You are on page 1of 339

Report Errors in the PDF - ebooks@affairscloud.com Copyright 2014-2023 @ AffairsCloud.

com 1
This PAID PDF is provided by AffairsCloud’s dedicated team that works
diligently to provide aspirants with high-quality content. We recommend
you to purchase this PDF subscription and seize the opportunity to learn
efficiently.
Help Us to Grow & Provide Quality Service

Click here to Download the CareersCloud APP


Click here to Join the Telegram Channel
AffairsCloud Hindu Vocabs

Suggestions & Feedback are welcomed


Support@affairscloud.com

Report Errors in the PDF - ebooks@affairscloud.com Copyright 2014-2023 @ AffairsCloud.com 2


Report Errors in the PDF - ebooks@affairscloud.com Copyright 2014-2023 @ AffairsCloud.com 3
Report Errors in the PDF - ebooks@affairscloud.com Copyright 2014-2023 @ AffairsCloud.com 4
Current Affairs Q&A PDF – September 2023

Table of Contents
NATIONAL AFFAIRS ...................................................................................................................................................... 6
INTERNATIONAL AFFAIRS ......................................................................................................................................... 38
GOVT SCHEMES ............................................................................................................................................................ 51
VISITS............................................................................................................................................................................. 56
BANKING AND FINANCE ............................................................................................................................................. 59
ECONOMY AND BUSINESS .......................................................................................................................................... 92
MoU’s AND AGREEMENTS ........................................................................................................................................ 102
APPOINTMENTS AND RESIGNATIONS ................................................................................................................... 115
AWARDS AND RECOGNITIONS ................................................................................................................................ 127
SUMMITS, CONFERENCES ......................................................................................................................................... 137
COMMITTEE AND MEETING .................................................................................................................................... 139
INDEX ........................................................................................................................................................................... 140
ACQUISITION AND MERGERS .................................................................................................................................. 141
DEFENCE...................................................................................................................................................................... 144
SCIENCE AND TECHNOLOGY .................................................................................................................................... 148
SPORTS ........................................................................................................................................................................ 164
BOOKS AND AUTHORS.............................................................................................................................................. 175
OBITUARY ................................................................................................................................................................... 177
IMPORTANT DAYS ..................................................................................................................................................... 184
ENVIRONMENT........................................................................................................................................................... 206
APP and WEB PORTAL.............................................................................................................................................. 208
CURRENT STATIC BANKING .................................................................................................................................... 211
CA STATIC GK ............................................................................................................................................................. 217

Report Errors in the PDF - ebooks@affairscloud.com Copyright 2014-2023 @ AffairsCloud.com 5


NATIONAL AFFAIRS
1. निम्ननिखित में से निस उत्पाद िो हाि ही में (अगस्त’23 में) भौगोनिि संिेत रनिस्ट्र ी से भौगोनिि संिेत
(GI) टै ग िा दिाा प्राप्त हुआ है ?
1)जम्मू और कश्मीर का भद्रवाह राजमाश (J&K)
2)जम्मू और कश्मीर का रामबन सुलाई शहद (J&K)
3)ससक्किम का चोकुवा चावल (जादु ई चावल)
4)1 और 2 दोनोों
5)2 और 3 दोनोों
उत्तर- 4)1 और 2 दोिों
स्पष्टीिरण:
चेन्नई (तसमलनाडु ) क्कथित भौगोसलक सोंकेत रसजस्ट्र ी ने जम्मू & कश्मीर (J&K) के भद्रवाह राजमाश (लाल राजमा) और
रामबन सुलाई शहद और असम के चोकुवा चावल (मैसजक राइस) को भौगोसलक सोंकेत (GI) टै ग प्रदान सकए हैं।
• भौगोसलक सोंकेत रसजस्ट्र ी उद्योग और आों तररक व्यापार सोंवर्धन सवभाग (DPIIT), वासिज्य और उद्योग मोंत्रालय
(MoCI) के तहत कायध करती है।

2. अगस्त 2023 में, __________ (राज्य) में िािरापार परमाणु ऊिाा पररयोििा इिाई -3 (KAPP3) में भारत
िे पहिे स्वदे शी रूप से नििनसत 700-मेगािाट इिेखरर ि (MWe) दबाियुक्त भारी पािी ररएरर (PHWR)
िे पूरी क्षमता पर पररचािि शुरू निया।
1)ओसडशा
2)गुजरात
3)मध्य प्रदे श
4)केरल
5)महाराष्ट्र
उत्तर- 2)गुिरात
स्पष्टीिरण:
31 अगस्त, 2023 को, गुिरात में काकरापार परमािु ऊजाध पररयोजना इकाई-3 (KAPP3) में भारत के पहले स्वदे शी
रूप से सवकससत 700-मेगावाट इलेक्कररक (MWe) दबावयुक्त भारी पानी ररएरर (PHWR) ने पूरी क्षमता पर
पररचालन शुरू कर सदया है ।
• इसने 30 जून, 2023 को वासिक्कज्यक पररचालन शुरू सकया िा लेसकन अब तक यह अपनी 90% क्षमता पर काम
कर रहा िा।
• KAPP-3 और KAPP-4 उन्नत सुरक्षा सुसवर्ाओों के साि 700 MW इकाई आकार की स्वदे शी रूप से सडजाइन
की गई PHWR की भारत की पहली जोडी है।
i.परमािु ऊजाध सवभाग (DAE) के तहत एक सावधजसनक क्षेत्र का उद्यम (PSE), न्यूखियर पािर िॉपोरे शि ऑफ इं निया
निनमटे ि (NPCIL) को परमािु ऊजाध ररएररोों के सडजाइन, सनमाधि, कमीशसनोंग और सोंचालन का काम सौोंपा गया है।
ii.700 MW के परमािु ऊजाध सोंयोंत्रोों का सनमाधि रावतभाटा, राजथिान (RAPS 7 और 8) और हररयािा के गोरखपुर
(GHAVP 1 और 2) में चल रहा है।

Report Errors in the PDF - ebooks@affairscloud.com Copyright 2014-2023 @ AffairsCloud.com 6


3. िौि सा मंत्रािय अरू बर 2023 में ग्लोबि इं नियाAI 2023 सम्मेिि िा पहिा संस्करण आयोनित िरिे
िे निए तैयार है ?
1)सवज्ञान एवों प्रौद्योसगकी मोंत्रालय
2)कौशल सवकास और उद्यसमता मोंत्रालय
3)सशक्षा मोंत्रालय
4)इलेररॉसनक्स और सूचना प्रौद्योसगकी मोंत्रालय
5)वासिज्य एवों उद्योग मोंत्रालय
उत्तर-4)इिेररॉनिक्स और सूचिा प्रौद्योनगिी मंत्रािय
स्पष्टीिरण:
इिेररॉनिक्स और सूचिा प्रौद्योनगिी मंत्रािय (MeitY) अरू बर 2023 (अथिायी रूप से 14/15 अरू बर 2023 के
सलए योजना बनाई गई है ) में ग्लोबल इों सडयाAI 2023 सम्मेलन का पहला सोंस्करि आयोसजत करने के सलए तैयार है।
i.राजीव चन्द्रशेखर, राज्य मोंत्री (MoS), कौशल सवकास और उद्यसमता मोंत्रालय (MSDE), और MeitY सम्मेलन की
सोंचालन ससमसत की अध्यक्षता कर रहे हैं। ससमसत में MeitY के सडसजटल अिधव्यवथिा सलाहकार समूह के सदस्य और
AI के क्षेत्र के अन्य प्रमुख नेता शासमल हैं।

4. भारत सरिार िे उस स्वायत्त संगठि िा िाम बताइए निसे हाि ही में (नसतंबर’23 में) यूनििनसाटी अिुदाि
आयोग यूनििनसाटी अिुदाि आयोग (UGC) िी 'िी िोिो' श्रेणी िे तहत 'िीम्ि टू बी यूनििनसाटी' िा दिाा
नदया गया है।
1)राष्ट्रीय शैसक्षक अनुसोंर्ान और प्रसशक्षि पररषद
2)राष्ट्रीय अध्यापक सशक्षा पररषद
3)केंद्रीय माध्यसमक सशक्षा बोडध
4)अक्कखल भारतीय तकनीकी सशक्षा पररषद
5)राष्ट्रीय मुक्त सवद्यालयी सशक्षा सोंथिान
उत्तर- 1)राष्टरीय शैनक्षि अिुसंधाि और प्रनशक्षण पररषद
स्पष्टीिरण:
राष्ट्रीय शैसक्षक अनुसोंर्ान और प्रसशक्षि पररषद (NCERT) ने यूसनवससधटी अनुदान आयोग (UGC) की 'डी नोवो' श्रेिी
के तहत 'डीम्ड टू बी यूसनवससधटी' का दजाध प्रदान सकया।
i.इसकी घोषिा कौशल सवकास और उद्यसमता मोंत्रालय (MSDE) के केंद्रीय मोंत्री र्मेंद्र प्रर्ान ने NCERT के 63वें
थिापना सदवस (1 ससतोंबर 2023)के सलए आयोसजत कायधक्रम के दौरान की।
• मानद यूसनवससधटी का दजाध यूसनवससधटी अनुदान आयोग (UGC) अनधनियम, 1956 की र्ारा 3 के तहत सदया
गया है।
ii.इस क्कथिसत के साि, NCERT डीम्ड-टू -बी-यूसनवससधटी के रूप में एक नया सोंथिान थिासपत करने के सलए UGC को
आवेदन कर सकता है , जो असितीय और उभरते क्षेत्रोों में अध्ययन और अनुसोंर्ान करे गा जो सकसी भी मौजूदा सोंथिान
िारा पेश नहीों सकए जाते हैं।
iii.नई क्कथिसत के साि NCERT को अपनी स्नातक, स्नातकोत्तर और डॉररे ट सडग्री प्रदान करने और अपना स्वयों का
पाठ्यक्रम और पाठ्यक्रम सडजाइन करने की स्वायत्तता होगी।
iv.अजमेर, भोपाल, भुवनेश्वर, मैसूर और सशलाोंग में NCERT के क्षेत्रीय सशक्षा सोंथिान (REI) वतधमान में सोंबोंसर्त राज्योों में
थिानीय यूसनवससधटीोों से सोंबद्ध हैं और नई क्कथिसत के बाद NCERT से सोंबद्ध होोंगे।

Report Errors in the PDF - ebooks@affairscloud.com Copyright 2014-2023 @ AffairsCloud.com 7


5. िेंद्रीय िागररि उड्डयि मंत्री ज्योनतरानदत्य M. नसंनधया िे हाि ही में (नसतंबर’23 में) निस राज्य में क्षेत्रीय
ििेखरनिटी योििा (RCS)-UDAN (उडे दे श िा आम िागररि) िे तहत नििनसत उत्केिा हिाई अड्डे िा
उद् घाटि निया है ?
1)ओसडशा
2)पसिम बोंगाल
3)झारखण्ड
4)मध्य प्रदे श
5)महाराष्ट्र
उत्तर- 1)ओनिशा
स्पष्टीिरण:
केंद्रीय मोंत्री ज्योसतरासदत्य M. ससोंसर्या, नागररक उड्डयन मोंत्रालय (MoCA) ने नागररक उड्डयन मोंत्रालय के राज्य मोंत्री
(MoS) जनरल सवजय कुमार ससोंह (सेवासनवृत्त) के साि ओनिशा के कालाहाोंडी सजले में उत्केिा हिाई अड्डे और ओसडशा
के उत्केला और भुवनेश्वर के बीच सीर्ी उडान का उद् घाटन सकया।
i.ओसडशा सरकार के स्वासमत्व वाला उत्केला हवाई अड्डा, MoCA की क्षेत्रीय कनेक्करसवटी योजना (RCS)-UDAN (उडे
दे श का आम नागररक) के तहत सवकससत सकया गया है , यह क्षेत्रीय हवाई अड्डा 31.07 करोड रुपये की लागत पर आता
है।
ii.31 अगस्त 2023 से उत्केला हवाई अड्डे के सोंचालन के साि, ओसडशा में अब भुवनेश्वर, झारसुगुडा, जयपोर और
राउरकेला के बाद कुल 5 हवाई अड्डे होोंगे।
iii.उत्केला हवाई अड्डे का रनवे 917 मीटर (2,995 फीट) लोंबा और 30 मीटर चौडा है।

6. नसतंबर 2023 में, निश्व स्वास्थ्य संगठि (WHO) िे "___________ (राज्य/UT) घोषणा" िे रूप में WHO
पारं पररि नचनित्सा िैनश्वि नशिर सम्मेिि 2023 िा पररणाम दस्तािेज़ िारी निया।
1)उत्तर प्रदे श
2)गुजरात
3)सदल्ली
4)महाराष्ट्र
5)कनाधटक
उत्तर- 2)गुिरात
स्पष्टीिरण:
सवश्व स्वास्थ्य सोंगठन (WHO) ने "गुिरात घोषिा" के रूप में WHO पारं पररि नचनित्सा िैनश्वि नशिर सम्मेिि
2023 का पररिाम दस्तावेज़ जारी सकया।
• यह स्वदे शी ज्ञान, जैव सवसवर्ता और पारों पररक, पूरक और एकीकृत सचसकत्सा के प्रसत वैसश्वक प्रसतबद्धताओों की
पुसष्ट् करता है।
• आयुवेद, योग और प्राकृसतक सचसकत्सा, यूनानी, ससद्ध और होम्योपैिी (AYUSH), भारत सरकार (GoI) और
WHO के मोंत्रालय िारा सोंयुक्त रूप से आयोसजत WHO का पारों पररक सचसकत्सा वैसश्वक सशखर सम्मेलन
"टु वड्ध स हेल्थ एों ड वेल-बीइों ग फॉर ऑल" 17-18 अगस्त 2023 को गाोंर्ीनगर, गुजरात, भारत में आयोसजत
सकया गया िा।

Report Errors in the PDF - ebooks@affairscloud.com Copyright 2014-2023 @ AffairsCloud.com 8


7. नसतंबर 2023 में िेंद्रीय िौशि नििास और उद्यनमता & नशक्षा मंत्री धमेंद्र प्रधाि द्वारा निए गए िॉन्च िे
संबंध में निम्ननिखित में से िौि सा नबंदु "सही" है /हैं ?
A) िेंद्रीय मंत्री धमेंद्र प्रधाि िे िौशि नििास और उद्यनमता मंत्रािय (MSDE) और नशक्षा मंत्रािय (MoE) िे
"एिुअि िैपेनसटी नबखडं ग प्लाि (ACBP)", "िो योर नमनिस्ट्र ी इं िक्शि मॉड्यूि" और उच्च नशक्षा निभाग
(DoHE), MoE िे "टर े निंग मॉड्यूि फॉर िायरे रसा " िारी निया।
B) ये पहि MSDE, DoHE, राष्टरीय शैनक्षि अिुसंधाि और प्रनशक्षण पररषद (NCERT) और अखिि भारतीय
तििीिी नशक्षा पररषद (AICTE) द्वारा शुरू िी गई थी।
C) ये सभी पहि नमशि िमायोगी िे तहत शुरू िी गई हैं , निसिा उद्दे श्य सरिारी अनधिाररयों िो भनिष्य
िे निए तैयार होिे और िुशि सािािनिि सेिा नितरण िे निए सक्षम बिािा है।
1)केवल A
2)केवल A & B
3)केवल B & C
4)केवल A & C
5)सभी A, B & C
उत्तर- 4)िेिि A & C
स्पष्टीिरण:
िेंद्रीय मंत्री धमेंद्र प्रधाि, कौशल सवकास और उद्यसमता
मोंत्रालय (MSDE) और सशक्षा मोंत्रालय (MoE) ने "एनुअल
कैपेससटी सबक्कडोंग प्लान (ACBP)”, MSDE के “नो योर
समसनस्ट्र ी इों डक्शन मॉड्यूल" और उच्च सशक्षा सवभाग
(DoHE), MoE के "टर े सनोंग मॉड्यूल फॉर डायरे रसध "
जारी सकया।
• ये पहल MSDE, DoHE, स्कूल सशक्षा और
साक्षरता सवभाग (DoSEL) और क्षमता निमााण
आयोग (CBC) िारा शुरू की गई है ।
• ये सभी पहल नमशि िमायोगी के तहत शुरू की गई हैं।
• इसका उद्दे श्य सरकारी असर्काररयोों को भसवष्य के सलए तैयार होने और कुशल सावधजसनक सेवा सवतरि के
सलए सक्षम बनाना है।

8. उस सरिारी एिेंसी/मंत्रािय िा िाम बताइए निसिे हाि ही में (नसतंबर’23 में) स्मारिों िे संरक्षण िे
निए 'अिॉप्ट ए हेररटे ि 2.0' िायाक्रम 'इं नियि हेररटे ि' ऐप और ई-अिुमनत पोटा ि िॉन्च निया है।
1)गृह मोंत्रालय
2)साोंस्कृसतक सोंसार्न और प्रसशक्षि केंद्र
3)भारतीय पुरातत्व सवेक्षि
4)सोंस्कृसत मोंत्रालय
5)पयधटन मोंत्रालय

Report Errors in the PDF - ebooks@affairscloud.com Copyright 2014-2023 @ AffairsCloud.com 9


उत्तर-3)भारतीय पुरातत्व सिेक्षण
स्पष्टीिरण:
भारतीय पुरातत्व सिेक्षण (ASI) ने समवेत ऑसडटोररयम, इों सदरा गाों र्ी राष्ट्रीय कला केंद्र (IGNCA), नई सदल्ली, सदल्ली
में स्मारकोों के सोंरक्षि के सलए कॉपोरे ट सहतर्ारकोों के साि सहयोग करने के सलए 'अिॉप्ट ए हेररटे ि 2.0' िायाक्रम
शुरू सकया।
• कायधक्रम का शुभारों भ केंद्रीय सोंस्कृसत, पयधटन और उत्तर पूवी क्षेत्र (DoNER) के सवकास मोंत्रालय के मोंत्री G
सकशन रे ड्डी, सोंस्कृसत और सवदे श मोंत्रालय की राज्य मोंत्री (MoS) मीनाक्षी लेखी के साि सकया गया।
• कायधक्रम में 'इं नियि हेररटे ि' िाम िा ऐप और ई-अिुमनत पोटा ि लॉन्च सकया गया।
i.'अडॉप्ट ए हेररटे ज 2.0' के तहत, ASI कॉपोरे ट सहतर्ारकोों को अपने कॉपोरे ट सामासजक उत्तरदासयत्व (CSR) फोंड का
उपयोग करके ऐसतहाससक स्मारकोों में सुसवर्ाएों बढाने के सलए आमोंसत्रत करता है।
• दे शभर में ASI के सोंरक्षि में 3696 स्मारक हैं ।
ii.प्राचीन स्मारक और पुराताक्कत्वक थिल और अवशेष (AMASR) असर्सनयम 1958 के अनुसार सवसभन्न स्मारकोों के सलए
सुसवर्ाएों माोंगी जाएों गी।

9. उस िंपिी िा िाम बताइए निसिे हाि ही में (नसतंबर'23 में) नशक्षा मंत्रािय (MoE), िौशि नििास &
उद्यनमता मंत्रािय (MSDE), िे साथ नमििर 3 साि िी अिनध िे निए 'एिुिेशि टू एं टर े प्रेन्योरनशप:
एम्पोिेररं ग ए ििरे शि ऑफ़ स्ट्ूिें ट्स, एिु िेटसा, एं ि एं टर े प्रेन्योसा' शुरू निया है।
1)टाटा कोंसल्टें सी ससवधसेज
2)इों फोससस
3)सवप्रो
4)IBM
5)मेटा
उत्तर- 5)मेटा
स्पष्टीिरण:
3 ससतोंबर, 2023 को, सशक्षा मोंत्रालय (MoE), कौशल सवकास & उद्यसमता मोंत्रालय (MSDE), और मेटा (सजसे पहले
फेसबुक के नाम से जाना जाता िा) के बीच नई सदल्ली, सदल्ली में 3 साि िी अिनध िे निए एक सहयोग शुरू सकया
गया िा, सजसका नाम 'एजुकेशन टू एों टर े प्रेन्योरसशप: एम्पोवेररों ग ए जनरे शन ऑफ़ स्ट्ूडें ट्स, एडु केटसध, एों ड एों टर े प्रेन्योसध'
िा।
• यह सहयोग केंद्रीय मोंत्री र्मेंद्र प्रर्ान, MoE & MSDE िारा लॉन्च सकया गया िा।
• मेटा और प्रमुख सोंथिानोों, अिाध त् राष्ट्रीय उद्यसमता और लघु व्यवसाय सवकास सोंथिान (NIESBUD), अक्कखल
भारतीय तकनीकी सशक्षा पररषद (AICTE), और केंद्रीय माध्यसमक सशक्षा बोडध (CBSE) के बीच तीन आशय पत्रोों
(LoI) का भी आदान-प्रदान सकया गया।

10. राष्टरपनत द्रौपदी मुमूा िे निस राज्य/UT में राष्टरनपता 'महात्मा गांधी' िी 12 फुट ऊंची प्रनतमा िा अिािरण
निया और 'गांधी िानटिा' िा उद् घाटि निया?
1)उत्तर प्रदे श
2)महाराष्ट्र
3)गुजरात
4)सदल्ली

Report Errors in the PDF - ebooks@affairscloud.com Copyright 2014-2023 @ AffairsCloud.com 10


5)मध्य प्रदे श
उत्तर-4)नदल्ली
स्पष्टीिरण:
4 ससतोंबर 2023 को, राष्ट्रपसत द्रौपदी मुमूध ने राष्ट्रसपता ‘महात्मा गांधी’ की 12 फुट ऊंची प्रनतमा का अनावरि सकया
और नई सदल्ली, नदल्ली में राजघाट के पास गाोंर्ी दशधन में 'गांधी िानटिा' का उद् घाटन सकया।
• गाोंर्ी वासटका में महात्मा गाों र्ी की अलग-अलग मुद्राओों में मूसतधयाों थिासपत की गई हैं।
• सवशेष रूप से बच्चोों के सलए आगोंतुक अनुभव को बढाने के सलए गाोंर्ी वासटका के भीतर एक सेल्फी पॉइों ट
थिासपत सकया गया है।

11. नसतंबर 2023 में नशक्षा मंत्रािय द्वारा निए गए िॉन्च िे संबंध में निम्ननिखित में से िौि सा नबंदु "सही"
है/हैं ?
A) िेंद्रीय मंत्री धमेंद्र प्रधाि, नशक्षा मंत्रािय (MoE) िे िई नदल्ली, नदल्ली में माििीय नमशि - नशक्षि प्रनशक्षण
िायाक्रम िा शुभारं भ निया।
B) पोटा ि क्षमता निमााण िायाक्रमों में रुनच रििे िािे नशक्षिों िे निए निश्वनिद्यािय अिुदाि आयोग (UGC)
द्वारा स्थानपत निया गया है।
C) MoE िे िक्षाओं में रचिात्मि अनभव्यखक्त नििनसत िरिे बच्चों िो आनटा नफनशयि इं टेनििेंस (AI)
सीििे में मदद िरिे िे निए माइक्रोसॉफ़्ट िॉपोरे शि िे साथ एि समझौता ज्ञापि (MoU) पर भी हस्ताक्षर
निए।
1)केवल A
2)केवल A & B
3)केवल B & C
4)केवल A & C
5)सभी A, B & C
उत्तर- 2)िेिि A & B
स्पष्टीिरण:
5 ससतोंबर, 2023 को, यानी राष्ट्रीय सशक्षक सदवस 2023 के
अवसर पर, केंद्रीय मोंत्री र्मेंद्र प्रर्ान, सशक्षा मोंत्रालय (MoE)
ने कौशल भवन, नई सदल्ली , सदल्ली में सवश्वसवद्यालय अनुदान
आयोग (UGC) िारा मालवीय समशन - सशक्षक प्रसशक्षि
कायधक्रम का शुभारों भ सकया।
• उन्ोोंने कायधक्रम के पोटध ल
(https://mmc.ugc.ac.in/) का भी उद् घाटन सकया
और इसकी सूचना सववरसिका जारी की।
• यह पोटध ल क्षमता सनमाधि कायधक्रमोों में रुसच रखने वाले सशक्षकोों के सलए UGC िारा थिासपत सकया गया है।
• मालवीय समशन - सशक्षक प्रसशक्षि कायधक्रम का उद्दे श्य सशक्षकोों के प्रसशक्षि की गुिवत्ता में सुर्ार करना,
सशक्षकोों में नेतृत्व कौशल का सनमाध ि करना और राष्ट्रीय सशक्षा नीसत (NEP) 2020 के लक्ष्ोों को साकार करने में
मदद करना है।
• मालवीय समशन (MM) के तहत उच्च सशक्षा सोंथिानोों (HEI) में 111 मानव सोंसार्न सवकास केंद्र (HRDC) की
पहचान की गई है । इन HRDC का नाम बदलकर मदन मोहन मालवीय सशक्षक प्रसशक्षि केंद्र रखा जाएगा।

Report Errors in the PDF - ebooks@affairscloud.com Copyright 2014-2023 @ AffairsCloud.com 11


• MoE ने एिोब एक्सप्रेस एक्कप्लकेशन का उपयोग करके कक्षाओों में रचनात्मक असभव्यक्कक्त सवकससत करके
बच्चोों को आनटा नफनशयि इं टेनििेंस (AI) सीखने में मदद करने के सलए एक अमेररकी बहुराष्ट्रीय कोंप्यूटर
सॉफ्टवेयर कोंपनी एिोब के साि एक समझौता ज्ञापन (MoU) पर भी हस्ताक्षर सकए।

12. निम्ननिखित में से निस मंत्रािय/संगठि िे हाि ही में (नसतंबर’23 में) G20 (बीस िा समूह) पयाटि और
SDG (सतत नििास िक्ष्य) िै शबोिा िा अिािरण निया है ?
1)पयधटन मोंत्रालय
2)सोंयुक्त राष्ट्र सवश्व पयधटन सोंगठन
3)कौशल सवकास और उद्यसमता मोंत्रालय
4)1 और 2 दोनोों
5)2 और 3 दोनोों
उत्तर- 4)1 और 2 दोिों
स्पष्टीिरण:
5 ससतोंबर, 2023 को पयाटि मंत्रािय ने सोंयुक्त राष्ट्र सवश्व पयधटन सोंगठन (UNWTO) के सहयोग से G20 (बीस िा
समूह) पयाटि और SDG (सतत सवकास लक्ष्) िै शबोिा का अनावरि सकया।
• डै शबोडध को एक आभासी समारोह में पयधटन मोंत्रालय के केंद्रीय मोंत्री गोंगापुरम सकशन रे ड्डी िारा लॉन्च सकया
गया िा।
• G20 पयधटन और SDG डै शबोडध G20 पयधटन कायध समूह का एक पररिाम है।
• डै शबोडध को भारत की G20 अध्यक्षता के नेतृत्व में और UNWTO की सवशेषज्ञ ज्ञान साझेदारी के साि
सवकससत सकया गया है।

13. नसतंबर 2023 में भारत िे पहिे सोिर नसटी िा उद् घाटि निस शहर में निया गया?
1)चोंद्रपुर, महाराष्ट्र
2)तुनी, आों ध्र प्रदे श
3)चुरू, राजथिान
4)साोंची, मध्य प्रदे श
5)बाोंकुरा, पसिम बोंगाल
उत्तर-4)सांची, मध्य प्रदे श
स्पष्टीिरण:
मध्य प्रदे श (MP) के मुख्यमोंत्री (CM) सशवराज ससोंह चौहान ने भारत िे पहिे सोिर नसटी के रूप में MP में रायसेन
सजले के साोंची शहर का उद् घाटन सकया।
• यह प्रर्ान मोंत्री (PM) नरें द्र मोदी के 2070 तक हर राज्य में एक सोलर ससटी सवकससत करने के दृसष्ट्कोि के
अनुरूप है।
• साोंची के नागौरी और गुलगाोंव में क्रमशः 3 MW और 5 MW के सोलर सोंयोंत्र शहर की सवद् युत और कृसष
जरूरतोों को पूरा करते हैं।
• इसके अलावा, साोंची शहर के भीतर 8 MW का सग्रड-कनेरेड सोलर सोंयोंत्र सनमाध िार्ीन है।
• मध्य प्रदे श ऊजाध सवकास सनगम सलसमटे ड (MPUVNL) ने सोलर ससटी पररयोजना की नोडल एजेंसी के रूप में
कायध सकया।

Report Errors in the PDF - ebooks@affairscloud.com Copyright 2014-2023 @ AffairsCloud.com 12


• MPUVNL ने साोंची के लोगोों को ऊजाध बचत के बारे में सशसक्षत करने के सलए 'ऊजाध साक्षरता असभयान'
चलाया।

14. नसतंबर 2023 में मंनत्रमंिि िी मंिूरी िे संबंध में निम्ननिखित में से िौि सा नबंदु "सही" है /हैं ?
A) मंनत्रमंिि िे बैटरी एििी स्ट्ोरे ि नसस्ट्म (BESS) िे नििास िे निए 3,760 िरोड रुपये िी िायनबनिटी
गैप फंनिं ग (VGF) िी एि योििा िो मंिूरी दी।
B)मंनत्रमंिि िे उत्तर प्रदे श और नबहार िे निए औद्योनगि नििास योििा (IDS), 2017 िे निए 1164.53
िरोड रुपये िी अनतररक्त रानश िो मंिूरी दी।
C) िेंद्रीय गृह मंत्रािय (MHA) िे मेनििि प्रिेश में सरिारी स्कूिों में पढ़िे िािे छात्रों िे निए 10% िोटा
िागू िरिे िे पुिुचेरी िे एि प्रस्ताि िो मंिूरी दे दी है
1)केवल A
2)केवल A & B
3)केवल B & C
4)केवल A & C
5)सभी A, B & C
उत्तर- 4)िेिि A & C
स्पष्टीिरण:
प्रर्ान मोंत्री (PM) िरें द्र मोदी की अध्यक्षता में केंद्रीय मोंसत्रमोंडल ने 6 ससतोंबर, 2023 को सनम्नसलक्कखत प्रस्तावोों को मोंजूरी
दे दी है :
• केंद्रीय मोंसत्रमोंडल ने बैटरी एनजी स्ट्ोरे ज ससस्ट्म (BESS) सवकससत करने के उद्दे श्य से वायसबसलटी गैप फोंसडों ग
(VGF) की एक योजना को मोंजूरी दे दी है। यह योजना नवीकरिीय ऊजाध एकीकरि को बढाने और
असर्कतम सबजली माोंग लागत को कम करने के सरकार के प्रयासोों का सहस्सा है । इसे पहली बार केंद्रीय बजट
2023 में पेश सकया गया िा।
• सरकारी योगदान: कुल पररयोजना लागत का 40%, रासश 3,760 िरोड रुपये , VGF के रूप में प्रदान सकया
गया। पररयोजना लागत का शेष 60% भाग लेने वाली कोंपसनयोों िारा सनवेश सकया जाना है।
• मोंसत्रमोंडल ने नहमाचि प्रदे श और उत्तरािण्ड के सलए औद्योसगक सवकास योजना (IDS), 2017 के सलए
1164.53 िरोड रुपये की असतररक्त रासश को मोंजूरी दी। वासिज्य और उद्योग मोंत्रालय, उद्योग और आों तररक
व्यापार सोंवर्धन सवभाग ने सहमाचल प्रदे श और उत्तराखोंड के सलए औद्योसगक सवकास योजना 2017 के सलए
केंद्रीय क्षेत्र योजना के तहत असतररक्त र्न के आवोंटन का प्रस्ताव रखा।
• केंद्रीय गृह मोंत्रालय (MHA) ने पुिुचेरी में मेसडकल प्रवेश में सरकारी स्कूलोों में पढने वाले छात्रोों के सलए 10%
िोटा लागू करने के पुडुचेरी के एक प्रस्ताव को मोंजूरी दे दी है । केंद्रीकृत प्रवेश ससमसत (CENTAC) सवसभन्न
उच्च सशक्षा पाठ्यक्रमोों में प्रवेश के सलए छात्रोों की सूची को अोंसतम रूप दे ने के सलए सजम्मेदार है।
• पात्रता: NEET-योग्य छात्रोों के सलए सजन्ोोंने पुडुचेरी में सरकारी स्कूलोों में कक्षा I-XII तक पढाई की है।

15. नसतंबर 2023 में निस शहर में भारत िे पहिे 500 नििो-िोल्ट-एम्पीयर (kVA) अंिरग्राउं ि पािर
टर ांसफामार स्ट्े शि िा उद् घाटि निया गया?
1)कडपा, आों ध्र प्रदे श
2)जोर्पुर, राजथिान
3)चेन्नई, तसमलनाडु

Report Errors in the PDF - ebooks@affairscloud.com Copyright 2014-2023 @ AffairsCloud.com 13


4)बेंगलुरु, कनाधटक
5)कुरनूल, आों ध्र प्रदे श
उत्तर-4)बेंगिुरु, ििााटि
स्पष्टीिरण:
5 ससतोंबर 2023 को, कनाधटक सरकार के ऊजाध मोंत्री K J जॉजध ने मल्लेश्वरम, बेंगलुरु (ििााटि) में 15वें एवेन्यू पर
भारत के पहले 500 सकलो-वोल्ट-एम्पीयर (kVA) अोंडरग्राउों ड पावर टर ाोंसफामधर स्ट्े शन का उद् घाटन सकया।
i.यह सुसवर्ा 1.9 करोड रुपये की लागत से बेंगलुरु इलेक्कररससटी सप्लाई कोंपनी सलसमटे ड (BESCOM) और ब्रुहट
बेंगलुरु महानगर पासलका (BBMP) के सहयोग से सवकससत की गई िी।
ii.सबजली सोंबोंर्ी कायध BESCOM िारा और सससवल कायध BBMP िारा सकये जाते हैं ।

16. नसतंबर 2023 में, __________ िण प्रदू षण िे निए उत्सिाि व्यापार योििा (ETS) िागू िरिे िािा भारत
िा दू सरा शहर बि गया।
1)पुिे
2)अहमदाबाद
3)मुोंबई
4)गाोंर्ीनगर
5)इों दौर
उत्तर- 2)अहमदाबाद
स्पष्टीिरण:
गुजरात सरकार के पयधटन, साोंस्कृसतक गसतसवसर्याों , वन और पयाधवरि, जलवायु पररवतधन मोंत्री मुलुभाई बेरा ने
अहमदाबाद, गुजरात में उत्सजधन व्यापार योजना (ETS) लाइव टर े सडों ग का उद् घाटन सकया।
• इस लॉन्च के साि, अहमदाबाद सूरत (गुजरात) के बाद िण प्रदू षण के सलए ETS लागू करने वाला भारत के
साि-साि गुजरात का दू सरा शहर बन गया।
• कि प्रदू षि के सलए ETS औद्योसगक उत्सजधन को सवसनयसमत करने और गुजरात के प्रदू षि स्तर को कम करने
में मदद करे गा।
• योजना में अहमदाबाद के 118 उद्योगोों के बीच लाइव टर े सडों ग 5 ससतोंबर 2023 से शुरू हो गई है ।

17. निस राज्य सरिार िे हाि ही में (नसतंबर'23 में) टर ांसिेंिर समुदाय िे निए मुख्यमंत्री राज्य सामानिि
सुरक्षा पेंशि योििा िे प्रस्ताि िो मंिूरी दे दी है और इसे अन्य नपछडा िगा (OBC) श्रेणी में शानमि निया है ?
1)ओसडशा
2)महाराष्ट्र
3)झारखण्ड
4)पसिम बोंगाल
5)मध्य प्रदे श
उत्तर-3)झारिंि
स्पष्टीिरण:
मुख्यमोंत्री (CM) हे मोंत सोरे न की अध्यक्षता में झारिंि मोंसत्रमोंडल ने अपनी सावधभौसमक पेंशन योजना 'मुख्यमंत्री राज्य
सामानिि सुरक्षा पेंशि योििा' के तहत टर ाोंसजेंडर समुदाय को शासमल करने और प्रसत माह 1000 रुपये की सवत्तीय
सहायता का लाभ उठाने को मोंजूरी दे दी।

Report Errors in the PDF - ebooks@affairscloud.com Copyright 2014-2023 @ AffairsCloud.com 14


• सजनकी आयु 18 वषध या उससे असर्क है और उनके पास मतदाता पहचान पत्र है , वे योजना के सलए पात्र होोंगे।
i.मोंसत्रमोंडल ने उन्ें सरकारी नौकररयोों में आरक्षि का लाभ सुसनसित करने के सलए अन्य सपछडा वगध (OBC) श्रेणी के
तहत शासमल करने की भी मोंजूरी दे दी है।
ii.जो लोग सकसी भी जासतगत आरक्षि के दायरे में नहीों आते हैं उन्ें सूची में 46वें नोंबर पर OBC श्रेिी में शासमल सकया
जाएगा।

18. भौगोलिक संकेत रलिस्ट्र ी ने हाि ही में (लसतंबर’23 में) कोरापुट कािािीरा चावि को भौगोलिक संकेत
(GI) टै ग प्रदान लकया है।
कोरापुट कािािीरा चावि ___________ (राज्य) से संबंलित है।
1)पश्चिम बंगाल
2)आं ध्र प्रदे श
3)केरल
4)ओश्चिशा
5)तश्चमलनािु
उत्तर-4)ओलिशा
स्पष्टीकरण:
चेन्नई (तश्चमलनािु ) स्थित भौगोश्चलक संकेत रश्चिस्ट्र ी ने ओलिशा के 'कोरापुट कािािीरा चावि' को भौगोश्चलक संकेत
(GI) टै ग प्रदान श्चकया है, श्चिसे 'चावल का रािकुमार' भी कहा िाता है।
i.कालािीरा चावल की खेती श्चवशेष रूप से ओश्चिशा के कोरापुट श्चिले की िनिाश्चतयों और श्चकसानों द्वारा पीश्च़ियों से की
िाती रही है।
• कोरापुट कालािीरा चावल (कक्षा 30 - चावल) को माल के अंतगगत 'कृश्चष' के रूप में वगीकृत श्चकया गया िा।
ii.भौगोश्चलक संकेत रश्चिस्ट्र ी उद्योग और आं तररक व्यापार संवर्गन श्चवभाग (DPIIT), वाश्चिज्य और उद्योग मंत्रालय
(MoCI) के तहत कायग करती है।

19. लकस राज्य ने हाि ही में (लसतंबर’23 में) कृष्ण िन्माष्टमी के लहस्से के रूप में दही हांिी उत्सव मनाया है?
1)मध्य प्रदे श
2)गुिरात
3) पश्चिम बंगाल
4)महाराष्ट्र
5) उत्तर प्रदे श
उत्तर-4)महाराष्टर
स्पष्टीकरण:
महाराष्टर ने 7 श्चसतंबर 2023 को भगवान कृष्ण की ियंती, कृष्ण िन्माष्टमी के श्चहस्से के रूप में दही हांिी उत्सव
मनाया। इस उत्सव को गोपाल काला या उत्लोत्सवम के नाम से भी िाना िाता है।
• 800 से अश्चर्क गोश्चवंद पिकों (समूहों) ने मानव श्चपराश्चमि तैयार श्चकए और वारली, अंर्ेरी, िोगेश्वरी और ठािे
सश्चहत मुंबई (महाराष्ट्र) के श्चवश्चभन्न उपनगरीय इलाकों में दही हां िी तोड़ने के श्चलए एक-दू सरे के साि प्रश्चतस्पर्ाग
की।
• महाराष्ट्र सरकार ने लगभग 75,000 गोश्चवंदों को बीमा प्रदान श्चकया है ।

Report Errors in the PDF - ebooks@affairscloud.com Copyright 2014-2023 @ AffairsCloud.com 15


20. नसतंबर 2023 में सामानिि न्याय और अनधिाररता मंत्रािय (MoSJE) िे तहत निििांग व्यखक्तयों िे
सशखक्तिरण निभाग (DePWD) द्वारा निए गए िॉन्च िे संबंध में निम्ननिखित में से िौि सा/से नबंदु "सही"
है/हैं ?
A) सामानिि न्याय और अनधिाररता मंत्रािय (MoSJE) िे तहत निििांग व्यखक्तयों िे सशखक्तिरण निभाग
(DePWD) िे समािेनशता िो बढ़ािा दे िे और निििांग व्यखक्तयों (PwD) िो सशक्त बिािे िे निए 5 अग्रणी
पहि शुरू िी।
B) PwD िे निए नशिायत समाधाि प्रनक्रया िो सरि और तेि िरिे िे निए िॉमि सनिासेि सेंटर (CSC) ई-
गििेंस सनिास इं निया निनमटे ि द्वारा नििनसत एि ऑििाइि िेस मॉनिटररं ग पोटा ि िॉन्च निया गया था।
C) DEPwD द्वारा शुरू निया गया PM-DAKSH पोटा ि एि व्यापि निनिटि प्लेटफॉमा है निसे PwD िो
िौशि प्रनशक्षण और रोिगार िे अिसरों िी तिाश में सशक्त बिािे िे निए निज़ाइि निया गया है।
1)सभी A, B & C
2)केवल A & B
3)केवल B & C
4)केवल A & C
5)केवल A
उत्तर- 4)िेिि A & C
स्पष्टीिरण:
सामासजक न्याय और असर्काररता मोंत्रालय (MoSJE) के तहत सवकलाों ग व्यक्कक्तयोों के सशक्कक्तकरि सवभाग (DePWD)
ने समावेसशता को बढावा दे ने और सवकलाोंग व्यक्कक्तयोों (PwD) को सशक्त बनाने के सलए 5 अग्रिी पहल शुरू की।
i.PwD के सलए सशकायत समार्ान प्रसक्रया को सरल और तेज करने के सलए सवकलाोंग व्यक्कक्तयोों के सलए मुख्य आयुक्त
(CCPD) के कायाधलय िारा सवकससत एक ऑििाइि िेस मॉनिटररं ग पोटा ि लॉन्च सकया गया िा।
ii.इन पहलोों की शुरुआत नई सदल्ली में डॉ. अम्बेडकर इों टरनेशनल सेंटर (DAIC) में राज्य मोंत्री (MoS), MoSJE प्रसतमा
भौमी िारा की गई।
iii.अन्य 4 पहिें थी ं:
• DEPwD ने बैचलर ऑफ आसकधटे क्चर कायधक्रमोों के भीतर सावधभौसमक पहुोंच पाठ्यक्रम शुरू करने के सलए
COA के साि साझेदारी की है।
• DEPwD ने अनुसोंर्ान उद्दे श्योों के सलए UDID पोटध ल के माध्यम से गुमनाम डे टा जारी सकया। यह सवसभन्न स्तरोों
पर अोंतदृधसष्ट् प्रदान करे गा, गहरी समझ की सुसवर्ा प्रदान करे गा और लसक्षत हस्तक्षेपोों की जानकारी दे गा।
• DEPwD िारा शुरू सकया गया PM-DAKSH पोटध ल एक व्यापक सडसजटल प्लेटफॉमध है सजसे PwD को कौशल
प्रसशक्षि और रोजगार के अवसरोों की तलाश में सशक्त बनाने के सलए सडज़ाइन सकया गया है।
• DEPwD िारा पुक्कस्तका, प्रवेश के रास्ते: सवकलाोंगता असर्कारोों पर न्यायालय, सवकलाोंगता असर्कारोों से सोंबोंसर्त
भारत के सवोच्च न्यायालय और उच्च न्यायालयोों के उल्लेखनीय सनिधयोों का सोंकलन है।

21. निस राज्य िे राज्यपाि िे हाि ही में (नसतंबर’23 में) "मण्यम निप्लि सारनध अल्लूरी सीताराम रािू " पर
छह निशेष ििर िारी निए हैं ?
1)गिेशी लाल (ओसडशा के राज्यपाल)
2)िावरचोंद गेहलोत (कनाधटक के राज्यपाल)
3)S अब्दु ल नज़ीर (आों ध्र प्रदे श के राज्यपाल)
4)C.P. रार्ाकृष्णन (झारखोंड के राज्यपाल)

Report Errors in the PDF - ebooks@affairscloud.com Copyright 2014-2023 @ AffairsCloud.com 16


5)तसमसलसाई सौोंदयधराजन (तेलोंगाना के राज्यपाल)
उत्तर-3)S अब्दु ि िज़ीर (आं ध्र प्रदे श िे राज्यपाि)
स्पष्टीिरण:
आों ध्र प्रदे श (AP) िे राज्यपाि S अब्दु ि िज़ीर ने डाक सटकट सोंग्रह के माध्यम से गुमनाम नायकोों को सम्मासनत करने
के सलए आजादी का अमृत महोत्सव के भाग "मण्यम निप्लि सारनध अल्लूरी नसताराम रािू " पर छह सवशेष कवर
जारी सकए।
• सोंचार मोंत्रालय के डाक सवभाग िारा प्रकासशत ये कवर मोनोग्राफ के रूप में अल्लूरी के वीरतापूिध कायों और
उनके लेक्कफ्टनेंटोों के बसलदान का सोंकलन है।
• यह ररलीज़ सचली पोस्ट् की शताब्दी का भी स्मरि कराती है।
• 1922 में, अल्लूरी सीताराम राजू के नेतृत्व में क्राोंसतकाररयोों ने सब्रसटश असर्काररयोों को सोंदेश भेजने का एक
अनोखा तरीका इस्तेमाल सकया। वे अोंग्रेजोों को डराने के सलए एक तीर चलाते हैं , साि में समचध का एक गुच्छा और
एक टाइप सकया हुआ सोंदेश, सजसे 'समरापकाया तप" या "सचली पोस्ट्' कहा जाता है।
• उनका पहला हमला 1922 में सचोंतापल्ली पुसलस स्ट्े शन पर िा, सजसके कारि रम्पा क्राोंसत हुई।

22. नसतंबर 2023 में, िेंद्रीय रक्षा मंत्री राििाथ नसंह िे सीमा सडि संगठि (BRO) द्वारा निनमात 2941 िरोड
रुपये िी 90 बुनियादी ढांचा पररयोििाओं िा उद् घाटि निया।
पररयोििाओं िा निमााण भारत िे सीमािती राज्यों और िेंद्र शानसत प्रदे शों में निया गया है।
1)4
2)11
3)3
4)5
5)9
उत्तर- 2)11
स्पष्टीिरण:
रक्षा मोंत्रालय के केंद्रीय मोंत्री राजनाि ससोंह ने जम्मू -कश्मीर में सबश्नाह-कौलपुर-फूलपुर रोड पर दे वक सब्रज पर BRO
िारा आयोसजत एक समारोह में सीमा सडक सोंगठन (BRO) िारा सनसमधत 90 बुनियादी ढांचा पररयोििाओं का
उद् घाटन सकया।
• पररयोजनाओों का सनमाधि भारत के 11 सीमािती राज्यों और िेंद्र शानसत प्रदे शों में सकया गया है।
• BRO ने इन पररयोजनाओों का सनमाधि अत्यार्ुसनक तकनीक का उपयोग करते हुए कम समय में 2941 िरोड
रुपये में पूरा सकया।
i.बुसनयादी ढाोंचा पररयोजनाओों में अरुिाचल प्रदे श में नेसचपु सुरोंग; पसिम बोंगाल में दो हवाई क्षेत्र, दो हे लीपैड, 22
सडकें और 63 पुल शासमल है।
ii.इन 90 पररयोजनाओों में से अरुिाचल प्रदे श में 36 पररयोजनाएों हैं , लद्दाख में 26 पररयोजनाएों हैं , जम्मू & कश्मीर में
11 पररयोजनाएों हैं , समजोरम में पाोंच पररयोजनाएों हैं , सहमाचल प्रदे श में तीन पररयोजनाएों हैं।
• ससक्किम, उत्तराखोंड और पसिम बोंगाल में प्रत्येक में दो-दो हैं।
• एक पररयोजना नागालैंड में, एक राजथिान में और एक अोंडमान एवों सनकोबार िीप समूह में है।

Report Errors in the PDF - ebooks@affairscloud.com Copyright 2014-2023 @ AffairsCloud.com 17


23. निस शहर में, िेंद्रीय नशक्षा और िौशि नििास और उद्यनमता मंत्री धमेंद्र प्रधाि िे हाि ही में
(नसतंबर’23 में) निद्या समीक्षा िेंद्र (VSK) िा उद् घाटि निया और 141 प्रधािमंत्री स्कूि फॉर राइनिंग इं निया
(PM SHRI) स्कूिों िी आधारनशिा रिी?
1)पुिे, महाराष्ट्र
2)गाोंर्ीनगर, गुजरात
3)लखनऊ, उत्तर प्रदे श
4)दे हरादू न, उत्तराखोंड
5)पटना, सबहार
उत्तर-4)दे हरादू ि, उत्तरािंि
स्पष्टीिरण:
केंद्रीय मोंत्री र्मेंद्र प्रर्ान, सशक्षा मोंत्रालय (MoE) और कौशल सवकास और उद्यसमता मोंत्रालय (MSDE) ने उत्तराखोंड के
मुख्यमोंत्री (CM) पुष्कर ससोंह र्ामी के साि, उत्तरािंि िे दे हरादू ि में सवद्या समीक्षा केंद्र (VSK) का उद् घाटन सकया।
i.उन्ोोंने दे हरादू न, उत्तराखोंड में 141 प्रर्ानमोंत्री स्कूल फॉर राइसजोंग इों सडया (PM SHRI) स्कूलोों और 40 सबस्तरोों वाले
नेताजी सुभाष चोंद्र बोस आवासीय सवद्यालय (NSCBAV) (आवासीय सवद्यालय) की आर्ारसशला भी रखी।
ii.PM SHRI स्कूल उत्कृष्ट्ता हाससल करने के सलए मौजूदा स्कूलोों के सलए मॉडल के रूप में काम करें गे और
NSCBAV वोंसचत बच्चोों को गुिवत्तापूिध सशक्षा प्रदान करे गा।

24. निस संगठि िे इं नियि इं स्ट्ीट्यूट ऑफ िॉरपोरे ट अफेयसा (IICA) और िेशिि स्ट्ॉि एक्सचेंि ऑफ
इं निया निनमटे ि (NSE) िे साथ नमििर हाि ही में (नसतंबर’23 में) मुंबई, महाराष्टर में नबििेस
ररस्पॉखिनबनिटी एं ि सस्ट्े िेनबनिटी ररपोनटिं ग (BRSR) पर एि िायाशािा िा आयोिि निया है ?
1) यूनाइटे ड नेशोंस एजुकेशनल, साइों सटसफक एों ड कल्चरल आगेनाइजेशन
2) यूनाइटे ड नेशोंस डे वलपमेंट प्रोग्राम
3) यूनाइटे ड नेशोंस सचडर्न्ध फण्ड
4) इों टरनेशनल मोनेटरी फण्ड
5) वडध बैंक
उत्तर-3)यूिाइटे ि िेशंस नचडिा फण्ड
स्पष्टीिरण:
12 ससतोंबर, 2023 को, इों सडयन इों स्ट्ीट्यूट ऑफ कॉपोरे ट अफेयसध (IICA) ने UNICEF (यूनाइटे ड नेशोंस सचडर्न्ध फण्ड
) और नेशनल स्ट्ॉक एक्सचेंज ऑफ इों सडया सलसमटे ड (NSE) के सहयोग से मुोंबई, महाराष्ट्र में NSE पररसर में सबजनेस
ररस्पॉक्कर्न्सबसलटी एों ड सस्ट्े नेसबसलटी ररपोसटिं ग (BRSR) पर एक िायाशािा का आयोजन सकया।
i.उद्दे श्य:कायधशाला का उद्दे श्य BRSR ढाोंचे की व्यापक समझ प्रदान करना है ।
ii.BRSR ढाोंचा: यह नेशनल गाइडलाइर्न् फॉर ररस्पॉक्कर्न्बल सबजनेस कोंडर (NGRBC) के 9 ससद्धाोंतोों पर आर्ाररत है।
यह शीषध 1000 सूचीबद्ध कोंपसनयोों को अपने ESG प्रदशधन की ररपोटध करने के सलए बाध्य करता है , जो सजम्मेदार
व्यावसासयक आचरि के प्रसत उनके समपधि को दशाधता है।

Report Errors in the PDF - ebooks@affairscloud.com Copyright 2014-2023 @ AffairsCloud.com 18


25. नसतंबर 2023 में मंनत्रमंिि िी मंिूरी िे संबंध में निम्ननिखित में से िौि सा नबंदु "गित" है ?
1)केंद्रीय मोंसत्रमोंडल ने सवत्तीय वषध 2023-24 से 2027-28 तक 5 वषों में 84 लाख LPG कनेक्शन जारी करने के सलए
प्रर्ान मोंत्री उज्ज्वला योजना (PMUY) 2.0 के सवस्तार को मोंजूरी दे दी।
2)केंद्रीय मोंसत्रमोंडल ने 7210 करोड रुपये के सवत्तीय पररव्यय के साि 4 वषों (2023 से शुरू) के सलए केंद्रीय क्षेत्र
योजना के रूप में ई-कोटध प्रोजेर चरि III को मोंजूरी दे दी है ।
3)केंद्रीय मोंसत्रमोंडल ने उस MoU को मोंजूरी दे दी सजस पर जून 2023 में भारत के इलेररॉसनक्स और सूचना प्रौद्योसगकी
मोंत्रालय (Meity) और आमेसनया गिराज्य के उच्च तकनीक उद्योग मोंत्रालय िारा हस्ताक्षर सकए गए िे।
4)केंद्रीय मोंसत्रमोंडल ने एक और MoU को भी मोंजूरी दे दी, सजस पर जून 2023 में भारत सरकार के Meity और
एों टीगुआ और बारबुडा के सूचना, सोंचार प्रौद्योसगकी, उपयोसगता और ऊजाध मोंत्रालय के बीच हस्ताक्षर सकए गए िे।
5)केंद्रीय मोंसत्रमोंडल ने 9 से 10 ससतोंबर, 2023 तक नई सदल्ली में भारत िारा पहली बार आयोसजत 18वें G20 सशखर
सम्मेलन की सफलता का जश्न मनाने के सलए एक प्रस्ताव पाररत सकया है।
उत्तर- 1)िेंद्रीय मंनत्रमंिि िे नित्तीय िषा 2023-24 से 2027-28 ति 5 िषों में 84 िाि LPG ििेक्शि िारी
िरिे िे निए प्रधाि मंत्री उज्ज्विा योििा (PMUY) 2.0 िे निस्तार िो मंिूरी दे दी।
स्पष्टीिरण:
प्रर्ान मोंत्री (PM) नरें द्र मोदी की अध्यक्षता में केंद्रीय मोंसत्रमोंडल ने 13 ससतोंबर 2023 को सनम्नसलक्कखत प्रस्तावोों को मोंजूरी
दे दी है ।
• केंद्रीय मोंसत्रमोंडल ने 9 से 10 ससतोंबर, 2023 तक नई सदल्ली में भारत िारा पहली बार आयोसजत 18वें G20
सशखर सम्मेलन की सफलता का जश्न मनाने के सलए एक प्रस्ताव पाररत सकया है।
• केंद्रीय मोंसत्रमोंडल ने सवत्तीय वषध 2023-24 से 2025-26 तक 3 िषों में 75 िाि LPG (तरलीकृत पेटरोसलयम
गैस) कनेक्शन जारी करने के सलए प्रर्ान मोंत्री उज्ज्वला योजना (PMUY) 2.0 के सवस्तार को मोंजूरी दे दी।
• केंद्रीय मोंसत्रमोंडल ने 7210 करोड रुपये के सवत्तीय पररव्यय के साि 4 वषों (2023 से शुरू) के सलए केंद्रीय क्षेत्र
योजना के रूप में ई-कोटध प्रोजेर चरि III को मोंजूरी दे दी है ।
• भारत और 3 अन्य दे शोों के बीच MoU को मोंजूरी: केंद्रीय मोंसत्रमोंडल ने समझौता ज्ञापन (MoU) को मोंजूरी दे
दी, सजस पर जून 2023 में भारत के इलेररॉसनक्स और सूचना प्रौद्योसगकी मोंत्रालय (Meity) और आमेसनया
गिराज्य के उच्च तकनीक उद्योग मोंत्रालय िारा हस्ताक्षर सकए गए िे।
• माननीय प्रर्ान मोंत्री की अध्यक्षता में केंद्रीय मोंसत्रमोंडल ने 12 जून 2023 को भारत गिराज्य के इलेररॉसनक्स
और सूचना प्रौद्योसगकी मोंत्रालय औरससएरा सलयोन गिराज्य के सूचना और सोंचार मोंत्रालय के बीच समझौता
ज्ञापन (MoU) पर हस्ताक्षर करने को मोंजूरी दी।
• केंद्रीय मोंसत्रमोंडल ने एक और MoU को भी मोंजूरी दे दी, सजस पर जून 2023 में भारत सरकार के Meity और
एों टीगुआ & बारबुडा के सूचना, सोंचार प्रौद्योसगकी, उपयोसगता और ऊजाध मोंत्रालय के बीच हस्ताक्षर सकए गए
िे।
• आसिधक मामलोों की मोंसत्रमोंडल ससमसत (CCEA) ने नेशनल स्ट्ॉक एक्सचेंज (NSE) सलसमटे ड और बॉम्बे स्ट्ॉक
एक्सचेंज सलसमटे ड (BSE) ऑफ इों सडया में सूचीबद्ध सावधजसनक सलसमटे ड भारतीय फामाधस्युसटकल कोंपनी M/s
सुवेन फामाधस्यूसटकल्स सलसमटे ड में सवदे शी सनवेश के सलए प्रत्यक्ष सवदे शी सनवेश (FDI) प्रस्ताव को मोंजूरी दे दी।

Report Errors in the PDF - ebooks@affairscloud.com Copyright 2014-2023 @ AffairsCloud.com 19


26. उस सरिारी एिेंसी/NGO िा िाम बताइए निसिे हाि ही में (नसतंबर'23 में) िंनबत मामिों िो सुिझािे
और सरिारी िायााियों िे भीतर स्वच्छता (स्वच्छता) िो बढ़ािा दे िे िे निए अपिा निशेष अनभयाि 2.0 पूरा
निया है।
1)सचोंतन
2)NITI आयोग
3)ATREE
4)सोंकल्पतरु फाउों डेशन
5)मुक्कक्त
उत्तर- 2)NITI आयोग
स्पष्टीिरण:
नेशनल इों स्ट्ीट्यूशन फॉर टर ाोंसफॉसमिंग इों सडया (NITI आयोग)
ने अपना निशेष अनभयाि 2.0 सफलतापूवधक पूरा कर सलया
है, सजसका उद्दे श्य लोंसबत मामलोों को सुलझाने और सरकारी
कायाधलयोों के भीतर क्लीनलीनेस (स्वच्छता) को बढावा दे ना है।
i.कासमधक, लोक सशकायत और पेंशन मोंत्रालय के तहत
प्रशाससनक सुर्ार और लोक सशकायत सवभाग (DARPG) िारा
शुरू सकया गया सवशेष असभयान 2.0 नवोंबर 2022 से अगस्त 2023 तक चलाया गया िा।
ii.असभयान अवसर् के दौरान लगभग 95% सावधजसनक सशकायतोों और अपीलोों का सफलतापूवधक समार्ान सकया गया।

27. नसतंबर 2023 में, िेंद्रीय िौशि नििास और उद्यनमता मंत्री धमेंद्र प्रधाि िे खस्कि इं निया
________________ (SID) प्लेटफॉमा िॉन्च निया।
1)डे वलपमेंट
2)डै शबोडध
3)सडपाटध मेंट
4)डर ाइव
5)सडसजटल
उत्तर- 5)निनिटि
स्पष्टीिरण:
केंद्रीय कौशल सवकास और उद्यसमता मोंत्री र्मेंद्र प्रर्ान (MSDE) ने क्कस्कल इों सडया निनिटि (SID) प्लेटफॉमध लॉन्च सकया
है।
• SID कौशल, सशक्षा, रोजगार और उद्यसमता के सलए भारत का सडसजटल पक्किक इों फ्रास्ट्र क्चर (DPI) है।
• सडसजटल प्लेटफ़ॉमध में एक वेबसाइट और एक मोबाइल एक्कप्लकेशन दोनोों शासमल हैं सजसे राष्ट्रीय कौशल सवकास
सनगम (NSDC) िारा सवकससत सकया गया िा। यह कई भारतीय भाषाओों में उपलब्ध है।
• क्कस्कल इों सडया सडसजटल प्लेटफॉमध इच्छु क उद्यसमयोों के सलए सवसभन्न प्रकार के उद्योग-सवसशष्ट् कौशल पाठ्यक्रम,
कैररयर के अवसर और सहायता प्रदान करता है ।
• इसमें गूगल, माइक्रोसॉफ्ट, ससस्को और क्कस्कल काउों ससल फॉर ग्रीन जॉब्स (SCGJ) के पाठ्यक्रमोों की सूची शासमल
है।

Report Errors in the PDF - ebooks@affairscloud.com Copyright 2014-2023 @ AffairsCloud.com 20


28. नसतंबर 2023 में, िॉपोरे ट मामिों िे मंत्रािय (MCA) िे िंपिी िािूि सनमनत िा िायािाि
____________ ति बढ़ा नदया।
1)16 जनवरी 2024
2)16 ससतोंबर 2025
3)16 ससतोंबर 2026
4)16 ससतोंबर 2024
5)16 माचध 2024
उत्तर- 4)16 नसतंबर 2024
स्पष्टीिरण:
कॉरपोरे ट मामलोों के मोंत्रालय (MCA) ने कोंपनी कानून ससमसत का कायधकाल 1 साि बढाकर 16 नसतंबर, 2024 तक
कर सदया है।
• कोंपनी कानून ससमसत की थिापना 2019 में एक वषध (2020) के कायधकाल के साि की गई िी, और इसका
कायधकाल साल दर साल क्रसमक रूप से बढाया गया िा।
• ससमसत को कोंपनी असर्सनयम, 2013 और सीसमत दे यता भागीदारी (LLP) असर्सनयम, 2008 के कायाधन्वयन से
सोंबोंसर्त सवसभन्न मामलोों पर सरकार की जाोंच करने और ससफाररश करने का काम सौोंपा गया है।
• वतधमान में, कोंपनी कानून ससमसत में MCA के ससचव डॉ. मनोज गोसवल की अध्यक्षता में 11 सदस्य शासमल हैं ।

29. निम्ननिखित में से निस एसोनसएशि/NGO/संग्रहािय िा निदे शी अंशदाि निनियमि अनधनियम (FCRA)
िाइसेंस हाि ही में (नसतंबर'23 में) गृह मंत्रािय (MHA) द्वारा रद्द िर नदया गया है ?
1)हेल्पएज भारत
2)श्रीसनवास मक्कल्लया मेमोररयल सिएटर क्राफ्ट् स सोंग्रहालय
3)SELF EMPLOYED WOMEN’S ASSOCIATION (SEWA)
4)1 और 2 दोनोों
5)2 और 3 दोनोों
उत्तर- 5)2 और 3 दोिों
स्पष्टीिरण:
गृह मोंत्रालय (MHA) ने बाि रक्षा भारत, श्रीनििास मखल्लया मेमोररयि नथएटर क्राफ्ट् स संग्रहािय और SELF
EMPLOYED WOMEN’S ASSOCIATION (SEWA) का सवदे शी योगदान सवसनयमन असर्सनयम (FCRA) लाइसेंस रद्द
कर सदया है।
i.गैर-सरकारी सोंगठनोों (NGO) के सलए नवीकरिीय FCRA लाइसेंस की समय सीमा (30 ससतोंबर 2023)से पहले लाइसेंस
रद्द कर सदए गए िे।
ii.बाल रक्षा भारत यूनाइटे ड सकोंगडम (UK) क्कथित NGO "सेव द सचडरेन" की भारत इकाई है जो पूरे भारत में लगभग 16
राज्योों में सोंचासलत होती है।
iii.श्रीसनवास मक्कल्लया मेमोररयल सिएटर सशल्प सोंग्रहालय की थिापना स्वतोंत्रता सेनानी उल्लाल श्रीसनवास मक्कल्लया की
स्मृसत में कठपुतली सनमाधि, मुखौटा सनमाधि, कागज की आकृसतयााँ , बाोंस सशल्प और अन्य जैसे पारों पररक हस्तसशल्प में
प्रसशक्षि प्रदान करने के सलए की गई िी।

Report Errors in the PDF - ebooks@affairscloud.com Copyright 2014-2023 @ AffairsCloud.com 21


30. निस अंतरााष्टरीय संगठि िे हाि ही में (नसतंबर’23 में) महामारी िी तैयारी िे निए एि स्वास्थ्य दृनष्टिोण
िो मिबूत िरिे िे निए पशुपािि और िे यरी निभाग (AH&D) िे साथ सहयोग निया है ?
1)सवश्व स्वास्थ्य सोंगठन
2)सवश्व पशु स्वास्थ्य सोंगठन
3)सोंयुक्त राष्ट्र मानव सनपटान कायधक्रम
4)वडध वाइड फोंड फॉर नेचर
5)अोंतराधष्ट्रीय मु द्रा कोष
उत्तर-2)निश्व पशु स्वास्थ्य संगठि
स्पष्टीिरण:
सवश्व पशु स्वास्थ्य सोंगठन (WOAH) के सहयोग से मत्स्य पालन पशुपालन और डे यरी मोंत्रालय के तहत पशुपालन और
डे यरी सवभाग (AH&D) ने "ररस्क-बेथड मैनेजमेंट ऑफ़ क्कस्पल ओवर इवेंट्स इन वाइडलाइफ इन इों सडया" शीषधक से
एक बहु-क्षेत्रीय कायधशाला का आयोजन सकया।
i.कायधशाला का उद्दे श्य एक स्वास्थ्य दृसष्ट्कोि, अोंतर-क्षेत्रीय सहयोग और महामारी सोंबोंर्ी तैयाररयोों के महत्व को
मजबूत करना है।
ii.चार मुख्य उद्दे श्य:
• वन्यजीव-उत्पसत्त रोग जोक्कखम सवश्लेषि के बारे में सहतर्ारकोों का ज्ञान बढाना
• भारत के जोक्कखम मूल्ाोंकन और प्रबोंर्न का व्यापक अोंतर सवश्लेषि करना
• रोग फैलने के पररदृश्योों का अनुकरि करना
• प्रासोंसगक सहतर्ारकोों के बीच सोंचार और जागरूकता को बढावा दे ना।
iii.'एक स्वास्थ्य' लोगोों, जानवरोों और पयाधवरि के स्वास्थ्य को सोंतुसलत और अनुकूसलत करने के सलए एक एकीकृत,
एकीकृत दृसष्ट्कोि है।

31. नसतंबर 2023 में, िेंद्रीय मत्स्य पािि, पशुपािि और िे यरी मंत्री पुरुषोत्तम रूपािा िे भारतीय िृनष
अिुसंधाि पररषद (ICAR)- ______________ िे समथाि से एग्रीिल्चर इं श्योरें स िंपिी ऑफ इं निया निनमटे ि
(AIC) द्वारा नििनसत झी ंगा फसि बीमा योििा शुरू िी
1)केंद्रीय समुद्री मत्स्य अनुसोंर्ान सोंथिान
2)राष्ट्रीय जलकृसष सोंसार्न ब्यूरो
3)केंद्रीय मत्स्य प्रौद्योसगकी सोंथिान
4)राष्ट्रीय मछली आनुवोंसशक सोंसार्न ब्यूरो
5)केंद्रीय खारा जल जलकृसष सोंथिान
उत्तर-5)िेंद्रीय िारा िि िििृनष संस्थाि
स्पष्टीिरण:
केंद्रीय मत्स्य पालन, पशुपालन और डे यरी मोंत्रालय (MoFAHD) के मोंत्री पुरुषोत्तम रूपाला ने भारतीय कृसष अनुसोंर्ान
पररषद (ICAR) – केंद्रीय खारा जलजीव पालन सोंथिान (CIBA) के समिधन से एग्रीकल्चर इों श्योरें स कोंपनी ऑफ इों सडया
सलसमटे ड (AIC) िारा सवकससत झी ंगा फसि बीमा योििा शुरू की।
i.यह योजना 14 ससतोंबर 2023 को गुजरात के नवसारी में नवसारी कृसष सवश्वसवद्यालय पररसर में ICAR-CIBA के झीोंगा
सकसान कॉन्क्क्लेव 2023 के दू सरे सोंस्करि के दौरान शुरू की गई िी।
ii.कॉन्क्क्लेव के दौरान, CIBA ने 2 समझौता ज्ञापनोों (MoU) पर हस्ताक्षर सकए:

Report Errors in the PDF - ebooks@affairscloud.com Copyright 2014-2023 @ AffairsCloud.com 22


iii.CIBA ने NFDB िारा प्रस्तासवत प्रीसमयम सक्कब्सडी के साि जलीय कृसष के सलए फसल बीमा लागू करने के सलए
राष्ट्रीय मत्स्य सवकास बोडध (NFDB) के साि एक MoU पर हस्ताक्षर सकए।
iv.CIBA ने प्रौद्योसगकी सहायता प्रदान करने के सलए गुजरात के मछली सकसान उत्पादक सोंगठन (FFPO) के साि एक
MoU पर भी हस्ताक्षर सकए।

32. नसतंबर 2023 में, भौगोनिि संिेत रनिस्ट्र ी (GIR) िे िाद्य सामग्री श्रेणी िे तहत ___________ (राज्य) िे
'अत्रेयपुरम पूथारे िुिु' िो भौगोनिि संिेत (GI) टै ग प्रदाि निए।
1)केरल
2)तसमलनाडु
3)आों ध्र प्रदे श
4)तेलोंगाना
5)कनाधटक
उत्तर-3)आं ध्र प्रदे श
स्पष्टीिरण:
भौगोसलक सोंकेत रसजस्ट्र ी (GIR) ने आं ध्र प्रदे श की चावल और गुड से बनी समठाई 'अत्रेयपुरम पूथारे िुिुह’ को
भौगोसलक सोंकेत (GI) टै ग प्रदान सकया है।
उद्योग सोंवर्धन और आों तररक व्यापार सवभाग (DIPIT), वासिज्य और उद्योग मोंत्रालय (MoCI) के तहत GIR ने 'खाद्य
सामग्री' श्रेिी के तहत 'अत्रेयपुरम पूिारे कुलुह’ समठाई सकस्म को पोंजीकृत सकया िा।
• आत्रेयपुरम आों ध्र प्रदे श के कोनसीमा सजले का एक गाों व है।
i.GI ससटध सफकेट हैंडओवर समारोह दामोदरम सोंजीवय्या नेशनल लॉ यूसनवससधटी (DSNLU), सवशाखापत्तनम में
"ज्योग्रासफकल इों सडकेशर्न् एों ड GI ससटध सफकेट हैंडओवर सेरेमनी फॉर अत्रेयपुरम पूिारे कुलुह" पर राष्ट्रीय कायधशाला
के अवसर पर आयोसजत सकया गया िा।

33. हाि ही में (नसतंबर'23 में) निस राज्य/UT िे उधमपुर रे ििे स्ट्े शि िा िाम बदििर "शहीद िैप्टि तुषार
महािि रे ििे स्ट्े शि" िर नदया है ?
1)सहमाचल प्रदे श
2)लद्दाख
3)जम्मू और कश्मीर
4)उत्तराखोंड
5)उत्तर प्रदे श
उत्तर-3)िम्मू-िश्मीर
स्पष्टीिरण:
जम्मू और कश्मीर (J&K) प्रशासन ने जम्मू-कश्मीर के उर्मपुर सजले में उधमपुर रे ििे स्ट्े शि का नाम बदलकर
"शहीद कैप्टन तुषार महािि रे ििे स्ट्े शि" करने का आदे श पाररत सकया है।
• नाम बदलने को गृह मोंत्रालय (MHA) िारा मोंजूरी दे दी गई िी।
i.J&K के उर्मपुर की आदशध कॉलोनी के कैप्टन तुषार महाजन 9 PARA (भारतीय सेना के सवशेष बल) के एक असर्कारी
िे।

Report Errors in the PDF - ebooks@affairscloud.com Copyright 2014-2023 @ AffairsCloud.com 23


ii.फरवरी 2016 में पुलवामा सजले में जम्मू और कश्मीर उद्यसमता सवकास सोंथिान (JKEDI) भवन पर आतोंकवादी हमले
के दौरान वह शहीद हो गए। हमले के दौरान अपने सािी सेना कसमधयोों की रक्षा करते हुए उन्ोोंने एक आतोंकवादी को
गोली मार दी।

34. नसतंबर 2023 में, रक्षा अनधग्रहण पररषद (DAC) िे निनभन्न हनथयार प्रणानियों और प्लेटफामों िे पूंिी
अनधग्रहण प्रस्तािों िे निए ____________ रुपये िी आिश्यिता िी स्वीिृनत (AoN) प्रदाि िी।
1)55,000 करोड
2)45,000 करोड
3)40,000 करोड
4)30,000 करोड
5)35,000 करोड
उत्तर- 2)45,000 िरोड
स्पष्टीिरण:
रक्षा असर्ग्रहि पररषद (DAC) ने 15 ससतोंबर 2023 को केंद्रीय रक्षा मोंत्री राजनाि ससोंह की अध्यक्षता में अपनी बैठक में
45,000 िरोड रुपये की सवसभन्न हसियार प्रिासलयोों और प्लेटफामों के पूोंजी असर्ग्रहि प्रस्तावोों के सलए आवश्यकता
की स्वीकृसत (AoN) प्रदान की।
• ये प्रस्ताव भारतीय रक्षा बलोों की पररचालन क्षमताओों को मजबूत करने और रक्षा पररयोजनाओों में स्वदे शी सामग्री
के समावेश को बढाने का प्रयास करते हैं।
• रक्षा असर्ग्रहि पररषद (DAC) ने नौ खरीद प्रस्तावोों को मोंजूरी दे दी है , और ये सभी बाय इों सडयन-स्वदे शी रूप
से सडजाइन, सवकससत और सनसमधत (IDDM) श्रेिी के तहत स्वदे शी सवक्रेताओों से प्राप्त की जाएों गी।
• रक्षा मोंत्री राजनाि ससोंह ने उच्चस्तरीय रक्षा असर्ग्रहि पररषद की बैठक के दौरान शीषध सैन्य असर्काररयोों और
सरकारी नौकरशाहोों को रक्षा क्षेत्र में IDDM पररयोजनाओों के सलए न्यूनतम 60-65% स्वदे शी सामग्री सुसनसित
करने का सनदे श सदया।
• वतधमान में, रक्षा पररयोजनाओों में स्वदे शी सामग्री की सीमा 50% है।

35. नसतंबर 2023 में, NITI आयोग िे सदस्य प्रोफेसर रमेश चंद िे िृनष में िे टा संबंधी शासि चुिौनतयों िा
समाधाि िरिे िे निए __________ (निभाग/मंत्रािय/सरिारी एिेंसी) द्वारा नििनसत िृनष सांखख्यिी िे निए
एिीिृत पोटा ि (UPAg) िॉन्च निया।
1)सवज्ञान एवों प्रौद्योसगकी मोंत्रालय
2)ग्रामीि और औद्योसगक सवकास में अनुसोंर्ान केंद्र
3)कृसष एवों सकसान कल्ाि सवभाग
4)फेडरे शन ऑफ इों सडयन चैंबसध ऑफ कॉमसध एों ड इों डस्ट्र ी
5)साोंक्कख्यकी और कायधक्रम कायाधन्वयन मोंत्रालय
उत्तर-3)िृनष एिं निसाि िल्याण निभाग
स्पष्टीिरण:
नेशनल इों स्ट्ीट्यूशन फॉर टर ाोंसफॉसमिंग इों सडया (NITI) आयोग के सदस्य प्रोफेसर रमेश चंद ने कृसष और सकसान
कल्ाि मोंत्रालय (MoA&FW) के तहत कृसष और सकसान कल्ाि सवभाग (DoA&FW) िारा सवकससत कृसष
साोंक्कख्यकी के सलए एकीकृत पोटध ल (UPAg) लॉन्च सकया, तासक कृसष में डे टा से सोंबोंसर्त शासन चुनौसतयोों का समार्ान
सकया जा सके।

Report Errors in the PDF - ebooks@affairscloud.com Copyright 2014-2023 @ AffairsCloud.com 24


i.कृसष साोंक्कख्यकी के सलए एकीकृत पोटध ल (UPAg) एक उन्नत कृसष डे टा प्रबोंर्न मोंच है जो फसल अनुमान उत्पन्न करता
है और मूल्, व्यापार, खरीद और स्ट्ॉक जैसे कृसष साों क्कख्यकी प्रदान करने वाली अन्य प्रिासलयोों के साि एकीकृत होता
है।
ii.UPAg का इरादा सूसचत सनिधय लेने में सक्षम बनाने के सलए सोंपूिध अोंतदृधसष्ट् प्रदान करके नीसत सनमाधताओों, शोर्कताधओों
और सकसानोों ससहत कृसष सहतर्ारकोों की मदद करना है।

36. नसतंबर 2023 में प्रधाि मंत्री (PM) िरें द्र मोदी द्वारा निए गए हाि ही में निए गए समनपात/िॉन्च िे संबंध
में निम्ननिखित में से िौि सा नबंदु "सही" है /हैं ?
A) प्रधाि मंत्री (PM) िरें द्र मोदी िे निश्विमाा ियंती िे अिसर पर इं निया इं टरिेशिि िन्वेंशि एं ि एक्सपो
सेंटर (IICC) िे चरण -1 िो समनपात निया, निसे यशोभूनम िे रूप में भी िािा िाता है।
B) उन्ोंिे इसिे िोगो, टै गिाइि ''सम्माि समथा समृखि'' और पोटा ि िे साथ िारीगरों और नशल्पिारों िे
18 फोिस क्षेत्रों िो ििर िरिे िािी PM निश्विमाा योििा भी िॉन्च िी।
C) इस अिसर पर उन्ोंिे एि अिुिूनित स्ट्ाम्प शीट, एि टू ि निट ई-बुििेट और िीनियो िारी निया।
1)केवल A
2)केवल A & B
3)केवल B & C
4)केवल A & C
5)सभी A, B & C
उत्तर -5)सभी A, B & C
स्पष्टीिरण:
17 ससतोंबर, 2023 को, यानी निश्विमाा ियंती के अवसर पर, भारत के प्रर्ान मोंत्री (PM) िरें द्र मोदी ने इों सडया
इों टरनेशनल कन्वेंशन एों ड एक्सपो सेंटर (IICC) के चरण -1 को राष्ट्र को समसपध त सकया, सजसे यशोभूनम भी कहा जाता
है, जो िारका सेरर 25, नई सदल्ली, सदल्ली में क्कथित है ।
i.IICC भारत का सबसे बडा सम्मेलन और प्रदशधनी केंद्र है सजसका कुल इनडोर क्षेत्र 300,000m2 है।
ii.उन्ोोंने इसके लोगो, टै गलाइन ''सम्माि समथा समृखि'' और पोटध ल (https://pmvishwakarma.gov.in/) के साि
कारीगरोों और सशल्पकारोों के 18 फोकस क्षेत्रोों को कवर करने वाली PM निश्विमाा योििा भी लॉन्च की।
iii.इस अवसर पर उन्ोोंने एक अनुकूसलत स्ट्ाम्प शीट, एक टू ि निट ई-बुििेट और वीसडयो जारी सकया।
iv.लॉन्च/ररलीज़ IICC, नई सदल्ली से सकया गया िा।

37. निस मंत्रािय िे हाि ही में (नसतंबर’23 में) एि एिीिृत िायाािय पररसर और एि ई-िॉिेट (भारतिोष
अनग्रम िमा) सुनिधा 'उडाि भिि' िा उद् घाटि निया है ?
1)गृह मोंत्रालय
2)सवत्त मोंत्रालय
3)नागररक उड्डयन मोंत्रालय
4)सडक पररवहन और राजमागध मोंत्रालय
5)कॉपोरे ट मामलोों का मोंत्रालय
उत्तर-3)िागररि उड्डयि मंत्रािय

Report Errors in the PDF - ebooks@affairscloud.com Copyright 2014-2023 @ AffairsCloud.com 25


स्पष्टीिरण:
18 ससतोंबर 2023 को, िागररि उड्डयि मंत्रािय (MoCA) के केंद्रीय मोंत्री ज्योसतरासदत्य ससोंसर्या ने सदल्ली के
सफदरजोंग हवाई अड्डे पर एक एकीकृत कायाधलय पररसर 'उडाि भिि' और MoCA के सलए एक ई-िॉिेट
(भारतकोष असग्रम जमा) सुनिधा का उद् घाटन सकया।
i.उडान भवन, नया एकीकृत कायाधलय सभी सोंगठनोों को एक ही छत के नीचे एक साि काम करने की अनुमसत दे गा और
इससे MoCA के तहत सवसभन्न सनयामक प्रासर्करिोों जैसे DGCA, BCAS, AAIB, AERA और AAI के बीच बेहतर समन्वय
होगा।
ii.न्यू इों टीग्रेटेड ऑसफस कॉम्प्लेक्स में G+3 लेवल जमीन के ऊपर और बाकी 3 बेसमेंट लेवल के रूप में बनाए गए हैं
और इसमें 1270 असर्कारी रह सकते हैं।
iii.नई इमारत, प्रर्ान मोंत्री के 'सोंकल्प से ससक्कद्ध' के आह्वान के साि अपने तालमेल पर जोर दे ती है।
iv. ई-वॉलेट भारत कोष पोटध ल में सवसभन्न सनयामक अनुमोदनोों के सलए प्रसोंस्करि शुल्क के सलए सवशेष रूप से
उपयोगी होगा।

38. नसतंबर 2023 में िेंद्रीय नित्त मंत्री निमािा सीतारमण और िेंद्रीय िृनष और निसाि िल्याण मंत्री
(MoAFW) िरें द्र नसंह तोमर द्वारा निए गए हाि ही िे िॉन्च िे संबंध में निम्ननिखित में से िौि सा/से नबंदु
"सही" है/हैं?
A) िेंद्रीय मंनत्रयों िे निसाि ऋण पोटा ि (KRP) िा उद् घाटि निया, निसिा उद्दे श्य निसािों िो KCC योििा
िे तहत सखििी िािे ऋण दे िा है , और उन्ें MISS िे माध्यम से सखििी िािे िृनष ऋण प्राप्त िरिे में
सहायता िरिा है।
B) िायाक्रम िे दौराि, घर-घर KCC अनभयाि भी िॉन्च निया गया, िो 1 अरू बर, 2023 से 31 नदसंबर,
2023 ति चिेगा, निसिा िक्ष्य प्रधािमंत्री निसाि सम्माि निनध (PM-KISAN) िे िगभग 1.5 िरोड
िाभानथायों िो िोडिा है , िो अभी ति KCC योििा से िही ं िुडे हैं।
C) िेदर िेटििा िे टा नसस्ट्म्स (WINDS) पहि िे प्रभाि िो बढ़ािे िे निए WINDS मैिुअि भी िॉन्च निया
गया।
1)सभी A, B & C
2)केवल A & B
3)केवल B & C
4)केवल A & C
5)केवल A
उत्तर- 1)सभी A, B & C
स्पष्टीिरण:
19 ससतोंबर, 2023 को केंद्रीय मोंत्री निमािा सीतारमण, सवत्त मोंत्रालय; केंद्रीय कृसष और सकसान कल्ाि मोंत्रालय
(MoAFW) के केंद्रीय मोंत्री िरें द्र नसंह तोमर ने पूसा कॉम्प्लेक्स, नई सदल्ली, सदल्ली में एक कायधक्रम के दौरान सकसान
ररन पोटध ल (KRP), मौसम सूचना नेटवकध डे टा ससस्ट्म (WINDS) मैिुअि और डोर-टू -डोर सकसान क्रेसडट काडध
(KCC) असभयान (घर-घर KCC अनभयाि) का उद् घाटन सकया।
• ये पहल कृसष-ऋि (KCC & MISS) और फसल बीमा (PMFBY/RWBCIS) पर केंसद्रत हैं ।
• MISS का मतलब सोंशोसर्त ब्याज सहायता योजना, PMFBY का मतलब प्रर्ानमोंत्री फसल बीमा योजना और
RWBCIS का मतलब पुनगधसठत मौसम आर्ाररत फसल बीमा योजना है।

Report Errors in the PDF - ebooks@affairscloud.com Copyright 2014-2023 @ AffairsCloud.com 26


• सकसान ऋि पोटध ल (KRP) का उद्दे श्य सकसानोों को KCC योजना के तहत सक्कब्सडी वाले ऋि दे ना और MISS के
माध्यम से सक्कब्सडी वाले कृसष ऋि प्राप्त करने में सहायता करना है ।
• कायधक्रम के दौरान, घर-घर KCC असभयान भी लॉन्च सकया गया, जो 1 अरू बर, 2023 से 31 सदसोंबर, 2023
तक चलेगा, सजसका लक्ष् प्रर्ानमोंत्री सकसान सम्मान सनसर् (PM-KISAN) के लगभग 1.5 करोड लाभासिधयोों को
जोडना है , जो अभी तक KCC योजना से नहीों जुडे हैं।
• WINDS मैनुअल को वेदर नेटवकध डे टा ससस्ट्म्स (WINDS) पहल के प्रभाव को बढाने के सलए लॉन्च सकया गया
है, सजसे कृसष पर सूसचत मौसम सोंबोंर्ी सनिधय लेने के सलए उन्नत मौसम डे टा एनासलसटक्स का उपयोग करने के
सलए जुलाई 2023 में लॉन्च सकया गया िा।

39. भारतीय िौसेिा िे िौसेिा ििाचार और स्वदे शीिरण संगठि (NIIO) सेनमिार 'स्वाििंबि 2023' िा
___________ संस्करण अरू बर, 2023 में िई नदल्ली, नदल्ली में आयोनित होिे िािा है।
1)तीसरा
2)चौिा
3)दू सरा
4)पाोंचवाों
5)पहला
उत्तर- 3)दू सरा
स्पष्टीिरण:
भारतीय नौसेना के नौसेना नवाचार और स्वदे शीकरि सोंगठन (NIIO) सेनमिार 'स्वाििंबि 2023' का दू सरा सोंस्करि
4-5 अरू बर, 2023 को नई सदल्ली, सदल्ली में आयोसजत होने वाला है।
i.सेसमनार में 'SPRINT चैलेंजेज' पहल के तहत स्ट्ाटध -अप िारा सवकससत स्वदे शी प्रौद्योसगसकयोों/उत्पादोों के 75 प्रोटोटाइप
प्रदसशधत सकए जाएों गे।
ii.SPRINT भारतीय नौसेना, रक्षा नवाचार सोंगठन (DIO), रक्षा उत्कृष्ट्ता के सलए नवाचार (iDEX), NIIO और प्रौद्योसगकी
सवकास त्वरि सेल (TDAC) के बीच एक सहयोगी पहल है।
िोट: सेसमनार का पहला सोंस्करि जुलाई 2022 में नई सदल्ली, सदल्ली में आयोसजत सकया गया िा। प्रर्ान मोंत्री मोदी ने
'SPRINT' पहल के एक भाग के रूप में स्ट्ाटध -अप/MSME के सलए 75 चुनौसतयोों की शुरुआत की।

40. हाि ही में (लसतंबर’23 में) लकस भारत सरकार कायाािय/प्रालिकरण ने दू रसंचार क्षेत्र के लिए भारतीय
मानक समय (IST) टर े सेबि प्राथलमक संदभा समय घडी (PRTC) लवकलसत करने के लिए वैज्ञालनक और
औद्योलगक अनुसंिान पररषद (CSIR)-राष्टरीय भौलतक प्रयोगशािा (NPL) के साथ एक समझौते पर हस्ताक्षर
लकए हैं?
1)राष्ट्रीय सूचना श्चवज्ञान केंद्र
2)भारतीय टे लीफोन इं िस्ट्र ीि श्चलश्चमटे ि
3)भारतीय दू रसंचार श्चनयामक प्राश्चर्करि
4)उन्नत अनुसंर्ान एवं श्चवकास केंद्र
5)टे लीमैश्चटक्स श्चवकास केंद्र
उत्तर-5)टे िीमैलटक्स लवकास केंद्र

Report Errors in the PDF - ebooks@affairscloud.com Copyright 2014-2023 @ AffairsCloud.com 27


स्पष्टीकरण:
नई श्चदल्ली, श्चदल्ली में टे लीमैश्चटक्स श्चवकास केंद्र (C-DOT) और वैज्ञाश्चनक और औद्योश्चगक अनुसंर्ान पररषद (CSIR)-
राष्ट्रीय भौश्चतक प्रयोगशाला (NPL) ने सभी दू रसंचार सेवाओं को एक संदभग समय के साि श्चसंक्रनाइज़ करने के श्चलए
भारतीय मानक समय (IST) टर े सेबल प्रािश्चमक संदभग समय घड़ी (PRTC) श्चवकश्चसत करने के श्चलए एक समझौते पर
हस्ताक्षर श्चकए।
i.समय घड़ी एक क्षेत्रीय नेश्चवगेशन उपग्रह प्रिाली पर आर्ाररत होगी श्चिसे नेश्चवगेशन श्चवद इं श्चियन कांस्ट्ेलेशन (NavIC)
कहा िाता है।
ii.दू रसंचार श्चवभाग (DoT) की दू रसंचार प्रौद्योश्चगकी श्चवकास श्चनश्चर् (TTDF) योिना के तहत समझौते पर हस्ताक्षर श्चकए
गए।
iii.C-DOT के मुख्य कायगकारी अश्चर्कारी (CEO) रािकुमार उपाध्याय की उपस्थिश्चत में समझौते पर हस्ताक्षर श्चकए गए
iv. प्रािश्चमक संदभग समय घड़ी (PRTC) का लक्ष्य सभी दू रसंचार सेवा प्रदाताओं (TSP) और इं टरनेट सेवा प्रदाताओं
(ISP) को ± 20 नैनोसेकंि के भीतर प्रत्यक्ष IST टर ै सेश्चबश्चलटी प्रदान करना है।

41. लसतंबर 2023 में, केंद्रीय कौशि लवकास और उद्यलमता मंत्री िमेंद्र प्रिान ने राष्टरीय कौशि लवकास लनगम
(NSDC) और __________ (बैंक) के सहयोग से 'स्किल्स ऑन व्हील्स' पहि शुरू की।
1)एस्क्सस बैंक
2)भारतीय स्ट्े ट बैंक
3)इं िसइं ि बैंक
4)ICICI बैंक
5)HDFC बैंक
उत्तर-3)इं िसइं ि बैंक
स्पष्टीकरण:
केंद्रीय मंत्री र्मेंद्र प्रर्ान, कौशल श्चवकास और उद्यश्चमता मंत्रालय (MSDE) ने राष्ट्रीय कौशल श्चवकास श्चनगम (NSDC)
और इं िसइं ि बैंक के सहयोग से 'स्किि ऑन व्हील्स' पहि शुरू की। MSDE मंत्री और लोकसभा अध्यक्ष श्री ओम
श्चबड़ला ने संयुक्त रूप से कायगक्रम समारोह के उद् घाटन के दौरान पहल शुरू की।
• ग्रामीि युवाओं, श्चवशेषकर मश्चहलाओं को प्रमुख श्चिश्चिटल कौशल से लैस करना और उन्हें नौकरी और भश्चवष्य
के श्चलए तैयार करना है।
• यह पहल ग्रामीि पररवारों की आिीश्चवका में सुर्ार के श्चलए प्रासंश्चगक कौशल प्रश्चशक्षि के साि 5 वषों की
अवश्चर् में 60,000 युवाओं को सशक्त बनाएगी।

42. उस राज्य सरकार का नाम बताइए लिसने हाि ही में (लसतंबर'23 में) सरकारी स्वास्थ्य सुलविाओं पर
नविात लशशुओ ं के लिए िन्म पंिीकरण प्रमाण पत्र तत्काि िारी करने में सक्षम बनाने के लिए अपने मां नविात
टर ै लकंग ऐप (MaNTrA) को िन्म पंिीकरण प्रणािी के साथ एकीकृत लकया है।
1)महाराष्ट्र
2)उत्तर प्रदे श
3)मध्य प्रदे श
4)पश्चिम बंगाल
5)ओश्चिशा
उत्तर- 2)उत्तर प्रदे श

Report Errors in the PDF - ebooks@affairscloud.com Copyright 2014-2023 @ AffairsCloud.com 28


स्पष्टीकरण:
उत्तर प्रदे श (UP) सरकार ने अपने मां नवाित टर ै श्चकंग ऐप (MaNTrA) को िन्म पंिीकरण प्रणािी के साि एकीकृत
श्चकया है, ताश्चक UP में सरकारी स्वास्थ्य सुश्चवर्ाओं पर नविात श्चशशुओं के श्चलए िन्म पंिीकरि प्रमाि पत्र तत्काल िारी
श्चकया िा सके।
i.इससे नागररक सुश्चवर्ा ब़िे गी और माता-श्चपता को अलग से आवेदन करने की आवश्यकता समाप्त हो िाएगी।
ii.इस श्चवकास के साि, UP नागररक पंिीकरि प्रिाली (CRS) पोटग ल के माध्यम से सत्याश्चपत सरकारी स्वास्थ्य संथिानों
के श्चलए 'स्वत: संज्ञान' स्वचाश्चलत िन्म पंिीकरि लागू करने वाला पहला राज्य बन गया है।
iii.MaNTrA ऐप, श्चिसे मूल रूप से UNICEF के सहयोग से UP में श्चवकश्चसत MaNTrA ऐप, लेबर रूम में सेवाओं की
गुिवत्ता की श्चनगरानी करता है और UP में सावगिश्चनक स्वास्थ्य सुश्चवर्ाओं में सफलतापूवगक तैनात श्चकया गया है ।

43. भारत सरकार के लकस लवत्तीय संस्थान ने हाि ही में (लसतंबर'23 में) 100 करोड रुपये का "ओलिशा
स्ट्ाटा अप ग्रोथ फंि (OSGF)" िॉन्च करने के लिए स्ट्ाटा अप ओलिशा के साथ सहयोग लकया है?
1)नेशनल बैंक ऑफ फाइनेंश्चसंग इं फ्रास्ट्र क्चर एं ि िे वलपमेंट
2)इं श्चिया इं फ्रास्ट्र क्चर फाइनेंस कंपनी श्चलश्चमटे ि
3)स्माल इं िस्ट्र ीि िे वलपमेंट बैंक ऑफ़ इं श्चिया
4)एक्सपोटग -इम्पोटग बैंक ऑफ़ इं श्चिया
5)इं िस्स्ट्र यल फाइनेंस कॉपोरे शन ऑफ़ इं श्चिया
उत्तर-3)स्माि इं िस्ट्र ीि िे विपमेंट बैंक ऑफ़ इं लिया
स्पष्टीकरण:
ओश्चिशा सरकार के सूक्ष्म, लघु और मध्यम उद्यम (MSME) श्चवभाग की एक पहल स्ट्ाटग अप ओश्चिशा ने स्माि इं िस्ट्र ीि
िे विपमेंट बैंक ऑफ़ इं लिया (SIDBI) के सहयोग से 100 करोड़ रुपये का "ओश्चिशा स्ट्ाटग अप ग्रोि फंि (OSGF)"
लॉन्च श्चकया।
i.SIDBI नए फंि के श्चलए फंि मैनेिर के रूप में काम करे गा और श्चनवेश रिनीश्चत तैयार करने और फंि के पोटग फोश्चलयो
के प्रबंर्न के श्चलए श्चिम्मेदार होगा।
ii.ओश्चिशा स्ट्ाटग अप ग्रोि फंि एक केंद्रीकृत टर ै श्चकंग और कायागन्वयन तंत्र के माध्यम से राज्य स्ट्ाटग अप में श्चनवेश करने
के श्चलए फंि-ऑफ-फंि वाहन संचाश्चलत करता है।

44. लसतंबर 2023 में भारत सरकार (GoI) द्वारा शुरू लकए गए राष्टरीय पुरिारों के नए सेट के संबंि में
लनम्नलिस्कित में से कौन सा लबंदु "सही" है /हैं ?
A)भारत सरकार (GoI) ने लवज्ञान, प्रौद्योलगकी और नवाचार के क्षेत्र में राष्टरीय पुरिारों का एक नया सेट,
राष्टरीय लवज्ञान पुरिार (RVP) पेश लकया है।
B)RVP में 3 लवज्ञान रत्न पुरिार, साथ ही 25 लवज्ञान श्री, 25 लवज्ञान युवा-शांलत स्वरूप भटनागर और 3
लवज्ञान टीम पुरिार शालमि होंगे।
C) लवज्ञान रत्न (VR) पुरिार लवज्ञान और प्रौद्योलगकी के लकसी भी क्षेत्र में की गई िीवन भर की उपिस्कियों
और योगदान को मान्यता दे ने के लिए लनिााररत है।
1)सभी A, B & C
2)केवल A & B
3)केवल B & C
4)केवल A & C

Report Errors in the PDF - ebooks@affairscloud.com Copyright 2014-2023 @ AffairsCloud.com 29


Report Errors in the PDF - ebooks@affairscloud.com Copyright 2014-2023 @ AffairsCloud.com 30
5)केवल A
उत्तर- 1)सभी A, B & C
स्पष्टीकरण:
भारत सरकार (GoI) ने श्चवज्ञान, प्रौद्योश्चगकी और नवाचार के क्षेत्र में राष्टरीय पुरिारों का एक नया सेट, राष्टरीय
लवज्ञान पुरिार (RVP) पेश श्चकया है।
i.RVP भारत में श्चवज्ञान, प्रौद्योश्चगकी और नवाचार के क्षेत्र में सवोच्च मान्यता में से एक बनने के श्चलए तैयार है, और यह
श्चवश्चभन्न श्चवज्ञान श्चवभागों द्वारा श्चदए गए लगभग 300 पुरस्कारों को 56 केंद्र प्रशाश्चसत पुरस्कारों से प्रश्चतथिाश्चपत RVP 13
िोमेन में श्चदया िाएगा।
ii.RVP में 3 श्चवज्ञान रत्न पुरस्कार, साि ही 25 श्चवज्ञान श्री, 25 श्चवज्ञान युवा-शांश्चत स्वरूप भटनागर और 3 श्चवज्ञान टीम
पुरस्कार शाश्चमल होंगे।
• लवज्ञान रत्न (VR) पुरिार श्चवज्ञान और प्रौद्योश्चगकी के श्चकसी भी क्षेत्र में की गई िीवन भर की
उपिस्कियों & योगदान को मान्यता दे ने के श्चलए श्चनर्ागररत है।
• श्चवज्ञान श्री (VS) पुरस्कार श्चवज्ञान और प्रौद्योश्चगकी के श्चकसी भी क्षेत्र में श्चवश्चशष्ट् योगदान को मान्यता दे ने
के श्चलए श्चनर्ागररत है ।
• श्चवज्ञान युवा-शांश्चत स्वरूप भटनागर (VY-SSB) पुरस्कार का उद्दे श्य 45 वषग से कम आयु के युवा
वैज्ञाश्चनकों को पहचानना और प्रोत्साश्चहत करना है श्चिन्होंने श्चवज्ञान और प्रौद्योश्चगकी के श्चकसी भी क्षेत्र में
उत्कृष्ट् योगदान श्चदया है।
• शांश्चत स्वरूप भटनागर (SSB) पुरस्कार 1958 से सात िोमेन में वैज्ञाश्चनक और औद्योश्चगक अनुसंर्ान
पररषद (CSIR) द्वारा श्चदए गए हैं।
• श्चवज्ञान टीम (VT) पुरस्कार तीन या अश्चर्क वैज्ञाश्चनकों/शोर्कतागओ/ं नवप्रवतगकों की एक टीम को प्रदान
श्चकया िाना है, श्चिन्होंने एक टीम में काम करते हुए श्चवज्ञान और प्रौद्योश्चगकी के श्चकसी भी क्षेत्र में असार्ारि
योगदान श्चदया है।

45. प्रधाि मंत्री निसाि सम्माि निनध (PM-KISAN) योििा िे निए आनटा नफनशयि इं टेनििेंस (AI) चैटबॉट
िा िाम बताएं निसे हाि ही में (नसतंबर'23 में) िृनष और निसाि िल्याण मंत्रािय िे राज्य मंत्री (MoS)
िैिाश चौधरी द्वारा िॉन्च निया गया था।
1)सकसान ई-सलसा
2)सकसान ए-सना
3)सकसान ई-समत्र
4)सकसान ई-ज़रा
5)सकसान ई-वीरा
उत्तर-3)निसाि ई-नमत्र
स्पष्टीिरण:
कृसष और सकसान कल्ाि मोंत्रालय (MoA&FW) के राज्य मोंत्री (MoS) कैलाश चौर्री ने नई सदल्ली, सदल्ली में
प्रर्ानमोंत्री सकसान सम्मान सनसर् (PM-KISAN) योजना के सलए एक AI (कृसत्रम बुक्कद्धमत्ता) चैटबॉट "निसाि ई-नमत्र"
लॉन्च सकया।
i.चैटबॉट PM-KISAN योजना की दक्षता और पहुों च को बढाएगा, सजससे सकसानोों को उनकी पूछताछ के सलए त्वररत,
स्पष्ट् और आसानी से सुलभ प्रसतसक्रया समलेगी।

Report Errors in the PDF - ebooks@affairscloud.com Copyright 2014-2023 @ AffairsCloud.com 31


ii.MoA&FW ने AI चैटबॉट् स को सरकारी योजनाओों में सफलतापूवधक एकीकृत सकया। यह केंद्र सरकार की प्रमुख
योजना के साि एकीकृत पहला AI चैटबॉट है।
iii.PM-KISAN सशकायत प्रबोंर्न प्रिाली में AI चैटबॉट की शुरूआत का उद्दे श्य सकसानोों को उपयोगकताध के अनुकूल
और सुलभ मोंच के साि सशक्त बनाना है।

46. नसतंबर 2023 में 'भारतीय प्रिाश स्तंभ उत्सि' या 'इं नियि िाइटहाउस फेखस्ट्िि' िा पहिा संस्करण
निस राज्य में आयोनित निया गया था?
1)कनाधटक
2)गोवा
3)ओसडशा
4)केरल
5)पसिम बोंगाल
उत्तर- 2)गोिा
स्पष्टीिरण:
पयधटक थिलोों के रूप में लाइटहाउस को बढावा दे ने के सलए 3 सदवसीय कायधक्रम 'भारतीय प्रकाश स्तोंभ उत्सव' या
'इं नियि िाइटहाउस फेखस्ट्िि' का पहिा संस्करण 23 से 25 ससतोंबर 2023 तक गोिा में आयोसजत सकया गया
िा।
i.इस महोत्सव का उद् घाटन 23 ससतोंबर 2023 को पिजी, गोवा के फोटध अगुआडा में बोंदरगाह, जहाजरानी और
जलमागध मोंत्रालय (MoPSW) के केंद्रीय मोंत्री सबाधनोंद सोनोवाल िारा सकया गया िा।
ii.ऐसतहाससक थिलोों यानी प्रकाशस्तोंभोों को पयधटन थिलोों के रूप में बढावा दे ने के सलए यह त्यौहार भारत के सभी
प्रकाशस्तोंभोों में मनाया गया। यह तटीय क्षेत्रोों में प्रकाशस्तोंभोों की अमूल् भूसमका पर भी प्रकाश डालता है।
iii. केंद्र सरकार ने इसके सलए 75 लाइटहाउस का चयन सकया है । यह पररवतधन 'लाइटहाउस हेररटे ज टू ररज्म'
असभयान के तहत सकया जाएगा।

47. नसतंबर 2023 ति स्वच्छ भारत नमशि (ग्रामीण) चरण II िे संबंध में निम्ननिखित में से िौि सा/से नबंदु
"सही" है/हैं?
A) भारत िे स्वच्छ भारत नमशि (ग्रामीण) चरण II िे तहत प्रमुि मीि िा पत्थर हानसि निया है , नमशि िे
चरण II िे तहत 75% गांिों िे िुिे में शौच मुक्त (ODF) प्लस िा दिाा हानसि निया है।
B) िुि 4.43 िाि गांिों िे िुद िो ODF प्लस घोनषत निया है , िो 2024-25 ति स्वच्छ भारत नमशि -
ग्रामीण (SBM-G) चरण II िे िक्ष्यों िो प्राप्त िरिे िी नदशा में एि महत्वपूणा िदम है।
C) राज्यों/UT में 100% ODF प्लस मॉिि गांि - अंिमाि और नििोबार द्वीप समूह, दादरा िगर हिेिी और
दमि दीि, िम्मू और िश्मीर और नसखिम है।
1)सभी A, B & C
2)केवल A & B
3)केवल B & C
4)केवल A & C
5)केवल A
उत्तर- 1)सभी A, B & C

Report Errors in the PDF - ebooks@affairscloud.com Copyright 2014-2023 @ AffairsCloud.com 32


स्पष्टीिरण:
भारत ने स्वच्छ भारत समशन (ग्रामीि) चरि II के तहत प्रमुख मील का पत्थर हाससल सकया है , समशन के चरण II के
तहत 75% गांिों ने खुले में शौच मुक्त (ODF) प्लस िा दिाा हाससल सकया है ।
• एक ODF प्लस मॉडल गाोंव अपने नाम के साि ODF का दजाध जोडता है और इसमें ठोस अपसशष्ट् और तरल
अपसशष्ट् प्रबोंर्न दोनोों की व्यवथिा होती है।
• कुल 4.43 िाि गांिों ने खुद को ODF प्लस घोसषत सकया है , जो 2024-25 तक स्वच्छ भारत समशन - ग्रामीि
(SBM-G) चरण II के लक्ष्ोों को प्राप्त करने की सदशा में एक महत्वपूिध कदम है।
• शीषा प्रदशाि िरिे िािे राज्य/UT निन्ोंिे 100% ODF प्लस गांि हानसि निए: अोंडमान और सनकोबार
िीप समूह, दादरा और नगर हवेली और दमन और दीव, गोवा, गुजरात, सहमाचल प्रदे श, जम्मू और कश्मीर,
कनाधटक, केरल, लद्दाख, पुडुचेरी, ससक्किम, तसमलनाडु , तेलोंगाना और सत्रपुरा हैं।
• राज्योों/UT में 100% ODF प्लस मॉिि गांि - अंिमाि & नििोबार द्वीप समूह, दादरा नगर हवेली और
दमन दीव, जम्मू और कश्मीर और ससक्किम हैं।
• 2025 तक सभी गाोंवोों को ODF प्लस बनाने का लक्ष् रखा गया है।

48. नसतंबर 2023 में भारत िे प्रधाि मंत्री (PM) िरें द्र मोदी द्वारा निए गए हानिया उद् घाटि िे संबंध में
निम्ननिखित में से िौि सा नबंदु "गित" है ?
1)प्रर्ान मोंत्री ने पूरे उत्तर प्रदे श में लगभग 1115 करोड रुपये की लागत से सनसमधत 16 अटल आवासीय सवद्यालयोों
(आवासीय सवद्यालयोों) का उद् घाटन सकया।
2)PM ने कनेक्करसवटी और यात्री अनुभव को बढाने के सलए 24 ससतोंबर, 2023 को वीसडयो कॉन्क्फ्रेंससोंग के माध्यम से 11
राज्योों को जोडने वाली नौ वोंदे भारत एक्सप्रेस टर े नोों को हरी झोंडी सदखाई।
3)केरल को अलाप्पुझा के रास्ते कासरगोड-सतरुवनोंतपुरम मागध पर पहली ऑरें ज वोंदे भारत एक्सप्रेस समली।
4)PM ने सबहार के पटना में एक अोंतरराष्ट्रीय सक्रकेट स्ट्े सडयम की आर्ारसशला रखी।
5)PM ने नई सदल्ली के सवज्ञान भवन में भारत के पहले अोंतराध ष्ट्रीय वकील सम्मे लन 2023 का भी उद् घाटन सकया।
उत्तर-4)PM िे नबहार िे पटिा में एि अंतरराष्टरीय नक्रिेट स्ट्े नियम िी आधारनशिा रिी।
स्पष्टीिरण:
प्रर्ान मोंत्री नरें द्र मोदी ने 23 और 24 ससतोंबर 2023 के दौरान उल्लेखनीय उद् घाटन सकए िे।
• प्रर्ानमोंत्री नरें द्र मोदी ने उत्तर प्रदे श के वारािसी में रुद्राक्ष अोंतराध ष्ट्रीय सहयोग और कन्वेंशन सेंटर में काशी
सोंसद साोंस्कृसतक महोत्सव 2023 का उद् घाटन सकया।
• प्रर्ान मोंत्री ने पूरे उत्तर प्रदे श में लगभग 1115 िरोड रुपये की लागत से सनसमधत 16 अटि आिासीय निद्याियों
(आवासीय सवद्यालयोों) का उद् घाटन सकया।
• श्री मोदी ने क्षेत्र में खेल और प्रसतयोसगताओों को बढावा दे ने के सलए काशी सोंसद खेल प्रसतयोसगता के पोंजीकरि
के सलए पोटध ल लॉन्च सकया।
• प्रर्ान मोंत्री ने कनेक्करसवटी और यात्री अनुभव को बढाने के सलए 24 ससतोंबर, 2023 को वीसडयो कॉन्क्फ्रेंससोंग के
माध्यम से 11 राज्योों को जोडने वाली िौ िंदे भारत एक्सप्रेस टर े िों को हरी झोंडी सदखाई।
❖ राजथिान, तसमलनाडु , तेलोंगाना, आों ध्र प्रदे श, कनाधटक, सबहार, पसिम बोंगाल, केरल, ओसडशा, झारखोंड
और गुजरात
• केरल को अलाप्पुझा के रास्ते कासरगोड-सतरुवनोंतपुरम मागध पर पहली ऑरें ज वोंदे भारत एक्सप्रेस समली। वोंदे
भारत सेवाओों की कुल सोंख्या 68 तक पहुोंच जाएगी, सजसमें अपने पूवधवसतधयोों की तुलना में 25 सुर्ारोों के साि
छोटे 8-कोच सोंस्करि शासमल होोंगे।

Report Errors in the PDF - ebooks@affairscloud.com Copyright 2014-2023 @ AffairsCloud.com 33


• प्रर्ानमोंत्री नरें द्र मोदी ने उत्तर प्रदे श के वारािसी में एक अोंतरराष्ट्रीय सक्रकेट स्ट्े सडयम की आर्ारसशला रखी।
❖ वारािसी में अोंतरराष्ट्रीय सक्रकेट स्ट्े सडयम की क्षमता 30,000 दशधकोों की होगी.
❖ इसे लगभग 450 करोड रुपये की लागत से गाोंजरी, राजातालाब, वारािसी में सवकससत सकया जा रहा है
(उत्तर प्रदे श सरकार ने स्ट्े सडयम के सलए भूसम असर्ग्रहि के सलए 121 करोड रुपये खचध सकए, जबसक
भारतीय सक्रकेट कोंटर ोल बोडध [BCCI] इसके सनमाधि पर 330 करोड रुपये खचध होोंगे।
• प्रर्ान मोंत्री नरें द्र मोदी ने नई सदल्ली के सवज्ञान भवन में अोंतराध ष्ट्रीय वकील सम्मेलन 2023 का उद् घाटन सकया।
दे श में पहली बार ऐसी पहल की गई है .

49. निस राज्य सरिार िे हाि ही में (नसतंबर'23 में) अपिे 7िें टाइगर ररििा, निसिा िाम 'िीरांगिा
दु गाािती टाइगर ररििा' है, िा अिािरण निया है ?
1)उत्तर प्रदे श
2)पसिम बोंगाल
3)महाराष्ट्र
4)ओसडशा
5)मध्य प्रदे श
उत्तर-5)मध्य प्रदे श
स्पष्टीिरण:
मध्य प्रदे श(MP) राज्य सरकार ने अपने 7िें टाइगर ररजवध 'िीरांगिा दु गाािती टाइगर ररििा' का अनावरि सकया है ,
जो MP के सागर, दमोह और नरससोंहपुर सजलोों में 2,339 वगध सकलोमीटर के क्षेत्र में फैला हुआ है। इसका कुछ भाग
नमधदा और यमुना नदी घासटयोों के अोंतगधत आता है।
• यह अनावरि राष्ट्रीय बाघ सोंरक्षि प्रासर्करि (NTCA) से मोंजूरी के बाद हुआ है।
• इस टाइगर ररजवध की घोषिा वन्यजीव (सोंरक्षि) असर्सनयम, 1972 (1972 का 53)की र्ारा 38V की उप-र्ारा
(iv) के खोंड (ii) के प्रावर्ानोों के अनुसार है ।
• इसमें नौरादे ही वन्यजीव अभ्यारण्य और वीराोंगना दु गाध वती वन्यजीव अभ्यारण्य के क्षेत्र शासमल होोंगे।
िोट- MP में अन्य छह टाइगर ररजवध कान्ा, बाों र्वगढ, सतपुडा, पेंच, पन्ना और सोंजय-डु बरी हैं।

50. हाि ही में (नसतंबर'23 में) निस िंपिी िे इं नियि ऑयि िॉपोरे शि निनमटे ि (IOCL) िो अपिी तरह िी
पहिी दो हाइिर ोिि फ्यूि सेि इिेखरर ि व्हीिि (FCEV) बसें नििीिर िी ं?
1)मसहोंद्रा & मसहोंद्रा सलसमटे ड
2)हुोंडई मोटर कोंपनी
3)टाटा मोटसध सलसमटे ड
4)मारुसत सुजुकी इों सडया
5)अशोक लीलैंड सलसमटे ड
उत्तर-3)टाटा मोटसा निनमटे ि
स्पष्टीिरण:
25 ससतोंबर, 2023 को, टाटा मोटसा निनमटे ि ने इों सडयन ऑयल कॉपोरे शन सलसमटे ड (IOCL) को अपनी तरह की
पहली दो हाइडर ोजन ईोंर्न सेल इलेक्कररक वाहन (FCEV) बसें सौोंपीों।
i.इन्ें केंद्रीय मोंत्री हरदीप ससोंह पुरी, पेटरोसलयम एवों प्राकृसतक गैस मोंत्रालय (MoPNG); राज्य मोंत्री (MoS) रामेश्वर तेली,
MoPNG; IOCL के अध्यक्ष श्रीकाोंत मार्व वैद्य ससहत अन्य लोगोों नेहरी झोंडी सदखाई।

Report Errors in the PDF - ebooks@affairscloud.com Copyright 2014-2023 @ AffairsCloud.com 34


ii.इों सडयन ऑयल कॉपोरे शन सलसमटे ड एक महारत्न केंद्रीय सावधजसनक क्षेत्र उद्यम है।
• जून 2021 में, टाटा मोटसध ने भारत में हाइडर ोजन-आर्ाररत प्रोटोन एक्सचेंज मेम्ब्रेन (PEM) ईोंर्न-सेल
प्रौद्योसगकी के परीक्षि के सलए 15 FCEV बसोों की आपूसतध के सलए IOCL से एक टें डर जीता।
• 35 यासत्रयोों के आसानी से चढने और बैठने के सलए सडज़ाइन की गई इन 12-मीटर बसोों को पुिे, महाराष्ट्र में
टाटा मोटसध के R&D (अनुसोंर्ान एवों सवकास) केंद्र में सवकससत सकया गया िा।

51. नसतंबर 2023 में रसायि और उिारि मंत्रािय (MoC&F) द्वारा हाि ही में निए गए िॉन्च िे संबंध में
निम्ननिखित में से िौि सा/से नबंदु "सही" है /हैं?
A) िेंद्रीय मंत्री िॉ. मिसुि मंिानिया, रसायि और उिारि मंत्रािय (MoC&F) िे भारत में फामाा-मेिटे ि क्षेत्र
में अिुसंधाि और नििास और ििाचार पर राष्टरीय िीनत शुरू िी।
B) फामाास्यूनटिल्स निभाग िे 5,000 िरोड रुपये िे बिट िे साथ फामाा -मेिटे ि में अिुसंधाि और ििाचार
िो बढ़ािा दे िा (PRIP ) क्षेत्र िे निए योििा शुरू िी है।
C) PRIP क्षेत्र िे निए एि योििा शुरू िी गई थी, निसिा िक्ष्य अिुसंधाि बुनियादी ढांचे िो बढ़ािर
भारतीय फामाास्युनटिि क्षेत्र िो ििाचार-संचानित नििास िी ओर स्थािांतररत िरिा था।
1)केवल A
2)केवल A & B
3)केवल B & C
4)केवल A & C
5)सभी A, B & C
उत्तर -5)सभी A, B & C
स्पष्टीिरण:
26 ससतोंबर, 2023 को, रसायन और उवधरक मोंत्रालय (MoC&F) के केंद्रीय मोंत्री डॉ. मनसुख मोंडासवया ने नई सदल्ली में
एक कायधक्रम के दौरान भारत में फामाध -मेडटे क सेरर में अनुसोंर्ान और सवकास और नवाचार पर राष्ट्रीय नीसत और
फामाध -मेडटे क में अनुसोंर्ान और नवाचार को बढावा दे ना (PRIP) क्षेत्र के सलए योजना शुरू की।
• पारों पररक दवाओों, फाइटो-फामाधस्यूसटकल्स और सचसकत्सा उपकरिोों ससहत फामाध स्यूसटकल्स में R&D को
बढावा दे ने के सलए भारत में फामाध -मेडटे क क्षेत्र में अनु सोंर्ान एवों सवकास और नवाचार पर राष्ट्रीय नीसत हैं ।
• फामाधस्यूसटकल्स सवभाग ने 5,000 करोड रुपये के बजट के साि PRIP योजना शुरू की है , सजसका उद्दे श्य
अनुसोंर्ान बुसनयादी ढाोंचे को बढाकर भारतीय फामाधस्युसटकल क्षेत्र को नवाचार-सोंचासलत सवकास की ओर
थिानाोंतररत करना है। इस योजना में दो घटक शासमल हैं :
• घटक A: अनुसोंर्ान बुसनयादी ढाोंचे को मजबूत करने के सलए 700 करोड रुपये के बजट के साि NIPER
(राष्ट्रीय फामाध स्युसटकल सशक्षा और अनुसोंर्ान सोंथिान) में 7 उत्कृष्ट्ता केंद्र (CoE) की थिापना करना है।
• घटक B: 4,250 करोड रुपये के बजट के साि नई रासायसनक सोंथिाओों, जसटल जेनररक, सचसकत्सा उपकरिोों,
स्ट्े म सेल िेरेपी, अनाि दवाओों और एों टी-माइक्रोसबयल प्रसतरोर् जैसे 6 प्रािसमकता वाले क्षेत्रोों में अनुसोंर्ान को
बढावा दे ना है।

Report Errors in the PDF - ebooks@affairscloud.com Copyright 2014-2023 @ AffairsCloud.com 35


52. नसतंबर 2023 में, िििातीय मामिों िे मंत्रािय िे तहत िििातीय छात्रों िे निए राष्टरीय
नशक्षा_____________ (NESTS) िे हाि ही में (नसतंबर'23 में) 54 एििव्य मॉिि आिासीय निद्याियों (EMRS)
में 'अमेज़़ॅि फ्यूचर इं िीनियर प्रोग्राम' िे चरण II िा शुभारं भ निया है।
1)सपोटध
2)ससवधस
3)स्कूल
4)सोसायटी
5)स्कीम
उत्तर- 4)सोसायटी
स्पष्टीिरण:
जनजातीय मामलोों के मोंत्रालय के तहत जनजातीय छात्रोों के सलए राष्ट्रीय सशक्षा सोसायटी (NESTS) ने आों ध्र प्रदे श,
गुजरात, मध्य प्रदे श, ओसडशा, राजथिान और तेलोंगाना में फैले 54 एकलव्य मॉडल आवासीय सवद्यालयोों (EMRS) में
'अमेज़़ॅि फ्यूचर इं िीनियर प्रोग्राम' का दू सरा चरि शुरू सकया।
i.दू सरे चरि में कोसडों ग, लॉसजकल सीक्वेंससोंग, लसनिंग लूप्स और िॉक प्रोग्रासमोंग के अलावा एक उन्नत िॉक प्रोग्रासमोंग
और आसटध सफसशयल इों टेसलजेंस (AI) पाठ्यक्रम शासमल होगा।
ii.यह कायधक्रम क्रमशः कोंप्यूटर सवज्ञान के बुसनयादी ससद्धाोंतोों से लेकर AI के बुसनयादी ससद्धाोंतोों की पेशकश करके
कक्षा छह से कक्षा 9 तक के छात्रोों को शासमल करता है।

53. नसतंबर 2023 में, भारतीय िाद्य सुरक्षा और मािि प्रानधिरण (FSSAI) िे ___________ िो 'िाद्य पशु' िे
रूप में मान्यता दी।
1)बद्री
2)मालवी
3)गाओलाओ
4)समिुन
5)बरगुर
उत्तर-4)नमथुि
स्पष्टीिरण:
भारतीय खाद्य सुरक्षा और मानक प्रासर्करि (FSSAI) ने नमथुि (वैज्ञासनक नाम-बोस फ्रोंटसलस), गोजातीय (गाय जैसा
जानवर) को 'िाद्य पशु' के रूप में मान्यता दी है।
• समिुन की सवशेषताएों इसकी पारों पररक अर्ध-पालतूकरि और न्यूनतम मानवीय हस्तक्षेप के साि एक मुक्त-श्रेिी
वन पाररक्कथिसतकी तोंत्र में पनपने की क्षमता है।
• समिुन माोंस, जो सवशेष रूप से अपनी कम वसा सामग्री के सलए जाना जाता है , में स्वास्थ्य के प्रसत जागरूक
उपभोक्ताओों के सलए एक प्रीसमयम माोंस उत्पाद बनने की क्षमता है।
• समिुन, सजसे अक्सर "पहासडयोों का मवेशी" कहा जाता है , अरुिाचल प्रदे श और नागालैंड का राज्य पशु है , जो
इसके गहन साोंस्कृसतक महत्व को उजागर करता है।
i.भारतीय कृसष अनुसोंर्ान पररषद (ICAR) ने समिुन सकसानोों को प्रसतस्पर्ी कीमतोों पर व्यवसाय सोंचासलत करने में
सहायता के सलए "खरीदार" और "सवक्रेता" के रूप में पोंजीकृत करने के सलए M-ANITRA ऐप लॉन्च सकया।
ii.िागािैंि में ICAR - राष्ट्रीय समिुन अनुसोंर्ान केंद्र (NRCM) ने 1 ससतोंबर 2023 को पहला राष्ट्रीय समिुन सदवस मनाया।

Report Errors in the PDF - ebooks@affairscloud.com Copyright 2014-2023 @ AffairsCloud.com 36


54.भारतीय िाद्य सुरक्षा और मािि प्रानधिरण (FSSAI) िे ऑििाइि िाद्य सुरक्षा अिुपािि प्रणािी
(FoSCoS) पोटा ि में _____________ िा एि िया प्रािधाि पेश निया।
1)सवशेष श्रेिी
2)जोक्कखम मूल्ाोंकन
3)गुिवत्ता सनयोंत्रि
4)आयात मोंजूरी
5)पोषि सोंबोंर्ी लेबसलोंग
उत्तर - 1)निशेष श्रेणी
स्पष्टीिरण:
भारतीय खाद्य सुरक्षा और मानक प्रासर्करि (FSSAI) ने खाद्य व्यवसाय क्षेत्र में मसहलाओों और टर ाोंसजेंडर उद्यसमयोों के
सलए लैंसगक समानता और समान अवसरोों को बढावा दे ने के सलए ऑनलाइन िाद्य सुरक्षा अिुपािि प्रणािी (FoSCoS)
पोटध ल में 'निशेष श्रेणी' का एक नया प्रावर्ान पेश सकया।
i.इस प्रावर्ान के अनुसार, लाइसेंससोंग और पोंजीकरि असर्कारी 'सवशेष श्रेिी' आवेदनोों को इस तरह से प्रािसमकता दें गे
सक सनयसमत आवेदनोों के साि एक-से-एक अनुपात सुसनसित हो, जब तक सक सकसी भी श्रेिी में कोई लोंसबत आवेदन न
हो।

55. निम्ननिखित में से िौि सी योििा उि संपीनडत बायो-गैस (CBG) संयंत्रों और बायोगैस संयंत्र िो नित्तीय
सहायता प्रदाि िरे गी िो गैल्विाइनिंग ऑगेनिि बायो-एग्रो ररसोसेि धि (GOBARdhan) िे निए एिीिृत
पंिीिरण पोटा ि पर पंिीिृत हैं ?
1)अटल पेंशन योजना (APY)
2)प्रर्ानमोंत्री सुरक्षा बीमा योजना (PMSBY)
3)प्रर्ानमोंत्री जीवन ज्योसत बीमा योजना (PMJJBY)
4)स्ट्ैं ड अप इों सडया योजना
5)माकेट डे वलपमेंट अससस्ट्ें स (MDA) योजना
उत्तर-5)मािेट िे ििपमेंट अनसस्ट्ें स (MDA) योििा
स्पष्टीिरण:
सोंपीसडत बायो-गैस (CBG) सोंयोंत्र और बायोगैस सोंयोंत्र गैल्वनाइसजोंग ऑगेसनक बायो-एग्रो ररसोसेज र्न (GOBARdhan)
के सलए एकीकृत पोंजीकरि पोटध ल पर पोंजीकृत रसायन और उवधरक मोंत्रालय (MoC&F) के तहत उवधरक सवभाग की
माकेट डे वलपमेंट अससस्ट्ें स (MDA) योजना के तहत सवत्तीय सहायता के सलए पात्र हैं।
• GOBARdhan सोंयोंत्रोों से प्राप्त जैसवक उवधरकोों के उत्पादन और उपयोग को प्रोत्सासहत करने के सलए MDA
योजना तीन साल की अवसर् (FY 2023-24 से FY 2025-26) में 1451.82 करोड रुपये के बजट के साि शुरू
की गई है ।

56. उस मंत्रािय िा िाम बताइए निसिे हाि ही में (नसतंबर'23 में) अंतरााष्टरीय नक्रिेट पररषद (ICC), भारतीय
नक्रिेट िंटर ोि बोिा (BCCI) और संयुक्त राष्टर बाि िोष (UNICEF) िे साथ िैंनगि समािता िो बढ़ािा दे िे िे
निए CRIIIO 4 GOOD िॉन्च निया है।
1)सशक्षा मोंत्रालय
2)मसहला एवों बाल सवकास मोंत्रालय
3)युवा मामले और खेल मोंत्रालय

Report Errors in the PDF - ebooks@affairscloud.com Copyright 2014-2023 @ AffairsCloud.com 37


4)सामासजक न्याय और असर्काररता मोंत्रालय
5)कौशल सवकास और उद्यसमता मोंत्रालय
उत्तर- 1)नशक्षा मंत्रािय
स्पष्टीिरण:
नशक्षा मंत्रािय के केंद्रीय मोंत्री र्मेंद्र प्रर्ान ने गुजरात के अहमदाबाद में नरें द्र मोदी स्ट्े सडयम में लडसकयोों और लडकोों
के बीच लैंसगक समानता को बढावा दे ने के सलए एक नया ऑनलाइन, जीवन कौशल सीखने का मॉड्यूल "CRIIIO 4
GOOD" लॉन्च सकया।
i.यह कायधक्रम अोंतराध ष्ट्रीय सक्रकेट पररषद (ICC), भारतीय सक्रकेट कोंटर ोल बोडध (BCCI) और सोंयुक्त राष्ट्र बाल कोष
(UNICEF) के सहयोग से शुरू सकया गया िा।
ii.CRIIIO 4 GOOD एक असभनव युवा जुडाव है जो लैंसगक समानता और सभी के सलए समान अवसरोों को बढावा दे ता
है। यह बच्चोों को बदलती दु सनया में सवकससत होने के सलए आवश्यक आवश्यक जीवन कौशल से लैस करे गा।

INTERNATIONAL AFFAIRS
1. निस संगठि/माििानधिार संनध नििायों िे हाि ही में (अगस्त’23 में) िए नदशानिदे श िारी निए हैं
निन्ोंिे बच्चों िे स्वच्छ, स्वस्थ और नटिाऊ िातािरण में रहिे िे अनधिारों िी पुनष्ट िी है ?
1)सोंयुक्त राष्ट्र पयाधवरि कायधक्रम
2)नस्लीय भेदभाव उन्मूलन ससमसत
3)सोंयुक्त राष्ट्र सवकास कायध क्रम
4)बाल असर्कारोों ससमसत
5)अत्याचार के सवरुद्ध ससमसत
उत्तर-4)बाि अनधिारों सनमनत
स्पष्टीिरण:
28 अगस्त 2023 को, बाि अनधिारों पर संयुक्त राष्टर (UN) सनमनत ने नए सदशासनदे श जारी सकए, सजन्ें औपचाररक
रूप से "सामान्य सटप्पिी सोंख्या 26 (2023)" के रूप में जाना जाता है , जो सदस्य राज्योों की सरकारोों को बढते जलवायु
सोंकट के क्कखलाफ बच्चोों के असर्कारोों की रक्षा के सलए कारध वाई करने का आह्वान करता है।
• यह पहली बार है जब ससमसत ने बच्चोों के स्वच्छ, स्वथि और सटकाऊ वातावरि में रहने के असर्कारोों की पुसष्ट् की
है।
i.अपने 93वें सत्र के दौरान, बाल असर्कारोों पर सोंयुक्त राष्ट्र ससमसत ने आसर्काररक तौर पर सामान्य सटप्पिी सों ख्या 26
(2023)को अपनाया है ।
ii.सदशासनदे श बाल असर्कारोों पर UN कन्वेंशन के तहत राज्य के दासयत्वोों की व्यापक व्याख्या प्रदान करते हैं , सजसे
196 दे शोों िारा अनुमोसदत सकया गया है।
iii.1989 में अपनाई गई यह सोंसर् जीवन, स्वास्थ्य, स्वच्छ पेयजल और अक्कस्तत्व और सवकास ससहत बच्चोों के असर्कारोों
की रूपरे खा तैयार करती है ।

2. भारत िे साथ निस दे श िे हाि ही में (अगस्त’23 में) रणिीनति स्वच्छ ऊिाा साझेदारी िे तहत
ििीिरणीय ऊिाा प्रौद्योनगिी एक्शि प्लेटफॉमा (RETAP) िॉन्च निया है ?
1)यूनाइटे ड सकोंगडम
2)फ़्ाोंस

Report Errors in the PDF - ebooks@affairscloud.com Copyright 2014-2023 @ AffairsCloud.com 38


3)सोंयुक्त राज्य
4)ब्राज़ील
5)सोंयुक्त अरब अमीरात
उत्तर-3)संयुक्त राज्य
स्पष्टीिरण:
संयुक्त राज्य (US) ऊजाध सवभाग (DOE) और भारत के नवीन और नवीकरिीय ऊजाध मोंत्रालय (MNRE) ने एक बैठक
में रणिीनति स्वच्छ ऊिाा साझेदारी के तहत US-भारत नवीकरिीय ऊजाध प्रौद्योसगकी कारध वाई प्लेटफामध (RETAP)
को एक साि लॉन्च सकया।
i.नई और उभरती नवीकरिीय प्रौद्योसगसकयोों को आगे बढाने के सलए जून 2023 में RETAP की घोषिा की गई िी।
ii.बैठक का नेतृत्व DOE के उप ससचव डे सवड तुकध और MNRE ससचव भूसपोंदर ससोंह भल्ला ने सकया।

3. नसतंबर 2023 में, बायोिायिनसाटी एं ि ििेिानटि इं टरगििामेंटि साइं स-पॉनिसी प्लेटफामा ऑि


बायोिायिनसाटी एं ि इिोनसस्ट्म सनिासेि (IPBES) िे असेसमेंट ररपोटा ऑि इििेनसि एनियि स्पीशीि एं ि
दे यर िण्ट्र ोि (निसे "इििेनसि एनियि स्पीशीि ररपोटा " िे रूप में िािा िाता है )” शीषाि से एि ररपोटा
प्रिानशत िी।
इििेनसि एनियि स्पीशीि िी िैनश्वि आनथाि िागत 2019 में सािािा _____________ अमेररिी िॉिर से
अनधि हो गई, 1970 िे बाद से हर दशि में िागत िम से िम चौगुिी हो गई है।
1)124 सबसलयन
2)423 सबसलयन
3)534 सबसलयन
4)312 सबसलयन
5)256 सबसलयन
उत्तर- 2)423 नबनियि
स्पष्टीिरण:
बायोडायवससधटी एों ड कर्न्ेवधसटव पर सोंयुक्त राष्ट्र (UN) की अग्रिी सोंथिा, इों टरगवनधमेंटल साइों स-पॉसलसी प्लेटफामध ऑन
बायोडायवससधटी एों ड इकोससस्ट्म ससवधसेज (IPBES) ने असेसमेंट ररपोटध ऑन इनवेससव एसलयन स्पीशीज एों ड दे यर
कण्ट्र ोल (सजसे "इनवेससव एसलयन स्पीशीज ररपोटध " के रूप में जाना जाता है )” शीषधक से एक ररपोटध प्रकासशत की।
• ररपोटध के अनुसार, इनवेससव एसलयन स्पीशीज की पहचान दु सनया भर में बायोडायवससधटी के नुकसान के मुख्य
प्रत्यक्ष चालकोों में से एक के रूप में की गई िी, और दु सनया भर में कम से कम 37,000 सवदे शी प्रजासतयााँ
थिासपत की गईों।
• ररपोटध से यह भी पता चलता है सक 3,500 से असर्क आक्रामक प्रजासतयााँ मानवीय गसतसवसर्योों के कारि
दु सनया भर के सवसभन्न क्षेत्रोों और बायोम में प्रवेश कर चुकी हैं , सजससे मानव स्वास्थ्य, खाद्य सुरक्षा और
बायोडायवससधटी प्रभासवत हो रही है।
• इििेनसि एनियि स्पीशीि िी िैनश्वि आनथाि िागत 2019 में सालाना 423 नबनियि अमेररिी िॉिर
से असर्क हो गई, 1970 के बाद से हर दशक में लागत कम से कम चौगुनी हो गई है।

Report Errors in the PDF - ebooks@affairscloud.com Copyright 2014-2023 @ AffairsCloud.com 39


4. नसतंबर 2023 में, सूया िेपाि िाठमांिू िनिंग सानहत्य महोत्सि (KLF) िा ________ संस्करण िाठमांिू,
िेपाि में आयोनित निया गया था।
1)पाोंचवाों
2)तीसरा
3)दू सरा
4)चौिा
5)छठा
उत्तर- 3)दू सरा
स्पष्टीिरण:
सूया िेपाि काठमाोंडू कसलोंग सासहत्य महोत्सव (KLF) का दू सरा सोंस्करि 1 से 3 ससतोंबर 2023 तक काठमाों डू, नेपाल
में आयोसजत सकया गया िा।
i.इस कायधक्रम का उद् घाटन 1 ससतोंबर 2023 को नेपाल के सवदे श मोंत्री NP सऊद ने सकया िा और यह 3 ससतोंबर
2023 को लसलतपुर, नेपाल में सफलतापूवधक सोंपन्न हुआ।
• महोत्सव का केंद्रीय सवषय शक्कक्त, भक्कक्त सससवलाइजेशन कनेक्शन : नेपाल एज सेंटर ऑफ ग्लोबल िॉट है।
• यह उत्सव सासहक्कत्यक और साोंस्कृसतक गसतसवसर्योों को बढावा दे ने के माध्यम से दसक्षि एसशयाई सोंस्कृसत के
आदान-प्रदान के सलए आयोसजत सकया जाता है।
• भारत, नेपाल, भूटान, बाोंग्लादे श और श्रीलोंका के कलाकारोों और लेखकोों ने सासहत्य, सोंगीत, नृत्य, कसवता और
अन्य माध्यमोों में अपना काम प्रदसशधत सकया।
• 'यशस्वी सासहत्य सम्मान' पुरस्कार प्रोफेसर अवर्ेश प्रर्ान, असभनेत्री और लेक्कखका सदव्या दत्ता, प्रोफेसर मार्व
प्रसाद पोखरे ल और असभनेत्री मनीषा कोइराला को सदया गया।

5. संयुक्त राष्टर (UN) िे "प्रोग्रेस ऑि द सस्ट्े िेबि िे ििपमेंट गोल्स: द िेंिर स्नैपशॉट 2023" ररपोटा िारी िी,
निसमें मुख्य रूप से ___________ (िषा) ति निंग समािता प्राप्त िरिे िी नदशा में मौिूदा रुझाि, अंतराि
और हानिया असफिताओं पर प्रिाश िािा गया।
1)2027
2)2025
3)2026
4)2030
5)2028
उत्तर- 4)2030
स्पष्टीिरण:
संयुक्त राष्टर (UN) की "प्रोग्रेस ऑन द सस्ट्े नेबल डे वलपमेंट गोल्स: द जेंडर स्नैपशॉट 2023" शीषधक वाली ररपोटध में
कहा गया है सक स्वास्थ्य, सशक्षा, रोजगार और सत्ता के हॉल में दु सनया भर में मसहलाओों के क्कखलाफ पूवाधग्रहोों के कारि
दु सनया सस्ट्े नेबल डे वलपमेंट गोल्स (SDG) 5: निंग समािता को प्राप्त करने की सदशा में अपने प्रयासोों में सवफल रही
है।
• यह UN मसहला (लैंसगक समानता और मसहलाओों के सशक्कक्तकरि के सलए UN इकाई) और UN आसिधक और
सामासजक मामलोों के सवभाग (DESA) िारा सोंयुक्त रूप से सनसमधत वासषधक श्रृोंखला का नवीनतम सोंस्करि है ।
• ररपोटध सभी 17 SDG में लैंसगक समानता की प्रगसत का व्यापक सवश्लेषि प्रदान करती है।

Report Errors in the PDF - ebooks@affairscloud.com Copyright 2014-2023 @ AffairsCloud.com 40


• ररपोटध 2030 तक लैंसगक समानता हाससल करने की सदशा में मौजूदा रुझानोों, अोंतरालोों और हाल की
असफलताओों पर भी प्रकाश डालती है ।

6. नसतंबर 2023 में िारी U.S. न्यूि एं ि िडा ररपोटा िी सिाश्रेष्ठ दे शों िी रैं निंग 2023 िे 8िें संस्करण िे
अिुसार, भारत िे __________ रैं ि हानसि िी, िबनि _________ (दे श) िगातार दू सरी बार दु निया िे सिाश्रेष्ठ
दे श िे रूप में शीषा पर रहा।
1)29वााँ; स्वीडन
2)30वााँ; क्कस्वट् ज़रलैंड
3)29वााँ; कनाडा
4)30वााँ; स्वीडन
5)40वााँ; क्कस्वट् ज़रलैंड
उत्तर- 2)30िााँ; खस्वट् ज़रिैंि
स्पष्टीिरण:
U.S. न्यूि एं ि िडा ररपोटा िी सवधश्रेष्ठ दे शोों की रैं सकोंग 2023 के 8िें संस्करण के अनुसार, भारत ने 40.8 के समग्र
स्कोर के साि 2023 में 30िी ं रैं क हाससल करने के सलए अपनी रैं सकोंग में एक थिान का सुर्ार सकया। 2022 में भारत
85 में से 31वें थिान पर िा।
• इस सूची में 87 दे शोों में से क्कस्वट् जरलैंड लगातार दू सरी बार और कुल समलाकर छठी बार शीषध पर है।
• क्कस्वट् जरलैंड ने सभी 10 सबरैं सकोंग में शीषध 25 में जगह बनाई।
• क्कस्वट् जरलैंड के बाद कनाडा दू सरे थिान पर और स्वीडन तीसरे थिान पर है।
• रैं सकोंग वैसश्वक सवपिन सोंचार कोंपनी WPP और उसके स्वासमत्व वाले BAV ब्राोंड एनासलसटक्स टू ल और व्हाटध न
स्कूल ऑफ़ द यूसनवससधटी ऑफ़ पेनससलवेसनया, सोंयुक्त राज्य अमेररका (US) के सहयोग से सवकससत की गई
है।
• रैं सकोंग में 87 दे शोों, 73 सवशेषताओों पर 17,000 से असर्क उत्तरदाताओों का मूल्ाों कन सकया गया।

7. नसतंबर 2023 में, भारत OIML (इं टरिेशिि ऑगािाइिेशि ऑफ िीगि मेटरोिॉिी) प्रमाणपत्र िारी िरिे
िािा दु निया िा __________ दे श बि गया।
1)15वाों
2)14वाों
3)16वाों
4)13वाों
5)10वाों
उत्तर- 4)13िां
स्पष्टीिरण:
भारत को दु सनया में कहीों भी वजन और माप बेचने के सलए अोंतरराष्ट्रीय स्तर पर स्वीकृत OIML (इों टरनेशनल
ऑगधनाइजेशन ऑफ लीगल मेटरोलॉजी) पैटनध अनुमोदन प्रमाि पत्र जारी करने के सलए एक प्रासर्करि के रूप में
मान्यता दी गई है। इसके साि ही भारत दु सनया का 13िां दे श बन गया जो OIML प्रमाणपत्र जारी कर सकता है।
i.उपभोक्ता मामलोों के मोंत्रालय के तहत उपभोक्ता मामलोों का सवभाग अब OIML प्रमािपत्र जारी करे गा।
ii.अोंतराधष्ट्रीय बाजार में वजन या माप बेचने के सलए OIML पैटनध अनुमोदन प्रमािपत्र असनवायध है ।

Report Errors in the PDF - ebooks@affairscloud.com Copyright 2014-2023 @ AffairsCloud.com 41


iii.अन्य दे श जो OIML प्रमािपत्र जारी कर सकते हैं वे ऑस्ट्र े सलया, क्कस्वट् जरलैंड, चीन, चेक गिराज्य, जमधनी, डे नमाकध,
फ्राोंस, यूनाइटे ड सकोंगडम (UK), जापान, नीदरलैंड, स्वीडन और स्लोवासकया हैं।

8. भारत िे साथ िौि सा दे श हाि ही में (नसतंबर’23 में) ध्रुिीय और आिानटि िि में भारतीय िानििों िो
प्रनशनक्षत िरिे िे निए सहमत हुआ है ?
1)रूस
2)फ़्ाोंस
3)ऑस्ट्र े सलया
4)यूनाइटे ड सकोंगडम
5)स्पेन
उत्तर- 1)रूस
स्पष्टीिरण:
भारत और रूस रूस के व्लासदवोस्तोक में रूसी समुद्री प्रसशक्षि सोंथिान में ध्रुिीय और आिानटि िि में भारतीय
िानििों को प्रसशसक्षत करने पर सहमत हुए, सजसका नाम जीआई एडसमरल नेवेल्स्की के नाम पर रखा गया है और यह
ससम्युलेटर सुसवर्ाओों से सुसक्कित है।
i.भारतीय नासवकोों को प्रसशसक्षत करने का सनिधय केंद्रीय बोंदरगाह, जहाजरानी और जलमागध मोंत्री सबाधनोंद सोनोवाल
और A.O. चेकुनकोव, रूस के व्लासदवोस्तोक में सुदूर पूवध और आकधसटक के सवकास के सलए रूसी सोंघ के मोंत्री के
बीच एक बैठक के दौरान हुआ।
ii.केंद्रीय मोंत्री सबाधनोंद सोनोवाल ने 10 से 13 ससतोंबर 2023 तक रूस के व्लासदवोस्तोक में आयोसजत 8वीों पूवी आसिधक
मोंच की बैठक में भी भाग सलया।
iii.भारत और रूस समुद्री सहयोग को व्यापक बनाने के सलए व्लासदवोस्तोक और चेन्नई के बीच उत्तरी समुद्री मागध
(NSR) के साि-साि पूवी समुद्री गसलयारे (EMC) जैसे नए पररवहन गसलयारोों का उपयोग करने की सोंभावना तलाशने
के सलए तैयार हैं ।

9. निम्ननिखित में से निस भारतीय निरासत स्थि िो हाि ही में (नसतंबर'23 में) सांस्कृनति निरासत श्रेणी िे
तहत संयुक्त राष्टर शैनक्षि, िैज्ञानिि और सांस्कृनति संगठि (UNESCO) िी निश्व निरासत सूची में शानमि
निया गया है ?
1)पसिम बोंगाल में शाोंसतसनकेतन
2)कनाधटक में होयसला की पसवत्र मण्डली
3)महाराष्ट्र में सहरि पत्थर के स्मारक
4)1 और 2 दोनोों
5)2 और 3 दोनोों
उत्तर- 4)1 और 2 दोिों
स्पष्टीिरण:
पसिम बोंगाल (WB) में शाोंसतसनकेतन और ििााटि में होयसिा िे पनित्र मण्डिी को सोंयुक्त राष्ट्र शैसक्षक, वैज्ञासनक
और साोंस्कृसतक सोंगठन (UNESCO) की सवश्व सवरासत सूची में अोंसकत सकया गया िा।
• शांनतनििेति भारत का 41िां और होयसिा िा पनित्र मण्डिी 42िां सवश्व र्रोहर थिल है।
• दोनोों को साोंस्कृसतक सवरासत श्रेिी के अोंतगधत सूचीबद्ध सकया गया िा।

Report Errors in the PDF - ebooks@affairscloud.com Copyright 2014-2023 @ AffairsCloud.com 42


• इसकी घोषिा 10 से 25 ससतोंबर 2023 तक सऊदी अरब के ररयाद में आयोसजत UNESCO की सवश्व र्रोहर
ससमसत के 45वें सवस्ताररत सत्र के दौरान की गई।
िोट: वतधमान में, भारत में कुल 42 UNESCO सवश्व र्रोहर थिल हैं , सजनमें 7 प्राकृसतक सवरासत थिल, 1 समसश्रत
सवरासत थिल और 34 साों स्कृसतक सवरासत थिल शासमल हैं ।

10. निम्ननिखित में से निस संगठि िे हाि ही में (नसतंबर’23 में) "ग्लोबि टर ें ि्स इि चाइड मोिेटरी पािटी
अिॉनििं ग टू इं टरिेशिि पािटी िाइि" ररपोटा तैयार िी है , निसमें प्रमुि निष्कषा नििािा गया है नि दु निया
में िुि अत्यनधि गरीब आबादी िा 52.5% बच्चे हैं ?
1)सवश्व बैंक समूह
2)सोंयुक्त राष्ट्र बाल कोष
3)सोंयुक्त राष्ट्र मानव सनपटान कायधक्रम
4)1 और 2 दोनोों
5)2 और 3 दोनोों
उत्तर- 4)1 और 2 दोिों
स्पष्टीिरण:
निश्व बैंि समूह और सोंयुक्त राष्ट्र बाल कोष (UNICEF) िारा तैयार की गई ररपोटध "ग्लोबि टर ें ि्स इि चाइड
मोिेटरी पािटी अिॉनििं ग टू इं टरिेशिि पािटी िाइि" के अनुसार, 2022 में, दु सनया की कुल चरम गरीब
आबादी में बच्चोों की सहस्सेदारी 52.5% िी, यानी दु सनया में हर दू सरा अत्यसर्क गरीब व्यक्कक्त एक बच्चा है ।
• लगभग 11.5% (52 समसलयन) भारतीय बच्चे अत्योंत गरीब घरोों में रहते हैं।
• यह सवश्व बैंक और UNICEF की तीसरी ररपोटध िी, सजसमें 2016 और 2020 की उनकी दो ररपोटें शासमल िीों।
• ररपोटध ने 2013 से 2022 तक बाल गरीबी में अनुमासनत रुझान प्रस्तुत सकए, जो तीन अोंतरराष्ट्रीय गरीबी रे खाओों
2.15 अमेररकी डॉलर (अत्यसर्क गरीबी), 3.65 अमेररकी डॉलर (सनम्न मध्यम आय), और 6.85 अमेररकी डॉलर
(ऊपरी मध्यम आय) पर आर्ाररत है।
• यह ररपोटध ससतोंबर 2022 में अपनाई गई नई वैसश्वक गरीबी रे खा पर आर्ाररत िी, सजसमें सवश्व बैंक ने अपने गरीबी
और असमानता प्लेटफॉमध पर डे टा अपडे ट सकया िा।

11. लसतंबर 2023 में लवश्व स्वास्थ्य संगठन (WHO) द्वारा िारी ग्लोबि ररपोटा ऑन हाइपरटें शन: द रे स अगेंस्ट्
ए साइिेंट लकिर के अनुसार, दु लनया ___________(वषा) तक बढे हुए रक्तचाप (अलनयंलत्रत उच्च रक्तचाप) के
प्रसार में 25% की कमी के स्वैस्किक वैलश्वक िक्ष्य को पूरा करने के रास्ते पर नही ं है।
1)2028
2)2025
3)2035
4)2030
5)2040
उत्तर- 2)2025
स्पष्टीकरण:
श्चवश्व स्वास्थ्य संगठन (WHO) ने उच्चरक्तदाब (उच्च रक्तचाप-BP) के वैश्चश्वक प्रभाव पर अपनी पहली ररपोटग िारी की,
श्चिसका शीषगक, "ग्लोबि ररपोटा ऑन हाइपरटें शन: द रे स अगेंस्ट् ए साइिेंट लकिर" है, साि ही िीवन बचाने के
श्चलए प्रभावी उच्चरक्तदाब दे खभाल पर श्चसफाररशें भी िारी की।

Report Errors in the PDF - ebooks@affairscloud.com Copyright 2014-2023 @ AffairsCloud.com 43


• ररपोटग से पता चलता है श्चक उच्चरक्तदाब से पीश्चड़त हर 5 में से लगभग 4 लोगों को पयागप्त उपचार नहीं श्चमल
रहा है। यह अनुमान लगाया गया है श्चक कवरे ि को ब़िाकर 2023 और 2050 के बीच 76 श्चमश्चलयन मौतों को
रोका िा सकता है।
• दु श्चनया 2025 तक ब़िे हुए रक्तचाप (अश्चनयंश्चत्रत उच्चरक्तदाब) की व्यापकता में 25% की कमी के स्वैस्िक
वैश्चश्वक लक्ष्य को पूरा करने की राह पर नहीं है।श्चवश्व स्वास्थ्य संगठन (WHO) ने उच्चरक्तदाब (उच्च रक्तचाप-
BP) के वैश्चश्वक प्रभाव पर अपनी पहली ररपोटग िारी की, श्चिसका शीषगक, "ग्लोबल ररपोटग ऑन हाइपरटें शन: द
रे स अगेंस्ट् ए साइलेंट श्चकलर" है, साि ही िीवन बचाने के श्चलए प्रभावी उच्चरक्तदाब दे खभाल पर श्चसफाररशें
भी िारी की।
• 1990 और 2019 के बीच, उच्चरक्तदाब (BP 140/90 mmHg या इससे अश्चर्क या उच्चरक्तदाब के श्चलए दवा
लेने वाले) के साि रहने वाले व्यस्क्तयों की संख्या 650 श्चमश्चलयन से दोगुनी होकर 1.3 श्चबश्चलयन हो गई है।
• ररपोटग के अनुसार, यश्चद भारत की लगभग 50% उच्चरक्तदाब से ग्रस्त आबादी अपने रक्तचाप को श्चनयंश्चत्रत कर
ले तो 2040 तक भारत में लगभग 4.6 श्चमश्चलयन मौतों को रोका िा सकता है।
• ररपोटग से पता चलता है श्चक भारत में 30-79 वषग की आयु के अनुमाश्चनत 188.3 श्चमश्चलयन वयस्क उच्चरक्तदाब से
पीश्चड़त हैं। 50% श्चनयंत्रि दर प्राप्त करने के श्चलए, उच्चरक्तदाब से पीश्चड़त 67 श्चमश्चलयन से अश्चर्क लोगों का प्रभावी
ढं ग से इलाि करने की आवश्यकता होगी।

12. उस संगठन का नाम बताइए लिसने हाि ही में (लसतंबर'23 में) सतत लवकास िक्ष्यों (SDG) की उपिस्कि
को आगे बढाने के लिए अंतरााष्टरीय मानकीकरण संगठन (ISO) के साथ एक आशय पत्र (SoI) पर हस्ताक्षर
लकए हैं।
1)श्चवश्व बैंक
2)अंतरागष्ट्रीय मुद्रा कोष
3)संयुक्त राष्ट्र पयागवरि कायगक्रम
4)संयुक्त राष्ट्र श्चवकास कायगक्रम
5)िलवायु पररवतगन पर संयुक्त राष्ट्र फ्रेमवकग कन्वेंशन
उत्तर-4)संयुक्त राष्टर लवकास कायाक्रम
स्पष्टीकरण:
अंतरागष्ट्रीय मानकीकरि संगठन (ISO) और संयुक्त राष्ट्र श्चवकास कायगक्रम (UNDP) ने सतत श्चवकास लक्ष्यों (SDG) की
उपलस्ि को आगे ब़िाने में एक उपकरि के रूप में अंतराग ष्ट्रीय मानकों का उपयोग करने के इरादे के एक आशय
पत्र(SoI) पर हस्ताक्षर श्चकए हैं।
• सहयोग का उद्दे श्य UNDP के SDG प्रभाव मानकों और प्रासंश्चगक ISO मानकों के आर्ार पर SDG के श्चलए
पहला अंतरराष्ट्रीय मानक श्चवकश्चसत करना भी है ।
• यह साझेदारी SDG17 - लक्ष्यों के श्चलए साझेदारी के अनुरूप है।
• यह सहयोग दस्तावेज़ीकरि और प्रमािन के श्चलए एक सामान्य आर्ार श्चवकश्चसत करे गा, श्चिससे संगठनों और
कंपश्चनयों के श्चलए अपने प्रयासों को संरेस्खत करना और SDG की श्चदशा में प्रगश्चत का दस्तावेिीकरि करना
आसान हो िाएगा।
• यह सहयोग व्यवसायों और श्चनवेशकों के लाभ के श्चलए नए प्रबंर्न मानक तैयार करे गा।

Report Errors in the PDF - ebooks@affairscloud.com Copyright 2014-2023 @ AffairsCloud.com 44


13. लसतंबर 2023 में लवश्व स्वास्थ्य संगठन (WHO) द्वारा िारी ‘ग्रोथ ऑफ़ अल्ट्र ा-प्रोसेस्ि फूि् स इन इं लिया-
एन एनालिलसस ऑफ़ टर ें ि्स, इश्यूि एं ि पॉलिसी रे कमेंिेशन्स’ ररपोटा के अनुसार, भारत का अल्ट्र ा-प्रोसेस्ि
फूि क्षेत्र 2011 से 2021 तक िुदरा लबक्री मूल्य में_______________की चक्रवृस्कि वालषाक वृस्कि दर (CAGR) से
बढा।
1)14.34%
2)13.37%
3)15.31%
4)12.34%
5)16.48%
उत्तर- 2)13.37%
स्पष्टीकरण:
श्चवश्व स्वास्थ्य संगठन (WHO) की ररपोटग "ग्रोि ऑफ़ अल्ट्र ा-प्रोसेथि फूि् स इन इं श्चिया- एन एनाश्चलश्चसस ऑफ़ टर ें ि्स,
इश्यूि एं ि पॉश्चलसी रे कमेंिेशन्स" के अनुसार, भारत का अल्ट्र ा-प्रोसेस्ि फूि सेक्टर 2011 से 2021 तक खुदरा
श्चबक्री मूल्य में 13.37% की चक्रवृस्ि वाश्चषगक वृस्ि दर (CAGR) से ब़िा है।
i.यह ररपोटग भारत के श्चलए WHO कंटर ी ऑश्चफस द्वारा इं श्चियन काउं श्चसल फॉर ररसचग ऑन इं टरनेशनल इकोनॉश्चमक
ररलेशंस (ICRIER) के सहयोग से श्चवकश्चसत की गई है । ररपोटग में स्वथि िीवन शैली के श्चलए पोषि संबंर्ी बदलाव का
आह्वान श्चकया गया है।
ii.श्चिश्चटश हाटग फाउं िेशन के अनुसार, अल्ट्र ा-प्रोसेथि फूि पदािग लंबी शेल्फ लाइफ वाली वस्तुएं हैं श्चिनमें आमतौर पर
पांच या अश्चर्क सामश्चग्रयां होती हैं, श्चिनमें संरक्षक, इमल्सीफायर, श्चमठास और कृश्चत्रम रं ग और स्वाद शाश्चमल हैं।
iii.2011 से 2021 तक खुदरा श्चबक्री की मात्रा में, पेय पदािों की सबसे बड़ी श्चहस्सेदारी िी, इसके बाद चॉकलेट और
चीनी कन्फेक्शनरी, और तैयार सुश्चवर्ा फूि दू सरे और तीसरे थिान पर िे।

14. उस भारतीय नौसेना िहाि (INS) का नाम बताइए लिसने हाि ही में (लसतंबर’23 में) भारत और श्रीिंका
के बीच IN-SLN िाइवेक्स 23 के लहस्से के रूप में महत्वपूणा लमलश्रत गैस गोतािोरी प्रलशक्षण सत्र आयोलित
लकया है ।
1)INS श्चनरूपक
2)INS अस्त्रर्ारिी
3)INS श्चनरीक्षक
4)INS सवेक्षक
5)INS संध्याक
उत्तर-3)INS लनरीक्षक
स्पष्टीकरण:
भारतीय नौसेना िहाि (INS) लनरीक्षक (A-15), भारतीय नौसेना के गोताखोरी सहायता और पनिु ब्बी बचाव पोत
(DSRV) ने महत्वपूिग लमलश्रत गैस गोतािोरी प्रश्चशक्षि सत्र सफलतापूवगक आयोश्चित श्चकए।
• यह भारत और श्रीलंका के बीच आठ श्चदवसीय गोताखोरी प्रश्चशक्षि पररश्चनयोिन अभ्यास (14-21 श्चसतंबर
2023) IN-SLN िाइवेक्स 23 का एक श्चहस्सा है।
• यह पड़ोसी दे शों में क्षमता श्चनमागि के श्चलए भारत की ‘नेबरहुि फस्ट्ग ’ नीश्चत के अनु रूप है।
• श्चमश्चश्रत गैस गोताखोरी में लगभग 21% ऑक्सीिन और 79% नाइटर ोिन के मानक वायु श्चमश्रि के अलावा
अन्य श्वास गैसों का उपयोग शाश्चमल है।

Report Errors in the PDF - ebooks@affairscloud.com Copyright 2014-2023 @ AffairsCloud.com 45


i.भारतीय तट रक्षक (ICG) प्रदू षि-श्चनयंत्रि पोत (PCV) ‘समुद्र प्रहरी’ ने बैंकॉक, िाईलैंि में खलोंग टोई बंदरगाह पर
प्रदू षि प्रश्चतश्चक्रया टे बल-टॉप अभ्यास और प्रदशगन श्चकया।
ii.यह अभ्यास िहाि की बैंकॉक की चार श्चदवसीय यात्रा (17-20 श्चसतंबर 2023) के दौरान आयोश्चित कायगक्रमों का
श्चहस्सा है।

15. नसतंबर 2023 में ििािाई नथंि टैं ि फ्रेिर इं स्ट्ीट्यूट द्वारा िारी इििोनमि फ्रीिम ऑफ़ द िडा (EFW)
2023 एिुअि ररपोटा िे अिुसार, ____________ (दे श) 8.56 िी रे नटं ग िे साथ हांगिांग िो पीछे छोडिर
दु निया िी सबसे मुक्त अथाव्यिस्था बि गया है , िबनि भारत सूचिांि में _______ स्थाि नदया गया।
1)न्यूजीलैंड; 97
2)क्कस्वट् जरलैंड; 87
3)ससोंगापुर; 82
4)क्कस्वट् जरलैंड; 97
5)ससोंगापुर; 87
उत्तर-5)नसंगापुर; 87
स्पष्टीिरण:
कनाडाई सिोंक टैं क फ्रेजर इों स्ट्ीट्यूट िारा जारी इििोनमि फ्रीिम ऑफ़ द िडा (EFW) 2023 एिुअि ररपोटा के
अनुसार, 8.56 रे सटों ग के साि नसंगापुर दु सनया की सबसे मुक्त अिधव्यवथिा है। इों डेक्स में भारत 87िें थिान पर है।
• हाोंगकाोंग और क्कस्वट् जरलैंड क्रमशः 8.55 और 8.47 की रे सटों ग के साि दू सरी और तीसरी सबसे मुक्त अिधव्यवथिा
हैं।
• 1970 में EFW इं िेक्स की शुरुआत के बाद से हांगिांग िे शीषध थिान बरिरार रखा है। पहली बार हाों गकाों ग
अपने नोंबर एक थिान से सफसलकर दू सरे थिान पर आ गया है।
• सनचले पाोंच दे श - यमन (161), सूडान (162), सीररया (163), सजम्बाब्वे (164)और वेनेजुएला (165)हैं।
• 2023 EFW ररपोटध में भारत एक थिान सफसलकर 6.62 रे सटों ग के साि 87वें थिान पर है। EFW 2022 की
एनुअल ररपोटध में भारत को 6.64 रे सटों ग समली और वह 86वें थिान पर रहा।
• नई सदल्ली क्कथित स्वतोंत्र सिोंक-टैं क 'सेंटर फॉर सससवल सोसाइटी' इों डेक्स के सह-प्रकाशकोों में से एक है ।

16. निम्ननिखित में से निस भारतीय सशस्त्र बि िे िाउं टर टे रररज्म फीड टर े निंग एक्सरसाइि (FTX) 2023
पर एसोनसएशि ऑफ साउथईस्ट् एनशयि िेशंस (ASEAN) िे रक्षा मंनत्रयों िी बैठि (ADMM) प्लस
एक्सपटा िनििंग ग्रुप (EWG) में भाग निया।
1)भारतीय सेना
2)भारतीय नौसेना
3)भारतीय वायुसेना
4)1 और 3 दोनोों
5)2 और 3 दोनोों
उत्तर-1)भारतीय सेिा
स्पष्टीिरण:
भारतीय सेना काउों टर टे रररज्म फीड टर े सनोंग एक्सरसाइज (FTX) 2023 पर एसोससएशन ऑफ साउिईस्ट् एसशयन
नेशोंस (ASEAN) के रक्षा मोंसत्रयोों की बैठक (ADMM) प्लस एक्सपटा िनििंग ग्रुप (EWG) में भाग लेती है।
i.अभ्यास का मुख्य उद्दे श्य आतोंकवाद-सनरोर् के क्षेत्र में क्षेत्रीय सहयोग को मजबूत करना और बढावा दे ना है ।

Report Errors in the PDF - ebooks@affairscloud.com Copyright 2014-2023 @ AffairsCloud.com 46


ii.ADMM-प्लस ASEAN और उसके आठ सोंवाद साझेदारोों ऑस्ट्र े सलया, चीन, भारत, जापान, न्यूजीलैंड, कोररया
गिराज्य, रूस और सोंयुक्त राज्य अमेररका (USA) के सलए एक मोंच है।

17. नसतंबर 2023 में, संयुक्त राष्टर नििास िायाक्रम (UNDP) िे स्थायी नित्त िो समथाि और बढ़ािे िी बढ़ती
िैनश्वि आिश्यिता िे निए एि सनक्रय प्रनतनक्रया िे रूप में अनभिि ___________ िॉन्च निया।
1)जलवायु सवत्त पहल
2)ई-लसनिंग प्रोग्राम
3)ग्रीन फाइनेंस लैब
4)पीयर2पीयर लेंसडों ग मोबाइल एक्कप्लकेशन
5)ई-कॉमसध वेबसाइट
उत्तर-2)ई-िनििंग प्रोग्राम
स्पष्टीिरण:
सोंयुक्त राष्ट्र सवकास कायधक्रम (UNDP) ने थिायी सवत्त को समिधन और बढाने की बढती वैसश्वक आवश्यकता के प्रसत
ससक्रय प्रसतसक्रया के रूप में एक असभनव ई-िनििंग प्रोग्राम शुरू सकया।
• सतत सवत्त SDG महत्वाकाों क्षाओों का आर्ार है , जो दु सनया भर के व्यक्कक्तयोों को राष्ट्रीय SDG लक्ष्ोों के साि र्न
सोंरेक्कखत करने और सभी के लाभ के सलए एक सावधभौसमक सटकाऊ सवत्त ढाोंचा तैयार करने के सलए सशक्त बनाता
है।
• UNDP के सस्ट्े नेबल फाइनेंस हब (SFH ) ने थिायी सवत्त पहल को आगे बढाने में महत्वपूिध भूसमका सनभाई है।

18. निम्ननिखित में से निस इिाई िे इं टरिेशिि इं स्ट्ीट्यूट फॉर पॉपुिेशि साइं सेि (IIPS) िे साथ नमििर
'हमारे बुिुगों िी संस्थागत प्रनतनक्रयाओं िी दे िभाि: इं निया एनिंग ररपोटा 2023' िा अिािरण निया।
1)सवश्व स्वास्थ्य सोंगठन (WHO)
2)भारतीय सचसकत्सा अनुसोंर्ान पररषद (ICMR)
3)सोंयुक्त राष्ट्र जनसोंख्या कोष (UNFPA) भारत
4)अोंतराधष्ट्रीय बचाव ससमसत (IRC)
5)एमनेस्ट्ी इों टरनेशनल
उत्तर -3)संयुक्त राष्टर ििसंख्या िोष (UNFPA) भारत
स्पष्टीिरण:
सोंयुक्त राष्ट्र जनसोंख्या कोष (UNFPA) भारत ने इों टरनेशनल इों स्ट्ीट्यूट फॉर पॉपुलेशन साइों सेज (IIPS) के साि समलकर
‘िेयररं ग फॉर आिर एडसा इं स्ट्ीटू शिि रे स्पॉिेस: इं निया एनिंग ररपोटा 2023’ का अनावरि सकया।
• इों सडया एसजोंग ररपोटध 2023 भारत में वृद्ध व्यक्कक्तयोों की जीवन क्कथिसतयोों और कल्ाि की गहन समीक्षा का
प्रसतसनसर्त्व करती है ।
• इों सडया एसजोंग ररपोटध 2023 अपनी सपछली जारी ररपोटध इों सडया एसजोंग ररपोटध - 2017 पर आर्ाररत है।
• भारत की आबादी में वररष्ठ नागररकोों की सहस्सेदारी 10% है। 2036 तक यह 14.36% हो जाएगी, 2050 में यह
20.8% होगी

Report Errors in the PDF - ebooks@affairscloud.com Copyright 2014-2023 @ AffairsCloud.com 47


19. नसतंबर 2023 में िारी टाइम्स हायर एिुिेशि (THE) िडा यूनििनसाटी रैं निंग 2024 िे संबंध में
निम्ननिखित में से िौि सा/से नबंदु "सही" है /हैं?
A) भारतीय निज्ञाि संस्थाि, बेंगिुरु (IISc बेंगिुरु) भारतीय संस्थािों में िंबर 1 स्थाि पर है , िो 2017 िे बाद
पहिी बार 201-250 बैंि रैं निंग में आया है।
B) रैं निंग में 80 भारतीय संस्थािों िो शानमि िरिे िे साथ, भारत सूची में तीसरा सबसे अनधि प्रनतनिनधत्व
िािा दे श बि गया।
C) यूिाइटे ि निंगिम (UK) िी ऑक्सफोिा यूनििनसाटी िगातार आठिें साि िडा यूनििनसाटी रैं निंग 2024 में
प्रथम स्थाि पर रही।
1)केवल A
2)केवल A & B
3)केवल B & C
4)केवल A & C
5)सभी A, B & C
उत्तर- 4)िेिि A & C
स्पष्टीिरण:
'टाइम्स हायर एजुकेशन (THE) िडा यूनििनसाटी रैं निंग 2024' के अनुसार, भारतीय सवज्ञान इों स्ट्ीट्यूट, बेंगलुरु (IISc
बेंगिुरु) भारतीय इों स्ट्ीटू ट् स में िंबर 1 स्थाि पर है , जो 2017 के बाद पहली बार 201-250 बैंड रैं सकोंग में आया है।
IISc बेंगलुरु 2023 में 251-300 बैंड से ऊपर चला गया है।
• सवश्व यूसनवससधटी रैं सकोंग 2024 में शासमल 91 भारतीय इं स्ट्ीटू ट् स के साि, भारत सूची में चौिा सबसे असर्क
प्रसतसनसर्त्व वाला दे श है। 2023 में, भारत के 75 इों स्ट्ीटू ट् स को सूची में शासमल सकया गया िा।
• यूनाइटे ड सकोंगडम (UK) की यूनििनसाटी ऑफ ऑक्सफोिा िगातार आठिें साि THE वडध यूसनवससधटी
रैं सकोंग 2024 में पहले थिान पर है , जबसक सोंयुक्त राज्य अमेररका (USA) की स्ट्ै नफोडध यूसनवससधटी और
मैसाचुसेट्स इों स्ट्ीट्यूट ऑफ टे क्नोलॉजी (MIT) क्रमशः दू सरे और तीसरे थिान पर हैं।
• सवश्व सवश्वसवद्यालय रैं सकोंग 2024 में 43 भारतीय सवश्वसवद्यालयोों को 1,000 से नीचे थिान सदया गया है।
• 2024 रैं सकोंग में 20 नए भारतीय सवश्वसवद्यालयोों को शासमल सकया गया।
िोट- .'द वडध यूसनवससधटी रैं सकोंग' के 20वें सोंस्करि में 108 दे शोों और क्षेत्रोों के 1,904 सवश्वसवद्यालय शासमल िे।

20. हाि ही में (नसतंबर'23 में) संयुक्त राष्टर शैनक्षि, िैज्ञानिि और सांस्कृनति संगठि (UNESCO) िी निश्व
निरासत सूची में नितिे िए स्थि िोडे गए, नििमें भारत िे पनिम बंगाि में शांनतनििेति और भारत िे
ििााटि में होयसिा िे पनित्र समूह शानमि हैं ?
1)30
2)40
3)62
4)50
5)42
उत्तर- 5)42

Report Errors in the PDF - ebooks@affairscloud.com Copyright 2014-2023 @ AffairsCloud.com 48


स्पष्टीिरण:
सवश्व र्रोहर ससमसत (WHC) का 45 िां सत्र ररयाद, सऊदी अरब में हुआ, जो 25 ससतोंबर को समाप्त हुआ। इस सत्र में
सोंयुक्त राष्ट्र शैसक्षक, वैज्ञासनक और साोंस्कृसतक सोंगठन (UNESCO) िी निश्व धरोहर सूची में अन्य महत्वपूिध सनिधयोों
के साि-साि 42 िए स्थिों को जोडा गया।
• ससमसत की अध्यक्षता UNESCO में सऊदी अरब की थिायी प्रसतसनसर् H.H. राजकुमारी हाइफ़ा अल मोसग्रन ने
की।
• सऊदी अरब साम्राज्य (KSA) ने WHC के 45वें सत्र के सलए मेजबान दे श के रूप में कायध सकया।
i.भारत के पनिम बंगाि में शांनतनििेति और ििााटि में होयसिा िे पनित्र स्थिों को सोंयुक्त राष्ट्र शैसक्षक,
वैज्ञासनक और साोंस्कृसतक सोंगठन (UNESCO) की सवश्व र्रोहर सूची में जोडा गया है ...पूरी जानकारी के सलए यहाों
क्कक्लक करें
ii.42 नए अोंसकत थिलोों में से, 33 साोंस्कृसतक हैं और 9 प्राकृसतक हैं। इससे 168 दे शोों में UNESCO की सवश्व र्रोहर
थिलोों की कुल सोंख्या 1199 (993 साोंस्कृसतक, 227 प्राकृसतक, 39 समसश्रत) हो गई है। ससमसत ने 5 मौजूदा सवश्व र्रोहर
थिलोों के सवस्तार को भी मोंजूरी दी।

21. नसतंबर 2023 में व्यापार और नििास पर संयुक्त राष्टर सम्मेिि (UNCTAD) द्वारा िारी समुद्री पररिहि
2023 ररपोटा िी समीक्षा में िहा गया है नि िीिाबोिाइनिंग नशनपंग िी िागत प्रनत िषा ___________ अमेररिी
िॉिर से अनधि होगी क्ोंनि उत्सिाि में िृखि िारी है।
1)200 सबसलयन
2)400 सबसलयन
3)100 सबसलयन
4)500 सबसलयन
5)300 सबसलयन
उत्तर- 3)100 नबनियि
स्पष्टीिरण:
व्यापार और सवकास पर सों युक्त राष्ट्र सम्मे लन (UNCTAD) की समुद्री पररवहन की समीक्षा 2023 में समुद्री पररवहन
को डीकाबोनाइज करने के सलए कारध वाई करने और ससस्ट्म-व्यापी सहयोग करने का आग्रह सकया गया। ररपोटध में
चेतावनी दी गई है सक उत्सजधन में वृक्कद्ध जारी रहने के कारि िीिाबोिाइनिंग नशनपंग की लागत प्रसत वषध 100
नबनियि अमेररिी िॉिर से असर्क होगी।
i.UNCTAD सशसपोंग उद्योग में चल रहे काबधन उत्सजधन और सनयामक असनसितताओों को दू र करने के सलए स्वच्छ ईोंर्न,
सडसजटल समार्ान और एक न्यायसोंगत पररवतधन की आवश्यकता पर जोर दे ता है।
ii.समुद्री पररवहन 2023 की समीक्षा 27 ससतोंबर, 2023 को (सवश्व समुद्री सदवस 2023 - 28 ससतोंबर 2023 से पहले)
लॉन्च की गई िी।
iii.सशसपोंग प्रभुत्व और उत्सजधन:
• वैसश्वक व्यापार मात्रा का 80% से असर्क सहस्सा है।
• वैसश्वक ग्रीनहाउस गैस उत्सजधन में लगभग 3% का योगदान दे ता है ।
• सपछले दशक में उत्सजधन में 20% की वृक्कद्ध हुई।

Report Errors in the PDF - ebooks@affairscloud.com Copyright 2014-2023 @ AffairsCloud.com 49


22. नसतंबर 2023 में, इं टरिेशिि एयर टर ांसपोटा एसोनसएशि (IATA) िे ____________ (राज्य) में आगामी
िेिर अंतरााष्टरीय एयरपोटा िे निए तीि अक्षर िा िोि 'DXN' आिंनटत निया।
1)ओसडशा
2)उत्तर प्रदे श
3)पसिम बोंगाल
4)सबहार
5)झारखण्ड
उत्तर- 2)उत्तर प्रदे श
स्पष्टीिरण:
दु सनया भर की एयरलाइनोों के सलए व्यापार सोंघ, इों टरनेशनल एयर टर ाोंसपोटध एसोससएशन (IATA) ने उत्तर प्रदे श (UP)
के न्यू ओखला औद्योसगक सवकास प्रासर्करि (िोएिा) िे िेिर में आगामी ग्रीनफीड एयरपोटध के सलए तीि अक्षर
िा िोि 'DXN' आवोंसटत सकया है।
• कोड 'DXN' एयरपोटध की नोएडा, सदल्ली और पसिमी UP से सनकटता का प्रतीक है।
i.IATA िारा सौोंपा गया यह तीन-अक्षर वाला कोड सडज़ाइनर होगा, सजसका उपयोग एयरलाइों स और टर ै वल एजेंससयाों
अोंतरराष्ट्रीय स्तर पर, नोएडा इों टरनेशनल एयरपोटध की पहचान करने के सलए करें गी।
ii.यह सुसनसित करता है सक यात्री सवसभन्न प्लेटफामों पर एयरपोटध की आसानी से पहचान कर सकें।
iii.2024 के अोंत तक यह पहल पूरी तरह से लागू हो जाएगी। एयरपोटध सोंचालन शुरू होते ही कोड ससक्रय हो जाएगा।

23. निस िंपिी िे हाि ही में (नसतंबर'23 में) 35 िषों िे निए रिरिाि सेिाओं िे साथ 120 िंदे भारत टर े िों
िे निमााण िे निए भारतीय रे ििे िे साथ एि आपूनता अिु बंध पर हस्ताक्षर निए हैं ?
1)काइनेट रे लवे सॉल्ूशोंस
2)जापान टर ाोंसपोटध इों जीसनयररों ग कोंपनी
3)कैनेसडयन पैसससफक रे लवे
4)बॉम्बासडध यर पररवहन
5)स्ट्ै डलर रे ल AG
उत्तर- 1)िाइिेट रे ििे सॉल्यूशंस
स्पष्टीिरण:
भारतीय रे ििे ने िंदे भारत टर े िों के सनमाधि के सलए भारत-रूसी सोंयुक्त उद्यम (JV) नििेट रे ििे सॉल्यूशंसके साि
एक आपूनता अिुबंध पर हस्ताक्षर सकए हैं। अनुबोंर् के अनुसार, काइनेट रे लवे सॉल्ूशोंस 35 वषों तक रखरखाव
सेवाओों के साि-साि 120 वोंदे भारत टर े नोों का सनमाधि करे गा।
i.काइनेट रे लवे सॉल्ूशोंस वोंदे भारत टर े नोों के उत्पादन के सलए मेटरोवैगनमैश (MWM) और लोकोमोसटव इलेररॉसनक
ससस्ट्म्स (LES), रूस और रे ल सवकास सनगम सलसमटे ड (RVNL), भारत के बीच एक JV है। MWM रे ल उपकरि
सनमाधता, टर ाोंसमैशहोक्कडोंग JSC ग्रुप (टर ाोंसमैश) की सहायक कोंपनी है।
ii.वोंदे भारत टर े न पररयोजना के सलए MWM, LES और RVNL के बीच सहयोग को भारतीय और रूसी भागीदारोों के
बीच बहुमत के स्वासमत्व के मुद्दे पर शुरू से ही लगातार चुनौसतयोों का सामना करना पडा।
iii.सववाद के कारि भारतीय रे लवे के साि अनुबोंर् पर हस्ताक्षर करने में ससतोंबर 2023 तक की दे री हुई। भारतीय
रे लवे ने सदसोंबर 2022 में वोंदे भारत टर े न सेट के सलए बोसलयाों आमोंसत्रत की िीों और उन्ें माचध 2023 में खोला गया िा।

Report Errors in the PDF - ebooks@affairscloud.com Copyright 2014-2023 @ AffairsCloud.com 50


GOVT SCHEMES
1. निस राज्य सरिार िे हाि ही में (अगस्त'23 में) मुख्यमंत्री निशुिि अन्नपूणाा फूि पैिेट (MNAFP) योििा
शुरू िी है ?
1)गुजरात
2)महाराष्ट्र
3)राजथिान
4)पसिम बोंगाल
5)ओसडशा
उत्तर-3)रािस्थाि
स्पष्टीिरण:
रािस्थाि के मुख्यमोंत्री (CM) अशोक गहलोत ने स्वतोंत्रता सदवस (15 अगस्त 2023)के अवसर पर मुख्यमोंत्री सन:शुल्क
अन्नपूिाध फूड पैकेट योजना (MNAFP) (मुख्यमोंत्री मुफ्त अन्नपूिाध फूड पैकेट योजना) शुरू की।
• यह योजना राजथिान सरकार की प्रमुख कल्ािकारी योजनाओों में से एक है , जो हर महीने 10.4 समसलयन से
असर्क पररवारोों को मुफ्त भोजन पैकेट सवतररत करना चाहती है।
• इस योजना से 50 समसलयन व्यक्कक्तयोों को लाभ होगा।
i.इस योजना के तहत, राष्टरीय िाद्य सुरक्षा अनधनियम (NFSA) के तहत आने वाले पररवारोों को हर महीने उसचत मूल्
की दु कानोों (FPS) से अन्नपूिाध भोजन पैकेट मुफ्त में प्राप्त होोंगे।
ii.प्रत्येक खाद्य पैकेट में 1 सकलोग्राम चना दाल, चीनी, नमक, 1 लीटर सोयाबीन पररष्कृत खाद्य तेल, 100 ग्राम समचध
पाउडर और र्सनया पाउडर और 50 ग्राम हल्दी होगी।

2. नसतंबर 2023 में शुरू िी गई 'मेरा नबि मेरा अनधिार' योििा िे संबंध में निम्ननिखित में से िौि सा नबंदु
"सही" है/हैं?
A) नित्त मंत्रािय िे रािस्व निभाग िे तहत िेंद्रीय अप्रत्यक्ष िर और सीमा शुल्क (CBIC) िे 'मेरा नबि मेरा
अनधिार' िामि एि चािाि प्रोत्साहि योििा शुरू िी।
B) यह योििा 3 राज्यों: असम, गुिरात और हररयाणा और 3 िेंद्र शानसत प्रदे शों: पुिुचेरी, दमि और दीि
और दादरा और िगर हिेिी में शुरू िी गई थी।
C) यह पहि मानसि और त्रैमानसि ििी-िर ॉ िे माध्यम से 10,000 रुपये से 1 िरोड रुपये ति िे ििद
पुरस्कार प्रदाि िरती है।
1)केवल A
2)केवल A & B
3)केवल B & C
4)केवल A & C
5)सभी A, B & C
उत्तर -5)सभी A, B & C
स्पष्टीिरण:
सवत्त मोंत्रालय के राजस्व सवभाग के तहत केंद्रीय अप्रत्यक्ष कर और सीमा शुल्क (CBIC) ने 'मेरा नबि मेरा अनधिार'
नामक एक चालान प्रोत्साहन योजना शुरू की।

Report Errors in the PDF - ebooks@affairscloud.com Copyright 2014-2023 @ AffairsCloud.com 51


• इस योजना का उद्दे श्य नागररकोों और उपभोक्ताओों को सामान या सेवाओों की व्यवसाय-से-उपभोक्ता (B2C)
खरीदारी करते समय सवक्रेता से माल और सेवा कर (GST) चालान माों गने के सलए प्रोत्सासहत करना है।
• यह योजना 3 राज्यों (असम, गुजरात और हररयािा) और 3 िेंद्र शानसत प्रदे शों (UT) (पुडुचेरी, दमन और
दीव और दादरा और नगर हवेली) में शुरू की गई िी।
i.उद्दे श्य: इस योजना का उद्दे श्य GST प्रसक्रयाओों और राजस्व सोंग्रह को बढाना है।
• यह पहल माससक और त्रैमाससक लकी-डर ॉ के माध्यम से 10,000 रुपये से 1 िरोड रुपये तक के नकद
अवाडध प्रदान करती है ।
• यह अप्रत्यक्ष रूप से कर चोरी को रोकने, जवाबदे ही को बढावा दे ने और औपचाररक अिधव्यवथिा के सवकास
का समिधन करने में योगदान दे ता है।
• मानदों ड: न्यूनतम खरीद मूल् 200 रुपये वाले चालान लकी डर ा के सलए पात्र हैं , और व्यक्कक्त एक महीने में
असर्कतम 25 चालान अपलोड कर सकते हैं।

3. निस राज्य सरिार िे हाि ही में (नसतंबर'23 में) निशेष रूप से निििांग बच्चों िो सशक्त बिािे िे निए
मुख्यमंत्री िेि क्षमता, पुिनिामााण आिांक्षाएं और आिीनििा (SABAL) योििा शुरू िी है ?
1)कनाधटक
2)गुजरात
3)सहमाचल प्रदे श
4)झारखण्ड
5)मध्य प्रदे श
उत्तर-3)नहमाचि प्रदे श
स्पष्टीिरण:
4 ससतोंबर, 2023 को, सहमाचल प्रदे श (HP) के मुख्यमोंत्री (CM) सुखसवोंदर ससोंह सुक्खू ने सवध सशक्षा असभयान और
सोंपकध फाउों डेशन िारा सोंयुक्त रूप से आयोसजत कायधक्रम के दौरान HP के हमीरपुर सजले के नादौन से मुख्यमंत्री
िेि क्षमता, पुिनिामााण आिांक्षाएं और आिीनििा (SABAL) का शुभारों भ सकया।
• इस योजना का उद्दे श्य राज्य में सवशेष रूप से सवकलाोंग बच्चोों को मुख्यर्ारा में जोडकर उनके जीवन में सुर्ार
करना है।
i.राज्य सरकार ने सवशेष रूप से सवकलाोंग बच्चोों को आवश्यक सुसवर्ाएों और अवसर प्रदान करने के सलए हमीरपुर,
सोलन और सशमला सजलोों में लगभग 400 स्कूलोों को समिधन दे ने की योजना बनाई है।
ii.CM ने क्कस्वफ्ट चैट ऐप पर 'अभ्यास सहमाचल' और 'सशक्षक सहायता' चैटबॉट भी लॉन्च सकए, और यह कन्वसेशनल
आसटध सफसशयल इों टेसलजेंस (AI) िारा सोंचासलत है। ये चैटबॉट छात्रोों के सलए सुसवर्ाजनक पाठ सोंशोर्न का समिधन करते
हैं, कक्षाओों में बेहतर सशक्षि और सशक्षि के सलए क्कक्वज़ और शैसक्षक वीसडयो पेश करते हैं ।

4. नसतंबर 2023 में, ििीि और ििीिरणीय ऊिाा मंत्रािय (MNRE) िे प्रधाि मंत्री निसाि ऊिाा सुरक्षाम
उत्थाि महानभयाि योििा (PM-KUSUM योििा) िे घटि C िे तहत स्वदे शी घरे िू सामग्री आिश्यिता
(DCR) िी आिश्यिता िे प्रािधाि िे निए छूट िो ____________ ति बढ़ा नदया है।
1)31 जनवरी 2024
2)30 नवोंबर 2023
3)31 सदसोंबर 2023
4)30 अप्रैल 2024

Report Errors in the PDF - ebooks@affairscloud.com Copyright 2014-2023 @ AffairsCloud.com 52


5)31 माचध 2024
उत्तर- 5)31 माचा 2024
स्पष्टीिरण:
नवीन और नवीकरिीय ऊजाध मोंत्रालय (MNRE) ने प्रर्ान मोंत्री सकसान ऊजाध सुरक्षाम उत्थान महासभयान योजना (PM-
KUSUM योििा) के घटक C के तहत स्वदे शी घरे लू सामग्री आवश्यकता (DCR) की आवश्यकता के प्रावर्ान के
सलए छूट को 31 माचा 2024 तक बढा सदया है।
• 20 जून, 2023 को या उससे पहले सौोंपी गई पररयोजनाओों के सलए सोलर सेल्स के सलए DCR आवश्यकता को
माफ कर सदया गया िा।
• यह अगस्त 2022 में MNRE िारा जारी ज्ञापन की सनरों तरता है , सजसने DCR छूट को 20 जून 2023 तक बढा
सदया िा। यह सनिधय इस मामले पर उद्योग का प्रसतसनसर्त्व प्राप्त करने के बाद आया है।
• सकसानोों को खेती के सलए सौर ससोंचाई पोंप थिासपत करने के सलए सक्कब्सडी दे ने के सलए MNRE िारा माचध 2019
में प्रर्ान मोंत्री सकसान ऊजाध सुरक्षा उत्थान महासभयान योजना (PM-KUSUM योजना) शुरू की गई िी।

5. निस राज्य सरिार िे हाि ही में (नसतंबर'23 में) गृनहनणयों िो नित्तीय सहायता प्रदाि िरिे िे निए गृह
आधार स्वीिृनत आदे श नितररत निए हैं ?
1)कनाधटक
2)गोवा
3)गुजरात
4)झारखण्ड
5)महाराष्ट्र
उत्तर- 2)गोिा
स्पष्टीिरण:
गोिा के मुख्यमोंत्री (CM) प्रमोद सावोंत ने गृसहसियोों को आसिधक रूप से समिधन दे ने, उनकी स्वतोंत्रता को बढावा दे ने
और उनके जीवन की गुिवत्ता में सुर्ार करने के सलए, रवीोंद्र भवन, साोंखली (गोवा) में आयोसजत एक कायधक्रम में कई
लाभासिधयोों को गृह आधार मोंजूरी आदे श सवतररत सकए।
• यह पहल मसहला एवों बाल सवकास सनदे शालय िारा आयोसजत की गई िी।
• CM ने योजना, साोंक्कख्यकी और मूल्ाोंकन सनदे शालय िारा आयोसजत स्वयोंपुरा गोवा कायधक्रम के तहत एक
सडसजटल पहल, गोवा के उद्यसमयोों और स्वयों सहायता समूहोों (SHG) को सशक्त बनाने के उद्दे श्य से चिथ-ए-
बािार भी लॉन्च सकया।
• गोवा सरकार गृह आर्ार और दयानोंद सामासजक सुरक्षा योजना (DSSS) के तहत 2,000 रुपये और गृह
आर्ार के तहत 1,500 रुपये प्रदान करती है।
i.गोवा के CM प्रमोद सावोंत ने "गोिा टै क्सी ऐप" लॉन्च सकया, एक टै क्सी सेवा ऐप सजसके माध्यम से लोग तटीय राज्य
के भीतर कहीों भी यात्रा करने के सलए ऑनलाइन कैब बुक कर सकते हैं। यह 24/7 बुसकोंग सेवा है।
• "गोवा टै क्सी ऐप" सपछले 6 महीनोों से मनोहर अोंतराध ष्ट्रीय हवाई अड्डे (MIA), MoPA (गोवा) में चालू है।

Report Errors in the PDF - ebooks@affairscloud.com Copyright 2014-2023 @ AffairsCloud.com 53


6. उस राज्य िा िाम बताइए निसिे हाि ही में (नसतंबर'23 में) मनहिाओं िे निए प्रनत माह 1000 रुपये िी
नित्तीय सहायता प्रदाि िरिे िे निए एि मनहिा अनधिार अिुदाि योििा ििैगिार मगनिर उरीमई (थोगई)
नथट्टम शुरू िी है।
1)तेलोंगाना
2)केरल
3)कनाधटक
4)आों ध्र प्रदे श
5)तसमलनाडु
उत्तर-5)तनमििािु
स्पष्टीिरण:
तनमििािु (TN) के मुख्यमोंत्री (CM) MK स्ट्ासलन ने पररवारोों के मुक्कखया के रूप में पहचानी जाने वाली मसहलाओों के
सलए प्रसत माह 1000 रुपये की सवत्तीय सहायता प्रदान करने के सलए एक मनहिा अनधिार अिुदाि योििा, कलैगनार
मगसलर उरीमई (िोगई) सिट्टम की शुरुआत की।
i.यह योजना TN के पूवध CM और द्रसवड मुनेत्र कडगम (DMK) के सोंथिापक C.N.अन्नादु रई(15 ससतोंबर) की जयोंती के
अवसर पर शुरू की गई िी।
ii.यह 2021 के सवर्ानसभा चुनावोों से पहले स्ट्ासलन िारा सदए गए 7 आश्वासनोों में से एक िा।
iii.12,000 रुपये प्रसत वषध का अनुदान मसहलाओों की आजीसवका में सुर्ार, उनके जीवन स्तर को बढाने और उन्ें समाज
में आत्म-सम्मान के साि जीने में सक्षम बनाने के सलए सनर्ाधररत है।

7. निस राज्य सरिार िे हाि ही में (नसतंबर'23 में) श्रम बि िे िल्याण िे निए 'मुख्यमंत्री श्रनमि िल्याण
योििा (MMSKY)' िामि एि िई योििा शुरू िी है ?
1)उत्तर प्रदे श
2)असम
3)मध्य प्रदे श
4)महाराष्ट्र
5)अरुिाचल प्रदे श
उत्तर-5)अरुणाचि प्रदे श
स्पष्टीिरण:
अरुणाचि प्रदे श (AR) के मुख्यमोंत्री (CM), पेमा खाों डू ने राज्य की श्रम शक्कक्त के कल्ाि के सलए 'मुख्यमोंत्री श्रसमक
िल्याण योजना (MMSKY)' नामक एक नई योजना शुरू की।
i.यह योजना भारतीय मजदू र सोंघ (BMS) की राज्य इकाई िारा न्योकुम लापाों ग मैदान, ईटानगर, AR में आयोसजत एक
कायधक्रम के दौरान शुरू की गई िी।
ii.यह कायधक्रम सवश्वकमाध जयोंती के उपलक्ष् में आयोसजत सकया गया िा। BMS 17 ससतोंबर 2023 को सवश्वकमाध जयोंती
की पूवध सोंध्या पर राष्ट्रीय मजदू र सदवस के रूप में मनाया जाता है ।
iii.इसके तहत, अरुिाचल प्रदे श भवन और अन्य सनमाध ि श्रसमक कल्ाि बोडध (APBOCWWB) के साि पोंजीकृत श्रसमकोों
के सलए प्रोत्साहन को सोंशोसर्त सकया गया है।

Report Errors in the PDF - ebooks@affairscloud.com Copyright 2014-2023 @ AffairsCloud.com 54


8. लकस राज्य सरकार ने हाि ही में (लसतंबर’23 में) गभावती मलहिाओं, बच्चों और लकशोररयों की स्वास्थ्य स्कस्थलत
में सुिार के लिए ‘मुख्यमंत्री संपूणा पुलष्ट योिना’और ‘पद पुलष्ट योिना’ शुरू की है?
1)महाराष्ट्र
2)मध्य प्रदे श
3)ओश्चिशा
4)पश्चिम बंगाल
5)झारखण्ड
उत्तर-3)ओलिशा
स्पष्टीकरण:
ओलिशा के मुख्यमंत्री (CM) नवीन पटनायक ने राज्य में गभगवती मश्चहलाओं, बच्चों और श्चकशोररयों की स्वास्थ्य स्थिश्चत में
सुर्ार के श्चलए ‘मुख्यमंत्री संपूणा पुलष्ट योिना’और ‘पद पुलष्ट योिना’नाम से दो नई योिनाएं शुरू कीं।
i.इस योिना का उद्दे श्य ओश्चिशा में श्चकशोर लड़श्चकयों, गभग वती मश्चहलाओं और कुपोश्चषत बच्चों की पोषि स्थिश्चत में सुर्ार
करना है।
ii.ममता योिना (2011 में शुरू) राज्य में गभगवती मश्चहलाओं और नविात श्चशशुओं के अिे स्वास्थ्य को सुश्चनश्चित कर
रही है। मुख्यमंत्री संपूिग पुश्चष्ट् योिना से िरूरतमंदों के बीच पोषि में और सुर्ार होगा।
iii.‘पद पुश्चष्ट् योिना’ के तहत दू रदराि और आश्चदवासी इलाकों के बच्चों को उनके ही गांवों और पररक्षेत्रों में उच्च गुिवत्ता
वाला पका हुआ भोिन श्चदया िाएगा।

9. निस राज्य िे हाि ही में (नसतंबर'23 में) घर बिािे िे निए सहायता प्रदाि िरिे िे निए मुख्यमंत्री ग्रामीण
आिास न्याय योििा शुरू िी है ?
1)झारखोंड
2)छत्तीसगढ
3)महाराष्ट्र
4)मध्य प्रदे श
5)उत्तर प्रदे श
उत्तर- 2)छत्तीसगढ़
स्पष्टीिरण:
छत्तीसगढ़ के मुख्यमोंत्री (CM) भूपेश बघेल ने राज्य भर में योजना के पात्र लाभासिधयोों को घर बनाने के सलए सहायता
प्रदान करने के उद्दे श्य से "मुख्यमंत्री ग्रामीण आिास न्याय योििा" शुरू की है ।
i.इस योजना में वे लाभािी शासमल हैं सजनके पास है प्रर्ानमोंत्री (PM) आवास योजना का लाभ नहीों समला जो सामासजक-
आसिधक सवेक्षि 2011 (आसर्काररक तौर पर सामासजक आसिधक और जासत जनगिना - SECC 2011 के रूप में जाना
जाता है ) पर आर्ाररत है।
ii.यह योजना क्रमशः IAP (पहाडी और दु गधम क्षेत्र) और गैर-IAP (मैदानी क्षेत्र) सजलोों में सवभासजत करके प्रसत पररवार
1,30,000 रुपये और 1,20,000 रुपये की सवत्तीय सहायता प्रदान करती है ।
iii.इसके अलावा, पात्र पररवारोों को स्वच्छ भारत समशन ग्रामीि (SBM-G) और महात्मा गाों र्ी राष्ट्रीय ग्रामीि रोजगार
गारों टी असर्सनयम (MNREGA) के असभसरि के माध्यम से उनकी पात्रता के अनुसार 12,000 रुपये समलेंगे।

Report Errors in the PDF - ebooks@affairscloud.com Copyright 2014-2023 @ AffairsCloud.com 55


VISITS
1. 20वें ASEAN-भारत लशिर सम्मेिन और 18वें पूवी एलशया लशिर सम्मेिन में भाग िेने के लिए प्रिान
मंत्री नरें द्र मोदी की िकाताा यात्रा के संबंि में लनम्नलिस्कित में से कौन सा लबंदु "सही" है /हैं ?
A) 20वां ASEAN-भारत लशिर सम्मेिन िकाताा, इं िोनेलशया में आयोलित लकया गया था और इसकी सह-
अध्यक्षता इं िोनेलशया गणराज्य के राष्टरपलत िोको लविोिो और प्रिान मंत्री नरें द्र मोदी ने 'आलसयान मामिे:
लवकास का केंद्र' लवषय पर की थी।
B)18वें पूवी एलशया लशिर सम्मेिन (EAS) ने क्षेत्र को लवकास के केंद्र के रूप में बनाए रिने और बढावा दे ने
पर EAS नेताओं के वक्तव्य को अपनाया।
C)बैठक के दौरान, भारत ने लििी, लतमोर-िेस्ते में एक भारतीय दू तावास स्थालपत करने की घोषणा की।
1) केवल A
2) केवल A & B
3) केवल B & C
4) केवल A & C
5) सभी A, B & C
उत्तर -5)सभी A, B & C
स्पष्टीकरण:
भारत के प्रर्ान मंत्री (PM) नरें द्र मोदी ने इं िोनेश्चशया गिराज्य के राष्ट्रपश्चत िोको श्चविोिो के श्चनमंत्रि पर 6 से 7 श्चसतंबर
2023 तक िकाताग, इं िोनेश्चशया की आश्चर्काररक यात्रा की।
• 20वां ASEAN-भारत लशिर सम्मेिन िकाताग, इं िोनेश्चशया में आयोश्चित श्चकया गया िा और इसकी सह-
अध्यक्षता इं िोनेश्चशया गिराज्य के राष्ट्रपश्चत िोको श्चविोिो और प्रर्ान मं त्री नरें द्र मोदी ने 'ASEAN मामिे:
लवकास का केंद्र' श्चवषय पर की िी।
• भारतीय पक्ष ने लििी, लतमोर-िेस्ते में एक भारतीय दू तावास थिाश्चपत करने की घोषिा की।
• बैठक के दौरान दो संयुक्त वक्तव्य, एक समुद्री सहयोग पर और दू सरा खाद्य सुरक्षा पर अपनाया गया।
• प्रर्ानमंत्री नरें द्र मोदी ने आश्चसयान-भारत सहयोग को मिबूत करने के श्चलए 12 सूत्री योिना का अनावरि
श्चकया।
• 2022-2023 में, भारत और आश्चसयान के बीच श्चद्वपक्षीय व्यापार 131.5 लबलियन अमेररकी िॉिर तक पहुंच
गया, िो भारत के कुल व्यापार का 11.3% है।
• 18वें पूवी एलशया लशिर सम्मेिन (EAS) ने क्षेत्र को श्चवकास के केंद्र के रूप में बनाए रखने और ब़िावा दे ने
पर EAS नेताओं के वक्तव्य को अपनाया और क्षेत्रीय और अंतरराष्ट्रीय मुद्दों की एक श्चवस्तृत श्रृंखला पर श्चवचारों
का आदान-प्रदान श्चकया।
• EAS रिनीश्चतक बातचीत के श्चलए इं िो-पैश्चसश्चफक का प्रमुख मंच है और इसकी थिापना 2005 में हुई िी।
इसमें अब 18 सदस्य हैं।

2. नसतंबर 2023 में भारत िी राष्टरपनत द्रौपदी मुमूा िी गुिरात यात्रा िे संबंध में निम्ननिखित में से िौि सा/से
नबंदु "सही" है/हैं?
A) राष्टरपनत द्रौपदी मुमूा िे अपिी गुिरात यात्रा िे दौराि 'राष्टरीय ई-निधाि एखप्लिेशि' (NeVA) िा उद् घाटि
निया, निसिा उद्दे श्य राज्य निधािसभा िे िामिाि िो 'निनिटि हाउस' में बदििर िागि रनहत बिािा
है।

Report Errors in the PDF - ebooks@affairscloud.com Copyright 2014-2023 @ AffairsCloud.com 56


B) राष्टरपनत द्रौपदी मुमूा िे गुिरात िे गांधीिगर में रािभिि से िस्तुतः आयुष्माि भि अनभयाि िे साथ-साथ
आयुष्माि भि पोटा ि भी िॉन्च निया।
C) राष्टरपनत द्रौपदी मुमूा िे िई नदल्ली, नदल्ली में भारतीय िृनष अिुसंधाि पररषद (ICAR) िन्वेंशि सेंटर, राष्टरीय
िृनष निज्ञाि िेंद्र में आयोनित एि िायाक्रम में निसािों िे अनधिारों पर पहिे िैनश्वि संगोष्ठी (GSFR) िा
उद् घाटि निया।
1)केवल A
2)केवल A & B
3)केवल B & C
4)केवल A & C
5)सभी A, B & C
उत्तर -5)सभी A, B & C
स्पष्टीिरण:
भारत की राष्ट्रपसत द्रौपदी मुमूध 12 से 13 ससतोंबर, 2023 तक गुजरात की दो सदवसीय यात्रा पर िीों।
i.राष्ट्रपसत द्रौपदी मुमूा ने अपनी 2 सदवसीय गुजरात यात्रा पर, 'राष्टरीय ई-निधाि एखप्लिेशि' (NeVA) का उद् घाटन
सकया, सजसका उद्दे श्य राज्य सवर्ानसभा के कामकाज को 'सडसजटल हाउस' में बदलकर कागज रसहत बनाना है।
ii.13 ससतोंबर 2023 को, राष्ट्रपसत द्रौपदी मुमूध ने गुजरात के गाों र्ीनगर में राजभवन से वस्तुतः आयुष्मान भव असभयान के
साि-साि आयुष्माि भि पोटा ि लॉन्च सकया।
• आयुष्मान भव असभयान और पोटध ल का शुभारों भ सावधभौसमक स्वास्थ्य कवरे ज (UHC) प्राप्त करने और सभी के
सलए स्वास्थ्य सेवा सुसनसित करने की सदशा में एक महत्वपूिध कदम है।
iii.12 ससतोंबर 2023 को, राष्ट्रपसत द्रौपदी मुमूध ने भारतीय कृसष अनुसोंर्ान पररषद (ICAR) कन्वेंशन सेंटर, राष्ट्रीय कृसष
सवज्ञान केंद्र, नई सदल्ली, सदल्ली में आयोसजत एक कायधक्रम में सकसानोों के असर्कारोों पर पहले वैसश्वक सोंगोष्ठी (GSFR)
का उद् घाटन सकया।

3. नसतंबर 2023 में सऊदी अरब िे प्रधाि मंत्री (PM) नहि रॉयि हाइिेस (HRH) नप्रंस मोहम्मद नबि सिमाि
नबि अब्दु िअज़ीज़ अि-सऊद िी भारत यात्रा िे संबंध में निम्ननिखित में से िौि सा/से नबंदु "सही" है /हैं?
A) सऊदी अरब िे प्रधाि मंत्री (PM) और भारत िे प्रधाि मंत्री िरें द्र मोदी िे भारत - सऊदी रणिीनति
साझेदारी पररषद (SPC) िी पहिी बैठि िी सह-अध्यक्षता िी।
B) भारतीय ििीि और ििीिरणीय ऊिाा मंत्रािय (MNRE) और सऊदी ऊिाा मंत्रािय िे अपिी ऊिाा
साझेदारी िो मिबूत िरिे , भारत िे स्थायी ऊिाा में पररिताि और िैनश्वि िििायु पररिताि शमि प्रयासों
िा समथाि िरिे िे निए एि MoU पर हस्ताक्षर निए हैं।
C) तििीिी क्षेत्र में सहयोग और SME बैंि िी स्थापिा िे निए भारतीय ररििा बैंि (RBI) और सऊदी अरब
िे िघु और मध्यम उद्यम (SME ) बैंि िे बीच एि MoU पर हस्ताक्षर निए गए।
1)केवल A
2)केवल A & B
3)केवल B & C
4)केवल A & C
5)सभी A, B & C
उत्तर- 2)िेिि A & B

Report Errors in the PDF - ebooks@affairscloud.com Copyright 2014-2023 @ AffairsCloud.com 57


स्पष्टीिरण:
9-11 ससतोंबर, 2023 तक, सऊदी अरब साम्राज्य के क्राउन सप्रोंस और प्रर्ान मोंत्री (PM) सहज रॉयल हाइनेस (HRH)
सप्रोंस मोहम्मद सबन सलमान सबन अब्दु लअज़ीज़ अल सऊद ने भारत के प्रर्ान मोंत्री (PM) नरें द्र मोदी के सनमोंत्रि पर
भारत की राजकीय यात्रा की। यह सप्रोंस की भारत की दू सरी राजकीय यात्रा है , महामसहम ने इससे पहले फरवरी 2019
में राजकीय यात्रा पर भारत का दौरा सकया िा।
i.क्राउन सप्रोंस और भारत के PM ने भारत-सऊदी रिनीसतक साझेदारी पररषद (SPC) की पहली बैठक की सह-
अध्यक्षता की, सजसे अरू बर 2019 में PM नरें द्र मोदी की ररयाद, सऊदी अरब की राजकीय यात्रा के दौरान दोनोों
नेताओों िारा एक समझौते पर हस्ताक्षर करने के साि थिासपत सकया गया िा।
• इस SPC ने यूनाइटे ड सकोंगडम (UK), फ्राोंस और चीन के बाद भारत को चौिा दे श बना सदया सजसके साि सऊदी
अरब ने ऐसी साझेदारी की।
ii.भारतीय नवीन और नवीकरिीय ऊजाध मोंत्रालय (MNRE) और सऊदी ऊिाा मंत्रािय ने अपनी ऊजाध साझेदारी को
मजबूत करने, भारत के थिायी ऊजाध में पररवतधन और वैसश्वक जलवायु पररवतधन शमन प्रयासोों का समिधन करने के सलए
एक MoU पर हस्ताक्षर सकए हैं।
iii.तकनीकी क्षेत्र में सहयोग और SME बैंक की थिापना के सलए भारतीय लघु उद्योग सवकास बैंक (SIDBI) और
सऊदी अरब के लघु और मध्यम उद्यम (SME) बैंि के बीच एक MoU पर हस्ताक्षर सकए गए।

4. नसतंबर 2023 में प्रधाि मंत्री (PM) िरें द्र मोदी िी मध्य प्रदे श और छत्तीसगढ़ यात्रा िे संबंध में निम्ननिखित
में से िौि सा/से नबंदु "सही" है/हैं ?
A)मध्य प्रदे श िी यात्रा िे दौराि, PM िे बीिा में 50,700 िरोड रुपये से अनधि िी पररयोििाओं िी
आधारनशिा रिी, निसमें पेटरोिेनमिि िॉम्प्लेक्स शानमि है।
B) PM िे MP िे िमादापुरम नििे में 'नबििी और ििीिरणीय ऊिाा निनिमााण क्षेत्र' िामि दस
पररयोििाओं िी आधारनशिा भी रिी।
C) रायगढ़, छत्तीसगढ़ िी अपिी यात्रा िे दौराि, उन्ोंिे छत्तीसगढ़ िे िौ नििों में 'नक्रनटिि िेयर ब्लॉि' िी
आधारनशिा रिी और एि िाि नसिि सेि परामशा िािा भी नितररत निए।
1)सभी A, B & C
2)केवल A & B
3)केवल B & C
4)केवल A & C
5)केवल A
उत्तर- 1)सभी A, B & C
स्पष्टीिरण:
14 ससतोंबर, 2023 को भारत के प्रर्ान मोंत्री (PM) िरें द्र मोदी ने मध्य प्रदे श (MP) के बीना और छत्तीसगढ के रायगढ
का दौरा सकया, सजसका सववरि इस प्रकार है :
• PM ने MP के बीना में 50,700 िरोड रुपये से असर्क की पररयोजनाओों का नशिान्यास सकया।
• पररयोजनाओों में बीना में पेटरोिेनमिि िॉम्प्लेक्स, नमधदापुरम में एक सबजली और नवीकरिीय ऊजाध
सवसनमाधि क्षेत्र, इों दौर में IT (सूचना प्रौद्योसगकी) पाकध, रतलाम में मेगा औद्योसगक पाकध; और MP भर में 6 नए
औद्योसगक क्षेत्र शासमल हैं।
• भारत पेटरोसलयम कॉपोरे शन सलसमटे ड (BPCL) िारा बीना ररफाइनरी में पेटरोकेसमकल कॉम्प्लेक्स की
आर्ारसशला रखी गई है। कॉम्प्लेक्स को ~49,000 करोड रुपये की लागत से सवकससत सकया जाएगा।

Report Errors in the PDF - ebooks@affairscloud.com Copyright 2014-2023 @ AffairsCloud.com 58


• PM ने MP के नमधदापुरम सजले में 'सबजली और नवीकरिीय ऊजाध सवसनमाधि क्षेत्र' नामक दस पररयोजनाओों की
आर्ारसशला भी रखी।
• सबजली और नवीकरिीय ऊजाध सवसनमाधि क्षेत्र, सजसे 460 करोड रुपये से असर्क के पररव्यय के साि सवकससत
सकया जाएगा, आसिधक सवकास और नौकररयोों को बढावा दे गा।
• रायगढ, छत्तीसगढ की अपनी यात्रा के दौरान, उन्ोोंने महत्वपूिध रे ल क्षेत्र की पररयोजनाओों को समसपधत सकया,
और छत्तीसगढ के नौ सजलोों में 'सक्रसटकल केयर िॉक' की आर्ारसशला रखी और एक लाख ससकल सेल
परामशध काडध भी सवतररत सकए।
• उन्ोोंने रायगढ में सावधजसनक कायधक्रम में लगभग 6,350 करोड रुपये की रे ल क्षेत्र की पररयोजनाएों समसपधत
कीों, जो क्षेत्र में यासत्रयोों की आवाजाही के साि-साि माल ढु लाई को सुसवर्ाजनक बनाकर सामासजक आसिधक
सवकास को गसत प्रदान करें गी।
• छत्तीसगढ पूवध रे ल पररयोजना चरि- I मल्टी-मोडल कनेक्करसवटी के सलए PM गसतशक्कक्त का एक सहस्सा है ,
सजसमें गारे -पेलमा स्पर के साि खरससया से र्रमजयगढ तक 124.8 सकलोमीटर रे ल लाइन और छाल, बरौद,
दु गाधपुर और अन्य कोयला खदानोों को जोडने वाली 3 फीडर लाइनें शासमल हैं।

BANKING AND FINANCE


1. निस िंपिी िे हाि ही में (अगस्त’23 में) AI-आधाररत अिुसंधाि प्रणािी द्वारा संचानित निश्व िा पहिा पूणा
स्वचानित फंि, एि पोटा फोनियो मैिेिमेंट सनिास (PMS) िॉन्च निया है ?
1)चाल्सध श्वाब कॉपोरे शन
2)कैसपटल वाया
3)सनष्ठा
4)मोहरा
5)सावटध
उत्तर-5)सािटा
स्पष्टीिरण:
भारत की अग्रिी सनवेश सलाहकार फमों में से एक, सािटा ने हैदराबाद, तेलोंगाना में दु निया िा पहिा पूरी तरह से
स्वचानित फंि पोटध फोसलयो मैनेजमेंट ससवधस (PMS) 'द एि एस्ट्र ा फंि' लॉन्च सकया. यह सनवेश सनिधय लेने के सलए
आसटध सफसशयल इों टेसलजेंस (AI) अनुसोंर्ान प्रिाली एडवाोंथड प्रोसेस ऑटोमेशन एों ड ररसचध टे क्नोलॉजी (APART) का
उपयोग करता है।
i.फोंड ने सनवेश पर असर्क ररटनध प्राप्त करने की सोंभावनाओों को बढाया।
ii.फोंड को पहले सदन लगभग 31 करोड रुपये का सनवेश प्राप्त हुआ और FY24 के अोंत तक 350 करोड रुपये के प्रबोंर्न
के तहत सोंपसत्त (AUM) हाससल करने की योजना है ।

2. निस बैंि िे हाि ही में (अगस्त'23 में) GST चािाि िा उपयोग िरिे सूक्ष्म, िघु और मध्यम उद्यमों (MSME)
िो निनिटि क्रेनिट प्रिाह िी सुनिधा िे निए एि मोबाइि एखप्लिेशि, गुि्स एं ि सनिासेि टै क्स (GST) सहाय
ऐप िॉन्च निया है ?
1)इों सडयन बैंक
2)पोंजाब नेशनल बैंक
3)बैंक ऑफ इों सडया

Report Errors in the PDF - ebooks@affairscloud.com Copyright 2014-2023 @ AffairsCloud.com 59


4)स्ट्े ट बैंक ऑफ इों सडया
5)बैंक ऑफ बडौदा
उत्तर-2)पंिाब िेशिि बैंि
स्पष्टीिरण:
सावधजसनक क्षेत्र के बैंक, पंिाब िेशिि बैंि (PNB) ने िस्तु एिं सेिा िर (GST) सहाय योििा पर आर्ाररत एक
मोबाइल एक्कप्लकेशन, PNB GST सहाय ऐप लॉन्च सकया। यह एक एों ड-टू -एों ड सडसजटल उत्पाद है सजसके तहत GST
चालान का उपयोग करके ऋि सदया जा सकता है।
• इस पहुोंच के साि, PNB GST चालान का उपयोग करके सूक्ष्म, लघु और मध्यम उद्यमोों (MSME) को सडसजटल
क्रेसडट प्रवाह की सुसवर्ा दे ने वाला पहला सावधजसनक क्षेत्र बैंक (PSB) बन गया है ।
• यह पहल भारत में MSME क्षेत्र और सडसजटल क्रेसडट पाररक्कथिसतकी तोंत्र के सवकास को आगे बढाने की बैंक की
रिनीसत के अनुरूप भी है ।
i.ऐप MSME उर्ारकताधओों के सलए सकसी भी मैन्युअल हस्तक्षेप को खत्म करने में मदद करने के सलए एों ड-टू -एों ड
सडसजटल ऋि प्रसक्रया को सक्षम बनाता है और प्रसक्रया को लागत प्रभावी और सुचारू बनाता है।
ii.यह सेवा ऋि रासश को सीर्े बैंक में रखे गए उर्ारकताध के चालू खाते में जमा कर दे गी।
iii.PNB GST सहाय ऐप आवेदक/उर्ारकताध और बैंक के बीच की दू री को पाट दे गा।

3. ओपि-सोसा हाइपरिेिर फैनिि निप्लॉयमेंट हेल्पर िा िाम बताइए निसे हाि ही में (अगस्त’23 में) िेशिि
पेमेंट्स िॉरपोरे शि ऑफ इं निया (NPCI) द्वारा िुबेरिेट्स िस्ट्सा पर हाइपरिेिर फैनिि आधाररत ब्लॉिचेि
िेटििा िी तैिाती और प्रबंधि िो सुव्यिखस्थत िरिे िे निए िॉन्च निया गया था।
1)फाल्कन
2)इरोहा
3)टर ॉन
4)कॉडाध
5)सॉटू ि
उत्तर- 1)फाल्कि
स्पष्टीिरण:
नेशनल पेमेंट्स कॉरपोरे शन ऑफ इों सडया (NPCI) ने कुबेरनेट क्लस्ट्सध पर हाइपरलेजर फैसब्रक आर्ाररत िॉकचेन
नेटवकध की तैनाती और प्रबोंर्न को सुव्यवक्कथित करने के सलए एक ओपन-सोसध हाइपरलेजर फैसब्रक सडप्लॉयमेंट हेल्पर
'फाल्कि' लॉन्च सकया है।
• 'फाल्कन' को पेश करने का प्रािसमक लक्ष् कुबेरनेट्स वातावरि के भीतर हाइपरलेजर फैसब्रक नेटवकध के
सवसभन्न घटकोों को थिासपत करने, कॉक्कन्क्फ़गर करने और प्रबोंसर्त करने की जसटल प्रसक्रया को सरल बनाना है।
• इन घटकोों में फैसब्रक नोड् स, पीयसध, ऑडध रसध और चैनल शासमल हैं।
फाल्कि िे बारे में:
i.फाल्कन जनरल पक्किक लाइसेंस (GPL) लाइसेंस (GPL-3.0) के तहत बनाया गया एक ओपन-सोसध प्रोजेर है।
ii.फाल्कन से िॉकचेन डे वलपसध को नेटवकध के साि-साि वेब3 समार्ानोों की कुशल, सवश्वसनीय और स्वचासलत तैनाती
की सुसवर्ा के सलए सवतररत लेजर तकनीक का उपयोग करने में मदद करने की उम्मीद है।
िोट: NPCI ने 2020 में भुगतान को तेज और सुरसक्षत बनाने के सलए िॉकचेन तकनीक आर्ाररत "वज्र प्लेटफॉमध" लॉन्च
सकया है।

Report Errors in the PDF - ebooks@affairscloud.com Copyright 2014-2023 @ AffairsCloud.com 60


4. उस िंपिी िा िाम बताइए निसिे हाि ही में (अगस्त'23 में) एि िए टर े निं ग ऐप शेयर.मािेट िे साथ
ऑििाइि स्ट्ॉि िोनिंग व्यिसाय में प्रिेश निया है।
1) PayU
2)फ्रीचाजध
3)रे जरपे
4)भारतपे
5)फोनपे
उत्तर- 5)फोिपे
स्पष्टीिरण:
30 अगस्त 2023 को, भारतीय सडसजटल भुगतान प्लेटफॉमध फोिपे ने अपनी सहायक कोंपनी फोनपे वेल्थ ब्रोसकोंग के
तहत एक नए टर े सडों ग ऐप, शेयर.मािेट के साि ऑनलाइन स्ट्ॉक ब्रोसकोंग व्यवसाय में प्रवेश सकया।
i.ऐप का उपयोग स्ट्ॉक, म्यूचुअल फोंड और एक्सचेंज-टर े डेड फोंड या एक्सचेंज-टर े डेड फोंड (ETF) में सनवेश करने के
सलए सकया जा सकता है।
ii.फोनपे के अनुसार, शेयर.माकेट असर्क ररयायती ब्रोकररों ग मूल् (24 सेंट या 0.05%, जो भी कम हो), और व्यापक
बाजार बुक्कद्धमत्ता और वेल्थबास्केट् स नामक एक मात्रात्मक अनुसोंर्ान-आर्ाररत तकनीकी मोंच प्रदान करता है , जो
उपयोगकताधओों के सलए मोबाइल ऐप और वेब दोनोों के रूप में उपलब्ध होगा।

5. निस िाइफ इं श्योरें स िंपिी िे हाि ही में (अगस्त’23 में) एि सुरक्षा और बचत-उन्मुि िाइफ इं श्योरें स
योििा शुरू िी है , निसमें मिी बैि बेनिनफट और गारं टीि इििम (GI) से संबंनधत नििल्प शानमि हैं ?
1)मैक्स लाइफ इों श्योरें स
2)बजाज आसलयाोंज लाइफ इों श्योरें स
3)ICICI प्रूडेंसशयल लाइफ इों श्योरें स
4)SBI लाइफ इों श्योरें स
5)टाटा AIA लाइफ इों श्योरें स
उत्तर-3)ICICI प्रूिेंनशयि िाइफ इं श्योरें स
स्पष्टीिरण:
ICICI प्रूडेंसशयल लाइफ इों श्योरें स कोंपनी सलसमटे ड (ICICI प्रूिेंनशयि िाइफ) ने "ICICI प्रू GIFT प्रो" लॉन्च सकया है
जो एक सुरक्षा और बचत-उन्मुख लाइफ इों श्योरें स योजना है। नई योजना पॉसलसीर्ारकोों को, सजन्ें "लाइफ एसोडध " भी
कहा जाता है , अपनी पॉसलसी को अनुकूसलत करने के सलए सवकल्पोों की एक सवस्तृत श्रृोंखला प्रदान करती है ।
i.यह पॉसलसीर्ारक को कई प्रकार की लचीलेपन की पेशकश करता है , सजसमें मनी बैक बेसनसफट और गारों टीड
इनकम (GI) से सोंबोंसर्त सवकल्प शासमल हैं।

6. उस िंपिी िा िाम बताइए निसिे हाि ही में (अगस्त’23 में) गेस्ट् चेि-आउट िे निए उन्नत ऑििाइि
भुगताि सुरक्षा िे निए ALT ID (िैिखल्पि पहचाििताा) समाधाि िॉन्च निया है।
1)वीज़ा
2)अमेररकन एक्सप्रेस
3)कैसपटल वन
4)मास्ट्रकाडध
5)टे नसेंट

Report Errors in the PDF - ebooks@affairscloud.com Copyright 2014-2023 @ AffairsCloud.com 61


उत्तर- 4)मास्ट्रिािा
स्पष्टीिरण:
मास्ट्रिािा ने गेस्ट् चेि-आउट प्रनक्रयाओं के सलए उन्नत ऑनलाइन भुगतान सुरक्षा के सलए ALT ID (वैकक्कल्पक
पहचानकताध ) समाधाि लॉन्च सकया है क्ोोंसक यह लेनदे न के दौरान सोंवेदनशील काडध जानकारी के जोक्कखम को रोक
दे गा।
i.ALT ID समार्ान गेस्ट् चेक-आउट प्रसक्रयाओों के दौरान उपयोगकताधओों िारा प्रदान सकए गए वास्तसवक काडध नोंबरोों या
काडध र्ारकोों के सलए एक वैकक्कल्पक पहचानकताध बनाता है जो अपने काडध को सहे जे सबना लेनदे न करते हैं।
ii.इससलए, यह अनसर्कृत पहुोंच या डे टा उल्लोंघनोों की क्कथिसत में सोंवेदनशील काडध डे टा को उजागर करने की भेद्यता को
कम करता है।
iii.ALT ID समार्ान काडध र्ारकोों को कई लाभ प्रदान करे गा, जैसे व्यापारी वेबसाइटोों पर काडध नोंबरोों का भोंडारि न
होना और सोंभासवत डे टा उल्लोंघनोों से सुरक्षा।

7. निस िंपिी िो हाि ही में (नसतंबर’23 में) गैर-बैंनिंग नित्त िंपिी (NBFC) िा व्यिसाय शुरू िरिे िे
निए भारतीय ररििा बैंि (RBI) से पंिीिरण प्रमाणपत्र प्राप्त हुआ है ?
1)सहन्क्दुजा लीलैंड फाइनेंस सलसमटे ड
2)क्कक्लक्स फाइनेंस इों सडया प्राइवेट सलसमटे ड
3)ठाकुर सफनइन्वेस्ट् प्राइवेट सलसमटे ड
4)PTC इों सडया फाइनेंसशयल ससवधसेज सलसमटे ड
5)बजाज ऑटो कोंज्यूमर फाइनेंस सलसमटे ड
उत्तर-5)बिाि ऑटो िंज्यूमर फाइिेंस निनमटे ि
स्पष्टीिरण:
बजाज ऑटो सलसमटे ड की पूिध स्वासमत्व वाली सहायक कोंपनी बिाि ऑटो िंज्यूमर फाइिेंस निनमटे ि (BACFL)
को गैर-बैंसकोंग सवत्त कोंपनी (NBFC) का व्यवसाय शुरू करने के सलए भारतीय ररजवध बैंक (RBI) से पोंजीकरि
प्रमािपत्र प्राप्त हुआ है।
i.सावधजसनक जमा स्वीकार सकए सबना NBFC का व्यवसाय शुरू करने/चलाने के सलए भारतीय ररजवध बैंक असर्सनयम,
1934 की र्ारा 45-IA के तहत पोंजीकरि का प्रमािन 31 अगस्त 2023 को जारी सकया गया िा।

8. नसतंबर 2023 ति िमाचारी भनिष्य निनध संगठि (EPFO) िे संबंध में निम्ननिखित में से िौि सा/से नबंदु
"सही" है/हैं?
A) सरिार िे िमाचारी भनिष्य निनध संगठि (EPFO) िो इखिटी और संबंनधत पररसंपनत्तयों में नििेश िी
नबक्री या मोचि से प्राप्त आय िा नििेश िरिे िी अिुमनत दी है।
B) EPFO इखिटी और संबंनधत उपिरणों में न्यूितम 10% और अनधितम 25% नििेश िर सिता है।
C) EPFO बॉम्बे स्ट्ॉि एक्सचेंि (BSE) या िेशिि स्ट्ॉि एक्सचेंि (NSE) में सूचीबि िंपनियों िे शेयरों में
5000 िरोड रुपये या उससे अनधि िे बािार पूंिीिरण िे साथ नििेश िर सिता है।
1)केवल A
2)केवल A & B
3)केवल B & C
4)केवल A & C
5)सभी A, B & C

Report Errors in the PDF - ebooks@affairscloud.com Copyright 2014-2023 @ AffairsCloud.com 62


उत्तर- 4)िेिि A & C
स्पष्टीिरण:
सरकार ने कमधचारी भसवष्य सनसर् सोंगठन (EPFO) को इक्कक्वटी और सोंबोंसर्त पररसोंपसत्तयोों में सनवेश की सबक्री या मोचन
से प्राप्त आय का सनवेश करने की अनुमसत दी है। सपछली दो असर्सूचनाओों के नीचे सोंशोर्न करने के सलए 24 अगस्त,
2023 को एक नया खोंड जोडा गया िा।
• असर्सूचना में सक्कम्मलन (सदनाोंक 1 ससतोंबर, 2023): अनुमत कायों की सूची में "इक्कक्वटी और सोंबोंसर्त सनवेशोों में
सनवेश की सबक्री या मोचन" जोडा गया।
• EPFO इक्कक्वटी और सोंबोंसर्त उपकरिोों में न्यूितम 5% और अनधितम 15% सनवेश कर सकता है ।
• EPFO बॉम्बे स्ट्ॉक एक्सचेंज (BSE) या नेशनल स्ट्ॉक एक्सचेंज (NSE) में सूचीबद्ध कोंपसनयोों के शेयरोों में
5000 िरोड रुपये या उससे अनधि के बाजार पूोंजीकरि के साि सनवेश कर सकता है।
• BSE या NSE सूचीबद्ध इक्कक्वटी में न्यूनतम 65% सनवेश के साि म्यूचुअल फोंड में भी सनवेश सकया जा सकता है ।
EPF0 के वतधमान असनवायध मौजूदा सनवेश पैटनध की सूची।

9. निस िंपिी िे हाि ही में (नसतंबर’23 में) ग्रामीण भारत में 1.5 िाि व्यिसाय संिाददाता (BC) िो िुशि
बिािे िे निए 'भारत में स्केनिंग एिेंट व्यिहायाता और गुणित्ता' पररयोििा शुरू िरिे िे निए ग्रामीण
फाउं िेशि फॉर सोशि इम्पैर (GFSI) िे साथ सहयोग निया है ?
1)फोनपे
2)पेटीएम
3)PayNearby
4)पॉसलसीबाजार
5)मोसबक्कक्वक
उत्तर-3)PayNearby
स्पष्टीिरण:
PayNearby ने ग्रामीि फाउों डेशन फॉर सोशल इम्पैर (GFSI) के सहयोग से 1.5 िाि व्यिसाय संिाददाता (BC)
को कुशल बनाने और ग्रामीि भारत में आवश्यक कौशल, ज्ञान और समिधन के माध्यम से 15,000 मसहला BC को
सूचीबद्ध करने के सलए 'भारत में स्केसलोंग एजेंट व्यवहायधता और गुिवत्ता' पररयोजना शुरू की।
• यह पररयोजना सबल एों ड मेसलोंडा गेट्स फाउों डेशन (BMGF) िारा समसिधत है।
• पररयोजना का कायधकाल दो वषध है।
• PayNearby के सडस्ट्र ीब्यूशन-एस-ए-ससवधस नेटवकध (DaaS) का उपयोग ग्रामीि फाउों डेशन िारा वासिज्य,
सशक्षा, मनोरों जन, क्रेसडट प्रबोंर्न, बीमा और अन्य जैसे सवसभन्न डोमेन में अनुरूप सडसजटल और सवत्तीय समार्ान
पेश करने के सलए सकया जाएगा।

10. उस बैंि िा िाम बताइए निसिे हाि ही में (नसतंबर’23 में) मोबाइि-फस्ट्ा VISA नसग्नेचर मेटि सह-
िांिेि क्रेनिट िािा िॉन्च िरिे िे निए OneCard िे साथ साझेदारी िी है।
1)पोंजाब नेशनल बैंक
2)इों सडयन बैंक
3)केनरा बैंक
4)बैंक ऑफ बडौदा
5)बैंक ऑफ इों सडया
उत्तर- 2)इं नियि बैंि

Report Errors in the PDF - ebooks@affairscloud.com Copyright 2014-2023 @ AffairsCloud.com 63


स्पष्टीिरण:
OneCard के साि साझेदारी में इं नियि बैंि ने सामासजक दू री के मद्दे नजर सोंभासवत उपयोगकताधओों तक आसान
पहुोंच प्रदान करने के सलए अत्यार्ुसनक, मोबाइल-फस्ट्ध , सोंपकध रसहत, VISA नसग्नेचर मेटि सह-िांिेि क्रेनिट िािा
पेश सकए।
• यह पहल वास्तसवक समय में लेनदे न पर नज़र रखने , खचों को प्रबोंसर्त करने, खरीदारी को EMI में पररवसतधत
करने, पुरस्कार भुनाने, पुनभुधगतान करने, माससक बजट की योजना बनाने , क्रेसडट सीमा को समायोसजत करने
और सनबाधर् भुगतान की सुसवर्ा प्रदान करने जैसी सुसवर्ाएों एक मोबाइल एक्कप्लकेशन के भीतर लाती है ।
स्ट्े ट बैंक ऑफ मॉरीशस (SBM) बैंक, साउि इों सडयन बैंक, कैिोसलक सीररयन बैंक (CSB), फेडरल बैंक, बैंक ऑफ
बडौदा फाइनेंसशयल कुछ अन्य OneCard सह-ब्राोंडेड काडध जारीकताध हैं ।
ii.OneCard एक सह-ब्राोंडेड क्रेसडट काडध है जो भारतीय ररजवध बैंक (RBI) िारा अनुमोसदत बैंकोों और सवत्तीय सोंथिानोों
िारा OneCard के साि साझेदारी में जारी सकया गया है।
• OneCard को FPL टे क्नोलॉजीज िारा एक पूिध-स्ट्ै क स्वासमत्व प्रौद्योसगकी प्लेटफ़ॉमध पर लॉन्च और प्रबोंसर्त सकया
गया है।

11. उस िंपिी िा िाम बताइए निसिे हाि ही में (नसतंबर'23 में) व्यापाररयों िो मोबाइि और िािा दोिों से
भुगताि स्वीिार िरिे में सक्षम बिािे िे निए उद्योग िा पहिा साउं िबॉक्स िॉन्च निया है।
1)भारतपे
2)फोनपे
3)पेटीएम
4)रे जरपे
5)फ्रीचाजध
उत्तर-3)पेटीएम
स्पष्टीिरण:
पेटीएम (वन97 कम्युसनकेशोंस सलसमटे ड) ने उद्योग का पहला पेटीएम काडध साउों डबॉक्स लॉन्च सकया, जो व्यापाररयोों को
'टै प एं ि पे' सुसवर्ा के साि साउं िबॉक्स के माध्यम से सभी वीज़ा, मास्ट्रकाडध , एमेक्स और रुपे नेटवकध पर मोबाइि
और िािा दोनोों से भुगताि स्वीकार करने में सक्षम बनाता है।
• साउों डबॉक्स व्यापारी और ग्राहक को सलक्कक्वड सक्रस्ट्ल सडथप्ले (LCD) स्क्रीन के माध्यम से ऑसडयो और
सवज़ुअल भुगतान पुसष्ट्करि प्रदान करता है।
• टै प एों ड पे: पेटीएम काडध साउों डबॉक्स में अोंतसनधसहत 'टै प एों ड पे' कायधक्षमता शासमल है , जो व्यापाररयोों को
5,000 रुपये तक के काडध भुगतान स्वीकार करने की अनुमसत दे ती है।
• कनेक्करसवटी: यह सडवाइस तेज भुगतान अलटध के सलए तेज चौिी पीढी (4G) नेटवकध कनेक्करसवटी का दावा
करता है।

12. नसतंबर 2023 में भारतीय ररज़िा बैंि (RBI) द्वारा उठाए गए िदम िे संबंध में निम्ननिखित में से िौि सा/से
नबंदु "सही" है/हैं?
A) RBI पररपत्र निसिा शीषाि 'UPI िे माध्यम से बैंिों में पूिा-स्वीिृत क्रेनिट िाइिों िा संचािि' है , िे
यूनिफाइि पेमेंट्स इं टरफेस (UPI) में फंनिं ग िातों िे रूप में पूिा-स्वीिृत क्रेनिट िाइिों िो शानमि िरिे िी
घोषणा िी।

Report Errors in the PDF - ebooks@affairscloud.com Copyright 2014-2023 @ AffairsCloud.com 64


B) पररपत्र भुगताि बैंिों, िघु नित्त बैंिों और क्षेत्रीय ग्रामीण बैंिों िो छोडिर, अिुसूनचत िानणखज्यि बैंिों पर
िागू होता है।
C) बैंि अपिी बोिा -अिुमोनदत िीनतयों िे अिुसार इि क्रेनिट िाइिों िे उपयोग िे निए क्रेनिट सीमा, क्रेनिट
अिनध, ब्याि दरों सनहत नियम और शतें स्थानपत िर सिते हैं।
1)केवल A
2)केवल A & B
3)केवल B & C
4)केवल A & C
5)सभी A, B & C
उत्तर -5)सभी A, B & C
स्पष्टीिरण:
भारतीय ररजवध बैंक (RBI) के पररपत्र सजसका शीषधक 'UPI के माध्यम से बैंकोों में पूवध-स्वीकृत क्रेसडट लाइनोों का सोंचालन'
है, ने यूसनफाइड पेमेंट्स इों टरफेस (UPI) में फोंसडों ग खातोों के रूप में पूवध-स्वीकृत क्रेसडट लाइनोों को शासमल करने की
घोषिा की।
• यह सनिधय क्रेसडट लाइनोों को फोंसडों ग खाते के रूप में शासमल करके UPI के दायरे को व्यापक बनाता है।
• UPI में क्रेसडट लाइन एक पूवध-अनुमोसदत क्रेसडट रासश को सोंदसभधत करती है जो एक बैंक अपने ग्राहकोों को
प्रदान करता है। यह क्रेसडट लाइन ग्राहकोों को लेनदे न या भुगतान करने की अनुमसत दे ती है , भले ही उस समय
उनके बैंक खाते में पयाध प्त र्नरासश न हो।
i.यह पररपत्र भुगतान बैंकोों, लघु सवत्त बैंकोों और क्षेत्रीय ग्रामीि बैंकोों को छोडकर, अिुसूनचत िानणखज्यि बैंिों पर
लागू होता है।
ii.यह सनदे श भुगतान और सनपटान प्रिाली असर्सनयम, 2007 (2007 िा अनधनियम 51)की र्ारा 18 के साि पसठत
धारा 10(2)के तहत जारी सकया गया है।
iii.बैंक अपनी बोडध -अनुमोसदत नीसतयोों के अनुसार इन क्रेसडट लाइनोों के उपयोग के सलए क्रेसडट सीमा, क्रेसडट अवसर्,
ब्याज दरोों ससहत सनयम और शतें थिासपत कर सकते हैं ।

13. उस बैंि िा िाम बताइए निसिे हाि ही में (नसतंबर'23 में) ििीि फ्रंट-एं ि बैंनिंग अिुप्रयोगों िी तैिाती
िी सुनिधा िे निए एि सुरनक्षत और अिुिूििीय िंप्यूनटं ग बुनियादी ढांचा प्रदाि िरिे िे निए इं टरिेशिि
नबििेस मशीि िॉरपोरे शि (IBM) िे साथ सहयोग निया है।
1)पोंजाब नेशनल बैंक
2)इों सडयन बैंक
3)बैंक ऑफ बडौदा
4)यूसनयन बैंक ऑफ इों सडया
5)केनरा बैंक
उत्तर- 2)इं नियि बैंि
स्पष्टीिरण:
इं नियि बैंि सलसमटे ड, एक सावधजसनक क्षेत्र का बैंक (PSB), नवीन फ्रोंट-एों ड बैंसकोंग अनुप्रयोगोों की तैनाती की सुसवर्ा के
सलए एक सुरसक्षत और अनुकूलनीय कोंप्यूसटों ग बुसनयादी ढाोंचा प्रदान करने के सलए IBM (इों टरनेशनल सबजनेस मशीर्न्
कॉपोरे शन) के साि सहयोग करता है।

Report Errors in the PDF - ebooks@affairscloud.com Copyright 2014-2023 @ AffairsCloud.com 65


i.यह सहयोग भारतीय बैंक के कोर बैंसकोंग सॉल्ूशन (CBS) वकधलोड को IBM सवधर पर चलाने का मागध प्रशस्त करता
है।
ii.यह सहयोग इों सडयन बैंक की पहल प्रोजेर PARADISE (सवतररत पयाधवरि में सोंसार्न एकत्रीकरि िारा प्रदशधन
सोंवर्धन) का एक सहस्सा है।

14. उस िाइफ इं श्योरें स िंपिी िा िाम बताइए निसिे हाि ही में (नसतंबर'23 में) 'िाइफ ACE' योििा शुरू
िी है , िो एि िॉि-निंक्ि, पानटा नसपेनटं ग, व्यखक्तगत िीिि बीमा बचत योििा है।
1)मैक्स लाइफ इों श्योरें स
2)बजाज आसलयाोंज लाइफ इों श्योरें स
3)ICICI प्रूडेंसशयल लाइफ इों श्योरें स
4)SBI लाइफ इों श्योरें स
5)टाटा AIA लाइफ इों श्योरें स
उत्तर-2)बिाि आनियांि िाइफ इं श्योरें स
स्पष्टीिरण:
बिाि आनियांि िाइफ इं श्योरें स कोंपनी सलसमटे ड ने पेश सकया बिाि आनियांि िाइफ ACE एक नॉन-सलोंक्ड,
पासटध ससपेसटों ग, व्यक्कक्तगत जीवन बीमा बचत योजना है। यह योजना ग्राहकोों को उनकी व्यक्कक्तगत सवत्तीय आवश्यकताओों
के अनुसार नकदी प्रवाह को सडजाइन करने में असर्क लचीलापन प्रदान करती है ।
• पारों पररक बचत उत्पादोों के सवपरीत, यह योजना ग्राहकोों को अपने आय प्रवाह को सडजाइन करने के सलए पसोंद
की शक्कक्त प्रदान करती है।
• प्रवेश के समय न्यूनतम और असर्कतम आयु - 18 से 55 वषध (लक्ष् सोंरक्षि लाभ (GPB) के साि) & 0 से 60
वषध (GPB के सबना) है।
• पररपक्वता आयु - 85 वषध की आयु तक (GPB के साि) और 100 वषध की आयु तक (GPB के सबना) है।
• प्रीसमयम भुगतान की आवृसत्त - वासषधक, अर्धवासषधक, त्रैमाससक और माससक है।

15. निस बैंि िे हाि ही में (नसतंबर’23 में) सेंटरि बैंि निनिटि िरें सी (CBDC) िे निए यूनिफाइि पेमेंट
इं टरफेस (UPI) इं टरऑपरे नबनिटी िी शुरुआत िी है ?
1)भारतीय स्ट्े ट बैंक
2)कनाधटक बैंक
3)RBL बैंक
4)IDBI बैंक
5)केनरा बैंक
उत्तर- 1)भारतीय स्ट्े ट बैंि
स्पष्टीिरण:
भारत के सबसे बडे सावधजसनक क्षेत्र के बैंक, भारतीय स्ट्े ट बैंि (SBI) ने अपने सडसजटल रुपए (ईरूपी), सजसे सेंटरल
बैंक सडसजटल करें सी (CBDC) भी कहा जाता है , के सलए यूसनफाइड पेमेंट इों टरफेस (UPI) इों टरऑपरे सबसलटी की
शुरुआत की है , सजससे ग्राहकोों के सलए पहुोंच और सुसवर्ा बढ गई है।
• 'ई-रूपी बाय SBI' एक्कप्लकेशन के माध्यम से, CBDC उपयोगकताध अब मचेंट UPI क्कक्वक ररस्पाोंस (QR) कोड को
सहजता से स्कैन कर सकते हैं , सजससे तेज और सुरसक्षत लेनदे न की सुसवर्ा समलती है।
• नोट - IDFC फस्ट्ध बैंक ने भी अपने CBDC के साि UPI QR कोड के एकीकरि की घोषिा की।

Report Errors in the PDF - ebooks@affairscloud.com Copyright 2014-2023 @ AffairsCloud.com 66


i.भारत के अग्रिी क्रेसडट काडध जारीकताध SBI काडध ने सी-सूट असर्काररयोों और उच्च-नेट-विध व्यक्कक्तयोों के सलए अपने
सुपर-प्रीसमयम काडध 'AURUM' की नवीनतम सुसवर्ाओों का अनावरि सकया है। AURUM एक सवसशष्ट्, केवल-
आमोंत्रि-िारा-क्रेसडट काडध है जो काडध र्ारकोों को असितीय सवशेषासर्कार प्रदान करने के सलए सडज़ाइन की गई चुसनोंदा
सुसवर्ाओों से सुसक्कित है ।

16. निस िंपिी िे हाि ही में (नसतंबर'23 में) िेशिि पेमेंट िॉरपोरे शि ऑफ इं निया (NPCI) िे सहयोग से
भारत िा पहिा व्हाइट िेबि ATM (WLA) यूनिफाइि पेमेंट इं टरफेस (UPI) -ऑटोमेटेि टे िर मशीि (ATM)
िॉन्च निया है ?
1)CCएवेन्यू पेमेंट गेटवे
2)इों स्ट्ामोजो पेमेंट गेटवे
3)सहताची पेमेंट ससवधसेज
4)कैशफ्री पेमेंट ससवधसेज
5)रे जरपे पेमेंट गेटवे
उत्तर -3)नहताची पेमेंट सनिासेि
स्पष्टीिरण:
सहताची सलसमटे ड की 100% सहायक कोंपनी और एक एकीकृत भुगतान समार्ान प्रदाता नहताची पेमेंट सनिासेि प्राइिेट
निनमटे ि ने नेशनल पेमेंट कॉरपोरे शन ऑफ इों सडया (NPCI) के सहयोग से व्हाइट लेबल ATM (WLA) के रूप में भारत
का पहला यूसनफाइड पेमेंट इों टरफेस (UPI) -ऑटोमेटेड टे लर मशीन (ATM) लॉन्च सकया, जो सुरसक्षत काडध लेस नकद
सनकासी की पेशकश करता है।
• UPI-केवल व्हाइट-लेबल ATM भौसतक काडध की आवश्यकता को समाप्त करके ग्राहक सुरक्षा में सुर्ार करे गा।
• 'UPI ATM' का लॉन्च पारों पररक ATM के साि UPI की सुसवर्ा और सुरक्षा को एकीकृत करता है।
• वतधमान में, सहताची पेमेंट नकद जमा सुसवर्ा प्रदान करने वाला एकमात्र WLA ऑपरे टर है।

17. निस बैंि िे हाि ही में (नसतंबर'23 में) सेंटरि बैंि निनिटि िरें सी (CBDC) और यूनिफाइि पेमेंट इं टरफेस
(UPI) इं टरऑपरे नबनिटी िॉन्च िी है ?
1)इों सडयन बैंक
2)पोंजाब नेशनल बैंक
3)RBL बैंक
4)कनाधटक बैंक
5)इों डसइों ड बैंक
उत्तर-2)पंिाब िेशिि बैंि
स्पष्टीिरण:
पोंजाब नेशनल बैंक (PNB) ने अपने PNB सडसजटल रुपया ऐप में सेंटरल बैंक सडसजटल करें सी (CBDC) और यूसनफाइड
पेमेंट इों टरफेस (UPI) इं टरऑपरे नबनिटी लॉन्च की।
• PNB सडसजटल रुपया ऐप उपयोगकताध इन व्यापाररयोों के UPI QR पर खरीदारी करने के सलए अपने CBDC
वॉलेट का उपयोग कर सकते हैं , भले ही उनके पास CBDC वॉलेट न हो।
• इों टरऑपरे सबसलटी वह तकनीकी अनुकूलता है जो एक पेमेंट ससस्ट्म को अन्य भुगतान प्रिासलयोों के साि काम
करने की अनुमसत दे ती है।

Report Errors in the PDF - ebooks@affairscloud.com Copyright 2014-2023 @ AffairsCloud.com 67


• CBDC वॉलेट में RBI िारा जारी सडसजटल रुपये (ई रुपया) है और यह रुपये का एक टोकनयुक्त सडसजटल
सोंस्करि है। सडसजटल रुपया सबल्कुल एक सोंप्रभु मुद्रा की तरह कायध करता है और कागजी मुद्रा के साि 1:1 में
सवसनमेय है।

18. नसतंबर 2023 में िारी एनशयि िे ििपमेंट बैंि (ADB) िे 2023 टर े ि फाइिेंस गैप्स, ग्रोथ एं ि िॉि सिे िे
अिुसार, 2022 में िैनश्वि टर े ि फाइिेंस गैप बढ़िर ______________ हो गया।
1)5.0 सटर सलयन अमेररकी डॉलर
2)4.5 सटर सलयन अमेररकी डॉलर
3)3.2 सटर सलयन अमेररकी डॉलर
4)2.5 सटर सलयन अमेररकी डॉलर
5)1.2 सटर सलयन अमेररकी डॉलर
उत्तर-4)2.5 नटर नियि अमेररिी िॉिर
स्पष्टीिरण:
एसशयन डे वलपमेंट बैंक (ADB) िे 2023 टर े ड फाइनेंस गैप्स, ग्रोि एों ड जॉब्स सवे (8िें संस्करण) के अनुसार, वैसश्वक
टर े ड फाइनेंस गैप्स 2020 में 1.7 सटर सलयन अमेररकी डॉलर और 2018 में 1.5 सटर सलयन अमेररकी डॉलर से बढकर 2022
में 2.5 नटर नियि अमेररिी िॉिर (वैसश्वक व्यापाररक व्यापार का 10%) हो गया।
• इससे पता चलता है सक वैसश्वक अिधव्यवथिा अभी भी COVID-19 महामारी के प्रभाव से उबरने के सलए सोंघषध
कर रही है ।
• टर े ड फाइनेंस गैप्स आयात और सनयाधत का समिधन करने के सलए सवत्तपोषि के अनुरोर्ोों और अनुमोदनोों के बीच
का अोंतर है ।
• प्रासोंसगक आपूसतध श्रृोंखलाओों और टर े ड फाइनेंस गैप्स पर प्रभाव का मूल्ाोंकन करने के सलए, 2023 व्यापार अोंतराल
सवेक्षि पहली बार (सडसजटलीकरि के साि) पयाधवरि, सामासजक और शासन (ESG) मुद्दोों पर केंसद्रत है।

19. अगस्त 2023 में आयोनित REC निनमटे ि िी 5िी ं िानषाि आम बैठि (AGM) िे संबंध में निम्ननिखित में
से िौि सा नबंदु "सही" है /हैं ?
A)REC निनमटे ि 2025 ति अपिे हररत पररयोििा ऋण पोटा फोनियो िो 5 िाि िरोड रुपये ति बढ़ािे िे
निए प्रनतबि है , िो 10 गुिा से अनधि िी िृखि दशााता है।
B)REC निनमटे ि िे 2023-24 िे निए 1.20 िाि िरोड रुपये िे अपिे बािार उधार िायाक्रम िे तहत मांिटे ि
िीि अरें िसा एं ि बूिरिसा (MLAB) िे रूप में दो निस्तों में 1.15 नबनियि अमेररिी िॉिर िुटाए।
C)REC निनमटे ि िे EXIM बैंि ऑफ इं निया से 100 नमनियि अमेररिी िॉिर िा निदे शी मुद्रा सािनध ऋण
प्राप्त निया।
1)केवल A
2)केवल A & B
3)केवल B & C
4)केवल A & C
5)सभी A, B & C
उत्तर-3)िेिि B & C

Report Errors in the PDF - ebooks@affairscloud.com Copyright 2014-2023 @ AffairsCloud.com 68


स्पष्टीिरण:
6 ससतोंबर, 2023 को REC निनमटे ि (पूवध में ग्रामीि सवद् युतीकरि सनगम सलसमटे ड ) की 5िी ं वासषधक आम बैठक (AGM)
वीसडयो कॉन्क्फ्रेंससोंग के माध्यम से आयोसजत की गई िी। बैठक की अध्यक्षता इसके अध्यक्ष एवों प्रबोंर् सनदे शक (CMD)
सववेक कुमार दे वाों गन ने की।
i.AGM िी मुख्य बातें:
• REC सलसमटे ड 2030 तक अपने हररत पररयोजना ऋि पोटध फोसलयो को 3 िाि िरोड रुपये तक बढाने के
सलए प्रसतबद्ध है , जो 10 गुना से असर्क की वृक्कद्ध दशाधता है।
• सवद् युत मोंत्रालय ने REC को भारत के सवकास में योगदान दे ने वाले गैर-सबजली बुसनयादी ढाोंचे और रसद क्षेत्रोों को
ऋि प्रदान करने का असर्कार सदया है।
• REC सलसमटे ड ने 2023-24 के सलए 1.20 लाख करोड रुपये के अपने बाजार उर्ार कायधक्रम के तहत माोंडटे ड
लीड अरें जसध और बुकरनर (MLAB) के रूप में छह बैंकोों के कोंसोसटध यम से अगस्त 2023 में दो सकस्तोों में 1.15
नबनियि अमेररिी िॉिर जुटाए।
• REC सलसमटे ड ने EXIM (एक्सपोटा इम्पोटा ) बैंि ऑफ इं निया से 100 नमनियि अमेररिी िॉिर का निदे शी
मुद्रा सािनध ऋण प्राप्त सकया। यह EXIM बैंक िारा REC को सदया जाने वाला पहला सावसर् ऋि है।

20. नसतंबर 2023 में भारतीय प्रनतभूनत और निनिमय बोिा (SEBI) द्वारा िारी अिुमोदि/पररपत्र िे संबंध में
निम्ननिखित में से िौि सा नबंदु "सही" है /हैं ?
A)SEBI िे ऑििाइि स्ट्ॉि िोनिंग फमा , ग्रो म्यूचुअि फंि िो अपिा पहिा इं िेक्स फंि 'ग्रो निफ्टी टोटि
मािेट् स इं िेक्स फंि' िॉन्च िरिे िी मंिूरी दे दी। यह फंि न्यू फंि ऑफररं ग (NFO) िे िररए िुटाया िाएगा।
B)भारतीय प्रनतभूनत और निनिमय बोिा (SEBI) िे एि पररपत्र िारी िर िहा नि SEBI नििेशि सुरक्षा और
नशक्षा िोष (IPEF) में धि िमा िरिा िेिि ऑििाइि मोि िे माध्यम से निया िािा चानहए।
C)यह पररपत्र भारतीय प्रनतभूनत और निनिमय बोिा अनधनियम, 1992 िी धारा 11(1)िे तहत प्रदत्त शखक्तयों
िा प्रयोग िरते हुए िारी निया गया है।
1)केवल A
2)केवल A & B
3)केवल B & C
4)केवल A & C
5)सभी A, B & C
उत्तर -5)सभी A, B & C
स्पष्टीिरण:
भारतीय प्रसतभूसत और सवसनमय बोडध (SEBI) ने ऑनलाइन स्ट्ॉक ब्रोसकोंग फमध , ग्रो म्यूचुअल फोंड को अपना पहला इों डेक्स
फोंड 'ग्रो निफ्टी टोटि मािेट् स इं िेक्स फंि' लॉन्च करने की मोंजूरी दे दी। यह फोंड न्यू फोंड ऑफररों ग (NFO) के
जररए जुटाया जाएगा।
• लॉन्च सकया गया इों डेक्स फोंड एक पैससव म्यूचुअल फोंड है (जो सनफ्टी या सेंसेक्स जैसे माकेट इों डेक्स की नकल
करता है) और इसे ससतोंबर 2023 के अोंत तक लॉन्च सकया जाएगा।
• भारतीय प्रसतभूसत और सवसनमय बोडध (SEBI) ने एक पररपत्र जारी कर कहा सक SEBI सनवेशक सुरक्षा और सशक्षा
कोष (IPEF) में र्न जमा करना केवल ऑनलाइन मोड के माध्यम से सकया जाना चासहए।
• यह पररपत्र भारतीय प्रसतभूसत और सवसनमय बोडध असर्सनयम, 1992 की धारा 11(1)के तहत प्रदत्त शक्कक्तयोों का
प्रयोग करते हुए जारी सकया गया है।

Report Errors in the PDF - ebooks@affairscloud.com Copyright 2014-2023 @ AffairsCloud.com 69


• नए सदशासनदे शोों ने 2020 में पहले जारी SEBI सदशासनदे शोों को प्रसतथिासपत कर सदया, सजसमें सनर्ाधररत सकया गया
िा सक रासश IPEF में ऑनलाइन मोड या सडमाोंड डर ाफ्ट (DD) के माध्यम से जमा की जाएगी।

21. उस िंपिी िा िाम बताइए निसिे हाि ही में (नसतंबर'23 में) उद्यमों और निक्रेताओं दोिों िे निए नबििेस-
टू -नबििेस (B2B) निनिटि भुगताि िो फायदे मंद बिािे िे निए KredX (नमनियंस िेंचसा प्राइिेट निनमटे ि) िे
साथ साझेदारी िी है।
1)वीज़ा
2)अमेररकन एक्सप्रेस
3)मास्ट्रकाडध
4)कैसपटल वन
5)सडस्कवर काडध
उत्तर- 3)मास्ट्रिािा
स्पष्टीिरण:
KredX (समसनयोंस वेंचसध प्राइवेट सलसमटे ड), भारत का सबसे बडा सप्लाई चेन फाइनेंस प्लेटफॉमध , ने सबजनेस-टू -
सबजनेस (B2B) सडसजटल भुगतान को उद्यमोों और सवक्रेताओों दोनोों के सलए फायदे मोंद बनाने के सलए दु सनया भर में
दू सरी सबसे बडी भुगतान प्रसोंस्करि कोंपनी मास्ट्रिािा , इों क. के साि साझेदारी की है।
i.इस साझेदारी के तहत, मास्ट्रकाडध अपनी वासिक्कज्यक काडध सेवा को KredX प्लेटफॉमध के साि एकीकृत करे गा,
सजससे B2B भुगतान, सवशेषकर काडध से जुडी जसटलताओों को दू र सकया जा सकेगा।
ii.गसतशील छूट, शीघ्र भुगतान सवकल्प और मूल् खोज तोंत्र जैसी सुसवर्ाओों वाला मोंच उद्यमोों और सवक्रेताओों को अपने
नकदी प्रवाह को बढाने में सक्षम करे गा।
iii.आसटध सफसशयल इों टेसलजें स (AI) िारा सोंचासलत, प्लेटफ़ॉमध व्यवसायोों के सलए एों ड-टू -एों ड प्रोक्ोर-टू -पे समार्ान के
रूप में कायध करता है , सजससे उन्ें चालान के तेज़ और असर्क कुशल समलान और प्रसोंस्करि में मदद समलती है।

22. निस िंपिी िे हाि ही में (नसतंबर’23 में) सूक्ष्म, िघु और मध्यम उद्यमों (MSME) िे निए एि क्रेनिट िािा
'SimplySAVE मचेंट िािा ' िॉन्च निया है ?
1)HDFC काडध
2)SBI काडध
3)अमेररकन एक्सप्रेस
4)वीज़ा इों क
5)मास्ट्रकाडध
उत्तर- 2)SBI िािा
स्पष्टीिरण:
भारत की सबसे बडी प्योर-प्ले क्रेसडट काडध कोंपनी, SBI काड्ध स एों ड पेमेंट ससवधसेज सलसमटे ड (SBI िािा ) ने सूक्ष्म, लघु
और मध्यम उद्यमोों (MSME) के सलए एक क्रेसडट काडध , 'SimplySAVE मचेंट SBI िािा ' लॉन्च सकया। काडध को
MSME व्यापाररयोों के सलए ब्याज मुक्त अल्पकासलक ऋि सवकल्प और अन्य सवशेष लाभ प्रदान करने के सलए सडज़ाइन
सकया गया है।
i.SimplySAVE मचेंट SBI काडध का अनावरि मुोंबई, महाराष्ट्र में ग्लोबल सफनटे क फेस्ट् में भारतीय स्ट्े ट बैं क (SBI) के
अध्यक्ष श्री सदनेश खारा िारा सकया गया।

Report Errors in the PDF - ebooks@affairscloud.com Copyright 2014-2023 @ AffairsCloud.com 70


ii.यह काडध MSME को ब्याज मुक्त अल्पकासलक ऋि प्रदान करके औपचाररक ऋि तक आसान और समय पर पहुोंच
की आवश्यकता को पूरा करे गा।
iii.यह काडध RuPay नेटवकध पर उपलब्ध है और इसे सवसभन्न UPI-सक्षम तृतीय-पक्ष ऐप्स के माध्यम से भुगतान के सलए
यूसनफाइड पेमेंट्स इों टरफ़ेस (UPI) से भी जोडा जा सकता है।

23. नसतंबर 2023 में मुंबई, महाराष्टर में आयोनित ग्लोबि नफिटे ि फेस्ट् (GFF) िे संबंध में निम्ननिखित में से
िौि सा/से नबंदु "सही" है /हैं?
A) ग्लोबि नफिटे ि फेस्ट् (GFF) िा संयुक्त रूप से भारतीय राष्टरीय भुगताि निगम (NPCI), भारतीय भुगताि
पररषद (PCI) और नफिटे ि अनभसरण पररषद (FCC) द्वारा आयोनित निया गया था।
B) भारतीय ररििा बैंि (RBI) िे गििार शखक्तिांत दास िे हाि ही में ग्लोबि नफिटे ि फेस्ट् 2023 में भारतीय
राष्टरीय भुगताि निगम (NPCI) द्वारा नििनसत पांच िए इिोिेनटि भुगताि समाधािों िी घोषणा िी।
C) ऑििाइि PSB िोि निनमटे ि िे संस्थापि और CEO टॉम ग्रीििुि िे नफिटे ि पसािैनिटी ऑफ द ईयर -
गल्फ िोऑपरे शि िंटर ीि (GCC) श्रेणी िे तहत ग्लोबि नफिटे ि अिाि्ा स 2023 िीता।
1)केवल A
2)केवल A & B
3)केवल B & C
4)केवल A & C
5)सभी A, B & C
उत्तर- 2)िेिि A & B
स्पष्टीिरण:
ग्लोबल सफनटे क फेस्ट् (GFF) 5 से 7 ससतोंबर, 2023 तक सजयो वडध कन्वेंशन सेंटर, मुोंबई, महाराष्ट्र, भारत में आयोसजत
सबसे बडा सफनटे क 3-सदवसीय सम्मेलन है। यह सोंयुक्त रूप से भारतीय राष्ट्रीय भुगतान सनगम (NPCI), भारतीय भुगतान
पररषद (PCI) और सफनटे क असभसरि पररषद (FCC) िारा आयोसजत सकया गया िा।
• िीम: ग्लोबल कोलैबोरे शन फॉर ए रे स्पोोंससबल फाइनेंससयल इकोससस्ट्म -इों क्लूससव | रे सससलएों ट | सस्ट्े नेबल
• मुोंबई में ग्लोबल सफनटे क फेस्ट् 2023 में सवत्त मोंत्री सनमधला सीतारमि के सोंबोर्न में भारत के सवत्तीय पाररक्कथिसतकी
तोंत्र और इसकी चुनौसतयोों के कई महत्वपूिध पहलुओों को शासमल सकया गया।
• भारतीय ररजवध बैंक (RBI) के गवनधर शक्कक्तकाों त दास ने हाल ही में ग्लोबल सफनटे क फेस्ट् 2023 में भारतीय
राष्ट्रीय भुगतान सनगम (NPCI) िारा सवकससत पाोंच नए इनोवेसटव भुगतान समार्ानोों की घोषिा की।
• लुलु फाइनेंसशयल होक्कडोंग्स के प्रबोंर् सनदे शक अदीब अहमद िे नफिटे ि पसािैनिटी ऑफ द ईयर - गल्फ
िोऑपरे शि िंटर ीि (GCC) श्रेणी के तहत ग्लोबल सफनटे क अवाड्ध स 2023 जीता।
• भारतीय सवसशष्ट् पहचान प्रासर्करि (UIDAI) ने मुोंबई में ग्लोबल सफनटे क फेक्कस्ट्वल (GFF) 2023 में एक
महत्वपूिध प्रभाव डाला, "रीइमेसजन आर्ार #टु गेदर" सवषय के तहत एक उद्योग बैठक का आयोजन सकया।
• दे श के सबसे बडे ऋिदाता भारतीय स्ट्े ट बैंक (SBI) ने ग्लोबि नफिटे ि फेस्ट् 2023 में 'नेशन फस्ट्ध टर ाोंसजट
काडध ' का अनावरि सकया है , सजसे रुपे नेशनल कॉमन मोसबसलटी काडध (NCMC) प्रीपेड काडध के रूप में भी
जाना जाता है।

Report Errors in the PDF - ebooks@affairscloud.com Copyright 2014-2023 @ AffairsCloud.com 71


24. नसतंबर 2023 में, ____________ ओपि िेटििा फॉर निनिटि िॉमसा (ONDC)-सक्षम प्रस्ताि पेश िरिे
िािा भारत िा पहिा निदे शी बैंि बि गया।
1)स्ट्ैं डडध चाटध डध बैंक इों सडया
2)DBS बैंक इों सडया
3)HSBC इों सडया
4)CSB बैंक इों सडया
5)ससटीबैंक इों सडया
उत्तर-2)HSBC इं निया
स्पष्टीिरण:
हाोंगकाोंग और शोंघाई बैंसकोंग कॉपोरे शन (HSBC) इं निया ने मुोंबई, महाराष्ट्र में आयोसजत ग्लोबल सफनटे क फेस्ट् (GFF)
2023 के दौरान ONDC प्लेटफॉमध में भाग लेने के सलए कॉपोरे ट् स के सलए वन-स्ट्ॉप-शॉप प्रस्ताव 'ओपन नेटवकध फॉर
सडसजटल कॉमसध (ONDC) इि ए बॉक्स' लॉन्च सकया है।
i.इस लॉन्च के साि, HSBC इों सडया ONDC-सक्षम प्रस्ताव पेश करने वाला भारत का पहला सवदे शी बैंक बन गया है।
ii.ONDC इन ए बॉक्स को प्रोटीन ईगॉव टे क्नोलॉजीज सलसमटे ड (प्रोटीन) और शायर ओमनीचैनल प्राइवेट सलसमटे ड
(आद्या) के साि साझेदारी में सवकससत सकया गया िा।

25. उस बैंि िा िाम बताइए िो हाि ही में (नसतंबर'23 में) हाई-िेट-िथा व्यखक्तयों (HNI) और मनहिा ग्राहिों
िो सेिा प्रदाि िरिे िािा मेटि िे नबट िािा िािा पेश िरिे िािा पहिा सािािनिि क्षेत्र िा बैंि (PSB) बि
गया है।
1)यूसनयन बैंक ऑफ इों सडया
2)बैंक ऑफ बडौदा
3)बैंक ऑफ इों सडया
4)पोंजाब नेशनल बैंक
5)इों सडयन बैंक
उत्तर-1)यूनियि बैंि ऑफ इं निया
स्पष्टीिरण:
यूसनयन बैंक ऑफ इों सडया (UBI) ने नेशनल पेमेंट्स कॉरपोरे शन ऑफ इों सडया (NPCI) के सहयोग से दो नए डे सबट काडध
वेररएों ट, HNI एम्पीररयो रुपे मेटि िे नबट िािा और रुपे एम्पािर हर िे नबट िािा पेश सकए हैं।
• इसके साि, UBI हाई-नेट-विध व्यक्कक्तयोों (HNI) और मसहला ग्राहकोों को सेवा प्रदान करने वाला मेटल डे सबट
काडध काडध पेश करने वाला पहला सावधजसनक क्षेत्र का बैंक (PSB) बन गया है।
• ये काडध रुपे नेटवकध पर जारी सकए जाते हैं।
• काडों का अनावरि मुोंबई, महाराष्ट्र में ग्लोबल सफनटे क फेस्ट् (GFF) 2023 में UBI के MD & CEO (प्रबोंर्
सनदे शक और मुख्य कायधकारी असर्कारी), A. मसिमेखलाई और NPCI के COO (मुख्य पररचालन असर्कारी),
प्रवीिा राय िारा सकया गया।

26. निस िघु नित्त बैंि (SFB) िे हाि ही में (नसतंबर'23 में) अपिे बैंनिंग समाधािों िी श्रृंििा िा निस्तार
िरिे िे निए िेनिथ+ मेटि क्रेनिट िािा पेश निया है ?
1)सूयोदय SFB
2)इक्कक्वटास SFB

Report Errors in the PDF - ebooks@affairscloud.com Copyright 2014-2023 @ AffairsCloud.com 72


3)जना SFB
4)AU SFB
5)उिीवन SFB
उत्तर-4)AU SFB
स्पष्टीिरण:
AU स्मॉि फाइिेंस बैंि निनमटे ि, भारतीय ररजवध बैंक (RBI) िारा अनुमोसदत लघु सवत्त बैंक (SFB) ने बैंसकोंग
समार्ानोों की अपनी सीमा का सवस्तार करते हुए िेनिथ+ मेटि क्रेनिट िािा पेश सकया।
• इस सुपर प्रीसमयम क्रेसडट काडध में वैसश्वक यात्रा से लेकर प्रीसमयम लाउों ज एक्सेस , व्यक्कक्तगत कोंसीयजध सेवाओों
जैसे असीसमत मनोरों जन सवकल्पोों और काडध एक्करवेशन पर 5000 रुपये के लक्जरी ब्राोंड वाउचर या ररवाडध
पॉइों ट्स के साि एक स्वागत योग्य लाभ शासमल है।
• काडध के लाभोों में वैसश्वक यात्रा के सलए 0.99% का सबसे कम सवदे शी मुद्रा (फोरे क्स) माकधअप शासमल है ।

27. उस स्मॉि फाइनेंस बैंक (SFB) का नाम बताइए लिसने हाि ही में (लसतंबर’23 में) स्ट्ार हेल्थ एं ि अिाइि
इं श्योरें स कंपनी और बिाि आलियांि िाइफ इं श्योरें स कंपनी के साथ बैंकएश्योरें स समझौता लकया है।
1) ESAF SFB
2) AU SFB
3) उत्कषग SFB
4) इस्िटास SFB
5) सूयोदय SFB
उत्तर-2)AU SFB
स्पष्टीकरण:
AU स्मॉल फाइनेंस बैंक (AU SFB) ने हाल ही में स्ट्ार हेल्थ एं ि अलाइि इं श्योरें स कंपनी (स्ट्ार हेल्थ), और बिाि
आश्चलयांि लाइफ इं श्योरें स कंपनी (बिाि आलियांि) के साि बैंकएश्योरें स समझौता श्चकया है। यह रिनीश्चतक कदम
AU बैंक को इन साझेदाररयों के माध्यम से अपने ग्राहकों को इं श्योरें स उत्पादों की एक श्रृंखला की पेशकश करने की
अनुमश्चत दे ता है।
i.AU बैंक के ग्राहक अब बैंक की शाखाओं और श्चिश्चिटल चैनलों के माध्यम से स्वास्थ्य इं श्योरें स और सामान्य इं श्योरें स
पॉश्चलश्चसयों िैसे इं श्योरें स उत्पादों तक पहुंच सकते हैं।
• इस साझेदारी के तहत, AU SFB बिाि आश्चलयांि लाइफ इं श्योरें स से इं श्योरें स उत्पादों की एक श्रृंखला
श्चवतररत करे गा। इन उत्पादों में टमग इं श्योरें स, बचत योिना और सेवाश्चनवृश्चत्त उत्पाद शाश्चमल हैं।
• स्ट्ार हेल्थ के साि साझेदारी के मामले में, AU SFB स्ट्ार हेल्थ से स्वास्थ्य इं श्योरें स योिनाएं श्चवतररत करे गा।

28. निस िाइफ इं श्योरें स िंपिी िे हाि ही में (नसतंबर’23 में) इििम िाभ िॉन्च निया है , िो पॉनिसी अिनध
िे दौराि नित्तीय सुरक्षा और िगातार आय स्ट्र ीम दोिों प्रदाि िरिे िे निए एि व्यखक्तगत बचत योििा है ?
1)मैक्स लाइफ इों श्योरें स
2)बजाज आसलयाोंज लाइफ इों श्योरें स
3)ICICI प्रूडेंसशयल लाइफ इों श्योरें स
4)भारती AXA लाइफ इों श्योरें स
5)टाटा AIA लाइफ इों श्योरें स
उत्तर-4)भारती AXA िाइफ इं श्योरें स

Report Errors in the PDF - ebooks@affairscloud.com Copyright 2014-2023 @ AffairsCloud.com 73


स्पष्टीिरण:
भारती AXA िाइफ इं श्योरें स ने भारती AXA लाइफ इििम िाभ लॉन्च सकया है , जो एक व्यक्कक्तगत बचत योजना है
जो पॉसलसी अवसर् के दौरान सवत्तीय सुरक्षा और लगातार आय प्रवाह दोनोों प्रदान करती है।
• यह लाइफ इों श्योरें स कवरे ज के साि एक गैर-सलोंक्ड, गैर-भागीदारी वाली व्यक्कक्तगत बचत योजना है।
• यह लॉन्च इों श्योरें स को सरल बनाने और अपने उपभोक्ताओों को असर्कतम सुरक्षा प्रदान करने के उद्दे श्य के
एक सहस्से के रूप में सकया गया िा।
i.पॉसलसीर्ारक 10 से 12 वषों के सलए गारों टीकृत आय की उम्मीद कर सकते हैं , सजसमें प्रारों सभक भुगतान वासषधक
प्रीसमयम के 100% के बराबर होगा और 10 साल की अवसर् के सलए 5 साल के बाद 150% तक बढ जाएगा, या 12
साल की अवसर् के सलए 6 साल के बाद 200% हो जाएगा।

29. उस भुगताि िंपिी िा िाम बताइए निसिे हाि ही में (नसतंबर’23 में) "ऑटोपे ऑि QR (खिि ररस्पांस)"
िॉन्च िरिे िे निए िेशिि पेमेंट्स िॉरपोरे शि ऑफ इं निया (NPCI) िे साथ सहयोग निया है , िो 2-चरणीय
प्रनक्रया िे माध्यम से ग्राहि िो शानमि िरिे में सक्षम सुनिधा है।
1)कैशफ्री पेमेंट्स इों सडया प्राइवेट सलसमटे ड
2)रे जरपे सॉफ्टवेयर प्राइवेट सलसमटे ड
3)भारतपे सलसमटे ड
4)इन्फीबीम एवेन्यूज़ सलसमटे ड
5)पेटीएम प्राइवेट सलसमटे ड
उत्तर- 1)िैशफ्री पेमेंट्स इं निया प्राइिेट निनमटे ि
स्पष्टीिरण:
िैशफ्री पेमेंट्स इं निया प्राइिेट निनमटे ि ने नेशनल पेमेंट्स कॉरपोरे शन ऑफ इों सडया (NPCI) के सहयोग से "ऑटोपे
ऑि QR (क्कक्वक ररस्पॉर्न्)" लॉन्च सकया, जो 2-चरिीय प्रसक्रया के माध्यम से ग्राहक को शासमल करने में सक्षम सुसवर्ा
है।
i.2 चरि की प्रसक्रया में QR कोड को स्कैन करना और यूसनफाइड पेमेंट्स इों टरफेस (UPI) ऐप्स के माध्यम से ई-जनादे श
को मोंजूरी दे ना शासमल है।
ii.कैशफ्री पेमेंट्स कोंपनी के उत्पादोों को शक्कक्त प्रदान करने वाले मुख्य भुगतान और बैंसकोंग बुसनयादी ढाोंचे के सनमाधि के
सलए बैंकोों के साि काम करता है।
iii.यह शॉपीफाय, सवक्स, पेपाल, अमेज़न पे, पेटीएम और गूगल पे जैसे प्रमुख प्लेटफामों के साि भी एकीकृत है।

30. पयाटि मंत्रािय और इन्वेस्ट् इं निया िे साथ निस िंपिी िे हाि ही में (नसतंबर’23 में) भारत िे इनतहास
और संस्कृनत िा पता िगािे िे निए Priceless.com िॉन्च निया है ?
1)वीज़ा इों क
2)मास्ट्रकाडध
3)अमेररकन एक्सप्रेस
4)कैसपटल वन
5)सडस्कवर काडध
उत्तर- 2)मास्ट्रिािा

Report Errors in the PDF - ebooks@affairscloud.com Copyright 2014-2023 @ AffairsCloud.com 74


स्पष्टीिरण:
पयाटि मंत्रािय और इन्वेस्ट् इं निया के सहयोग से 'मास्ट्रिािा ' ने भारत के इसतहास और सोंस्कृसत (नवीनतम यात्रा
रुझानोों का उपयोग करके) का पता लगाने के सलए Priceless.com लॉन्च सकया है ।
• Priceless.com पयधटन मोंत्रालय की "दे िो अपिा दे श" पहल का समिधन करे गा।
i.priceless.com मास्ट्रकाडध के वैसश्वक अमूल् प्लेटफॉमध का सवस्तार है जो दु सनया भर के मास्ट्रकाडध काडध र्ारकोों को
सवशेष अनुभव और सवशेषासर्कार प्रदान करता है।
ii.यह थिानीय और अोंतराध ष्ट्रीय यासत्रयोों के सलए दे श की सवसवर् सवरासत का अनुभव करने का मागध प्रशस्त करता है ।

31. निस भारतीय प्रौद्योनगिी संस्थाि (IIT) िे हाि ही में (नसतंबर'23 में) स्ट्ाटा अप पाररखस्थनतिी तंत्र िा समथाि
िरिे िे निए ICICI बैंि िे साथ एि MoU पर हस्ताक्षर निए हैं ?
1)IIT मद्रास
2)IIT गुवाहाटी
3)IIT खडगपुर
4)IIT कानपुर
5)IIT इों दौर
उत्तर-4)IIT िािपुर
स्पष्टीिरण:
उत्तर प्रदे श (UP) में भारतीय प्रौद्योसगकी सोंथिान (IIT),िािपुर ने स्ट्ाटध अप इनक्ूबेशन एों ड इनोवेशन सेंटर (SIIC) IIT
िािपुर में स्ट्ाटध अप पाररक्कथिसतकी तोंत्र का समिधन करने के सलए ICICI बैंि सलसमटे ड के साि एक समझौता ज्ञापन
(MoU) पर हस्ताक्षर सकए हैं।
i.साझेदारी शुरुआती चरि के उद्यमोों के सलए एक व्यापक सहायता प्रिाली प्रदान करती है ।
ii.इस MoU के तहत, IIT कानपुर और ICICI बैंक SIIC में स्ट्ाटध अप और इनोवेटसध को आवश्यक उपकरिोों और ज्ञान
से लैस करने के सलए सोंयुक्त प्रयासोों की सुसवर्ा प्रदान करें गे।
iii.ICICI बैंक सनजी इक्कक्वटी (PE) और वेंचर कैसपटल (VC) समिधन के साि जु डते हुए IIT कानपुर में नवप्रवतधकोों,
इनक्ूबेसटयोों और छात्रोों को लेनदे न से पहले और बाद के अनुपालन में मदद करता है।

32. उस बैंि िा िाम बताइए निसिे हाि ही में (नसतंबर'23 में) I-प्रोसेस सनिासेि (इं निया) प्राइिेट निनमटे ि
(I-प्रोसेस) िो पूणा स्वानमत्व िािी सहायि िंपिी में बदििे िे निए भारतीय ररििा बैंि (RBI) से मंिूरी प्राप्त
िी है।
1)HDFC बैंक
2)ICICI बैंक
3)YES बैंक
4)एक्कक्सस बैंक
5)इों डसइों ड बैंक
उत्तर- 2)ICICI बैंि
स्पष्टीिरण:
भारतीय ररजवध बैंक (RBI) ने सवसशष्ट् शतों के अर्ीन, I-प्रोसेस ससवधसेज (इों सडया) प्राइवेट सलसमटे ड (I-प्रोसेस) को पूिध
स्वासमत्व वाली सहायक कोंपनी में बदलने के सलए ICICI बैंि सलसमटे ड को मोंजूरी दे दी है।
• ICICI बैंक के बोडध सदस्योों ने फरवरी 2023 में पहले ही प्रस्ताव को मोंजूरी दे दी िी।

Report Errors in the PDF - ebooks@affairscloud.com Copyright 2014-2023 @ AffairsCloud.com 75


• रूपाोंतरि की अनुमासनत लागत 15.40 िरोड रुपये (अोंसतम सनर्ाधरि के अर्ीन) है।
• ICICI बैंक के पास सफलहाल I-प्रोसेस में 19% सहस्सेदारी है।

33. निस इं श्योरें स िंपिी िे हाि ही में (नसतंबर'23 में) एि ही योििा िे तहत स्वास्थ्य और िाइफ इं श्योरें स
िे िाभों िो नमिािर एि नसक्ोर अनििंग्स एं ि िेििेस एििांटेि (SEWA) योििा िॉन्च िी है ?
1)मैक्स लाइफ इों श्योरें स
2)बजाज आसलयाोंज लाइफ इों श्योरें स
3)ICICI प्रूडेंसशयल लाइफ इों श्योरें स
4)भारती AXA लाइफ इों श्योरें स
5)टाटा AIA लाइफ इों श्योरें स
उत्तर-1)मैक्स िाइफ इं श्योरें स
स्पष्टीिरण:
मैक्स िाइफ इं श्योरें स िंपिी निनमटे ि ने एक नई इों श्योरें स प्लान "मैक्स लाइफ ससक्ोर असनिंग्स एों ड वेलनेस एडवाोंटेज
(SEWA) प्लाि" लॉन्च की है जो एक ही प्लान के तहत स्वास्थ्य और लाइफ इों श्योरें स के लाभोों को जोडती है ।
• यह बीमार्ारक को अपने पररवार की स्वास्थ्य सोंबोंर्ी जरूरतोों को पूरा करने के साि-साि भसवष्य के खचों के
सलए सवत्तीय बैकअप सुसनसित करने की अनुमसत दे ता है ।
• यह बीमार्ारक के सलए स्वास्थ्य, सुरक्षा, बचत और कल्ाि लाभोों को एकीकृत करता है और एक व्यापक प्लान
प्रदान करता है जो जीवन कवर के साि अस्पताल में भती, गहन दे खभाल इकाई (ICU) शुल्क आसद को कवर
करता है।
• SEWA प्लान 2 वेररएों ट: एलीट वेररएों ट और लाइट वेररएों ट में उपलब्ध है।

34. नसतंबर 2023 में भारतीय ररििा बैंि (RBI) द्वारा िारी नदशानिदे शों िे संबंध में निम्ननिखित में से िौि सा
नबंदु "सही" है/हैं?
A)RBI िे निम्मेदार ऋण आचरण पर नदशानिदे श िारी निए िो निनियनमत संस्थाओं (RE) िो सभी मूि चि
या अचि संपनत्त दस्तािेिों िो िारी िरिे और उधारितााओ ं द्वारा व्यखक्तगत ऋण िे पूणा पुिभुागताि या
निपटाि िे 30 नदिों िे भीतर निसी भी रनिस्ट्र ी में पंिीिृत शुल्क िो हटािे िा निदे श दे ते हैं।
B) ये निदे श उि सभी मामिों पर िागू होते हैं िहां मू ि चि/अचि संपनत्त दस्तािेिों िी ररहाई 1 नदसंबर, 2023
िो या उसिे बाद होती है।
C)RBI िे RBI (िानसनफिेशि, िैल्यूएशि एं ि ऑपरे शि ऑफ़ इन्वेस्ट्मेंट पोटा फोनियो ऑफ़ िमनशायि बैंक्स)
निरे क्शंस, 2023 िारी िरिे उन्ें िैनश्वि माििों िे साथ संरेखित िरिे िे निए बैंिों द्वारा नििेश िो िगीिृत
िरिे पर अपिे नदशानिदे शों िो भी संशोनधत निया, िो 1 अप्रैि, 2024 से प्रभािी होगा।
1)केवल A
2)केवल A & B
3)केवल B & C
4)केवल A & C
5)सभी A, B & C
उत्तर -5)सभी A, B & C

Report Errors in the PDF - ebooks@affairscloud.com Copyright 2014-2023 @ AffairsCloud.com 76


स्पष्टीिरण:
13 ससतोंबर, 2023 को, भारतीय ररजवध बैंक (RBI) ने सजम्मेदार ऋि आचरि पर सदशासनदे श जारी सकए, जो सवसनयसमत
सोंथिाओों (RE) को सभी मूल चल या अचल सोंपसत्त दस्तावेजोों को जारी करने और ऋिकताधओों िारा व्यक्कक्तगत ऋि के
पूिध पुनभुधगतान या सनपटान के 30 नदिों के भीतर सकसी भी रसजस्ट्र ी में पोंजीकृत शुल्क को हटाने का सनदे श दे ता है।
• 'रे स्पोोंससबल लेंसडों ग कोंडर– ररलीज़ ऑफ़ मूवेबल / इम्मूवेबल प्रॉपटी डॉक्ुमेंट्स ऑन रीपेमेंट/ सेटलमेंट ऑफ़
पसधनल लोर्न्' की आसर्काररक असर्सूचना है।
• ये सनदे श उन सभी मामलोों पर लागू होते हैं जहाों मूल चल/अचल सोंपसत्त दस्तावेजोों की ररहाई 1 नदसंबर, 2023
को या उसके बाद होती है ।
• यसद RE सोंपसत्त दस्तावेज़ जारी करने में दे री करता है या ऋि सनपटान के 30 सदनोों के बाद चाजध सोंतुसष्ट् फॉमध
दाक्कखल करने में सवफल रहता है , तो उन्ें ऋिकताध को कारि बताना होगा।
• यसद दे री RE की गलती है , तो उन्ें दे री के प्रत्येक सदन के सलए ऋिकताध को 5,000 रुपये का मुआवजा दे ना
होगा।
i.RBI ने RBI (क्लासससफकेशन, वैल्ूएशन एों ड ऑपरे शन ऑफ़ इन्वेस्ट्मेंट पोटध फोसलयो ऑफ़ कमसशधयल बैंक्स )
सडरे क्शोंस, 2023 जारी करके उन्ें वैसश्वक मानकोों के साि सोंरेक्कखत करने के सलए बैंकोों िारा सनवेश को वगीकृत करने
पर अपने सदशासनदे शोों को भी सोंशोसर्त सकया, जो 1 अप्रैि, 2024 से प्रभावी होगा।
• मानदों डोों में बदलाव वासिक्कज्यक बैंकोों के सनवेश पोटध फोसलयो के वगीकरि, मूल्ाोंकन और सोंचालन के सलए
मौजूदा मानदों डोों के सोंबोंर् में 14 जनवरी, 2022 को चचाध पत्र (DP) में प्रस्तासवत सोंशोर्नोों के बाद सकया गया है।
• सोंशोसर्त सनदे शोों के कायाध न्वयन के साि, RBI (क्लासससफकेशन, वैल्ूएशन एों ड ऑपरे शन ऑफ़ इन्वेस्ट्मेंट
पोटध फोसलयो ऑफ़ कमसशधयल बैंक्स) सडरे क्शोंस, 2021 सदनाोंक 25 अगस्त, 2021 को सनरस्त कर सदया जाएगा।

35. निस िंपिी िे हाि ही में (नसतंबर'23 में) सूक्ष्म, िघु और मध्यम उद्यमों (MSME) िे निए आसाि नित्तपोषण
नििल्प प्रदाि िरिे िे निए भारतीय िघु उद्योग नििास बैंि (SIDBI) िे साथ एि MoU पर हस्ताक्षर निए हैं ?
1)सुजलॉन एनजी सलसमटे ड
2)टाटा पावर सोलर ससस्ट्म्स सलसमटे ड
3)रीन्यू सलसमटे ड
4)एज़्योर पावर ग्लोबल सलसमटे ड
5)NTPC ग्रीन एनजी सलसमटे ड
उत्तर-2)टाटा पािर सोिर नसस्ट्म्स निनमटे ि
स्पष्टीिरण:
सूक्ष्म, लघु और मध्यम उद्यमोों (MSME) के सलए एक आसान सवत्तपोषि सवकल्प प्रदान करने के सलए टाटा पावर सोलर
ससस्ट्म्स सलसमटे ड (TPSSL), और भारतीय लघु उद्योग सवकास बैंक (SIDBI) के बीच एक समझौता ज्ञापन (MoU) पर
हस्ताक्षर सकए गए।
• TPSSL टाटा पावर ररन्यूएबल एनजी सलसमटे ड (TPREL) की पूिध स्वासमत्व वाली सहायक कोंपनी है जबसक TPREL
टाटा पावर की सहायक कोंपनी है।
• TPSSL और SIDBI, SIDBI की 4E (एों ड टू एों ड एनजी एसफसशएों सी) योजना के माध्यम से अनुकूसलत सवत्तपोषि
समार्ान प्रदान करके MSME के बीच सौर ऊजाध अपनाने को प्रोत्सासहत करें गे।
• इस पहल का उद्दे श्य CO2 पदसचह्न को कम करना और भारत के स्वच्छ ऊजाध लक्ष्ोों में योगदान करना है।

Report Errors in the PDF - ebooks@affairscloud.com Copyright 2014-2023 @ AffairsCloud.com 77


36. उस िंपिी िा िाम बताइए निसिे हाि ही में (नसतंबर'23 में) हाइपरUPI िॉन्च िरिे िे निए यस बैंि िे
साथ साझेदारी िी है , िो िेशिि पेमेंट्स िॉरपोरे शि ऑफ इं निया (NPCI) द्वारा नििनसत एि प्लग-इि
सॉफ्टिेयर िे ििपमेंट टू िनिट (SDK) है।
1)पेटीएम
2)जसपे टे क्नोलॉजीज
3)फोनपे
4)भारतपे
5)रे जरपे सॉफ्टवेयर
उत्तर- 2)िसपे टे क्नोिॉिीि
स्पष्टीिरण:
यस बैंक सलसमटे ड (YES बैंि) ने नेशनल पेमेंट्स कॉरपोरे शन ऑफ इों सडया (NPCI) िारा सवकससत एक प्लग-इन
सॉफ्टवेयर डे वलपमेंट टू लसकट (SDK) हाइपरUPI लॉन्च करने के सलए भारत की अग्रिी भुगतान कोंपनी िुस्पे
टे क्नोिॉिीि प्राइिेट निनमटे ि (िुसपे) के साि साझेदारी की, जो मचेंट ऐप के सलए इन-ऐप यूसनफाइड पेमेंट्स
इों टरफेस (UPI) भुगतान को सशक्त बनाता है।
• इसके साि, व्यापारी अब ग्राहकोों को परे शानी मुक्त एक-क्कक्लक UPI अनुभव प्रदान कर सकते हैं । इससे ग्राहकोों
को मचेंट ऐप के भीतर UPI-आर्ाररत लेनदे न करने की भी अनुमसत समलेगी।
• यह प्लग-इन SDK भारत में सबसे सवश्वसनीय UPI-ऑन-क्लाउड स्ट्ै क में से एक पर बनाया गया है , जो कई
उपभोक्ता और व्यापारी ऐप्स के सलए UPI भुगतान का भी समिधन करता है।
• गुल्लक, एक अग्रिी सफनटे क ऐप, जसपे के UPI प्लगइन SDK, हाइपरUPI को तैनात करने वाला पहला व्यापारी
बन गया है।
• NPCI भारत में एक मजबूत भुगतान और सनपटान बुसनयादी ढाोंचा बनाने के सलए भुगतान और सनपटान प्रिाली
असर्सनयम, 2007 के प्रावर्ानोों के तहत भारतीय ररजवध बैंक (RBI) और भारतीय बैंक सोंघ (IBA) की एक पहल
है।

37. हाि ही में (नसतंबर'23 में) निस बैंि िे निस्कपे अिाउं ट-टू -अिाउं ट (A2A) शुरू िरिे िे निए गोनिस्क
टे क्नोिॉिीि प्राइिेट निनमटे ि िे साथ साझेदारी िी है ?
1)एक्कक्सस बैंक
2)यस बैंक
3)ICICI बैंक
4)HDFC बैंक
5)इों डसइों ड बैंक
उत्तर- 2)यस बैंि
स्पष्टीिरण:
मुोंबई (महाराष्ट्र) क्कथित भुगतान सफनटे क कोंपनी गोसब्रस्क टे क्नोलॉजीज प्राइवेट सलसमटे ड (निस्कपे) और यस बैंि ने
सब्रस्कपे अकाउों ट-टू -अकाउों ट (A2A) पेश करने के सलए साझेदारी की है , जो सनयाधतकोों और आयातकोों को सशक्त बनाने
के सलए सडज़ाइन सकया गया एक अत्यार्ुसनक समार्ान है।
i.यह सहयोग सीमा पार भुगतान की दु सनया में एक महत्वपूिध मील का पत्थर दशाधता है , सजससे सवसनमाधि और सेवा क्षेत्रोों
में कायधरत सूक्ष्म, लघु और मध्यम उद्यमोों (MSME) को लाभ होगा।

Report Errors in the PDF - ebooks@affairscloud.com Copyright 2014-2023 @ AffairsCloud.com 78


िोट: सब्रस्कपे ने वैंकूवर, सब्रसटश कोलोंसबया, कनाडा में क्कथित अपनी सहायक कोंपनी सब्रस्कपे टे क्नोलॉजीज इों क की थिापना
के माध्यम से अपनी पहुोंच को सीमाओों से परे बढाया है।

38. हाि ही में (नसतंबर'23 में) निस िंपिी िे भारत िा पहिा संपिा रनहत भुगताि पहििे योग्य ON-THE-
GO(OTG) ररं ग, 7 ररं ग िॉन्च िरिे िे निए िेशिि पेमेंट िॉरपोरे शि ऑफ इं निया (NPCI) िे साथ सहयोग
निया है ?
1)फाइनेंसपीयर प्राइवेट सलसमटे ड
2)जसपे टे क्नोलॉजीज प्राइवेट सलसमटे ड
3)सफसवध प्राइवेट सलसमटे ड
4)परसफयोस सॉफ्टवेयर सॉल्ूशोंस
5)सलवक्कक्वक टे क्नोलॉजी (इों सडया) प्राइवेट सलसमटे ड
उत्तर-5)नििखिि टे क्नोिॉिी (इं निया) प्राइिेट निनमटे ि
स्पष्टीिरण:
नेशनल पेमेंट कॉरपोरे शन ऑफ इों सडया (NPCI) ने 7 ररं ग नामक रुपे ON-THE-GO(OTG) ररों ग लॉन्च की है , जो भारत
में बनी पहली सोंपकध रसहत भुगतान पहनने योग्य ररों ग है ।
i.इस अोंगूठी का अनावरि ग्लोबल सफनटे क फेस्ट्, 2023 (GFF 2023)में सकया गया, जो 5 से 7 ससतोंबर 2023 के दौरान
मुोंबई, महाराष्ट्र में आयोसजत सकया गया िा।
ii.7 ररों ग को भारत में सडज़ाइन, असेंबल, सनसमधत और पेटेंट कराया गया है। यह अोंगूठी 7 (सेवेनररों ग इनोवेशन प्राइवेट
सलसमटे ड) और नििखिि टे क्नोिॉिी (इं निया) प्राइिेट निनमटे ि के सहयोग से बनाई गई िी।
iii.7 ररों ग एक सचकनी, पहनने योग्य अोंगूठी है जो सज़रकोसनया ससरे समक (ZrO2)नामक एक बहुत मजबूत और खरोोंच-
प्रसतरोर्ी सामग्री से बनी है सजसका उपयोग एयरोस्पेस अनुप्रयोगोों में भी सकया जाता है।

39. उस बैंि िा िाम बताइए निसिे हाि ही में (नसतंबर'23 में) िीज़ा और िसपे िे सहयोग से एि "िचुाअि
िमनशायि िािा " िॉन्च निया है।
1)एक्कक्सस बैंक
2)ICICI बैंक
3)इों डसइों ड बैंक
4)HDFC बैंक
5)YES बैंक
उत्तर-3)इं िसइं ि बैंि
स्पष्टीिरण:
इं िसइं ि बैंि ने िीज़ा और िसपे के साि साझेदारी में काडध र्ारकोों को कॉरपोरे ट् स और टर ै वल एजेंटोों के सलए सीमा
पार लेनदे न पर पूिध सनयोंत्रि दे ने के सलए सुरक्षा की असतररक्त परतोों के साि एक "िचुाअि िमनशायि िािा " लॉन्च
सकया। यह काडध वीज़ा पेएबल्स ऑटोमेशन प्लेटफ़ॉमध पर बनाया गया है।
• यह काडध उन ग्राहकोों को लाभ पहुोंचाता है जो सवसभन्न सवदे शी मुद्राओों में एकासर्क बुसकोंग करते हैं ।
• काडध उच्च स्तर का सनयोंत्रि, कम जसटलता और पूिध पारदसशधता प्रदान करता है ।
• काडध र्ारक वचुधअल काडध तैयार कर सकते हैं और इसकी साख जैसे काडध नोंबर, काडध सत्यापन मूल् (CVV)
कोड और समाक्कप्त सतसियोों को अनुकूसलत कर सकते हैं।

Report Errors in the PDF - ebooks@affairscloud.com Copyright 2014-2023 @ AffairsCloud.com 79


40. नसतंबर 2023 में भारतीय ररज़िा बैंि (RBI) िी हाि ही में िारी िे संबंध में निम्ननिखित में से िौि सा/से
नबंदु "सही" है/हैं?
A)RBI िे िषा 2023-24 िे निए NBFC िे निए स्केि आधाररत निनियमि िे तहत अपर िेयर (UL)/NBFC-
UL में 15 गैर-बैंनिंग नित्तीय िंपनियों (NBFC) िे िाम िारी निए।
B)NBFC-UL (अपर िेयर) में NBFC शानमि हैं निन्ें निशेष रूप से RBI द्वारा मापदं िों और स्कोररं ग पिनत िे
आधार पर बढ़ी हुई नियामि आिश्यिताओं िी गारं टी िे रूप में पहचािा िाता है।
C)TMF नबििेस सनिासेि निनमटे ि, पूिा में टाटा मोटसा फाइिेंस निनमटे ि, स्कोररं ग पिनत िे आधार पर NBFC-
UL (िॉि-बैंनिंग फाइिेंनशयि िंपिी - अपर िेयर) िे रूप में अहाता प्राप्त िरती है।
1)केवल A
2)केवल A & B
3)केवल B & C
4)केवल A & C
5)सभी A, B & C
उत्तर -5)सभी A, B & C
स्पष्टीिरण:
15 ससतोंबर, 2023 को, भारतीय ररजवध बैंक (RBI) ने वषध 2023-24 के सलए NBFC के सलए स्केि आधाररत निनियमि
के तहत अपर लेयर (UL)/NBFC-UL में 15 गैर-बैंसकोंग सवत्तीय कोंपसनयोों (NBFC) के नाम जारी सकए। यहाों क्कक्लक करें
2023-24 के सलए NBFC-UL की सूची यहाों दी गई है।.
i.अरू बर 2021 में, RBI ने स्केल आर्ाररत सवसनयमन (SBR) जारी सकया: NBFC के सलए एक सोंशोसर्त सनयामक ढाोंचा
(ढाोंचा) जो NBFC को उनकी सोंपसत्त के आकार और स्कोररों ग पद्धसत के अनुसार सनम्नसलक्कखत में वगीकृत करता है :
• NBFC-BL (बेस लेयर): इसमें मुख्य रूप से 1,000 करोड रुपये से कम सोंपसत्त वाली गैर-जमा स्वीकार करने
वाली NBFC शासमल हैं ।
• NBFC-ML (समसडल लेयर): इसमें 1,000 करोड रुपये से असर्क की सोंपसत्त वाली सभी जमा लेने वाली NBFC
और गैर-जमा लेने वाली NBFC और सनम्नसलक्कखत गसतसवसर्याों करने वाली NBFC शासमल हैं।
• NBFC-UL (अपर िेयर): अपर लेयर में NBFC शासमल हैं सजन्ें सवशेष रूप से RBI िारा मापदों डोों और स्कोररों ग
पद्धसत के आर्ार पर बढी हुई सनयामक आवश्यकताओों की गारों टी के रूप में पहचाना जाता है।
• सोंपसत्त के आकार के मामले में शीषध 10 पात्र NBFC अन्य कारकोों की परवाह सकए सबना हमेशा ऊपरी स्तर पर
रहेंगे।
• NBFC-TL (टॉप लेयर): यसद RBI सोंभासवत प्रिालीगत जोक्कखम में पयाधप्त वृक्कद्ध को पहचानता है तो NBFC-UL
को टॉप लेयर में ले जाया जाएगा। सफलहाल, टॉप लेयर आदशध रूप से खाली रहेगी।
• TMF नबििेस सनिासेि निनमटे ि, पूवध में टाटा मोटसध फाइनेंस सलसमटे ड, स्कोररों ग पद्धसत के आर्ार पर NBFC-
UL (नॉन-बैंसकोंग फाइनेंसशयल कोंपनी - अपर लेयर) के रूप में अहधता प्राप्त करती है।
• हालााँसक, चल रहे व्यवसाय पुनगधठन के कारि नवीनतम समीक्षा में इसे वतधमान में NBFC-UL के रूप में सूचीबद्ध
नहीों सकया गया है ।

Report Errors in the PDF - ebooks@affairscloud.com Copyright 2014-2023 @ AffairsCloud.com 80


41. उस बैंि िा िाम बताइए निसिे हाि ही में (नसतंबर'23 में) सभी िेंद्रों पर प्रनतनदि िम से िम चार घंटे
निनित आउटिेटों पर बुनियादी बैंनिंग से िाएं प्रदाि िरिे िे निए 'SAATHI (समग्र समािेशि िे निए सतत
पहुंच और संरेखित प्रौद्योनगिी)' िॉन्च निया है।
1)पोंजाब नेशनल बैंक
2)बैंक ऑफ बडौदा
3)इों सडयन बैंक
4)भारतीय स्ट्े ट बैंक
5)बैंक ऑफ इों सडया
उत्तर-3)इं नियि बैंि
स्पष्टीिरण:
इं नियि बैंि ने सबजनेस कॉरे स्पोोंडेंट (BC) चैनल के माध्यम से सभी केंद्रोों पर प्रसतसदन कम से कम चार घोंटे सनसित
आउटलेटोों पर बुनियादी बैंनिंग सेिाएं प्रदान करने के सलए IB (इं नियि बैंि) SAATHI (समग्र समावेशन के सलए
सतत पहुोंच और सोंरेक्कखत प्रौद्योसगकी) नामक एक नई पहल शुरू की है।
i.यह पहल शाोंसत लाल जैन िारा शुरू की गई जो चेन्नई, तसमलनाडु में इों सडयन बैंक के प्रबोंर् सनदे शक (MD) और मुख्य
कायधकारी असर्कारी (CEO) हैं।
ii.यह कायधक्रम ग्राहकोों के सलए बैंसकोंग सेवाओों तक पहुोंच और उपयोग को आसान बनाता है , और यह ग्राहकोों को
बुसनयादी और मूल् वसर्धत दोनोों सेवाएों भी प्रदान करता है।
iii.इस पहल को लागू करने के सलए, इों सडयन बैंक को 15 कॉपोरे ट सबजनेस कॉरे स्पॉन्डें ट्स (CBC) के साि 5,000 से
असर्क नव सनयुक्त नए BC (माचध 2024 तक) की आवश्यकता है।
• वतधमान में, उनके पास केवल 10,750 BC और 10 CBC हैं।

42. निस िघु नित्त बैंि (SFB) िे हाि ही में (नसतंबर’23 में) मध्यम और बडी व्यािसानयि संस्थाओं िे निए
मैखक्समा सेनिंग्स अिाउं ट और नबििेस मैखक्समा िरं ट अिाउं ट िॉन्च निया है ?
1)सूयोदय SFB
2)AU SFB
3)इक्कक्वटास SFB
4)उज्ज्जीवन SFB
5)उत्कषध SFB
उत्तर-4)उज्ज्िीिि SFB
स्पष्टीिरण:
उिीवन स्मॉल फाइनेंस बैंक सलसमटे ड (उज्ज्िीिि SFB) ने मैखक्समा सेनिंग्स अकाउों ट और नबििेस मैखक्समा िरं ट
अिाउं ट लॉन्च सकया है जो सवशेष रूप से मध्यम और बडी व्यावसासयक सोंथिाओों के सलए सडज़ाइन सकया गया है।
• दोनोों खाते एक RuPay सेलेर डे सबट काडध की पेशकश करते हैं जो कल्ाि सेवाओों, सवशेष व्यापारी ऑफ़र,
हवाई अड्डे के लाउों ज तक पहुोंच और एक प्रीसमयम स्वास्थ्य जाोंच तक पहुोंच प्रदान करता है।
• मैक्कक्समा बचत खाते पर वासषधक ब्याज दर 7.5% तक है और ग्राहक 1 लाख रुपये से अकाउों ट खोल सकते हैं।
• शेष रासश पात्रता मानदों ड को पूरा करने के सलए सावसर् जमा (FD) में 15 लाख रुपये या उससे असर्क बनाए
रखने की लचीलापन (यह सवकल्प अन्य मानक सेसवोंग्स अकाउों ट के साि उपलब्ध नहीों है )।

Report Errors in the PDF - ebooks@affairscloud.com Copyright 2014-2023 @ AffairsCloud.com 81


43. निस बैंि िे हाि ही में (नसतंबर'23 में) ििीिरणीय ऊिाा पररयोििाओं िे निए सह-उधार और ऋण
नसंनििेशि िो बढ़ािा दे िे और सुनिधा प्रदाि िरिे िे निए भारतीय ििीिरणीय ऊिाा नििास एिेंसी
(IREDA) िे साथ एि MoU पर हस्ताक्षर निए हैं ?
1)बैंक ऑफ बडौदा
2)बैंक ऑफ इों सडया
3)बैंक ऑफ महाराष्ट्र
4)यूसनयन बैंक ऑफ इों सडया
5)केनरा बैंक
उत्तर-3)बैंि ऑफ महाराष्टर
स्पष्टीिरण:
भारतीय नवीकरिीय ऊजाध सवकास एजेंसी (IREDA) ने दे श भर में नवीकरिीय ऊजाध पररयोजनाओों के सलए सह-उर्ार
और ऋि ससोंसडकेशन को बढावा दे ने और सुसवर्ा प्रदान करने के सलए बैंक ऑफ महाराष्ट्र (BoM) के साि एक महत्वपूिध
समझौता ज्ञापन (MoU) पर हस्ताक्षर सकए हैं।
• इस समझौते के तहत, बैंक ऑफ महाराष्ट्र IREDA िारा जारी बाोंड (पेशकश के सनसदध ष्ट् सनयमोों और शतों के
अनुसार) में सनवेश कर सकता है।
• IREDA के अध्यक्ष और प्रबोंर् सनदे शक (CMD) प्रदीप कुमार दास की उपक्कथिसत में MoU पर हस्ताक्षर सकए गए।
• IREDA, नवीन और नवीकरिीय ऊजाध मोंत्रालय (MNRE) के प्रशाससनक सनयोंत्रि के तहत भारत सरकार का एक
समनी रत्न (श्रेिी - I) उद्यम है ।

44. उस बीमा िंपिी िा िाम बताइए निसिे हाि ही में (नसतंबर'23 में) दीघािानिि पूंिी प्रशंसा िे निए एि
िया फंि ऑफररं ग (NFO) इमनििंग इखिटी फंि िॉन्च निया है।
1)एक्साइड लाइफ इों श्योरें स
2)भारती AXA लाइफ इों श्योरें स
3)HDFC लाइफ इों श्योरें स
4)SBI लाइफ इों श्योरें स
5)मैक्स लाइफ इों श्योरें स
उत्तर-2)भारती AXA िाइफ इं श्योरें स
स्पष्टीिरण:
भारती AXA िाइफ इं श्योरें स ने भारती AXA िाइफ इमनििंग इखिटी फंि नामक अपनी नई फोंड पेशकश (NFO)
लॉन्च की। यह भारती AXA का पहला समड-कैप फोंड है , सजसका उद्दे श्य समड-कैप कोंपसनयोों के पोटध फोसलयो में सनवेश
के माध्यम से दीघािानिि पूंिी प्रशोंसा प्रदान करना है ।
i.भारती AXA लाइफ इों श्योरें स ने 13 साल बाद लॉन्च सकया समड-कैप फोंड; समड-कैप फोंड की आक्कखरी लॉक्कन्चोंग 2010
में हुई िी।
ii.सनवेशक भारती AXA लाइफ के तीन यूसनट सलोंक्ड इों श्योरें स प्लान (ULIP) के माध्यम से भारती AXA लाइफ के इमसजिंग
इक्कक्वटी फोंड में सनवेश कर सकते हैं :
• भारती AXA लाइफ वेल्थ प्रो
• भारती AXA लाइफ ग्रो वेल्थ
• भारती AXA लाइफ वेल्थ मैक्कक्समाइज़र

Report Errors in the PDF - ebooks@affairscloud.com Copyright 2014-2023 @ AffairsCloud.com 82


iii.NFO सकसी सनवेश कोंपनी िारा लॉन्च सकए जा रहे सकसी भी फोंड के सलए पहली सदस्यता पेशकश है और NFO के
दौरान, सनवेशक म्यूचुअल फोंड योजना की इकाइयोों को अोंसकत मूल् पर खरीद सकते हैं , जो आमतौर पर 10 रुपये प्रसत
यूसनट की एक सनसित कीमत पर सनर्ाधररत सकया जाता है।

45. निस बैंि िे हाि ही में (नसतंबर'23 में) अनििासी भारतीयों (NRI) िे निए गैर-आिासीय बाहरी (NRE)
और अनििासी साधारण (NRO) िाते आसािी से िोििे िे निए एि निनिटि सुनिधा शुरू िी है ?
1)भारतीय स्ट्े ट बैंक
2)बैंक ऑफ इों सडया
3)पोंजाब नेशनल बैंक
4)बैंक ऑफ बडौदा
5)इों सडयन बैंक
उत्तर- 1)भारतीय स्ट्े ट बैंि
स्पष्टीिरण:
भारतीय स्ट्े ट बैंक (SBI) ने असनवासी भारतीयोों (NRI) के सलए SBI के मोबाइल ऐप YONO के माध्यम से गैर-आवासीय
बाहरी (NRE) और असनवासी सार्ारि (NRO) िाते (बचत और चालू खाते दोनोों) आसानी से खोलने के सलए एक
सडसजटल सुसवर्ा शुरू की है।
i.यह सेवा नए बैंक (NTB) ग्राहिों के सलए NRI बैंसकोंग आवश्यकताओों के सलए वन-स्ट्ॉप समार्ान के रूप में सडज़ाइन
की गई है तासक उन्ें सरल, कुशल और सटीक खाता खोलने की प्रसक्रया प्रदान की जा सके।
ii.एक NRE खाता भारत में एक NRI के नाम पर उसकी सवदे शी कमाई के सलए खोला जाता है , जबसक एक NRO खाता
भारत में असजधत आय, जैसे सकराया, लाभाोंश, पेंशन और ब्याज के प्रबोंर्न के सलए है ।
iii.जून 2023 तक, बैंक का जमा आर्ार 45.31 लाख करोड रुपये से असर्क है , सजसमें CASA (चालू खाता और बचत
खाता) अनुपात 42.88% और असग्रम 33 लाख करोड रुपये से असर्क है ।

46. लसतंबर 2023 में िारी भारतीय ररज़वा बैंक (RBI) के लवत्तीय पररसंपलत्तयों के प्रवाह और घरे िू दे नदाररयों के
िे टा के संबंि में लनम्नलिस्कित में से कौन सा/से लबंदु "सही" है /हैं ?
A) पररवारों की शुि लवत्तीय बचत FY23 में GDP के िगभग सात दशक के लनचिे स्तर 6.1% पर आ गई, िो
FY22 में 7.2% थी।
B) राष्टरीय लवत्तीय स्कस्वच (NFS) का उपयोग FY221 में 11.5% से घटकर FY23 में 5.1% हो गया।
C) घरे िू लवत्तीय दे नदाररयों में पयााप्त वृस्कि दे िी गई, िो FY23 में GDP का 5.8% हो गई, िो FY22 में 3.8%
थी।
1)केवल A
2)केवल A & B
3)केवल B & C
4)केवल A & C
5)सभी A, B & C
उत्तर-3)केवि B & C

Report Errors in the PDF - ebooks@affairscloud.com Copyright 2014-2023 @ AffairsCloud.com 83


स्पष्टीकरण:
पररवारों की शुि लवत्तीय बचत FY23 में GDP के 5.1% के लगभग पांच दशक के लनचिे स्तर पर आ गई, िो FY22
में 7.2% से कम है। भारतीय ररिवग बैंक (RBI) के आं कड़ों के अनुसार, यह श्चपछले श्चवत्त वषग की तुलना में उल्लेखनीय
कमी को श्चचश्चित करता है, िहां यह 7.2% (FY22) िा।
• श्चसतंबर 2023 को िारी RBI के घरे लू श्चवत्तीय संपश्चत्तयों और दे नदाररयों के प्रवाह के आं कड़ों के अनुसार, राष्ट्रीय
श्चवत्तीय स्स्वच (NFS) का उपयोग FY221 में 11.5% से घटकर FY23 में 5.1% हो गया।
• घरे िू लवत्तीय दे नदाररयों में पयागप्त वृस्ि दे खी गई, िो FY23 में GDP का 5.8% हो गई, िो FY22 में 3.8% िी।
इससे पता चलता है श्चक पररवार अपनी उपभोग िरूरतों को पूरा करने के श्चलए उर्ार पर श्चनभगर हो गए हैं।

47. लनम्नलिस्कित में से कौन से सहकारी बैंक को हाि ही में (लसतंबर’23 में) भारतीय ररज़वा बैंक (RBI) ने बैंलकंग
लवलनयमन अलिलनयम, 1949, लवशेष रूप से िारा 47 A (1)I के साथ िारा 46 (4)(i) और 56 के तहत अपने
अलिकार का हवािा दे ने के लिए मौलद्रक दं ि िगाया है।?
1)अभ्युदय सहकारी बैंक श्चलश्चमटे ि (महाराष्ट्र)
2)लालबाग सहकारी बैंक श्चलश्चमटे ि (गुिरात)
3)मेहसािा श्चलश्चमटे ि के सहकारी बैंक (गुिरात)
4)हररि नागररक सहकारी बैंक श्चलश्चमटे ि (गुिरात)
5)राष्ट्रीय सहकारी बैंक श्चलश्चमटे ि (महाराष्ट्र)
उत्तर -1)अभ्युदय सहकारी बैंक लिलमटे ि (महाराष्टर)
स्पष्टीकरण:
भारतीय ररज़वग बैंक (RBI) ने बैंलकंग लवलनयमन अलिलनयम, 1949, श्चवशेष रूप से र्ारा 47 A (1)I के साि र्ारा 46
(4)(i) और 56 के तहत अपने अश्चर्कार का हवाला दे ते हुए चार सहकारी बैंकों पर मौलद्रक दं ि लगाया है।
• िािबाग सहकारी बैंक लिलमटे ि (गुिरात) - '5 लाख रुपये के श्चलए अन्य बैंकों के साि िमा राश्चश रखने' और
'िमा श्चदशा श्चनदे श, 2016 पर ब्याि दर' पर आरबीआई के श्चनदे शों का अनुपालन न करना है।
• सहकारी बैंक ऑफ मेहसाणा लिलमटे ि (गुिरात) - '3.50 लाख रुपये के श्चलए श्चनदे शकों आश्चद को ऋि और
अश्चग्रम - ज़मानत या गारं टर के रूप में श्चनदे शक - स्पष्ट्ीकरि' और 'अन्य बैंकों के साि िमा की श्चनयुस्क्त' पर
आरबीआई के श्चनदे शों का अनुपालन न करना है।
• हाररि नागररक सहकारी बैंक लिलमटे ि (गुिरात) - '3 लाख रुपये के श्चलए नकद आरश्चक्षत अनुपात
(सीआरआर) के रखरखाव', 'अन्य बैंकों के साि िमा राश्चश की श्चनयुस्क्त', और 'िमा पर ब्याि दर - श्चदशाश्चनदे श,
2016' पर आरबीआई के श्चनदे शों का अनुपालन न करना है।
• राष्टरीय सहकारी बैंक लिलमटे ि (महाराष्टर) - 1 लाख रुपये के श्चलएिमा खातों-प्रािश्चमक (शहरी) सहकारी बैंकों
के रखरखाव पर आरबीआई के श्चनदे शों का अनु पालन न करना है।

48. लसतंबर 2023 में, मेटा ने भारत में व्हाट् सएप लबिनेस के लिए यूलनफाइि पेमेंट इं टरफेस (UPI) भुगतान
शुरू करने की घोषणा की।
लनम्नलिस्कित में से कौन सी कंपलनयां प्रारं लभक भुगतान भागीदार हैं?
1)भारतपे
2)रे िरपे
3)पेयू
4)1 और 2 दोनों

Report Errors in the PDF - ebooks@affairscloud.com Copyright 2014-2023 @ AffairsCloud.com 84


5)2 और 3 दोनों
उत्तर- 5)2 और 3 दोनों
स्पष्टीकरण:
मेटा (श्चिसे पहले फेसबुक के नाम से िाना िाता िा) के मुख्य कायगकारी अश्चर्कारी (CEO) माकग िुकरबगग ने भारत में
व्हाट् सएप लबिनेस के श्चलए यूश्चनफाइि पेमेंट इं टरफेस (UPI) पेमेंट्स शुरू करने की घोषिा की। यह ग्राहकों को
अपने पसंदीदा UPI श्चवकल्प का उपयोग करके व्हाट् सएप चैट एस्िकेशन पर सीर्े व्यवसायों को भुगतान करने की
अनुमश्चत दे ता है।
i.महाराष्ट्र के मुंबई में श्चियो वर्ल्ग कन्वेंशन सेंटर में आयोश्चित व्हाट् सएप श्चबिनेस पर केंश्चद्रत मेटा कन्वसेशन इवेंट में यह
घोषिा की गई।
ii.स्वीकृत भुगतान श्चवश्चर्यों में व्हाट् सएप, क्रेश्चिट / िे श्चबट कािग , नेटबैंश्चकंग, गूगल पे, फोनपे, पेटीएम और अश्चर्क UPI
श्चवकल्प शाश्चमल हैं।
iii.प्रारं श्चभक पेमेंट भागीदार रे िरपे और पेयू हैं।

49. उस कंपनी का नाम बताइए लिसने हाि ही में (लसतंबर’23 में) 1,18,826 करोड रुपये की नवीकरणीय और
थमाि ऊिाा पररयोिनाओं को लवत्तपोलषत करने के लिए पावर फाइनेंस कॉपोरे शन लिलमटे ि (PFC) के साथ एक
MoU पर हस्ताक्षर लकए हैं।
1)SJVN श्चलश्चमटे ि
2)रूरल इलेस्रर श्चफकेशन कॉपोरे शन श्चलश्चमटे ि
3)NLC इं श्चिया श्चलश्चमटे ि
4)नेशनल िमगल पावर कॉपोरे शन श्चलश्चमटे ि
5)पावर श्चग्रि कॉपोरे शन ऑफ इं श्चिया
उत्तर- 1)SJVN लिलमटे ि
स्पष्टीकरण:
SJVN लिलमटे ि (श्चिसे पहले सतलुि िल श्चवद् युत श्चनगम श्चलश्चमटे ि के नाम से िाना िाता िा) ने 1,18,826 करोड रुपये
की नवीकरिीय और िमगल ऊिाग पररयोिनाओं को श्चवत्तपोश्चषत करने के श्चलए भारत के अग्रिी गैर-बैंश्चकंग श्चवत्तीय श्चनगम
(NBFC) पावर फाइनेंस कॉपोरे शन श्चलश्चमटे ि (PFC) के साि एक समझौता ज्ञापन (MoU) पर हस्ताक्षर श्चकए हैं ।
i.इस पररयोिना में लगभग 12178 मेगावाट (MW) की कुल क्षमता के साि 660 MW िमगल उत्पादन पररयोिना के
साि नवीकरिीय ऊिाग (RE) उद्यम (सौर, हाइिर ो और पंप स्ट्ोरे ि) शाश्चमल हैं।
ii.PFC पररयोिना का लगभग 67-76% अथिायी रूप से श्चवत्तपोश्चषत करे गा। टमग लोन के रूप में प्रस्ताश्चवत फंि 80000
करोड़ रुपये से 90000 करोड़ रुपये तक होगा।

50. भारत िे निस नित्तीय संस्थाि/नियामि नििाय िे हाि ही में (नसतंबर’23 में) उत्तर प्रदे श ििीि और
ििीिरणीय ऊिाा नििास एिेंसी (UPNEDA) िे साथ अयोध्या, उत्तर प्रदे श में आिासीय इिाइयों िे सौरिरण
और िैि-ऊिाा और ििीिरणीय पररयोििा िे नित्तपोषण िे निए एि MoU पर हस्ताक्षर निए हैं ?
1)भारतीय ररजवध बैंक
2)भारतीय सनयाधत-आयात बैंक
3)राष्ट्रीय कृसष और ग्रामीि सवकास बैंक
4)भारतीय लघु उद्योग सवकास बैंक
5)भारतीय प्रसतभूसत एवों सवसनमय बोडध

Report Errors in the PDF - ebooks@affairscloud.com Copyright 2014-2023 @ AffairsCloud.com 85


उत्तर-4)भारतीय िघु उद्योग नििास बैंि
स्पष्टीिरण:
उत्तर प्रदे श नवीन और नवीकरिीय ऊजाध सवकास एजेंसी (UPNEDA) और भारतीय लघु उद्योग सवकास बैंक (SIDBI)
के बीच अयोध्या, उत्तर प्रदे श (UP) में आवासीय इकाइयोों के सौरकरि और िैि-ऊिाा और ििीिरणीय पररयोििा
नित्तपोषण के सलए एक समझौता ज्ञापन (MoU) पर हस्ताक्षर सकए गए हैं।
i.यह समझौता UP में SIDBI के लखनऊ कायाधलय में आयोसजत एक कायधक्रम के दौरान सकया गया।
ii.यह MoU सौर ऊजाध को बढावा दे ने और पयाधवरि अनुकूल पररयोजनाओों के सलए नवाचारोों को प्रोत्सासहत करने में
मदद करे गा।
iii.गौरतलब है सक UP सरकार ने अयोध्या को सोलर ससटी के रूप में सवकससत करने के सलए नासमत सकया है .
iv.कायधक्रम के दौरान, सूक्ष्म, लघु और मध्यम उद्यमोों (MSME) क्षेत्र के सलए 'सौर/हररत पररयोजनाओों के सवत्तपोषि के
सलए प्रचार असभयान' के माध्यम से SIDBI िारा की जा रही सडसजटल और नवीनतम पहलोों के बारे में जागरूकता पैदा
करने पर जोर सदया गया।

51. निस बैंि िे हाि ही में (नसतंबर'23 में) 'NEO फॉर नबििेस', भारतीय सूक्ष्म, िघु और मध्यम उद्यमों
(MSME) िे निए एि निनशष्ट िि-स्ट्ॉप टर ांिैक्शि बैंनिंग प्लेटफॉमा बिाया है ?
1)ICICI बैंक
2)एक्कक्सस बैंक
3)HDFC बैंक
4)YES बैंक
5)इों डसइों ड बैंक
उत्तर- 2)एखक्सस बैंि
स्पष्टीिरण:
21 ससतोंबर, 2023 को, एखक्सस बैंि ने भारतीय सूक्ष्म, लघु और मध्यम उद्यमोों (MSME) के सलए एक सवसशष्ट् वन-स्ट्ॉप
टर ाोंजैक्शन बैंसकोंग प्लेटफॉमध 'NEO फॉर नबििेस' लॉन्च सकया।
i.यह अपनी तरह का पहला व्यापक सडसजटल प्रस्ताव है जो MSME की वास्तसवक, वतधमान और उभरती लेनदे न बैंसकोंग
आवश्यकताओों का समिधन करता है।
ii.इसमें बैंक-ग्रेड सुरक्षा सुसवर्ाओों के साि-साि समकालीन UX(उपयोगकताध अनुभव)/UI (उपयोगकताध इों टरफ़ेस) है।
iii."NEO फॉर सबजनेस" "NEO बाय एक्कक्सस बैंक" ब्राोंसडों ग के सहस्से के रूप में नवीनतम लॉन्च है। एक्कक्सस बैंक िारा
NEO अपने कॉपोरे ट बैंसकोंग ग्राहकोों/ग्राहकोों के सलए सडसजटल बैंसकोंग प्रस्ताव है।

52. उस स्मॉि फाइिेंस बैंि निनमटे ि (SFB) िा िाम बताइए निसिे हाि ही में (नसतंबर'23 में) ऑटोमेटेि
टे िर मशीि (ATM) पर यूनिफाइि पेमेंट इं टरफेस (UPI) िा उपयोग िरिे इं टरऑपरे बि िािा िेस िैश
निदिर ॉि (ICCW) सेिा शुरू िी है।
1)सूयोदय SFB
2)AU SFB
3)इक्कक्वटास SFB
4)उिीवन SFB
5)उत्कषध SFB
उत्तर-5)उत्कषा SFB

Report Errors in the PDF - ebooks@affairscloud.com Copyright 2014-2023 @ AffairsCloud.com 86


स्पष्टीिरण:
उत्कषध स्मॉल फाइनेंस बैंक सलसमटे ड (USFBL) ने उत्कषध स्मॉल फाइनेंस बैंक ऑटोमेटेड टे लर मशीन (ATM) से
यूसनफाइड पेमेंट इों टरफेस (UPI) सक्षम काडध लेस कैश सनकासी की अनुमसत दे ने के उद्दे श्य से इों टरऑपरे बल काडध लेस
कैश सवदडर ॉल (ICCW) सेवा शुरू की।
i.यह भुगतान और सनपटान प्रिाली असर्सनयम, 2007 (2007 का असर्सनयम 51)की र्ारा 18 के साि पसठत र्ारा 10(2)के
तहत 2022 में भारतीय ररजवध बैंक (RBI) िारा जारी असर्सूचना 'ATM पर ICCW' के अनुरूप है।
ii.ICCW को UPI-ATM के नाम से भी जाना जाता है ; यह त्वररत प्रसतसक्रया (QR) कोड के माध्यम से सनकासी को सक्षम
बनाता है।
iii.ATM के माध्यम से काडध -रसहत नकद सनकासी भारत में कुछ बैंकोों िारा ऑन-अस और ऑफ-अस आर्ार पर पेश
सकया जाने वाला लेनदे न का एक अनुमत तरीका है।

53. भारत िे निस नित्तीय संस्थाि/नियामि नििाय िे हाि ही में (नसतंबर'23 में) भारत िे नफिटे ि क्षेत्र िो
बढ़ािा दे िे िे निए निनिटि िेंिसा एसोनसएशि ऑफ इं निया (DLAI) िे साथ एि MoU पर हस्ताक्षर निए हैं ?
1)भारतीय ररजवध बैंक
2)भारतीय सनयाधत-आयात बैंक
3)राष्ट्रीय कृसष और ग्रामीि सवकास बैंक
4)भारतीय लघु उद्योग सवकास बैंक
5)भारतीय प्रसतभूसत एवों सवसनमय बोडध
उत्तर-4)भारतीय िघु उद्योग नििास बैंि
स्पष्टीिरण:
23 ससतोंबर, 2023 को, एक-दू सरे की सवशेषज्ञता का उपयोग करके भारत िे नफिटे ि क्षेत्र को बढावा दे ने के सलए
भारतीय लघु उद्योग सवकास बैंक (SIDBI) और सडसजटल लेंडसध एसोससएशन ऑफ इों सडया (DLAI) के बीच एक समझौता
ज्ञापन (MoU) पर हस्ताक्षर सकए गए हैं।
i.गोवा में DLAI िारा आयोसजत UnConvlave2023 कायधक्रम के दौरान MoU पर हस्ताक्षर सकए गए।
ii.MoU बैंक-उर्ार सेवा प्रदाता (LSP) गठबोंर्न और सह-उर्ार व्यवथिा ससहत सडसजटल ऋि साझेदारी के सलए मानक
प्रोटोकॉल बनाने पर केंसद्रत है।
iii.SIDBI पात्र DLAI सदस्य NBFC (गैर-बैंसकोंग सवत्तीय कोंपनी) को सोंसार्न सहायता प्रदान करे गा और असाइनमेंट और
प्रसतभूसतकरि लेनदे न का पता लगाएगा।

54. नसतंबर 2023 में भारतीय प्रनतभूनत और निनिमय बोिा (SEBI) द्वारा हाि ही में िी गई मंिूरी िे संबंध में
निम्ननिखित में से िौि सा/से नबंदु "सही" है /हैं?
A) SEBI िे मौिूदा LC ढांचे िे उपायों िे माध्यम से ऋण बािार से नित्तपोषण िी िरूरतों िो पूरा िरिे िे
निए बडे निगमों (LC) िे निए ढांचे में िचीिापि प्रदाि िरिे िे प्रस्ताि िो मंिूरी दे दी।
B) SEBI बोिा िे नििेश सिाहिारों िे निए बढ़ी हुई योग्यता और अिुभि आिश्यिताओं िे अिुपािि िी
समय सीमा नसतंबर 2023 से बढ़ािर नसतंबर 2025 िर दी।
C) SEBI बोिा िे ििा में िूबे नििेशिों िे निए ऋण-सूचीबि संस्थाओं (िंपनियों िो छोडिर), ररयि-एस्ट्े ट
इन्वेस्ट्मेंट टर स्ट्् स (REIT) और इं फ्रास्ट्र क्चर इन्वेस्ट्मेंट टर स्ट्् स (InvIT) में दािा ि िी गई रिम ति पहुंच िो
आसाि बिािे िे निए संशोधिों िो मंिूरी दे दी है।
1)केवल A

Report Errors in the PDF - ebooks@affairscloud.com Copyright 2014-2023 @ AffairsCloud.com 87


2)केवल A & B
3)केवल B & C
4)केवल A & C
5)सभी A, B & C
उत्तर -5)सभी A, B & C
स्पष्टीिरण:
भारतीय प्रसतभूसत और सवसनमय बोडध (SEBI) ने 21 ससतोंबर, 2023 को मुोंबई, महाराष्ट्र में आयोसजत अपनी 202वीों बोडध
बैठक के दौरान प्रसतभूसत बाजार में सवसभन्न रुझानोों और इसके सलए ससक्रय रूप से योजना बनाने के सलए SEBI के
दृसष्ट्कोि पर चचाध की। SEBI ने सनम्नसलक्कखत प्रस्तावोों को भी मोंजूरी दी:
i.SEBI ने मौजूदा LC ढाोंचे के उपायोों के माध्यम से ऋि बाजार से सवत्तपोषि की जरूरतोों को पूरा करने के सलए बडे
निगमों (LC) िे निए ढांचे में लचीलापन प्रदान करने के प्रस्ताव को मोंजूरी दे दी।
• LC को पररभासषत करने के सलए मौसद्रक सीमा बढाना है ।
• वतधमान सनयमोों के अनुसार, LC वे हैं सजनके पास कम से कम 100 िरोड रुपये की बकाया दीघधकासलक उर्ारी,
'AA और उससे ऊपर' की क्रेसडट रे सटों ग और दीघधकासलक उर्ार (एक वषध से असर्क) के साि खुद को सवत्तपोसषत
करने का लक्ष् होना चासहए।
ii.SEBI बोडध ने सनवेश सलाहकारोों के सलए बढी हुई योग्यता और अनुभव आवश्यकताओों के अनुपालन की समय सीमा
नसतंबर 2023 से बढाकर ससतोंबर 2025 कर दी और इन आवश्यकताओों को व्यक्कक्तयोों, प्रमुख असर्काररयोों और सनवेश
सलाहकारोों से जुडे व्यक्कक्तयोों पर लागू सकया।
iii.SEBI बोडध ने कजध में डूबे सनवेशकोों के सलए ऋि-सूचीबद्ध सोंथिाओों (कोंपसनयोों को छोडकर), ररयल-एस्ट्े ट इन्वेस्ट्मेंट
टर स्ट्् स (REIT) और इों फ्रास्ट्र क्चर इन्वेस्ट्मेंट टर स्ट्् स (InvIT) में दावा न की गई रकम तक पहुोंच को आसान बनाने के सलए
सोंशोर्नोों को मोंजूरी दे दी है ।
• इस सोंबोंर् में सनवेशक सोंरक्षि और सशक्षा सनसर् (IPEF) सवसनयम, सलक्कस्ट्ोंग दासयत्व और प्रकटीकरि आवश्यकताएाँ
(LODR) सवसनयम, InvIT सवसनयम और REIT सवसनयम में सोंशोर्न सकए जाएों गे।
• इसके साि, सनवेशक अपनी दावा न की गई रासश का दावा करने के सलए सीर्े सोंबोंसर्त सूचीबद्ध सोंथिाओों, REIT
और InvIT से सोंपकध कर सकते हैं , सजससे एक आसान दावा प्रसक्रया सुसनसित हो सकेगी।

55. नसतंबर 2023 में भारतीय ररििा बैंि (RBI) द्वारा उठाए गए हानिया िदम िे संबंध में निम्ननिखित में से
िौि सा नबंदु "सही" है /हैं ?
A) RBI िे द साहेबराि दे शमुि िो-ऑपरे नटि बैंि निनमटे ि, मुंबई, महाराष्टर िे द िॉसमॉस िो-ऑपरे नटि बैंि
निनमटे ि, पुणे, महाराष्टर िे साथ एिीिरण िी योििा िो मंिूरी दे दी।
B) RBI िे भारतीय स्ट्े ट बैंि (1.30 िरोड रुपये), इं नियि बैंि (1.62 िरोड रुपये) और बैंि ऑफ बडौदा (2
िरोड रुपये) पर मौनद्रि िुमाािा िगाया।
C) RBI िे मुंबई, महाराष्टर में एि गैर-बैंनिंग नित्तीय िंपिी (NBFC) फेिबैंि फाइिेंनशयि सनिासेि निनमटे ि
पर 8.80 िाि रुपये िा िुमाािा भी िगाया।
1)केवल A
2)केवल A & B
3)केवल B & C
4)केवल A & C
5)सभी A, B & C

Report Errors in the PDF - ebooks@affairscloud.com Copyright 2014-2023 @ AffairsCloud.com 88


उत्तर- 4)िेिि A & C
स्पष्टीिरण:
भारतीय ररज़वध बैंक (RBI) ने 21-25 ससतोंबर, 2023 तक सनम्नसलक्कखत सनिधय सलए:
• 25 ससतोंबर, 2023 को, RBI ने बैंसकोंग सवसनयमन (BR) असर्सनयम, 1949 की र्ारा 56 के साि पसठत र्ारा 44A
की उप-र्ारा (4)के तहत प्रदत्त अपनी शक्कक्त का प्रयोग करते हुए द साहेबराि दे शमुि िो-ऑपरे नटि बैंि
निनमटे ि, मुोंबई, महाराष्ट्र के द िॉसमॉस िो-ऑपरे नटि बैंि निनमटे ि, पुिे, महाराष्ट्र के साि समामेलन की
योजना को मोंजूरी दी।
• 21 ससतोंबर, 2023 को, RBI ने भारतीय स्ट्े ट बैंि (1.30 करोड रुपये), इं नियि बैंि(1.62 करोड रुपये), पंिाब
& नसंध बैंक(1 िरोड रुपये) पर BR असर्सनयम की र्ारा 46(4)(i) और 51 (1)के साि र्ारा 47A(1)(c) के
प्रावर्ानोों के तहत प्रदत्त शक्कक्तयोों का उपयोग करते हुए मौसद्रक जुमाधना लगाया।
• RBI ने NBFC (ररज़वध बैंक) सदशा-सनदे श, 2016 में र्ोखार्डी की सनगरानी के कुछ प्रावर्ानोों का अनुपालन न
करने के सलए मुोंबई में एक गैर-बैंसकोंग सवत्तीय कोंपनी (NBFC) फेडबैंक फाइनेंसशयल ससवधसेज सलसमटे ड पर 8.80
लाख रुपये का जुमाधना लगाया।
• 25 ससतोंबर, 2023 को RBI ने अपयाधप्त पूोंजी और कमाई की सोंभावनाओों के बीच कपोल को-ऑपरे सटव बैंक
सलसमटे ड, मुोंबई, महाराष्ट्र का लाइसेंस रद्द कर सदया है ।
• इसके बाद, तत्काल प्रभाव से BR असर्सनयम, 1949 की र्ारा 56 के साि पढी गई र्ारा 5 (b) में पररभासषत जमा
स्वीकार करने और चुकाने ससहत बैंसकोंग गसतसवसर्योों का सोंचालन करने से तुरोंत प्रसतबोंसर्त कर सदया गया है ।

56. नसतंबर 2023 में, पूिािािा नफििॉपा निनमटे ि िो _____________ (बैंि) िे साथ सह-िांिेि क्रेनिट िािा
िारी िरिे िे निए भारतीय ररििा बैंि (RBI) से मंिूरी नमिी।
1)YES बैंक
2)HDFC बैंक
3)RBL बैंक
4)एक्कक्सस बैंक
5)इों डसइों ड बैंक
उत्तर-5)इं िसइं ि बैंि
स्पष्टीिरण:
पूिािािा नफििॉपा निनमटे ि को इं िसइं ि बैंि सलसमटे ड के साि सह-ब्राोंडेड क्रेसडट काडध जारी करने के सलए भारतीय
ररजवध बैंक (RBI) िी मंिूरी समल गई है।
i.पूनावाला सफनकॉपध सलसमटे ड (पहले मैग्मा सफनकॉपध के नाम से जाना जाता िा) एक गैर-बैंसकोंग सवत्त कोंपनी (NBFC) है
जो साइरस पूनावाला समूह से सोंबोंसर्त है।
ii.सह-ब्राोंडेड क्रेसडट काडध को अनुकूल क्रेसडट इसतहास वाले ग्राहकोों को लाभ प्रदान करने के सलए सडज़ाइन सकया गया
है। क्रेसडट काडध 3 महीने के भीतर लॉन्च करने की तैयारी है।

57. निस बैंि िे हाि ही में (नसतंबर'23 में) 55,000 िरोड रुपये िे सह-नित्त बुनियादी ढांचा पररयोििा ऋण
िे निए REC निनमटे ि िे साथ एि MoU पर हस्ताक्षर निए हैं ?
1)बैंक ऑफ बडौदा
2)इों सडयन बैंक
3)पोंजाब नेशनल बैंक

Report Errors in the PDF - ebooks@affairscloud.com Copyright 2014-2023 @ AffairsCloud.com 89


4)बैंक ऑफ इों सडया
5)स्ट्े ट बैंक ऑफ इों सडया
उत्तर-3)पंिाब िेशिि बैंि
स्पष्टीिरण:
26 ससतोंबर, 2023 को, REC निनमटे ि (पूवध में ग्रामीि सवद् युतीकरि सनगम सलसमटे ड) और पंिाब िेशिि बैंि (PNB)
के बीच एक समझौता ज्ञापन (MoU) पर हस्ताक्षर सकए गए हैं , जो एक कोंसोसटध यम व्यवथिा के माध्यम से सबजली क्षेत्र
और इों फ्रास्ट्र क्चर एों ड लॉसजक्कस्ट्क्स क्षेत्र में सोंभासवत सवत्त पोषि के अवसरोों की तलाश करने के सलए सहयोग के सलए है ।
• दोनोों सोंथिाएों सोंयुक्त रूप से अगिे 3 िषों में 55,000 िरोड रुपये का सवत्तपोषि प्रदान करती हैं।
• REC सलसमटे ड 2030 तक हररत पररयोजनाओों के सलए अपने ऋि पोटध फोसलयो को 3 लाख करोड रुपये तक
सवस्ताररत करने का प्रयास कर रहा है ।
िोट: REC सलसमटे ड एक गैर-बैंसकोंग सवत्त कोंपनी (NBFC) है , जो पूरे भारत में सबजली क्षेत्र के सवत्तपोषि और सवकास पर
ध्यान केंसद्रत करती है।

58. निम्ननिखित में से निस एसेट मैिेिमेंट िंपिी (AMC) िे हाि ही में (नसतंबर'23 में) िेंद्रीय माध्यनमि
नशक्षा बोिा (CBSE) िे साथ नमििर "सीिो पैसों िी भाषा" िामि एि नििेशि नशक्षा और िागरूिता
पहि शुरू िी है।
1)HDFC एसेट मैनेजमेंट कोंपनी सलसमटे ड (HDFC AMC)
2)ICICI प्रूडेंसशयल एसेट मैनेजमेंट कोंपनी सलसमटे ड (ICICI प्रूडेंसशयल)
3)कोटक मसहोंद्रा एसेट मैनेजमेंट कोंपनी सलसमटे ड (KMAMCL)
4)एक्कक्सस एसेट मैनेजमेंट कोंपनी सलसमटे ड
5)सनप्पॉन इों सडया म्यूचुअल फोंड
उत्तर -3)िोटि मनहंद्रा एसेट मैिेिमेंट िंपिी निनमटे ि (KMAMCL)
स्पष्टीिरण:
िोटि मनहंद्रा एसेट मैिेिमेंट िंपिी निनमटे ि (KMAMCL) ने केंद्रीय माध्यसमक सशक्षा बोडध (CBSE) के सहयोग
से "सीखो पैसे की भाषा" नामक एक सनवेशक सशक्षा और जागरूकता पहल शुरू की है।
• पहल के सहस्से के रूप में, 75,000 सशक्षकोों (उनमें से 50% मसहला सशक्षक हैं ) को सवत्तीय ज्ञान से लैस सकया
जाएगा।
• इस कायधक्रम की दे खभाल सेंटर फॉर इन्वेस्ट्मेंट एजुकेशन एों ड लसनिंग (CIEL) के 500 कुशल प्रसशक्षकोों िारा
की जाएगी, जो मुोंबई, महाराष्ट्र में क्कथित है ।

59. निम्ननिखित में से निस म्यूचुअि फंि िंपिी िे हाि ही में (नसतंबर'23 में) भारत िा पहिा निफ्टी 500
ETF (एक्सचेंि टर े िेि फंि) िॉन्च निया है ?
1)बोंर्न म्युचुअल फोंड
2)आसदत्य सबडला सन लाइफ म्यूचुअल फोंड
3)UTI म्यूचुअल फोंड
4)मोतीलाल ओसवाल म्यूचुअल फोंड
5)SBI म्यूचुअल फोंड
उत्तर-4)मोतीिाि ओसिाि म्यूचुअि फंि

Report Errors in the PDF - ebooks@affairscloud.com Copyright 2014-2023 @ AffairsCloud.com 90


स्पष्टीिरण:
मोतीिाि ओसिाि म्यूचुअि फंि ने भारत का पहला सनफ्टी 500 ETF(एक्सचेंज टर े डेड फोंड) "मोतीलाल ओसवाल
सनफ्टी 500 EFT" लॉन्च सकया, जो सनफ्टी 500 कुल ररटनध इों डेक्स की नकल/टर ै सकोंग करने वाली एक ओपन-एों डे ड
योजना है।
• यह फोंड दीघधकासलक पूोंजी वृक्कद्ध चाहने वाले सनवेशकोों के सलए आदशध सवकल्प है।
• सनवेशक केवल 500 रुपये प्रसत योजना/सवकल्प के साि जुड सकते हैं और 1 रुपये के गुिक में सनवेश कर
सकते हैं , सजसमें कोई ऊपरी सनवेश सीमा नहीों है।

60. नसतंबर 2023 में भारतीय ररििा बैंि (RBI) द्वारा हाि ही में उठाए गए िदम िे संबंध में निम्ननिखित में से
िौि सा/से नबंदु "सही" है /हैं?
A) RBI िे RBI (अखिि भारतीय नित्तीय संस्थािों िे निए बेसि III पूंिी ढांचे, एक्सपोिर मािदं ि, महत्वपूणा
नििेश, िगीिरण, मूल्यांिि और नििेश पोटा फोनियो मािदं िों और संसाधि िुटािे िे मािदं िों पर नििेिपूणा
निनियम) नदशानिदे श, 2023 िारी निए िो 1 अप्रैि 2024 से िागू हो
B) ये RBI द्वारा RBI अनधनियम, 1934 िी धारा 45L द्वारा प्रदत्त शखक्तयों िा प्रयोग िरते हुए िारी निए गए थे।
C) RBI िे िानसि, महाराष्टर में िानसि नििा नगरिा सहिारी बैंि निनमटे ि िा िाइसेंस रद्द िर नदया, क्ोंनि
बैंि िे पास पयााप्त पूंिी और िमाई िी संभाििाएं थी ं।
1)केवल A
2)केवल A & B
3)केवल B & C
4)केवल A & C
5)सभी A, B & C
उत्तर -5)सभी A, B & C
स्पष्टीिरण:
भारतीय ररजवध बैंक (RBI) ने RBI (अक्कखल भारतीय सवत्तीय सोंथिानोों के सलए बेसल III पूोंजी ढाोंचे, एक्सपोजर मानदों ड,
महत्वपूिध सनवेश, वगीकरि, मूल्ाोंकन और सनवेश पोटध फोसलयो मानदों डोों और सोंसार्न जुटाने के मानदों डोों पर सववेकपूिध
सवसनयम) नदशानिदे श, 2023 जारी सकए जो 1 अप्रैल 2024 से लागू हो
i.इन्ें RBI िारा RBI अनधनियम, 1934 की धारा 45L िारा प्रदत्त शक्कक्तयोों का प्रयोग करते हुए जारी सकया गया िा।
ii.प्रयोज्यता:
• ये पाोंच अक्कखल भारतीय सवत्तीय सोंथिानोों (AIFI) पर लागू होोंगे।
• भारतीय सनयाधत-आयात बैंक (EXIM) बैंक
• राष्ट्रीय कृसष और ग्रामीि सवकास बैंक (NABARD)
• अवसोंरचना और सवकास के सवत्तपोषि के सलए राष्ट्रीय बैंक (NABFID)
• राष्ट्रीय आवास बैं क (NHB)
• भारतीय लघु उद्योग सवकास बैंक (SIDBI)
iii.25 ससतोंबर 2023 के एक आदे श के साि, RBI ने नाससक, महाराष्ट्र में नाससक सजला सगरना सहकारी बैंक सलसमटे ड
का लाइसेंस रद्द कर सदया, क्ोोंसक बैंक के पास पयाधप्त पूोंजी और कमाई की सोंभावनाएों िीों।
• बैंक बैंसकोंग सवसनयमन असर्सनयम, 1949 की र्ारा 56 के साि पढी गई र्ारा 11(1)और र्ारा 22 (3)(d) के
प्रावर्ानोों का अनुपालन नहीों करता है ।

Report Errors in the PDF - ebooks@affairscloud.com Copyright 2014-2023 @ AffairsCloud.com 91


61. उस बैंि िा िाम बताइए निसिे हाि ही में (नसतंबर'23 में) िोर-स्ट्े प गोड िोि सेिा शुरू िरिे िे निए
साहीबंधु िे साथ सहयोग निया है।
1)साउि इों सडयन बैंक
2)कनाधटक बैंक
3)HDFC बैंक
4)RBL बैंक
5)इों डसइों ड बैंक
उत्तर- 2)ििााटि बैंि
स्पष्टीिरण:
कनाधटक बैंक सलसमटे ड (KBL) ने साहीबंधु के सहयोग से अपने ग्राहकोों के सलए "KBL-स्विध बोंर्ु" नामक एक असभनव
िोर-स्ट्े प गोड िोि सेिा शुरू की।
i.यह गोड लोन प्रसक्रयाओों के पूिध सडसजटलीकरि को सक्षम बनाता है , सजससे बैंक ग्राहकोों को सीर्े उनके दरवाजे पर
स्विध ऋि सेवाएों प्रदान कर सकता है। यह अपनेगोड लोन पोटध फोसलयो का भी सवस्तार करता है।
ii.साहीबोंर्ु, द मसिपाल ग्रुप िारा समसिधत गोड लोन के सलए एक प्रमुख एग्रीगेटर प्लेटफॉमध है ।

ECONOMY AND BUSINESS


1. अगस्त 2023 में िारी राष्टरीय सांखख्यिी िायाािय (NSO), सांखख्यिी और िायाक्रम िायाान्वयि मंत्रािय
(MOSPI) द्वारा िारी आं िडों िे अिुसार, भारत िा सिि घरे िू उत्पाद (GDP) 2023-2024 िी अप्रैि-िूि
नतमाही (Q1FY24) में एि साि िे उच्च स्तर ________ हो गया।
1)7.8%
2)5.9%
3)6.4%
4)7.0%
5)7.2%
उत्तर- 1)7.8%
स्पष्टीिरण:
साोंक्कख्यकी और कायधक्रम कायाधन्वयन मोंत्रालय (MOSPI) के राष्ट्रीय साोंक्कख्यकी कायाध लय (NSO) िारा जारी आों कडोों के
अनुसार, भारत िा सकल घरे लू उत्पाद (GDP) 2023-2024 की अप्रैल-जून सतमाही (Q1FY24)में एक साल के उच्च
स्तर 7.8% तक बढ गया, जबसक FY23 की जनवरी-माचध सतमाही (Q4FY23)में 6.1% की वृक्कद्ध हुई िी।
• यह वृक्कद्ध मजबूत उपभोग माोंग और सेवा क्षेत्र में उच्च गसतसवसर्योों से प्रेररत है।
• भारत सबसे तेजी से बढती प्रमुख अिधव्यवथिाओों में से एक बना हुआ है , और FY24 में 6.5% की दर से बढने
की उम्मीद है।
• भारत सरकार के मुख्य आसिधक सलाहकार वेंकटरामन अनोंत नागेश्वरन ने आों कडोों की जानकारी दी।
• 2011-12 में Q1FY24 के सलए मूल कीमतोों पर सकल मूल् वसर्धत (GVA) Q1FY23 में 11.9% और Q4FY23
में 6.5% की तुलना में 7.8% बढी।

Report Errors in the PDF - ebooks@affairscloud.com Copyright 2014-2023 @ AffairsCloud.com 92


2. उस िंपिी िा िाम बताइए निसिे हाि ही में (अगस्त’23 में) अक्षय ऊिाा (RE), ग्रीि हाइिर ोिि और
िीिाबोिाइिेशि िे क्षेत्रों में सहयोग िरिे िे निए िेशिि थमाि पािर िॉरपोरे शि (NTPC ) निनमटे ि िे
साथ एि MoU पर हस्ताक्षर निए हैं।
1)भारत पेटरोसलयम सलसमटे ड
2)ऑयल एों ड नेचुरल गैस कॉपोरे शन सलसमटे ड
3)ऑयल इों सडया सलसमटे ड
4)इों सडयन ऑयल कॉपोरे शन सलसमटे ड
5)सहोंदुस्तान पेटरोसलयम सलसमटे ड
उत्तर-3)ऑयि इं निया निनमटे ि
स्पष्टीिरण:
31 अगस्त, 2023 को, NTPC निनमटे ि (सजसे पहले नेशनल िमधल पावर कॉपोरे शन के नाम से जाना जाता िा) और
ऑयि इं निया निनमटे ि (OIL) के बीच नवीकरिीय ऊजाध (RE), ग्रीन हाइडर ोजन और इसके डे ररवेसटव और भू -तापीय
ऊजाध के उपयोग ससहत डीकाबोनाइजेशन पहल के क्षेत्रोों में सहयोग का पता लगाने के सलए एक समझौता ज्ञापन
(MoU) पर हस्ताक्षर सकए गए िे।
• NTPC सलसमटे ड सवद् युत मोंत्रालय के अर्ीन एक महारत्न केंद्रीय सावधजसनक क्षेत्र उपक्रम (CPSU) है।
• OIL पेटरोसलयम और प्राकृसतक गैस मोंत्रालय (MoPNG) के तहत एक महारत्न CPSU है।
• MoU के माध्यम से, वे RE क्षेत्र में अपनी उपक्कथिसत का सवस्तार करें गे और भारत को अपने 2070 नेट शून्य
उत्सजधन लक्ष् को प्राप्त करने में मदद करने की अपनी प्रसतबद्धता के सहस्से के रूप में थिायी समार्ान
तलाशेंगे।
• NTPC 2032 तक 60 GW (गीगा वाट) RE क्षमता हाससल करने के सलए प्रसतबद्ध है ।

3. नसतंबर 2023 में, मूिीि िे अपिे ग्लोबि मैक्रो आउटिुि में 2023 िे निए भारत िे नििास अिुमाि िो
5.5% से बढ़ािर ________ िर नदया।
1)6.9%
2)6.7%
3)5.9%
4)6.2%
5)5.8%
उत्तर- 2)6.7%
स्पष्टीिरण:
वैसश्वक रे सटों ग एजेंसी मूिीि ने अपने ग्लोबल मैक्रो आउटलुक में Q1FY24 में मजबूत आसिधक सवकास के बीच 2023
कैलेंडर वषध के सलए भारत के सवकास अनुमान को 5.5% से बढाकर 6.7% कर सदया है।
i.सेवा क्षेत्र की मजबूत वृक्कद्ध और पूोंजीगत व्यय में वृक्कद्ध ने सवत्त वषध 2023-24 की पहली सतमाही अप्रैल से जून
(Q1FY24)में भारत की प्रभावशाली 7.8% GDP वृक्कद्ध में योगदान सदया, जो Q1FY24 की इसी अवसर् के सलए RBI के
8% के अनुमान से कम िा।
ii.भारत के सलए मूडीज का हासलया सवकास पूवाधनुमान, 2023 में 6.7% आसिधक सवकास दर की भसवष्यवािी करता है ,
जो सवत्तीय वषध 2023-24 (FY24)के सलए भारतीय ररजवध बैंक (RBI) के 6.5% के अनुमान से िोडा असर्क है।

Report Errors in the PDF - ebooks@affairscloud.com Copyright 2014-2023 @ AffairsCloud.com 93


4. उस िंपिी िा िाम बताइए निसिे हाि ही में (नसतंबर'23 में) ई-िॉमसा नियाात िो बढ़ािा दे िे िे निए िाि
निभाग िे साथ साझेदारी िी है।
1)फेडे क्स
2)िू डाटध
3)सदल्लीवरी
4)सबगफुट ररटे ल सॉल्ूशन
5)ईकाटध
उत्तर- 4)नबगफुट ररटे ि सॉल्यूशि
स्पष्टीिरण:
5 ससतोंबर, 2023 को, सोंचार मोंत्रालय के तहत िाि निभाग ने इों सडया पोस्ट् की व्यापक उपक्कथिसत और सशसपोंग
समार्ानोों का उपयोग करके ई-कॉमसध सनयाध त को बढावा दे ने के सलए नबगफुट ररटे ि सॉल्यूशि के साि साझेदारी की
है, जो नशपरॉिेट िांि के रूप में काम करता है।
• इस पर हस्ताक्षर महासनदे शक डाक सेवा आलोक शमाध , मोंजू कुमार, मुख्य पोस्ट्मास्ट्र जनरल सदल्ली और
सासहल गोयल, CEO, सशपरॉकेट की उपक्कथिसत में सकए गए।
i.यह समझौता डाक घर सनयाधत केंद्रोों (DNK) और सशपरोकेट के बीच तकनीकी एकीकरि को सक्षम बनाता है।
ii.सशपरॉकेट का उपयोग करने वाले भारत-आर्ाररत सवक्रेता सीर्े सशपरॉकेट प्लेटफॉमध से ई-PBE (सनयाधत का डाक
सबल) और सशसपोंग लेबल उत्पन्न कर सकते हैं।

5. निस िंपिी िे हाि ही में (नसतंबर'23 में) भारत िी शॉटा रें ि एयर निफेंस नसस्ट्म िो बढ़ािे िे निए इज़राइि
एयरोस्पेस इं िस्ट्र ीि निनमटे ि (IAI) िे साथ एि MoU पर हस्ताक्षर निए हैं ?
1)सहन्क्दुस्तान एयरोनॉसटक्स सलसमटे ड
2)सडफेंस ररसचध एों ड डे वलपमेंट आगेनाईजेशन
3)भारत हेवी इलेक्कररकल्स सलसमटे ड
4)भारत डायनेसमक्स सलसमटे ड
5)भारत इलेररॉसनक्स सलसमटे ड
उत्तर-5)भारत इिेररॉनिक्स निनमटे ि
स्पष्टीिरण:
भारत इिेररॉनिक्स निनमटे ि (BEL) और इज़राइल की अग्रिी एयरोस्पेस और रक्षा कोंपनी इज़राइल एयरोस्पेस
इों डस्ट्र ीज सलसमटे ड (IAI) ने शॉटध रें ज एयर सडफेंस ससस्ट्म के क्षेत्र में भारत की बढती जरूरतोों को पूरा करने में सहयोग
के सलए एक समझौता ज्ञापन (MoU) पर हस्ताक्षर सकए।
i.BEL के सनदे शक (सवपिन) K V सुरेश कुमार और IAI के समसाइल ससस्ट्म सडवीजन के उपाध्यक्ष और महाप्रबोंर्क असव
एसलशा ने कनाधटक के बेंगलुरु में BEL के अध्यक्ष और प्रबोंर् सनदे शक (CMD) भानु प्रकाश श्रीवास्तव की उपक्कथिसत में
MoU पर हस्ताक्षर सकए।
ii.MoU का उद्दे श्य IAI और BEL की क्षमताओों को मजबूत करना है और यह भारत सरकार (GoI) की 'आत्मसनभधर
भारत' और 'मेक इन इों सडया' नीसतयोों के अनुरूप है।

Report Errors in the PDF - ebooks@affairscloud.com Copyright 2014-2023 @ AffairsCloud.com 94


6. उस िाउि िंप्यूनटं ग सेिा िंपिी िा िाम बताइए निसिे हाि ही में (नसतंबर'23 में) सरिारी अनधिाररयों
िो साइबर सुरक्षा िौशि में प्रनशनक्षत िरिे िे निए भारतीय िंप्यूटर आपातिािीि प्रनतनक्रया टीम (CERT-
In) िे साथ साझेदारी िी है।
1)अमेज़न वेब ससवधसेज इों सडया
2)गूगल क्लाउड प्लेटफामध
3)ओरे कल क्लाउड इों फ्रास्ट्र क्चर
4)IBM क्लाउड
5)माइक्रोसॉफ्ट एज़्योर इों सडया
उत्तर- 2)गूगि िाउि प्लेटफामा
स्पष्टीिरण:
गूगल िारा प्रस्तुत गूगल क्लाउड प्लेटफ़ॉमध (GCP) ने सरकारी असर्काररयोों को साइबर सुरक्षा कौशल में प्रसशसक्षत करने
के सलए इलेररॉसनक्स और सूचना प्रौद्योसगकी मोंत्रालय (MeitY) के तहत भारतीय कोंप्यूटर आपातकालीन प्रसतसक्रया टीम
(CERT-In) के साि साझेदारी की है।
• CERT-In MeitY का सहस्सा है जो साइबर सुरक्षा खतरोों, हैसकोंग और अन्य साइबर-सोंबोंसर्त मुद्दोों से सनपटता है।
• इस साझेदारी का उद्दे श्य भारत में साइबर सुरक्षा पररदृश्य को मजबूत करना है।
• लगभग 1000 सरकारी असर्कारी जनरे सटव आसटध सफसशयल इों टेसलजेंस (AI) के उपयोग, साइबर सुरक्षा AI
हैकिॉन और वास्तसवक दु सनया के अनुप्रयोगोों और पररदृश्योों ससहत साइबर रक्षा सवोत्तम प्रिाओों में प्रसशक्षि
लेंगे।
• ओपन नेटवकध फॉर सडसजटल कॉमसध (ONDC) और गूगल क्लाउड ने जेनरे सटव AI के साि भारत में ई-कॉमसध
को आगे बढाने के सलए सहयोग सकया है।

7. निस बैंि िे हाि ही में (नसतंबर'23 में) मौिूदा और िए दोिों िािा धारिों िो यात्रा िाभ प्रदाि िरिे िे
निए एि अनभिि प्रस्ताि पेश िरिे िे निए खियरनटर प िे साथ साझेदारी िी है ?
1)ICICI बैंक
2)YES बैंक
3)एक्कक्सस बैंक
4)HDFC बैंक
5)इों डसइों ड बैंक
उत्तर-3)एखक्सस बैंि
स्पष्टीिरण:
क्किपकाटध इों सडया प्राइवेट सलसमटे ड की सहायक कोंपनी क्कक्लयरसटर प ने खियरनटर प के माध्यम से बुसकोंग करने वाले
मौजूदा और नए काडध र्ारकोों दोनोों को यात्रा लाभ प्रदान करने के सलए एक असभनव प्रस्ताव पेश करने के सलए भारत के
सबसे बडे सनजी क्षेत्र के बैंकोों में से एक, एखक्सस बैंि के साि साझेदारी की है ।
• इस सहयोग से करीब 12.5 नमनियि ग्राहिों को लाभ होगा।
i.यह घरे लू उडान बुसकोंग के सलए सवशेषासर्कारोों की एक सवस्तृत श्रृोंखला, जैसे सक 1,200 रुपये की सीटोों की बुसकोंग में
छूट, मुफ्त भोजन, सुसवर्ा शुल्क से छूट, और िेक्समैक्स के तहत 1 रुपये के मामूली शुल्क के सलए उडानोों को रद्द
करने और पुनसनधर्ाधररत करने के सवकल्प प्रदान करे गा।

Report Errors in the PDF - ebooks@affairscloud.com Copyright 2014-2023 @ AffairsCloud.com 95


8. हाि ही में (लसतंबर’23 में) लकस गठबंिन ने 6G वायरिेस प्रौद्योलगलकयों पर सहयोग के अवसरों का पता
िगाने के लिए भारत 6G एिायंस के साथ एक MoU पर हस्ताक्षर लकए हैं?
1) ग्लोबल टे ल्को AI एलायंस
2) नेक्स्ट G अलायंस
3) O-RAN एलायंस
4) हैंिसेट एलायंस खोलें
5) ICT एलायंस
उत्तर- 2)नेक्स्ट G अिायंस
स्पष्टीकरण:
एलायंस फॉर टे लीकम्युश्चनकेशंस इं िस्ट्र ी सॉल्यूशंस (ATIS) के नेक्स्ट G एिायंस और भारत 6G एिायंस (B6GA) ने
6G वायरलेस प्रौद्योश्चगश्चकयों पर सहयोग के अवसरों का पता लगाने के श्चलए एक समझौता ज्ञापन (MoU) पर हस्ताक्षर
श्चकए।
• MoU अनुसंर्ान और श्चवकास प्रािश्चमकताओं को संरेस्खत करने की सुश्चवर्ा प्रदान करे गा िो एक सामान्य 6G
दृश्चष्ट्कोि का समिगन करते हैं और सुरश्चक्षत और श्चवश्वसनीय दू रसंचार के साि-साि लचीली आपूश्चतग श्रृंखलाएं
बनाते हैं।
i.नेक्स्ट G एलायंस 6G पर प्रारं श्चभक फोकस के साि श्चनिी क्षेत्र के नेतृत्व वाले प्रयासों के माध्यम से अगले दशक में
उत्तर अमेररकी मोबाइल प्रौद्योश्चगकी नेतृत्व को आगे ब़िाने की एक पहल है।
ii.भारत 6G एलायंस को अगली पी़िी की वायरलेस तकनीक में नवाचार और सहयोग को ब़िावा दे ने के श्चलए संचार
मंत्रालय के तहत दू रसंचार श्चवभाग (DoT) द्वारा लॉन्च श्चकया गया िा।

9.उन दो भारतीय कंपलनयों के नाम बताइए लिन्ोंने हाि ही में (लसतंबर’23 में) भारत में आलटा लफलशयि
इं टेलििेंस (AI) सुपर कंप्यूटर बनाने के लिए USA स्कस्थत लचप लनमााता NVIDIA के साथ एक अिग साझेदारी
की है।
1)टाटा समूह
2) श्चवप्रो श्चलश्चमटे ि
3)ररलायंस इं िस्ट्र ीि श्चलश्चमटे ि
4) दोनों 1 & 2
5) दोनों 1 & 3
उत्तर- 5)दोनों 1& 3
स्पष्टीकरण:
संयुक्त राज्य अमेररका (USA) स्थित श्चचप श्चनमागता NVIDIA कॉपोरे शन ने भारत में आश्चटगश्चफश्चशयल इं टेश्चलिेंस (AI) सुपर
कंप्यूटर बनाने के श्चलए ररलायंस इं िस्ट्र ीि श्चलश्चमटे ि और टाटा समूह के साि अलग से साझेदारी की है ।
• साझेदारी का उद्दे श्य भारत में AI समार्ान श्चवकश्चसत करने के श्चलए कंप्यूश्चटंग बुश्चनयादी ढांचे और िेटफामों को
आगे ब़िाना है।
• NVIDIA कॉपोरे शन एक प्रौद्योश्चगकी कंपनी है िो ग्राश्चफक्स प्रोसेश्चसंग यूश्चनट् स (GPU) के श्चििाइन और श्चनमागि
के श्चलए िानी िाती है।
i.ररिायंस और NVIDIA भारत का अपना फाउं िेशन लािग लैंग्वेि मॉिल (LLM) श्चवकश्चसत करने के श्चलए सहयोग
करें गे, िो िेनेररक AI अनुप्रयोगों के श्चलए उपयुक्त होगा और भारतीय भाषाओं की एक श्चवस्तृत श्रृंखला में श्चनदे श श्चदया
िाएगा।

Report Errors in the PDF - ebooks@affairscloud.com Copyright 2014-2023 @ AffairsCloud.com 96


ii.टाटा और NVIDIA अपनी श्रेिी में सवगश्रेष्ठ प्रदशगन प्राप्त करने के श्चलए अगली पी़िी के NVIDIA GH200 ग्रेस हॉपर
सुपरश्चचप द्वारा संचाश्चलत AI सुपरकंप्यूटर के श्चनमागि के श्चलए सहयोग करें गे।

10. उस िंपिी िा िाम बताइए निसिे हाि ही में (नसतंबर’23 में) ग्रीि हाइिर ोिि िा उत्पादि िरिे िे निए
NTPC ग्रीि एििी निनमटे ि (NGEL) िे साथ एि MoU पर हस्ताक्षर निए हैं।
1)रीन्यू पावर सलसमटे ड
2)नायरा एनजी सलसमटे ड
3)ररलायोंस पावर सलसमटे ड
4)टाटा पावर सलसमटे ड
5)अदानी टर ाोंससमशन सलसमटे ड
उत्तर-2)िायरा एििी निनमटे ि
स्पष्टीिरण:
NTPC ग्रीन एनजी सलसमटे ड (NGEL) ने नायरा एनजी के कैसप्टव उपयोग के सलए ग्रीि हाइिर ोिि का उत्पादन करने
और डीकाबोनाइजेशन प्रसक्रया में उनकी मदद करने और काबधन फुटसप्रोंट को कम करने के सलए िायरा एििी निनमटे ि
के साि समझौता ज्ञापन (MoU) पर हस्ताक्षर सकए।
i.यह साझेदारी हाइडर ोजन पररयोजनाओों को सवकससत करने के सलए NTPC की पहल के साि सोंरेक्कखत है और इसका
उद्दे श्य भारत को सेल्फ-ररलायेंट (आत्मसनभधर भारत) बनाना है।
ii.MoU पर NGELके मुख्य कायधकारी असर्कारी (CEO) मोसहत भागधव और नायरा एनजी के तकनीकी प्रमुख अमर
कुमार और अन्य असर्काररयोों की उपक्कथिसत में हस्ताक्षर सकए गए।

11. उस िंपिी िा िाम बताइए निसिे हाि ही में (नसतंबर'23 में) भारत में हाई-स्पीि एल्यूमीनियम रे ि िोचों
िे निए उच्च-पररशुिता िािे एक्सट्रूिे ि उत्पादों िे निमााण िे निए इटिी खस्थत मेटरा SpA, रोिें गो नसयािो िे
साथ एि प्रौद्योनगिी साझेदारी पर हस्ताक्षर निए हैं।
1)वेदाोंता सलसमटे ड
2)नेशनल एल्ुमीसनयम कोंपनी सलसमटे ड
3)सहोंडाल्को इों डस्ट्र ीज सलसमटे ड
4)भारत एल्ुमीसनयम कोंपनी सलसमटे ड
5)सहोंदुस्तान सजोंक सलसमटे ड
उत्तर-3)नहंिाल्को इं िस्ट्र ीि निनमटे ि
स्पष्टीिरण:
नहंिाल्को इं िस्ट्र ीि निनमटे ि ने भारत में हाई-स्पीड एल्यूमीनियम रे ि िोचों के सलए उच्च पररशुद्धता वाले एक्सट्रूडे ड
उत्पादोों के सनमाधि के सलए इटली क्कथित मेटरा SpA रोिें गो सायािो के साि एक प्रौद्योसगकी साझेदारी पर हस्ताक्षर सकए।
i.रे लवे उद्योग में एक्सट्रूडे ड एल्ुमीसनयम महत्वपूिध है क्ोोंसक यह वजन घटाने और याों सत्रक शक्कक्त को जोडता है।
ii.सहोंडाल्को ने वोंदे भारत टर े नोों के सलए यात्री कोच बनाने में 2,000 िरोड रुपये का सनवेश करने की योजना बनाई है
और यह प्रौद्योसगकी साझेदारी इस प्रयास के सलए अत्यार्ुसनक तकनीक प्रदान करे गी।

Report Errors in the PDF - ebooks@affairscloud.com Copyright 2014-2023 @ AffairsCloud.com 97


12. निस िंपिी िे हाि ही में (नसतंबर'23 में) अपिे यानत्रयों िो व्यखक्तगत और परे शािी मुक्त िमीिी अिुभि
प्रदाि िरिे िे निए "प्रोिेर अनभिंदि" िॉन्च निया है ?
1)इों सडगो
2)सवस्तारा
3)स्पाइसजेट
4)एयर इों सडया
5)एलायोंस एयर
उत्तर-4)एयर इं निया
स्पष्टीिरण:
टाटा सोंस के स्वासमत्व वाली एयर इं निया ने अपने यासत्रयोों को व्यक्कक्तगत और परे शानी मुक्त ग्राउों ड अनुभव प्रदान करने
के सलए "प्रोिेर अनभिंदि" (सहोंदी में असभवादन) लॉन्च सकया।
• प्रोजेर असभनोंदन के सहस्से के रूप में, एयर इों सडया के मेहमानोों को सहायता और सहायता प्रदान करने के सलए
सवशेष रूप से प्रसशसक्षत सेवा आश्वासन असर्कारी (SAO) भारत के 16 प्रमुि हिाई अड्डों पर तैनात सकए जाएों गे।
• 16 प्रमुख हवाई अड्डोों में अहमदाबाद (गुजरात), बेंगलुरु (कनाधटक), कोक्कच्च और कोसझकोड (केरल), चेन्नई
(तसमलनाडु ), सदल्ली, गोवा, गुवाहाटी (असम), हैदराबाद (तेलोंगाना), कोलकाता (पसिम बोंगाल), वारािसी और
लखनऊ (उत्तर प्रदे श), मुोंबई, नागपुर, पुिे (महाराष्ट्र), और सवशाखापत्तनम (आों ध्र प्रदे श) शासमल हैं।

13. नसतंबर 2023 में, नफच रे नटं ग्स िे चािू नित्त िषा (FY 2023-24)िे निए भारत िे सिि घरे िू उत्पाद (GDP)
िे नििास पूिाािुमाि िो __________ पर बरिरार रिा।
1)5.9%
2)6.3%
3)6.0%
4)6.9%
5)5.2%
उत्तर- 2)6.3%
स्पष्टीिरण:
नफच रे नटं ग्स ने ग्लोबल इकोनॉसमक आउटलुक - ससतोंबर 2023 नामक अपनी सवशेष ररपोटध में चालू सवत्त वषध (FY 2023-
24)के सलए भारत के सकल घरे लू उत्पाद (GDP) की वृक्कद्ध का अनुमान 6.3% और FY 2024-2025 के सलए 6.5% की
अनुमासनत वृक्कद्ध दर को बरकरार रखा है।
• इससे पहले जून 2023 में रे सटों ग एजेंसी ने अपने अनुमान को 6% से सोंशोसर्त कर 6.3% कर सदया िा।
• सख्त मौसद्रक नीसत और खराब सनयाधत के बावजूद, अिधव्यवथिा बढने का प्रयास कर रही है।
• FY 2023-24 की पहली सतमाही (Q1 - अप्रैल से जून) में भारतीय अिधव्यवथिा 7.8% की दर से बढी। सफच को
उम्मीद है सक 2023 के सलए RBI की बेंचमाकध ब्याज दर 6.5% पर रहेगी।

14. नसतंबर 2023 में, िेशिि हाईिे िॉनिखस्ट्क्स मैिेिमेंट निनमटे ि (NHLML) िे _____________ (शहर) में
मल्टी मॉिि िॉनिखस्ट्क्स पािा (MMLP) िे नििास िे निए एि समझौते पर हस्ताक्षर निए।
1)बेंगलुरु, कनाधटक
2)चेन्नई, तसमलनाडु
3)हैदराबाद, तेलोंगाना

Report Errors in the PDF - ebooks@affairscloud.com Copyright 2014-2023 @ AffairsCloud.com 98


4)मुोंबई, महाराष्ट्र
5)अहमदाबाद, गुजरात
उत्तर-1)बेंगिुरु, ििााटि
स्पष्टीिरण:
नेशनल हाईवे लॉसजक्कस्ट्क्स मैनेजमेंट सलसमटे ड (NHLML) ने ििााटि के बेंगिुरु में मुडेसलोंगनहल्ली में मल्टी मॉडल
लॉसजक्कस्ट्क्स पाकध (MMLP) के सवकास के सलए एक समझौते पर हस्ताक्षर सकए हैं ।
i.समझौते पर सरकारी सवशेष प्रयोजन वाहन (SPV) बेंगलुरु MMLP प्राइवेट सलसमटे ड और कोंसेशनेयर SPV PATH
बेंगलुरु लॉसजक्कस्ट्क्स पाकध प्राइवेट सलसमटे ड के बीच हस्ताक्षर सकए गए।
ii.प्रस्तासवत MMLP को 1,770 करोड रुपये की अनुमासनत लागत पर सावधजसनक सनजी भागीदारी (PPP) (सडजाइन,
सनमाधि, सवत्त, सोंचालन और हस्ताोंतरि (DBFOT) मॉडल के तहत सवकससत सकया जाएगा।
iii.यह पररयोजना PM गसत शक्कक्त राष्ट्रीय मास्ट्र प्लान के तहत दे श में लागू की गई पहली और सबसे बडी MMLP बनने
के सलए तैयार है ।

15. लसतंबर 2023 में, एलशयाई लवकास बैंक (ADB) ने लवत्तीय वषा 2022-2023 (FY23)के लिए भारत के सकि
घरे िू उत्पाद (GDP) के अनुमान को 6.4% से घटाकर ________ कर लदया।
1)6.1%
2)5.9%
3)6.0%
4)6.2%
5)6.3%
उत्तर- 5)6.3%
स्पष्टीकरण:
एश्चशयाई श्चवकास बैंक (ADB) ने अपने एश्चशयाई श्चवकास आउटलुक (ADO) श्चसतंबर 2023 में श्चवत्तीय वषग 2022-2023
(FY23)के श्चलए भारत के सकल घरे लू उत्पाद (GDP) के अनुमान को 6.4% (ADO अप्रैल 2023 अनुमान) से एक
प्रश्चतशत घटाकर 6.3% कर श्चदया।
i.यह श्चगरावट र्ीमी श्चनयागत, वैश्चश्वक भू -रािनीश्चतक तनाव और खरीफ सीिन (िुलाई-अरू बर) की कटाई के समय या
रबी सीिन (अरू बर-अप्रैल) के दौरान कृश्चष उत्पादन पर अश्चनयश्चमत वषाग के संभाश्चवत प्रभाव के कारि है।
ii.FY 2023-2024 (FY24)के श्चलए GDP अनुमान 6.7% पर बरकरार रखा गया है ।
iii.FY25 में, यश्चद प्रत्यक्ष श्चवदे शी श्चनवेश (FDI) प्रवाह बड़ा होता है, खासकर श्चवश्चनमागि क्षेत्र में, तो आश्चिगक वृस्ि अश्चर्क हो
सकती है।

16. इं लिया रे लटं ग्स एं ि ररसचा (इं ि-रा) ने हाि ही में (लसतंबर 2023 में) FY24 के लिए भारत के GDP लवकास
अनुमान को __________ से संशोलित कर 6.2% कर लदया है, िो फरवरी 2023 में अनुमालनत था।
1)5.6%
2)5.9%
3)5.7%
4)5.5%
5)5.0%
उत्तर- 2)5.9%

Report Errors in the PDF - ebooks@affairscloud.com Copyright 2014-2023 @ AffairsCloud.com 99


स्पष्टीकरण:
इं श्चिया रे श्चटंग्स एं ि ररसचग (इं ि-रा) ने FY24 के श्चलए भारत की GDP वृस्ि का अनुमान फरवरी 2023 में अनुमाश्चनत
5.9% से संशोश्चर्त कर 6.2% कर श्चदया।
• यह वृस्ि श्चनरं तर सरकारी पूंिीगत व्यय, भारतीय उद्योग िगत और बैंकों की श्चिलीवरे ज्ड बैलेंस शीट और एक
नए श्चनिी कॉपोरे ट पूंिीगत व्यय चक्र की संभावना के कारि है।
• FY24 की पहली श्चतमाही (1QFY24)में, श्चतमाही GDP की वृस्ि 7.8% िी और FY24 की शेष तीन श्चतमाश्चहयों में
इसके र्ीमा होने का अनुमान है।
• श्चनिी अंश्चतम उपभोग व्यय (PFCE) FY24 में 6.9% ब़िने की उम्मीद है, िबश्चक FY23 में यह 7.5% िी।

17. लसतंबर 2023 में, S&P (स्ट्ैं ििा & पूअसा) ग्लोबि ने FY24 के लिए भारत की लवकास भलवष्यवाणी को 5.9%
से बढाकर _________ कर लदया।
1)6.8%
2)6.2%
3)6.4%
4)6.6%
5)6.9%
उत्तर- 4)6.6%
स्पष्टीकरण:
18 श्चसतंबर, 2023 को, S&P (स्ट्ैं ििग & पूअसग) ग्लोबल ने अप्रैल-िून श्चतमाही (Q1FY24)में मिबूत वृस्ि के बीच श्चवत्त
वषग 2024 के श्चलए भारत की श्चवकास भश्चवष्यवािी को अगस्त 2023 में अनुमाश्चनत 5.9% से ब़िाकर 6.6% कर श्चदया।
i.FY25 में, यह अगस्त 2023 में अनुमाश्चनत 6.1% की तुलना में 6.2% की दर से ब़िे गी।
ii.भारत की मुद्रास्फीश्चत दर अगस्त 2023 में 5.1% की भश्चवष्यवािी की तुलना में 2023 में तेिी से ब़िकर 6% हो िाएगी।

18. हाि ही में (लसतंबर'23 में) लकस कंपनी ने नॉवे के लवत्तीय बुलनयादी ढांचे को बढाने के लिए BankID
BankAxept AS के साथ साझेदारी की है?
1)इं फोश्चसस श्चलश्चमटे ि
2)टाटा कंसल्ट्ें सी सश्चवगसेि
3)कॉश्चििेंट श्चलश्चमटे ि
4)लासगन एं ि टु िो श्चलश्चमटे ि
5)टे क मश्चहंद्रा श्चलश्चमटे ि
उत्तर-2)टाटा कंसल्ट्ें सी सलवासेि
स्पष्टीकरण:
टाटा समूह की एक श्चहस्से, टाटा कंसल्ट्ें सी सलवासेि (TCS) ने नॉवे के राष्ट्रीय भुगतान और इलेररॉश्चनक पहचान प्रिाली
BankID BankAxept AS के साि एक संचालन कमांि सेंटर थिाश्चपत करने और प्रबंश्चर्त करने के श्चलए साझेदारी की
है िो नॉवे के लवत्तीय बुलनयादी ढांचे की लचीलापन, सुरक्षा और उपलिता को ब़िाएगा।
i.BankID नॉवेश्चियन eID सत्यापन प्रिाली है, श्चिसका उपयोग 90% से अश्चर्क आबादी, बैंक, सरकारी एिेंश्चसयां और
वाश्चिस्ज्यक उद्यम करते हैं।
ii.समझौते के श्चहस्से के रूप में, TCS सभी BankID BankAxept AS उपयोगकतागओं और ग्राहकों के श्चलए ओस्लो, नॉवे
में 24×7 ऑपरे शन कमांि सेंटर का श्चनमागि और प्रबंर्न करे गी।

Report Errors in the PDF - ebooks@affairscloud.com Copyright 2014-2023 @ AffairsCloud.com 100


19. नसतंबर 2023 में नित्त मंत्रािय िे आनथाि मामिों िे निभाग द्वारा िारी मानसि आनथाि समीक्षा - अगस्त
2023 िे अिुसार, नित्तीय िषा 2023-24 (FY24)िे निए भारत िी िास्तनिि सिि घरे िू उत्पाद (GDP) िृखि
__________ पर बरिरार रिी गई थी।
1)6.5%
2)5.9%
3)5.2%
4)6.0%
5)6.9%
उत्तर- 1)6.5%
स्पष्टीिरण:
सवत्त मोंत्रालय के तहत आनथाि मामिों िे निभाग िारा जारी माससक आसिधक समीक्षा - अगस्त 2023 में सवत्त वषध 2023-
24 (FY24 ) के सलए भारत िे िास्तनिि सिि घरे िू उत्पाद (GDP) िी िृखि के सलए अपने अनुमान को 6.5% पर
बरकरार रखा गया है।
i.कॉपोरे ट लाभप्रदता, सनजी क्षेत्र के पूोंजी सनमाधि, बैंक ऋि वृक्कद्ध और सनमाधि क्षेत्र में गसतसवसर् में उज्ज्वल थिानोों से सवकास
दृसष्ट्कोि के सलए जोक्कखमोों की भरपाई की गई िी।
ii.उपभोग और सनवेश के सलए उच्च घरे लू माों ग ने FY24 (अप्रैल-जून 2023)की पहली सतमाही (Q1)में GDP सवकास दर
को बढा सदया।
iii.उच्च आवृसत्त सोंकेतक (HFI) बताते हैं सक FY24 (जुलाई-ससतोंबर 2023)की दू सरी सतमाही (Q2)अच्छी तरह से आकार
ले रही है।

20. निस िंपिी िे हाि ही में (नसतंबर'23 में) मुंबई अपतटीय क्षेत्रों से तेि िी नबक्री िे निए तेि और प्रािृनति
गैस निगम निनमटे ि (ONGC) िे साथ एि समझौते पर हस्ताक्षर निए हैं ?
1)सहन्क्दुस्तान पेटरोसलयम कॉपोरे शन सलसमटे ड
2)इों सडयन ऑयल कॉपोरे शन सलसमटे ड
3)भारत पेटरोसलयम कॉपोरे शन सलसमटे ड
4)ऑयल इों सडया सलसमटे ड
5)मैंगलोर ररफाइनरी एों ड पेटरोकेसमकल्स सलसमटे ड
उत्तर-1)नहन्दु स्ताि पेटरोनियम िॉपोरे शि निनमटे ि
स्पष्टीिरण:
भारत में सबसे बडी कच्चे तेल और प्राकृसतक गैस कोंपनी, तेल और प्राकृसतक गै स सनगम सलसमटे ड (ONGC) ने मुंबई
(महाराष्टर) िे अपतटीय क्षेत्रों से सहोंदुस्तान पेटरोसलयम कॉपोरे शन सलसमटे ड (HPCL) को कच्चे तेल की आपूसतध करने के
सलए एक समझौते पर हस्ताक्षर सकए।
• यह समझौता HPCL की मुोंबई ररफाइनरी को प्रसत वषध लगभग 4.5 समसलयन टन कच्चे तेल की सबक्री को कवर
करता है।
• ONGC और HPCL पेटरोसलयम और प्राकृसतक गैस मोंत्रालय (MoPNG) के तहत महारत्न केंद्रीय सावधजसनक क्षेत्र
उपक्रम (CPSU) हैं । HPCL ONGC की सहायक कोंपनी है।
• सवपिन स्वतोंत्रता के बाद मुोंबई ऑफशोर कच्चे तेल की सबक्री के सलए यह ONGC का दू सरा कायधकाल समझौता
है।

Report Errors in the PDF - ebooks@affairscloud.com Copyright 2014-2023 @ AffairsCloud.com 101


िोट: ONGC मुोंबई के तट से दू र अरब सागर में अपने सोंसार्नोों से प्रसत वषध 13-14 समसलयन टन कच्चे तेल का उत्पादन
करती है।

21. उस िंपिी िा िाम बताइए निसिे हाि ही में (नसतंबर'23 में) छात्रों िो िौशि और िई तििीिों से
पररनचत िरािे िे निए नशक्षा मंत्रािय (MoE) और अखिि भारतीय तििीिी नशक्षा पररषद (AICTE) िे साथ
एि MoU पर हस्ताक्षर निए हैं।
1)सवप्रो सलसमटे ड
2)टाटा कोंसल्टें सी ससवधसेज
3)गूगल
4)इन्फोससस सलसमटे ड
5)माइक्रोसॉफ्ट
उत्तर-5)माइक्रोसॉफ्ट
स्पष्टीिरण:
सशक्षा मोंत्रालय (MoE), अक्कखल भारतीय तकनीकी सशक्षा पररषद (AICTE) और माइक्रोसॉफ्ट ने छात्रोों को नई
प्रौद्योसगसकयोों के कौशल और अनुभव से लैस करने के सलए एक समझौता ज्ञापन (MoU) पर हस्ताक्षर सकए हैं ।
i.सहयोग का उद्दे श्य छात्रोों को कौशल सवकास, परामशध और पररयोजना सवकास के अवसर प्रदान करना है , साि ही AI,
क्लाउड, डे टा साइों स, डे टा एनासलसटक्स और सुरक्षा के क्षेत्र में सशक्षकोों को सशक्त बनाना है।
ii.AICTE के मौजूदा पाठ्यक्रम को पूरक करने और छात्रोों को माइक्रोसॉफ्ट प्रौद्योसगसकयोों के साि व्यावहाररक अनुभव
प्रदान करने के सलए माइक्रोसॉफ्ट अपने केंद्रीकृत प्रसशक्षि और कौशल मोंच के साि माइक्रोसॉफ्ट लनध को एकीकृत
करने के सलए तैयार है।
iii.माइक्रोसॉफ्ट उद्योग अोंतदृध सष्ट् और मागधदशधन के सलए भारत में एज़्योर डे वलपर समुदाय में छात्रोों को भी शासमल करे गा।

MoU’s AND AGREEMENTS


1. उस मंत्रािय िा िाम बताइए निसिे हाि ही में (अगस्त’23 में) सफाई िमाचाररयों, हाथ से मैिा ढोिे िािों,
िचरा बीििे िािों और उििे पररिारों िे सामानिि-आनथाि उत्थाि िो बढ़ािे िे निए राष्टरीय सफाई
िमाचारी नित्त और नििास निगम (NSKFDC) िे साथ एि MoU पर हस्ताक्षर निए हैं।
1)ग्रामीि सवकास मोंत्रालय
2)सामासजक न्याय एवों असर्काररता मोंत्रालय
3)गृह मोंत्रालय
4)पोंचायती राज मोंत्रालय
5)आवास और शहरी मामलोों का मोंत्रालय
उत्तर -2)सामानिि न्याय एिं अनधिाररता मंत्रािय
स्पष्टीिरण:
सामानिि न्याय एिं अनधिाररता मंत्रािय (MoSJE) ने सफाई कमधचाररयोों, हाि से मैला ढोने वालोों, कचरा बीनने वालोों
और उनके पररवारोों के सामासजक-आसिधक उत्थान को बढाने के सलए राष्ट्रीय सफाई कमधचारी सवत्त और सवकास सनगम
(NSKFDC) के साि एक समझौता ज्ञापन (MoU) पर हस्ताक्षर सकए। .
• सवत्तीय वषध (FY) 2023-24 और 2024-25 के सलए हस्ताक्षररत MoU पूरे भारत में हासशए पर रहने वाले समुदायोों
को सशक्त बनाने पर केंसद्रत होगा।

Report Errors in the PDF - ebooks@affairscloud.com Copyright 2014-2023 @ AffairsCloud.com 102


i.सहयोगात्मक प्रयास का उद्दे श्य उन गसतसवसर्योों का सवस्तार करना है जो सुरक्षा, सशक्षा, कौशल सवकास, उद्यसमता और
दीघधकासलक रोजगार की सोंभावनाओों को बढावा दे ते हैं , सजससे स्वच्छता कमधचाररयोों के समग्र सवकास को सक्षम सकया
जा सके।
ii.इस सहयोग का उद्दे श्य हासशए पर रहने वाले समुदायोों को सशक्त बनाने वाले कायधक्रमोों को प्रभावी ढों ग से लागू करने
के सलए सवशेषज्ञता और सोंसार्नोों को साझा करना है।
iii.MoU लसक्षत कल्ाि कायधक्रमोों के सलए र्न का असर्क प्रभावी ढों ग से आवोंटन और उपयोग करके समावेशी सवकास
में तेजी लाने के उद्दे श्य पर जोर दे ता है।

2. अगस्त 2023 में िािूि और न्याय राज्य मंत्री (MoL&J) द्वारा निए गए िॉन्च िे संबंध में निम्ननिखित में से
िौि सा/से नबंदु "सही" है /हैं?
A) िािूि और न्याय मंत्रािय िे राज्य मंत्री अिुाि राम मेघिाि िे संचार और सूचिा प्रौद्योनगिी िे सहयोग से
िािूिी िाििारी और सिाह प्रदाि िरिे िे निए टे िी-िॉ 2.0 िॉन्च निया।
B) टे िी-िॉ-2.0, िो टे िी-िॉ और न्याय बंधु ऐप िो एिीिृत िरता है , इसिे साथ ही इसिा सूचिात्मि ई-
ट्यूटोररयि भी िारी निया गया है।
C) िेंद्रीय मंत्री िे िॉयस ऑफ बेनिनफनशयरीि बुििेट िा चौथा संस्करण भी िॉन्च निया, िो उि व्यखक्तयों
िी िास्तनिि िीिि िी िहानियों पर प्रिाश िािता है निििे िीिि पर टे िी-िॉ िा सिारात्मि प्रभाि
पडा।
1)केवल A
2)केवल A & B
3)केवल B & C
4)केवल A & C
5)सभी A, B & C
उत्तर -5)सभी A, B & C
स्पष्टीिरण:
अिुाि राम मेघिाि, राज्य मोंत्री (MoS) स्वतोंत्र प्रभार (IC) कानून और न्याय मोंत्रालय (MoL&J) ने लीगल जानकारी
और सलाह प्रदान करने के सलए न्याय सवभाग (DoJ), MoL&J की एक पहल, टे ली-लॉ 2.0 लॉन्च सकया। सोंचार और
सूचना प्रौद्योसगकी के. आयोजन की कुछ मुख्य बातें ये हैं
• “टे िी-िॉ" डॉक्ूमेंटरी की स्क्रीसनोंग, सजसमें पाोंच वषों (2017-2022)में टे ली-लॉ की यात्रा को दशाधया गया है।
• "टे िी-िॉ-2.0" का लॉन्च, जो टे िी-िॉ और न्याय बंधु ऐप को एकीकृत करता है , साि ही इसके सूचनात्मक ई-
ट्यूटोररयल को भी जारी सकया गया है।
• केंद्रीय मोंत्री ने वॉयस ऑफ बेसनसफसशयरीज बुकलेट का चौिा सोंस्करि भी लॉन्च सकया, जो उन व्यक्कक्तयोों की
वास्तसवक जीवन की कहासनयोों पर प्रकाश डालता है सजनके जीवन पर टे ली-लॉ का सकारात्मक प्रभाव पडा।

3. निस िंपिी िे हाि ही में (अगस्त’23 में) सूक्ष्म, िघु और मध्यम उद्यम (MSME) नियाातिों िे निए सीमा
पार रसद िो सरि बिािे िे निए इं निया पोस्ट् िे साथ एि MoU पर हस्ताक्षर निए हैं ?
1)मीशो
2)क्किपकाटध
3)अमेज़़ॅन
4)इों सडयामाटध

Report Errors in the PDF - ebooks@affairscloud.com Copyright 2014-2023 @ AffairsCloud.com 103


5)सजयो माटध
उत्तर-3)अमेज़़ॅि
स्पष्टीिरण:
ई-कॉमसध पर ध्यान केंसद्रत करने वाली अमेररकी बहुराष्ट्रीय प्रौद्योसगकी कोंपनी अमेज़़ॅि ने सूक्ष्म, लघु और मध्यम उद्यम
(MSME) नियाातिों के सलए सीमा पार रसद को सरल बनाने के सलए इं निया पोस्ट् के साि एक समझौता ज्ञापन
(MoU) पर हस्ताक्षर सकए हैं।
• 31 अगस्त 2023 को नई सदल्ली, सदल्ली में आयोसजत (चौिे) अमेज़़ॅन सोंभव सशखर सम्मेलन 2023 के दौरान
MoU पर हस्ताक्षर सकए गए िे।
i.इवेंट के दौरान, अमेज़़ॅन ने एक जेनरे सटव आसटध सफसशयल इों टेसलजेंस (AI) आर्ाररत व्यक्कक्तगत सडसजटल सहायक
"अमेज़़ॅन सह-AI (Amazon Sah-AI)" पेश सकया।
ii.अमेज़़ॅन सोंभव सशखर सम्मेलन का चौिा सोंस्करि 2023 में हुआ, जो छोटे व्यवसायोों और उपभोक्ताओों के सलए
नवाचार, सवत्तीय समावेशन और क्कथिरता पर केंसद्रत िा।
iii.कायधक्रम के दौरान, सवज्ञान और प्रौद्योसगकी राज्य मोंत्री (स्वतोंत्र प्रभार) सजतेंद्र ससोंह ने इों सडया पोस्ट् और अमेज़़ॅन के
बीच 10 साल की साझेदारी की स्मृसत में पोस्ट्ल स्ट्ाम्प का अनावरि सकया।

4. हाि ही में (अगस्त 2023 में) लकस कंपनी ने रक्षा सेवाओं के पूवा सैलनकों के लिए रोिगार के अवसरों की
सुलविा के लिए पुनवाास महालनदे शािय (DGR) के साथ एक MoU पर हस्ताक्षर लकए हैं?
1)टाटा कोंसल्टें सी ससवधसेज
2)HCL टे क्नोलॉजीज सलसमटे ड
3)टे क मसहों द्रा सलसमटे ड
4)सवप्रो सलसमटे ड
5)जेनपैर इों सडया प्राइवेट सलसमटे ड
उत्तर-5)िेिपैर इं निया प्राइिेट निनमटे ि
स्पष्टीिरण:
रक्षा मोंत्रालय (MoD) के भूतपूवध सैसनक कल्ाि सवभाग के तहत पुनवाधस महासनदे शालय (DGR) ने रक्षा सेवाओों के पूवध
सैसनकोों के सलए रोजगार के अवसरोों की सुसवर्ा के सलए िेिपैर इं निया प्राइिेट निनमटे ि के साि एक समझौता
ज्ञापन (MoU) पर हस्ताक्षर सकए।
i.यह साझेदारी कॉपोरे ट कोंपसनयोों और पूवध सैसनकोों को एक साझा मोंच पर लाती है तासक पूवध सैसनकोों को कुशल
जनशक्कक्त प्रदान की जा सके और उन्ें उनकी सैन्य सेवा के बाद दू सरा कैररयर सदया जा सके।
ii.मुोंबई (महाराष्ट्र) क्कथित जेनपैर इों सडया एक वैसश्वक पेशेवर सेवा फमध है जो बुक्कद्धमान व्यापार सोंचालन से वाएों प्रदान
करती है।

5. GoI िे निस सािािनिि क्षेत्र िे नित्तीय संस्थािों िे हाि ही में (नसतंबर'23 में) ररन्यूएबि एििी (RE)
पररयोििाओं िे सह-नित्तपोषण िे निए इं नियि ररन्यूएबि एििी िे ििपमेंट एिेंसी निनमटे ि (IREDA) िे
साथ एि MoU पर हस्ताक्षर निए हैं ?
1)इों डक्कस्ट्रयल फाइनेंस कॉपोरे शन ऑफ़ इों सडया
2)नेशनल बैंक ऑफ फाइनेंससोंग इों फ्रास्ट्र क्चर एों ड डे वलपमेंट
3)इों सडया इों फ्रास्ट्र क्चर फाइनेंस कोंपनी सलसमटे ड
4)एक्सपोटध -इम्पोटध बैंक ऑफ़ इों सडया

Report Errors in the PDF - ebooks@affairscloud.com Copyright 2014-2023 @ AffairsCloud.com 104


5)स्माल इों डस्ट्र ीज डे वलपमेंट बैंक ऑफ़ इों सडया
उत्तर-3)इं निया इं फ्रास्ट्र क्चर फाइिेंस िंपिी निनमटे ि
स्पष्टीिरण:
4 ससतोंबर, 2023 को, ररन्यूएबल एनजी (RE) पररयोजनाओों के सह-सवत्तपोषि के सलए इों सडयन ररन्यूएबल एनजी
डे वलपमेंट एजेंसी सलसमटे ड (IREDA) और इों सडया इों फ्रास्ट्र क्चर फाइनेंस कोंपनी सलसमटे ड (IIFCL) के बीच एक
समझौता ज्ञापन (MoU) पर हस्ताक्षर सकए गए।
• यह MoU ग्रीन फाइनेंससोंग में IREDA और इों फ्रास्ट्र क्चर फाइनेंससोंग में IIFCL की सवशेषज्ञता का उपयोग करे गा।
i.MoU लघु जलसवद् युत पररयोजनाओों ससहत सभी RE पररयोजनाओों के सलए सह-ऋि/सह-उत्पसत्त और ऋि
ससोंसडकेशन के सलए IREDA और IIFCL को सशक्त बनाएगा।
ii.IREDA RE क्षेत्र के सवकास के सलए IIFCL को अपनी तकनीकी-सवत्तीय सवशेषज्ञता प्रदान करे गा।
• दोनोों सोंगठन 3-4 साल के सलए IREDA उर्ार के सलए ब्याज दरें तय करें गे।

6. निम्ननिखित में से निस बैंि िे हाि ही में (नसतंबर’23 में) ििीिरणीय ऊिाा पररयोििाओं िे सह-
नित्तपोषण िे निए भारतीय ििीिरणीय ऊिाा नििास एिेंसी निनमटे ि (IREDA) िे साथ MoU पर हस्ताक्षर
निए हैं ?
1)यूसनयन बैंक ऑफ इों सडया
2)बैंक ऑफ बडौदा
3)इों सडयन बैंक
4)1 और 2 दोनोों
5)2 और 3 दोनोों
उत्तर- 4)1 और 2 दोिों
स्पष्टीिरण:
5 ससतोंबर 2023 को, भारतीय नवीकरिीय ऊजाध सवकास एजेंसी सलसमटे ड (IREDA) ने यूसनयन बैंक ऑफ इों सडया
(UBI) और बैंक ऑफ बडौदा (BoB) के साि समझौता ज्ञापन (MoU) पर हस्ताक्षर सकए।
i.ये समझौते IREDA को सवसभन्न प्रकार की नवीकरिीय ऊजाध पररयोजनाओों के सलए ऋि ससोंसडकेशन और सह-ऋि
दे ने में UBI और BoB के साि काम करने में सक्षम बनाएों गे , सजसमें थिासपत और अत्यार्ुसनक नवीकरिीय ऊजाध
प्रौद्योसगकी दोनोों शासमल हैं।
ii.IREDA नवीन और नवीकरिीय ऊजाध मोंत्रालय (MNRE) के प्रशाससनक सनयोंत्रि के तहत एक समनी रत्न (श्रेिी- I)
केंद्रीय सावधजसनक क्षेत्र उद्यम (CPSE) है ।

7. निस राज्य िी िाद्य प्रसंस्करण सोसायटी िे हाि ही में (नसतंबर'23 में) न्यूितम 7,500 सूक्ष्म-िाद्य
प्रसंस्करण इिाइयों िो उन्नत और स्थानपत िरिे िे निए भारतीय स्ट्े ट बैंि (SBI) िे साथ एि समझौता ज्ञापि
पर हस्ताक्षर निए हैं ?
1)महाराष्ट्र
2)गुजरात
3)आों ध्र प्रदे श
4)ओसडशा
5)पसिम बोंगाल
उत्तर-3)आं ध्र प्रदे श

Report Errors in the PDF - ebooks@affairscloud.com Copyright 2014-2023 @ AffairsCloud.com 105


स्पष्टीिरण:
आों ध्र प्रदे श सरकार (AP) के कृसष और सवपिन सवभाग के तहत आं ध्र प्रदे श खाद्य प्रसोंस्करि सोसायटी (APFPS) ने
आों ध्र प्रदे श में न्यूनतम 7,500 सूक्ष्म-खाद्य प्रसोंस्करि इकाइयोों के उन्नयन और थिापना के सलए भारतीय स्ट्े ट बैंक
(SBI) के साि एक समझौता ज्ञापन (MoU) पर हस्ताक्षर सकए।
• यह समझौता प्रर्ानमोंत्री सूक्ष्म खाद्य प्रसोंस्करि उद्यम औपचाररकीकरि (PMFME) योजना के तहत सवत्तीय
वषध 2023-24 (FY24)के दौरान सनष्पासदत सकया जाएगा।
• इस सहयोग का उद्दे श्य आों ध्र प्रदे श में खाद्य प्रसोंस्करि उद्योग के सवकास को बढावा दे ना है।
• इस MoU के सहस्से के रूप में, SBI एग्री इों फ्रास्ट्र क्चर फोंड (AIF) के तहत सूक्ष्म और लघु उद्यमोों के सलए क्रेसडट
गारों टी फोंड टर स्ट् (CGTMSE) के सदशासनदे शोों का पालन करते हुए पात्र लाभासिधयोों को 10 लाख रुपये तक के
सोंपासश्वधक-मुक्त ऋि की पेशकश करे गा।

8. उस राज्य सरिार िा िाम बताइए निसिे हाि ही में (नसतंबर'23 में) एनशया िे सबसे बडे निखस्ट्र र िूनिंग
नसस्ट्म (DCS) िो नििनसत िरिे िे निए िेशिि सेंटरि िूनिंग िंपिी PJSC (टै िीि) िे साथ एि MoU पर
हस्ताक्षर निए हैं।
1)पसिम बोंगाल
2)कनाधटक
3)मध्य प्रदे श
4)तेलोंगाना
5)हररयािा
उत्तर-4)तेिंगािा
स्पष्टीिरण:
तेिंगािा सरिार ने हैदराबाद, तेलोंगाना में एसशया के सबसे बडे सडक्कस्ट्रर कोसलोंग ससस्ट्म (DCS) को सवकससत करने
के सलए नेशनल सेंटरल कूसलोंग कोंपनी PJSC (टै िीि), अबू र्ाबी (सोंयुक्त अरब अमीरात-UAE) आर्ाररत कूसलोंग-एस-
ए-ससवधस प्रदाता के साि समझौता ज्ञापन (MoU) पर हस्ताक्षर सकए हैं ।
• टै ब्रीड हैदराबाद फामाध ससटी (HPC), तेलोंगाना के सलए 1,25,000 प्रशीतन टन (RT) शीतलन बुसनयादी ढाोंचे को
सवकससत करने के सलए 200 समसलयन अमरीकी डालर का सनवेश करने के सलए तैयार है।
• यह पररयोजना सावधजसनक सनजी भागीदारी (PPP) के आर्ार पर शुरू की जा रही है।
i.तेलोंगाना सरकार ने है दराबाद के साइबराबाद के मौजूदा और आगामी वासिक्कज्यक सजलोों और अन्य समसश्रत उपयोग
वाले सवकास क्षेत्रोों में सजला कूसलोंग बुसनयादी ढाोंचे का पता लगाने के सलए तब्रीड के साि एक MoU पर हस्ताक्षर सकए
हैं, जो 200 मेगावाट (MW) से असर्क सबजली की माोंग को कम करने की क्षमता प्रदान करते हैं ।

9. उस िंपिी िा िाम बताइए निसिे हाि ही में (नसतंबर'23 में) भारत िे निमािि क्षेत्र िो बढ़ािे िे निए
भारतीय रे ििे िे गनत शखक्त निश्वनिद्यािय (GSV) ििोदरा िे साथ एि MoU पर हस्ताक्षर निए हैं।
1)लॉकहीड मासटध न
2)बोइों ग
3)एयरबस
4)BAE ससस्ट्म्स
5)डसॉल्ट एसवएशन
उत्तर-3)एयरबस

Report Errors in the PDF - ebooks@affairscloud.com Copyright 2014-2023 @ AffairsCloud.com 106


स्पष्टीिरण:
7 ससतोंबर, 2023 को, भारतीय रे लवे के गसत शक्कक्त सवश्वसवद्यालय (GSV) ििोदरा (गुजरात) और यूरोपीय बहुराष्ट्रीय
एयरोस्पेस सनगम एयरबस ने भारत के सवमानन क्षेत्र को बढाने के सलए सहयोग के सलए एक समझौता ज्ञापन (MoU)
पर हस्ताक्षर सकए हैं।
• गसत शक्कक्त सवश्वसवद्यालय वडोदरा, गुजरात में रे ल मोंत्रालय के अर्ीन एक केंद्रीय सवश्वसवद्यालय है। यह पररवहन
और लॉसजक्कस्ट्क्स क्षेत्र में भारत का पहला सवश्वसवद्यालय है।
• MoU में वडोदरा में C295 सवमान सुसवर्ा थिासपत करने के सलए दो सोंथिाओों के बीच साझेदारी शासमल है।
• यह छात्रोों और पेशेवरोों के सलए क्षेत्र-सवसशष्ट् प्रसशक्षि कायधक्रमोों, सहयोगात्मक अनुसोंर्ान, सोंकाय के सलए उद्योग
प्रदशधन, छात्र इों टनधसशप, प्लेसमेंट और छात्रवृसत्त के सनमाधि का समिधन करे गा।

10. PM नरें द्र मोदी और पीपुल्स ररपस्किक ऑफ बांग्लादे श की PM शेि हसीना के बीच आदान-प्रदान लकए
गए समझौता ज्ञापन (MoU) के संबंि में लनम्नलिस्कित में से कौन सा लबंदु "सही" है ?
A) भारतीय राष्टरीय भुगतान लनगम (NPCI) और बांग्लादे श बैंक के बीच लिलिटि भुगतान तंत्र में सहयोग पर
समझौता ज्ञापन।
B) 2023-2025 के लिए भारत और बांग्लादे श के बीच सांिृलतक आदान-प्रदान कायाक्रम (CEP) के
नवीनीकरण पर समझौता ज्ञापन।
C)भारतीय कृलष अनुसंिान पररषद (ICAR) और बांग्लादे श कृलष अनुसंिान पररषद (BARC) ने कृलष
अनुसंिान और लशक्षा में सहयोग को बढावा दे ने के लिए एक समझौता ज्ञापन पर हस्ताक्षर लकए।
1) केवल A
2) केवल A & B
3) केवल B & C
4) केवल A & C
5) सभीA, B & C
उत्तर -5)सभी A, B & C
स्पष्टीकरण:
प्रर्ान मंत्री (PM) नरें द्र मोदी और पीपल्स ररपस्िक ऑफ बांग्लादे श की PM शेि हसीना ने भारत और बां ग्लादे श
के बीच श्चद्वपक्षीय सहयोग को मिबूत करने के श्चलए 3 समझौता ज्ञापनों (MoU) का आदान-प्रदान श्चकया।
i.PM मोदी ने बांग्लादे श की PM शेख हसीना के साि श्चद्वपक्षीय बैठक की, िो 9 से 10 श्चसतंबर 2023 तक नई श्चदल्ली में
आयोश्चित G-20 िीिसा सलमट में भाग लेने के श्चलए अश्चतश्चि के रूप में भारत आ रही िीं।
ii. 3 समझौता ज्ञापन िे,
• भारतीय राष्ट्रीय भुगतान श्चनगम (NPCI) और बांग्लादे श बैंक के बीच श्चिश्चिटल भुगतान तंत्र में सहयोग पर
समझौता ज्ञापन।
• 2023-2025 के श्चलए भारत और बांग्लादे श के बीच सांस्कृश्चतक आदान-प्रदान कायगक्रम (CEP) के
नवीनीकरि पर समझौता ज्ञापन।
• कृश्चष अनुसंर्ान और श्चशक्षा में सहयोग को ब़िावा दे ने के श्चलए भारतीय कृश्चष अनुसंर्ान पररषद (ICAR) और
बांग्लादे श कृश्चष अनुसंर्ान पररषद (BARC) के बीच समझौता ज्ञापन।

Report Errors in the PDF - ebooks@affairscloud.com Copyright 2014-2023 @ AffairsCloud.com 107


11. लकस दे श के न्यालयक कॉिेि ने हाि ही में (लसतंबर’23 में) न्यालयक लशक्षा और अनुसंिान में सहयोग को
आगे बढाने पर भारत की राष्टरीय न्यालयक अकादमी (NJA) के साथ एक समझौता ज्ञापन (MoU) पर हस्ताक्षर
लकए हैं।
1)फ्रांस
2)यूनाइटे ि श्चकंगिम
3)श्चसंगापुर
4)संयुक्त राज्य अमेररका
5)मलेश्चशया
उत्तर-3)लसंगापुर
स्पष्टीकरण:
भारत की राष्ट्रीय न्याश्चयक अकादमी (NJA), िो भारत के सवोच्च न्यायालय के श्चनदे शों के तहत काम करती है, ने
न्याश्चयक श्चशक्षा और अनुसंर्ान में सहयोग को आगे ब़िाने पर श्चसंगापुर के सवोच्च न्यायालय के लसंगापुर न्यालयक
कॉिेि के साि एक समझौता ज्ञापन (MoU) पर हस्ताक्षर श्चकए।
• CJI DY चंद्रचूड़ और श्चसंगापुर के सीिे सुंदरे श मेनन की उपस्थिश्चत में न्याश्चयक सहयोग के क्षेत्र में भारत के
सुप्रीम कोटग और श्चसंगापुर के सुप्रीम कोटग के बीच एक और समझौता ज्ञापन पर भी हस्ताक्षर श्चकए गए।
• भारत के मुख्य न्यायार्ीश (CJI) िॉ. र्नंिय यशवंत (DY) चंद्रचूड़ की श्चसंगापुर की आश्चर्काररक यात्रा के दौरान
समझौता ज्ञापन पर हस्ताक्षर श्चकए गए।

12. उस िंपिी िा िाम बताइए निसिे हाि ही में (नसतंबर’23 में) पूरे भारत में िौसेिा िनमायों और पररिारों
िी व्यखक्तगत यात्रा और आिागमि िे निए निश्वसिीय गनतशीिता समाधाि प्रदाि िरिे िे निए भारतीय
िौसेिा िे साथ एि MoU पर हस्ताक्षर निए हैं।
1)उबर
2)िािाकार
3)क्कक्वक राइड
4)ओला शेयर
5)दोगो
उत्तर- 1)उबर
स्पष्टीिरण:
भारतीय िौसेिा ने भारत भर में नौसेना कसमधयोों और पररवारोों की व्यक्कक्तगत यात्रा और आवागमन के सलए सवश्वसनीय,
सुसवर्ाजनक, सुरसक्षत और सकफायती गसतशीलता समार्ान प्रदान करने के सलए अग्रिी राइडशेयररों ग ऐप M/s उबरके
साि एक समझौता ज्ञापन (MoU) पर हस्ताक्षर सकए।
i.यह MoU नौसेना स्ट्ाफ के प्रमुख (CNS) के ‘SHIPS FIRST’ दृसष्ट्कोि के तहत 'हैप्पी पसधनेल' के दृसष्ट्कोि के
अनुरूप है और सशस्त्र बलोों में यह पहली पहल है।
ii.यह पररवतधनकारी पररवतधन के सलए प्रौद्योसगकी को अपनाने के भारत सरकार (GoI) के 'सडसजटल इों सडया' दृसष्ट्कोि
को भी बढावा दे ता है ।

Report Errors in the PDF - ebooks@affairscloud.com Copyright 2014-2023 @ AffairsCloud.com 108


13. निस राज्य िे पररिहि निभाग िे हाि ही में (नसतंबर 23 में) ओपि मोनबनिटी िेटििा िे दायरे िा निस्तार
िरिे िे निए ओपि िेटििा फॉर निनिटि िॉमसा (ONDC) िे साथ एि MoU पर हस्ताक्षर निए हैं ?
1)आों ध्र प्रदे श
2)कनाधटक
3)तसमलनाडु
4)तेलोंगाना
5)केरल
उत्तर-5)िेरि
स्पष्टीिरण:
ओपन नेटवकध फॉर सडसजटल कॉमसध (ONDC), वासिज्य और उद्योग मोंत्रालय (MoCI) के तहत उद्योग और आों तररक
व्यापार सोंवर्धन सवभाग (DPIIT) और िेरि सरकार के पररवहन सवभाग की एक पहल ने केरल में ओपन मोसबसलटी
नेटवकध के दायरे का सवस्तार करने के सलए एक समझौता ज्ञापन (MoU) पर हस्ताक्षर सकए।
i.समझौते ने केरल राज्य के बाकी सहस्सोों को कवर करने के सलए कोक्कच्च ओपन मोसबसलटी नेटवकध का सवस्तार सकया,
सजसे अब केरल ओपन मोसबसलटी नेटवकध (KOMN) कहा जाता है , सजसे नवोंबर 2022 में कोक्कच्च, केरल में लॉन्च सकया
गया िा।
ii.यह पररयोजना वतधमान में केरल में चल रहे पररवहन के सभी तरीकोों को ONDC नेटवकध पर एकीकृत करे गी और
उपयोगकताध के अनुकूल खरीदार और सवक्रेता ऐप्स के माध्यम से पररवहन के सभी तरीकोों तक सस्ती और सुसवर्ाजनक
पहुोंच प्रदान करे गी।

14. उस संगठि िा िाम बताइए निसिे हाि ही में (नसतंबर’23 में) िृनष में िे टा-संचानित ििाचारों िो बढ़ािा
दे िे िे निए राष्टरीय िृनष और ग्रामीण नििास बैंि (NABARD) िे साथ एि MoU पर हस्ताक्षर निए हैं।
1)कृसष सवकास के सलए अोंतराध ष्ट्रीय कोष
2)खाद्य एवों कृसष सोंगठन
3)सोंयुक्त राष्ट्र सवकास कायध क्रम
4)अोंतराधष्ट्रीय मु द्रा कोष
5)सोंयुक्त राष्ट्र मानव सनपटान कायधक्रम
उत्तर-3)संयुक्त राष्टर नििास िायाक्रम
स्पष्टीिरण:
12 ससतोंबर, 2023 को, सोंयुक्त राष्ट्र सवकास कायधक्रम (UNDP) और राष्ट्रीय कृसष और ग्रामीि सवकास बैं क
(NABARD) ने छोटे सकसानोों का समिधन करने के सलए कृसष और खाद्य प्रिासलयोों में डे टा-सोंचासलत नवाचारोों के
सोंयुक्त सवकास के सलए तकनीकी सहयोग पर 5 साि के समझौता ज्ञापन (MoU) पर हस्ताक्षर सकए।
i.उद्दे श्य: सटकाऊ कृसष को बढावा दे ना, सवोत्तम प्रिाओों को साझा करके और सनवेश को अनुकूसलत करके भेद्यता को
कम करना
ii.इस MoU के तहत, NABARD और UNDP छोटे सकसानोों की आजीसवका बढाने के सलए उत्पाद सवकास, प्रौद्योसगकी
हस्ताोंतरि, सामूसहक जलवायु कारध वाई और कृसष नीसत समिधन के सलए ओपन-सोसध डे टा साझा करें गे।

Report Errors in the PDF - ebooks@affairscloud.com Copyright 2014-2023 @ AffairsCloud.com 109


15. नसतंबर 2023 में, श्रम और रोिगार मंत्रािय (MoLE) िे अपिे राष्टरीय िररयर सेिा (NCS) पोटा ि िे साथ
एिीिृत िरिे िे निए निनभन्न िौिरी पोटा िों, नियोक्ताओं और िौशि प्रदाताओं िे साथ MoU पर हस्ताक्षर
निए।
NCS पोटा ि िे साथ एिीिृत िरिे िे निए निम्ननिखित में से निस संस्था िे MoU पर हस्ताक्षर निए थे?
1)हायरमी
2)TCS iON
3)रीलेवल
4)1 और 2 दोनोों
5)2 और 3 दोनोों
उत्तर- 4)1 और 2 दोिों
स्पष्टीिरण:
12 ससतोंबर, 2023 को श्रम और रोजगार मोंत्रालय (MoLE) ने अपने राष्ट्रीय कैररयर सेवा (NCS) पोटध ल के साि एकीकृत
करने के सलए सवसभन्न नौकरी पोटध लोों, सनयोक्ताओों और कौशल प्रदाताओों के साि समझौता ज्ञापन (MoU) पर हस्ताक्षर
सकए।
i.यह सहयोग सनजी नौकरी पोटध लोों को NCS प्लेटफॉमध पर अपनी नौकरी के अवसर साझा करने की अनुमसत दे ता है ,
सजससे NCS में पोंजीकृत नौकरी चाहने वालोों के सलए रोजगार के अवसरोों का सवस्तार होता है ।
ii.इस साझेदारी से असोंगसठत क्षेत्र के 30 लाख से असर्क ईश्रम पोंजीकृत श्रसमकोों को लाभ होगा जो NCS में शासमल हो
गए हैं।
iii.वे सोंथिाएाँ सजनके साि MoU पर हस्ताक्षर सकए गए:
• टीमलीज HRटे क (फ्रेशसधवडध )
• मॉन्स्टर डॉट कॉम इों सडया प्राइवेट सलसमटे ड को फाउों डइट कहा जाता है
• QUESS CORP सलसमटे ड
• सडसलवरी टर ै क (VSS टे क)
• KARPAGA असेसमेंट APP MATRIX ससवधसेज प्राइवेट सलसमटे ड (हायरमी)
• QUIKR INDIA प्राइवेट सलसमटे ड
• TCS iON
• Firstjob.co.in

16. उस राज्य सरिार िा िाम बताइए निसिे हाि ही में (नसतंबर'23 में) सभी बैंि रनहत ग्राम पंचायतों (GP)
में बैंनिंग सेिाएं प्रदाि िरिे िे निए 6 सािािनिि क्षेत्र िे बैंिों (PSB) िे साथ एि MoU पर हस्ताक्षर निए हैं।
1)आों ध्र प्रदे श
2)पसिम बोंगाल
3)गुजरात
4)कनाधटक
5)ओसडशा
उत्तर-5)ओनिशा

Report Errors in the PDF - ebooks@affairscloud.com Copyright 2014-2023 @ AffairsCloud.com 110


स्पष्टीिरण:
13 ससतोंबर 2023 को, ओनिशा सरकार ने ग्राहक सेवा प्वाइों ट-प्लस (CSP प्लस) बैंसकोंग आउटलेट के माध्यम से ओसडशा
के सभी बैंक रसहत ग्राम पोंचायतोों (GP) में बैंसकोंग सेवाएों प्रदान करने के सलए 6 सावधजसनक क्षेत्र के बैंकोों (PSB) के साि
एक समझौता ज्ञापन (MoU) पर हस्ताक्षर सकए।
i.6 PSB : भारतीय स्ट्े ट बैंक (SBI), पोंजाब नेशनल बैंक (PNB), यूसनयन बैंक ऑफ इों सडया (UBI), UCO बैंक ऑफ
इों सडया, बैंक ऑफ इों सडया (BoI), और बैंक ऑफ बडौदा (BoB) हैं।
ii.ओसडशा सभी ग्राम पोंचायतोों को बैंसकोंग सेवाएों प्रदान करने के सलए ऐसा अनुकरिीय मॉडल अपनाने वाला भारत का
पहला राज्य है।
• सवत्तीय समावेशन सरकार का उच्च प्रािसमकता वाला एजेंडा है। यह एक महत्वपूिध चुनौती बनी हुई है क्ोोंसक
ओसडशा में बैंसकोंग पहुोंच काफी कम है और सभी सजलोों में एक समान नहीों है।
• 6798 GP में से, लगभग 65% GP (4373 GP) में ईोंट-और-मोटाधर शाखाएों नहीों हैं ।

17. निस भारतीय प्रौद्योनगिी संस्थाि (IIT) िे हाि ही में (नसतंबर’23 में) भारत में िाद्य सुरक्षा, पोषण,
िििायु िचीिापि और आिीनििा में सुधार िे निए संयुक्त राष्टर निश्व िाद्य िायाक्रम (WFP) िे साथ एि
MoU पर हस्ताक्षर निए हैं ?
1)IIT सदल्ली
2)IIT कानपुर
3)IIT खडगपुर
4)IIT मद्रास
5)IIT बॉम्बे
उत्तर-5)IIT बॉम्बे
स्पष्टीिरण:
सोंयुक्त राष्ट्र सवश्व खाद्य कायधक्रम (WFP) और मुोंबई (महाराष्ट्र) में भारतीय प्रौद्योसगकी सोंथिान, बॉम्बे (IITB) ने भारत में
खाद्य सुरक्षा, पोषि, जलवायु लचीलापन और आजीसवका में सुर्ार के सलए एक समझौता ज्ञापन (MoU) पर हस्ताक्षर
सकए हैं।
i.MoU िा उद्दे श्य:
• खाद्य और पोषि सुरक्षा से सोंबोंसर्त साक्ष् की पहचान, सवश्लेषि और उत्पन्न करने के सलए एक व्यापक ज्ञान
प्रबोंर्न प्रिाली थिासपत करना है।
• सरकारी खाद्य सुरक्षा नेट कायधक्रमोों का समिधन करने के सलए प्रौद्योसगकी और नवाचारोों के सलए एक पाररक्कथिसतकी
तोंत्र सवकससत करना है।
• खाद्य और पोषि सुरक्षा पर कारध वाई योग्य साक्ष् उत्पन्न करने के सलए प्रशाससनक डे टा सेट का उपयोग करके
वास्तसवक समय सनगरानी प्रिाली और डै शबोडध बनाना है।

18. उस कंपनी का नाम बताइए लिसने हाि ही में (लसतंबर'23 में) क्रमशः हाइिर ोिन ईंिन सेि फेरी और
मीलियम-स्पीि इं िन के लिए िॉयि् स रलिस्ट्र और कैटरलपिर इं क के साथ दो गैर-बाध्यकारी समझौता
ज्ञापन (MoU) पर हस्ताक्षर लकए हैं।
1)कोचीन श्चशपयािग श्चलश्चमटे ि
2)गोवा श्चशपयािग श्चलश्चमटे ि
3)गािग न रीच श्चशपश्चबर्ल्सग एं ि इं िीश्चनयसग श्चलश्चमटे ि

Report Errors in the PDF - ebooks@affairscloud.com Copyright 2014-2023 @ AffairsCloud.com 111


4)मझगांव िॉक श्चशपश्चबर्ल्सग श्चलश्चमटे ि
5)श्चहंदुस्तान श्चशपयािग श्चलश्चमटे ि
उत्तर-3)गािा न रीच लशपलबल्डसा एं ि इं िीलनयसा लिलमटे ि
स्पष्टीकरण:
गािग न रीच श्चशपश्चबर्ल्सग एं ि इं िीश्चनयसग (GRSE) लिलमटे ि ने लंदन, यूनाइटे ि श्चकंगिम (UK) में रक्षा और सुरक्षा
उपकरि अंतरागष्ट्रीय (DSEI) प्रदशगनी के दौरान श्चवदे शी कंपश्चनयों के साि दो गैर-बाध्यकारी समझौता ज्ञापनों (MoU)
पर हस्ताक्षर श्चकए, िो लंदन के रॉयल श्चवरोररया िॉक में एक्सेल में 12 से 15 श्चसतंबर 2023 तक हुआ िा।
i.ये MoU नवीन हररत ऊिाग प्रौद्योश्चगश्चकयों को आगे ब़िाने और श्चटकाऊ, लागत प्रभावी वैकस्ल्पक ईंर्न को ब़िावा दे ने
की भारत की प्रश्चतबिता के अनुरूप हैं।
ii.दो गैर-बाध्यकारी और गोपनीय प्रकृश्चत के MoU:
• हाइिर ोिन ईंिन सेि फ़ेरी के श्चवकास के श्चलए िॉयि् स रलिस्ट्र के साि।
• श्चवशेष रूप से भारतीय नौसेना (IN) और भारतीय तटरक्षक (ICG) के श्चलए मीलियम-स्पीि इं िन के उत्पादन,
श्चबक्री और सेवा में संभाश्चवत सहयोग के श्चलए कैटरलपिर इं क के साि।

19. उस संगठन का नाम बताइए लिसने हाि ही में (लसतंबर’23 में) एक उभरती अथाव्यवस्था कायाक्रम
लवकलसत करने के लिए इं लिया ग्लोबि फोरम (IGF) के साथ एक Mou पर हस्ताक्षर लकए हैं।
1)अंतरागष्ट्रीय मुद्रा कोष
2)श्चवश्व बैंक
3)अंतरागष्ट्रीय श्रम संगठन
4)श्चवश्व सरकार श्चशखर सम्मेलन
5)श्चवश्व बौस्िक संपदा संगठन
उत्तर-4)लवश्व सरकार लशिर सम्मेिन
स्पष्टीकरण:
20 श्चसतंबर, 2023 को लंदन, यूनाइटे ि श्चकंगिम (UK) स्थित इं श्चिया ग्लोबल फोरम (IGF) और श्चवश्व सरकार श्चशखर
सम्मेलन (WGS) ने एक उभरती अिगव्यवथिा कायगक्रम श्चवकश्चसत करने के श्चलए एक समझौता ज्ञापन (MoU) पर
हस्ताक्षर श्चकए हैं ।
i.यह साझेदारी IGF को दु श्चनया भर में WGS के ज्ञान भागीदारों में शाश्चमल होने की भी अनुमश्चत दे गी। यह अंतराग ष्ट्रीय
सहयोग और सवोत्तम प्रिाओं को साझा करने को भी ब़िावा दे गा।
ii.उच्च-स्तरीय चचााएँ: यह पहल सरकारी अश्चर्काररयों, व्यापाररक नेताओं, अिगशास्स्त्रयों और श्चवशेषज्ञों को शाश्चमल
करते हुए सतत आश्चिगक श्चवकास, भू-आश्चिगक दृश्चष्ट्कोि और नवीन समार्ानों पर चचाग की सुश्चवर्ा प्रदान करे गी।
iii.ग्लोबि साउथ िे विपमेंट: इसका उद्दे श्य ग्लोबल साउि में सतत आश्चिगक श्चवकास और श्चवकास को ब़िावा दे ना है।

20. भारत सरिार िे निस निभाग िे हाि ही में (नसतंबर’23 में) िशा मुक्त भारत अनभयाि (NMBA) िा
संदेश फैिािे िे निए अखिि निश्व गायत्री पररिार िे साथ एि MoU पर हस्ताक्षर निए हैं ?
1)गृह सवभाग
2)न्याय सवभाग
3)कानूनी मामलोों का सवभाग
4)राजभाषा सवभाग
5)सामासजक न्याय & असर्काररता सवभाग

Report Errors in the PDF - ebooks@affairscloud.com Copyright 2014-2023 @ AffairsCloud.com 112


उत्तर-5)सामानिि न्याय & अनधिाररता निभाग
स्पष्टीिरण:
22 ससतोंबर, 2023 को, सामासजक न्याय और असर्काररता मोंत्रालय (MoSJ&E) के तहत सामानिि न्याय &
अनधिाररता निभाग और अक्कखल सवश्व गायत्री पररवार ने अक्कखल सवश्व गायत्री पररवार के बैनर तले युवाओों, मसहलाओों,
छात्रोों और समुदाय के बीच नशा मुक्त भारत असभयान (NMBA) के सोंदेश को फैलाने के सलए एक समझौता ज्ञापन
(MoU) पर हस्ताक्षर सकए।
i.सदल्ली के नई सदल्ली क्कथित डॉ. अोंबेडकर इों टरनेशनल सेंटर में केंद्रीय मोंत्री डॉ. वीरें द्र कुमार, MoSJ&E, डॉ. सचन्मय
पोंड्या, अक्कखल सवश्व गायत्री पररवार ससहत अन्य लोगोों की उपक्कथिसत में MoU पर हस्ताक्षर सकए गए।
i.AIIMS (अक्कखल भारतीय आयुसवधज्ञान सोंथिान), नई सदल्ली में राष्ट्रीय औषसर् सनभधरता उपचार केंद्र (NDDTC) के
माध्यम से सामासजक न्याय और असर्काररता सवभाग िारा मादक द्रव्योों के उपयोग पर सकए गए पहले व्यापक राष्ट्रीय
सवेक्षि के अनुसार, भारत में शराब सबसे असर्क इस्तेमाल सकया जाने वाला मनो-ससक्रय पदािध है , इसके बाद
कैनसबस और ओसपओइड हैं।

21. निस संगठि/सरिारी एिेंसी िे हाि ही में (नसतंबर'23 में) व्यापार, नििेश, गुणित्ता माििों और
तििीिी हस्तांतरण िे क्षेत्र में सहयोग नििनसत िरिे िे निए ईराि-भारत संयुक्त चैंबर ऑफ िॉमसा िे
साथ एि MoU पर हस्ताक्षर निए हैं ?
1)फ़ूड सेफ्टी एों ड स्ट्ैं डड्ध स अिॉररटी ऑफ इों सडया
2)फेडरे शन ऑफ इों सडयन चैंबसध ऑफ कॉमसध एों ड इों डस्ट्र ी
3)एग्रीकल्चरल एों ड प्रोसेथड फ़ूड प्रोडर् स एक्सपोटध डे वलपमेंट अिॉररटी
4)फेडरे शन ऑफ इों सडयन एक्सपोटध ऑगधनाइजेशन
5)कॉन्क्फ़ेडरे शन ऑफ इों सडयन इों डस्ट्र ी
उत्तर-4)फेिरे शि ऑफ इं नियि एक्सपोटा ऑगािाइिेशि
स्पष्टीिरण:
22 ससतोंबर, 2023 को, ईराि-भारत संयुक्त चैंबर ऑफ िॉमसा ने व्यापार, सनवेश, गुिवत्ता मानकोों और तकनीकी
हस्ताोंतरि क्षेत्रोों में सहयोग सवकससत करने के सलए नई सदल्ली क्कथित फेडरे शन ऑफ इों सडयन एक्सपोटध ऑगधनाइजेशन
(FIEO) के साि एक समझौता ज्ञापन (MoU) पर हस्ताक्षर सकए।
i.MoU पर ईरान-भारत सोंयुक्त चैंबर ऑफ कॉमसध के उपाध्यक्ष महदी रों गरोना; और FIEO के महासनदे शक (DG) और
मुख्य कायधकारी असर्कारी (CEO) अजय शाई ने हस्ताक्षर सकए।

22. उस संस्थाि िा िाम बताइए निसिे हाि ही में (नसतंबर'23 में) इं िीनियररं ग िे निनभन्न क्षेत्रों में
अिादनमि सहयोग िो बढ़ािा दे िे और रक्षा से संबंनधत प्रौद्योनगनियों िी िैज्ञानिि समझ में सुधार िे निए
भारतीय िौसेिा (IN) िे साथ एि MoU पर हस्ताक्षर निए हैं।
1)भारतीय सवज्ञान सोंथिान (IISc) बेंगलुरु
2)भारतीय प्रौद्योसगकी सोंथिान (IIT) बॉम्बे
3)भारतीय प्रौद्योसगकी सोंथिान (IIT) सदल्ली
4)भारतीय प्रौद्योसगकी सोंथिान (IIT) खडगपुर
5)भारतीय प्रौद्योसगकी सोंथिान (IIT) कानपुर
उत्तर -1)भारतीय निज्ञाि संस्थाि (IISc) बेंगिुरु

Report Errors in the PDF - ebooks@affairscloud.com Copyright 2014-2023 @ AffairsCloud.com 113


स्पष्टीिरण:
इों जीसनयररों ग पर सवसभन्न क्षेत्रोों में अकादसमक सहयोग को बढावा दे ने, रक्षा से सोंबोंसर्त प्रौद्योसगसकयोों की वैज्ञासनक समझ में
सुर्ार करने और नई प्रौद्योसगसकयोों का सनमाधि करने के सलए भारतीय नौसेना (IN) और भारतीय सवज्ञान सोंथिान (IISc),
बेंगिुरु, ििााटि के बीच एक समझौता ज्ञापन (MoU) पर हस्ताक्षर सकए गए।
i.MoU पर IN की ओर से सहायक चीफ ऑफ मेटेररयल (डॉकयाडध & रे सफट) ररयर एडसमरल K श्रीसनवास और IISc के
रसजस्ट्र ार कैप्टन श्रीर्र वाररयर (सेवासनवृत्त) और फाउों डेशन फॉर साइों स इनोवेशन एों ड डे वलपमेंट (FSID) बेंगलुरु के
सनदे शक प्रोफेसर B गुरुमूसतध ने हस्ताक्षर सकए। FSID को IISc िारा बढावा सदया जाता है।
ii.प्राकृसतक रे सफ्रजरें ट पर काम करने वाले भसवष्य के सलए तैयार टर ाोंससक्रसटकल काबधन डाइऑक्साइड (CO2)आर्ाररत
एयर कोंडीशनर (AC) सोंयोंत्र के सवकास के सलए IISc के साि सहयोग करता है।

23.उस िंपिी िा िाम बताइए निसिे हाि ही में (नसतंबर’23 में) नशक्षा मंत्रािय (MoE), और िौशि नििास
और उद्यनमता मंत्रािय (MSDE) िे तहत निभागों िे साथ 8 समझौता ज्ञापिों (MoU) पर हस्ताक्षर निए, तानि
भारतीय युिाओं िो भनिष्य िे निए तैयार निनिटि िौशि िे साथ सशक्त बिािे िे निए अिुरूप पाठ्यक्रम
पेश निए िा सिें।
1)इन्फोससस
2)मेटा
3)माइक्रोसॉफ्ट
4)IBM
5)गूगल
उत्तर- 4)IBM
स्पष्टीिरण:
IBM (इों टरनेशनल सबजनेस मशीन कॉपोरे शन) ने सशक्षा मोंत्रालय (MoE), और कौशल सवकास और उद्यसमता मोंत्रालय
(MSDE) के तहत सवभागोों के साि 8 समझौता ज्ञापनोों (MoU) पर हस्ताक्षर सकए, तासक भारतीय युवाओों को भसवष्य के
सलए तैयार सडसजटल कौशल के साि सशक्त बनाने के सलए अनुरूप पाठ्यक्रम पेश सकए जा सकें।
i.यह साझेदारी आसटध सफसशयल इों टेसलजेंस (AI) (जेनरे सटव AI ससहत), साइबर सुरक्षा, क्लाउड कोंप्यूसटों ग और
व्यावसासयक सवकास जैसी उभरती प्रौद्योसगसकयोों पर स्कूली सशक्षा, उच्च सशक्षा और व्यावसासयक कौशल में सशक्षासिधयोों
के सलए IBM के सीखने के मोंच IBM खस्कल्सनबड के सह-सनमाधि और पहुोंच पर ध्यान केंसद्रत करती है।

24. निस दे श िे चुिाि आयोग िे हाि ही में (नसतंबर'23 में) चुिािी प्रबंधि और प्रशासि िो बढ़ािे में
सहयोग िे निए भारत िे चुिाि आयोग (ECI) िे साथ एि समझौता ज्ञापि (MoU) पर हस्ताक्षर निए हैं।
1)सेशेल्स
2)श्रीलोंका
3)दसक्षि अफ़्ीका
4)ससोंगापुर
5)मालदीव
उत्तर-1)सेशेल्स
स्पष्टीिरण:
सेशेल्स के चुनाव आयोग (ECS) ने चुनावी प्रबोंर्न और प्रशासन को बढाने में सहयोग के सलए भारत के चुनाव आयोग
(ECI) के साि एक समझौता ज्ञापन (MoU) पर हस्ताक्षर सकए हैं।

Report Errors in the PDF - ebooks@affairscloud.com Copyright 2014-2023 @ AffairsCloud.com 114


• MoU पर ECS के अध्यक्ष डै नी लुकास और ECI के मुख्य चुनाव आयुक्त राजीव कुमार ने हस्ताक्षर सकए।
• इससे चुनाव प्रबोंर्न और असर्काररयोों के प्रसशक्षि में सवोत्तम प्रिाओों का आदान-प्रदान भी मजबूत होगा।

25. निस राज्य सरिार िे हाि ही में (नसतंबर'23 में) अपिी भू-तापीय क्षमता िा पता िगािे और नििनसत
िरिे िे निए िॉिेनियि नियोटे खक्निि इं स्ट्ीट्यूट (NGI) िे साथ एि MoU पर हस्ताक्षर निए हैं ?
1)आों ध्र प्रदे श
2)असम
3)कनाधटक
4)अरुिाचल प्रदे श
5)पसिम बोंगाल
उत्तर-4)अरुणाचि प्रदे श
स्पष्टीिरण:
सवश्व पयधटन सदवस (WTD) 2023 (27 ससतोंबर 2023)के अवसर पर, अरुणाचि प्रदे श सरकार ने राज्य में उपलब्ध गमध
झरनोों के माध्यम से अपनी भू -तापीय क्षमता का पता लगाने और सवकससत करने के सलए नॉवेसजयन सजयोटे क्कक्नकल
इों स्ट्ीट्यूट (NGI ) के साि एक समझौता ज्ञापन (MoU) पर हस्ताक्षर सकए।
i.MoU का उद्दे श्य WTD 2023 का सवषय "टू ररज्म एों ड ग्रीन इों वेस्ट्मेंट” से मेल खाता है।
ii.रॉयल नॉवेसजयन दू तावास, नई सदल्ली, NGI के माध्यम से पररयोजना का तकनीकी रूप से समिधन करे गा।
iii.अरुिाचल प्रदे श की ओर से, MoU पर रे पो रोन्या (राज्य में सवज्ञान और प्रौद्योसगकी ससचव) और रासजोंदर भसीन (NGI
के एक तकनीकी सवशेषज्ञ) ने हस्ताक्षर सकए।

APPOINTMENTS AND RESIGNATIONS


1. अगस्त 2023 में, ििरि िाइस ओनिगुई न्गुएमा िो ____________ (दे श) िा संक्रमणिािीि राष्टरपनत नियुक्त
निया गया था।
1)इसियोसपया
2)गैबॉन
3)नाइजीररया
4)तोंजासनया
5)केन्या
उत्तर- 2)गैबॉि
स्पष्टीिरण:
30 अगस्त, 2023 को, गैबोनीज़ ररपक्किकन गाडध के प्रमुख ििरि िाइस ओनिगुई न्गुएमा को सवधसम्मसत से गैबॉि
के सोंक्रमिकालीन राष्ट्रपसत और सोंथिानोों के सों क्रमि और बहाली के सलए ससमसत के अध्यक्ष के रूप में सनयुक्त सकया
गया िा।
i.वह सोमवार 4 ससतोंबर, 2023 को सोंवैर्ासनक न्यायालय के समक्ष सों क्रमिकालीन राष्ट्रपसत के रूप में शपि लेंगे।
ii.अफ्रीकी सोंघ (AU) की शाोंसत और सुरक्षा पररषद ने सैन्य तख्तापलट के बाद गैबॉन को "तुरोंत सनलोंसबत" करने का
फैसला सकया है ।

Report Errors in the PDF - ebooks@affairscloud.com Copyright 2014-2023 @ AffairsCloud.com 115


2. हाि ही में (अगस्त’23 में) रे ििे बोिा िी पहिी मनहिा अध्यक्ष और मुख्य िायािारी अनधिारी (CEO) िे रूप
में निसे नियुक्त निया गया है ?
1)जया वमाध ससन्ा
2)मार्बी पुरी बुच
3)रे खा शमाध
4)ममता कुमारी
5)मीनाक्षी नेगी
उत्तर- 1)िया िमाा नसन्ा
स्पष्टीिरण:
कैसबनेट की सनयुक्कक्त ससमसत (ACC) ने रे लवे बोडध के अध्यक्ष और मुख्य कायधकारी असर्कारी (CEO) के रूप में िया
िमाा नसन्ा की सनयुक्कक्त को मोंजूरी दे दी है। 1905 में रे लवे बोडध की थिापना के बाद से वह इस पद को सोंभालने वाली
पहली मसहला बनीों।
i.उन्ोोंने असनल कुमार लाहोटी की जगह ली सजनका अध्यक्ष और CEO के रूप में कायधकाल 31 अगस्त 2023 को
समाप्त हो गया।
ii.जया वमाध ससन्ा 31 अगस्त 2024 तक इस पद पर रहेंगी। पहले उन्ें 1 अरू बर 2023 को सेवासनवृत्त होना िा, लेसकन
उनका कायधकाल समाप्त होने तक उन्ें सफर से सनयुक्त सकया जाएगा।
iii.इस सनयुक्कक्त से पहले, वह रे लवे बोडध में सदस्य (सोंचालन और व्यवसाय सवकास) के रूप में कायधरत िीों।

3. नसतंबर 2023 में, िेंद्र सरिार िे आम (िोिसभा) और राज्य चुिाि एि साथ िरािे िे निए "एि राष्टर एि
चुिाि" िी संभाििा तिाशिे िे निए _____________ िे िेतृत्व में एि सनमनत िा गठि निया।
1)मोहम्मद हासमद अोंसारी
2)वेंकैया नायडू
3)राम नाि कोसवन्द
4)प्रसतभा पासटल
5)सुशील कुमार सशोंदे
उत्तर-3)राम िाथ िोनिन्द
स्पष्टीिरण:
1 ससतोंबर, 2023 को िेंद्र सरिार ने "एि राष्टर एि चुिाि" की सोंभावना तलाशने के सलए भारत के पूवध राष्ट्रपसत राम
िाथ िोनिन्द के नेतृत्व में एक ससमसत का गठन सकया, सजसका उद्दे श्य आम (लोकसभा) और राज्य चुनाव एक साि
कराना है।
• इसकी जानकारी केंद्रीय मोंत्री प्रह्लाद जोशी, सोंसदीय कायध मोंत्रालय ने साझा की।
i.केंद्र सरकार एक राष्ट्र, एक चुनाव पर एक सवर्ेयक पेश करने की योजना बना रही है सजसमें लोकसभा और सभी राज्य
सवर्ानसभाओों के सलए एक ही सदन या एक सवसशष्ट् समय सीमा के भीतर एक साि चुनाव कराने की पररकल्पना की गई
है।
ii.भारत में, जब मौजूदा सरकार का कायधकाल समाप्त हो जाता है या सकसी कारि से वह भोंग हो जाती है , तो सोंसद
सदस्योों (MP) के सलए आम चुनाव और राज्य सवर्ानसभा चुनाव अलग-अलग आयोसजत सकए जाते हैं।
iii.यह अवर्ारिा पूरी तरह से नई नहीों है , क्ोोंसक भारत में पहले 1951-52, 1957, 1962 और 1967 में एक साि
लोकसभा और सवर्ानसभा चुनाव हुए िे, हालाोंसक समय से पहले सवघटन के कारि उन्ें बोंद कर सदया गया िा।

Report Errors in the PDF - ebooks@affairscloud.com Copyright 2014-2023 @ AffairsCloud.com 116


4. हाि ही में (नसतंबर’23 में) निस दे श िे थमाि शिमुगरत्नम िो 9िें राष्टरपनत िे रूप में चुिा है ?
1)मलेसशया
2)ससोंगापुर
3)िाईलैंड
4)इों डोनेसशया
5)सफलीपीोंस
उत्तर- 2)नसंगापुर
स्पष्टीिरण:
भारतीय मूल के ससोंगापुर में जन्मे अिधशास्त्री थमाि शिमुगरत्नम (66 वषध) को 2.48 समसलयन से असर्क वैर् वोटोों में से
70.4% (1,746,427 वोट) हाससल करने के बाद नसंगापुर के 9िें राष्टरपनत के रूप में चुना गया है।
i.वह 2017 में सबना सकसी प्रसतयोसगता के सनवाधसचत ससों गापुर की पहली मसहला राष्ट्रपसत हलीमा याकूब की जगह लेंगे,
सजनका कायधकाल 13 ससतोंबर, 2023 को समाप्त होगा।
ii.िमधन शनमुगरत्नम ने ससोंगापुर सरकार के सनवेश सनगम (GIC) के मुख्य सनवेश असर्कारी (CIO) के पूवध मुख्य सनवेश
असर्कारी Ng कोक सोोंग को हराया, सजनके पास 15.72% वोट (390,041)िे, और टै न सकन सलयान, NTUC आय
मुख्य कायधकारी असर्कारी (CEO), सजन्ोोंने 13.88% वोट (344,292)हाससल सकए।

5. नसतंबर 2023 में, दीपि गुप्ता िे __________________ (बैंि) िे अंतररम MD और मुख्य िायािारी अनधिारी
(CEO) िे रूप में पदभार संभािा।
1)बोंर्न बैंक
2)इों डसइों ड बैंक
3)आरबीएल बैंक
4)करूर वैश्य बैंक
5)कोटक मसहोंद्रा बैंक
उत्तर-5)िोटि मनहंद्रा बैंि
स्पष्टीिरण:
िोटि मनहंद्रा बैंि सलसमटे ड के सोंयुक्त प्रबोंर् सनदे शक (MD) दीपि गुप्ता ने 31 सदसोंबर 2023 तक बैंक के अोंतररम
MD और मुख्य कायधकारी असर्कारी (CEO) का पदभार सोंभाला। वह तब तक इस पद पर बने रहें गे जब तक भारतीय
ररजवध बैंक (RBI) इसके उत्तरासर्कारी का प्रस्ताव नहीों करता।
i.दीपक गुप्ता की सनयुक्कक्त 1 ससतोंबर 2023 से MD & CEO के पद से उदय कोटक के शीघ्र इस्तीफे के बाद हुई है। MD
& CEO के रूप में उनका कायधकाल 31 सदसोंबर 2023 को समाप्त हो रहा है ।
ii.उदय कोटक गैर-कायधकारी सनदे शक के रूप में कोटक मसहों द्रा बैंक का सहस्सा बने रहेंगे।

6. हाि ही में (नसतंबर’23 में) नसंगापुर अंतरााष्टरीय मध्यस्थता िेंद्र (SIMC) िे अंतरााष्टरीय मध्यस्थता पैिि िे
सदस्य िे रूप 1में निसे नियुक्त निया गया है ?
1)S.अब्दु ल नजीर
2)NV रमि
3)अजय रस्तोगी
4)कृष्ण मुरारी
5)सदनेश माहेश्वरी

Report Errors in the PDF - ebooks@affairscloud.com Copyright 2014-2023 @ AffairsCloud.com 117


उत्तर- 2)NV रमण
स्पष्टीिरण:
भारत के पूिा मुख्य न्यायार्ीश (CJI) नुिलपसत वेंकट रमि (NV रमण) को ससोंगापुर अोंतराध ष्ट्रीय मध्यथिता केंद्र (SIMC)
के अोंतराधष्ट्रीय मध्यथिता पैनल का सदस्य नियुक्त सकया गया है।
• ससोंगापुर में SIMC के अध्यक्ष जॉजध सलम िारा उन्ें सनयुक्कक्त पत्र प्रस्तुत सकया गया।
i.न्यायार्ीश रमना ने ससोंगापुर कन्वेंशन वीक (SC वीक), ससोंगापुर के कानून मोंत्रालय (समनलॉ), अोंतराध ष्ट्रीय व्यापार कानून
पर सोंयुक्त राष्ट्र आयोग (UNCITRAL) और 20 से असर्क भागीदार सोंगठनोों िारा आयोसजत वासषधक सम्मेलन में भाग
सलया।
i.भारत के प्रमुख बहुराष्ट्रीय सनगम (MNC) न्यायमूसतध रमि के नेतृत्व में मध्यथिता पहल का समिधन करने के सलए एक
साि आए हैं।

7. उस व्यखक्तत्व िा िाम बताइए निसे हाि ही में (नसतंबर’23 में) स्वराि टर ै रसा िा िांि एं बेसिर नियुक्त
निया गया है।
1)आसदत्य रॉय कपूर
2)रोसहत शमाध
3)अक्षय कुमार
4)महेंद्र ससोंह र्ोनी
5)सैफ अली खान
उत्तर-4)महेंद्र नसंह धोिी
स्पष्टीिरण:
श्री आनोंद मसहों द्रा की अध्यक्षता वाली मसहों द्रा & मसहोंद्रा सलसमटे ड की सहायक कोंपनी स्वराि टर ै रसा ने पूवध भारतीय
कप्तान महेंद्र नसंह धोिी को अपना नया ब्राोंड एों बेसडर नासमत सकया है। यह घोषिा कोंपनी के हल्के टर ै ररोों के लॉन्च
के दौरान की गई िी।
i.स्वराज टर ै रसध के अनुसार, यह ररश्ता भारतीय सकसानोों को अत्यार्ुसनक और भरोसेमोंद मशीनीकरि समार्ान ii.प्रदान
करने के प्रसत उसके समपधि को मजबूत करे गा।
ii.ब्राोंड एों बेसडर के रूप में, MS र्ोनी ब्राोंड के सलए नए लॉन्च सकए गए सवज्ञापन असभयान में शासमल होोंगे।

8. नसतंबर 2023 में, सातो नििमैि िो 5िें िायािाि िे निए ___________ (दे श) िे प्रधाि मंत्री (PM) िे रूप
में चुिा गया था।
1)वानूआतू
2)तुवालु
3)पलाऊ
4)समोआ
5)सफजी
उत्तर- 1)िािूआतू
स्पष्टीिरण:
4 ससतोंबर 2023 को, वानूआतू गिराज्य की सोंसद ने पूवध प्रर्ान मोंत्री (PM) और पीपुल्स प्रोग्रेससव पाटी के नेता सातो
सकलमैन को वानूआतू के नए PM के रूप में चुना। यह िािूआतू के PM के रूप में सातो नििमैि िा 5 िां िायािाि
है।

Report Errors in the PDF - ebooks@affairscloud.com Copyright 2014-2023 @ AffairsCloud.com 118


i.इससे पहले, उन्ोोंने सदसोंबर 2010 से अप्रैल 2011 तक; मई से जून 2011; जून 2011 से माचध 2013; और 2015 से
2016 तक PM के रूप में कायध सकया।
ii.गुप्त मतदान में सातो सकलमैन को 23 के मुकाबले 27 वोट चुने गए। यह पररिाम अपील की अदालत िारा सनवतधमान
PM इश्माएल कलसाकाऊ के क्कखलाफ असवश्वास प्रस्ताव के पररिामोों को बरकरार रखने के बाद आया।

9. हाि ही में (अगस्त’23 में) िेशिि एसोनसएशि ऑफ सॉफ्टिेयर एं ि सनिास िंपिीि (NASSCOM) िे
अध्यक्ष िे रूप में निसे नियुक्त निया गया है ?
1)R चन्द्रशेखर
2)राजेश नाोंसबयार
3)रे खा M मेनन
4)ररशद प्रेमजी
5)केशव मुरुगेश
उत्तर- 2)रािेश िांनबयार
स्पष्टीिरण:
नेशनल एसोससएशन ऑफ सॉफ्टवेयर एों ड ससवधस कोंपनीज (NASSCOM) ने िॉनग्निेंट इं निया के अध्यक्ष और प्रबोंर्
सनदे शक (CMD) रािेश िांनबयार को अपना अध्यक्ष सनयुक्त सकया है ।
• वह अिंत माहेश्वरी (माइक्रोसॉफ्ट इों सडया के पूवध अध्यक्ष) का थिान लेंगे, सजन्ें अप्रैल 2023 में NASSCOM के
अध्यक्ष (2023-2024)के रूप में सनयुक्त सकया गया िा।
• अप्रैल 2023 में, राजेश नाोंसबयार को 2023-24 की अवसर् के सलए NASSCOM के उपाध्यक्ष के रूप में सनयुक्त
सकया गया िा।
• वह NASSCOM अध्यक्ष दे बजानी घोष के साि 2023-2025 के सलए NASSCOM कायधकारी पररषद का समिधन
करें गे।
• कायधकारी पररषद का लक्ष् 2030 तक सॉफ्टवेयर उद्योग को 500 सबसलयन अमेररकी डॉलर का बनाना है।

10. उस मंत्रािय िा िाम बताइए निसिे हाि ही में (नसतंबर'23 में) सुप्रीम िोटा (SC) िे तदथा निशेषज्ञ पैिि
िी िगह एि िई स्थायी िेंद्रीय अनधिार प्राप्त सनमनत (CEC) िी स्थापिा िी है।
1)कानून और न्याय मोंत्रालय
2)सोंसदीय कायध मोंत्रालय
3)पयाधवरि, वन और जलवायु पररवतधन मोंत्रालय
4)गृह मोंत्रालय
5)सामासजक न्याय और असर्काररता मोंत्रालय
उत्तर-3)पयाािरण, िि और िििायु पररिताि मंत्रािय
स्पष्टीिरण:
पयाधवरि, वन और जलवायु पररवतधन मोंत्रालय ((MoEF&CC) ने एक नई थिायी केंद्रीय असर्कार प्राप्त ससमसत (CEC)
की स्थापिा की है , सजसने उसी नाम के एक तदिध सवशेषज्ञ पैनल की जगह ली है , सजसने वन और पयाधवरि के मुद्दोों के
मामलोों में सुप्रीम कोटध (SC) की सहायता की िी।
i.अब, CEC को पयाधवरि सोंबोंर्ी मुद्दोों पर एक थिायी वैर्ासनक सनकाय के रूप में थिासपत सकया गया है , और भारत
सरकार (GoI) का CEC के सोंसवर्ान पर पूिध सनयोंत्रि है ।
• यह सवकास वन सोंरक्षि (सोंशोर्न) सवर्ेयक, 2023 के बाद हुआ।

Report Errors in the PDF - ebooks@affairscloud.com Copyright 2014-2023 @ AffairsCloud.com 119


• इस सोंबोंर् में, केंद्र सरकार नए CEC में सदस्योों को नामाोंसकत और सनयुक्त करे गी।
• इसमें एक अध्यक्ष, एक सदस्य ससचव और तीन सवशेषज्ञ सदस्य शासमल होोंगे।
• पयाधवरि, वासनकी, या वन्यजीव क्षेत्रोों में न्यूनतम 25 वषों का अनुभव या सरकार में पयाध प्त प्रशाससनक
सवशेषज्ञता वाला अध्यक्ष, असर्कतम तीन वषों का कायधकाल पूरा करे गा।

11. हाि ही में (लसतंबर’23 में) एसोलसएशन ऑफ इन्वेस्ट्मेंट बैंकसा ऑफ इं लिया (AIBI) के अध्यक्ष के रूप में
लकसे लनयुक्त लकया गया है?
1) श्चमश्चलंद V. दलवी
2) अनय P खरे
3)महावीर लुिावत
4)अिुगन मेहरा
5) प्रेम D'कुन्हा
उत्तर-3)महावीर िुणावत
स्पष्टीकरण:
एसोश्चसएशन ऑफ इन्वेस्ट्मेंट बैंकसग ऑफ इं श्चिया (AIBI) ने पैंटोमैि कैश्चपटल एिवाइिसग प्राइवेट श्चलश्चमटे ि के समूह
संथिापक और प्रबंर् श्चनदे शक (MD) महावीर िुणावत को श्चसतंबर 2023 से शुरू होने वाले दो साि के श्चलए अपना
अध्यक्ष श्चनयुक्त श्चकया है।
• वह एस्क्सस कैश्चपटल श्चलश्चमटे ि के MD (कॉपोरे ट फाइनेंस श्चिवीिन (CFD) अनय P िरे की िगह लेंगे।
• AIBI ने J.M. फाइनेंश्चशयल के MD अिुान मेहरा और ICICI श्चसक्योररटीि में SVP (वररष्ठ उपाध्यक्ष) & प्रमुख -
ECM श्चनष्पादन, प्रेम D'कुन्ा को अपना उपाध्यक्ष श्चनयुक्त श्चकया।
• AIBI भारतीय प्रश्चतभूश्चत और श्चवश्चनमय बोिग (SEBI) और श्चवश्चभन्न वैर्ाश्चनक प्राश्चर्करिों के श्चलए श्चनवेश बैंकरों का
एकमात्र प्रश्चतश्चनश्चर् श्चनकाय है ।

12. नसतंबर 2023 में, भारतीय ररििा बैंि (RBI) िे तीि साि िे निए ____________ (बैंि) िे प्रबंध निदे शि
(MD) और मुख्य िायािारी अनधिारी (CEO) िे रूप में संदीप बख्शी िी पुि: नियुखक्त िो मंिूरी दे दी।
1)HDFC बैंक
2)YES बैंक
3)ICICI बैंक
4)एक्कक्सस बैंक
5)इों डसइों ड बैंक
उत्तर-3)ICICI बैंि
स्पष्टीिरण:
भारतीय ररजवध बैंक (RBI) ने 4 अरू बर, 2023 से 3 अरू बर 2026 तक तीि साि के सलए ICICI बैंक सलसमटे ड के प्रबोंर्
सनदे शक (MD) और मुख्य कायधकारी असर्कारी (CEO) के रूप में संदीप बख्शी की पुनसनधयुक्कक्त को मोंजूरी दे दी है।
i.यह उनके कायधकाल का दू सरा सवस्तार है ; पहला सवस्तार 2021 में दो साल के सलए हुआ जो 3 अरू बर 2023 को
समाप्त होने वाला है।
ii.वह 2018 से चोंदा कोचर के बाद ICICI बैंक के MD और CEO के रूप में कायधरत हैं।

Report Errors in the PDF - ebooks@affairscloud.com Copyright 2014-2023 @ AffairsCloud.com 120


13. हाि ही में (नसतंबर'23 में) मास्ट्रिािा इं निया िे गैर-िायािारी अध्यक्ष िे रूप में निसे नियुक्त निया गया
है?
1)कासतधक रामनािन
2)गौतम अग्रवाल
3)सनक्कखल साहनी
4)रजनीश कुमार
5)राम श्रीर्र
उत्तर-4)रििीश िुमार
स्पष्टीिरण:
भारतीय स्ट्े ट बैंक (SBI) के पूिा अध्यक्ष रििीश िुमार को मास्ट्रिािा इं निया के गैर-कायधकारी अध्यक्ष के रूप में
सनयुक्त सकया गया है ।
i.रजनीश कुमार मास्ट्रकाडध की दसक्षि एसशया कायधकारी नेतृत्व टीम का मागधदशधन करें गे , सजसका नेतृत्व गौतम अग्रवाल,
दसक्षि एसशया के सडवीजन अध्यक्ष और दे श कॉपोरे ट असर्कारी, भारत करें गे।
ii.वह वतधमान में भुगतान स्ट्ाटध अप भारतपे के अध्यक्ष के रूप में कायधरत हैं ।
iii.पुस्तक - रजनीश कुमार ने 2021 में पेंगुइन वाइसकोंग िारा प्रकासशत 'द कस्ट्ोसडयन ऑफ टर स्ट्: ए बैंकसध मेमॉयर'
पुस्तक सलखी।

14. नसतंबर 2023 में, भारतीय ररििा बैंि (RBI) िे 3 साि िी अिनध िे निए __________ (बैंि) िे प्रबंध निदे शि
(MD) और मुख्य िायािारी अनधिारी (CEO) िे रूप में शनशधर िगदीशि िी पुिनिायुखक्त िो मंिूरी दे दी।
1)भारतीय स्ट्े ट बैंक
2)YES बैंक
3)एक्कक्सस बैंक
4)ICICI बैंक
5)HDFC बैंक
उत्तर-5)HDFC बैंि
स्पष्टीिरण:
भारतीय ररजवध बैंक (RBI) ने 27 अरू बर 2023 से 26 अरू बर 2026 तक 3 साल की अवसर् के सलए HDFC बैंि
सलसमटे ड के प्रबोंर् सनदे शक (MD) और मुख्य कायधकारी असर्कारी (CEO) के रूप में शनशधर िगदीशि की पुनसनधयुक्कक्त
को मोंजूरी दे दी है।
i.अरू बर 2020 से वह हाउससोंग डे वलपमेंट फाइनेंस कॉरपोरे शन (HDFC) बैंक के MD और CEO रहे हैं , जब उन्ोोंने
आसदत्य पुरी (बैंक के सबसे लोंबे समय तक सेवा दे ने वाले CEO (ससतोंबर 1994 से 26 वषध) से पदभार सोंभाला िा)।
ii.शसशर्र जगदीशन 1996 में सवत्त समारोह में प्रबोंर्क के रूप में HDFC बैंक में शासमल हुए और 1999 में सवत्त के
व्यवसाय प्रमुख बने। उन्ें 2008 में मुख्य सवत्तीय असर्कारी के रूप में सनयुक्त सकया गया िा।

15. हाि ही में (नसतंबर'23 में) 2023-2024 अिनध िे निए ऑनिट ब्यूरो ऑफ सिाु िेशि (ABC) िे अध्यक्ष िे
रूप में निसे चुिा गया है ?
1)सगरीश अग्रवाल
2)प्रशाोंत कुमार
3)श्रीसनवासन K. स्वामी

Report Errors in the PDF - ebooks@affairscloud.com Copyright 2014-2023 @ AffairsCloud.com 121


4)शैलेश गुप्ता
5)मोसहत जैन
उत्तर-3)श्रीनििासि K. स्वामी
स्पष्टीिरण:
श्रीसनवासन K. स्वामी होंसा समूह के कायधकारी अध्यक्ष श्रीसनवासन K. स्वामी को 2023-2024 िी अिनध के सलए ऑसडट
ब्यूरो ऑफ सकुधलेशन (ABC) के अध्यक्ष के रूप में चुना गया है। उन्ोोंने प्रताप पवार की जगह ली, सजन्ोोंने 2022-23
के कायधकाल के सलए अध्यक्ष के रूप में कायध सकया।
• श्रीसनवासन K. स्वामी वषध 2022-2023 के सलए ब्यूरो के उपाध्यक्ष िे।
• उन्ें 2016 में एडवरटाइसजोंग एजेंसीज एसोससएशन ऑफ इों सडया (AAAI) िारा लाइफटाइम अचीवमेंट अवाडध
से सम्मासनत सकया गया िा।
• वह वतधमान में एसशयन फेडरे शन ऑफ एडवरटाइसजोंग एसोससएशन के अध्यक्ष के रूप में कायधरत हैं।
i.पररषद में प्रकाशक सदस्योों का प्रसतसनसर्त्व करने वाले मलयाला मनोरमा कोंपनी सलसमटे ड के मुख्य एसोससएट सोंपादक
और सनदे शक ररयाद मैथ्यू को 2023-2024 के सलए ABC के उपाध्यक्ष के रूप में चुना गया है।

16. नसतंबर 2023 में राज्यसभा द्वारा उठाए गए िदम िे संबंध में निम्ननिखित में से िौि सा/से नबंदु "सही"
है/हैं ?
A) भारत िे उपराष्टरपनत, राज्यसभा िे पदे ि सभापनत िगदीप धििड िे आठ सदस्यीय पैिि िे साथ 50
प्रनतशत (4 सदस्य) मनहिा सांसदों िे साथ उपाध्यक्षों िे पैिि िा पुिगाठि निया है।
B) चार िई मनहिा: िांता िदा म (उत्तर प्रदे श), सुनमत्रा बाखिि (मध्य प्रदे श), ममता मोहंता (ओनिशा) और
गीता उफा चंद्रप्रभा (उत्तर प्रदे श) उपाध्यक्ष हैं।
C) भारत िे उपराष्टरपनत, राज्यसभा िे पदे ि सभापनत िगदीप धििड िे उपाध्यक्षों िे पैिि में 13 मनहिा
सांसदों िा गठि निया है।
1)सभी A, B & C
2)केवल A & C
3)केवल B & C
4)केवल A & C
5)केवल A
उत्तर- 1)सभी A, B & C
स्पष्टीिरण:
भारत िे उपराष्टरपनत, राज्यसभा के पदे न सभापसत जगदीप र्नखड ने 13 ससतोंबर 2023 से 50 प्रनतशत (4 सदस्य)
मसहला साोंसदोों के साि आठ सदस्यीय पैिि के साि उपाध्यक्षोों के पैनल का पुनगधठन सकया है।
• राज्यसभा (भारतीय सोंसद का ऊपरी सदन) के इसतहास में पहली बार पैनल में मसहला सदस्योों को समान
प्रसतसनसर्त्व सदया गया है ।
• 18-22 ससतोंबर, 2023 तक आयोसजत सोंसद के सवशेष सत्र के पहले सदन, उच्च सदन में जगदीप र्नखड िारा
चार नई मसहला उपाध्यक्षोों की घोषिा की गई।
i.चार िई मनहिा: िांता िदा म (उत्तर प्रदे श), सुनमत्रा बाखिि (मध्य प्रदे श), ममता मोहंता (ओसडशा) और गीता
उफा चंद्रप्रभा (उत्तर प्रदे श) उपाध्यक्ष हैं ।
ii.पैनल के पुरुष सदस्य -अक्कखलेश प्रसाद ससोंह (सबहार), नारायि दास गुप्ता (सदल्ली), V. सवजयसाई रे ड्डी (आों ध्र प्रदे श)
और शाोंतनु सेन (पसिम बोंगाल) हैं।

Report Errors in the PDF - ebooks@affairscloud.com Copyright 2014-2023 @ AffairsCloud.com 122


iii.भारत िे उपराष्टरपनत, राज्यसभा िे पदे ि सभापसत जगदीप र्नखड ने उपाध्यक्षोों के पैनल में 13 मसहला साोंसदोों
का गठन सकया है। मसहला आरक्षि सवर्ेयक या नारी शक्कक्त वोंदन असर्सनयम 2023 पर चचाध के सलए एक सदन के सलए
पैनल का पुनगधठन सकया गया िा।
• मसहला आरक्षि सवर्ेयक 2023, सजसे आसर्काररक तौर पर सोंसवर्ान (128वाों सों शोर्न) सवर्ेयक, 2023 कहा
जाता है और आमतौर पर नारी शक्कक्त वोंदन असर्सनयम के रूप में जाना जाता है , भारतीय सोंसद में पेश सकया
गया है।
• यह कानून मसहलाओों को सीर्े सनवाधसचत लोकसभा और राज्य सवर्ानसभाओों में 33 प्रसतशत सीटें आवोंसटत करने
के सलए है।

17. उस व्यखक्तत्व िा िाम बताइए निसे हाि ही में (नसतंबर'23 में) पेंगुइि रैं िम हाउस िे स्थायी मुख्य िायािारी
अनधिारी (CEO) िे रूप में िानमत निया गया है।
1)प्रशाोंत समश्रा
2)शेष शेषासद्र
3)सनहार मालवीय
4)लसलत ससोंह
5)सनताशा दे वसर
उत्तर-3)निहार माििीय
स्पष्टीिरण:
भारतीय मूल के अमेररकी प्रकाशन कायधकारी निहार माििीय को न्यूयॉकध (सोंयुक्त राज्य अमेररका, USA) क्कथित
अोंतराधष्ट्रीय प्रकाशन समूह, पेंगुइि रैं िम हाउस का थिायी मुख्य कायधकारी असर्कारी (CEO) नासमत सकया गया है।
i.सनहार मालवीय 1 जनवरी 2023 से 9 महीने के सलए अोंतररम CEO के रूप में कायधरत हैं।
ii.अोंतररम CEO के रूप में सनयुक्कक्त के बाद से, वह बटे ल्समैन (पेंगुइन रैं डम हाउस की मूल कोंपनी) समूह प्रबोंर्न ससमसत
(GMC) के सदस्य रहे हैं।
iii.वह माकधस डोहले का थिान लेंगे, सजन्ोोंने 2022 में साइमन एों ड शूस्ट्र को हाससल करने के पेंगुइन रैं डम हाउस के
असफल प्रयास के बाद पद छोड सदया िा।

18. हाि ही में (नसतंबर'23 में) भाभा परमाणु अिुसंधाि िेंद्र (BARC) िे 14िें निदे शि िे रूप में निसे नियुक्त
निया गया है ?
1)अचधना शमाध
2)सववेक भसीन
3)उमेश दानी
4)तपन कुमार घाोंटी
5)जयकुमार
उत्तर- 2)नििेि भसीि
स्पष्टीिरण:
15 ससतोंबर 2023 को, नििेि भसीि ने भारत की प्रमुख परमािु अनुसोंर्ान सुसवर्ा, भाभा परमािु अनुसोंर्ान केंद्र
(BARC) के 14िें निदे शि के रूप में कायधभार सोंभाला, सजसका मुख्यालय मुोंबई, महाराष्ट्र में है ।
• इस सनयुक्कक्त से पहले, सववेक BARC के परमािु ईोंर्न समूह (NFG) के सनदे शक के रूप में कायधरत िे।

Report Errors in the PDF - ebooks@affairscloud.com Copyright 2014-2023 @ AffairsCloud.com 123


i.वह अजीत कुमार मोहोंती का थिान लेंगे, सजन्ें मई 2023 में प्रर्ान मोंत्री कायाधलय (PMO) के तहत परमािु ऊजाध सवभाग
(DAE) के ससचव और परमािु ऊजाध आयोग के अध्यक्ष के रूप में सनयुक्त सकया गया िा।
ii.BARC की थिापना होमी जहाोंगीर भाभा ने 1954 में परमािु ऊजाध प्रसतष्ठान, टर ॉम्बे (AEET) के नाम से की िी। 1966 में
AEET का नाम बदलकर BARC कर सदया गया।

19. हाि ही में (नसतंबर'23 में) 3 साि िी अिनध िे निए एनशयाई नििास बैंि (ADB) िे उपाध्यक्ष (मािेट
सॉल्यूशंस) िे रूप में निसे नियुक्त निया गया है ?
1)जयोंत कुमार
2)आसतश मािुर
3)भागधव दासगुप्ता
4)अजय कुमार
5)प्रवीि शमाध
उत्तर-3)भागाि दासगुप्ता
स्पष्टीिरण:
22 ससतोंबर, 2023 को एसशयाई सवकास बैंक (ADB) ने भागाि दासगुप्ता को 3 साि की अवसर् के सलए उपाध्यक्ष
(मािेट सॉल्यूशंस) सनयुक्त सकया।
i.इस सनयुक्कक्त से पहले, वह 2009 से ICICI लोम्बाडध जनरल इों श्योरें स कोंपनी सलसमटे ड के प्रबोंर् सनदे शक (MD) और मुख्य
कायधकारी असर्कारी (CEO) के रूप में कायधरत िे।
ii.उन्ोोंने ICICI लोम्बाडध के MD और CEO पद से इस्तीफा दे सदया है। MD और CEO के रूप में उनका कायधकाल
अप्रैल 2024 में समाप्त होने वाला िा।

20. हाि ही में (नसतंबर'23 में) निस िंपिी िे बिमािी अग्रिाि िो िया अध्यक्ष नियुक्त निया है ?
1)मसहोंद्रा एयरोस्पेस सलसमटे ड
2)टाटा एडवाोंथड ससस्ट्म्स सलसमटे ड
3)सहन्क्दुस्तान एयरोनॉसटक्स सलसमटे ड
4)भारत इलेररॉसनक्स सलसमटे ड
5)भारत हेवी इलेक्कररकल्स सलसमटे ड
उत्तर-2)टाटा एििांस्ि नसस्ट्म्स निनमटे ि
स्पष्टीिरण:
बिमािी अग्रिाि को टाटा एडवाोंथड ससस्ट्म्स सलसमटे ड (TASL), एयरोस्पेस और रक्षा समार्ान प्रभाग और TATA ग्रुप
की होक्कडोंग कोंपनी टाटा सोंस की पूिध स्वासमत्व वाली सहायक कोंपनी के िए अध्यक्ष के रूप में सनयुक्त सकया गया है।
i.सनयुक्कक्त से पहले, वह टाटा सोंस में वररष्ठ सलाहकार और टाटा इलेररॉसनक्स प्राइवेट सलसमटे ड (TEPL) के अध्यक्ष के
रूप में कायधरत िे।
ii.वह नििय नसंह का थिान लेंगे जो इस पद से सेवासनवृत्त हो गए हैं। वह 2018 से TASL के अध्यक्ष के रूप में कायधरत
िे और वह टाटा टर स्ट् के उपाध्यक्ष के रूप में भी कायधरत हैं।

Report Errors in the PDF - ebooks@affairscloud.com Copyright 2014-2023 @ AffairsCloud.com 124


21. हाि ही में (नसतंबर'23 में) ICICI िोम्बािा ििरि इं श्योरें स निनमटे ि िे प्रबंध निदे शि (MD) और मुख्य
िायािारी अनधिारी (CEO) िे रूप में निसे नियुक्त निया गया है ?
1)मुरली सशवरामन
2)सोंदीप जगदीश बत्रा
3)राकेश झा
4)सोंजीव मोंत्री
5)आलोक कुमार अग्रवाल
उत्तर-4)संिीि मंत्री
स्पष्टीिरण:
ICICI िोम्बािा ििरि इं श्योरें स निनमटे ि ने भागधव दासगुप्ता के पद से इस्तीफे के बाद संिीि मंत्री को कोंपनी का
प्रबोंर् सनदे शक (MD) और मुख्य कायधकारी असर्कारी (CEO) सनयुक्त सकया।
i.सोंजीव मोंत्री वतधमान में ICICI लोम्बाडध जनरल इों श्योरें स के कायधकारी सनदे शक (2015 से) के रूप में कायधरत हैं।
ii.सोंजीव मोंत्री 1 सदसोंबर 2023 या भारतीय बीमा सनयामक और सवकास प्रासर्करि (IRDAI) से अनुमोदन की तारीख, जो
भी बाद में हो, से लगातार पाों च वषों की अवसर् या अपनी सेवासनवृसत्त की तारीख के सलए MD और CEO का पद सोंभालेंगे।
जो भी पहले हो, IRDAI और कोंपनी के सदस्योों के अनुमोदन के अर्ीन है।
iii.हालााँसक, 30 नवोंबर 2023 तक भागधव दासगुप्ता MD & CEO बने रहेंगे।

22. नसतंबर 2023 में, िैनबिेट िी नियुखक्त सनमनत (ACC) िे 1 िषा िी अिनध िे निए __________ (नियामि
नििाय) िे उप-राज्यपाि M रािेश्वर राि िी पुि: नियुखक्त िो मंिूरी दी।
1)भारतीय बीमा सवसनयामक और सवकास प्रासर्करि
2)भारतीय प्रसतभूसत एवों सवसनमय बोडध
3)भारतीय ररजवध बैंक
4)पेंशन फोंड सनयामक और सवकास प्रासर्करि
5)राष्ट्रीय कृसष और ग्रामीि सवकास बैंक
उत्तर-3)भारतीय ररििा बैंि
स्पष्टीिरण:
कैसबनेट की सनयुक्कक्त ससमसत (ACC) ने 9 अरू बर 2023 से 1 िषा की अवसर् के सलए भारतीय ररजवध बैंक (RBI) के DG
M रािेश्वर राि की पुनसनधयुक्कक्त को मोंजूरी दे दी है।
• प्रारों भ में, उन्ें अरू बर 2020 में 3 साल की अवसर् के सलए RBI के DG के रूप में सनयुक्त सकया गया िा।
i.RBI असर्सनयम 1934 के अनुसार, गवनधर के अलावा, RBI को मौसद्रक नीसत सवभाग का नेतृत्व करने के सलए 4 DG, 2
रैं क के भीतर से, एक वासिक्कज्यक बैंकर और एक अन्य अिधशास्त्री की आवश्यकता होती है ।
ii.RBI के वतधमान 4 DG स्वामीनािन J, T. रबी शोंकर, M. राजेश्वर राव और डॉ. M.D. पात्रा हैं।

23. हाि ही में (नसतंबर'23 में) यूिीिो िे पहिे भारतीय िांि एं बेसिर िे रूप में निसे िानमत निया गया है ?
1)कैटरीना कैफ
2)आसलया भट्ट
3)कोंगना रनौत
4)दीसपका पादु कोन
5)सकयारा आडवािी

Report Errors in the PDF - ebooks@affairscloud.com Copyright 2014-2023 @ AffairsCloud.com 125


उत्तर- 1)िैटरीिा िैफ
स्पष्टीिरण:
बॉलीवुड असभनेत्री िैटरीिा िैफ को जापानी बहुराष्ट्रीय फैशन ररटे लर ब्राों ड यूिीिो का पहिा भारतीय ब्राोंड एों बेसडर
नासमत सकया गया है।
• एक साल के समझौते के तहत असभनेत्री को कोंपनी की रोजमराध के पररर्ानोों की 'लाइफसवयर' रें ज की सवज्ञापन
सफल्ोों में सदखाया जाएगा।
• यूनीक्लो जापान के सबसे बडे फैशन समूह फास्ट् ररटे सलोंग का सहस्सा है सजसके आठ ब्राोंड हैं सजनमें यूनीक्लो,
GU, थ्योरी, PLST, कॉम्पटाइर डे स कॉटसनयसध, सप्रोंसेस टै म-टै म, J ब्राोंड और हेल्ुट लैंग शासमल हैं। यूनीक्लो ने
2019 में भारत में अपना पहला स्ट्ोर खोला।
• क्कस्वट् जरलैंड के टे सनस क्कखलाडी रोजर फेडरर 2018 से यूनीक्लो ग्लोबल ब्राोंड एों बेसडर हैं।

24. हाि ही में (नसतंबर'23 में) इं टेि इं निया िे अध्यक्ष िे रूप में निसे नियुक्त निया गया है ?
1)गोकुल सुब्रमण्यम
2)सवक्रम चौहान
3)सनवृसत्त राय
4)ससचन कट्टी
5)दे बजानी घोष
उत्तर- 1)गोिुि सुिमण्यम
स्पष्टीिरण:
सोंयुक्त राज्य अमेररका (USA) क्कथित अग्रिी सडसजटल सचप सनमाधता इों टेल कॉपोरे शन ने गोिुि सुिमण्यम को इं टेि
इं निया का अध्यक्ष सनयुक्त सकया है।
i.इों टेल इों सडया के अध्यक्ष के रूप में , वह भारत में इों टेल के समग्र इों जीसनयररों ग और सडजाइन सोंचालन के सलए सजम्मेदार
होोंगे, सजसमें नवाचार, क्रॉस-ग्रुप दक्षता और साइट से इों टेल उत्पादोों के सनष्पादन को शासमल करना शासमल है।
ii.वह वतधमान में क्लाइों ट कोंप्यूसटों ग ग्रुप (CCG) के उपाध्यक्ष (VP) हैं और क्लाइों ट प्लेटफॉमध और ससस्ट्म के महाप्रबोंर्क
के रूप में कायध करते हैं। वह CCG में मौजूदा अग्रिी क्लाइों ट प्लेटफॉमध और ससस्ट्म के अलावा इों टेल इों सडया के अध्यक्ष
की भूसमका भी सोंभालेंगे।
iii.इों टेल इों सडया सोंयुक्त राज्य अमेररका (USA) के बाहर कोंपनी का सबसे बडा सडजाइन और इों जीसनयररों ग केंद्र है , सजसमें
बेंगलुरु (कनाधटक) और हैदराबाद (तेलोंगाना) में अत्यार्ुसनक सडजाइन सुसवर्ाएों हैं।

25. हाि ही में (नसतंबर'23 में) निस बैंि िे KN मधुसूदिि िो अंशिानिि अध्यक्ष नियुक्त निया है ?
1)साउि इों सडयन बैंक
2)RBL बैंक
3)HDFC बैंक
4)र्नलक्ष्मी बैंक
5)ICICI बैंक
उत्तर-4)धििक्ष्मी बैंि

Report Errors in the PDF - ebooks@affairscloud.com Copyright 2014-2023 @ AffairsCloud.com 126


स्पष्टीिरण:
भारतीय ररजवध बैंक (RBI) ने 26 ससतोंबर, 2023 से 3 साल की अवसर् के सलए सत्रशूर (केरल) क्कथित धििक्ष्मी बैंि के
अोंशकासलक अध्यक्ष के रूप में KN मधुसूदिि की सनयुक्कक्त को मोंजूरी दे दी है । वह 9 नवोंबर 2022 से बैंक के बोडध में
एक स्वतोंत्र सनदे शक रहे हैं।
i.KN मर्ुसूदनन ने बैंक के अोंशकासलक अध्यक्ष और स्वतोंत्र सनदे शक G सुब्रमोसनया अय्यर की जगह ली, सजन्ोोंने सदसोंबर
2021 में इस्तीफा दे सदया िा। उनके इस्तीफे के बाद, बैंक 21 महीने तक सबना अध्यक्ष के रहा िा।
ii.KN मर्ुसूदनन सूक्ष्म, लघु और मध्यम उद्यम (MSME) कोंपसनयोों ससहत सवसभन्न सनजी सलसमटे ड कोंपसनयोों के प्रबोंर्
सनदे शक हैं। उन्ोोंने 1983 में सससवल सनमाध ि में कदम रखा।

AWARDS AND RECOGNITIONS


1. अगस्त 2023 में घोनषत रे मि मैग्सेसे अिािा (2023) िे 65िें संस्करण िा नििेता निम्ननिखित में से िौि सा
नबंदु 'िही ं' है?
1)रसव कन्नन: भारत से "हीरो फॉर होसलक्कस्ट्क हैल्थकेयर"।
2)समररयम कोरोनेल-फेरर: सफलीपीोंस से "वीमेन इन पीस-सबक्कडोंग पायसनयर"
3)यूजेसनयो लेमोस: सतमोर-लेस्ते से "फ़ूड सोवेररगोंती सवशनरी"
4)कोरवी रक्षोंद: बाोंग्लादे श से "एजुकेशन-फॉर-ऑल चैंसपयन"।
5)सु नान-चेंग: िाईलैंड से "हैल्थ केयर अवेलेबल टू द पुअर"।
उत्तर- 5)सु िाि-चेंग: थाईिैंि से "हैल्थ िेयर अिेिेबि टू द पुअर"।
स्पष्टीिरण:
सफलीपीोंस क्कथित रे मन मैग्सेसे अवाडध फाउों डेशन (RMAF) ने रे मि मैग्सेसे अिािा (2023)के 65िें संस्करण की घोषिा
की। 2023 रे मन मैग्सेसे अवाडध 11 नवोंबर 2023 को सफलीपीोंस के मनीला में आयोसजत सकया जाएगा।
• रसव कन्नन: भारत से "हीरो फॉर होसलक्कस्ट्क हैल्थकेयर"।
• समररयम कोरोनेल-फेरर: सफलीपीोंस से "वीमेन इन पीस-सबक्कडोंग पायसनयर"
• यूजेसनयो लेमोस: सतमोर-लेस्ते से "फ़ूड सोवेररगोंती सवशनरी"
• कोरवी रक्षोंद: बाोंग्लादे श से "एजुकेशन-फॉर-ऑल चैंसपयन"।
िोट: रे मन मैग्सेसे अवाडध को 'एसशया का नोबेल पुरस्कार' माना जाता है। भारत को कुल 59 पुरस्कार समले हैं , जबसक
सफलीपीोंस को 65 पुरस्कार समले हैं , जो इसे सभी दे शोों में सवोच्च बनाता है।

2. अगस्त 2023 में आयोनित 26िें ई-गििेंस पर राष्टरीय सम्मेिि (NCeG) िे दौराि प्रस्तुत ई-गििेंस 2022-
23 (NAeG 2023)िे निए राष्टरीय पुरस्कार िे संबंध में निम्ननिखित में से िौि सा नबंदु "सही" है ?
A)राष्टरीय अपराध ररिॉिा ब्यूरो (NCRB) िी राष्टरीय स्वचानित नफंगरनप्रंट पहचाि प्रणािी (NAFIS) टीम िे
निनिटि पररिताि, श्रेणी -1 िे निए सरिारी प्रनक्रया पुिरा चिा में उत्कृष्टता िे तहत स्वणा पुरस्कार िीता।
B)ओनिशा सरिार िो श्रम िाहि सुनिधा प्रणािी िे निए निनिटि पररिताि श्रेणी 1 िे निए सरिारी प्रनक्रया
पुिरा चिा में स्वणा पुरस्कार नमिा।
C)पंचायती राि मंत्रािय िी SVAMITVA (ग्राम आबादी िा सिेक्षण और ग्रामीण क्षेत्रों में उन्नत प्रौद्योनगिी िे
साथ मािनचत्रण) योििा िो िागररि िेंनद्रत सेिाएं प्रदाि िरिे िे निए उभरती प्रौद्योनगनियों िे अिुप्रयोग,
श्रेणी 2 में स्वणा पुरस्कार प्राप्त हुआ।
1)केवल A

Report Errors in the PDF - ebooks@affairscloud.com Copyright 2014-2023 @ AffairsCloud.com 127


2)केवल A & B
3)केवल B & C
4)केवल A & C
5)सभी A, B & C
उत्तर-4)िेिि A & C
स्पष्टीिरण:
नागररकोों को बेहतर सुसवर्ाएों प्रदान करने पर सवचार-सवमशध करने के सलए 26िां ई-गवनेंस पर राष्ट्रीय सम्मे लन (NCeG)
25 और 26 अगस्त 2023 को सब्रसलयोंट कन्वेंशन सेंटर, इं दौर, मध्य प्रदे श (MP) में आयोसजत सकया गया िा।
• सम्मेलन के दौरान, ई-गवनेंस पहल के अनुकरिीय कायाधन्वयन के सलएई-गवनेंस के सलए राष्ट्रीय पुरस्कार 2022-
23 (NCeG 2023) प्रदान सकए गए।
• 26वीों NCeG का आयोजन 'सवकससत भारत, एम्पावररों ग ससटीजर्न्' सवषय पर सकया गया िा।
• राष्ट्रीय अपरार् ररकॉडध ब्यू रो (NCRB) की राष्ट्रीय स्वचासलत सफोंगरसप्रोंट पहचान प्रिाली (NAFIS) टीम ने
DARPG, कासमधक, लोक सशकायत और पेंशन मोंत्रालय से NAeG 2022-23 के सडसजटल पररवतधन (श्रेिी -1)के
सलए सरकारी प्रसक्रया रीइों जीसनयररों ग में उत्कृष्ट्ता के तहत स्वणा पुरस्कार जीता।
• पनिम बंगाि सरिार को सुनिधा िाहि सुनिधा प्रणािी के सलए कासमधक, लोक सशकायत और पेंशन मोंत्रालय
से NAeG 2022-23 की सडसजटल पररवतधन श्रेणी 1 के सलए सरकारी प्रसक्रया पुनरध चना में स्वणा पुरस्कार भी
समला।
• पोंचायती राज मोंत्रालय की SVAMITVA (ग्राम आबादी का सवेक्षि और ग्रामीि क्षेत्रोों में उन्नत प्रौद्योसगकी के साि
मानसचत्रि) योजना को NAeG 2022-23 की श्रेिी 2, नागररक केंसद्रत सेवाएों प्रदान करने के सलए उभरती
प्रौद्योसगसकयोों के अनुप्रयोग में स्विध पुरस्कार समला।

3. हाि ही में (नसतंबर’23 में) मॉस्को, रूस में आयोनित पहिे निक्स BRICS फोरम में िडा इिोिेशि अिािा से
निसे सम्मानित निया गया है ?
1)ग़ज़ल अलघ
2)मनीष समश्रा
3)आलोक बरुआ
4)असवष्कार श्रीवास्तव
5)शाोंता िौटम
उत्तर-5)शांता थौटम
स्पष्टीिरण:
तेलोंगाना की पहिी मसहला चीफ इनोवेशन ऑसफसर (CIO) शांता थौटम को मॉस्को, रूस में आयोसजत पहिे
BRICS इिोिेशि फोरम में वडध इनोवेशन अवाडध से सम्मासनत सकया गया। क्लाउड ससटी अवर्ारिा फोरम का
मुख्य सवषय िा।
• यह अवाडध असभनव समार्ानोों को लागू करने के व्यवक्कथित प्रयास के माध्यम से सतत सवकास लक्ष् (SDG) -4
में उनके योगदान के सलए सदया गया है।
i.यह अवाडध सवश्व सवकास सोंगठन िारा थिासपत सकया गया िा, जो एक अों तरराष्ट्रीय गैर-सरकारी सोंगठन (NGO) है,
सजसे सोंयुक्त राष्ट्र आसिधक और सामासजक पररषद (ECOSOC) के साि सवशेष परामशधदात्री दजाध प्राप्त है ।
ii.यह अवाडध सवकासशील दे शोों के उन नेताओों को सम्मासनत करने के सलए सदया जाता है सजन्ोोंने सतत सवकास लक्ष्ोों
(SDG) की उपलक्कब्ध के सलए व्यक्कक्तगत योगदान सदया है।

Report Errors in the PDF - ebooks@affairscloud.com Copyright 2014-2023 @ AffairsCloud.com 128


4. निम्ननिखित में से निस व्यखक्तत्व िो हाि ही में (नसतंबर’23 में) ग्लोबि फाइिेंस पनत्रिा द्वारा ग्लोबि फाइिेंस
सेंटरि बैंिर ररपोटा िािा 2023 में A+ िा दिाा नदया गया है ?
1)सपयरे वुन्क्श (सचली)
2)माकध कानी (इों ग्लैंड)
3)मागधरीटा डे लगाडो (स्पेन)
4)सदसमतार राडे व (मेक्कक्सको)
5)शक्कक्तकाोंत दास (भारत)
उत्तर-5)शखक्तिांत दास (भारत)
स्पष्टीिरण:
सोंयुक्त राज्य अमेररका (USA) क्कथित ग्लोबल फाइनेंस पसत्रका ने ग्लोबल फाइनेंस सेंटरल बैंकर ररपोटध काडध 2023 में
भारतीय ररििा बैंि (RBI) िे गििार शक्कक्तकाोंत दास को A+ रे सटों ग दी है।
i.शक्कक्तकाोंत दास के साि, क्कस्वस नेशनल बैंक, क्कस्वट् जरलैंड के चेयरपसधन िॉमस J जॉडध न और स्ट्े ट बैंक ऑफ सवयतनाम,
सवयतनाम के गवनधर गुयेन िी होोंग को A+ रे सटों ग दी गई।
ii.1994 से ग्लोबल फाइनेंस िारा प्रसतवषध प्रकासशत सेंटरल बैंकर ररपोटध काडध , यूरोपीय सोंघ, पूवी कैरे सबयाई सेंटरल बैंक,
सेंटरल अफ्रीकी राज्योों के बैंक और सेंटरल बैंक ऑफ वेस्ट् अफ्रीकन स्ट्े ट्स ससहत 101 प्रमुख दे शोों, क्षेत्रोों और सजलोों के
सेंटरल बैंक गवनधरोों को ग्रेड दे ते हैं।

5. निस रे ििे स्ट्े शि िो हाि ही में (नसतंबर'23 में) IGBC (इं नियि ग्रीि नबखडं ग िाउं नसि)-भारतीय उद्योग
पररसंघ (CII) द्वारा प्लेनटिम िी उच्चतम रे नटं ग िे साथ "ग्रीि रे ििे स्ट्े शि" िे रूप में प्रमानणत निया गया है ?
1)मुोंबई सेंटरल रे लवे स्ट्े शन
2)सवजयवाडा रे लवे स्ट्े शन
3)गुवाहाटी रे लवे स्ट्े शन
4)चोंडीगढ रे लवे स्ट्े शन
5)वडोदरा रे लवे स्ट्े शन
उत्तर-2)निियिाडा रे ििे स्ट्े शि
स्पष्टीिरण:
आों ध्र प्रदे श (AP) के निियिाडा रे ििे स्ट्े शि को IGBC (इों सडयन ग्रीन सबक्कडोंग काउों ससल)-भारतीय उद्योग पररसोंघ
(CII) िारा प्लेसटनम की उच्चतम रे सटों ग के साि "ग्रीि रे ििे स्ट्े शि" के रूप में प्रमासित सकया गया है।
• रे सटों ग IGBC ग्रीन रे लवे स्ट्े शन रे सटों ग ससस्ट्म के तहत प्रस्तुत की गई िी।
• यह ससकोंदराबाद रे लवे सडवीजन (तेलोंगाना) के बाद प्लैसटनम रे सटों ग हाससल करने वाला दसक्षि मध्य रे लवे (SCR)
जोन के तहत दू सरा रे लवे स्ट्े शन बन गया है , सजसने 2018 में रे सटों ग हाससल की िी।
• इससे पहले 2019 में, सवजयवाडा रे लवे स्ट्े शन को IGBC - CII से गोड स्ट्ैं डडध रे सटों ग प्राप्त िी, जो 3 साल के
सलए वैर् है।

6. अगस्त 2023 में िेंद्रीय मंत्री भूपेन्द्र यादि, पयाािरण, िि और िििायु पररिताि मंत्रािय (MoEF&CC) द्वारा
प्रस्तुत 'स्वच्छ िायु सिेक्षण-2023' पुरस्कार िे संबंध में निम्ननिखित में से िौि सा/से नबंदु "सही" है /हैं ?
A) श्रेणी 1 में इं दौर (मध्य प्रदे श) िे पहिा स्थाि हानसि निया, उसिे बाद आगरा (उत्तर प्रदे श) और ठाणे
(महाराष्टर) क्रमशः दू सरे और तीसरे स्थाि पर रहे ।

Report Errors in the PDF - ebooks@affairscloud.com Copyright 2014-2023 @ AffairsCloud.com 129


B) महाराष्टर िे अमरािती िे शीषा स्थाि हानसि निया, उसिे बाद क्रमशः दू सरे और तीसरे स्थाि पर मुरादाबाद
(उत्तर प्रदे श) और गुंटूर (आं ध्र प्रदे श) रहे।
C) नहमाचि प्रदे श िे परिाणू िे शीषा स्थाि हानसि निया, उसिे बाद िािा अंब (नहमाचि प्रदे श) और अंगुि
(ओनिशा) क्रमशः दू सरे और तीसरे स्थाि पर रहे।
1)केवल A
2)केवल A & B
3)केवल B & C
4)केवल A & C
5)सभी A, B & C
उत्तर -5)सभी A, B & C
स्पष्टीिरण:
केंद्रीय मोंत्री भूपेन्द्र यादव, पयाधवरि, वन और जलवायु पररवतधन मोंत्रालय (MoEF&CC) ने 7 ससतोंबर 2023 को भोपाल,
मध्य प्रदे श में "नीले आकाश के सलए साफ हवा के अोंतराधष्ट्रीय सदवस" के अवसर पर 'स्वच्छ िायु सिेक्षण -2023'
सवजेताओों को पुरस्कार सदए हैं।
• MoEFCC की पहल के तहत केंद्रीय प्रदू षि सनयोंत्रि बोडध (CPCB) िारा 'स्वच्छ वायु सवेक्षि- 2023' आयोसजत
सकया गया िा।
• स्वच्छ वायु सवेक्षि का दू सरा सोंस्करि 2023 में हुआ।
i.'राष्टरीय स्वच्छ िायु शहर' पुरस्कार:
• श्रेणी 1: इं दौर (मध्य प्रदे श) ने पहला थिान हाससल सकया, उसके बाद आगरा (उत्तर प्रदे श) और ठािे (महाराष्ट्र)
क्रमशः दू सरे और तीसरे थिान पर रहे।
• श्रेणी 2: महाराष्ट्र के अमरावती ने शीषध थिान हाससल सकया, उसके बाद क्रमशः दू सरे और तीसरे थिान पर
मुरादाबाद (उत्तर प्रदे श) और गुोंटूर (आों ध्र प्रदे श) रहे ।
• श्रेणी 3: सहमाचल प्रदे श के परवानू ने शीषध थिान हाससल सकया, उसके बाद काला अोंब (सहमाचल प्रदे श) और
अोंगुल (ओसडशा) क्रमशः दू सरे और तीसरे थिान पर रहे । पुरस्कार के साि नकद मूल्, टर ॉफी और "राष्ट्रीय स्वच्छ
वायु शहर" शीषधक वाला प्रमाि पत्र सदया जाता है ।

7. नसतंबर 2023 में घोनषत निज्ञाि और प्रौद्योनगिी 2022 िे निए शांनत स्वरूप भटिागर (SSB) पुरस्कारों िे
संबंध में निम्ननिखित में से िौि सा/से नबंदु "सही" है /हैं ?
A) िैज्ञानिि और औद्योनगि अिुसंधाि पररषद (CSIR) िे महानिदे शि N ििैसेल्वी िे पूरे भारत में 12 युिा
िैज्ञानििों िो नितररत निए िािे िािे निज्ञाि और प्रौद्योनगिी 2022 िे निए शांनत स्वरूप भटिागर (SSB)
पुरस्कारों िी घोषणा िी।
B) नदप्यमि गांगुिी िे RCT (रैं िमाइज्ड िंटर ोड टर ायल्स) सनहत िैदानिि और अिुिादात्मि िोनिि -19
अिुसंधाि िे निए नचनित्सा निज्ञाि श्रेणी में पुरस्कार िीता।
C) अनश्विी िुमार िे TB(ट्यूबरिुिोनसस) पैथोनफनियोिॉिी में बायोनफि-प्रेररत संक्रमण िे निए बायोिॉिी
निज्ञाि श्रेणी िे तहत पुरस्कार िीता।
1)सभी A, B & C
2)केवल A & B
3)केवल B & C
4)केवल A & C

Report Errors in the PDF - ebooks@affairscloud.com Copyright 2014-2023 @ AffairsCloud.com 130


5)केवल A
उत्तर- 1)सभी A, B & C
स्पष्टीिरण:
11 ससतोंबर, 2023 को, वैज्ञासनक और औद्योसगक अनुसोंर्ान पररषद (CSIR) के महासनदे शक N कलैसेल्वी ने पूरे भारत
में 12 युिा िैज्ञानििों को सवतररत सकए जाने वाले सवज्ञान और प्रौद्योसगकी 2022 के सलए शाोंसत स्वरूप भटनागर (SSB)
पुरस्कारों की घोषिा की।
• यह घोषिा CSIR-नेशनल इों स्ट्ीट्यूट ऑफ साइों स कम्युसनकेशन एों ड पॉसलसी ररसचध (CSIR-NIScPR) के 'वन
वीक वन लैब' कायधक्रम के शुभारों भ कायधक्रम में राज्य मोंत्री (MoS) िॉ नितेंद्र नसंह (सवज्ञान और प्रौद्योसगकी
मोंत्रालय), उपाध्यक्ष, CSIR की उपक्कथिसत में की गई।
• दीप्यमि गांगुिी, CSIR-इों सडयन इों स्ट्ीट्यूट ऑफ केसमकल बायोलॉजी (IICB), कोलकाता, पसिम बोंगाल ने RCT
(रैं डमाइज्ड कोंटर ोड टर ायल्स) ससहत नैदासनक और टर ाोंसलेशनल कोसवड -19 अनुसोंर्ान के सलए नचनित्सा निज्ञाि
श्रेणी में पुरस्कार जीता।
• अनश्विी िुमार, CSIR-इों स्ट्ीट्यूट ऑफ माइक्रोसबयल टे क्नोलॉजी (IMTECH), चोंडीगढ ने TB(ट्यूबरकुलोससस)
पैिोसफसजयोलॉजी में बायोसफल्-प्रेररत सोंक्रमि के सलए बायोिॉिी निज्ञाि श्रेणी के तहत पुरस्कार जीता।

8. उस भारतीय व्यखक्तत्व िा िाम बताइए निन्ें हाि ही में (नसतंबर'23 में) फ्रांस सरिार द्वारा Chevalier dans
l’Ordre des Arts et des Lettres (िाइट ऑफ द ऑिा र ऑफ आट्ा स एं ि िेटसा ) से सम्मानित निया गया है।
1)सब्यसाची मुखजी
2)ऋतु कुमार
3)मनीष मल्होत्रा
4)तरुि तसहसलयानी
5)राहुल समश्रा
उत्तर-5)राहुि नमश्रा
स्पष्टीिरण:
फ्रांस सरकार ने भारतीय फैशन सडजाइनर राहुि नमश्रा को "Chevalier dans l’Ordre des Arts et des Lettres"
(नाइट ऑफ द ऑडध र ऑफ आट्ध स एों ड लेटसध) से सम्मासनत सकया है । यह पुरस्कार एक सडजाइनर के रूप में कला के
क्षेत्र में उनकी उपलक्कब्धयोों और भारतीय सशल्प और ज्ञान को अोंतरराष्ट्रीय स्तर पर बढावा दे ने की उनकी प्रसतबद्धता को
मान्यता दे ता है।
i.यह पुरस्कार फ्राोंस और भारत के बीच साोंस्कृसतक सोंबोंर्ोों और सहयोग में उनके योगदान का भी सम्मान करता है।
ii.12 ससतोंबर 2023 को, भारत में फ्राोंस के राजदू त इमैनुएल लेनैन ने फ्राोंस के सनवास, नई सदल्ली, सदल्ली में एक सवशेष
समारोह में राहुल समश्रा को पुरस्कार का प्रतीक सचन् प्रदान सकया।

9. नसतंबर 2023 में मॉनििंग िंसल्ट द्वारा िारी "ग्लोबि िीिर अप्रूिि रे नटं ग टर ै िर" सिेक्षण िे अिुसार, 76%
िी अप्रूिि रे नटं ग िे साथ निसे "मोस्ट् पॉपुिर ग्लोबि िडा िीिर" िानमत निया गया है ?
1)जो सबडे न (सोंयुक्त राज्य अमेररका के राष्ट्रपसत)
2)नरें द्र मोदी (भारत के प्रर्ान मोंत्री)
3)एन्डर े स मैनुअल लोपेज़ ओब्रेडोर (मेक्कक्सको के राष्ट्रपसत)
4)ऋसष सुनक (यूनाइटे ड सकोंगडम के प्रर्ान मोंत्री)
5)एलेन बसेट (क्कस्वट् ज़रलैंड के राष्ट्रपसत)

Report Errors in the PDF - ebooks@affairscloud.com Copyright 2014-2023 @ AffairsCloud.com 131


उत्तर -2)िरें द्र मोदी (भारत िे प्रधाि मंत्री)
स्पष्टीिरण:
सोंयुक्त राज्य अमेररका (USA) क्कथित कोंसक्कल्टोंग फमध मॉसनिंग कोंसल्ट िारा जारी "ग्लोबल लीडर अप्रूवल रे सटों ग टर ै कर"
सवेक्षि के अनुसार, भारत िे प्रधाि मंत्री (PM) िरें द्र मोदी को 76% (दु सनया में सबसे असर्क) की अनुमोदन रे सटों ग
के साि "मोस्ट् पॉपुिर ग्लोबि िडा िीिर" नासमत सकया गया है। मोदी को 18% अस्वीकृसत समली और 5% ने कोई
राय नहीों दी।
i.इससे पहले मोदी 2022 और 2021 में "सवाधसर्क लोकसप्रय वैसश्वक नेता" के रूप में सूची में शीषध पर िे।
ii.क्कस्वट् जरलैंड के राष्ट्रपसत एलेन बसेट दू सरे (64%) थिान पर हैं , उनके बाद मेक्कक्सको के राष्ट्रपसत एों डर े स मै नुअल लोपेज़
ओब्रेडोर तीसरे थिान (61%) हैं।

10. भारत िे उस व्यखक्तत्व िा िाम बताइए, निसिे अपिी टीम िे सदस्य िे साथ 'मैिेनििि इं िीनियररं ग'
श्रेणी िे तहत 2023 Ig िोबेि पुरस्कार िीता।
1)गोबीनाि राजेंद्रन
2)कृष्णास्वामी रामैया
3)मक्कल्लकाजुधन स्वामी
4)अनूप राजप्पन
5)श्यामरों जन दास महापात्र
उत्तर-4)अिूप रािप्पि
स्पष्टीिरण:
2023 Ig नोबेल पुरस्कार 14 ससतोंबर, 2023 को 33वें प्रिम वासषधक Ig नोबेल पुरस्कार समारोह में प्रदान सकए गए। Ig
नोबेल पुरस्कार हास्य वैज्ञासनक उपलक्कब्धयोों के सलए पुरस्कार है और इसका उद्दे श्य "उन उपलक्कब्धयोों का सम्मान करना
है जो पहले लोगोों को होंसाते हैं , और सफर उन्ें सोचने पर मजबूर करते हैं।”
• भारत, चीन, मलेसशया और सोंयुक्त राज्य अमेररका (USA) के वैज्ञासनकोों की एक टीम ने मृत मकसडयोों को सफर
से जीसवत करने और उन्ें याोंसत्रक पकडने वाले उपकरि के रूप में उपयोग करने के सलए 'मैकेसनकल
इों जीसनयररों ग' श्रेिी में 2023 Ig नोबेल जीता है।
• टीम िे सदस्य: फेय याप, जेन सलयू, अिूप रािप्पि (भारत), टर े वर सशमोकुसु और डै सनयल प्रेस्ट्न।
• मैसाचुसेट्स इों स्ट्ीट्यूट ऑफ टे क्नोलॉजी (MIT) सोंग्रहालय, कैक्कम्ब्रज (USA) में 11 नवोंबर, 2023 को 33वें प्रिम
वासषधक Ig नोबेल पुरस्कार सवजेताओों के सलए 'Ig नोबेल फेस-टू -फेस' नामक एक नया असतररक्त सािी कायधक्रम
आयोसजत सकया गया है।

11. नसतंबर 2023 में भारत िे उपराष्टरपनत िगदीप धििड से नितिे ििािारों िो संगीत िाटि अिादमी
अमृत पुरस्कार 2022 प्राप्त हुआ है ?
1)55
2)70
3)75
4)80
5)84
उत्तर- 5)84

Report Errors in the PDF - ebooks@affairscloud.com Copyright 2014-2023 @ AffairsCloud.com 132


स्पष्टीिरण:
16 ससतोंबर, 2023 को, भारत िे उपराष्टरपनत िगदीप धििड ने सवज्ञान भवन, नई सदल्ली में एक समारोह के दौरान
प्रदशधन कला के सवसभन्न क्षेत्रोों के 84 ििािारों को एकमुश्त सोंगीत नाटक अकादमी अमृत पुरस्कार 2022 प्रदान सकए।
• यह पुरस्कार 75 वषध से असर्क आयु के भारतीय कलाकारोों को सम्मासनत करने के सलए गसठत सकया गया िा,
सजन्ें अब तक अपने कररयर में कोई राष्ट्रीय सम्मान नहीों सदया गया है ।
• समारोह में कुल 84 कलाकारोों को इस प्रसतसष्ठत पुरस्कार से सम्मासनत सकया गया, सजनमें 70 पुरुष और 14
मसहलाएों शासमल िीों।
i.सूची में सवसभन्न भारतीय राज्योों के कलाकार जैसे आों ध्र प्रदे श से 3, अरुिाचल प्रदे श से 2, महाराष्ट्र से 6, उत्तर प्रदे श,
गुजरात से 3-3, पोंजाब और सदल्ली से 2-2 शासमल हैं ।
ii.4 पुरस्कार प्राप्तकताध सबहार, ओसडशा, पसिम बोंगाल, मसिपुर, मध्य प्रदे श, तसमलनाडु , केरल और कनाधटक से हैं , जबसक
असम और राजथिान से 5 प्रत्येक को उनके असार्ारि योगदान के सलए मान्यता दी गई।

12. लसतंबर 2023 में, भारत के नेशनि मेलिकि कमीशन (NMC) को तत्काि प्रभाव से __________ वषों की
अवलि के लिए वल्डा फेिरे शन फॉर मेलिकि एिुकेशन (WFME) द्वारा मान्यता का दिाा लदया गया था।
1)5
2)3
3)10
4)4
5)6
उत्तर-3)10
स्पष्टीकरण:
भारत के नेशनल मेश्चिकल कमीशन (NMC) को तत्काल प्रभाव से 10 वषों की अवश्चर् के श्चलए वर्ल्ग फेिरे शन फॉर
मेश्चिकल एिुकेशन (WFME) द्वारा मान्यता का दिाा श्चदया गया है।
i.इस मान्यता के श्चहस्से के रूप में, भारत के सभी 706 मौिूदा मेश्चिकल कॉलेि WFME मान्यता प्राप्त हो िाएं गे और
आने वाले 10 वषों में थिाश्चपत नए मेश्चिकल कॉलेि स्वचाश्चलत रूप से WFME मान्यता प्राप्त हो िाएं गे।
नोट: मान्यता प्राप्त एिेंश्चसयों की मान्यता का WFME कायगक्रम 2005 के बाद थिाश्चपत श्चकया गया िा।

13. नसतंबर 2023 में रािभाषा िीनता पुरस्कार 2022-23 िे संबंध में निम्ननिखित में से िौि सा/से नबंदु "सही"
है/हैं ?
A) पेंशि और पेंशिभोगी िल्याण निभाग (DoPPW) िो िगातार दू सरे िषा 300 से िम िमाचाररयों िािे
मंत्राियों/निभागों िी श्रेणी में सिाश्रेष्ठ प्रदशाि िरिे िािे (प्रथम) निभाग िे रूप में मान्यता दी गई है।
B) भारत िे नियंत्रि एिं महािेिा परीक्षि (CAG) िे िषा 2022-23 िे निए 300 से अनधि िनमायों िे साथ
"मंत्रािय/निभाग श्रेणी" में रािभाषा िीनत िे उत्कृष्ट िायाान्वयि िे निए सम्मानित निया।
C) नहंदुस्ताि पेटरोनियम िॉपोरे शि निनमटे ि िो िषा 2022-23 िे निए क्षेत्र 'A' में सािािनिि क्षेत्र िे उपक्रमों
िी श्रेणी में 'रािभाषा िीनता पुरस्कार' (तीसरा स्थाि) से सम्मानित निया गया है।
1)केवल A
2)केवल A & B
3)केवल B & C
4)केवल A & C

Report Errors in the PDF - ebooks@affairscloud.com Copyright 2014-2023 @ AffairsCloud.com 133


5)सभी A, B & C
उत्तर- 2)िेिि A & B
स्पष्टीिरण:
राजभाषा कीसतध पुरस्कार 2022-23, "आसर्काररक भाषा को लागू करने में उपलक्कब्धयोों" के सलए भारत सरकार िारा सदया
जाने वाला सबसे प्रसतसष्ठत पुरस्कार, सहोंदी सदवस समारोह 2023 में प्रस्तुत सकया गया।
• यह प्रसतसष्ठत पुरस्कार समारोह तीसरे अक्कखल भारतीय राजभाषा सम्मेलन (ऑल-इों सडया ऑसफससयल लैंग्वेज
काोंफ्रेंस) जो 14-15 ससतोंबर तक महाराष्ट्र के पु िे के बालेवाडी में श्री सशव छत्रपसत खेल पररसर में हुआ िा।
• पेंशि और पेंशिभोगी िल्याण निभाग (DoPPW) को प्रसतसष्ठत प्रिम राजभाषा कीसतध पुरस्कार, 2022-23
समला। यह लगातार दू सरा वषध है जब DoPPW को 300 से कम िमाचाररयों वाले मंत्राियों/निभागों की श्रेिी
में सवधश्रेष्ठ प्रदशधन करने वाले सवभाग के रूप में मान्यता दी गई है।
• पावर फाइनेंस कॉरपोरे शन (PFC) को वषध 2022-23 के सलए 'A' श्रेणी में प्रसतसष्ठत 'राजभाषा कीसतध पुरस्कार'
प्राप्त हुआ। PFC ने इस श्रेिी में तीसरा थिान हाससल सकया।
• भारत के सनयोंत्रक एवों महालेखा परीक्षक (CAG) ने वषध 2022-23 के सलए 300 से अनधि कसमधयोों के साि
"मोंत्रालय/सवभाग श्रेिी" में राजभाषा नीसत के उत्कृष्ट् कायाधन्वयन के सलए सम्मासनत सकया।

14. नसतंबर 2023 में, भारत िे िई नदल्ली, नदल्ली में ___________ (िषा) में सीमेंट िे रसायि निज्ञाि (ICCC) पर
17िी ं अंतरााष्टरीय िांग्रेस िी मेिबािी िे निए बोिी िीती।
1)2026
2)2027
3)2028
4)2025
5)2029
उत्तर- 2)2027
स्पष्टीिरण:
भारत ने 2027 में िई नदल्ली, नदल्ली में सीमेंट के रसायन सवज्ञान पर 17िी ं अोंतराधष्ट्रीय काों ग्रेस (ICCC) की मेजबानी की
बोली जीती। इस सनिधय की घोषिा 18 से 22 ससतोंबर 2023 तक बैंकॉक, िाईलैंड में चल रहे 16वें ICCC (2023)के
दौरान की गई िी।
i.भारत के अलावा, अन्य बोलीदाता क्कस्वट् जरलैंड और सोंयुक्त अरब अमीरात (UAE) से िे।
ii.भारत की बोली नेशनल काउों ससल फॉर सीमेंट एों ड सबक्कडोंग मैटेररयल्स (NCCBM/NCB) और भारतीय प्रौद्योसगकी
सोंथिान (IIT) सदल्ली ने लगाई िी। बोली NCCBM के महासनदे शक डॉ L P ससोंह िारा प्रस्तुत की गई िी; NCB के सोंयुक्त
सनदे शक डॉ. S K चतुवेदी और IIT सदल्ली के प्रोफेसर (सससवल इों जीसनयररों ग) डॉ. शशाोंक सबश्नोई भी इस मौके पर मौजूद
िे।
iii.17वीों ICCC भारत को सीमेंट क्षेत्र में वैसश्वक नेताओों, सवशेषज्ञोों और नवप्रवतधकोों को एक साि लाने का एक अनूठा
अवसर प्रदान करे गी। भारत ने इससे पहले 1992 में 9वीों काोंग्रेस की मेजबानी की िी।

15. नसतंबर 2023 में निम्ननिखित में से निस गांि िो पयाटि मंत्रािय (MoT) द्वारा भारत िे सिाश्रेष्ठ पयाटि गांि
2023 िे रूप में चुिा गया है ?
1)पसिम बोंगाल
2)असम

Report Errors in the PDF - ebooks@affairscloud.com Copyright 2014-2023 @ AffairsCloud.com 134


3)ससक्किम
4)मध्य प्रदे श
5)उत्तर प्रदे श
उत्तर- 2)असम
स्पष्टीिरण:
असम के मुख्यमोंत्री (CM) सहमोंत सबस्वा सरमा ने घोषिा की सक केंद्रीय पयधटन मोंत्रालय (MoT) ने असम के सोसनतपुर
सजले के नबश्विाथ घाट (सजसे गुप्त काशी भी कहा जाता है ) को भारत िे सिाश्रेष्ठ पयाटि गांि 2023 के रूप में चुना
है।
• MoT ने पसिम बोंगाल के मुसशधदाबाद सजले के निरीटे श्वरी गााँि को भारत के सवधश्रेष्ठ पयधटन गााँव के रूप में भी
चुना है।
• गाोंवोों का चयन 31 राज्योों और केंद्र शाससत प्रदे शोों (UT) के 791 आवेदनोों में से सकया गया िा।
• सबस्वनाि घाट साोंस्कृसतक समामेलन और गोंगमौ िान और नागसोंकर मोंसदर जैसे आध्याक्कत्मक और र्ासमधक थिलोों
के सलए जाना जाता है।
• सकरीटे श्वरी, भागीरिी नदी के पास क्कथित है , सजसमें सकरीटे श्वरी मोंसदर है , जो भारत के 51 शक्कक्तपीठोों में से एक
है।

16. उस व्यखक्तत्व िा िाम बताइए िो हाि ही में (नसतंबर'23 में) फीड ररसचा और एखप्लिेशि में िॉमाि E.
बोरिॉग पुरस्कार प्राप्त िरिे िािे तीसरे भारतीय िैज्ञानिि बिे हैं।
1)श्रीसनवास कुलकिी
2)रे नू खन्ना
3)स्वासत नायक
4)गगनदीप कोंग
5)असनल कुमार गुप्ता
उत्तर-3)स्वानत िायि
स्पष्टीिरण:
भारतीय वैज्ञासनक स्वानत िायि को 'फीड ररसचा एं ि एप्लीिेशि' के सलए नॉमधन E. बोरलॉग पुरस्कार 2023 का
प्राप्तकताध नासमत सकया गया है। वह फीड ररसचा और एप्लीिेशि में यह पुरस्कार पाने वाली तीसरी भारतीय और
पहिी ओनिया (ओनिशा) वैज्ञासनक बनीों। उन्ें ओसडशा में प्यार से सबहाना दीदी (बीज लेडी) कहा जाता है।
• पुरस्कार की घोषिा क्लाइमेट वीक न्यूयॉकध ससटी में की गई िी जो 17-24 ससतोंबर 2023 तक आयोसजत सकया
गया िा।
• फीड ररसचध और एप्लीकेशन के सलए नॉमधन E. बोरलॉग पुरस्कार, रॉकफेलर फाउों डेशन िारा प्रदान सकया गया
• यह पुरस्कार सवश्व खाद्य पुरस्कार फाउों डेशन िारा प्रसतवषध 40 वषध से कम आयु के सकसी व्यक्कक्त को प्रदान सकया
जाता है, सजसने वैसश्वक खाद्य सुरक्षा और पोषि में सुर्ार में महत्वपूिध योगदान सदया हो।
िोट: यह पुरस्कार पाने वाले अन्य दो भारतीय असदसत मुखजी (2012)और महासलोंगम गोसवोंदराज (2022)हैं।

17. नसतंबर 2023 ति, िहीदा रहमाि िो िषा 2021 िे निए ___________ दादा साहब फाल्के पुरस्कार से
सम्मानित निया िाएगा।
1)54वाों
2)50वाों

Report Errors in the PDF - ebooks@affairscloud.com Copyright 2014-2023 @ AffairsCloud.com 135


3)55वाों
4)53वाों
5)56वाों
उत्तर- 4)53िां
स्पष्टीिरण:
अनुभवी भारतीय असभनेता और पद्म पुरस्कार सवजेता वहीदा रहमान को भारतीय ससनेमा में उनके आजीवन योगदान के
सलए वषध 2021 के दादा साहेब फाल्के पुरस्कार से सम्मासनत सकया जाएगा। वह इस पुरस्कार की 53िी ं प्राप्तकताध
बनेंगी।
• वह दादा साहे ब फाल्के पुरस्कार पाने वाली 8वीों मसहला कलाकार भी होोंगी।
• यह पुरस्कार 69वें राष्ट्रीय सफल् पुरस्कार समारोह के दौरान प्रदान सकया जाएगा।
• भारत सरकार (GoI) ने उन्ें सफल् "रे शमा और शेरा" (1971)के सलए राष्ट्रीय सफल् पुरस्कार (सवधश्रेष्ठ असभनेत्री)
से सम्मासनत सकया।
• उन्ें 1972 में पद्म श्री (कला) और 2011 में पद्म भूषि (कला) से सम्मासनत सकया गया िा।
िोट- 1969 में थिासपत दादा साहेब फाल्के पुरस्कार, भारतीय ससनेमा में र्ुोंडीराज गोसवोंद फाल्के (दादा साहे ब फाल्के)
के योगदान को मनाने के सलए भारत सरकार िारा शुरू सकया गया िा।

18. 26 नसतंबर 2023 िो िैज्ञानिि और औद्योनगि अिुसंधाि पररषद (CSIR) िे 82 िें स्थापिा नदिस पर
िेंद्रीय निज्ञाि और प्रौद्योनगिी राज्य मंत्री नितेंद्र नसंह से नितिे प्रख्यात िैज्ञानििों िो निज्ञाि और प्रौद्योनगिी
(S&T) 2022 िे निए शांनत स्वरूप भटिागर (SSB) पुरस्कार प्राप्त हुए?
1)10
2)12
3)20
4)15
5)25
उत्तर- 2)12
स्पष्टीिरण:
वैज्ञासनक एवों औद्योसगक अनुसोंर्ान पररषद (CSIR) ने 26 नसतंबर 2023 को नई सदल्ली, सदल्ली में भारत मोंडपम में
आयोसजत एक कायधक्रम के साि अपना 82िां स्थापिा नदिस मनाया।
i.सवज्ञान और प्रौद्योसगकी मोंत्रालय (MoST) के केंद्रीय राज्य मोंत्री (स्वतोंत्र प्रभार) डॉ. सजतेंद्र ससोंह ने कायधक्रम को सोंबोसर्त
सकया और 45 िषा से कम आयु के 12 प्रख्यात िैज्ञानििों को सवज्ञान और प्रौद्योसगकी (S&T) 2022 के सलए शाोंसत
स्वरूप भटनागर (SSB) पुरस्कार प्रदान सकए।
िोट: CSIR, प्रर्ान मोंत्री नरें द्र मोदी के नेतृत्व वाला एक प्रमुख राष्ट्रीय अनुसोंर्ान और सवकास (R&D) सोंगठन, MoST की
प्रशाससनक दे खरे ख में काम करता है ।

19. नसतंबर 2023 में िायर एं ि प्लाखस्ट्ि प्रोिर् स (WPP) द्वारा आयोनित िंतार िांिZ टॉप 75 मोस्ट्
िैल्यूएबि इं नियि िांि्स ररपोटा िे 10िें संस्करण िे अिुसार, ____________ (िंपिी/बैंि) िे िगातार दू सरे िषा
भारत िे मोस्ट् िैल्यूएबि िांि िे रूप में अपिा शीषा स्थाि बरिरार रिा।
1)इों फोससस सलसमटे ड
2)HDFC बैंक

Report Errors in the PDF - ebooks@affairscloud.com Copyright 2014-2023 @ AffairsCloud.com 136


3)एयरटे ल सलसमटे ड
4)टाटा कोंसल्टें सी ससवधसेज
5)HCL टे क्नोलॉजीज सलसमटे ड
उत्तर-4)टाटा िंसल्टें सी सनिासेि
स्पष्टीिरण:
वायर एों ड प्लाक्कस्ट्क प्रोडर् स (WPP) और िंतार िारा आयोसजत कोंतार िांिZ टॉप 75 मोस्ट् िैल्यूएबि इं नियि
िांि्स ररपोटा में, टाटा िंसल्टें सी सनिासेि (TCS) ने 43 सबसलयन अमेररकी डॉलर के ब्राोंड मूल् के साि लगातार दू सरे
वषध भारत के मोस्ट् वैल्ूएबल के रूप में अपना शीषध थिान बरकरार रखा।
i.हाउससोंग डे वलपमेंट फाइनेंस कॉरपोरे शन (HDFC) बैंक और इों फोससस क्रमशः 33,612 अमेररकी डॉलर और 24,170
अमेररकी डॉलर के ब्राोंड मूल् के साि दू सरे और तीसरे थिान पर हैं ।
• भारत के शीषध 75 ब्राोंडोों का सोंयुक्त ब्राोंड मूल् 379 सबसलयन अमेररकी डॉलर है , जो 2022 से 4% की सगरावट
है।
• 2023 भारतीय ब्राोंड रैं सकोंग में चार ििागंतुि और दो पुि: प्रिेशिताा शासमल हैं
❖ फोनपे (21वीों रैं क) भारत का अग्रिी सडसजटल भुगतान एक्कप्लकेशन बन गया है।
❖ अन्य नवागोंतुकोों में सफनटे क ब्राोंड क्रेड, शेयरचैट (फोटो और वीसडयो शेयररों ग ऐप), और स्ट्ार (मनोरों जन
मोंच) शासमल हैं।
• ऑटोमोसटव श्रेिी ने दो सबसे तेजी से उभरने वाले उत्पाद तैयार सकए।
❖ कई सफल उत्पाद लॉन्च और BMW के साि 10 साल की साझेदारी से TVS के मूल् में 59% की वृक्कद्ध
हुई और 24 थिानोों की छलाोंग लगाई गई, सजससे इसे यूरोप, अमेररका और कनाडा जैसे बाजारोों में बढत
हाससल हुई।
❖ हैचबैक से SUV की ओर प्रािसमकता में बदलाव और इलेक्कररक वाहनोों की माों ग के कारि मसहों द्रा ने अपने
मूल् में 48% की वृक्कद्ध की।
❖ इसके असतररक्त, 40% साल-दर-साल बदलाव के साि सनफीस्ट् (54वाों सबसे मूल्वान ब्राोंड) तीसरा तेजी
से बढने वाला ब्राोंड है।

SUMMITS, CONFERENCES
1. नसतंबर 2023 में, िई नदल्ली में आयोनित नद्व-िानषाि िौसेिा िमांिर सम्मेिि िे दू सरे संस्करण िे समुद्री
अिसंरचिा पररप्रेक्ष्य योििा(MIPP) __________ (िषा) िारी निया गया था।
1)2023-29
2)2023-37
3)2023-30
4)2023-35
5)2023-27
उत्तर- 2)2023-37
स्पष्टीिरण:
4 ससतोंबर, 2023 को, सि-वासषधक नौसेना कमाोंडर सम्मेलन का दू सरा सोंस्करि नई सदल्ली, सदल्ली में आयोसजत सकया
गया िा जहाों रक्षा मोंत्रालय के राज्य मोंत्री (MoS) अजय भट्ट ने वररष्ठ नौसेना नेतृत्व के साि बातचीत की।
• पहला सोंस्करि माचध 2023 में INS (भारतीय नौसेना जहाज) सवक्राोंत पर आयोसजत सकया गया िा।

Report Errors in the PDF - ebooks@affairscloud.com Copyright 2014-2023 @ AffairsCloud.com 137


• समुद्री अवसोंरचना पररप्रेक्ष् योजना 2023-37 (MIPP): यह योजना सरकार के क्कथिरता लक्ष्ोों और व्यापक
नीसत सनदे शोों के अनुरूप, अगले 15 वषों के सलए नौसेना की अवसोंरचना जरूरतोों को रे खाोंसकत करती है ।
• IRS सनयम और सवसनयम हैं डबुक: यह नौसैसनक जहाज सनमाधि में तकनीकी प्रगसत और आत्मसनभधरता को
शासमल करता है , यह हैंडबुक नौसैसनक लडाकोों के सनमाधि और वगीकरि को कवर करता है।
• 6 ससतोंबर, 2023 को, श्रृोंखला का दू सरा बाजध LSAM 16 (याडध 126) 30 साल की सेवा जीवन के साि भारतीय
सशसपोंग रसजस्ट्र (IRS) वगीकरि सनयमोों के तहत भारतीय नौसेना (IN) को सौोंप सदया गया है।

2. नसतंबर 2023 में िई नदल्ली में भारत िी अध्यक्षता में आयोनित 18िें G20 (ग्रुप ऑफ़ 20) िेताओं िे
नशिर सम्मेिि िे संबंध में निम्ननिखित में से िौि सा नबंदु "गित" है ?
1)18वें G20 सशखर सम्मेलन के दौरान, भारतीय प्रर्ान मोंत्री (PM) नरें द्र मोदी ने घोषिा की सक अफ्रीकी सोंघ को G20
के सदस्य के रूप में शासमल सकया गया है , सजससे यह 21वाों सदस्य बन गया है।
2)PM नरें द्र मोदी ने 18वें G20 सशखर सम्मेलन के दौरान नई सदल्ली घोषिा को सवधसम्मसत से अपनाने के बारे में एक
महत्वपूिध घोषिा की।
3)PM नरें द्र मोदी ने वैसश्वक जैव ईोंर्न गठबोंर्न (GBA) लॉन्च सकया, जो पेटरोल और डीजल ईोंर्न पर दु सनया की सनभधरता
को कम करने के उद्दे श्य से एक सामूसहक प्रयास है सजसमें अब तक कुल 19 दे श और 12 अोंतराध ष्ट्रीय सोंगठन शासमल
हो चुके हैं।
4)PM नरें द्र मोदी और अमे ररका के राष्ट्रपसत महामसहम श्री जो सबडे न ने वैसश्वक अवसोंरचना और सनवेश (PGII) और
भारत-मध्य पूवध-यूरोप आसिधक गसलयारे (IMEC) के सलए साझेदारी पर एक सवशेष कायधक्रम की सह-अध्यक्षता की।
5)PM नरें द्र मोदी ने G20 की अध्यक्षता इटली के राष्ट्रपसत सजयोसजधया मेलोनी को सौोंपी, जो 1 सदसोंबर, 2023 से समूह
की अध्यक्षता सोंभालेंगे।
उत्तर- 5)PM िरें द्र मोदी िे G20 िी अध्यक्षता इटिी िे राष्टरपनत नियोनिाया मेिोिी िो सौप
ं ी, िो 1 नदसंबर,
2023 से समूह िी अध्यक्षता संभािेंगे।
स्पष्टीिरण:
भारत की अध्यक्षता में दो सदवसीय 18िां G20 (ग्रुप ऑफ़ 20) नेताओों का सशखर सम्मेलन 9 से 10 ससतोंबर, 2023 तक
भारत मोंडपम अोंतराधष्ट्रीय प्रदशधनी और कन्वेंशन सेंटर, प्रगसत मैदान, नई सदल्ली में हुआ। यह अब तक का पहला G20
सशखर सम्मेलन है भारत िारा होस्ट् सकया गया।
• बैठक "वसुर्ैव कुटुों बकम" या "वन अिध – वन फ़ॅसमली – वन फ्यूचर" सवषय के तहत आयोसजत की गई िी। यह
एक पुराने सोंस्कृत ग्रोंि महा उपसनषद से प्रेररत है।
• 18वें G20 सशखर सम्मेलन के दौरान, भारतीय प्रर्ान मोंत्री नरें द्र मोदी ने घोषिा की सक अफ्रीिी संघ को G20
के सदस्य के रूप में शासमल सकया गया है , सजससे यह 21वाों सदस्य बन गया है।
• प्रर्ान मोंत्री नरें द्र मोदी ने 18वें G20 सशखर सम्मेलन के दौरान नई सदल्ली घोषिा को सवधसम्मसत से अपनाने के
बारे में एक महत्वपूिध घोषिा की।
• 9 ससतोंबर, 2023 को नई सदल्ली में आयोसजत G20 सशखर सम्मेलन में, प्रर्ान मोंत्री श्री नरें द्र मोदी ने वैसश्वक जैव
ईोंर्न गठबोंर्न (GBA) लॉन्च सकया, जो एक सामूसहक प्रयास है सजसका उद्दे श्य पे टरोल और डीजल ईोंर्न पर
दु सनया की सनभधरता को कम करना है। अब तक कुल 19 दे श और 12 अोंतरराष्ट्रीय सोंगठन इसमें शासमल होने
के सलए सहमत हो चुके हैं।
• नई सदल्ली में G-20 सशखर सम्मेलन के अवसर पर प्रर्ानमोंत्री श्री नरें द्र मोदी और अमरीका के राष्ट्रपसत श्री जो
सबडे न ने वैसश्वक अवसोंरचना और सनवेश (PGII) और भारत-मध्य पूवध-यूरोप आसिधक गसलयारे (IMEC) के सलए
भागीदारी पर एक सवशेष कायधक्रम की सह-अध्यक्षता की।

Report Errors in the PDF - ebooks@affairscloud.com Copyright 2014-2023 @ AffairsCloud.com 138


• दो सदवसीय G20 नेताओों का सशखर सम्मेलन 2023 10 ससतोंबर, 2023 को सोंपन्न हुआ। प्रर्ान मोंत्री नरें द्र मोदी
ने G20 िी अध्यक्षता िािीि के राष्ट्रपसत लूला डी ससल्वा को सौोंपी, जो इस वषध 1 सदसोंबर 2023 से समूह की
अध्यक्षता सोंभालेंगे।
• G20 नेताओों के सशखर सम्मेलन से इतर, सब्रसटश प्रर्ान मोंत्री ऋसष सुनक ने 9 ससतोंबर,2023 को नई सदल्ली के
भारत मोंडपम में ससोंगापुर के प्रर्ान मोंत्री ली सीन लूोंग के साि नई रिनीसतक साझेदारी बनाई।

COMMITTEE AND MEETING


1. नसतंबर 2023 में िई नदल्ली में आयोनित मंनत्रस्तरीय भारत-यूिाइटे ि निंगिम (UK) आनथाि और नित्तीय
िाताा (EFD) िे 12िें दौर िे संबंध में निम्ननिखित में से िौि सा नबंदु "सही" है /हैं ?
A) भारत और UK िे भारत में महत्वपूणा बुनियादी ढांचा क्षेत्रों िे निए दीघािानिि नििेश िो सुरनक्षत िरिे िे
निए िई नदल्ली में आयोनित मं नत्रस्तरीय भारत-UK EFD िे 12िें दौर िे दौराि UK-इं निया इं फ्रास्ट्र क्चर
फाइिेंनसंग निि (UKIIFB) िे िॉन्च िी घोषणा िी।
B) UKIIFB पहि भारत िी राष्टरीय अिसंरचिा पाइपिाइि िा समथाि िरिे िे निए निशेषज्ञता और नििेश
भी प्रदाि िरती है।
C) UKIIFB पहि िा िेतृत्व िेशिि इं स्ट्ीट्यूट फॉर टर ांसफॉनमिंग इं निया (NITI आयोग) और िंदि शहर द्वारा
संयुक्त रूप से निया िाता है।
1)केवल A
2)केवल A & B
3)केवल B & C
4)केवल A & C
5)सभी A, B & C
उत्तर -5)सभी A, B & C
स्पष्टीिरण:
भारत और यूनाइटे ड सकोंगडम (UK) ने भारत में महत्वपूिध बुसनयादी ढाोंचा क्षेत्रोों के सलए दीघधकासलक सनवेश को सुरसक्षत
करने के सलए 11 ससतोंबर 2023 को नई सदल्ली, सदल्ली में आयोसजत मंनत्रस्तरीय इं निया-UK इकनोसमक एों ड
फाइनेंससयल डायलाग (EFD) के 12िें दौर के दौरान UK-इों सडया इों फ्रास्ट्र क्चर फाइनेंससोंग सब्रज (UKIIFB) के लॉन्च की
घोषिा की।
• UKIIFB एक सहयोगी पहल है जो भारत में पयाधप्त बुसनयादी ढाोंचे के सनवेश के अवसरोों को अनलॉक करने की
सदशा में सनकट सहयोग में काम करने के सलए दोनोों दे शोों की सनरों तर प्रसतबद्धता का प्रतीक है।
• UKIIFB पहल भारत िी राष्टरीय अिसंरचिा पाइपिाइि का समिधन करने के सलए सवशेषज्ञता और सनवेश भी
प्रदान करती है।
• UKIIFB पहल का नेतृत्व नेशनल इों स्ट्ीट्यूट फॉर टर ाोंसफॉसमिंग इों सडया (NITI आयोग) और लोंदन शहर िारा
सोंयुक्त रूप से सकया जाता है।
• 2023 में लोंदन में आयोसजत फाइनेंससयल मासकधट डायलाग (FMD) ने बैंसकोंग, बीमा और पेंशन ससहत कई
उपक्षेत्रोों में दोनोों दे शोों के बीच सहयोग को गहरा करने का अवसर प्रदान सकया।
िोट: अगला (13वाों) मोंसत्रस्तरीय भारत-UK EFD 2024 में लोंदन, UK में आयोसजत सकया जाएगा।

Report Errors in the PDF - ebooks@affairscloud.com Copyright 2014-2023 @ AffairsCloud.com 139


INDEX
1. नसतंबर 2023 में सुरफशािा निनमटे ि द्वारा िारी 2023 निनिटि िानिटी ऑफ िाइफ (DQL) इं िेक्स िे
अिुसार, भारत िे 0.5154 िे रैं निंग इं िेक्स िे साथ __________ रैं ि हानसि िी है।
1)60वाों
2)40वाों
3)55वाों
4)65वाों
5)52वाों
उत्तर- 5)52िां
स्पष्टीिरण:
सुरफशाकध सलसमटे ड के वासषधक DQL इों डेक्स के 5वें सोंस्करि, 2023 सडसजटल क्वासलटी ऑफ लाइफ (DQL) इं िेक्स
के अनुसार, भारत 2022 में 59वें रैं क से 0.5154 रैं सकोंग इों डेक्स के साि 52िें रैं क पर पहुोंच गया है ।
i.121 दे शोों के सूचकाों क में 0.7902 रैं सकोंग सूचकाोंक के साि फ्राों स शीषध पर है , इसके बाद सफनलैंड, डे नमाकध, जमधनी
और लक्ज़मबगध हैं।
ii.एसशया (क्षेत्रीय रैं सकोंग) में भारत 13वें थिान पर है , जबसक ससोंगापुर (वैसश्वक रैं क 10) शीषध पर है ।
iii.सडसजटल क्वासलटी ऑफ लाइफ (DQL) इों डेक्स उन कारकोों की अोंतदृधसष्ट् प्रकट करता है जो दे श की सडसजटल भलाई
को प्रभासवत करते हैं और उन क्षेत्रोों को भसवष्य में सुर्ार के सलए प्रािसमकता दी जानी चासहए।
iv.यह अध्ययन दे श की सडसजटल जीवन गुिवत्ता में एक असितीय पररप्रेक्ष् प्रदान करता है और अध्ययन में दु सनया की
90% आबादी को शासमल सकया गया है।

2. हाि ही में (नसतंबर'23 में) नसतंबर 2023 में चैिानिनसस द्वारा िारी 2023 ग्लोबि नक्रप्टो एिॉप्शि इं िेक्स में
िौि सा दे श शीषा पर रहा है ?
1)चीन
2)भारत
3)जापान
4)सवयतनाम
5)सोंयुक्त राज्य अमेररका
उत्तर- 2)भारत
स्पष्टीिरण:
सक्रप्टो-सवश्लेषि फमध चैिानिनसस िारा जारी 2023 ग्लोबि नक्रप्टो एिॉप्शि इं िेक्स के अनुसार, भारत 154 दे शोों के
बीच 1.0 के स्कोर के साि इों डेक्स (रैं क 1)में शीषध पर है। इससे पहले, 2022 इों डेक्स में भारत चौिे थिान पर िा।
• नाइजीररया और सवयतनाम क्रमशः 0.642 और 0.568 के इं िेक्स स्कोर के साि दू सरे और तीसरे थिान पर हैं ।
• यह चौिा वासषधक ग्लोबल सक्रप्टो एडॉप्शन इों डेक्स है। इों डेक्स का पहला सोंस्करि 2020 में प्रकासशत हुआ िा।
• ररपोटध यह मापने के सलए ऑन-चेन डे टा और वास्तसवक दु सनया डे टा को जोडती है सक कौन से दे श जमीनी स्तर
पर सक्रप्टो अपनाने में दु सनया में अग्रिी हैं।
• ररपोटध का उद्दे श्य उन दे शोों को उजागर करना है जहाों आम लोग सक्रप्टोकरें सी का उपयोग करते हैं।

Report Errors in the PDF - ebooks@affairscloud.com Copyright 2014-2023 @ AffairsCloud.com 140


ACQUISITION AND MERGERS
1. अगस्त 2023 में, भारतीय प्रनतस्पधाा आयोग (CCI) िे ओरोिेि-िूिसि L.P. द्वारा ____________ (िंपिी) में
शेयरधाररता िे अनधग्रहण िो मंिूरी दे दी।
1)डे लॉइट टे क्नोलॉजी
2)सब्रसलयो होक्कडोंग्स, इों क.
3)IBM ग्लोबल सबजनेस ससवधसेज
4)एक्सेंचर टे क्नोलॉजी कोंसक्कल्टोंग
5)कैपजेसमनी SE
उत्तर -2)निनियो होखडं ग्स, इं ि.
स्पष्टीिरण:
भारतीय प्रसतस्पर्ाध आयोग (CCI) ने ओरोिेि-िूिि L.P. िारा निनियो होखडं ग्स, इं ि. में एक सनसित शेयरर्ाररता के
असर्ग्रहि को मोंजूरी दे दी है।
i.असर्ग्रहिकताध : ओरोजेन-ब्रूनसन L.P. प्रस्तासवत सोंयोजन को सक्षम करने के सलए सोंयुक्त राज्य अमेररका (USA) में
गसठत एक सीसमत कोंपनी है ।
• यह ऑरोजेन होक्कडोंग्स LLC के ग्रुप और अटै रोस ग्रुप, इों क. के ग्रुप का एक सहस्सा है ।
ii.लक्ष्: US क्कथित सब्रसलयो होक्कडोंग्स, इों क. एक वैसश्वक प्रौद्योसगकी परामशध और व्यवसाय समार्ान प्रदाता है जो बडे डे टा
एनासलसटक्स पर केंसद्रत है ।
• यह भारत में अपनी सक्कब्सडी "सब्रसलयो इों सडया टे क्नोलॉजीज प्राइवेट सलसमटे ड" के माध्यम से मौजूद है ।

2. नसतंबर 2023 में, भारतीय प्रनतस्पधाा आयोग (CCI) िे निस्तारा िे स्वानमत्व िािी टाटा SIA एयरिाइं स
निनमटे ि (TSAL) िे ______________ में चरण I नििय िो मंिूरी दे दी।
1)इों सडगो
2)स्पाइसजेट
3)गो फस्ट्ध
4)एयर इों सडया
5)अकासा एयर
उत्तर-4)एयर इं निया
स्पष्टीिरण:
भारतीय प्रसतस्पर्ाध आयोग (CCI) ने सवस्तारा के स्वासमत्व वाली टाटा SIA एयरलाइों स सलसमटे ड (TSAL) के एयर
इं निया सलसमटे ड (AIL) में पहले चरि के सवलय को मोंजूरी दे दी, सजसमें AIL जीसवत इकाई (मज्डध एों सटटी) के रूप में
उभरे गी। AIL ने अप्रैल 2023 में सवलय के सलए अनुरोर् सकया।
• ससोंगापुर एयरलाइों स सलसमटे ड (SIA) इकाई में 2,059 करोड रुपये के सनवेश के साि एयर इों सडया में 25.1%
सहस्सेदारी रखेगी।
• TSAL TSPL और SIA के बीच एक सोंयुक्त उद्यम है , सजसमें TSPL और SIA की कुल शेयरर्ाररता में क्रमशः
51% और 49% सहस्सेदारी है।
i.इकाई के पास नई सदल्ली (सदल्ली)-मुोंबई (महाराष्ट्र) मागध पर कुल उडानोों का 49% और नई सदल्ली (सदल्ली) -बेंगलुरु
(कनाधटक) मागध पर कुल उडान का 52% सहस्सा होगा।

Report Errors in the PDF - ebooks@affairscloud.com Copyright 2014-2023 @ AffairsCloud.com 141


3. नसतंबर 2023 में, भारतीय प्रनतस्पधाा आयोग (CCI) िे इग्नाइट िक्ज़मबगा होखडं ग्स S.à r.l द्वारा RHI मैग्नेनसटा
NV िे ___________ इखिटी शेयरों िे अनधग्रहण िो मंिूरी दे दी।
1)36.2%
2)20.6%
3)32.4%
4)29.9%
5)40.1%
उत्तर- 4)29.9%
स्पष्टीिरण:
भारतीय प्रसतस्पर्ाध आयोग (CCI) ने इग्नाइट िक्ज़मबगा होखडं ग्स S.à r.l.. िारा RHI मैग्नेनसटा NV के 29.9% इक्कक्वटी
शेयरोों के असर्ग्रहि को मोंजूरी दे दी है।
• अनधग्रहणिताा- इग्नाइट लक्ज़मबगध होक्कडोंग्स S.à r.l.
• िक्ष्य– RHI मैग्नेससटा NV
• इस असर्ग्रहि से RHI मैग्नेससटा इों सडया सलसमटे ड (RHIM इं निया) का अप्रत्यक्ष असर्ग्रहि हुआ, क्ोोंसक प्रस्तासवत
सोंयोजन के सहस्से के रूप में भारतीय कोंपनी का कोई शेयर नहीों खरीदा जा रहा िा।

4. नसतंबर 2023 में, GQG पाटा िसा िे IDFC फस्ट्ा बैंि िे प्रबंध निदे शि (MD) & मुख्य िायािारी अनधिारी
(CEO) V िैद्यिाथि से __________ रुपये में IDFC फस्ट्ा बैंि में 5.07 िरोड इखिटी शेयर हानसि निए।
1) 901.01 करोड
2) 478.70 करोड
3) 823.32 करोड
4) 725.05 करोड
5) 600.34 करोड
उत्तर- 2) 478.70 िरोड
स्पष्टीिरण:
GQG पाटा िसा ने IDFC फस्ट्ा बैंि के प्रबोंर् सनदे शक (MD) & मुख्य कायधकारी असर्कारी (CEO) V िैद्यिाथि से
लगभग 478.70 िरोड रुपये में IDFC फस्ट्ध बैंक में 5.07 करोड इक्कक्वटी शेयर (कुल इक्कक्वटी का 0.8%) हाससल सकए
हैं। यह लेनदे न 94.5 रुपये प्रसत शेयर की कीमत पर हुआ।
• 478.7 करोड रुपये की सबक्री आय में से, 229 करोड रुपये IDFC फस्ट्ध बैंक को बैंक के नए शेयरोों की
सदस्यता के सलए और 240.5 करोड रुपये स्ट्ॉक सवकल्पोों का उपयोग करने के सलए आयकर भुगतान के सलए
भुगतान करना होगा।
• 9.2 करोड रुपये पूवध-प्रसतबद्ध कारिोों के सलए योगदान होोंगे।

5. नसतंबर 2023 में, िैनश्वि नििेश फमा KKR & िंपिी इं ि अपिी िुि इखिटी नहस्सेदारी िो पूरी तरह से पतिा
आधार पर 1.17% से बढ़ािर ___________ िरिे िे निए ररिायंस ररटे ि िेंचसा निनमटे ि (RRVL) में 2,069.50
िरोड रुपये िा नििेश िरिे िे निए तैयार है।
1)2.09%
2)1.73%
3)1.42%

Report Errors in the PDF - ebooks@affairscloud.com Copyright 2014-2023 @ AffairsCloud.com 142


4)1.50%
5)2.14%
उत्तर-3)1.42%
स्पष्टीिरण:
वैसश्वक सनवेश फमध KKR & कोंपनी इों क, सजसे कोहलबगध क्रैसवस रॉबट्ध स एों ड कोंपनी के नाम से भी जाना जाता है , अपनी
कुल इक्कक्वटी सहस्सेदारी को पूरी तरह से पतला आर्ार पर 1.17% से बढाकर 1.42% करने के सलए ररलायोंस ररटे ल वेंचसध
सलसमटे ड (RRVL) में 2,069.50 िरोड रुपये का सनवेश करने के सलए तैयार है ।
i.यह सनवेश पूरी तरह से पतला आर्ार पर RRVL में 0.25% की असतररक्त इक्कक्वटी सहस्सेदारी में तब्दील हो जाएगा।
ii.यह सनवेश 8.361 लाख करोड रुपये (लगभग 100 सबसलयन अमेररकी डॉलर) के प्री-मनी इक्कक्वटी मूल् पर है।

6. लसतंबर 2023 मेंहाि ही में भारतीय प्रलतस्पिाा आयोग (CCI) की मंिूरी के संबंि में लनम्नलिस्कित में से कौन
सा लबंदु "सही" है?
A)CCI ने कुछ व्यस्कक्तयों के साथ PI अपॉच्र्युलनटीि फंि-I िीम-II द्वारा TVS क्रेलिट सलवासेि लिलमटे ि में कुछ
शेयरिाररता के प्रस्तालवत अलिग्रहण को मंिूरी दे दी है।
B)प्रस्तालवत संयोिन सेंटेिा मॉरीशस होस्कल्डं ग्स लिलमटे ि द्वारा क्वालिटी केयर इं लिया लिलमटे ि (QCIL) में कुि
इस्कक्वटी शेयरिाररता का िगभग 24.16% के अलिग्रहण से संबंलित है।
C)प्रस्तालवत संयोिन में टच हेल्थकेयर प्राइवेट लिलमटे ि से BCP एलशया II टॉपको IV प्राइवेट लिलमटे ि द्वारा
क्वालिटी केयर इं लिया लिलमटे ि की कुि इस्कक्वटी शेयरिाररता का ~ 72.49% पूरी तरह से पतिा आिार पर
अलिग्रहण करने की पररकल्पना की गई है।
1)केवल A
2)केवल A & B
3)केवल B & C
4)केवल A & C
5)सभी A, B & C
उत्तर -5)सभी A, B & C
स्पष्टीकरण:
20 श्चसतंबर, 2023 को भारतीय प्रश्चतस्पर्ाग आयोग (CCI) ने श्चनम्नश्चलस्खत प्रस्तावों को मंिूरी दे दी है:
i.2452991 ओंटाररयो श्चलश्चमटे ि द्वारा हाईवे इं फ्रास्ट्र क्चर टर स्ट् में अश्चतररक्त यूश्चनट होस्र्ल्ं ग के अश्चर्ग्रहि और 2743298
ओंटाररयो श्चलश्चमटे ि द्वारा हाईवे कंसेशन्सवन प्राइवेट श्चलश्चमटे ि में इस्िटी श्चहस्सेदारी के अश्चर्ग्रहि को मंिूरी दे दी।
ii.हाल ही में, भारतीय प्रश्चतस्पर्ाग आयोग (CCI) ने कुछ व्यस्क्तयों के साि PI अपॉच्र्युश्चनटीि फंि-I स्कीम-II द्वारा TVS
क्रेलिट सलवासेि लिलमटे ि में कुछ शेयरर्ाररता के प्रस्ताश्चवत अश्चर्ग्रहि को मंिूरी दे दी है।
• प्रस्ताश्चवत संयोिन अश्चर्ग्रहिकतागओं द्वारा टारगेट की 10.98% श्चहस्सेदारी (पूरी तरह से पतला आर्ार
पर) के अश्चर्ग्रहि से संबंश्चर्त है।
• PIOF-II लक्ष्य में लगभग 10.79% शेयरर्ाररता प्राप्त करे गा, िबश्चक व्यस्क्तगत अश्चर्ग्रहिकताग सामूश्चहक
रूप से लक्ष्य में 0.19% शेयरर्ाररता प्राप्त करें गे।
iii.प्रस्ताश्चवत संयोिन सेंटेिा मॉरीशस होस्कल्डंग्स श्चलश्चमटे ि द्वारा िाश्चलटी केयर इं श्चिया श्चलश्चमटे ि (QCIL) में कुल इस्िटी
शेयरर्ाररता का लगभग 24.16% के अश्चर्ग्रहि से संबंश्चर्त है।

Report Errors in the PDF - ebooks@affairscloud.com Copyright 2014-2023 @ AffairsCloud.com 143


iv.प्रस्ताश्चवत संयोिन में टच हेल्थकेयर प्राइवेट लिलमटे ि से BCP एश्चशया II टॉपको IV प्राइवेट श्चलश्चमटे ि (BCP एश्चशया
II / अश्चर्ग्रहिकताग) द्वारा क्वालिटी केयर इं लिया लिलमटे ि (टागेट) की कुल इस्िटी शेयरर्ाररता का ~ 72.49% पूरी
तरह से पतला आर्ार पर अश्चर्ग्रहि की पररकल्पना की गई है ।

7. लनम्नलिस्कित में से लकस बैंक में, HDFC एसेट मैनेिमेंट कंपनी लिलमटे ि को हाि ही में (लसतंबर’23 में) भारतीय
ररिवा बैंक (RBI) से भुगतान की गई शेयर पूंिी या वोलटं ग अलिकारों के 9.5% तक की अलतररक्त लहस्सेदारी
हालसि करने की मंिूरी लमि गई है?
1)कोटक मश्चहंद्रा बैंक
2)करूर वैश्य बैंक
3)DCB बैंक श्चलश्चमटे ि
4)1 और 2 दोनों
5)2 और 3 दोनों
उत्तर- 5)2 और 3 दोनों
स्पष्टीकरण:
HDFC एसेट मैनेिमेंट कंपनी श्चलश्चमटे ि (HDFC AMC) को करूर वैश्य बैंक श्चलश्चमटे ि (KVB) और DCB बैंक
लिलमटे ि की भुगतान की गई शेयर पूंिी या वोश्चटंग अश्चर्कारों के 9.5% तक की अश्चतररक्त श्चहस्सेदारी हाश्चसल करने के
श्चलए भारतीय ररिवग बैंक (RBI) से मंिूरी श्चमल गई।
• यह मंिूरी HDFC AMC द्वारा RBI को श्चकए गए आवेदन के संदभग में दी गई है।
• दोनों बैंकों में HDFC AMC की श्चहस्सेदारी हर समय बैंक की भुगतान की गई शेयर पूंिी या वोश्चटंग
अश्चर्कार के (व्यस्क्तगत रूप से) 9.5% से अश्चर्क नहीं होनी चाश्चहए।
i.RBI ने HDFC AMC को श्चनदे श श्चदया श्चक वह RBI की पूवग मंिूरी के श्चबना शेयरर्ाररता को 5% से कम न करें ।
• यश्चद कुल श्चहस्सेदारी 5% से कम हो िाती है, तो इसे बैंक की भुगतान की गई शेयर पूंिी या वोश्चटंग
अश्चर्कारों के 5% या उससे अश्चर्क तक ब़िाने के श्चलए RBI की पूवग मंिूरी की आवश्यकता होगी।
ii.RBI ने HDFC AMC को DCB बैंक के शेयर खरीदने के श्चलए एक साल का समय श्चदया है।
• अगर HDFC AMC एक साल के भीतर शेयर नहीं खरीदती है, तो RBI की मंिूरी रद्द कर दी िाएगी।

DEFENCE
1. 137 िनमायों िािी भारतीय सेिा िी टु िडी 31 अगस्त से 14 नसतंबर 2023 ति आयोनित होिे िािे
बहुराष्टरीय नत्र-सेिा संयुक्त सैन्य अभ्यास "अभ्यास BRIGHT STAR- 23" में भाग िेिे िे निए ____________
(दे श) िे निए रिािा हो गई है।
1)इों डोनेसशया
2)बाोंग्लादे श
3)श्रीलोंका
4)समस्र
5)जापान
उत्तर-4)नमस्र

Report Errors in the PDF - ebooks@affairscloud.com Copyright 2014-2023 @ AffairsCloud.com 144


स्पष्टीिरण:
137 िनमायों वाली भारतीय सेिा की टु कडी नमस्र के हम्माम में मोहम्मद नागुइब सैन्य अड्डे पर 31 अगस्त से 14
ससतोंबर 2023 तक आयोसजत होने वाले एक बहुराष्ट्रीय सत्र-सेवा संयुक्त सैन्य अभ्यास "अभ्यास BRIGHT STAR- 23"
में भाग लेने के सलए समस्र के सलए रवाना हो गई है।
• अभ्यास BRIGHT STAR-23 का नेतृत्व US CENTCOM (सोंयुक्त राज्य अमेररका सेंटरल कमाोंड) और समस्र की
सेना िारा सकया जाएगा।
• यह अभ्यास भारतीय सेना को रक्षा सहयोग बढाने के उद्दे श्य से अन्य सेनाओों के साि सवोत्तम प्रिाओों और
अनुभवोों को साझा करने का एक अनूठा अवसर प्रदान करे गा।
• 2023 संस्करि में, संयुक्त राज्य अमेररका, सऊदी अरब, ग्रीस और कतर सश्चहत 34 दे श अभ्यास BRIGHT
STAR-23 में भाग लेंगे। यह पश्चिम एश्चशया और उत्तरी अफ्रीका क्षे त्र में अब तक का सबसे बड़ा संयुक्त सैन्य
अभ्यास होगा।
• पहली बार, भारतीय सशस्त्र बल 549 कसमधयोों की कुल शक्कक्त के साि BRIGHT STAR अभ्यास में भाग ले रहे
हैं।
• भारतीय सेना का प्रसतसनसर्त्व 23 JAT बटासलयन की एक टु कडी िारा सकया जा रहा है।

2. निस दे श िी िौसेिा िे भारतीय िौसेिा (IN) िे साथ नमििर हाि ही में (नसतंबर'23 में) पैसेि अभ्यास
(PASSEX) िा आयोिि निया है ?
1)इों डोनेसशया
2)श्रीलोंका
3)बाोंग्लादे श
4)फ्राोंस
5)कजासकस्तान
उत्तर- 2)श्रीिंिा
स्पष्टीिरण:
भारतीय नौसेना (IN) और श्रीिंिा नौसेना (SLN) सोंयुक्त रूप से श्रीलोंका के कोलोंबो तट पर पैसेज अभ्यास (PASSEX)
का आयोजन कर रही हैं।
• भारतीय नौसेना जहाज (INS) सदल्ली और SLN जहाज सवजयबाहु ने अभ्यास में भाग सलया।
• यह अभ्यास 1 से 3 ससतोंबर 2023 तक INS सदल्ली की श्रीलोंका की दो सदवसीय यात्रा के दौरान हुआ।
• INS सदल्ली भारतीय नौसेना का पहला स्वदे शी रूप से सडजाइन और सनसमधत सनदे सशत समसाइल सवध्वोंसक है ,
सजसे मुोंबई, महाराष्ट्र में मझगाोंव डॉक सलसमटे ड िारा सनसमधत सकया गया है ।
• INS सदल्ली के कमाोंडर कैप्टन असभषेक कुमार और INS सदल्ली के चालक दल ने 1987-91 तक IPKF
ऑपरे शन के दौरान श्रीलोंका में अपने प्रािोों की आहुसत दे ने वाले भारतीय सैसनकोों के भारतीय शाोंसत रक्षा बल
(IPKF) स्मारक पर श्रद्धाोंजसल असपधत की।

3. निस दे श िी िौसेिा िे भारतीय िौसेिा िे साथ नमििर हाि ही में (नसतंबर’23 में) अरब सागर में
नद्वपक्षीय समुद्री अभ्यास िरुण (िरुण 2023)िा 21िां संस्करण आयोनित निया है ?
1)जापान
2)फ़्ाोंस
3)ऑस्ट्र े सलया

Report Errors in the PDF - ebooks@affairscloud.com Copyright 2014-2023 @ AffairsCloud.com 145


4)बाोंग्लादे श
5)इों डोनेसशया
उत्तर- 2)फ्रांस
स्पष्टीिरण:
भारतीय िौसेिा और फ्रांसीसी िौसेिा ने अरब सागर में सिपक्षीय समुद्री अभ्यास िरुण (िरुण 2023)के 21 िें
संस्करण के फेि II का आयोजन सकया। 3 सदवसीय अभ्यास में दोनोों दे शोों िारा सनदे सशत समसाइल सफ्रगेट, टैं कर,
समुद्री गश्ती सवमान और असभन्न हेलीकॉप्टरोों की भागीदारी दे खी गई।
• वरुि 23 दो चरिोों में आयोसजत सकया गया िा। 'िरुण-2023' िा फेि I 16 से 20 जनवरी 2023 तक भारत
के पसिमी समुद्री तट पर आयोसजत सकया गया िा।
• वरुि का 21वाों सोंस्करि भारत-फ्राोंसीसी रिनीसतक साझेदारी के 25वें वषध का प्रतीक है।

4. नसतंबर 2023 में, भारतीय सेिा िे सभी दु श्मि िे बख्तरबंद िाहिों िे खििाफ गनतशीिता प्रदाि िरिे िे
निए छह सौ स्वदे शी निनमात स्व-निखिय एं टी-टैं ि माइं स, "निभि" िो शानमि निया।
निभि िा निमााण भारतीय सेिा िे निए ___________ (िंपिी) द्वारा निया िा रहा है।
1)मसहोंद्रा & मसहोंद्रा सलसमटे ड
2)कल्ािी ग्रुप
3)टाटा मोटसध सलसमटे ड
4)मारुसत सुजुकी इों सडया
5)अशोक लीलैंड सलसमटे ड
उत्तर- 2)िल्याणी ग्रुप
स्पष्टीिरण:
भारतीय सेिा ने सभी दु श्मन के बख्तरबोंद वाहनोों के क्कखलाफ गसतशीलता प्रदान करने के सलए छह सौ स्वदे शी सनसमधत
स्व-सनक्किय एों टी-टैं क माइों स को शासमल सकया है , सजन्ें "निभि" के नाम से जाना जाता है।
i.भारतीय सेना के शस्त्रागार में सवभव की शुरूआत ने दु श्मन के बख्तरबोंद वाहनोों के क्कखलाफ उनकी सुरक्षा को बढाया
और उनकी समग्र पररचालन क्षमताओों को मजबूत सकया।
ii.निभि, प्वाइं ट-अटै ि एं टी-टैं ि गोिा-बारूद, भारत में रक्षा अनुसोंर्ान और सवकास सोंगठन (DRDO) के सहयोग
से सवकससत सकया गया िा। इस माइों स में ऑपरे टर की सुरक्षा सुसनसित करने के सलए सुरक्षा और एक्चुएशन तोंत्र हैं।
iii.सवभव का सनमाधि भारतीय सेना के सलए कल्ािी ग्रुप िारा सकया जा रहा है।

5. नसतंबर 2023 में, रक्षा मंत्रािय (MoD) िे भारतीय सेिा िे निए 150 से 500 km िी रें ि िािी ___________
सामररि बैनिखस्ट्ि नमसाइिों िी एि रे निमेंट िरीदिे िे प्रस्ताि िो मंिूरी दी।
1)प्रलय
2)अमोघा
3)सनभधय
4)प्रहार
5)सत्रशूल
उत्तर- 1)प्रिय

Report Errors in the PDF - ebooks@affairscloud.com Copyright 2014-2023 @ AffairsCloud.com 146


स्पष्टीिरण:
रक्षा मोंत्रालय (MoD) ने भारतीय सेना के सलए 150 से 500 सकलोमीटर (Km) की रें ज वाली 'प्रिय' सामररि
बैनिखस्ट्ि नमसाइिों की एक रे सजमेंट खरीदने के प्रस्ताव को मोंजूरी दे दी है।
• इन समसाइलोों को वास्तसवक सनयोंत्रि रे खा (LAC) और सनयोंत्रि रे खा (LoC) पर तैनात सकया जाएगा, जो क्रमशः
चीन और पासकस्तान की सीमा है।
• समसाइलोों को DRDO िारा सवकससत सकया गया िा। पारों पररक हसियारोों वाली इन समसाइलोों का इस्तेमाल
सामररक भूसमकाओों में सकया जाएगा।
• 'प्रलय' एक ठोस प्रिोदक रॉकेट मोटर और अन्य नई प्रौद्योसगसकयोों िारा सोंचासलत है। समसाइल मागधदशधन
प्रिाली में अत्यार्ुसनक नेसवगेशन और एकीकृत एसवयोसनक्स शासमल हैं।
• प्रलय एक अर्ध-बैसलक्कस्ट्क समसाइल है और सेना की सूची में सतह से सतह पर मार करने वाली सबसे लोंबी दू री
की समसाइल होगी। यह मोबाइल लॉन्चर से लॉन्च करने में सक्षम है।
• यह लगभग 350 से 700 सकलोग्राम (kg) का पारों पररक हसियार ले जाने में सक्षम है , जो इसे घातक दों डात्मक
क्षमता प्रदान करता है।

6. निस दे श िी िौसेिा िे भारतीय िौसेिा (IN) िे साथ हाि ही में (नसतंबर'23 में) SIMBEX 2023 िा 30िां
संस्करण शुरू निया है ?
1)ससोंगापुर
2)सेशेल्स
3)बाोंग्लादे श
4)इों डोनेसशया
5)श्रीलोंका
उत्तर-1)नसंगापुर
स्पष्टीिरण:
भारतीय नौसेना (IN) और ससोंगापुर गिराज्य की नौसेना (RSN) ससोंगापुर के तट पर दसक्षि चीन सागर के दसक्षिी सहस्सोों
में 21 से 28 ससतोंबर 2023 तक एक सोंयुक्त नौसैसनक युद्ध अभ्यास, ससोंगापुर-भारत समुद्री सिपक्षीय अभ्यास (SIMBEX
2023)के 30 िें सोंस्करि में भाग लेते हैं ।
• SIMBEX 2023 दो चरिोों में आयोसजत सकया जाता है सजसमें भूसम चरि और समुद्री चरि शासमल हैं।
• भूनम चरण:यह 21 से 24 ससतोंबर 2023 तक RSS ससोंगापुर - चाोंगी नेवल बेस, ससोंगापुर में आयोसजत सकया जाता
है।
• समुद्री चरण:यह 25 से 28 ससतोंबर 2023 तक अोंतरराष्ट्रीय जल क्षेत्र में दसक्षि चीन सागर के दसक्षिी इलाकोों में
आयोसजत सकया जाता है।
i.भारतीय नौसेना के राजपूत श्रेिी के सवध्वोंसक INS रिसवजय, कामोटाध श्रेिी के कावेट INS कवरत्ती, ससोंर्ुघोष श्रेिी की
डीजल-इलेक्कररक पनडु ब्बी INS ससोंर्ुकेसरी और एक P-8I समुद्री गश्ती सवमान इस अभ्यास में भाग ले रहे हैं ।

7. उस भारतीय िौसेिा िहाि (INS) िा िाम बताइए निसिे हाि ही में (नसतंबर'23 में) पहिे भारत-
इं िोिेनशया-ऑस्ट्रे निया नत्रपक्षीय समुद्री अभ्यास में भाग निया है।
1)INS सशवासलक
2)INS सतपुडा
3)INS सह्यासद्र

Report Errors in the PDF - ebooks@affairscloud.com Copyright 2014-2023 @ AffairsCloud.com 147


4)INS सत्रशूल
5)INS तलवार
उत्तर-3)INS सह्यानद्र
स्पष्टीिरण:
21-22 ससतोंबर, 2023 तक, भारतीय नौसेना (IN), रॉयल ऑस्ट्र े सलयाई नौसेना (RAN) और इों डोनेसशयाई नौसेना के बीच
पहिा नत्रपक्षीय समुद्री साझेदारी अभ्यास आयोसजत सकया गया िा, जहाों भारत का प्रसतसनसर्त्व युद्धपोत भारतीय
नौसेना जहाज (INS) सह्याद्री िारा सकया गया िा, जो सहोंद-प्रशाोंत में तैनात समशन है।
i.INS सह्यासद्र भारतीय नौसेना का स्वदे श सनसमधत युद्धपोत है सजसे सहोंद-प्रशाोंत क्षेत्र में तैनात सकया गया िा।
ii.INS सह्याद्री स्वदे शी रूप से सडजाइन और सनसमधत प्रोजेर-17 श्रेिी के मल्टीरोल स्ट्े ल्थ सफ्रगेट का तीसरा जहाज है।
यह मझगाोंव डॉक सलसमटे ड, मुोंबई, महाराष्ट्र में बनाया गया िा और इसकी कमान कैप्टन राजन कपूर के पास है ।

8. निस राज्य िी सेिा िे भारतीय सेिा िे साथ नमििर हाि ही में (नसतंबर'23 में) युि अभ्यास 23 िा 19िां
संस्करण शुरू निया है ?
1)श्रीलोंका
2)फ़्ाोंस
3)बाोंग्लादे श
4)सोंयुक्त राज्य अमेररका
5)इों डोनेसशया
उत्तर-4)संयुक्त राज्य अमेररिा
स्पष्टीिरण:
अभ्यास युि अभ्यास 23, भारतीय सेिा और संयुक्त राज्य की सेना के बीच वासषधक सोंयुक्त सैन्य प्रसशक्षि अभ्यास
का 19िां सोंस्करि फोटध वेनराइट, अलास्का, सोंयुक्त राज्य अमेररका (USA) में आयोसजत सकया जा रहा है ।
• यह 14 सदनोों तक चलने वाला अभ्यास है जो 25 ससतोंबर 2023 को शुरू हुआ और 8 अरू बर 2023 को समाप्त
होगा।
• युद्ध अभ्यास 23 का सवषय सोंयुक्त राष्ट्र (UN) जनादे श के अध्याय VII के तहत ‘एम्प्लॉयमेंट ऑफ़ एन इों टीग्रेटेड
बैटल ग्रुप इन माउों टेन/ एक्सटर ीम क्लाइमेसटक कोंडीशोंस’ है।
• भारतीय सेना की टु कडी में 350 कमी और US सेना की पहली सब्रगेड कॉम्बैट टीम की 1-24 इन्फैंटर ी बटासलयन
शासमल है , जो अभ्यास में भाग ले रही है।
• भारत की ओर से प्रमुख बटासलयन MARATHA लाइट इन्फैंटर ी रे सजमेंट से सोंबद्ध है ।

SCIENCE AND TECHNOLOGY


1. उस िेंद्रीय मंत्री िा िाम बताइए निन्ोंिे हाि ही में (अगस्त’23 में) अंतर-राज्य पररषद सनचिािय िा ई-
संसाधि िेब पोटा ि िॉन्च निया है।
1)असमत शाह (केंद्रीय गृह मोंत्री)
2)सनमधला सीतारमि (केंद्रीय सवत्त मोंत्री)
3)प्रह्लाद जोशी (केंद्रीय सोंसदीय कायध मोंत्री)
4)वीरें द्र कुमार (केंद्रीय सामासजक न्याय और असर्काररता मोंत्री)
5)सजतेंद्र ससोंह (सवज्ञान एवों प्रौद्योसगकी राज्य मोंत्री)

Report Errors in the PDF - ebooks@affairscloud.com Copyright 2014-2023 @ AffairsCloud.com 148


उत्तर-1)अनमत शाह (िेंद्रीय गृह मंत्री)
स्पष्टीिरण:
गृह मोंत्रालय (MHA) के केंद्रीय मोंत्री अनमत शाह ने गाोंर्ीनगर, गुजरात में पसिमी क्षेत्रीय पररषद की 26वीों बैठक के
दौरान अोंतर-राज्य पररषद ससचवालय, MHA का ई-सोंसार्न वेब पोटध ल लॉन्च सकया। यह पोटध ल क्षेत्रीय पररषदोों के कायों
को सक्षम करे गा।
• असमत शाह की अध्यक्षता में हुई बैठक में गुजरात के मुख्यमोंत्री (CM) भूपेन्द्रभाई पटे ल, महाराष्ट्र के CM एकनाि
सशोंदे, गोवा के CM प्रमोद सावोंत और दादरा और नगर हवेली और दमन और दीव के प्रशासक प्रफुल्ल खोडा
पटे ल और पसिमी क्षेत्र के राज्योों के अन्य गिमान्य व्यक्कक्त शासमल हुए।
i.यह पोटध ल 28 मई 1990 को अपने गठन के बाद से अोंतर-राज्य पररषद और इसकी थिायी ससमसत और 1957 में अपनी
थिापना के बाद से क्षेत्रीय पररषदोों और उनकी थिायी ससमसत की बैठकोों के कायधवृत्त और एजेंडे जैसे महत्वपूिध दस्तावेजोों
का भोंडार है।
ii.इसका उपयोग केंद्रीय मोंत्रालयोों/सवभागोों और राज्य सरकार/केंद्र शाससत प्रदे शोों (UT) िारा नीसतगत हस्तक्षेप के सलए
सकया जाएगा।

2. िौि सा मेटरो रे ििे हाि ही में (अगस्त’23 में) उि महािगरों िे निनशष्ट समूह िा नहस्सा बि गया है िो स्ट्ीि
थिा रे ि से एल्युमीनियम थिा रे ि में स्थािांतररत हो गए हैं ?
1)सूरत मेटरो रे लवे
2)नोएडा मेटरो रे लवे
3)कोलकाता मेटरो रे लवे
4)मुोंबई मेटरो रे लवे
5)सदल्ली मेटरो रे लवे
उत्तर-3)िोििाता मेटरो रे ििे
स्पष्टीिरण:
भारत सरकार के रे ल मोंत्रालय के तहत िोििाता मेटरो रे ििे ने अपने सभी आगामी गसलयारोों में नमनश्रत एल्यूमीनियम
थिा रे ि का उपयोग करने के साि-साि मौजूदा गसलयारोों में स्ट्ील िडध रे ल के साि रे टर ो-सफटमेंट का उपयोग करने का
सनिधय सलया है।
i.इस मेटरो रे लवे के साि, कोलकाता महानगरोों के उस सवसशष्ट् समूह का सहस्सा बन जाएगा जो स्ट्ील िडध रे ल से
एल्ुमीसनयम िडध रे ल में थिानाोंतररत हो गया है। समूह में लोंदन (यूनाइटे ड सकोंगडम-UK), मॉस्को (रूस), बसलधन (जमधनी),
म्यूसनख (जमधनी), और इस्ताोंबुल (तुकी) मेटरो शासमल हैं।
ii.िडध रे ल रे लगासडयोों, टर ामोों और अन्य रे ल वाहनोों को सवद् युत शक्कक्त प्रदान करने की एक सवसर् है।
iii.यह रे लवे टर ै क की दो मुख्य पटररयोों के साि-साि या उनके बीच जमीन के पास लगाई गई एक असतररक्त रे ल है।
इसमें सवद् युत र्ारा प्रवासहत होती है जो टर े न की सवद् युत प्रिोदन प्रिाली को शक्कक्त प्रदान करती है।

3. निस िंपिी िे हाि ही में (अगस्त’23 में) भारतीय िौसेिा िे निए पहिा िाइनिंग सपोटा क्राफ्ट (DSC)
'DSC A 20' (यािा 325)िॉन्च निया है ?
1)इों जीसनयसध इों सडया सलसमटे ड
2)टीटागढ रे ल ससस्ट्म्स सलसमटे ड
3)कोंटे नर कॉपोरे शन ऑफ इों सडया सलसमटे ड
4)रे ल सवकास सनगम सलसमटे ड

Report Errors in the PDF - ebooks@affairscloud.com Copyright 2014-2023 @ AffairsCloud.com 149


5)भारत हेवी इलेक्कररकल्स सलसमटे ड
उत्तर-2)टीटागढ़ रे ि नसस्ट्म्स निनमटे ि
स्पष्टीिरण:
टीटागढ़ रे ि नसस्ट्म्स निनमटे ि (TRSL), सजसे पहले टीटागढ वैगर्न् सलसमटे ड (TWL) के नाम से जाना जाता िा, ने
पसिम बोंगाल के टीटागढ सशपयाडध में भारतीय नौसेना के सलए एक डाइसवोंग सपोटध क्राफ्ट (DSC) 'DSC A 20' (यािा
325)लॉन्च सकया।
• यह TRSL िारा भारतीय नौसेना को सौोंपे जाने वाले पाों च DSC जहाजोों में से पहला है , सजसका कुल अनुबोंर् मूल्
लगभग 175 करोड रुपये है ।
• लॉन्च समारोह में भारतीय नौसेना के वाइस एडसमरल सों जय महें द्रू ने भाग सलया।
i.DSC एक अत्यार्ुसनक वॉटरक्राफ्ट है जो हर सोंभासवत पानी के नीचे के वातावरि में डाइसवोंग सोंचालन में सहायता के
सलए व्यापक प्रौद्योसगकी क्षमताओों से लैस है ।
ii.इन जहाजोों को भारतीय नौवहन रसजस्ट्र (IRS) के उसचत नौसेना सनयमोों और सवसनयमोों के अनुसार भारत में सडजाइन
और सनसमधत सकया गया है।

4. उस िंपिी िा िाम बताइए निसिे हाि ही में (अगस्त’23 में) एि उच्च-ररज़ॉल्यूशि एररयि िर ोि-आधाररत
नसंथेनटि एपचार रिार (SAR) नसस्ट्म पेश निया है।
1)ध्रुव स्पेस
2)स्काईरूट एयरोस्पेस
3)आद्यः एयरोस्पेस
4)असग्नकुल कॉसमॉस
5)GalaxEye
उत्तर-5)GalaxEye
स्पष्टीिरण:
SpaceTech स्ट्ाटध -अप कोंपनी GalaxEye ने एक उच्च-ररज़ॉल्ूशन एररयल डर ोन-आर्ाररत ससोंिेसटक एपचधर रडार
(SAR) नसस्ट्म पेश सकया है। यह अत्यार्ुसनक प्रौद्योसगकी बरसात या बादल की क्कथिसत में भी, हर मौसम में असार्ारि
रूप से सवस्तृत और उच्च-ररज़ॉल्ूशन वाली इमेसजोंग सक्षम बनाती है।
i.कहा जाता है सक डर ोन आर्ाररत SAR प्रिाली भारत में सकसी सनजी कोंपनी िारा सवकससत पहली ऐसी प्रौद्योसगकी है।
ii.GalaxEye की इन-हाउस सवकससत 'डे टा फ़्यूज़न प्रौद्योसगकी' अोंतररक्ष से सवस्तृत अोंतदृधसष्ट् और डे टा प्रदान करे गी,
सजससे उपग्रह समूहोों को वायुमोंडलीय बार्ाओों का सामना सकए सबना हर मौसम में इमेसजोंग करने का असर्कार समलेगा,
जो वतधमान एकल-सेंसर उपग्रहोों को "प्लेग" करते हैं।

5. अगस्त 2023 में, प्रोिेर 17A िे 7िें िीिनगरर िास स्ट्ील्थ नफ्रगेट, भारतीय िौसेिा िहाि (INS)
____________ िो मुंबई, महाराष्टर में मझगांि िॉि नशपनबडसा निनमटे ि (MDL) में िॉन्च निया गया था।
1)INS महेंद्रसगरर
2)INS तारासगरी
3)INS सहमसगरर
4)INS उदयसगरर
5)INS दू नासगरी
उत्तर- 1)INS महेंद्रनगरर

Report Errors in the PDF - ebooks@affairscloud.com Copyright 2014-2023 @ AffairsCloud.com 150


स्पष्टीिरण:
नीलसगरर क्लास स्ट्ील्थ सफ्रगेट भारतीय नौसेना जहाज (INS) महेंद्रनगरर को मझगाोंव डॉक सशपसबडसध सलसमटे ड
(MDL), मुोंबई, महाराष्ट्र में लॉन्च सकया गया िा।
• INS महेंद्रनगरर भारतीय नौसेना (IN) के प्रोिेर 17A का सातवाों और आक्कखरी स्ट्ील्थ सफ्रगेट है। यह MDL
िारा सनसमधत चौिा युद्धपोत भी है।
• जहाज का शुभारों भ भारत के उपराष्ट्रपसत जगदीप र्नखड की पत्नी डॉ. सुदेश र्नखड ने सकया।
• INS महेंद्रसगरर का नाम ओसडशा में क्कथित पूवी घाट की एक पवधत चोटी के नाम पर रखा गया है।
• इसकी लोंबाई 149 मीटर, चौडाई 17.8 मीटर, प्रक्षेपि भार 3450 टन और असर्कतम गसत 28 समुद्री मील तक
पहुोंचने में सक्षम है।

6. नसतंबर 2023 में िॉन्च निए गए आनदत्य-L1 (िैग्रेंि पॉइं ट 1)िे संबंध में निम्ननिखित में से िौि सा/से नबंदु
"सही" है/हैं?
A)भारतीय अंतररक्ष अिुसंधाि संगठि (ISRO) िे भारत िा पहिा िेधशािा-श्रेणी िा अंतररक्ष-आधाररत सौर
नमशि, आनदत्य-L1 (िैग्रेंि पॉइं ट 1)िॉन्च निया, निसे श्रीहररिोटा, आं ध्र प्रदे श (AP) से PSLV-C57 (ध्रुिीय
उपग्रह प्रक्षेपण याि) रॉिेट द्वारा िॉन्च निया गया।
B)PSLV-C57/आनदत्य-L1 नमशि िा उद्दे श्य अगिे 5 िषों में सूया, नििटतम तारे और सौर मंिि िी सबसे
बडी िस्तु िा अध्ययि िरिा है।
C)अंतररक्ष याि िो सूया-पृथ्वी प्रणािी िे िैग्रेंनियि नबंदु 1 (L1)िे आसपास हेिो िक्षा में तैिात िरिे िी
योििा है , िो पृथ्वी से िगभग 1.5 नमनियि नििोमीटर (km) दू र है।
1)केवल A
2)केवल A & B
3)केवल B & C
4)केवल A & C
5)सभी A, B & C
उत्तर -5)सभी A, B & C
स्पष्टीिरण:
2 ससतोंबर 2023 को, भारतीय अोंतररक्ष अनुसोंर्ान सोंगठन (ISRO) ने आं ध्र प्रदे श(AP) िे श्रीहररिोटा में दू सरे लॉन्च
पैड (SLP), सतीश र्वन अोंतररक्ष केंद्र - SDSC (पूवध में श्रीहररकोटा रें ज - SHAR) से PSLV-C57 (ध्रुवीय उपग्रह प्रक्षेपि
यान) रॉकेट िारा लॉन्च सकए गए भारत के पहले वेर्शाला-श्रेिी के अोंतररक्ष-आर्ाररत सौर समशन आनदत्य-L1 (लैग्रेंज
पॉइों ट 1)को लॉन्च सकया।
• PSLV-C57, PSLV की 59वीों उडान और PSLV-XL कॉक्कन्क्फ़गरे शन का उपयोग करने वाला 25वाों समशन है।
• PSLV-C57/आसदत्य-L1 समशन का उद्दे श्य अगिे 5 िषों में सूयध, सनकटतम तारे और सौर मोंडल की सबसे बडी
वस्तु का अध्ययन करना है।
• अोंतररक्ष यान को सूयध-पृथ्वी प्रिाली के िैग्रेंनियि नबंदु 1 (L1)के आसपास हेिो िक्षा में तैनात करने की योजना
है, जो पृथ्वी से लगभग 1.5 समसलयन सकलोमीटर (km) दू र है।
• आसदत्य-L1 एक टर ाोंस-लैग्रेंसजयन1 सक्कम्मलन पैंतरे बाज़ी से गुजरे गा और L1 के सलए अपना 110 सदवसीय प्रक्षेप
पि शुरू करे गा।
• समशन सनदे शक: तसमलनाडु के तेनकासी की ISRO मसहला वैज्ञासनक निगार शािी, आसदत्य-L1 समशन की
समशन सनदे शक हैं।

Report Errors in the PDF - ebooks@affairscloud.com Copyright 2014-2023 @ AffairsCloud.com 151


• अोंतररक्ष यान सवद् युत चुम्बकीय और कि सडटे ररोों का उपयोग करके प्रकाशमों डल, क्रोमोस्फीयर और सूयध की
सबसे बाहरी परतोों (कोरोना) का सनरीक्षि करने के सलए सात पेलोड ले जाता है।
• VELC, भारतीय खगोल भौसतकी सोंथिान (IIA), बेंगलुरु, कनाधटक में सवकससत सकया गया है , जो सौर कोरोना और
कोरोनल मास इजेक्शन की गसतशीलता का अध्ययन करे गा।
• SUIT, इों टर-यूसनवससधटी सेंटर फॉर एस्ट्र ोनॉमी एों ड एस्ट्र ोसफसजक्स (IUCAA), पुिे, महाराष्ट्र में सवकससत सकया
गया है, जो UV के सनकट सौर प्रकाशमोंडल और क्रोमोस्फीयर की छसव बनाएगा और UV के सनकट सौर
सवसकरि सभन्नता को भी मापेगा।

7. अगस्त 2023 में, SpaceX िे संयुक्त राज्य अमेररिा (USA) में फ्लोररिा से ___________ रॉिेट िे माध्यम से
अपिे 22 स्ट्ारनिंि इं टरिेट उपग्रहों िो िो अथा ऑनबाट (LEO) में िॉन्च निया।
1)फाल्कन 1e
2)ग्रासहॉपर
3)फाल्कन 9
4)स्ट्ारसशप
5)फाल्कन 5
उत्तर-3)फाल्कि 9
स्पष्टीिरण:
31 अगस्त 2023 को, स्पेस एक्सप्लोरे शन टे क्नोलॉजीज कॉपोरे शन (SpaceX) ने अपने 22 स्ट्ारसलोंक इों टरनेट उपग्रहोों
को लो अिध ऑसबधट (LEO) में लॉन्च सकया।
i.स्ट्ारसलोंक अोंतररक्ष यान को ले जाने वाले एक फाल्कि 9 रॉिेट ने सोंयुक्त राज्य अमेररका (USA) में िोररडा के
केप कैनावेरल स्पेस फोसध स्ट्े शन में स्पेस लॉन्च कॉम्प्लेक्स 40 (SLC-40) से उडान भरी।
ii.उडान भरने के बाद, फाल्कन 9 का पहला चरि पृथ्वी पर वापस आया, जो SpaceX डर ोन जहाज "ए शॉटध फॉल ऑफ
ग्रेसवटास" पर पहुोंच गया, जो अटलाोंसटक महासागर में तैनात िा।
iii.यह इस सवशेष बूस्ट्र के सलए 7 वाों लॉन्च और लैंसडों ग िा। इसने पहले क्रू -5, ग्लोबल पोसजशसनोंग ससस्ट्म III (GPS
III) अोंतररक्ष वाहन 06, इनमारसैट I6-F2, वासिक्कज्यक पुन: आपूसतध सेवा (CRS) -28, इों टेलसैट G-37 और अब दो
स्ट्ारसलोंक समशन लॉन्च सकए।

8. उस िंपिी िा िाम बताइए निसिे हाि ही में (नसतंबर'23 में) भारत िे पहिे आनटा नफनशयि इं टेनििेंस
(AI) सक्षम िाउं टर िर ोि 'इं द्रिाि' िो िॉन्च िरिे िे निए भारत इिेररॉनिक्स निनमटे ि (BEL) िे साथ
साझेदारी िी है।
1)सवप्रो PARI रोबोसटक्स
2)DiFACTO रोबोसटक्स
3)कुका रोबोसटक्स
4)ग्रेन रोबोसटक्स
5)इचासॉफ्ट रोबोसटक्स
उत्तर-4)ग्रेि रोबोनटक्स

Report Errors in the PDF - ebooks@affairscloud.com Copyright 2014-2023 @ AffairsCloud.com 152


स्पष्टीिरण:
हैदराबाद (तेलोंगाना) ग्रेिे रोबोनटक्स ने भारत इलेररॉसनक्स सलसमटे ड (BEL) के साि साझेदारी में 'इों द्रजाल' लॉन्च
सकया, जो भारत का पहला और एकमात्र काउों टर-डर ोन ससस्ट्म है जो डर ोन खतरोों का मुकाबला करने के सलए
आसटध सफसशयल इों टेसलजेंस (AI) और मशीन लसनिंग (ML) का उपयोग करता है।
i.डर ोन का अनावरि लेक्कफ्टनेंट जनरल गुरमीत ससोंह, जो उत्तराखोंड के राज्यपाल हैं , की उपक्कथिसत में सकया गया।
ii.इों द्रजाल डर ोन AI-सोंचासलत प्लेटफॉमध ग्रेनेOS पर काम करता है और यह दु सनया का एकमात्र स्वायत्त सवस्तृत क्षेत्र,
प्रसत-मानवरसहत सवमान प्रिाली (C-UAS) है जो सभी वगीकरिोों और सभी स्तरोों के स्वायत्त डर ोनोों से बचाव कर सकता
है।

9. निस िंपिी िे हाि ही में (नसतंबर'23 में) इिेनिटी िर ोि टर े निंग प्रोग्राम िॉन्च िरिे िे निए भारत िर ोि
एसोनसएशि और िेशिि िर ोि पायिट एसोनसएशि िे साथ साझेदारी िी है ?
1)गरुड एयरोस्पेस प्राइवेट सलसमटे ड
2)पारस सडफेंस एों ड स्पेस टे क्नोलॉजीज सलसमटे ड
3)र्ाक्षा अनमैन्ड ससस्ट्म्स प्राइवेट सलसमटे ड
4)िानोस टे क्नोलॉजीज प्राइवेट सलसमटे ड
5)टे कईगल प्राइवेट सलसमटे ड
उत्तर-1)गरुड एयरोस्पेस प्राइिेट निनमटे ि
स्पष्टीिरण:
गरुड एयरोस्पेस प्राइिेट निनमटे ि ने भारत िर ोि एसोनसएशि और िेशिि िर ोि पायिट एसोनसएशि के साि
चेन्नई, तसमलनाडु में इक्वेसलटी डर ोन प्रसशक्षि कायधक्रम शुरू सकया।
• 10 सदनोों का कायधक्रम सवशेष रूप से सवकलाों ग व्यक्कक्तयोों के सलए सडज़ाइन सकया गया है तासक उन्ें (सनः शुल्क)
प्रसशक्षि, सशक्षा और कौशल सवकास प्रदान सकया जा सके।
• यह कायधक्रम गरुड एयरोस्पेस की सवसवर्ता, समानता और समावेशन (DEandI) पहल को मजबूत करता है
• असग्नश्वर जयप्रकाश गरुड एयरोस्पेस प्राइवेट सलसमटे ड के सोंथिापक और मुख्य कायधकारी असर्कारी (CEO) हैं ,
सजसे 2015 में थिासपत सकया गया िा।

10. नमत्सुनबशी हेिी इं िस्ट्र ीि निनमटे ि (MHI) िे साथ निस स्पेस एिेंसी िे हाि ही में (नसतंबर'23 में) X-रे
िगोिीय उपग्रह X-रे इमेनिंग और स्पेररोस्कोपी नमशि (XRISM) और िूिर िैंिर स्माटा िैंिर फॉर
इन्वेखस्ट्गेशि मूि (SLIM) िो िे िािे िािे H-IIA िॉन्च व्हीिि िंबर 47 (H-IIA F47) िॉन्च निया है ?
1)जापान एयरोस्पेस एक्सप्लोरे शन एजेंसी
2)इों सडयन स्पेस ररसचध आगेनाईजेशन
3)स्पेस एक्सप्लोरे शन टे क्नोलॉजीज कॉपोरे शन
4)कैनेसडयन स्पेस एजेंसी
5)रोस्कोस्मोस
उत्तर-1)िापाि एयरोस्पेस एक्सप्लोरे शि एिेंसी
स्पष्टीिरण:
िापाि एयरोस्पेस एक्सप्लोरे शि एिेंसी (JAXA) ने समत्सुसबशी हेवी इों डस्ट्र ीज सलसमटे ड (MHI) के साि समलकर H-
IIA लॉन्च व्हीकल नोंबर 47 (H-IIA F47) लॉन्च सकया, जो X-रे खगोलीय उपग्रह X-रे इमेसजोंग और स्पेररोस्कोपी
समशन (XRISM) और लूनर लैंडर स्माटध लैंडर फॉर इन्वेक्कस्ट्गेशन मून (SLIM) को ले गया।

Report Errors in the PDF - ebooks@affairscloud.com Copyright 2014-2023 @ AffairsCloud.com 153


• रॉकेट को जापान के तनेगासशमा स्पेस केंद्र से लॉन्च सकया गया िा।
• इसके साि, जापान चोंद्रमा पर सॉफ्ट लैंसडों ग करने वाला पाोंचवाों दे श बन गया है ; अगस्त 2023 में चोंद्रयान 3
समशन के प्रक्षेपि के साि चौिा दे श भारत िा।
i.X-रे इमेसजोंग और स्पेररोस्कोपी समशन (XRISM) उपग्रह, JAXA, नेशनल एरोनॉसटक्स एों ड स्पेस एडसमसनस्ट्र े शन
(NASA) और यूरोपीय स्पेस एजेंसी (ESA) की एक सोंयुक्त पररयोजना है।
• उपग्रह का लक्ष् ब्रह्ाोंड में बहने वाली प्लाज़्मा हवाओों का सनरीक्षि करना है जो ससतारोों और आकाशगोंगाओों
के सवकास को समझने में मदद करती है।
• कहा जाता है सक XRISM ब्रह्ाोंड के सनमाधि से सोंबोंसर्त रहस्योों को सुलझाता है।
ii.SLIM दो उद्दे श्योों वाला छोटा स्पेस यान (2.4 मीटर) है
• उच्च पररशुद्धता लैंसडों ग तकनीक का प्रदशधन (सपन-पॉइों ट लैंसडों ग तकनीक और बार्ा पहचान तकनीक का
उपयोग करके) है।
• असर्क बार लूनर और ग्रहीय अन्वेषि समशनोों की अनुमसत दे ने के सलए एक हल्के लूनर और ग्रहीय जाोंच
प्रिाली का एहसास है।

11. उस रक्षा क्षेत्र िा िाम बताइए निसिे हाि ही में (नसतंबर'23 में) नमनिटर ी िॉम्बैट पैराशूट नसस्ट्म िी
प्रभाििाररता और फुिप्रूफ िायाप्रणािी िा परीक्षण निया है।
1)भारतीय वायु सेना
2)भारतीय नौसेना
3)राष्ट्रीय सुरक्षा गाडध
4)असम राइफल्स
5)सीमा सुरक्षा बल
उत्तर- 1)भारतीय िायु सेिा
स्पष्टीिरण:
भारतीय वायु सेना (IAF) ने नमनिटर ी िॉम्बैट पैराशूट नसस्ट्म की प्रभावकाररता और फुलप्रूफ कायधप्रिाली का
परीक्षि सकया, सजसे एररयल सडलीवरी ररसचध एों ड डे वलपमेंट एस्ट्े क्किशमेंट (ADRDE) िारा स्वदे शी रूप से सडजाइन
और सवकससत सकया गया है ।
• ADRDE रक्षा अनुसोंर्ान और सवकास सोंगठन (DRDO) की एक अनुसोंर्ान और सवकास (R&D) प्रयोगशाला
है।
• परीक्षण: भारतीय वायुसेना के एक टे स्ट् जम्पर ने जानबूझकर मुख्य पैराशूट को अलग कर सदया और
बैरोमेसटर क ऑटोमैसटक एक्करवेशन सडवाइस (AAD) से पहले ही ररजवध हैंडल को ससक्रय सकए सबना, ररजवध
पैराशूट को ररजवध स्ट्े सटक लाइन (RSL) के माध्यम से अपने आप खुलने सदया।

12. सामररक परमाणु हमिा सबमरीन का नाम बताइए लिसका हाि ही में (लसतंबर’23 में) उत्तर कोररया द्वारा
अनावरण लकया गया है।
1) िी शुन
2) हीरो श्चकम कुन ओक
3)दाई हांग िै न
4) कांग नाम 1
5) कोररयाई सबमरीन-III

Report Errors in the PDF - ebooks@affairscloud.com Copyright 2014-2023 @ AffairsCloud.com 154


उत्तर – 2)हीरो लकम कुन ओक
स्पष्टीकरण:
उत्तर कोररया ने अपनी पहिी ऑपरे शनल सामररक परमाणु हमिा सबमरीन, सबमरीन नंबर 841 लॉन्च की,
श्चिसका नाम "हीरो लकम कुन ओक" रखा गया। इसे श्चवशेष रूप से पानी के अंदर से रिनीश्चतक परमािु हश्चियार
लॉन्च करने के श्चलए श्चिज़ाइन श्चकया गया िा।
• लॉन्च समारोह में उत्तर कोररया के पूवी तट पर स्थित श्चसनफो श्चशपयािग में उत्तर कोररया के सवोच्च नेता श्चकम
िोंग उन ने भाग श्चलया।
• सबमरीन को उत्तर कोररया की थिापना की 75 वीं वषगगांठ (9 श्चसतंबर, 1948) से पहले लॉन्च श्चकया गया है।
उत्तर कोररया हर साल 9 श्चसतंबर को गणतंत्र की स्थापना के लदन के रूप में मनाता है।
• सबमरीन का नाम उत्तर कोररयाई नौसैश्चनक प्रमुख हीरो लकम कुन ओक के नाम पर रखा गया िा, श्चिन्होंने
1950 में कोररयाई युि के दौरान USS बाल्ट्ीमोर को िु बाने वाले ऑपरे शन का नेतृत्व श्चकया िा।

13. लकस कंपनी ने हाि ही में (लसतंबर’23 में) संयुक्त राज्य अमेररका (USA) की सरकार के साथ मास्ट्र लशप
ररपेयर एग्रीमेंट (MSRA) पर हस्ताक्षर लकए हैं?
1) गािग न रीच श्चशपश्चबर्ल्सग एं ि इं िीश्चनयसग श्चलश्चमटे ि
2) श्चशश्चपंग कॉपोरे शन ऑफ इं श्चिया
3) मझगांव िॉक श्चशपश्चबर्ल्सग श्चलश्चमटे ि
4) कोचीन श्चशपयािग श्चलश्चमटे ि
5) गोवा श्चशपयािग श्चलश्चमटे ि
उत्तर-3)मझगांव िॉक लशपलबल्डसा लिलमटे ि
स्पष्टीकरण:
रक्षा मंत्रालय (MoD) के तहत एक सावगिश्चनक क्षेत्र के उपक्रम (PSU) मझगांव िॉक लशपलबल्डसा लिलमटे ि (MDL)
ने संयुक्त राज्य अमेररका (US) की सरकार के साि एक मास्ट्र श्चशप ररपेयर एग्रीमेंट (MSRA) पर हस्ताक्षर श्चकए हैं,
श्चिसका प्रश्चतश्चनश्चर्त्व NAVSUP फ्लीट लॉश्चिस्स्ट्क्स सेंटर (FLC) योकोसुका, िापान द्वारा श्चकया िाता है ।
• MSRA US नौसेना और श्चनिी िहाि श्चनमागि ठे केदारों के बीच US नौसैश्चनक िहािों की ररपेयर के श्चलए
श्चशपयािों को पूवग-अनुमोदन दे ने के श्चलए एक कानूनी रूप से गैर-बाध्यकारी व्यवथिा है।
• इस समझौते के तहत, MDL US नौसेना के िहािों (USNS) के श्चलए आवश्यक यात्रा ररपेयर और रखरखाव
सेवाएं प्रदान करने के श्चलए बना है और यह भारत की समुद्री सुरक्षा को मिबूत करने का प्रतीक है।
• मझगांव िॉक श्चशपश्चबर्ल्सग श्चलश्चमटे ि, श्चिसे पहले मझगां व िॉक श्चलश्चमटे ि के नाम से िाना िाता िा, को 26
फरवरी, 1934 को मुंबई (महाराष्ट्र) में एक प्राइवेट श्चलश्चमटे ि कंपनी के रूप में शाश्चमल श्चकया गया िा।

14. निस संस्थाि िे हाि ही में (नसतंबर'23 में) राष्टरीय प्रािृनति संसाधि प्रबंधि प्रणािी (NNRMS) िायाक्रम
िे ढांचे िे भीतर िनिि संसाधिों और आपदा प्रबंधि िे क्षेत्रों में क्षमता निमााण िे निए भारतीय अंतररक्ष
अिुसंधाि संगठि (ISRO) िे साथ एि MoU पर हस्ताक्षर निए हैं ?
1)भारतीय भूवैज्ञासनक सवेक्षि प्रसशक्षि सोंथिान
2)भारतीय सवज्ञान सोंथिान, बेंगलुरु
3)केंद्रीय खनन एवों ईोंर्न अनुसोंर्ान सोंथिान
4)भारतीय भूवैज्ञासनक एवों र्ातुकमध सोंथिान
5)भारतीय सवज्ञान सशक्षा एवों अनुसोंर्ान सोंथिान कोलकाता

Report Errors in the PDF - ebooks@affairscloud.com Copyright 2014-2023 @ AffairsCloud.com 155


उत्तर- 1)भारतीय भूिैज्ञानिि सिेक्षण प्रनशक्षण संस्थाि
स्पष्टीिरण:
हैदराबाद (तेलोंगाना) में भारतीय भूवैज्ञासनक सवेक्षि प्रसशक्षि सोंथिान (GSITI) ने राष्ट्रीय प्राकृसतक सोंसार्न प्रबों र्न
प्रिाली(NNRMS) िायाक्रम के सहस्से के रूप में बेंगलुरु (कनाधटक) में भारतीय अोंतररक्ष अनुसोंर्ान सोंगठन (ISRO)
के साि 5 साल के समझौता ज्ञापन (MoU) पर हस्ताक्षर सकए हैं।
i.पररयोजना का प्रािसमक उद्दे श्य खसनज अन्वेषि में सडसजटल इमेज प्रोसेससोंग और भौगोसलक सूचना प्रिाली (GIS) के
अनुप्रयोग, खसनज अन्वेषि में उन्नत ररमोट सेंससोंग तकनीक और आपदा प्रबोंर्न में भू -सूचना सवज्ञान के अनुप्रयोगोों में
कुशल कायधबल (मानव सोंसार्न) के सनमाध ि के माध्यम से क्षमता सनमाधि है।
ii.GSITI और ISRO के बीच सहयोगात्मक पररयोजना का दायरा NNRMS कायधक्रम के ढाोंचे के भीतर खसनज सोंसार्नोों
और आपदा प्रबोंर्न के क्षेत्रोों में क्षमता सनमाधि के सलए प्रसशक्षि की एक श्रृोंखला आयोसजत करना है।

15. उस उपिरण िा िाम बताइए निसे हाि ही में (नसतंबर'23 में) भारतीय अंतररक्ष अिुसंधाि संगठि
(ISRO) द्वारा समुद्री अनभयािों िे दौराि मछु आरों िी सुरक्षा बढ़ािे िे निए नििनसत निया गया है।
1)सागर
2)नाभसमत्र
3)OFish
4)mKRISHI
5)फेमा
उत्तर- 2)िाभनमत्र
स्पष्टीिरण:
भारतीय अोंतररक्ष अनुसोंर्ान सोंगठन (ISRO) ने 'िभनमत्र' नामक अपने असभनव उपकरि का सफलतापूवधक परीक्षि
सकया है , सजसे गुजरात के अहमदाबाद में ISRO-अोंतररक्ष अनुप्रयोग केंद्र में सवकससत सकया गया िा। इस उपकरि का
उद्दे श्य समुद्री असभयानोों के दौरान मछु आरोों की सुरक्षा बढाना है।
i.परीक्षि केरल के कोल्लम के िी ंदिारा में हुआ, जहाों उपकरि को मछली पकडने वाले जहाज पर थिासपत सकया
गया और वैज्ञासनकोों और मत्स्य पालन सवभाग के असर्काररयोों की उपक्कथिसत में परीक्षि सकया गया।
ii.नभसमत्र एक उपग्रह-आर्ाररत सोंचार प्रिाली के आर्ार पर सोंचासलत होता है जो समुद्र और तट के बीच के अोंतर को
पाटते हुए दो-तरफा सोंचार की सुसवर्ा प्रदान करता है।

16. उस संगठि िा िाम बताइए निसिे हाि ही में (नसतंबर'23 में) आत्महत्या िी रोिथाम िे प्रयासों िो
मिबूत िरिे िे निए निज़ाइि निए गए 2 संसाधि [नप्रिेंनटं ग सुसाइि िा चौथा संस्करण: मीनिया पेशेिरों िे
निए एि संसाधि (2023 अपिे ट) और आत्महत्या और आत्महत्या िे प्रयासों िे िीनक्रनमििाइिेशि िे
स्वास्थ्य पहिुओ ं पर िीनत संनक्षप्त] िॉन्च निया है।
1)सोंयुक्त राष्ट्र बाल कोष
2)सवश्व स्वास्थ्य सोंगठन
3)सोंयुक्त राष्ट्र मानव सनपटान कायधक्रम
4)सफसलस्तीन शरिासिधयोों के सलए सोंयुक्त राष्ट्र राहत और कायध एजेंसी
5)शरिासिधयोों के सलए सोंयुक्त राष्ट्र उच्चायुक्त
उत्तर- 2)निश्व स्वास्थ्य संगठि

Report Errors in the PDF - ebooks@affairscloud.com Copyright 2014-2023 @ AffairsCloud.com 156


स्पष्टीिरण:
12 ससतोंबर 2023 को, सवश्व स्वास्थ्य सोंगठन (WHO) ने आत्महत्या रोकिाम प्रयासोों को मजबूत करने के सलए सडज़ाइन
सकए गए 2 सोंसार्न लॉन्च सकए। 2 सोंसार्न हैं
• आत्महत्या की रोकिाम का चौिा सोंस्करि: मीसडया पेशेवरोों के सलए एक सोंसार्न (2023 अद्यतन)।
• आत्महत्या और आत्महत्या के प्रयासोों को अपरार् की श्रेिी से बाहर करने के स्वास्थ्य पहलुओों पर नीसत
सोंसक्षप्त।
i.सोंसार्न दो क्षेत्रोों पर महत्वपूिध मागधदशधन प्रदान करते हैं जो आत्महत्या की रोकिाम के प्रयासोों के सलए महत्वपूिध हैं ,
अिाधत् मीसडया िारा आत्महत्या की सजम्मेदार ररपोसटिं ग और आत्महत्या और आत्महत्या के प्रयासोों को अपरार् से मुक्त
करना।
ii.WHO के अनुसार, आत्महत्या एक महत्वपूिध सावधजसनक स्वास्थ्य मुद्दा है और सवश्व स्तर पर प्रत्येक 100 मौतोों में से 1
आत्महत्या से होती है , जो हर साल 700,000 से असर्क लोगोों को प्रभासवत करती है । यह 15-29 वषध के लोगोों में मृत्यु
का चौिा प्रमुख कारि है।
iii.WHO का सप्रवेंसटों ग सुसाइड का चौिा सोंस्करि: मीसडया पेशेवरोों के सलए एक सोंसार्न (2023 अपडे ट) इों टरनेशनल
एसोससएशन फॉर सुसाइड सप्रवेंशन (IASP) के सहयोग से तैयार सकया गया है।
• तीसरा सोंस्करि 2017 में लॉन्च सकया गया िा। पहला सोंस्करि 2000 में और दू सरा सोंस्करि 2008 में जारी
सकया गया िा।
• यह सोंसार्न आत्महत्या की मीसडया ररपोसटिं ग के प्रभाव पर वतधमान साक्ष्ोों का साराों श प्रस्तुत करता है और
मीसडया पेशेवरोों को सजम्मेदारीपूवधक आत्महत्या की ररपोटध करने के बारे में व्यावहाररक मागधदशधन प्रदान करता
है।

17. निस िंपिी िे हाि ही में (नसतंबर'23 में) िाउि िंप्यूनटं ग िे माध्यम से अंतररक्ष-तििीिी ििाचारों िा
समथाि िरिे िे निए भारतीय अंतररक्ष अिु संधाि संगठि (ISRO) और भारतीय राष्टरीय अंतररक्ष संिधाि और
प्रानधिरण िेंद्र (IN-SPACe) िे साथ एि MoU पर हस्ताक्षर निए हैं ?
1)अमेज़न वेब ससवधसेज इों सडया
2)गूगल क्लाउड प्लेटफामध
3)ओरे कल क्लाउड इों फ्रास्ट्र क्चर
4)IBM क्लाउड
5)माइक्रोसॉफ्ट एज़्योर इों सडया
उत्तर- 1)अमेज़़ॅि िेब सनिासेि इं निया
स्पष्टीिरण:
अमेज़़ॅन वेब ससवधसेज (AWS) इं निया प्राइिेट निनमटे ि ने क्लाउड कोंप्यूसटों ग के माध्यम से अोंतररक्ष-तकनीकी
नवाचारोों का समिधन करने के सलए भारतीय अोंतररक्ष अनुसोंर्ान सोंगठन (ISRO) और भारतीय राष्ट्रीय अों तररक्ष सोंवर्धन
और प्रासर्करि केंद्र (IN-SPACe) के साि एक समझौता ज्ञापन (MoU) पर हस्ताक्षर सकए हैं।
i.क्लाउड कोंप्यूसटों ग कम लागत पर सािधक पररिाम प्राप्त करने के सलए बडी सोंख्या में कच्चे अोंतररक्ष डे टा के साि-साि
आसटध सफसशयल इों टेसलजेंस (AI), मशीन लसनिंग (ML) और एनासलसटक्स वकधलोड के सनष्पादन की त्वररत हैंडसलोंग प्रदान
करता है।

Report Errors in the PDF - ebooks@affairscloud.com Copyright 2014-2023 @ AffairsCloud.com 157


18. नसतंबर 2023 में, भारतीय हिाईअड्डा प्रानधिरण (AAI) िे िमीिी िेनिगेशि और िैंनिं ग सहायता िे
अंशांिि िे निए अपिी उडाि निरीक्षण इिाई (FIU) िे बेडे में उन्नत अत्याधुनिि उडाि निरीक्षण प्रणानियों
िे साथ दो िए ______________________ निमाि शानमि निए हैं।
1)CASA C-212
2)डोसनधयर-228
3)इलुसशन Il-76
4)B-360
5)Mil Mi-24
उत्तर-4)B-360
स्पष्टीिरण:
भारतीय हवाईअड्डा प्रासर्करि (AAI) ने जमीनी नेसवगेशन और लैंसडों ग सहायता के अोंशाोंकन के सलए अपनी उडान
सनरीक्षि इकाई (FIU) िे बेडे में उन्नत अत्यार्ुसनक उडाि निरीक्षण प्रणानियों के साि दो नए B-360 निमाि
शासमल सकए हैं ।
i.अोंतराधष्ट्रीय नागररक उड्डयन सोंगठन (ICAO) िारा सनसदध ष्ट् सवोत्तम अोंतराध ष्ट्रीय मानकोों को प्राप्त करने के सलए उडान
सनरीक्षि महत्वपूिध है।
ii.इन सवमानोों का उपयोग श्रेिी I, II और III इों स्ट्ूमेंट लैं सडों ग ससस्ट्म, डॉपलर वेरी हाई फ्रीक्वेंसी ओमनी रें ज (DVORs),
सडस्ट्ें स मेजररों ग इक्कक्वपमेंट (DME), नॉन-डायरे क्शनल बीकन (NDB), रडार, ग्राउों ड बेथड ऑग्मेंटेशन ससस्ट्म GBAS),
सप्रससजन एप्रोच पाि इों सडकेटर (PAPI), टै क्करकल एयर नेसवगेशन ससस्ट्म (TACAN) को कैसलब्रेट करने में सकया
जाएगा।

19. 15 नसतंबर 2023 िो, SpaceX िे संयुक्त राज्य अमेररिा (USA) में फ्लोररिा से ___________ रॉिेट िे
माध्यम से अपिे 22 स्ट्ारनिंि इं टरिेट उपग्रहों िो िो अथा ऑनबाट (LEO) में िॉन्च निया।
1)फाल्कन 1e
2)ग्रास हॉपर
3)फाल्कन 9
4)स्ट्ारसशप
5)फाल्कन 5
उत्तर-3)फाल्कि 9
स्पष्टीिरण:
15 ससतोंबर 2023 को, स्पेस एक्सप्लोरे शन टे क्नोलॉजीज कॉपोरे शन (SpaceX) ने 22 स्ट्ारनिंि इं टरिेट उपग्रहों को
लो अिध ऑसबधट (LEO) में लॉन्च सकया। फाल्कि 9 रॉिेट ने सोंयुक्त राज्य अमेररका (USA) के िोररडा में केप
कैनावेरल स्पेस फोसध स्ट्े शन में स्पेस लॉन्च कॉम्प्लेक्स 40 (SLC -40) से स्ट्ारसलोंक अोंतररक्ष यान लॉन्च सकया।
• इस समशन का समिधन करने वाले पहले चरि के बूस्ट्र की यह पाोंचवीों उडान िी, सजसने पहले क्रू -6, SES
O3b mPOWER और दो स्ट्ारसलोंक समशन लॉन्च सकए िे।
• यह SpaceX डर ोनसशप 'जस्ट् रीड द इों स्ट्रक्शोंस' पर उडान भरने के लगभग 8.5 समनट बाद पृथ्वी पर लौटा, जो
िोररडा तट से दू र अटलाोंसटक महासागर में तैनात िा।
• यह लॉन्च SpaceX का 2023 का 65वाों समशन और िोररडा के स्पेस कोस्ट् लॉन्च स्ट्े शन से 49वाों समशन है।

Report Errors in the PDF - ebooks@affairscloud.com Copyright 2014-2023 @ AffairsCloud.com 158


20. लसतंबर 2023 में, भारतीय वायु सेना (IAF) ने एक मािवाहक लवमान से स्वदे शी रूप से लवकलसत 'टाइप
V हेवी िर ॉप लसस्ट्म', मल्ट्ी-स्ट्े ि पैराशूट का सफितापूवाक परीक्षण लकया।
पैराशूट का लनमााण लकस कंपनी ने लकया है?
1)अश्चिकुल कॉसमॉस प्राइवेट श्चलश्चमटे ि
2)बेलाश्चटरक्स एयरोस्पेस प्राइवेट श्चलश्चमटे ि
3)स्काईरूट एयरोस्पेस प्राइवेट श्चलश्चमटे ि
4)ध्रुव स्पेस प्राइवेट श्चलश्चमटे ि
5)एयरबोश्चनगक्स श्चिफेंस एं ि स्पेस प्राइवेट श्चलश्चमटे ि
उत्तर-5)एयरबोलनाक्स लिफेंस एं ि स्पेस प्राइवेट लिलमटे ि
स्पष्टीकरण:
भारतीय वायु सेना (IAF) ने मालवाहक श्चवमान से स्वदे शी रूप से श्चवकश्चसत 'टाइप V हेवी िर ॉप लसस्ट्म' का
सफलतापूवगक परीक्षि श्चकया है िो हश्चियारों, गोला-बारूद और अन्य उपकरिों को दु गगम थिानों/युिक्षेत्रों में पैरािर ॉप
करने में सक्षम है।
• इस श्चसस्ट्म का श्चिज़ाइन एररयल श्चिलीवरी ररसचग एं ि िे वलपमेंट एस्ट्े स्िशमेंट (ADRDE), आगरा, उत्तर प्रदे श
(UP) द्वारा श्चवकश्चसत श्चकया गया है।
• टाइप V हेवी िर ॉप लसस्ट्म, एक पैराशूट है, श्चिसे 'मेक इन इं श्चिया' पहल के तहत श्चवकश्चसत श्चकया गया है।
i.टाइप V हैवी िर ॉप लसस्ट्म के बारे में:
• पैराशूट का श्चनमागि एयरबोलनाक्स लिफेंस एं ि स्पेस प्राइवेट लिलमटे ि द्वारा श्चकया गया है।
• यह एक मल्ट्ी-स्ट्े ि पैराशूट प्रिाली है श्चिसमें आठ मुख्य कैनोपी, तीन एक्सटर ै रर पैराशूट और एक िर ग
पैराशूट शाश्चमल हैं ।
• पैराशूट का विन 20 टन तक है और इसमें एक िेटफॉमग है िो एक श्चवशेष एल्यूमीश्चनयम-आर्ाररत र्ातु से
बना है।

21. लसतंबर 2023 में, कृलष और लकसान कल्याण सलचव, मनोि आहूिा ने ____________________ के लिए
सामान्य फसि अनुमान सवेक्षण (GCES) के लिए मोबाइि एस्किकेशन और वेब पोटा ि िॉन्च लकया।
1)पौर्ों/फसलों की बीमाररयों का पता लगाना और उनके श्चलए उपचार खोिना
2)पूरे भारत में कृश्चष प्रिाओं को बदलना
3)राज्य स्तरीय फसलों के क्षेत्रफल, उत्पादन और उत्पादकता के बारे में िानकारी प्राप्त करें
4)पूरे भारत में फसल की उपलिता के बारे में िानें
5)श्चवश्चभन्न राज्यों से नवीनतम भारतीय फसल वस्तुओं की मंिी कीमतों की िााँ च करें
उत्तर- 2)पूरे भारत में कृलष प्रथाओं को बदिना
स्पष्टीकरण:
21 श्चसतंबर, 2023 को, मनोि आहूिा, सश्चचव, कृश्चष और श्चकसान कल्याि मंत्रालय (MoAFW) के तहत कृश्चष और
श्चकसान कल्याि श्चवभाग (DA&FW) ने पूरे भारत में कृलष प्रथाओं को बदिने के श्चलए सामान्य फसल अनुमान
सवेक्षि (GCES) के श्चलए मोबाइल एस्िकेशन और वेब पोटग ल लॉन्च श्चकया।
i.यह श्चवकास, श्चवकास के दौरान सरकारी कायों की पहुं च, दायरे और पररिाम को ब़िाने के श्चलए श्चकया गया है ।
ii.ऐप और पोटग ल के संचालन की योिना 12 राज्यों में बनाई गई है और इसे अरू बर 2023 के तीसरे सप्ताह तक लॉन्च
श्चकया िाएगा। सभी राज्यों से साख सुश्चनश्चित करने के श्चलए GCES पोटग ल और एस्िकेशन को अपनाने का आग्रह श्चकया
िाता है।

Report Errors in the PDF - ebooks@affairscloud.com Copyright 2014-2023 @ AffairsCloud.com 159


iii.GCES का पोटग ल और मोबाइल एस्िकेशन DA&FW द्वारा श्चवकश्चसत श्चकया गया है।

22. उस व्यखक्तत्व िा िाम बताइए निसिे हाि ही में (नसतंबर'23 में) िोगों िो सरिारी िमाचाररयों िे
खििाफ भ्रष्टाचार िी नशिायतें दिा िरिे में सक्षम बिािे िे निए अपिी तरह िा पहिा सतिाता नशिायत
सूचिा प्रबंधि प्रणािी (VCIMS) पोटा ि िॉन्च निया है।
1)सवनय कुमार सक्सेना (सदल्ली के उपराज्यपाल)
2)रमेश बैस (महाराष्ट्र के राज्यपाल)
3)िावरचोंद गेहलोत (कनाधटक के राज्यपाल)
4)एडसमरल D K जोशी (अोंडमान और सनकोबार िीप के लेक्कफ्टनेंट गवनधर)
5)B.D. समश्रा (लद्दाख के उपराज्यपाल)
उत्तर-1)नििय िुमार सक्सेिा (नदल्ली िे उपराज्यपाि)
स्पष्टीिरण:
सदल्ली के उपराज्यपाल, नििय िुमार (VK) सक्सेिा ने अपनी तरह का पहला सतकधता सशकायत सूचना प्रबोंर्न
प्रिाली (VCIMS) पोटा ि लॉन्च सकया, जो लोगोों को कहीों से भी सकसी भी समय सरकारी कमधचाररयोों के क्कखलाफ
भ्रष्ट्ाचार की सशकायत दजध करने में सक्षम बनाता है ।
• सशकायतें सतकधता/भ्रष्ट्ाचार/ररश्वत की माों ग से सोंबोंसर्त हो सकती हैं।
• पारदसशधता और गोपनीयता के साि यह पोटध ल सशकायतोों की सनगरानी करे गा और त्वररत कारध वाई शुरू
करे गा।
• इस पोटध ल पर केवल एक बार पोंजीकरि की आवश्यकता है।
i.यह सुसनसित करने के सलए सक पोटध ल पर केवल वास्तसवक सशकायतें ही दजध की जाएों , सशकायतकताधओों के सलए एक
ई-अोंडरटे सकोंग जमा करना असनवायध है तासक लोक सेवकोों को गलत जानकारी दे ने के सलए IPC(भारतीय दों ड सोंसहता)
की र्ारा 182 के तहत उन पर मुकदमा चलाया जा सके।

23. पुरातत्वनिदों िे एि समूह िे हाि ही में (नसतंबर'23 में) निस दे श में 476,000 साि पहिे िी दु निया िी
सबसे पुरािी ििडी िी संरचिा िा प्रमाण िोिा है ?
1)समस्र
2)तोंज़ासनया
3)नामीसबया
4)युगाोंडा
5)जाक्कम्बया
उत्तर-5)िाखम्बया
स्पष्टीिरण:
पुरातत्वनिदों िे एि समूह ने दु निया िी सबसे पुरािी ििडी िी संरचिा, "एक पायदान से जुडे हुए इों टरलॉसकोंग
लॉग की एक जोडी" के साक्ष् की खोज की है , जो 476,000 साि पहिे िाखम्बया में कलम्बो नदी पर अफ्रीका के
दू सरे सबसे ऊोंचे झरने, कलाम्बो फॉल्स के पुराताक्कत्वक थिल पर है ।
i.इस सोंरचना के सनष्कषध 20 ससतोंबर 2023 को 'नेचर' पसत्रका में प्रकासशत हुए िे।
ii.कलम्बो नदी पर कलम्बो झरना, ताोंगासनका झील के सकनारे तोंजासनया के रुकवा क्षेत्र के साि जाक्कम्बया की सीमा पर
235 मीटर ऊोंचा एकल-बूोंद झरना है।

Report Errors in the PDF - ebooks@affairscloud.com Copyright 2014-2023 @ AffairsCloud.com 160


iii. इस झरने को 2009 में सोंयुक्त राष्ट्र शैसक्षक, वैज्ञासनक और साोंस्कृसतक सोंगठन (UNESCO) के सवश्व र्रोहर थिल में
शासमल सकया गया िा।

24. उस दे श िा िाम बताइए निसिे हाि ही में (नसतंबर'23 में) अपिे िए 5िी ं पीढ़ी िे मुख्य युिि टैं ि
(MBT), मिािा मािा 5/'बराि िा अिािरण निया है , निसिा नहिू में अिुिाद "िाइटनिंग" होता है।
1)सोंयुक्त अरब अमीरात
2)ईरान
3)जापान
4)इज़राइल
5)इों डोनेसशया
उत्तर-4)इज़राइि
स्पष्टीिरण:
इज़राइली रक्षा मोंत्रालय और इज़राइल रक्षा बलोों (IDF) ने अपने नए 5वीों पीढी के मुख्य युद्धक टैं क (MBT), मिािा
मािा 5 का अनावरि सकया, सजसे "बराि" के नाम से भी जाना जाता है , सजसका सहब्रू में अनुवाद "लाइटसनोंग" होता है ।
मकधवा 5 मकधवा IV का पररष्कृत सोंस्करि है ।
i.टैं क का सवकास पाोंच साल तक चला।
ii.बराक टैं क को रक्षा मोंत्रालय के बख्तरबोंद वाहन सनदे शालय और IDF के ग्राउों ड फोसेज और आमधडध कोर िारा
सोंयुक्त रूप से सवकससत सकया गया िा।
iii.टैं क के पहले मॉडल 401वीों सब्रगेड की 52वीों बख्तरबोंद बटासलयन को सौोंपे गए िे।

25. नसतंबर 2023 में िमोिोर G रनि द्वारा निस राज्य में भारतीय िौसेिा िा तीसरा नमसाइि सह गोिा
बारूद (MCA) बािा, यािा 77 (LSAM 9) िॉन्च निया गया था?
1)तसमलनाडु
2)केरल
3)आों ध्र प्रदे श
4)महाराष्ट्र
5)पसिम बोंगाल
उत्तर-3)आं ध्र प्रदे श
स्पष्टीिरण:
युद्धपोत उत्पादन अर्ीक्षक (सवशाखापत्तनम, आों ध्र प्रदे श) िमोिोर G रनि ने मेससध SECON इों जीसनयररों ग प्रोजेर्स
प्राइवेट सलसमटे ड के प्रक्षेपि थिल गुट्टेनादे वी, पूवी गोदावरी, आं ध्र प्रदे श में भारतीय नौसेना के तीसरे समसाइल सह
गोला-बारूद (MCA) बािा यािा 77 (LSAM 9) का शुभारों भ सकया।
i.स्वदे शी सनमाधताओों से प्राप्त प्रमुख और सहायक उपकरि/प्रिासलयोों वाला बाजध रक्षा मोंत्रालय की मेक इन इों सडया
पहल के अनुरूप है ।
ii.इस बजरे का सनमाधि भारतीय सशसपोंग रसजस्ट्र (IRS) के वगीकरि सनयमोों के तहत सकया गया है , सजसकी सेवा
अवसर् 30 वषध है।
iii.08 X MCA बाजध के सनमाधि और सवतरि के सलए अनुबोंर् मेससध SECON इों जीसनयररों ग प्रोजेर्स प्राइवेट सलसमटे ड,
सवशाखापत्तनम, एक सूक्ष्म, लघु और मध्यम उद्यम (MSME) सशपयाडध के साि सोंपन्न हुआ, जो भारत सरकार के
"आत्मसनभधर भारत"(सेल्फ -रे लाएों ट इों सडया ) पहल के साि सोंरेक्कखत है।

Report Errors in the PDF - ebooks@affairscloud.com Copyright 2014-2023 @ AffairsCloud.com 161


26. निस िंपिी िे हाि ही में (नसतंबर’23 में) राष्टरीय आपदा प्रबंधि प्रानधिरण (NDMA), राष्टरीय भूिंप
निज्ञाि िेंद्र (NCS) और पृथ्वी निज्ञाि मंत्रािय िे सहयोग से भारत में अपिा एं िर ॉइि भूिंप अिटा नसस्ट्म पेश
निया है।
1)मेटा
2)माइक्रोसॉफ्ट
3)गूगल
4)सेब
5)ज़ोहो
उत्तर-3)गूगि
स्पष्टीिरण:
गूगि ने राष्ट्रीय आपदा प्रबों र्न प्रासर्करि (NDMA) और राष्ट्रीय भूकोंप सवज्ञान केंद्र (NCS) और पृथ्वी सवज्ञान मोंत्रालय
के सहयोग से भारत में अपना एों डर ॉइड भूकोंप अलटध ससस्ट्म पेश सकया है।
i.यह साझेदारी समय पर भूिंप अिटा और सुरक्षा जानकारी प्रदान करने में केंद्र सरकार के प्रयासोों में सहायता
करे गी।
ii.एों डर ॉइड 5 या नए सोंस्करि चलाने वाले सडवाइस वाले एों डर ॉइड उपयोगकताधओों को यह सुसवर्ा प्राप्त होगी।
• अलटध प्राप्त करने के सलए, उपयोगकताधओों के पास Wi-Fiऔर/या सेलुलर डे टा कनेक्करसवटी होनी चासहए, और
एों डर ॉइड भूकोंप अलटध और थिान सेसटों ग्स दोनोों सक्षम होनी चासहए।

27. निस दे श िे हाि ही में (नसतंबर’23 में) "हाई िुि" िामि अपिी पहिी घरे िू निनमात पििु ब्बी िॉन्च िी
है?
1)दसक्षि कोररया
2)ताइवान
3)म्याोंमार
4)उत्तर कोररया
5)जापान
उत्तर- 2)ताइिाि
स्पष्टीिरण:
28 ससतोंबर 2023 को, ताइवान के राष्ट्रपसत साई इं ग-िेि ने ताइवान के दसक्षिी काऊशुोंग शहर में पनडु ब्बी के
सशपयाडध में एक समारोह के दौरान ताइिाि िी पहिी घरे लू सनसमधत पनडु ब्बी "हाई िुि" लॉन्च की।
• हाई कुन का चीनी भाषा में अिध है "पौरासिक समुद्री जीव" और अोंग्रेजी में इसे 'नरव्हल' कहा जाता है।
i.हाई कुन को ताइवान डॉलर (T$) 49.36 सबसलयन (1.53 सबसलयन अमेररकी डॉलर) की लागत से बनाया गया है।
ii.यह लॉकहीड मासटध न कॉपध की युद्ध प्रिाली का उपयोग करे गा और US (सोंयुक्त राज्य) सनसमधत माकध 48 हेवीवेट
टॉरपीडो ले जाएगा।
iii.डीजल-इलेक्कररक चासलत पनडु ब्बी को कई परीक्षिोों से गुजरना होगा और 2024 के अोंत तक नौसेना को सौोंप सदया
जाएगा।

Report Errors in the PDF - ebooks@affairscloud.com Copyright 2014-2023 @ AffairsCloud.com 162


28. नसतंबर 2023 में, चीि िे ___________ रॉिेट िा उपयोग िरिे "याओगि 33D" िामि एि िया ररमोट
सेंनसंग उपग्रह अंतररक्ष में िॉन्च निया।
1)लॉन्क्ग माचध 4 C
2)लॉन्क्ग माचध 3B
3)लॉन्क्ग माचध 2D
4)लॉन्क्ग माचध 4B
5)लाोंग माचध 5
उत्तर- 1)िॉन्ग माचा 4C
स्पष्टीिरण:
चीन ने एक ररमोट-सेंससोंग उपग्रह, "याओगन-33-04" लॉन्च सकया, सजसे याओगि 33D के नाम से भी जाना जाता है ,
सजसे उत्तर-पसिम चीन के सजउक्वान उपग्रह प्रक्षेपि केंद्र से िॉन्ग माचा-4C वाहक रॉकेट िारा लॉन्च सकया गया।
i.याओगन-33 04 को शोंघाई एकेडमी ऑफ स्पेसिाइट टे क्नोलॉजी िारा सवकससत सकया गया िा। लॉन्क्ग माचध -4C
तरल प्रिोदक िारा ईोंर्न भरने वाला तीन चरिोों वाला प्रक्षेपि यान है।
ii."याओगन-33 04" उपग्रह से प्राप्त जानकारी का उपयोग वैज्ञासनक प्रयोगोों, भूसम सोंसार्न सवेक्षि, फसल उपज
अनुमान और आपदा रोकिाम और राहत कायों के सलए सकया जाएगा।
iii.यह लॉन्च 2023 में चीन के 45वें रॉकेट प्रक्षेपि और लॉन्क्ग माचध वाहक रॉकेट श्रृोंखला के 489वें उडान समशन का
प्रतीक है ।

29. उस दे श िा िाम बताइए निसिे हाि ही में (नसतंबर'23 में) तीसरा सैन्य इमेनिंग उपग्रह िोर 3 िक्षा में
िॉन्च निया है।
1)फ्राोंस
2)ईरान
3)इों डोनेसशया
4)सोंयुक्त अरब अमीरात
5)जापान
उत्तर- 2)ईराि
स्पष्टीिरण:
ईराि के इस्लासमक ररवोल्ूशनरी गाडध कॉप्सध (IRGC) ने ईरान के शाहरौद अोंतररक्ष बोंदरगाह से तीसरे सैन्य इमेसजोंग
उपग्रह िोर 3 को सफलतापूवधक कक्षा में लॉन्च सकया है।
• उपग्रह को तीन चरि वाले कासेद वाहक िारा लॉन्च सकया गया िा सजसे IRGC िारा सवकससत सकया गया िा।
• फ़ारसी में, नोर 3 (सजसे नूर भी कहा जाता है ) का अिध "प्रकाश" है और कासेद का अिध "सोंदेशवाहक" है।
i.नूर-3 को पृथ्वी की सतह से 450 km(280 मील) की ऊोंचाई पर लोअर अिध ऑसबधट (LEO) में थिासपत सकया गया िा।
ii.नोर 3 उपग्रह का उपयोग IRGC के खुसफया उद्दे श्योों के सलए डे टा और सचत्र एकत्र करने के सलए सकया जाएगा।

Report Errors in the PDF - ebooks@affairscloud.com Copyright 2014-2023 @ AffairsCloud.com 163


SPORTS
1. अगस्त 2023 में, ज्यूररि, खस्वट् िरिैंि में आयोनित ज्यूररि िायमंि िीग 2023 में िीरि चोपडा िे पुरुषों
िी भािा फेंि स्पधाा में ___________ मीटर िे साथ ____________ स्थाि पर रहे।
1)दू सरा; 84.92
2)पहला; 86.23
3)दू सरा; 85.71
4)पहला; 84.92
5)तीसरा; 85.71
उत्तर- 3)दू सरा; 85.71
स्पष्टीिरण:
ओलोंसपक और सवश्व चैंसपयन िीरि चोपडा, एक भारतीय टर ै क और फीड एिलीट, ज्यूररख डायमोंड लीग 2023 में
पुरुषों िी भािा फेंि स्पधाा में दू सरे थिान पर रहे , जो 30-31 अगस्त, 2023 तक खस्वट् िरिैंि िे ज्यूररि के
लेटसजग्रोंड स्ट्े सडयम में हुआ िा।
• िीरि चोपडा 85.71 मीटर के सवधश्रेष्ठ के साि दू सरे थिान पर रहे।
• चेक गिराज्य के जैकब वाडलेज्च (85.86 मीटर) ने पहला थिान हाससल सकया और स्विध पदक जीता, और
जमधनी के जूसलयन वेबर (84.92 मीटर) तीसरे थिान पर रहे।
i.नीरज चोपडा के पास वतधमान में 23 डायमोंड लीग 2023 अोंक हैं और वह दू सरे थिान पर हैं , वह अगली बार 16 और
17 ससतोंबर 2023 को यूजीन (सोंयुक्त राज्य अमेररका - USA) में डायमोंड लीग 2023 सीज़न के फाइनल में भाग लेंगे।
ii.भारतीय एिलीट मुरली श्रीशोंकर ज्यूररख डायमोंड लीग 2023 में पुरुषोों की लोंबी कूद स्पर्ाध में 5वें थिान पर रहे और
वह यूजीन, USA में डायमोंड लीग फाइनल में पदापधि करने के सलए तैयार हैं।

2. हाि ही में (नसतंबर'23 में) फॉमूािा 1 (F1)इटानियि ग्रैंि नप्रक्स 2023 िा 93िां संस्करण निसिे िीता है
और िगातार सबसे अनधि (10िी ं) F1 िीत िा ररिॉिा बिाया है ?
1)मैक्स वेरस्ट्ै पेन
2)ससजधयो पेरेज़
3)कालोस सैन्क्ज़
4)सेबेक्कस्ट्यन वेट्टेल
5)लुईस हैसमल्टन
उत्तर- 1)मैक्स िेरस्ट्ै पेि
स्पष्टीिरण:
3 ससतोंबर, 2023 को, रे ड बुल के बेक्कियम-डच डर ाइवर मैक्स िेरस्ट्ै पेि ने मोोंज़ा ससकधट में आयोसजत इटानियि ग्रैंि
नप्रक्स िा 93 िां संस्करण जीता, सजसे आसर्काररक तौर पर फॉमूािा 1 (F1)सपरे ली ग्रैन प्रेसमयो डी'इटासलया 2023
के रूप में जाना जाता है , ऑटोडर ोमो नासज़योनेल मोोंज़ा, इटली के समलान के पास एक शहर मोोंज़ा में क्कथित है ।
• रे ड बुल के ससजधयो पेरेज़ (मेक्कक्सको) और फेरारी के कालोस सैन्क्ज़ जूसनयर (स्पेन) क्रमशः दू सरे और तीसरे
थिान पर रहे।
• यह जीत मैक्स वेरस्ट्ै पेन की 2023 F1 सीज़न की लगातार 10वीों जीत है और लगातार सबसे असर्क F1 जीत
का ररकॉडध थिासपत करती है।

Report Errors in the PDF - ebooks@affairscloud.com Copyright 2014-2023 @ AffairsCloud.com 164


3. उस फुटबॉि िब िा िाम बताइए निसिे हाि ही में (नसतंबर’23 में) ईस्ट् बंगाि फुटबॉि िब िो 1-0 से
हरािर िूरं ि िप 2023 टर ॉफी िा 132िां संस्करण िीता है।
1)मुोंबई ससटी
2)मोहन बागान सुपर जायोंट
3)चेन्नईसयन
4)राजथिान यूनाइटे ड
5)केरल िास्ट्सध
उत्तर-2)मोहि बागाि सुपर िायंट
स्पष्टीिरण:
मोहन बागान सुपर जायोंट ने ईस्ट् बोंगाल फुटबॉल क्लब को 1-0 से हराकर सववेकानन्द युबा भारती क्रीडाोंगन (साल्ट
लेक स्ट्े सडयम), कोलकाता, पसिम बोंगाल में आयोसजत िूरं ि िप 2023 टर ॉफी जीती।
i.मोहन बागान डूरों ड कप में 16 क्कखताब के साि ईस्ट् बोंगाल को पीछे छोडते हुए 17 क्कखताब जीतने वाली पहली टीम
बन गई।
ii.डूरों ड कप2023 3 अगस्त से 3 ससतोंबर 2023 तक आयोसजत सकया गया िा, इसका आयोजन डूरों ड फुटबॉल टू नाधमेंट
सोसाइटी (DFTS)- भारतीय सशस्त्र बलोों का एक सहस्सा, अक्कखल भारतीय फुटबॉल महासोंघ (AIFF) के सहयोग से
सकया गया िा।
iii. डूरों ड कप भारत और एसशया की सबसे पुरानी फुटबॉल प्रसतयोसगता है , और दु सनया की तीसरी सबसे पुरानी
प्रसतयोसगता भी है।

4. नसतंबर 2023 में, भारतीय पुरुष हॉिी टीम िे ओमाि िे सिािाह में आयोनित फाइिि में ______________
(दे श) िो हरािर पुरुष हॉिी5s एनशया िप सिािाह 2023 िा पहिा संस्करण िीता।
1)बाोंग्लादे श
2)ओमान
3)मलेसशया
4)जापान
5)पासकस्तान
उत्तर-5)पानिस्ताि
स्पष्टीिरण:
2 ससतोंबर, 2023 को, भारतीय पुरुष हॉिी टीम ने सुल्तान कबूस यूि कॉम्प्लेक्स फॉर कल्चर एों ड एों टरटे नमेंट- सपच
1(स्ट्े सडयम), सलालाह, ओमान में फाइनल में पानिस्ताि को पेनल्टी शूटआउट िारा 4-4 (2-0) से हराकर उद् घाटन
पुरुष हॉिी5s एनशया िप सिािाह 2023 जीता।
• इस जीत के साि, भारत ने FIH पुरुष हॉकी5s वडध कप ओमान 2024 के सलए क्वालीफाई कर सलया।
• 29 अगस्त से 2 ससतोंबर 2023 तक आयोसजत चैंसपयनसशप सीसनयर पुरुष वगध के सलए िी, और FIH पुरुष
हॉकी5s वडध कप ओमान 2024 के सलए क्वालीफाइों ग टू नाधमेंट के रूप में कायध सकया।
• भारतीय पुरुष हॉकी टीम को टर ॉफी के साि स्विध पदक सदया गया और पासकस्तान को रजत पदक और
ओमान को काोंस्य पदक समला।

Report Errors in the PDF - ebooks@affairscloud.com Copyright 2014-2023 @ AffairsCloud.com 165


5. नसतंबर 2023 में, भारत िे बािू, अिरबैिाि में आयोनित इं टरिेशिि शूनटं ग स्पोटा फेिरे शि (ISSF) निश्व
चैंनपयिनशप 2023 में __________पदि िीते और तीसरा स्थाि हानसि निया।
1)16
2)12
3)10
4)28
5)14
उत्तर- 5)14
स्पष्टीिरण:
भारत ने 14 अगस्त से 1 ससतोंबर 2023 तक बाकू, अजरबैजान में आयोसजत इों टरनेशनल शूसटों ग स्पोटध फेडरे शन (ISSF)
वडध चैंसपयनसशप 2023 में 14 पदि (6 स्विध और 8 काोंस्य) जीते और तीसरा स्थाि हाससल सकया।
• चीि 28 पदक (15 स्विध, 7 रजत और 6 काोंस्य) के साि पदि तानििा में शीषध पर रहा, उसके बाद यूक्रेन
12 पदक (6 स्विध, 4 रजत और 2 काोंस्य) के साि दू सरे थिान पर रहा।
i.ISSF िडा चैखम्पयिनशप 2023 पेररस 2024 ओलोंसपक के सलए एक क्वालीफाइों ग इवेंट है।
• राजेश्वरी कुमारी (मसहला टर ै प)
• ससफ्त कौर समरा (मसहला 50m राइफल 3 पोजीशन)
• अक्कखल श्योराि (पुरुष 50m राइफल 3 पोजीशन)
• मेहुली घोष (मसहला 10m एयर राइफल)
ii.ISSF वडध चैंसपयनसशप (2023)के 53वें सोंस्करि में राइफल, सपस्ट्ल और शॉटगन श्रेसियोों में 36 पदक स्पर्ाधओों के
सलए 101 दे शोों के 1,329 सनशानेबाजोों ने भाग सलया।

6. नसतंबर 2023 में आयोनित अंतरााष्टरीय टे बि टे निस महासंघ (ITTF) एनशयाई टे बि टे निस चैंनपयिनशप 2023
िे संबंध में निम्ननिखित में से िौि सा/से नबंदु "सही" है /हैं ?
A) शरथ िमि, सानथयाि ज्ञािसेिरि और हरमीत दे साई िी भारतीय पुरुष टीम िे अंतरााष्टरीय टे बि टे निस
महासंघ (ITTF) एनशयाई टे बि टे निस चैंनपयिनशप 2023 में िांस्य पदि िीता।
B) चीि िे फाइिि में चीिी ताइपे िो हरािर पुरुष टीम स्पधाा में स्वणा पदि िीता
C) सुतीथाा मुििी, अचािा नगरीश िामथ और श्रीिा अिुिा िी भारतीय मनहिा टीम िे मनहिा टीम स्पधाा में
पांचिां स्थाि हानसि निया, िबनि चीिी मनहिा टे बि टे निस टीम िे स्वणा पदि िीता।
1)केवल A
2)केवल A & B
3)केवल B & C
4)केवल A & C
5)सभी A, B & C
उत्तर -5)सभी A, B & C
स्पष्टीिरण:
शरथ िमि, सानथयाि ज्ञािसेिरि और हरमीत दे साई की भारतीय पुरुष टीम ने अोंतराध ष्ट्रीय टे बल टे सनस महासोंघ
(ITTF) एनशयाई टे बि टे निस चैंनपयिनशप 2023 में िांस्य पदि जीता।
• IITF एसशयन टे बल टे सनस चैंसपयनसशप 2023 टू नाधमेंट का 26वाों सोंस्करि है और यह पेररस 2024 ओलोंसपक के
सलए क्वालीसफकेशन टू नाधमेंट भी िा।

Report Errors in the PDF - ebooks@affairscloud.com Copyright 2014-2023 @ AffairsCloud.com 166


• भारत और दसक्षि कोररया को क्रमशः चीनी ताइपे और चीन के क्कखलाफ सेमीफाइनल में हारने के बाद काोंस्य
पदक से सोंतोष करना पडा। एसशयाई चैंसपयनसशप में सेमीफाइनल में हारने वाले दो क्कखलासडयोों को काोंस्य
पदक से सम्मासनत सकया जाता है।
• चीन ने फाइनल में चीनी ताइपे को हराकर पुरुष टीम स्पर्ाध में स्विध पदक जीता। चीनी ताइपे ने रजत पदक
जीता
• यह एसशयाई चैंसपयनसशप में उनका लगातार दू सरा पदक है। उन्ोोंने इससे पहले दोहा, कतर में 2021 एसशयाई
चैक्कम्पयनसशप में काोंस्य पदक जीता िा।
• सुतीिाध मुखजी, अचधना सगरीश कामि और श्रीजा अकुला की भारतीय मसहला टीम ने मसहला टीम स्पर्ाध में
पाोंचवाों थिान हाससल सकया।
• चीनी मसहला टे बल टे सनस टीम ने मसहला टीम स्पर्ाध के फाइनल में दसक्षि कोररया को हराकर स्विध पदक
जीता।

7. न्यूयॉका, संयुक्त राज्य अमेररका (USA) में आयोलित US ओपन 2023 के संबंि में लनम्नलिस्कित में से कौन
सा लबंदु "सही" है/हैं।
A) सलबाया के नोवाक िोकोलवच ने रूस के िे लनि मेदवेदेव को हराकर अपना चौथा US ओपन स्किताब और
24 वां ग्रैंि स्लैम स्किताब िीता।
B)आयाना सबािेंका ने मलहिा एकि फाइनि में कोको गॉफ को हराकर US ओपन 2023 में अपना पहिा ग्रैंि
स्लैम स्किताब िीता।
C)रािीव राम और िो सैलिसबरी ने फाइनि में रोहन बोपन्ना और मैथ्यू एबिे न को हराकर US ओपन में पुरुष
युगि का स्किताब िीता।
1) केवल A
2) केवल A & B
3) केवल B & C
4) केवल A & C
5) सभी A, B & C
उत्तर- 4)केवल A & C
स्पष्ट्ीकरि:
नोवाक िोकोलवच ने आिगर ऐश स्ट्े श्चियम (हािा कोटा ), न्यूयॉकग शहर, न्यूयॉकग, संयुक्त राज्य अमेररका (USA) में पुरुष
एकल फाइनल में िे श्चनयल मेदवेदेव को हराकर US ओपन 2023 में अपना 24 वां ग्रैंि स्लैम स्खताब िीता।
• US ओपन का 143वां संिरण 28 अगस्त से 10 श्चसतंबर 2023 (मेन-िर ा) तक आयोश्चित श्चकया गया िा।
i.कोको गॉफ़ (मश्चहला टे श्चनस संघ WTA- रैं श्चकंग 3) ने मश्चहला एकल फाइनल में आयगना सबालेंका (WTA रैं श्चकंग 1) को
हराकर US ओपन 2023 में अपना पहला ग्रैंि स्लैम स्खताब िीता।
ii.रािीव राम और िो सैलिसबरी ने फाइनल में रोहन बोपन्ना और मैथ्यू एबिे न को हराकर US ओपन में पुरुष युगल
का स्खताब िीता।
iii.गैलिएिा िािोव्स्िी और एररन राउटलिफ़ ने फाइनल में 2020 US ओपन चैंश्चपयन वेरा ज़्वोनारे वा और लौरा
सीगमंि को हराकर US ओपन में मश्चहला युगल का स्खताब िीता।
iv.हैरी हेलियोवारा और अन्ना िालिलनना ने फाइनल में िेश्चसका पेगुला और ऑस्स्ट्न क्राश्चिसेक को हराकर US
ओपन 2023 में श्चमश्चश्रत युगल स्खताब िीता।

Report Errors in the PDF - ebooks@affairscloud.com Copyright 2014-2023 @ AffairsCloud.com 167


8. अगस्त से नसतंबर 2023 ति िोििाता, पनिम बंगाि में आयोनित टाटा स्ट्ीि चेस इं निया रै नपि और
खब्लट् ि 2023 िे संबंध में निम्ननिखित में से िौि सा/से नबंदु "सही" है /हैं ?
A) फ्रांस िे मैखक्सम िानचएर-िाग्रेि िे अज़रबैिाििे तीमुर राद् िाबोि िो हरािर टाटा स्ट्ीि चेस इं निया
ओपि रै नपि खिताब िीता।
B) अिेक्जेंिर नग्रशुि (FIDE द्वारा प्रनतनिनधत्व) िे उज्बेनिस्ताि िे िोनदरबेि अब्दु सात्तोरोि िो हरािर टाटा
स्ट्ीि इं निया 2023 ओपि खब्लट् ि खिताब िीता।
C) भारत िी मनहिा ग्रैंिमास्ट्र नदव्या दे शमुि िे चीि िी िू िेििुि िो हरािर मनहिा िगा में टाटा स्ट्ीि
चेस इं निया रै नपि खिताब िीता।
1)केवल A
2)केवल A & B
3)केवल B & C
4)केवल A & C
5)सभी A, B & C
उत्तर -5)सभी A, B & C
स्पष्टीिरण:
टाटा स्ट्ील चेस इों सडया रै सपड एों ड क्किट् ज 2023 31 अगस्त से 9 ससतोंबर 2023 तक नेशनल लाइब्रेरी ऑफ इों सडया,
कोलकाता, पसिम बोंगाल में आयोसजत सकया गया िा।
• भारत की महाराष्ट्र की अोंतराधष्ट्रीय मास्ट्र सदव्या दे शमुख ने मसहला वगध में टाटा स्ट्ील चेस इों सडया रै सपड क्कखताब
जीता।
• तसमलनाडु के भारतीय चेस ग्रैंडमास्ट्र रमेशबाबू प्रगनानोंद ने ओपन रै सपड और ओपन क्किट् ज दोनोों स्पर्ाधओों
में तीसरा थिान हाससल सकया।
• 2018 में पहला सोंस्करि आयोसजत होने के बाद से यह टू नाधमेंट का 5वाों सोंस्करि है।
टाटा स्ट्ीि चेस इं निया रै नपि और खब्लट् ज़ 2023 िे नििेता:

श्रेणी रै नपि खब्लट् ज़

ओपि मैक्कक्सम वासचएर-लाग्रेव (फ्राोंस) अलेक्जेंडर सग्रशुक (FIDE िारा प्रस्तुत)


(पुरुष)
तीमुर राद् जाबोव (अज़रबैजान) नोसदरबेक अब्दु सत्तोरोव (उज़्बेसकस्तान)

मनहिा नदव्या दे शमुि (भारत) जू वेनजुन (चीन)

जू वेनजुन (चीन) कोनेरू हम्पी (भारत)

9. नसतंबर 2023 में, निस दे श िी पुरुष बास्केटबॉि टीम िे सनबाया िो हरािर पहिी बार अंतरााष्टरीय
बास्केटबॉि महासंघ (FIBA) बास्केटबॉि निश्व िप खिताब िीता?
1)स्पेन
2)कनाडा
3)जमधनी
4)अजेंटीना
5)फ़्ाोंस

Report Errors in the PDF - ebooks@affairscloud.com Copyright 2014-2023 @ AffairsCloud.com 168


उत्तर-3)िमािी
स्पष्टीिरण:
िमाि पुरुष बास्केटबॉि टीम ने सफलीपीोंस के मनीला में आयोसजत फाइनल में सनबाया िो हरािर अोंतराध ष्ट्रीय
बास्केटबॉल महासोंघ (FIBA) बास्केटबॉि निश्व िप 2023 जीता।
• FIBA बास्केटबॉल सवश्व कप का 19वाों सोंस्करि पहली बार 25 अगस्त से 10 ससतोंबर 2023 तक तीन मेजबान
दे शोों सफलीपीोंस, जापान और इों डोनेसशया में हुआ।
• यह जमधनी का पहला FIBA बास्केटबॉल सवश्व कप क्कखताब है।
• इस आयोजन में 32 टीमोों ने भाग सलया, सजन्ें 8 समूहोों में बाोंटा गया, सजनमें से प्रत्येक में 4 टीमें िीों।
• जमधनी 2006 के बाद अपने पहले फाइनल में FIBA सवश्व कप का ताज जीतने वाली पहली टीम बन गई, इससे
पहले स्पेन ने 2006 में यह क्कखताब जीता िा।

10. उस भारतीय टे निस खििाडी िा िाम बताइए िो हाि ही में (नसतंबर’23 में) टु ल्ि, ऑखस्ट्र या में आयोनित
NÖ OPEN 2023 में उपनििेता रहा।
1)युकी भाोंबरी
2)सुसमत नागल
3)रोहन बोपन्ना
4)प्रजनेश गुिेश्वरन
5)मुकुोंद शसशकुमार
उत्तर- 2)सुनमत िागि
स्पष्टीिरण:
भारतीय टे सनस क्कखलाडी सुनमत िागि 3 से 10 ससतोंबर 2023 तक टु ल्न, ऑक्कस्ट्रया में आयोसजत एसोससएशन ऑफ
टे सनस प्रोफेशनल्स (ATP) चैलेंजर 100 इवेंट, NÖ OPEN 2023 (EVN िारा सोंचासलत) में उपसवजेता रहे।
i.टु ल्न में TC टु ल्न सुसवर्ा में आयोसजत फाइनल में चेक गिराज्य के सवट कोसप्रवा ने सुसमत नागल को हराकर क्कखताब
जीता।
ii.नागल ने इवेंट के दौरान 60 रैं सकोंग अोंक असजधत सकए और ATP रैं सकोंग में 156वें (11 ससतोंबर 2023 तक) थिान पर
हैं।

11. नसतम्बर 2023 में गोिा िे राज्यपाि P.S. श्रीधरि नपल्लई िे अरू बर से ििंबर 2023 ति गोिा में
आयोनित होिे िािे ___________ राष्टरीय िेिों िे निए मशाि (टॉचा) िॉन्च िी।
1)38वाों
2)40वाों
3)37वाों
4)35वाों
5)36वाों
उत्तर- 3)37िां
स्पष्टीिरण:
गोवा के राज्यपाल P.S. श्रीर्रन सपल्लई ने गोवा के खेल और युवा मामलोों के सवभाग और गोवा के खेल प्रासर्करि िारा
दरबार हॉल, राजभवन, डोनापौला, गोिा में आयोसजत एक समारोह में 37वें राष्ट्रीय खेलोों के सलए मशाल (टॉचध) लॉन्च
की।

Report Errors in the PDF - ebooks@affairscloud.com Copyright 2014-2023 @ AffairsCloud.com 169


• 37वें राष्ट्रीय खेल 25 अरू बर, 2023 से 9 नवोंबर 2023 तक गोिा में आयोसजत होने वाले हैं।
• राज्यपाल ने 37वें राष्ट्रीय खे लोों का एों िम (िीम सॉन्क्ग) भी लॉन्च सकया, सजसमें भारतीय असभनेता असमताभ बच्चन
की आवाज है। राष्ट्रगान के सलए गोवा के वाद्ययों त्र घु मट का भी उपयोग सकया जाता है ।

12. उस भारतीय तीरं दाि िा िाम बताइए निसिे हाि ही में (नसतंबर’23 में) हमोनसिो 2023 हुंिई तीरं दािी
निश्व िप फाइिि में पुरुष िंपाउं ि में रित पदि िीता है।
1)प्रिमेश समार्ान जावकर
2)श्याम लाल मीना
3)सत्यदे व प्रसाद
4)असभषेक वमाध
5)अतनु दास
उत्तर- 1)प्रथमेश समाधाि िाििर
स्पष्टीिरण:
भारतीय तीरों दाज प्रथमेश समाधाि िाििर (सवश्व में 14वें थिान पर) ने हमोससलो 2023 हुोंडई तीरों दाजी सवश्व कप
फाइनल में पुरुषोों के कोंपाउों ड में रजत पदक जीता, जो 9 और 10 ससतोंबर 2023 को हमोससलो, मैक्कक्सको में आयोसजत
सकया गया िा।
• प्रिमेश जावकर ने स्विध पदक मुकाबले में डे नमाकध के मासियस फुलटध न (सवश्व में 14वें नोंबर के क्कखलाडी) के
क्कखलाफ प्रसतस्पर्ाध की।
• मैच टाई रहा क्ोोंसक दोनोों क्कखलासडयोों ने पाोंच सेट के बाद 148 अोंक बनाए। जबसक दोनोों तीरों दाजोों ने शूट-
ऑफ में 10 का स्कोर सकया, फुलटध न जीत गए क्ोोंसक उनका तीर केंद्र के करीब िा।

13. उस भारतीय बैिनमंटि खििाडी िा िाम बताइए निसिे हाि ही में (नसतंबर'23 में) मेदाि, इं िोिेनशया में
आयोनित BNI इं िोिेनशया मास्ट्सा 2023 में पुरुष एिि खिताब िीता है।
1)सप्रयाोंशु राजावत
2)समिुन मोंजूनाि
3)लक्ष् सेन
4)B. साई प्रिीत
5)सकरि जॉजध
उत्तर-5)निरण िॉिा
स्पष्टीिरण:
भारत के निरण िॉिा ने 5 से 10 ससतोंबर 2023 तक मेदान, इों डोनेसशया में आयोसजत BNI इं िोिेनशया मास्ट्सा
2023 में पुरुष एिि खिताब जीता।
• BNI इों डोनेसशया मास्ट्सध 2023 बैडसमोंटन वडध फेडरे शन (BWF) वडध टू र सुपर 100 टू नाधमेंट है।
• 2022 में ओसडशा ओपन में अपना पहला क्कखताब जीतने के बाद यह सकरि जॉजध का दू सरा BWF सुपर 100
क्कखताब है।
• पुरुष एकल: भारत के सकरि जॉजध ने फाइनल में जापान के कू ताकाहाशी को हराकर पुरुष एकल का क्कखताब
जीता।
• इस जीत ने उन्ें पुरुष एकल में कररयर की सवधश्रेष्ठ BWF 39वीों रैं क हाससल करने में मदद की।

Report Errors in the PDF - ebooks@affairscloud.com Copyright 2014-2023 @ AffairsCloud.com 170


• मसहला एकल: इों डोनेसशया की एस्ट्र नुरुमी टर ाई वाडोयो ने फाइनल में चीनी ताइपे की सचउ सपन-सचयान को
हराकर मसहला एकल का क्कखताब जीता।

14. नसतंबर 2023 में, भारतीय पुरुष फुटबॉि टीम िे नथम्पू, भूटाि में आयोनित फाइिि में ___________ (दे श)
िो हरािर दनक्षण एनशयाई फुटबॉि महासंघ (SAFF) U-16 चैखम्पयिनशप 2023 िा खिताब िीता।
1)भूटान
2)नेपाल
3)मालदीव
4)बाोंग्लादे श
5)पासकस्तान
उत्तर-4)बांग्लादे श
स्पष्टीिरण:
भारतीय अोंडर 16 (U-16) पुरुष फुटबॉि टीम ने फाइनल में बाों ग्लादे श को (2-0) से हराकर दसक्षि एसशयाई
फुटबॉल महासोंघ (SAFF) U-16 चैखम्पयिनशप 2023 का क्कखताब जीता। 2023 SAFF U16 चैक्कम्पयनसशप 2 से 10
ससतोंबर 2023 तक सिम्पू, भूटान के चाोंगसलसमिाोंग स्ट्े सडयम में आयोसजत की गई िी।
• 2023 SAFF U16 चैक्कम्पयनसशप SAFF U-15/U-16/U-17 चैक्कम्पयनसशप का 8वाों सोंस्करि है , जो SAFF
सदस्योों (भारत, नेपाल, बाों ग्लादे श, भूटान, मालदीव, पासकस्तान) की राष्ट्रीय टीमोों िारा लडी जाने वाली वासषध क
अोंतराधष्ट्रीय फुटबॉल प्रसतयोसगता है ।
• यह SAFF U-15/U-16/U-17 चैंसपयनसशप में भारत का 5वाों क्कखताब है।
• इसके साि ही भारत 5 क्कखताब (2017 और 2019 में U15 क्कखताब; 2013 और 2023 में U16 क्कखताब और
2022 में U-17 क्कखताब) के साि टू नाधमेंट की सबसे सफल टीम बन गई।

15. नसतंबर 2023 में िोिंबो, श्रीिंिा में आयोनित एनशया िप 2023 फाइिि िे 16िें संस्करण िे संबंध में
निम्ननिखित में से िौि सा नबंदु "गित" है ?
1)भारत ने एसशया कप 2023 के फाइनल में श्रीलोंका को हराकर ररकॉडध आठवीों बार एसशया कप जीता है।
2)भारतीय पुरुष सक्रकेट टीम के कप्तान रोसहत शमाध ने अपने कररयर में तीसरी बार एसशया कप टर ॉफी जीती और
सौरव गाोंगुली और MS र्ोनी के साि सवसशष्ट् सूची में शासमल हो गए।
3)एसशया कप में पासकस्तान के क्कखलाफ सवराट कोहली ने शतक लगाया और सबसे तेज 13,000 एक सदवसीय
अोंतराधष्ट्रीय (ODI) रन बनाने वाले बल्लेबाज बन गए।
4)रोसहत शमाध सवराट कोहली, ससचन तेंदुलकर, सौरव गाोंगुली, MS र्ोनी और राहुल द्रसवड के बाद 10,000 ODI रन
क्लब में प्रवेश करने वाले छठे भारतीय बन गए।
5)शुबमन सगल 2023 में 1,000 ODI रन बनाने वाले पहले बल्लेबाज बने।
उत्तर- 2)भारतीय पुरुष नक्रिेट टीम िे िप्ताि रोनहत शमाा िे अपिे िररयर में तीसरी बार एनशया िप टर ॉफी
िीती और सौरि गांगुिी और MS धोिी िे साथ निनशष्ट सूची में शानमि हो गए।
स्पष्टीिरण:
भारत ने श्रीलोंका के कोलोंबो के R प्रेमदासा स्ट्े सडयम में एसशया कप 2023 के फाइनल में गत चैंसपयन श्रीलोंका को
हराकर ररकॉडध आठवीों बार एसशया कप जीता है।
• एसशया कप 2023 के 16वें सोंस्करि की पहली बार 30 अगस्त से 17 ससतोंबर 2023 तक श्रीलोंका और
पासकस्तान िारा सह-मेजबानी की गई िी।

Report Errors in the PDF - ebooks@affairscloud.com Copyright 2014-2023 @ AffairsCloud.com 171


• आठ क्कखताब (सात ODI और एक T20I) के साि, भारत टू नाधमेंट के इसतहास में सबसे सफल टीम बन गई।
जहाों श्रीलोंका छह क्कखताब के साि दू सरी सबसे सफल टीम है और पासकस्तान ने दो क्कखताब जीते हैं।
• भारतीय पुरुष सक्रकेट टीम के कप्तान रोनहत शमाा ने अपने कररयर में दू सरी बार एसशया कप टर ॉफी जीती
और मोहम्मद अज़हरुद्दीि और MS धोिी के साि सवसशष्ट् सूची में शासमल हो गए।
❖ रोसहत से पहले अज़हरुद्दीन और र्ोनी ही ऐसे दो कप्तान िे सजन्ोोंने दो बार एसशया कप जीता िा।
अज़हर ने 1991 और 1995 में जबसक र्ोनी ने 2010 और 2016 में क्कखताब जीता िा।
❖ रोसहत ने पहली बार 2018 में सोंयुक्त अरब अमीरात (UAE) में एसशया कप जीता िा, जो ट्वें टी 20
अोंतराधष्ट्रीय (T20I) प्रारूप में खेला गया िा।
• सवराट कोहली ने एसशया कप में पासकस्तान के क्कखलाफ शतक लगाया और सबसे तेज 13,000 एक सदवसीय
अोंतराधष्ट्रीय (ODI) रन बनाने वाले बल्लेबाज बन गए।
• सवराट कोहली, ससचन तेंदुलकर, सौरव गाोंगुली, MS र्ोनी और राहुल द्रसवड के बाद रोसहत शमाध 10,000 ODI
रन क्लब में शासमल होने वाले छठे भारतीय बन गए हैं।
• शुबमन सगल 2023 में 1,000 ODI रन बनाने वाले पहले बल्लेबाज बने।

16. नसतंबर 2023 में, संयुक्त राज्य अमेररिा (USA) िे ओरे गि में आयोनित िायमंि िीग 2023 फाइिि में
िीरि चोपडा िे पुरुषों िी भािा फेंि स्पधाा में ___________ m िे साथ ____________ िो पूरा निया।
1)दू सरा; 84.92
2)पहला; 86.23
3)दू सरा; 83.80
4)पहला; 84.92
5)तीसरा; 85.71
उत्तर- 3)दू सरा; 83.80
स्पष्टीिरण:
टोक्ो 2020 ओलोंसपक स्विध पदक सवजेता िीरि चोपडा डायमोंड लीग 2023 फाइनल में पुरुषों िी भािा फेंि
स्पधाा में 83.80 m के सवधश्रेष्ठ थ्रो के साि दू सरे स्थाि पर रहे।
• चेक गिराज्य के जैकब वाडलेज्च ने 84.24 m के सवधश्रेष्ठ थ्रो के साि पुरुषोों की भाला फेंक का क्कखताब जीता,
जबसक सफनलैंड के ओसलवर हेलैंडर 83.74 m के साि तीसरे थिान पर रहे।
• 2016 और 2017 डायमोंड लीग चैंसपयनसशप जीतने के बाद जैकब वाडलेज्च के सलए 2023 का क्कखताब
डायमोंड लीग चैंसपयनसशप में उनका तीसरा क्कखताब िा।
• यूजीन में नीरज चोपडा का ररकॉडध : नीरज चोपडा के नाम यूजीन में 88.13 m का सवधश्रेष्ठ थ्रो का ररकॉडध है , जब
उन्ोोंने 2022 में सवश्व चैंसपयनसशप में रजत पदक जीता िा।
• डायमोंड लीग 2023 का फाइनल 16 से 17 ससतोंबर 2023 तक यूजीन, ओरे गन, सोंयुक्त राज्य अमेररका (USA)
में आयोसजत सकया गया िा।
िोट- 2023 डायमोंड लीग 14वाों सोंस्करि है (प्रायोजन कारि इसे वाोंडा डायमोंड लीग कहा जाता है ), 5 मई, 2023 से
17 ससतोंबर, 2023 तक आयोसजत सकया गया।

Report Errors in the PDF - ebooks@affairscloud.com Copyright 2014-2023 @ AffairsCloud.com 172


17. उस व्यखक्तत्व िा िाम बताइए निसिे हाि ही में (नसतंबर'23 में) फॉमूािा 1 (F1) नसंगापुर एयरिाइं स
नसंगापुर ग्रांि नप्रक्स 2023 िीता है।
1)लुईस हैसमल्टन
2)लैंडो नॉररस
3)कालोस सैन्क्ज़ जूसनयर
4)फनािंडो अलोोंसो
5)चाल्सध लेक्लर
उत्तर-3)िािोस सैन्ज़ िूनियर
स्पष्टीिरण:
फेरारी के स्पेन के कालोस सैन्क्ज़ वाज़क्वेज़ डी कास्त्रो (िािोस सैन्ज़ िूनियर) ने ससोंगापुर के मरीना बे स्ट्र ीट ससकधट में
आयोसजत फॉमूधला 1 (F1)ससोंगापुर एयरलाइों स ससोंगापुर ग्रैंड सप्रक्स 2023 जीता।
• यूनाइटे ड सकोंगडम (UK) के मैकलेरन डर ाइवर िैंिो िॉररस दू सरे थिान पर रहे जबसक UK के मससधडीज डर ाइवर
िुईस हैनमल्टि तीसरे थिान पर रहे।
• रे स में मैक्स वेरस्ट्ै पेन के पाों चवें थिान ने उनकी लगातार 10 F1 जीत का अोंत कर सदया।
• अब तक मैक्स वेरस्ट्ै पेन ने 2023 में 12 F1 ग्रैंड सप्रक्स जीते हैं और अभी भी सीज़न की चैंसपयनसशप का नेतृत्व
कर रहे हैं ।
i.F1 ससोंगापुर ग्राोंड सप्रक्स का 14वाों सोंस्करि 15 से 17 ससतोंबर 2023 तक आयोसजत सकया गया िा और ससोंगापुर ग्राोंड
सप्रक्स 2023, 2023 फॉमूधला वन (F1)सवश्व चैक्कम्पयनसशप का 15वाों दौर िा।

18. भारत लसतंबर 2023 में िािीि के ररयो िी िनेररयो में इं टरनेशनि शूलटं ग स्पोटा फेिरे शन (ISSF )
राइफि/लपस्ट्ि लवश्व कप 2023 में __________ पदकों के साथ पदक तालिका में पांचवें स्थान पर रहा।
1)चार
2)दो
3)तीन
4)पांच
5)छह
उत्तर- 2)दो
स्पष्टीकरण:
इं टरनेशनल शूश्चटंग स्पोटग फेिरे शन (ISSF ) राइफि/लपस्ट्ि लवश्व कप 2023 ररयो िी िनेररयो, िािील में 12 से 19
श्चसतंबर 2023 तक आयोश्चित श्चकया गया िा। भारत 2 पदक (1 स्वणा और 1 रित) के साि पदक ताश्चलका में 5वें
थिान पर रहा।
• श्चवश्व कप राइफल/श्चपस्ट्ल 2023 ररयो िी िनेररयो राइफल और श्चपस्ट्ल श्चनशानेबािों के श्चलए ISSF श्चवश्व कप
का अंश्चतम चरि िा।
i.इटली 3 पदक (2-स्विग, 1-कांस्य) के साि पदक ताश्चलका में शीषग पर है, चीन ने 8 पदक (1-स्विग, 4-रित, 3-कांस्य)
के साि दू सरा थिान हाश्चसल श्चकया।
ii.मश्चहलाओं की 10m एयर राइफल स्पर्ाग में भारत की इलावेश्चनल वलाररवन ने स्विग पदक िीता।
• 2019 में ISSF श्चवश्व कप ररयो में मश्चहलाओं की 10m एयर राइफल स्पर्ाग में स्विग पदक िीतने के बाद से यह
एलावेश्चनल का दू सरा व्यस्क्तगत ISSF श्चवश्व कप पदक िा।

Report Errors in the PDF - ebooks@affairscloud.com Copyright 2014-2023 @ AffairsCloud.com 173


iii.भारत की लनश्चि ने मश्चहलाओं की 50m राइफल 3 पोिीशन में रित पदक िीता, िबश्चक नॉवे की िीनत हेग
िु एस्ट्ै ि और िे नमाकग की स्ट्े फ़नी लॉरा स्कुराग ग्रंिसोई ने क्रमशः स्विग और कांस्य पदक िीते।
iv.19 वषीय श्चनिल ने ररले वन में 587 का ठोस स्कोर बनाकर मश्चहलाओं की 3 पोिीशन स्पर्ाग में िालीश्चफकेशन
राष्ट्रीय ररकॉिग बनाया।

19. उस व्यखक्तत्व िा िाम बताइए निसिे हाि ही में (नसतंबर'23 में) ग्रेटर िोएिा, उत्तर प्रदे श में आयोनित
इं नियि ऑयि ग्रैंि नप्रक्स ऑफ इं निया (मोटोGP भारत 2023)िीता है।
1)माको बेज़ेची
2)जॉजध मासटध न
3)फैसबयो क्वाटाधरो
4)पेडरो अकोस्ट्ा
5)अयुमु सासाकी
उत्तर- 1)मािो बेज़ेची
स्पष्टीिरण:
मूनी VR46 रे ससोंग टीम के इटली के मािो बेज़ेची ने बुद्ध इों टरनेशनल ससकधट (BIC), ग्रेटर नोएडा, उत्तर प्रदे श में
आयोसजत इों सडयन ऑयल ग्रैंड सप्रक्स ऑफ इों सडया (मोटोGP भारत 2023)जीता।
i.प्राइमा प्रामैक रे ससोंग के जॉजध मासटध न (स्पेन) और मॉन्स्टर एनजी यामाहा मोटोGP के फैसबयो क्वाटाधरो ((फ्राों स)) क्रमशः
दू सरे और तीसरे थिान पर रहे।
ii.मोटोGP इों सडया ग्राोंड सप्रक्स का उद् घाटन सोंस्करि 22 से 24 ससतोंबर 2023 तक आयोसजत सकया गया िा।
iii. इों सडया ग्राोंड सप्रक्स 2023, 2023 के मोटोGP सीज़न में 20 रे सोों में से 13वीों रे स िी। यह फॉमूधला 1 के बाद भारत में
सबसे बडी मोटर रे ससोंग प्रसतयोसगता है ।

20. उस व्यखक्तत्व िा िाम बताइए निसिे हाि ही में (नसतंबर'23 में) िापाि िे सुिुिा में आयोनित फॉमूािा
1 (F1)िापािी ग्रांि नप्रक्स 2023 िीता है।
1)लैंडो नॉररस
2)मैक्स वेरस्ट्ै पेन
3)ऑस्कर सपयास्त्री
4)लुईस हैसमल्टन
5)चाल्सध लेक्लर
उत्तर- 2)मैक्स िेरस्ट्ै पेि
स्पष्टीिरण:
रे ड बुल के बेक्कियम-डच डर ाइवर मैक्स िेरस्ट्ै पेि (डच ध्वज के तहत प्रसतस्पर्ाध ) ने 22 से 24 ससतोंबर 2023 तक
िापाि िे सुिुिा में सुजुका इों टरनेशनल रे ससोंग कोसध में आयोसजत फॉमूािा 1 (F1)िापािी ग्रांि नप्रक्स 2023
जीता।
• मैकलेरन के सब्रसटश-बेक्कियम डर ाइवर लैंडो नॉररस (सब्रसटश ध्वज के तहत प्रसतस्पर्ाध ) दू सरे थिान पर रहे जबसक
मैकलेरन के ऑस्ट्र े सलयाई रे ससोंग डर ाइवर ऑस्कर सपयास्त्री ने तीसरा थिान हाससल सकया।
• यह 2023 सीज़न की 16 रे सोों में मैक्स वेरस्ट्ै पेन की 13िी ं िीत और रे ड बुल का सीज़न का 15वाों क्कखताब है ।
यह उनका िगातार दू सरा िापािी ग्रांि नप्रक्स खिताब भी है , उन्ोोंने इससे पहले 2022 में यह क्कखताब जीता
िा।

Report Errors in the PDF - ebooks@affairscloud.com Copyright 2014-2023 @ AffairsCloud.com 174


• यह मैक्स वेरस्ट्ै पेन की कररयर की 48वीों जीत भी है। यह उनके कररयर की 29वीों और 2023 सीज़न में 9वीों
पोल पोजीशन िी।
i.रे ि बुि टीम ने 2023 F1 सीज़न में अब तक 623 अोंक असजधत करके F1 िंस्ट्र रसा चैंनपयिनशप 2023 जीती
• यह रे ड बुल का लगातार दू सरा कोंस्ट्र रसध क्कखताब है और 2005 में खेल में प्रवेश करने के बाद से उनका छठा
क्कखताब भी है।
• रे ड बुल ने 2010,2011,2012,2013,2022 में कोंस्ट्र रसध का क्कखताब जीता
िोट- जापानी ग्राोंड सप्रक्स 2023 िेिोिो िारा प्रायोसजत है।

BOOKS AND AUTHORS


1. अगस्त 2023 में, िेंद्रीय नशक्षा मंत्री धमेंद्र प्रधाि िे िई नदल्ली, नदल्ली में "िेट्स मूि फॉरििा " िामि एि
अनभिि िॉनमि बुि िॉन्च िी।
निम्ननिखित में से GoI िे निस स्वायत्त संगठि/संगठि िे िॉनमि बुि नििनसत िी है ?
1)सोंयुक्त राष्ट्र बाल कोष
2)राष्ट्रीय शैसक्षक अनुसोंर्ान और प्रसशक्षि पररषद
3)सोंयुक्त राष्ट्र शैसक्षक, वैज्ञासनक और साोंस्कृसतक सोंगठन
4)1 और 2 दोनोों
5)2 और 3 दोनोों
उत्तर- 4)5)2 और 3 दोिों
स्पष्टीिरण:
केंद्रीय मोंत्री धमेंद्र प्रधाि, सशक्षा मोंत्रालय (MoE) ने नई सदल्ली, सदल्ली के कौशल भवन में एक समारोह में "िेट्स मूि
फॉरििा " नामक एक असभनव कॉसमक बुक लॉन्च की। इस बुक का उद्दे श्य पूरे भारत में स्कूल जाने वाले बच्चोों के स्वास्थ्य
और कल्ाि को बढावा दे ना है।
• कॉसमक बुक को स्कूल सशक्षा और साक्षरता सवभाग, MoE और सोंयुक्त राष्ट्र शैसक्षक, वैज्ञासनक और साोंस्कृसतक
सोंगठन (UNESCO) नई सदल्ली के तहत राष्ट्रीय शैसक्षक अनुसोंर्ान और प्रसशक्षि पररषद (NCERT) िारा सोंयुक्त
रूप से सवकससत सकया गया िा।
• यह बुक सहोंदी, अोंग्रेजी, अससमया, बोंगाली, गुजराती, कन्नड, मलयालम, मराठी, उसडया, तसमल और तेलुगु में सप्रोंट
और सडसजटल दोनोों प्रारूपोों में उपलब्ध है ।
• यह आयुष्मान भारत असभयान के तहत स्कूल स्वास्थ्य और कल्ाि कायधक्रम (SHWP) के कायाधन्वयन के पूरक
के सलए एक इों टरै क्करव और शैसक्षक उपकरि के रूप में सडज़ाइन सकया गया है।
• कॉसमक बुक का एक इलेररॉसनक सोंस्करि सशक्षा मोंत्रालय (MoE), NCERT, UNESCO, और DIKSHA
वेबसाइटोों पर भी उपलब्ध होगा।

2. उस व्यस्कक्तत्व का नाम बताइए लिसने हाि ही में (लसतंबर'23 में) नई लदल्ली में "पीपुल्स G20" नामक एक
ईबुक का अनावरण लकया, िो G20 की अध्यक्षता के दौरान भारत की यात्रा का व्यापक दस्तावेि है।
1)अश्चमत शाह (केंद्रीय गृह मंत्री)
2)सुिह्मण्यम ियशंकर (केंद्रीय श्चवदे श मंत्री)
3)अपूवग चंद्रा (सूचना एवं प्रसारि मंत्रालय के सश्चचव)
4)रािीव चन्द्रशेखर (इलेररॉश्चनक्स और सूचना प्रौद्योश्चगकी राज्य मंत्री)

Report Errors in the PDF - ebooks@affairscloud.com Copyright 2014-2023 @ AffairsCloud.com 175


5)V. मुरलीर्रन (श्चवदे श राज्य मंत्री)
उत्तर-3)अपूवा चंद्रा (सूचना एवं प्रसारण मंत्रािय के सलचव)
स्पष्टीकरण:
सूचना और प्रसारि मंत्रालय (MIB) के सलचव अपूवा चंद्रा ने नई श्चदल्ली में "पीपुल्स G20" नामक एक ईबुक का
अनावरि श्चकया, िो G20 की अध्यक्षता के दौरान भारत की यात्रा का व्यापक दस्तावेिीकरि प्रदान करती है।
i.यह ई-बुक प्रेस सूचना ब्यूरो (PIB) की अनुसंर्ान इकाई द्वारा तैयार की गई िी।
ii.ईबुक को तीन भागों में बां टा गया है:
• पहला भाग 9 से 10 श्चसतंबर 2023 तक नई श्चदल्ली में आयोश्चित ऐश्चतहाश्चसक G20 श्चशखर सम्मेलन पर केंश्चद्रत है।
• दू सरा भाग शेरपा और श्चवत्त टर ै क के तहत श्चवश्चभन्न कायग समूहों द्वारा आयोश्चित बैठकों का सारांश प्रस्तुत करता
है।
• तीसरा भाग एक मनोरम फोटो श्चनबंर् प्रस्तुत करता है िो भारत की अध्यक्षता के दौरान पूरे भारत में आयोश्चित
िन-भागीदारी कायगक्रमों (G20 के बारे में िागरूकता कायगक्रम) का सार दशागता है।

3. लसतंबर 2023 में, भारत के चुनाव आयोग (ECI) ने व्यवस्कस्थत मतदाता लशक्षा और चुनावी भागीदारी (SVEEP)
कायाक्रम के तहत बच्चों में चुनाव, मतदान और चुनावी प्रलक्रया के बारे में िागरूकता फैिाने के लिए एक कॉलमक
बुक ‘चाचा चौिरी और चुनावी दं गि’ िारी की।
कॉलमक बुक ECI & _____________ (कंपनी) की एक संयुक्त पहि है।
1)प्राि कॉश्चमक्स
2)मनोि कॉश्चमक्स
3)राि कॉश्चमक्स
4)चंपक कॉश्चमक्स
5)िायमंि कॉश्चमक्स
उत्तर- 1)प्राण कॉलमक्स
स्पष्टीकरण:
भारत के चुनाव आयोग (ECI) ने व्यवस्थित मतदाता श्चशक्षा और चुनावी भागीदारी (SVEEP) कायगक्रम के तहत चुनावों,
मतदान और चुनावी प्रश्चक्रया के बारे में िागरूकता फैलाने के श्चलए एक कॉश्चमक बुक ‘चाचा चौिरी और चुनावी
दं गि’ िारी की। बच्चों के श्चलए चुनावी प्रश्चक्रया कॉश्चमक में चुनावी पहलुओं पर 10 लघु कहाश्चनयां हैं।
• कॉश्चमक्स बुक ECI और प्राण कॉलमक्स की एक संयुक्त पहल है।
• पुस्तक में चाचा चौर्री, साबू, राका, र्माका श्चसंह और श्चबल्लू िैसे हास्य पात्रों का उपयोग श्चकया गया है।
• ECI बच्चों के बीच 30,000 प्रश्चतयां मुफ्त में श्चवतररत करे गा।कॉश्चमक बुक्स की प्रश्चतयां श्चिश्चिटल रूप से
भी उपलि होंगी।
• SVEEP मतदाता श्चशक्षा, मतदाता िागरूकता फैलाने और मतदाता साक्षरता को ब़िावा दे ने के श्चलए ECI
का एक प्रमुख कायगक्रम है।

4. 'िेशि िॉनिंग - होनिखस्ट्ि एप्रोच फॉर यूनियि पखब्लि सनिास िमीशि(UPSC) नसनिि सनिासेि
एग्जानमिेशि' पुस्ति निसिे नििी है , निसे नसतंबर 2023 में िेंद्रीय पृथ्वी निज्ञाि मंत्री निरे ि ररनििू द्वारा
िॉन्च निया गया था?
1)सोंदीप नोंदूरी
2)सोनल गोयल

Report Errors in the PDF - ebooks@affairscloud.com Copyright 2014-2023 @ AffairsCloud.com 176


3)अजय ससोंह यादव
4)श्रीलाल शुक्ल
5)सनयाज़ खान
उत्तर- 2)सोिि गोयि
स्पष्टीिरण:
केंद्रीय मोंत्री निरे ि ररनििू, पृथ्वी सवज्ञान मोंत्रालय ने नई सदल्ली, सदल्ली में सत्रपुरा भवन की आवासीय आयुक्त IAS
असर्कारी सोिि गोयि िारा सलक्कखत पुस्तक 'िेशि िॉनिंग - होनिखस्ट्ि एप्रोच फॉर यूनियि पखब्लि सनिास
िमीशि(UPSC) नसनिि सनिासेि एग्जानमिेशि’ का सवमोचन सकया है।
i.पुस्तक UPSC गाइड िारा प्रकासशत की गई िी।
ii.पुस्तक का सवमोचन नई सदल्ली, सदल्ली में कॉक्कन्स्टट्यूशन क्लब ऑफ इों सडया में सकया गया।

5. उस भारतीय िंपिी िा िाम बताइए निसिे हाि ही में (नसतंबर'23 में) इं टरोिक्शि टू िॉरपोरे ट फाइिेंस
पुस्ति िे िैनश्वि (छठे ) संस्करण िो िॉन्च िरिे िे निए ऑस्ट्रे निया िे िोिोंगोंग निश्वनिद्यािय (UOW) िे
साथ सहयोग निया है।
1)एक्ूमेन एकेडमी सलसमटे ड
2)NSE एकेडमी सलसमटे ड
3)रूरल टे क्नोलॉजी एों ड सबजनेस इनक्ूबेटर
4)एजुकेट गल्सध सलसमटे ड
5)आसवष्कार इन्वेस्ट्मेंट एकेडमी सलसमटे ड
उत्तर-2)NSE अिादमी निनमटे ि
स्पष्टीिरण:
वोलोोंगोोंग सवश्वसवद्यालय (UOW), ऑस्ट्रे निया ने NSE एिेिमी निनमटे ि के सहयोग से मुोंबई, महाराष्ट्र में NSE इों सडया
(नेशनल स्ट्ॉक एक्सचेंज ऑफ इों सडया सलसमटे ड) मुख्यालय में इों टरोडक्शन टू कॉरपोरे ट फाइनेंस (छठा सोंस्करि) पुस्तक
का वैसश्वक सोंस्करि लॉन्च सकया।
i.यह पुस्तक UOW पररसरोों (हाोंगकाों ग और दु बई में ) के प्रिम वषध के व्याख्याताओों िारा सलखी गई है।
ii.पुस्तक UAE, भारत, मलेसशया, हाोंगकाों ग और ऑस्ट्र े सलया में शेयरोों के मूल्ाोंकन और व्यापार के ससद्धाोंतोों को शासमल
करती है ; यह पुस्तक कॉपोरे ट सवत्त पररदृश्य की जसटलताओों और बारीसकयोों को समझने में मदद करती है।
iii.पुस्तक का सवमोचन NSE के प्रबोंर् सनदे शक और मुख्य कायधकारी असर्कारी (CEO) आशीषकुमार चौहान, UOW के
कुलपसत और अध्यक्ष प्रोफेसर पेटरीससया M. डे सवडसन और UOW के वैसश्वक ब्राोंड एों बेसडर एडम सगलसक्रस्ट् ने सकया।

OBITUARY
1. अगस्त 2023 में, पूिा _____________ (दे श) गोििीपर िाि िोंगब्लोएि, निन्ोंिे 2 Federation
Internationale de Football Association (FIFA) निश्व िप फाइिि में िेिा था, िा निधि हो गया।
1)पुतधगाल
2)नीदरलैंड
3)अजेंटीना
4)फ्राोंस
5)मोरिो

Report Errors in the PDF - ebooks@affairscloud.com Copyright 2014-2023 @ AffairsCloud.com 177


उत्तर- 2)िीदरिैंि
स्पष्टीिरण:
िीदरिैंि के पूवध गोलकीपर िाि िोंगब्लोएि, सजन्ोोंने नीदरलैंड के सलए 2 Federation Internationale de
Football Association (FIFA) निश्व िप फाइिि में खेला िा, का सनर्न हो गया। उनका जन्म 1940 में एम्स्ट्डध म,
नीदरलैंड में हुआ िा।
i.वह "िॉिििा ऑगेि" के नाम से मशहूर नीदरलैंड टीम का सहस्सा िे , जो 1974 सवश्व कप (पसिम जमधनी के क्कखलाफ)
और 1978 सवश्व कप (अजेंटीना के क्कखलाफ) में उपसवजेता रही िी।
ii.उन्ोोंने 717 मैचोों के बाद 44 साल की उम्र में सोंन्यास ले सलया। उनकी अोंसतम उपक्कथिसत ने उन्ें डच पेशेवर मैच में
भाग लेने वाला अब तक का सबसे उम्रदराज क्कखलाडी बना सदया।

2. ____________ (दे श) नक्रिेट टीम िे पूिा िप्ताि हीथ नहल्टि स्ट्र ीि िा नसतंबर 2023 में निधि हो गया।
1)ऑस्ट्र े सलया
2)न्यूजीलैंड
3)सजम्बाब्वे
4)इों ग्लैंड
5)वेस्ट्इों डीज
उत्तर-3)निम्बाब्वे
स्पष्टीिरण:
3 ससतोंबर 2023 को सजम्बाब्वे सक्रकेट टीम के पूवध कप्तान हीथ नहल्टि स्ट्र ीि (हीि स्ट्र ीक) का 49 वषध की आयु में
दसक्षि-पसिमी निम्बाब्वे के माटाबेलेलैंड में सनर्न हो गया। उनका जन्म 16 माचध 1974 को सज़म्बाब्वे के बुलावायो में
हुआ िा।
i.हीि स्ट्र ीक ने 1993 और 2005 के बीच 65 टे स्ट् और 189 एक सदवसीय अोंतराध ष्ट्रीय (ODI) में सजम्बाब्वे का
प्रसतसनसर्त्व सकया।
ii.उन्ोोंने 2005 में 32 साल की उम्र में अोंतरराष्ट्रीय सक्रकेट से सोंन्यास ले सलया और इों क्कग्लश काउों टी, वारसवकशायर
काउों टी सक्रकेट क्लब (CCC) के सलए खेलने चले गए।

3. निनटश िैज्ञानिि_____________, िो 1996 में िोि भेड िॉिी िे निमााता थे , िा नसतंबर 2023 में निधि हो
गया।
1)सटम कॉकररल
2)सर इयान सवल्ुट
3)माइकल सबयरपाकध
4)पीटर सहग्स
5)टॉम सकबल
उत्तर-2)सर इयाि नििुट
स्पष्टीिरण:
10 ससतोंबर, 2023 को, सब्रसटश वैज्ञासनक सर इयाि नििुट, वह व्यक्कक्त सजन्ोोंने 1996 में एसडनबगध सवश्वसवद्यालय के
रोससलन इों स्ट्ीट्यूट में िॉिी भेड बनाने वाली टीम का नेतृत्व सकया िा, का 79 वषध की आयु में सनर्न हो गया। उनका
जन्म 7 जुलाई 1944 में है म्पटन लुसी, यूनाइटे ड सकोंगडम (UK) में हुआ िा।।

Report Errors in the PDF - ebooks@affairscloud.com Copyright 2014-2023 @ AffairsCloud.com 178


• सब्रसटश भ्रूिसवज्ञानी इयान सवल्ुट एसडनबगध सवश्वसवद्यालय में स्कॉसटश सेंटर फॉर रीजनरे सटव मेसडससन के अध्यक्ष
िे।
• 1998 में, उन्ें लॉडध लॉयड ऑफ़ सकल्गरन पुरस्कार और अमेररकन एकेडमी ऑफ़ अचीवमेंट का गोडन प्लेट
पुरस्कार समला।
• उन्ें 1999 में ऑडध र ऑफ द सब्रसटश एम्पायर (OBE) का असर्कारी और 2002 में रॉयल सोसाइटी (FRS) का
फेलो सनयुक्त सकया गया िा।

4. नसतंबर 2023 में, पािरपॉइं ट सॉफ़्टिेयर िे सह-निमााताओं में से एि, ____________ िा निधि हो गया।
1)चाल्सध क्किोंट
2)पॉल G. एलन
3)रॉबटध गक्कस्कर्न्
4)डे सनस ऑक्कस्ट्न
5)िॉमस वॉटसन
उत्तर-4)िे निस ऑखस्ट्ि
स्पष्टीिरण:
पािरपॉइं ट सॉफ्टिेयर के सह-सनमाधताओों में से एक, अमेररकी सॉफ्टवेयर डे वलपर िे निस ऑखस्ट्ि का निधि हो
गया। उनका जन्म 28 मई, 1947 को सोंयुक्त राज्य अमेररका (USA) के पेंससल्वेसनया के सपट् सबगध शहर में हुआ िा। वह
1985 से 1996 में अपनी सेवासनवृसत्त तक पावरपॉइों ट के प्रािसमक डे वलपर िे।
• ऑक्कस्ट्न ने रॉबटा गखस्कि के साि समलकर सॉफ्टवेयर कोंपनी फोरिॉट के सलए पावरप्वाइों ट बनाया और इसे
1987 में जारी सकया।
• माइक्रोसॉफ्ट ने तब फमध का असर्ग्रहि सकया और पावरपॉइों ट को अपने ऑसफस प्रोग्राम्स के सूट में एकीकृत
सकया।

5. नसतंबर 2023 में, प्रनसि रािमागा और यातायात निशेषज्ञ और राष्टरीय पररिहि योििा और अिुसंधाि िेंद्र
(NATPAC) िे संस्थापि निदे शि _______________ िा निधि हो गया।
1)NS. श्रीसनवासन
2)सुन्दर डे सनयल
3)V.S. सोंजय कुमार
4)अरुि चोंद्रन
5)वासुदेवन N
उत्तर- 1)N.S. श्रीनििासि
स्पष्टीिरण:
डॉ. NS. प्रससद्ध राजमागध और यातायात सवशेषज्ञ और राष्ट्रीय पररवहन योजना और अनुसोंर्ान केंद्र (NATPAC) के
सोंथिापक सनदे शक श्रीसनवासन का 89 वषध की आयु में चेन्नई, तसमलनाडु (TN) में उनके आवास पर निधि हो गया।
उनका जन्म नल्लूर, TN में हुआ िा।
• उन्ोोंने 1963 से 1976 तक 13 वषों तक केंद्रीय सडक अनुसोंर्ान सोंथिान, नई सदल्ली के यातायात और पररवहन
प्रभाग के प्रमुख के रूप में कायध सकया।
• 1976 से 1988 तक, उन्ोोंने NATPAC के कायधकारी सनदे शक के रूप में कायध सकया। उन्ोोंने केरल रोड फोंड
बोडध (KRFB) के सलाहकार के रूप में भी काम सकया।

Report Errors in the PDF - ebooks@affairscloud.com Copyright 2014-2023 @ AffairsCloud.com 179


i.उन्ें "वह व्यक्कक्त सजसने सदल्ली की सडकोों को सुव्यवक्कथित सकया", "रोसवोंग ऑगेनाइजर", "यातायात योजना में अग्रिी"
और "भारत में यातायात इों जीसनयररों ग के जनक" के रूप में स्वीकार सकया गया है।
ii.पुरस्कार: उन्ें वषध 1973-74 के सलए इों स्ट्ीट्यूशन ऑफ इों जीसनयसध (इों सडया) के भारत के राष्ट्रपसत पुरस्कार ससहत कई
पुरस्कारोों से सम्मासनत सकया गया िा।

6. नसतंबर 2023 में, स्वतंत्र ____________ (दे श) िे पहिे राष्टरपनत िे रूप में िाया िरिे िािे नमनसाया स्नेगुर िा
निधि हो गया।
1)मोल्दोवा
2)बेलारूस
3)आमेसनया
4)बुल्गाररया
5)एस्ट्ोसनया
उत्तर- 1)मोल्दोिा
स्पष्टीिरण:
स्वतोंत्र मोल्दोिा के पहिे राष्टरपनत के रूप में कायध करने वाले नमनसाया स्नेगुर का सनर्न हो गया है । उनका जन्म 17
जनवरी 1940 को मोल्दोवा के िोरे स्ट्ी सजले के सत्रफानेस्ट्ी गाोंव में हुआ िा।
• मोल्दोवन सुप्रीम काउों ससल की मोंजूरी के बाद वह 1990 में मोल्दोवन सोसवयत सोशसलस्ट् ररपक्किक के राष्ट्रपसत
बने।
• उन्ोोंने 27 अगस्त 1991 को मोल्दोवा गिराज्य को स्वतोंत्रता की घोषिा तक पहुोंचाया।
• स्वतोंत्रता के बाद, 1992 में, स्नेगुर और तत्कालीन रूसी राष्ट्रपसत बोररस येल्तससन ने मोल्दोवा के अलग हुए
टर ाोंसडे सनस्ट्र (टर ाोंससनक्कस्ट्रया) क्षेत्र पर सोंघषध को रोकने के सलए एक समझौते पर हस्ताक्षर सकए।

7. प्रनसि फ़्िोनटस्ट् और पद्म श्री पुरस्कार नििेता नसखिि िटे सि िीिा, प्रनसि 'नसखिि नसस्ट्सा' में से छोटी
िा नसतंबर 2023 में निधि हो गया।
उन्ें निस िषा ििा िे क्षेत्र में पद्मश्री पुरस्कार से सम्मानित निया गया था?
1)2004
2)2010
3)2005
4)2012
5)2008
उत्तर- 1)2004
स्पष्टीिरण:
प्रससद्ध बांसुरीिादि और पद्म श्री पुरस्कार सवजेता नसखिि िटे सि िीिा, जो तसमलनाडु की प्रससद्ध 'ससक्किल
ससस्ट्सध' जोडी में छोटी िीों, का चेन्नई, तसमलनाडु (TN) में सनर्न हो गया। उनका जन्म 9 ससतोंबर 1938 को हुआ िा।
i.ससक्किल बहनोों ने बाोंसुरी पर कनाध टक सोंगीत बजाने की 'गायकी' शैली को ससद्ध सकया। ससक्किल नीला अपनी बहन
ससक्किल कुोंजुमानी के साि ऑल इों सडया रे सडयो (AIR) और दू रदशधन (DD) के शीषध ग्रेड कलाकार िे।
• 2004 में, भारत सरकार ने नीला को ििा िे क्षेत्र में उनके योगदान के सलए भारत के चौिे सवोच्च नागररक
पुरस्कार 'पद्म श्री' से सम्मासनत सकया।

Report Errors in the PDF - ebooks@affairscloud.com Copyright 2014-2023 @ AffairsCloud.com 180


• चेन्नई, तसमलनाडु की सोंगीत अकादमी, मद्रास ने 2002 में ससक्किल ससस्ट्सध को "सोंगीत कलासनसर्" से सम्मासनत
सकया।
• सोंस्कृसत मोंत्रालय के तहत, 1989 में, उन्ें कनाधटक सोंगीत (वाद्य) के सलए सोंगीत नाटक अकादमी पुरस्कार समला।
• उन्ें 1973 में तसमलनाडु इयाल, इसाई, नाटक मोंद्रम से 'कलईमामसि' पुरस्कार समला।

8. नसतंबर 2023 में, ____________, एि प्रनसि भारतीय-अमेररिी िेिि-नफि निमााता, पत्रिार और िेिि।
1)सुमा जोसन
2)सचत्रा बनजी
3)सोंजेना अोंशू
4)गीता मेहता
5)सुकन्या कृष्णन
उत्तर-4)गीता मेहता
स्पष्टीिरण:
प्रससद्ध भारतीय-अमेररिी िेखििा-नफि निमााता, पत्रकार और लेक्कखका गीता मेहता का नई सदल्ली में सनर्न हो
गया। उनका जन्म 1943 में एक स्वतोंत्र कायधकताध और उडीसा (अब ओसडशा) के तीसरे मुख्यमोंत्री बीजू पटनायक के घर
हुआ िा। वह ओसडशा के मुख्यमोंत्री (CM) नवीन पटनायक की बडी बहन हैं।
i.उन्ोोंने 'कमाध कोला', 'स्नेक्स एों ड लैडसध: क्कग्लम्पसेस ऑफ मॉडनध इों सडया', 'ए ररवर सूत्र', 'राज' और 'एटरनल गिेश: फ्रॉम
बिध टू रीबिध' ससहत कई सकताबें सलखी हैं ।
ii.2019 में, उन्ें सासहत्य और सशक्षा के क्षेत्र में उत्कृष्ट् योगदान के सलए "सवदे शी" श्रेिी में भारत के चौिे सवोच्च नागररक
सम्मान पद्म श्री से सम्मासनत सकया गया िा। उन्ोोंने मई 2019 के चुनावोों से पहले समय का हवाला दे ते हुए पुरस्कार लेने
से इनकार कर सदया।

9. लसतंबर 2023 में, लहं दुस्तानी शास्त्रीय गायक और ग्वालियर घराने के सबसे प्रमुि प्रलतपादकों में से एक
_______________ का लनिन हो गया।
1)उस्ताद राश्चशद खान
2)श्चकशोरी अमोनकर
3)माश्चलनी रािुरकर
4)शुभा मुद्गल
5)नश्चचकेता यकुंिी
उत्तर-3)मालिनी रािुरकर
स्पष्टीकरण:
मालिनी रािुरकर, एक प्रश्चसि लहं दुस्तानी शास्त्रीय गालयका और ग्वाश्चलयर घराने की सबसे प्रमुख प्रश्चतपादकों में से
एक का 82 वषग की आयु में हैदराबाद, तेलंगाना में श्चनर्न हो गया। उनका िन्म 1941 में रािथिान के अिमेर में हुआ
िा।
i.उन्हें श्चहंदुस्तानी गायन संगीत में उनके योगदान के श्चलए 2001 में प्रश्चतश्चष्ठत संगीत नाटक अकादमी पुरस्कार सश्चहत कई
पुरस्कार प्राप्त हुए हैं।
ii.उन्होंने भारत के प्रमुख संगीत समारोहों में प्रदशगन श्चकया है, श्चिनमें गुनीदास सम्मेलन (मुंबई), तानसेन समारोह
(ग्वाश्चलयर, मध्य प्रदे श), सवाई गंर्वग महोत्सव (पुिे, महाराष्ट्र) और शं कर लाल महोत्सव (श्चदल्ली) शाश्चमल हैं ।

Report Errors in the PDF - ebooks@affairscloud.com Copyright 2014-2023 @ AffairsCloud.com 181


10. भरतिाट्यम िृत्यांगिा और पद्म भूषण पुरस्कार नििेता सरोिा िैद्यिाथि िा नसतंबर 2023 में निधि हो
गया।
उन्ें ििा िे क्षेत्र में पद्म भूषण पुरस्कार निस िषा प्राप्त हुआ?
1)2021
2)2020
3)2013
4)2019
5)2005
उत्तर- 3)2013
स्पष्टीिरण:
भरतनाट्यम नृत्याोंगना और पद्म भूषि पुरस्कार से सम्मासनत सरोिा िैद्यिाथि का 86 वषध की आयु में सदल्ली में सलोंफोमा
रोगोों के कारि सनर्न हो गया।
• सरोजा वैद्यनािन को कला श्रेिी में 2002 में पद्म श्री और 2013 में पद्म भूषण से सम्मासनत सकया गया िा।
• उन्ें 2008 में सोंगीत नाटक अकादमी पुरस्कार और 2023 में सोंगीत नाटक अकादमी फ़ेलोसशप (अकादमी
रत्न) पुरस्कार समला।
• वह सदल्ली के सरकारी सासहत्य कला पररषद सम्मान (1991), तसमलनाडु के सरकारी कलईमामसि उपासर्
(1999) और पोलाची तसमल इसाई सोंगम से भारत कलाई सुदर की उपासर् की भी प्राप्तकताध हैं।
• वह मध्य प्रदे श सरकार के कासलदास सम्मान (2009-10) की प्राप्तकताध भी िीों।
i.सरोजा वैद्यनािन ने भरतनाट्यम और कनाधटक सोंगीत पर कई सकताबें सलखी हैं , सजनमें 'द क्लाससकल डाोंस ऑफ
इों सडया', 'भरतनाट्यम - एन इन-डे प्ि स्ट्डी', 'कनाधटक सोंगीतम' और 'द साइों स ऑफ भरतनाट्यम' शासमल हैं।

11. नसतंबर 2023 में, 2006 से 2015 ति ____________ (दे श) िे पूिा राष्टरपनत नियोनिायो िेपोनिटािो िा निधि
हो गया।
1)ब्राज़ील
2)स्पेन
3)फ्राोंस
4)इटली
5)जमधनी
उत्तर-4)इटिी
स्पष्टीिरण:
इटिी के पूवध राष्ट्रपसत (2006 से 2015)और इतालवी कम्युसनस्ट् पाटी के नेता नियोनिायो िेपोनिटािो का 98 वषध की
आयु में रोम, इटली में सनर्न हो गया। उनका जन्म 1925 में नेपल्स, इटली में हुआ िा।
i.वह आर्ुसनक इतालवी गिराज्य (सितीय सवश्व युद्ध के बाद लोकताोंसत्रक युग) के इसतहास में सबसे लोंबे समय तक सेवा
करने वाले और सबसे लोंबे समय तक जीसवत रहने वाले राष्ट्रपसत िे।
ii.वह 1945 में इटासलयन कम्युसनस्ट् पाटी (PCI) में शासमल हुए और 2006 में इटली के 11वें राष्ट्रपसत बने और बाद में
2013 में दू सरे कायधकाल के सलए सफर से चुने गए और बाद में 2015 में पद से हट गए।
iii.वह इस पद पर दोबारा चुने जाने वाले इटली के पहले और एकमात्र राष्ट्रपसत िे।

Report Errors in the PDF - ebooks@affairscloud.com Copyright 2014-2023 @ AffairsCloud.com 182


12. अिुभिी मियािम नफि निदे शि और राष्टरीय पुरस्कार नििेता िुििनट्टि गीिगीस (KG) िॉिा िा
नसतंबर 2023 में निधि हो गया।
उन्ोंिे 1976 में ___________ नफि िे निए मियािम में सिाश्रेष्ठ फीचर नफि िा राष्टरीय नफि पुरस्कार िीता।
1)इराकल
2)पोंचवडी पालम
3)ओनाप्पुडवा
4)स्वप्नदानम्
5)एडसमन्टे वेररयेलु
उत्तर-4)स्वप्नदािम्
स्पष्टीिरण:
अनुभवी मलयालम सफल् सनदे शक और राष्ट्रीय पुरस्कार सवजेता िुििनट्टि गीिगीस (KG) िॉिा का 77 वषध की आयु
में केरल के कोक्कच्च में किनाड के पास सनर्न हो गया। उनका जन्म 26 मई 1946 को केरल के पिानामसिट्टा में हुआ
िा।
i.उन्ोोंने 1976 में अपनी पहली सफल् 'स्वप्नदािम्' के सलए मलयालम में सिाश्रेष्ठ फीचर नफि का राष्टरीय नफि
पुरस्कार जीता। सफल् ने सवधश्रेष्ठ सफल् और सवधश्रेष्ठ पटकिा के सलए 2 केरल राज्य सफल् पुरस्कार भी जीते।
ii.मलयालम ससनेमा में उनके आजीवन योगदान के सलए उन्ें 2015 के JV डै सनयल पुरस्कार से सम्मासनत सकया गया िा।
• JC डे सनयल पुरस्कार मलयालम ससनेमा में योगदान के सलए सवोच्च पुरस्कार है ।

13. पद्म पुरस्कार नििेता __________________, निन्ें भारत िी हररत क्रांनत िे ििि िे रूप में िािा िाता है ,
िा नसतंबर 2023 में निधि हो गया।
1)सनरपख टु टेज
2)दु गेश पटे ल
3)सैम सपत्रोदा
4)वगीस कुररयन
5)MS स्वामीनािन
उत्तर-5)MS स्वामीिाथि
स्पष्टीिरण:
भारतीय कृसष वैज्ञासनक और पद्म पुरस्कार नििेता मनकोम्बु सोंबाससवन स्वामीनािन (MS स्वामीिाथि), सजन्ें भारत
िी हररत क्रांनत िे ििि के रूप में जाना जाता है , का 98 वषध की आयु में चेन्नई, तसमलनाडु (TN) में सनर्न हो गया।
उनका जन्म 7 अगस्त 1925 को तसमलनाडु के कुोंभकोिम में हुआ िा।
• उन्ोोंने 1972-79 तक भारतीय कृसष अनुसोंर्ान पररषद (ICAR) के महासनदे शक (DG) और भारत सरकार (GoI),
कृसष अनुसोंर्ान और सशक्षा सवभाग के ससचव के रूप में कायध सकया।
• उन्ोोंने 2007 से 2013 तक भारत के सोंसद सदस्य(MP)(राज्यसभा - मनोनीत) के रूप में भी कायध सकया है।
• उन्ोोंने 1988 में चेन्नई में एक अनुसोंर्ान केंद्र M. S. स्वामीनािन ररसचध फाउों डेशन (MSSRF) की थिापना की
और इसके अध्यक्ष, एमेररटस अध्यक्ष और मुख्य सलाहकार के रूप में कायध सकया।
i.पुरस्कार:GoI ने उन्ें सवज्ञान और इों जीसनयररों ग के सलए पद्म श्री (1967), पद्म भूषि (1972)और पद्म सवभूषि (1989) से
सम्मासनत सकया।

Report Errors in the PDF - ebooks@affairscloud.com Copyright 2014-2023 @ AffairsCloud.com 183


• 1960 के दशक के दौरान भारत में उच्च उपज दे ने वाली गेहूों और चावल की सकस्मोों के सवकास और शुरूआत
का नेतृत्व करने के सलए उन्ें पहला सवश्व खाद्य पु रस्कार सवजेता नासमत सकया गया िा। उन्ें 1987 में यह पुरस्कार
समला।

14. नसतंबर 2023 में, ________________ (िंपिी) िे पूिा िायािारी अध्यक्ष अनश्वि सूयािांत दािी िा निधि हो
गया।
1)एसशयन पेंट्स सलसमटे ड
2)इों सडगो पेंट्स सलसमटे ड
3)बजधर पेंट्स सलसमटे ड
4)शालीमार पेंट्स सलसमटे ड
5)शीनलैक पेंट्स सलसमटे ड
उत्तर- 1)एनशयि पेंट्स निनमटे ि
स्पष्टीिरण:
भारत की अग्रिी पेंट और सजावट कोंपनी एनशयि पेंट्स निनमटे ि के पूवध कायधकारी अध्यक्ष अनश्वि सूयािांत दािी
का 79 वषध की आयु में सनर्न हो गया है। उनका जन्म 26 ससतोंबर, 1944 को मुोंबई, महाराष्ट्र में हुआ िा।
वह एसशयन पेंट्स के सह-सोंथिापकोों में से एक सूयधकाों त दानी के बेटे हैं।
i.वह 1970 में एसशयन पेंट्स बोडध में शासमल हुए और 1998 से 2009 तक एसशयन पेंट्स के उपाध्यक्ष और प्रबोंर् सनदे शक
(MD) के पद पर रहे।
ii.उन्ोोंने 1997 में एसशयन पेंट्स सलसमटे ड और PPG इों डस्ट्र ीज इों क, USA के बीच गसठत एक सोंयुक्त उद्यम (JV) PPG
एसशयन पेंट्स के बोडध में सोंथिापक सनदे शक के रूप में कायध सकया।
iii.उन्ोोंने 2018 से 2021 तक एसशयन पेंट्स के बोडध में गैर-कायधकारी अध्यक्ष के रूप में कायध सकया। उन्ोोंने 2021 में
अपने पद से इस्तीफा दे सदया।
िोट: एसशयन पेंट्स अब 19 दे शोों में पररचालन के साि एसशया की सबसे बडी पेंट कोंपसनयोों में से एक है।

IMPORTANT DAYS
1. 31 अगस्त 2023 िो संयुक्त राष्टर (UN) द्वारा दु निया भर में अफ्रीिी मूि िे िोगों िे निए ________ अंतरााष्टरीय
नदिस 2023 मिाया गया।
1)पाोंचवाों
2)तीसरा
3)छठा
4)चौिा
5)दू सरा
उत्तर- 2)तीसरा
स्पष्टीिरण:
दु सनया भर में अफ्रीकी मूल के लोगोों के असार्ारि योगदान को बढावा दे ने के सलए सोंयुक्त राष्ट्र (UN) िा अफ्रीिी मूि
िे िोगों िे निए अंतरााष्टरीय नदिस हर साल 31 अगस्त को दु सनया भर में मनाया जाता है।
• इस सदन का उद्दे श्य गुलामी के सशकार अफ्रीकी लोगोों के वोंशजोों के मानवासर्कारोों को बढावा दे ना और उनकी
रक्षा करना भी है।

Report Errors in the PDF - ebooks@affairscloud.com Copyright 2014-2023 @ AffairsCloud.com 184


i.31 अगस्त 2023 को अफ्रीकी मूल के लोगोों के सलए तीसरा अोंतराध ष्ट्रीय सदवस मनाया जाता है ।
ii.अफ़्ीकी मू ल के लोगोों के सलए पहला अोंतराध ष्ट्रीय सदवस 31 अगस्त 2021 को मनाया गया।

2. पूरे भारत में निश्व संस्कृत नदिस/निश्व-संस्कृत-नदिम 2023 िब मिाया गया?


1)29 अगस्त 2023
2)27 अगस्त 2023
3)30 अगस्त 2023
4)28 अगस्त 2023
5)31 अगस्त 2023
उत्तर- 5)31 अगस्त 2023
स्पष्टीिरण:
निश्व संस्कृत नदिस, सजसे 'निश्व-संस्कृत-नदिम' के नाम से भी जाना जाता है , हर साल सहोंदू कैलेंडर में श्रावि महीने की
पूसिधमा सतसि (पूसिधमा के सदन) को दु सनया की सबसे पुरानी ज्ञात भाषाओों में से एक और सभी भाषाओों की माों सोंस्कृत का
सम्मान करने के सलए पूरे भारत में मनाया जाता है , जो लगभग 3500 साल पहले भारत में उत्पन्न हुआ िा।
• सवश्व सोंस्कृत सदवस 2023 31 अगस्त 2023 को मनाया गया।
• पहला सवश्व सोंस्कृत सदवस 1969 में मनाया गया िा।

3. पूरे भारत में राष्टरीय पोषण सप्ताह (NNW) 2023 िब मिाया गया?
1)22 से 28 ससतोंबर 2023
2)24 से 30 ससतोंबर 2023
3)1 से 7 ससतोंबर 2023
4)25 से 31 ससतोंबर 2023
5)23 से 29 ससतोंबर 2023
उत्तर- 3)1 से 7 नसतंबर 2023
स्पष्टीिरण:
मानव जीवन में पोषि और स्वास्थ्य की महत्वपूिध भूसमका के बारे में लोगोों के बीच जागरूकता बढाने और स्वथि और
स्वच्छ खाने की आदतोों को बढावा दे ने के सलए 1 से 7 नसतंबर, 2023 तक पूरे भारत में राष्टरीय पोषण सप्ताह
(NNW) 2023 मनाया गया।
i.मसहला एवों बाल सवकास मोंत्रालय (MWCD) पूरे भारत में ससतोंबर 2023 में छठा राष्ट्रीय पोषि माह मना रहा है ।
ii.इसका उद्दे श्य "सुपोसषत भारत, साक्षर भारत, सशक्त भारत" (नुसटर शन-ररच इों सडया, एडु केटे ड इों सडया, एम्पॉवडध
इों सडया) पर केंसद्रत िीम के माध्यम से पूरे भारत में पोषि सोंबोंर्ी समझ को बढावा दे ना है।
iii.पहिा NWW 1 से 7 ससतोंबर 1982 तक मनाया गया िा।

4. निश्व िाररयि नदिस 2023 "सस्ट्े निंग िोिोिट सेरर फॉर द प्रेिेंट एं ि फ्यूचर ििरे शि" निषय िे साथ
दु निया भर में िब मिाया गया था?
1)1 ससतोंबर 2023
2)3 ससतोंबर 2023
3)30 अगस्त 2023
4)2 ससतोंबर 2023

Report Errors in the PDF - ebooks@affairscloud.com Copyright 2014-2023 @ AffairsCloud.com 185


5)31 अगस्त 2023
उत्तर- 4)2 नसतंबर 2023
स्पष्टीिरण:
निश्व िाररयि नदिस 2023 2 नसतंबर 2023 को दु सनया भर में "सस्ट्े सनोंग कोकोनट सेरर फॉर द प्रेजेंट एों ड फ्यूचर
जनरे शन" सवषय के साि मनाया गया।
i.यह सदन अोंतराधष्ट्रीय नाररयल समु दाय (ICC) के थिापना सदवस की याद में भी मनाया जाता है , सजसे पहले एसशयाई और
प्रशाोंत नाररयल समुदाय (APCC) के नाम से जाना जाता िा, सजसकी थिापना 2 ससतोंबर 1969 को हुई िी।
ii.भारत में, यह सदन नाररयल सवकास बोडध (CDB) के सहयोग से केरल, तसमलनाडु , कनाधटक, गोवा, पसिम बोंगाल, आों ध्र
प्रदे श, उडीसा आसद राज्योों में मनाया जाता है।
iii.सवश्व नाररयल सदवस पहिी बार 2 ससतोंबर 2009 को एसशयाई और प्रशाोंत नाररयल समुदाय (APCC) िारा मनाया गया
िा।

5. नशक्षा मंत्रािय, भारत सरिार (GoI) िे _____________ से एि सप्ताह ति चििे िािा साक्षरता अनभयाि
मिाया।
1)23 से 30 अगस्त 2023
2)24 से 31 अगस्त 2023
3)1 से 8 ससतोंबर 2023
4)3 से 10 ससतोंबर 2023
5)2 से 9 ससतोंबर 2023
उत्तर- 3)1 से 8 नसतंबर 2023
स्पष्टीिरण:
नशक्षा मंत्रािय, भारत सरकार (GoI) 8 ससतोंबर 2023 को अोंतराध ष्ट्रीय साक्षरता सदवस के उपलक्ष् में 1 नसतंबर से 8
नसतंबर, 2023 तक एक सप्ताह का साक्षरता असभयान शुरू करने के सलए तैयार है।
i.इस असभयान का उद्दे श्य सभी सहतर्ारकोों/लाभासिधयोों/नागररकोों के बीच अोंडरस्ट्ैं सडों ग लाइफलॉन्क्ग लसनिंग फॉर ऑल इन
सोसाइटी (ULLAS)-नव भारत साक्षरता कायधक्रम के बारे में जागरूकता बढाना है।
ii.इस साक्षरता असभयान पहल का उद्दे श्य दे श के नागररकोों के बीच कतधव्यबोर् (कतधव्य-चेतना) और जनभागीदारी
(नागररक भागीदारी) की भावना को बढावा दे ना है।
iii.यह व्यापक प्रयास साक्षरता को लोकसप्रय बनाने और भारत को सावधभौसमक सशक्षा प्राप्त करने के लक्ष् के करीब लाने
का प्रयास करता है।

6. संयुक्त राष्टर (UN) अंतरााष्टरीय चैररटी नदिस 2023 दु निया भर में िब मिाया गया?
1)1 ससतोंबर 2023
2)4 ससतोंबर 2023
3)2 ससतोंबर 2023
4)5 ससतोंबर 2023
5)3 ससतोंबर 2023
उत्तर- 4)5 नसतंबर 2023

Report Errors in the PDF - ebooks@affairscloud.com Copyright 2014-2023 @ AffairsCloud.com 186


स्पष्टीिरण:
स्वयोंसेवक और परोपकारी गसतसवसर्योों के माध्यम से दू सरोों की मदद करने के सलए दु सनया भर के लोगोों, गैर-सरकारी
सोंगठनोों (NGO) और सहतर्ारकोों को सोंवेदनशील बनाने और सोंगसठत करने के सलए सोंयुक्त राष्ट्र (UN) अंतरााष्टरीय चैररटी
नदिस 2023 दु सनया भर में 5 नसतंबर 2023 को मनाया गया।
• यह सदन मदर टे रेसा के कायों ससहत र्माधिध सोंगठनोों और व्यक्कक्तयोों के प्रयासोों को मान्यता दे ता है।
• 17 सदसोंबर 2012 को, सोंयुक्त राष्ट्र महासभा (UNGA) ने सोंकल्प A/RES/67/105 को अपनाया और हर साल
5 ससतोंबर, मदर टे रेसा की मृत्यु की सालसगरह को अोंतराध ष्ट्रीय चैररटी सदवस के रूप में नासमत सकया।
• मदर टे रेसा, सजन्ें कलकत्ता की सोंत टे रेसा भी कहा जाता है , का सनर्न 5 ससतोंबर 1997 को हुआ िा।
• पहिा अोंतराध ष्ट्रीय चैररटी सदवस 5 ससतोंबर 2013 को मनाया गया िा।

7. राष्टरीय नशक्षि नदिस 2023 िे संबंध में निम्ननिखित में से िौि सा/से नबंदु "सही" है /हैं ?
A) भारत रत्न िॉ. सिापल्ली राधािृष्णि िी ियंती मिािे िे निए 5 नसतंबर 2023 िो पूरे भारत में राष्टरीय नशक्षि
नदिस 2023 मिाया गया, निन्ोंिे स्वतंत्र भारत िे पहिे उपराष्टरपनत और दू सरे राष्टरपनत िे रूप में िाया निया।
B) राष्टरीय नशक्षि नदिस 2023 िे अिसर पर, भारत िी राष्टरपनत द्रौपदी मुमूा िे पूरे भारत में 75 चयनित नशक्षिों
िो राष्टरीय नशक्षि पुरस्कार 2023 से सम्मानित निया।
C) 75 सम्मानित िोगों में 50 स्कूि नशक्षि, उच्च नशक्षा िे 13 नशक्षि और नशक्षा मंत्रािय और गृह मंत्रािय िे
12 नशक्षि शानमि थे।
1)केवल A
2)केवल A & B
3)केवल B & C
4)केवल A & C
5)सभी A, B & C
उत्तर- 2)िेिि A & B
स्पष्टीिरण:
राष्टरीय नशक्षि नदिस 2023 पूरे भारत में 5 नसतंबर 2023 को प्रससद्ध सशक्षासवद् , भारत रत्न डॉ. सवधपल्ली रार्ाकृष्णन
की जयोंती मनाने के सलए मनाया गया, सजन्ोोंने स्वतोंत्र भारत के पहले उपराष्ट्रपसत और दू सरे राष्ट्रपसत के रूप में कायध
सकया।
• यह सदन पहली बार 5 ससतोंबर 1962 को डॉ. S रार्ाकृष्णन के 77िें िन्मनदि पर मनाया गया िा।
• तब से, 5 ससतोंबर को पूरे भारत में राष्ट्रीय सशक्षक सदवस के रूप में मनाया जाने लगा।
i.राष्टरीय नशक्षि नदिस 2023 (5 ससतोंबर 2023)के अवसर पर, भारत की राष्ट्रपसत द्रौपदी मुमूध ने सवज्ञान भवन, नई
सदल्ली, सदल्ली में भारत भर के 75 चयनित सशक्षकोों को राष्टरीय नशक्षि पुरस्कार 2023 से सम्मासनत सकया।
ii.75 सम्मानित लोगोों में 50 स्कूल सशक्षक, उच्च सशक्षा के 13 सशक्षक और िौशि नििास और उद्यनमता मंत्रािय
(MSDE) के 12 सशक्षक शासमल िे।
• प्रत्येक पुरस्कार में योग्यता प्रमाि पत्र, 50,000, रुपये का नकद पुरस्कार और एक रजत पदक सदया जाता है।

8. संयुक्त राष्टर (UN) िीिे आसमाि िे निए स्वच्छ िायु िा अंतरााष्टरीय नदिस 2023 दु निया भर में 'टु गेदर फॉर
िीि एयर' िी थीम िे साथ िब मिाया गया था?
1)3 ससतोंबर 2023
2)4 ससतोंबर 2023

Report Errors in the PDF - ebooks@affairscloud.com Copyright 2014-2023 @ AffairsCloud.com 187


3)5 ससतोंबर 2023
4)6 ससतोंबर 2023
5)7 ससतोंबर 2023
उत्तर- 5)7 नसतंबर 2023
स्पष्टीिरण:
सोंयुक्त राष्ट्र (UN) नीले आसमान के सलए स्वच्छ वायु का अोंतराध ष्ट्रीय सदवस सजसे सवश्व स्वच्छ वायु सदवस 2023 के रूप में
भी जाना जाता है , 7 नसतंबर 2023 को "टु गेदर फॉर िीि एयर" की िीम के साि दु सनया भर में मनाया गया।
i.7 ससतोंबर 2023 को नीले आसमान के सलए स्वच्छ वायु के अों तराध ष्ट्रीय सदवस का चौिा वासषधक उत्सव मनाया जाता है ।
ii.सवश्व स्वास्थ्य सोंगठन (WHO) ने 30 से असर्क अोंतरराष्ट्रीय सवशेषज्ञोों के सहयोग से सवशेष रूप से स्वास्थ्य कायधकताध ओों
के सलए तैयार पहला वायु प्रदू षि और स्वास्थ्य प्रसशक्षि टू लसकट (APHT) सवकससत सकया है। APHT को 2023 के अोंत
में जारी करने की तैयारी है।

9. संयुक्त राष्टर शैनक्षि, िैज्ञानिि और सांस्कृनति संगठि (UNESCO) द्वारा दु निया भर में अंतरााष्टरीय साक्षरता
नदिस (ILD) 2023 िब मिाया गया?
1)7 ससतोंबर 2023
2)5 ससतोंबर 2023
3)9 ससतोंबर 2023
4)6 ससतोंबर 2023
5)8 ससतोंबर 2023
उत्तर- 5)8 नसतंबर 2023
स्पष्टीिरण:
सोंयुक्त राष्ट्र शैसक्षक, वैज्ञासनक और साोंस्कृसतक सोंगठन (UNESCO) का अों तराध ष्ट्रीय साक्षरता सदवस (ILD)2023 दु सनया
भर में 8 नसतंबर 2023 को मनाया गया, तासक जनता को गररमा और मानवासर्कारोों के मामले के रूप में साक्षरता के
महत्व की याद सदलाई जा सके, और असर्क साक्षर और सटकाऊ समाज की सदशा में साक्षरता एजेंडे को आगे बढाया जा
सके।
• ILD िी 2023 िी थीम 'प्रमोसटों ग सलटरे सी फॉर ए वडध इन टर ाोंजीशन: सबक्कडोंग द फाउों डेशन फॉर सस्ट्े नेबल
एों ड पीसफुल सोसाइटीज' है।
• पहिा अोंतराध ष्ट्रीय साक्षरता सदवस 8 ससतोंबर 1967 को मनाया गया िा।
• 15 नवोंबर 2019 को पेररस (फ्राोंस) में UNESCO के 40वें आम सम्मेलन में, सदस्य राज्योों ने युवा और वयस्क
साक्षरता (2020-2025)के सलए एक नई UNESCO रिनीसत अपनाने पर सहमसत व्यक्त की।
• ILD 2023 (8 ससतोंबर 2023 को) के अवसर पर, UNESCO ने UNESCO अोंतराध ष्ट्रीय साक्षरता पुरस्कार (ILP) के
सवजेता की घोषिा की, जो प्रभावी और नवीन प्रिाओों को बढावा दे ने और गसतशील साक्षर समाजोों को प्रोत्सासहत
करने के सलए दु सनया भर में साक्षरता के क्षेत्र में उत्कृष्ट्ता और नवाचार को पुरस्कृत करता है।
• वतधमान में, 2 UNESCO ILP , UNESCO सकोंग सेजोोंग साक्षरता पुरस्कार (3 पुरस्कार) और साक्षरता के सलए
UNESCO कन्क्फ्यूसशयस पुरस्कार (3 पुरस्कार) है ।

10. दु निया भर में निश्व भौनति नचनित्सा (PT) नदिस 2023 िब मिाया गया?
1)4 ससतोंबर 2023
2)6 ससतोंबर 2023

Report Errors in the PDF - ebooks@affairscloud.com Copyright 2014-2023 @ AffairsCloud.com 188


3)8 ससतोंबर 2023
4)5 ससतोंबर 2023
5)7 ससतोंबर 2023
उत्तर- 3)8 नसतंबर 2023
स्पष्टीिरण:
निश्व भौनति नचनित्सा (PT) नदिस, सजसे सवश्व सफसजयोिेरेपी सदवस, 2023 के रूप में भी जाना जाता है , हर साल 8
नसतंबर 2023 को दु सनया भर में भौसतक सचसकत्सा के महत्व और दु सनया भर में व्यक्कक्तयोों के स्वास्थ्य, कल्ाि और जीवन
की गुिवत्ता में सुर्ार में इसकी भूसमका के बारे में जागरूकता बढाने के सलए मनाया जाता है।
• सवश्व भौसतक सचसकत्सा सदवस के 2023 अनभयाि िा निषय "आिधराइसटस एों ड द रोल ऑफ़ सफसजयोिेरेसपस्ट्"
है।
• 8 ससतोंबर की तारीख उस सदन को सचसह्नत करती है सजस सदन 1951 में WCPT की थिापना की गई िी।

11. पूरे भारत में लहमािय लदवस (लहमाियन लदवस) 2023 कब मनाया गया?
1) 11 श्चसतंबर 2023
2) 9 श्चसतंबर 2023
3) 12 श्चसतंबर 2023
4) 8 श्चसतंबर 2023
5) 7 श्चसतंबर 2023
उत्तर- 2) 9 लसतंबर 2023
स्पष्टीकरण:
लहमािय लदवस, श्चिसे लहमाियन लदवस के रूप में भी िाना िाता है , श्चहमालयी पाररस्थिश्चतकी तंत्र और क्षेत्र के
संरक्षि के उद्दे श्य से हर साल 9 लसतंबर को पूरे भारत में, श्चवशेष रूप से उत्तरािंि राज्य में मनाया िाता है।
• 9 लसतंबर 2023 को लहमािय लदवस का 14वां संिरण मनाया गया।
i.2014 में, उत्तराखंि के तत्कालीन मुख्यमंत्री (CM) हरीश रावत ने आश्चर्काररक तौर पर हर साल 9 श्चसतंबर को
'लहमािय लदवस' के रूप में घोश्चषत श्चकया।
ii.पहिा श्चहमालय श्चदवस 9 लसतंबर 2015 को मनाया गया िा।

12. 9 लसतंबर 2023 को दु लनया भर में मनाए गए लवश्व प्राथलमक लचलकत्सा लदवस 2023 का लवषय क्या था?
1) फस्ट्ग ऐि एं ि एक्सक्लूिेि पीपल
2) फस्ट्ग ऐि सेव्स लाइफ्स
3) लाइफलॉन्ग फस्ट्ग ऐि
4) फस्ट्ग ऐि इन द श्चिश्चिटल वर्ल्ग
5) फस्ट्ग ऐि एं ि रोि सेफ्टी
उत्तर- 4) फस्ट्ा ऐि इन द लिलिटि वल्डा
स्पष्टीकरण:
िीवन बचाने और चोटों को रोकने में प्रािश्चमक श्चचश्चकत्सा प्रश्चशक्षि के महत्व के बारे में िागरूकता पैदा करने के श्चलए
श्चवश्व प्रािश्चमक श्चचश्चकत्सा श्चदवस हर साल श्चसतंबर के दू सरे शश्चनवार को दु श्चनया भर में मनाया िाता है ।इस श्चदन का
उद्दे श्य आम िनता को बुश्चनयादी प्रािश्चमक श्चचश्चकत्सा कौशल सीखने के श्चलए प्रोत्साश्चहत करना भी है ।
• श्चवश्व प्रािश्चमक श्चचश्चकत्सा श्चदवस 2023 9 लसतंबर 2023 को मनाया गया।

Report Errors in the PDF - ebooks@affairscloud.com Copyright 2014-2023 @ AffairsCloud.com 189


• श्चवश्व प्रािश्चमक श्चचश्चकत्सा श्चदवस 2023 का श्चवषय "फस्ट्ा एि इन द लिलिटि वल्डा " है।
• श्चवषय श्चिश्चिटल टू ल (मोबाइल ऐप और ऑनलाइन वीश्चियो/पाठ्यक्रम) और संसार्नों के उपयोग को ब़िावा
दे ती है िो प्रािश्चमक श्चचश्चकत्सा श्चशक्षा का समिगन करते हैं।

13. संयुक्त राष्टर (UN) द्वारा लशक्षा को हमिे से बचाने के लिए अंतरााष्टरीय लदवस 2023 दु लनया भर में कब
मनाया गया?
1) 12 श्चसतंबर 2023
2) 9 श्चसतंबर 2023
3) 11 श्चसतंबर 2023
4) 8 श्चसतंबर 2023
5) 10 श्चसतंबर 2023
उत्तर- 2) 9 लसतंबर 2023
स्पष्टीकरण:
दु श्चनया भर में स्कूलों, छात्रों, श्चशक्षा कश्चमगयों और श्चशक्षकों पर हमलों के गंभीर मुद्दे के बारे में िागरूकता ब़िाने के श्चलए,
संयुक्त राष्ट्र (UN) लशक्षा को हमिे से बचाने के लिए अंतरााष्टरीय लदवस हर साल 9 लसतंबर को दु श्चनया भर में मनाया
िाता है।
• 9 श्चसतंबर 2023 को चौथा श्चशक्षा को हमले से बचाने के श्चलए अंतरााष्टरीय लदवस मनाया िाता है।
• पहिा श्चशक्षा को हमले से बचाने के श्चलए अं तराग ष्ट्रीय श्चदवस 9 श्चसतंबर 2020 को मनाया गया।
कायाक्रम 2023:
i.श्चशक्षा को हमले से बचाने के श्चलए चौिे अंतरागष्ट्रीय श्चदवस को श्चचश्चित करने के श्चलए, UNESCO, कतर राज्य, एिुकेशन
एबव ऑल (EAA) फाउं िेशन, UNICEF और संयुक्त राष्ट्र के वैश्चश्वक संचार श्चवभाग द्वारा एक मुख्य उच्च-स्तरीय
समारोह का सह-आयोिन श्चकया िाता है।
• यह कायगक्रम "िस्कस्ट्स फॉर लवस्कक्टम्स" की थीम पर केंश्चद्रत, श्चशक्षा की सुरक्षा के श्चलए प्रेरक कारग वाई के श्चलए
समश्चपगत एक सप्ताह तक चलने वाले स्मरिोत्सव का श्चहस्सा है।

14. दु लनया भर में लवश्व आत्महत्या रोकथाम लदवस (WSPD)2023 कब मनाया गया?
1) 8 श्चसतंबर 2023
2) 12 श्चसतंबर 2023
3) 10 श्चसतंबर 2023
4) 9 श्चसतंबर 2023
5) 11 श्चसतंबर 2023
उत्तर- 3) 10 लसतंबर 2023
स्पष्टीकरण:
श्चवश्व आत्महत्या रोकिाम (WSPD) श्चदवस प्रश्चतवषग 10 लसतंबर को दु श्चनया भर में मनाया िाता है ताश्चक िागरूकता पैदा
की िा सके श्चक आत्महत्या को रोका िा सकता है और आत्महत्या के मुद्दों पर ध्यान आकश्चषगत श्चकया िा सके और
आत्महत्या के आसपास के कलंक को कम श्चकया िा सके।
• वाश्चषगक उत्सव का आयोिन इं टरनेशनल एसोश्चसएशन फॉर सुसाइि श्चप्रवेंशन (IASP) द्वारा श्चकया िाता है और
श्चवश्व स्वास्थ्य संगठन (WHO) द्वारा सह-प्रायोश्चित श्चकया िाता है।
• 2021-2023 तक श्चवश्व आत्महत्या रोकिाम श्चदवस की श्चत्रवाश्चषगक श्चवषय "श्चक्रएश्चटंग होप थ्रू एक्शन" है।

Report Errors in the PDF - ebooks@affairscloud.com Copyright 2014-2023 @ AffairsCloud.com 190


• भारत अपनी राष्ट्रीय आत्महत्या रोकिाम रिनीश्चत (NSPS) के साि 2020 की तुलना में 2030 तक आत्महत्या
मृत्यु दर को 10% तक कम करे गा।

15. पूरे भारत में राष्टरीय िि शहीद नदिस 2023 िब मिाया गया?
1)11 ससतोंबर 2023
2)9 ससतोंबर 2023
3)12 ससतोंबर 2023
4)8 ससतोंबर 2023
5)10 ससतोंबर 2023
उत्तर- 1)11 नसतंबर 2023
स्पष्टीिरण:
भारत में वनोों और वन्यजीवोों की रक्षा के सलए अपने जीवन का बसलदान दे ने वाले लोगोों को याद करने और उनका सम्मान
करने के सलए 11 नसतंबर 2023 को पूरे भारत में राष्टरीय िि शहीद नदिस 2023 मनाया गया, जो ग्रह की भलाई के
सलए महत्वपूिध हैं।
i.इस सदन को 1730 में खेजडली नरसोंहार के वासषधक स्मरिोत्सव के रूप में भी मनाया जाता है।
ii.2023 कायधक्रम: वनोों और वन्यजीवोों की रक्षा के सलए अपने प्रािोों की आहुसत दे ने वाले वनवाससयोों की याद में, राष्ट्रीय
वन शहीद सदवस 11 ससतोंबर, 2023 को फॉरे स्ट्र मेमोररयल, ब्रैंसडस रोड, वन अनुसोंर्ान सोंथिान (FRI), उत्तराखोंड के
दे हरादू न पररसर में मनाया गया।

16. संयुक्त राष्टर (UN) अंतरााष्टरीय दनक्षण-दनक्षण सहयोग नदिस 2023 िो दु निया भर में "सॉनििे ररटी, इखिटी,
एं ि पाटा िरनशप: अििॉनिंग साउथ -साउथ िोऑपरे शि टू अचीि द SDG" निषय िे साथ िब मिाया गया?
1)10 ससतोंबर 2023
2)12 ससतोंबर 2023
3)9 ससतोंबर 2023
4)11 ससतोंबर 2023
5)8 ससतोंबर 2023
उत्तर- 2)12 नसतंबर 2023
स्पष्टीिरण:
संयुक्त राष्टर (UN) दसक्षि-दसक्षि सहयोग के सलए अोंतराध ष्ट्रीय सदवस 2023 दु सनया भर में 12 नसतंबर 2023 को
"सॉसलडे ररटी, इक्कक्वटी, एों ड पाटध नरसशप: अनलॉसकोंग साउि -साउि कोऑपरे शन टू अचीव द SDG" निषय के साि मनाया
गया।
• सवषय सतत सवकास लक्ष्ोों (SDG) के UN 2030 एजेंडे को प्राप्त करने में SSC के महत्व पर प्रकाश डालता है ।
• SSC के सलए पहला UN सदवस 19 सदसोंबर 2004 को मनाया गया िा।
• दसक्षि-दसक्षि सहयोग के सलए सोंयुक्त राष्ट्र कायाध लय (UNOSSC) ने UN में श्रीलोंका के थिायी समशन के साि
समलकर 12 ससतोंबर 2023 को न्यूयॉकध, सोंयुक्त राज्य अमेररका में SSC 2023-2025 पर उच्च स्तरीय ससमसत के
अध्यक्ष के रूप में एक उच्च स्तरीय पैनल चचाध का आयोजन सकया िा।

Report Errors in the PDF - ebooks@affairscloud.com Copyright 2014-2023 @ AffairsCloud.com 191


17. निश्व िचेि मस्कुिर निस्ट्र ॉफी नदिस 2023 िो दु निया भर में "ड्युचेि: िेनिंग बैररयसा" िी थीम िे साथ
िब मिाया गया?
1)8 ससतोंबर 2023
2)10 ससतोंबर 2023
3)7 ससतोंबर 2023
4)11 ससतोंबर 2023
5)9 ससतोंबर 2023
उत्तर- 3)7 नसतंबर 2023
स्पष्टीिरण:
सवश्व ड्यूचेन मस्कुलर सडस्ट्र ॉफी सदवस सजसे सवश्व ड्यूचेन जागरूकता सदवस (WDAD) 2023 के रूप में भी जाना
जाता है , 7 नसतंबर 2023 को दु सनया भर में “ड्युचेन: ब्रेसकोंग बैररयसध” की थीम के साि मनाया गया।
• WDAD जागरूकता भी बढाता है और ड्यूशेन और बेकर मस्कुलर सडस्ट्र ॉफी (DMD/BMD) से पीसडत लोगोों
के जीवन को बेहतर बनाने के सलए कारध वाई को प्रेररत करता है।
• 7 ससतोंबर 2023 को 10वाों सवश्व ड्यूचेन जागरूकता सदवस मनाया गया और पहला सवश्व ड्यूचेन जागरूकता
सदवस 2014 में मनाया गया िा।
• सवकलाोंग व्यक्कक्तयोों के सशक्कक्तकरि सवभाग (DEPwD) और सामासजक न्याय और असर्काररता मोंत्रालय
(MoSJE) ने 7 ससतोंबर 2023 को सवश्व ड्यूशेन मस्कुलर सडस्ट्र ॉफी (DMD) के बारे में जागरूकता बढाने के
उद्दे श्य से सवश्व ड्यूशेन मस्कुलर सडस्ट्र ॉफी सदवस मनाया।

18. पूरे भारत में नहंदी नदिस (नहंदी िे ) 2023 िब मिाया गया?
1)12 ससतोंबर 2023
2)14 ससतोंबर 2023
3)11 ससतोंबर 2023
4)13 ससतोंबर 2023
5)10 ससतोंबर 2023
उत्तर- 2)14 नसतंबर 2023
स्पष्टीिरण:
भारत की आसर्काररक भाषाओों में से एक के रूप में दे वनागरी सलसप में सलखी गई सहोंदी को अपनाने या घोसषत करने के
उपलक्ष् में 14 नसतंबर 2023 को पूरे भारत में सहोंदी सदवस (नहंदी िे ) 2023 मनाया गया।
• यह सदवस भारत की भाषाई सवसवर्ता का जश्न मनाने और उसे पहचानने के सलए मनाया जाता है , सजसमें भारत
की पहचान में सहोंदी भाषा के महत्व पर ध्यान सदया जाता है।
• पहिा सहोंदी सदवस 14 ससतोंबर 1953 को मनाया गया िा।
i.MHA के तहत राजभाषा सवभाग ने मूल रूप से सहोंदी में सकताबें सलखने को प्रोत्सासहत करने और राजभाषा को बढावा
दे ने के सलए 'राजभाषा गौरव पुरस्कार योजना' नाम से एक पुरस्कार योजना शुरू की।
ii. भाषा सवभाग ने राजभाषा कायाधन्वयन को प्रोत्सासहत करने के सलए पुरस्कार योजना 'राजभाषा कीसतध सपरास्कर' भी
शुरू की।

Report Errors in the PDF - ebooks@affairscloud.com Copyright 2014-2023 @ AffairsCloud.com 192


19. संयुक्त राष्टर (UN) अंतरााष्टरीय िोितंत्र नदिस 2023 "एम्पोिेररं ग द िेक्स्ट ििरे शि " िी थीम िे साथ दु निया
भर में िब मिाया गया?
1)13 ससतोंबर 2023
2)15 ससतोंबर 2023
3)12 ससतोंबर 2023
4)14 ससतोंबर 2023
5)11 ससतोंबर 2023
उत्तर- 2)15 नसतंबर 2023
स्पष्टीिरण:
सोंयुक्त राष्ट्र (UN) अंतरााष्टरीय िोितंत्र नदिस 2023 15 नसतंबर 2023 िो दु सनया भर में "एम्पोवेररों ग द नेक्स्ट
जनरे शन" की थीम के साि मनाया गया।
i.पहिा अोंतराधष्ट्रीय लोकतोंत्र सदवस 15 ससतोंबर 2008 को मनाया गया िा।
ii.15 ससतोंबर 1997 को, राष्ट्रीय सोंसदोों के एक अोंतरराष्ट्रीय सोंगठन, अोंतर-सोंसदीय सोंघ (IPU) ने लोकतोंत्र पर सावधभौसमक
घोषिा को अपनाया।
iii.2023 अोंतराधष्ट्रीय लोकतोंत्र सदवस सोंयुक्त राज्य अमेररका (USA) के न्यूयॉकध में सों युक्त राष्ट्र मुख्यालय में प्रायोसजत सतत
सवकास लक्ष् (SDG) मोंडप, नॉिध लॉन में एक लाइव कायधक्रम के साि मनाया गया।

20. 15 नसतंबर 2023 िो पूरे भारत में मिाए गए राष्टरीय इं िीनियसा नदिस 2023 िा निषय क्ा था?
1)इों जीसनयररों ग फॉर चेंज
2)इों जीसनयररों ग फॉर ए सेल्फ -रे सलअन्त इों सडया
3)इों जीसनयररों ग फॉर ए हे अल्थी प्लेनेट
4)इों जीसनयररों ग चैलेंजेज फॉर नॉलेज एरा
5)इों जीसनयररों ग फॉर ए सस्ट्े नेबल फ्यूचर
उत्तर- 5)इं िीनियररं ग फॉर ए सस्ट्े िेबि फ्यूचर
स्पष्टीिरण:
भारत रत्न सर मोक्षगुोंडम सवश्वेश्वरै या (15 ससतोंबर 1861)की जयोंती मनाने के सलए 'इं िीनियररं ग फॉर ए सस्ट्े िेबि
फ्यूचर' सवषय के साि 15 नसतंबर 2023 को पूरे भारत में राष्टरीय इं िीनियसा नदिस 2023 मनाया गया, सजन्ें भारत
में महानतम इों जीसनयरोों में से एक माना जाता है। ।
• इों स्ट्ीट्यूशन ऑफ इों जीसनयसध इों सडया (IEI) के अनुसार, समाज में उनके प्रमुख योगदान के सलए उन्ें सर MV
और 'आर्ुसनक मैसूर का जनक' भी कहा जाता है और उन्ें "भारत में आसिधक सनयोजन का अग्रदू त" भी कहा
जाता िा।
• 1955 में, इों जीसनयररों ग के क्षेत्र में उनके महत्वपूिध योगदान और राष्ट्र के प्रसत उनकी सेवा के सलए भारत सरकार
ने उन्ें 'भारत रत्न' से सम्मासनत सकया।
• पुस्तकें: ररकोंस्ट्र क्करोंग इों सडया (1920); प्लाोंड इकोनॉमी फॉर इों सडया (1934); मोमोयसध ऑफ़ माय वसकिंग लाइफ
(1951).

Report Errors in the PDF - ebooks@affairscloud.com Copyright 2014-2023 @ AffairsCloud.com 193


21. 16 लसतंबर 2023 को संयुक्त राष्टर द्वारा दु लनया भर में मनाए गए ओिोन परत संरक्षण के लिए अंतरााष्टरीय
लदवस 2023 का लवषय क्या था?
1)मॉक्कन्टरयल प्रोटोकॉल – कीसपोंग अस, आवर फ़ूड एों ड वैक्सीर्न् कूल
2)ग्लोबल कोऑपरे शन प्रोटे क्करोंग लाइफ ऑन अिध
3)मॉक्कन्टरयल प्रोटोकॉल: सफक्कक्सोंग द ओजोन लेयर एों ड रे डूससोंग क्लाइमेट चेंज
4)ओजोन फॉर लाइफ - 35 इयसध ऑफ ओजोन लेयर प्रोटे क्शन
5)मॉक्कन्टरयल प्रोटोकॉल: 32 इयसध एों ड हीसलोंग
उत्तर- 3)मॉखरर यि प्रोटोिॉि: नफखक्संग द ओिोि िेयर एं ि रे िूनसंग िाइमेट चेंि
स्पष्टीिरण:
सोंयुक्त राष्ट्र (UN) निश्व ओिोि नदिस या ओजोन परत के सोंरक्षि के सलए अोंतराध ष्ट्रीय सदवस 2023 16 ससतोंबर 2023
को “मॉखरर यि प्रोटोिॉि: नफखक्संग द ओिोि िेयर एं ि रे िूनसंग िाइमेट चेंि" सवषय के साि मनाया गया। इस
सदन को "वैसश्वक ओजोन सदवस" के रूप में भी जाना जाता है।
• ओजोन परत के सोंरक्षि के सलए पहला अोंतराध ष्ट्रीय सदवस 16 ससतोंबर 1995 को मनाया गया िा।

22. दु निया भर में अंतरााष्टरीय रे ि पांिा नदिस (IRPD) 2023 िब मिाया गया?
1)16 ससतोंबर 2023
2)15 ससतोंबर 2023
3)17 ससतोंबर 2023
4)14 ससतोंबर 2023
5)13 ससतोंबर 2023
उत्तर- 1)16 नसतंबर 2023
स्पष्टीिरण:
रे ड पाोंडा के सोंरक्षि के महत्व के बारे में जागरूकता पैदा करने और लोगोों को उन्ें बचाने के सलए प्रोत्सासहत करने के
सलए ससतोंबर के तीसरे शसनवार को दु सनया भर में अोंतराध ष्ट्रीय रे ड पाों डा सदवस (IRPD) मनाया जाता है ।
• अोंतराधष्ट्रीय रे ड पाोंडा सदवस (IRPD) 2023 16 नसतंबर 2023 को मनाया गया।
• IRPD 2022 17 ससतोंबर 2022 को मनाया गया। IRPD 2024 21 नसतंबर 2024 को मनाया जाएगा।
• पहिा अोंतराध ष्ट्रीय रे ड पाोंडा सदवस 18 ससतोंबर 2010 को मनाया गया िा।
• रे ड पाोंडा नेपाल, भारत, भूटान, चीन और म्याोंमार के जोंगल पहाडोों में व्यापक रूप से पाए जाते हैं।

23. #SeatheChange िी थीम िे साथ दु निया भर में अंतरााष्टरीय तटीय सफाई (ICC) नदिस 2023 िब मिाया
गया?
1)12 ससतोंबर 2023
2)15 ससतोंबर 2023
3)13 ससतोंबर 2023
4)16 ससतोंबर 2023
5)14 ससतोंबर 2023
उत्तर- 4)16 नसतंबर 2023

Report Errors in the PDF - ebooks@affairscloud.com Copyright 2014-2023 @ AffairsCloud.com 194


स्पष्टीिरण:
समुद्री कूडे की समस्या के बारे में जागरूकता बढाने के सलए हर साल ससतोंबर के तीसरे शसनवार को दु सनया भर में
अोंतराधष्ट्रीय तटीय सफाई (ICC) नदिस मनाया जाता है ।
i.प्लाक्कस्ट्क प्रदू षि पर ध्यान केंसद्रत करते हुए "#SeatheChange" िीम के साि 16 नसतंबर 2023 को अोंतराधष्ट्रीय
तटीय सफाई सदवस 2023 मनाया गया।
ii.पहली बार अोंतराधष्ट्रीय तटीय सफाई कायधक्रम की मेजबानी 1986 में सोंयुक्त राज्य अमेररका (USA) के वासशोंगटन राज्य
के पसिमी तट पर महासागर सोंरक्षि के सदस्योों सलोंडा मरासनस और कैिी ओ'हारा िारा की गई िी।
iii.ओशन कोंजरवेंसी के 38वें वासषधक अोंतराध ष्ट्रीय तटीय सफाई के उपलक्ष् में, ओशन कोंजरवेंसी,
Volunteercleanup.org ने समयामी वॉटरकीपर के साि 16 ससतोंबर 2023 को िोररडा (USA) के मागधरेट पेस पाकध में
समयामी-डे ड में एक वासषधक दे शव्यापी अोंतराध ष्ट्रीय तटीय सफाई का आयोजन सकया।

24. निश्व रोगी सुरक्षा नदिस 2023 िा निषय क्ा था िो 17 नसतंबर 2023 िो निश्व स्वास्थ्य संगठि (WHO)
द्वारा दु निया भर में मिाया गया था?
1)एों गेसजोंग पेशेंट्स फॉर पेशेंट सेफ्टी
2)मेसडकेशन सेफ्टी
3)मेसडकेशन सविाउट हामध
4)सेफ मैटरनल एों ड नूबोनध केयर
5)एर नाउ फॉर सेफ एों ड ररस्पेरफुल सचक्कल्ििध !
उत्तर- 1)एं गेनिंग पेशेंट्स फॉर पेशेंट सेफ्टी
स्पष्टीिरण:
सवश्व स्वास्थ्य सोंगठन (WHO) का निश्व रोगी सुरक्षा नदिस 2023 दु सनया भर में 17 नसतंबर 2023 को “एों गेसजोंग पेशेंट्स
फॉर पेशेंट सेफ्टी” निषय के साि मनाया गया।
i.WPSD 2023 का नारा है "एलीवेट द वॉइस ऑफ़ पेशेंट्स!" और पहला WPSD 17 ससतोंबर 2019 को मनाया गया िा।
ii.WHO ने फरवरी 2020 में "रोगी सुरक्षा का एक दशक 2020-2030" नामक एक प्रमुख पहल की थिापना की। यह
सोंयुक्त राष्ट्र (UN) सतत सवकास लक्ष्ोों (SDG) के अनुरूप है।
iii.2021 में 74वें WHA ने एक ऐसी दु सनया की दृसष्ट् से वैसश्वक रोगी सुरक्षा कायध योजना 2021-2030 को अपनाने के
सनिधय WHA74.13 को मोंजूरी दे दी, सजसमें एक ऐसी दु सनया की दृसष्ट् िी सजसमें स्वास्थ्य दे खभाल में सकसी को भी नुकसान
न हो, और हर रोगी को हर समय, हर जगह सुरसक्षत और सम्मानजनक दे खभाल समले।
iv.WPSD 2023 के अवसर पर, NATHEALTH, हेल्थकेयर फेडरे शन ऑफ इों सडया ने रोगी सुरक्षा को बढावा दे ने और
बढाने और स्वास्थ्य दे खभाल की गुिवत्ता में सुर्ार करने के सलए नेशनल एसक्रसडटे शन बोडध फॉर हॉक्कस्पटल्स एों ड हेल्थकेयर
प्रोवाइडसध (NABH) के साि एक समझौता ज्ञापन (MoU) पर हस्ताक्षर सकए।

25. दु निया भर में निश्व बांस नदिस 2023 िब मिाया गया?


1)14 ससतोंबर 2023
2)15 ससतोंबर 2023
3)16 ससतोंबर 2023
4)17 ससतोंबर 2023
5)18 ससतोंबर 2023
उत्तर- 5)18 नसतंबर 2023

Report Errors in the PDF - ebooks@affairscloud.com Copyright 2014-2023 @ AffairsCloud.com 195


स्पष्टीिरण:
बाोंस के बारे में जागरूकता पैदा करने और बाोंस की क्षमता को और असर्क उजागर करने के सलए 18 नसतंबर 2023
को दु सनया भर में निश्व बांस नदिस 2023 मनाया गया।
• सवश्व बाोंस सदवस के वासषधक आयोजन का नेतृत्व सवश्व बाोंस सोंगठन (WBO) िारा सकया जाता है , जो बाों स
सचसकत्सकोों के सलए एक अोंतरराष्ट्रीय समन्वय सनकाय है ।
• पहला बाोंस सदवस 18 ससतोंबर 2009 को बैंकॉक, िाईलैंड में मनाया गया िा। 18 ससतोंबर को िाईलैंड में प्रसतवषध
रॉयल िाई वन सदवस के रूप में मनाया जाता है।
i.सोंयुक्त राष्ट्र (UN) ने "सबक्कडोंग मैटेररयल्स एों ड द क्लाइमेट: कोंस्ट्र क्करोंग ए न्यू फ्यूचर " शीषधक से एक व्यापक ररपोटध का
अनावरि सकया है जो जैव-आर्ाररत सनमाधि सामग्री के सोंक्रमि में बाोंस की भूसमका पर प्रकाश डालता है।
ii.ररपोटध में पारों पररक सनमाध ि सामग्री जैसे सवरासत कोंक्रीट और स्ट्ील को सटकाऊ, नवीकरिीय सवकल्पोों के साि बदलने
के महत्व का उल्लेख सकया गया है।

26. संयुक्त राष्टर (UN) अंतरााष्टरीय समाि िेति नदिस 2023 दु निया भर में िब मिाया गया?
1)17 ससतोंबर 2023
2)15 ससतोंबर 2023
3)18 ससतोंबर 2023
4)16 ससतोंबर 2023
5)19 ससतोंबर 2023
उत्तर-3)18 नसतंबर 2023
स्पष्टीिरण:
समान मूल् के काम के सलए समान वेतन की उपलक्कब्ध की सदशा में दीघधकासलक प्रयासोों का प्रसतसनसर्त्व करने के सलए
सोंयुक्त राष्ट्र (UN) अंतरााष्टरीय समाि िेति नदिस प्रसतवषध 18 ससतोंबर को दु सनया भर में मनाया जाता है।
• 18 नसतंबर 2023 को चौिा अोंतराध ष्ट्रीय समान वेतन सदवस मनाया जाता है ।
• पहला अोंतराध ष्ट्रीय समान वेतन सदवस 18 ससतोंबर 2020 को मनाया गया।
• समान वेतन अोंतराध ष्ट्रीय गठबोंर्न (EPIC) का नेतृत्व अों तराध ष्ट्रीय श्रम सोंगठन (ILO), UN मसहला (लैंसगक समानता
और मसहलाओों के सशक्कक्तकरि के सलए UN इकाई), और आसिधक सहयोग और सवकास सोंगठन (OECD) िारा
सकया जाता है।

27. निश्व िि निगरािी नदिस (WWMD) 2023 दु निया भर में िब मिाया गया?
1)20 ससतोंबर 2023
2)18 ससतोंबर 2023
3)16 ससतोंबर 2023
4)19 ससतोंबर 2023
5)17 ससतोंबर 2023
उत्तर- 2)18 नसतंबर 2023
स्पष्टीिरण:
जल स्रोतोों की सनयसमत सनगरानी के महत्व के बारे में जागरूकता बढाने और दु सनया के जल सोंसार्नोों की सुरक्षा के महत्व
को सशसक्षत करने और पहचानने के सलए 18 नसतंबर 2023 को दु सनया भर में सवश्व जल सनगरानी सदवस (WWMD)
2023 मनाया गया।

Report Errors in the PDF - ebooks@affairscloud.com Copyright 2014-2023 @ AffairsCloud.com 196


i.सवश्व जल सनगरानी सदवस (WWMD) को 2003 में अमेररका के स्वच्छ जल फाउों डेशन (ACWF) िारा एक अोंतरराष्ट्रीय
आउटरीच कायधक्रम के रूप में घोसषत सकया गया िा।
ii.WWMD आसर्काररक तौर पर 18 ससतोंबर को मनाया जाता है । इसे शुरू में एक महीने बाद 18 अरू बर को सोंयुक्त
राज्य (US) स्वच्छ जल असर्सनयम की वषधगाोंठ को मान्यता दे ने के सलए चुना गया िा, सजसे 1972 में US काोंग्रेस िारा
अमेररका के जल सोंसार्नोों को बहाल करने और सोंरसक्षत करने के सलए असर्सनयसमत सकया गया िा।
iii.2007 में, आयोजकोों ने WWMD को 18 ससतोंबर तक थिानाोंतररत करने का सनिधय सलया। इस पररवतधन का कारि
वैसश्वक भागीदारी को प्रोत्सासहत करना िा, सजसमें वे क्षेत्र भी शासमल िे जहाों अरू बर के महीने में ठों ड का तापमान होता
है।
िोट: हर साल, लगभग 150 दे श 18 ससतोंबर को WWMD का सम्मान करने के सलए एक साि आते हैं।

28. 18 से 24 नसतंबर 2023 िो दु निया भर में मिाए गए बनधर िोगों िे अंतरााष्टरीय सप्ताह (IWDP) 2023 िा
निषय क्ा था?
1)सेलेब्रसटों ग सत्रसवोंग डे फ कम्युसनटीज
2)सबक्कडोंग इों क्लूससव कम्युसनटीज फॉर आल
3)ए वडध व्हेयर डे फ पीपल एवरीव्हेयर कैन साइन एनीव्हेयर!
4)रीसफरसमोंग डे फ पीपल ह्यूमन राइट् स
5)सवि साइन लैंग्वेजेज, एवरीवन इस इनक्लूडेड!
उत्तर- 3)ए िडा व्हेयर िे फ पीपि एिरीव्हेयर िैि साइि एिीव्हेयर!
स्पष्टीिरण:
अोंतराधष्ट्रीय बसर्र जन सप्ताह (IWDP) प्रसतवषध ससतोंबर के अोंसतम पूिध सप्ताह के दौरान दु सनया भर में मनाया जाता है और
ससतोंबर के अोंसतम रसववार को समाप्त होता है । IWDP सप्ताह उन लोगोों के असर्कारोों के बारे में जागरूकता बढाता है
जो बसर्र हैं या कम सुन पाते हैं /सुनने में अक्षम हैं।
i.IWPD 2023 18 से 24 नसतंबर 2023 तक "ए िडा व्हेयर िे फ पीपि एिरीव्हेयर िैि साइि एिीव्हेयर!" निषय
के साि मनाया जाता है।
ii.बसर्र लोगोों का अोंतराधष्ट्रीय सप्ताह (IWDP) एक पहल है सजसे पहली बार सवश्व बसर्र महासोंघ (WFD) िारा 1958 में
रोम, इटली में शुरू सकया गया िा।
iii.सोंयुक्त राष्ट्र (UN) IDSL प्रसतवषध 23 ससतोंबर को दु सनया भर में मनाया जाता है।

29. 21 लसतंबर 2023 को दु लनया भर में मनाए गए संयुक्त राष्टर (UN) अंतरााष्टरीय शांलत लदवस 2023 का लवषय
क्या था?
1)क्लाइमेट एक्शन फॉर पीस
2)शॉश्चपंग पीस टु गेदर
3)रीकवररं ग बेटर फॉर एन इिीटाि एं ि सस्ट्े नेबल वर्ल्ग
4)एन्ड रे श्चसस्म. श्चबर्ल् पीस
5)एक्शन्स फॉर पीस: आवर एं बीशन फॉर द #GlobalGoals
उत्तर- 5)एक्शन्स फॉर पीस: आवर एं बीशन फॉर द #GlobalGoals

Report Errors in the PDF - ebooks@affairscloud.com Copyright 2014-2023 @ AffairsCloud.com 197


स्पष्टीकरण:
संयुक्त राष्ट्र (UN) अंतराग ष्ट्रीय शांश्चत श्चदवस श्चिसे "श्चवश्व शां श्चत श्चदवस" के रूप में भी िाना िाता है, हर साल 21 श्चसतंबर को
दु श्चनया भर में मनाया िाता है और यह सभी दे शों और लोगों के भीतर और उनके बीच शांश्चत के श्चसिांतों को मिबूत
करने के श्चलए समश्चपगत है, िो 24 घंटे अश्चहंसा और युिश्चवराम की सुरक्षा की वकालत करता है।
i.21 श्चसतंबर 2023 को मनाए गए अंतराग ष्ट्रीय शां श्चत श्चदवस (IDP) 2023 का श्चवषय "एक्शन्स फॉर पीस: आवर एं बीशन
फॉर द #GlobalGoals के श्चलए हमारी महत्वाकांक्षा" है।
ii.पहला IDP 21 श्चसतंबर 1982 (श्चसतंबर 1982 का तीसरा मंगलवार) को मनाया गया िा।
iii.पीस बेल, श्चिसे "िापानी पीस बेल" के नाम से भी िाना िाता है , िून 1954 में िापान के संयुक्त राष्ट्र संघ (UNA) द्वारा
दान श्चकया गया िा।
iv.14 श्चसतंबर 2023 को एक युवा आयोिन कायगक्रम आयोश्चित श्चकया गया िा, िहां IDP के युवा आयोिन में युवा नेता,
कायगकताग और कलाकार अपनी आवाि उठा रहे िे और उन मुद्दों पर कारग वाई की मांग कर रहे िे िो उनके श्चलए
महत्वपूिग हैं।

30. लवश्व अल्जाइमर लदवस 2023 दु लनया भर में कब मनाया गया?


1)21 श्चसतंबर 2023
2)20 श्चसतंबर 2023
3)19 श्चसतंबर 2023
4)18 श्चसतंबर 2023
5)17 श्चसतंबर 2023
उत्तर- 1)21 लसतंबर 2023
स्पष्टीकरण:
लवश्व अल्जाइमर लदवस 2023 िागरूकता ब़िाने और अल्जाइमर रोग के आसपास मौिूद कलंक को चुनौती दे ने के
श्चलए 21 लसतंबर 2023 को दु श्चनया भर में मनाया गया, िो श्चक सबसे आम प्रकार का मनोभ्रंश और मनोभ्रंश के अन्य
रूप हैं।
• श्चवश्व अल्जाइमर श्चदवस श्चवश्व अल्जाइमर माह के दौरान मनाया िाता है।
i.श्चसतंबर का महीना दु श्चनया भर में प्रश्चतवषग श्चवश्व अल्जाइमर माह (एक महीने तक चलने वाला अश्चभयान) के रूप में मनाया
िाता है। यह अल्जाइमर रोग इं टरनेशनल (ADI) का अंतराग ष्ट्रीय अश्चभयान है ।
ii.ADI के अनुसार, श्चवश्व अल्जाइमर माह अश्चभयान 2023 का श्चवषय "कभी भी िल्दी नहीं, कभी बहुत दे र नहीं" है।
• श्चवश्व अल्जाइमर श्चदवस 2023 के अवसर पर, ADI ने मनोभ्रंश के िोस्खम को कम करने पर ध्यान केंश्चद्रत करते
हुए श्चवश्व अल्जाइमर ररपोटग 2023 लॉन्च की। 2023 की ररपोटग पत्रकाररता शैली में श्चलखी गई है और इसमें प्रमुख
केस अध्ययन शाश्चमल हैं।
• ADI के अनुसार, श्चवश्व स्तर पर, 2050 तक मनोभ्रंश से पीश्चड़त लोगों की संख्या लगभग तीन गुना हो िाएगी।

31. दु लनया भर में लवश्व गैंिा लदवस 2023 कब मनाया गया?


1)20 श्चसतंबर 2023
2)22 श्चसतंबर 2023
3)19 श्चसतंबर 2023
4)21 श्चसतंबर 2023
5)18 श्चसतंबर 2023

Report Errors in the PDF - ebooks@affairscloud.com Copyright 2014-2023 @ AffairsCloud.com 198


उत्तर- 2)22 लसतंबर 2023
स्पष्टीकरण:
गैंिा या राइनो (राइनो यूश्चनकॉश्चनगस) की भयानक स्थिश्चत के बारे में िागरूकता ब़िाने और इसकी आबादी के संरक्षि के
कदमों पर िोर दे ने के श्चलए 22 लसतंबर 2023 को दु श्चनया भर में लवश्व गैंिा लदवस 2023 मनाया गया।
• यह श्चदन गैंिों की सभी 5 प्रिाश्चतयों अिागत् अफ्रीका में सफेद और काले गैंिों और एश्चशया में महान एक
सींग वाले, िावन और सुमात्रा गैंिों की प्रिाश्चतयों को मनाने के श्चलए समश्चपगत है।
• 2010 में, श्चवश्व वन्यिीव कोष (WWF) - दश्चक्षि अफ्रीका ने इन िानवरों को बचाने के महत्व के बारे में
िागरूकता पैदा करने के श्चलए एक राष्ट्रीय संकट, श्चवश्व गैंिा श्चदवस की घोषिा की।
• भारत में गैंिे असम, पश्चिम बंगाल के कुछ श्चहस्सों और श्चबहार में पाए िाते हैं। WWF के अनुसार, आि
िंगलों में लगभग 3,700 भारतीय गैंिे हैं।
• माचग 2022 में हुई िनगिना के अनुसार, अकेले असम के कािीरं गा राष्ट्रीय उद्यान (KNP) में 2,613 गैंिे
हैं।

32. लवश्व गुिाब लदवस/कैंसर रोलगयों के कल्याण लदवस 2023 दु लनया भर में ___________, 2023 को मनाया गया।
1)20 श्चसतम्बर
2)18 श्चसतंबर
3)21 श्चसतंबर
4)19 श्चसतंबर
5)22 श्चसतंबर
उत्तर- 5)22 लसतंबर
स्पष्टीकरण:
लवश्व गुिाब लदवस, श्चिसे कैंसर रोश्चगयों के कल्याि के श्चलए श्चदवस 2023 के रूप में भी िाना िाता है, 22 लसतंबर
2023 को दु श्चनया भर में मनाया गया िा और यह दु श्चनया भर में कैंसर रोश्चगयों को समश्चपगत है और दु श्चनया भर में कैंसर से
बचे लोगों को प्रोत्साश्चहत करता है और स्वीकार करता है।
i.श्चवश्व गुलाब श्चदवस की उत्पश्चत्त का श्रेय मेश्चलंिा रोि नामक एक 12 वषीय कनािाई लड़की को श्चदया िाता है, श्चिसे
1994 में रक्त कैंसर (आस्स्कन ट्यूमर) के एक दु लगभ रूप का पता चला िा।
ii.श्चवश्व स्वास्थ्य संगठन (WHO) के अनुसार, वैश्चश्वक स्तर पर कैंसर मृत्यु का एक प्रमुख कारि है, श्चिसके कारि 2020
में लगभग 10 श्चमश्चलयन मौतें हुईं, या लगभग 6 में से 1 मौत हुई। सबसे आम कैंसर स्तन, फेफड़े , बृहदान्त्र और मलाशय
और प्रोस्ट्े ट कैंसर हैं ।

33. दु लनया भर में लवश्व कार-मुक्त लदवस 2023 कब मनाया गया?


1)18 श्चसतंबर 2023
2)21 श्चसतंबर 2023
3)19 श्चसतंबर 2023
4)22 श्चसतंबर 2023
5)20 श्चसतंबर 2023
उत्तर- 4)22 लसतंबर 2023

Report Errors in the PDF - ebooks@affairscloud.com Copyright 2014-2023 @ AffairsCloud.com 199


स्पष्टीकरण:
मोटर चालकों को एक श्चदन के श्चलए अपनी कारों को छोड़ने के श्चलए प्रोत्साश्चहत करने के श्चलए 22 लसतंबर 2023 को
दु श्चनया भर के शहरों में लवश्व कार-मुक्त लदवस 2023 मनाया गया।
• श्चवश्व कार मुक्त श्चदवस का श्चवचार 1973 के पेटरोश्चलयम संकट के बाद आया, िब लोगों ने कारों पर अपनी
श्चनभगरता को कम करने के तरीकों की तलाश शुरू कर दी।
• पहला राष्ट्रीय कार-मुक्त श्चदवस अश्चभयान 1997 में श्चिटे न में शुरू श्चकया गया िा।
i.श्चवश्व कार-मुक्त श्चदवस 2023 के पालन के एक भाग के रूप में, मध्य प्रदे श (MP) उच्च न्यायालय की इं दौर पीठ ने अपने
सभी कमगचाररयों को श्चवश्व कार-मुक्त श्चदवस के अवसर के संबंर् में कारों के बिाय पररवहन के वैकस्ल्पक सार्नों का
उपयोग करने के श्चलए कहा है।
ii.श्चवश्व कार मुक्त श्चदवस 2023 के अवसर पर, पश्चिम बंगाल (WB) के राज्यपाल CV आनंद बोस ने ‘कला क्रांश्चत श्चमशन’
के लोगो का अनावरि श्चकया और पश्चिम बंगाल के कोलकाता में रािभवन से एक साइश्चकल माचग को हरी झंिी श्चदखाई।

34. संयुक्त राष्टर (UN) अंतरााष्टरीय सांिेनति भाषा नदिस (IDSL) 2023 िो दु निया भर में "ए िडा व्हेयर िे फ
पीपि एिरीव्हेयर िैि नसग्न एिीव्हेय!” निषय िे साथ िब मिाया गया था?
1)19 ससतोंबर 2023
2)21 ससतोंबर 2023
3)23 ससतोंबर 2023
4)20 ससतोंबर 2023
5)22 ससतोंबर 2023
उत्तर-3)23 नसतंबर 2023
स्पष्टीिरण:
सोंयुक्त राष्ट्र (UN) अंतरााष्टरीय सांिेनति भाषा नदिस (IDSL) 2023 23 नसतंबर 2023 को दु सनया भर में "ए वडध
व्हेयर डे फ पीपल एवरीव्हेयर कैन ससग्न एनीव्हेय!" सवषय के साि मनाया गया।
i.यह सदन बसर्र लोगोों के अोंतराध ष्ट्रीय सप्ताह (IWDP) 2023 (18 से 24 ससतोंबर 2023 तक) के सहस्से के रूप में मनाया
जाता है।
ii.IWDP 2018 के भाग के रूप में 23 ससतोंबर 2018 को पहला अोंतराध ष्ट्रीय साों केसतक भाषा सदवस मनाया गया।
iii.भारतीय साोंकेसतक भाषा अनुसोंर्ान और प्रसशक्षि केंद्र (ISLRTC), नई सदल्ली (सदल्ली) ने 23 ससतोंबर 2023 को IDSL
2023 मनाया और सवसभन्न कायधक्रम और सामग्री लॉन्च की। यह कायधक्रम नई सदल्ली के भीम हॉल, डॉ. अोंबेडकर
इों टरनेशनल सेंटर जनपि में आयोसजत सकया गया िा।

35. दु निया भर में निश्व िदी नदिस (WRD) 2023 िब मिाया गया?
1)21 ससतोंबर 2023
2)24 ससतोंबर 2023
3)22 ससतोंबर 2023
4)25 ससतोंबर 2023
5)23 ससतोंबर 2023
उत्तर- 2)24 नसतंबर 2023

Report Errors in the PDF - ebooks@affairscloud.com Copyright 2014-2023 @ AffairsCloud.com 200


स्पष्टीिरण:
सवश्व नदी सदवस (WRD) प्रसतवषध ससतोंबर के चौिे रसववार को दु सनया भर में नसदयोों के महत्व के बारे में जागरूकता को
बढावा दे ने के सलए मनाया जाता है और इसका उद्दे श्य दु सनया भर में सभी नसदयोों की बेहतर सुरक्षा को प्रोत्सासहत करना
है।
• सवश्व नदी सदवस (WRD) 2023 24 नसतंबर 2023 को मनाया गया।
• WRD 2022 25 ससतोंबर 2022 को मनाया गया और WRD 2024 22 ससतोंबर 2024 को मनाया जाएगा।
• वह पहला WRD 2005 में मनाया गया िा और 12 दे शोों में मनाया गया िा।
i.WRD 2023 को मनाने के सलए, WRD ने प्रमुख नसदयोों में पैडसलोंग यात्राएों , नदी सम्मेलन, ऑनलाइन पैनल कायधक्रम
जैसे कायधक्रमोों की एक श्रृोंखला आयोसजत की, और नदी के सकनारे वृक्षारोपि और आवास बहाली पररयोजनाओों जैसी
सवसभन्न नई पररयोजनाएों शुरू की गई हैं ।
ii."नसदयोों के असर्कार" के सलए एक असभयान और "नसदयोों को गोद लें" कायधक्रम भी आयोसजत सकए गए।

36. 25 नसतंबर 2023 िो दु निया भर में मिाए गए 13िें निश्व फामाानसस्ट् नदिस (WPD) 2023 िा निषय क्ा
था?
1)फामाधससस्ट्: योर मेसडससर्न् एक्सपट् स
2)फामेसी यूनाइटे ड इन एक्शन फॉर ए हेअल्थीर वडध
3)फामेसी: ऑलवेज टर स्ट्े ड फॉर योर हेल्थ
4)सेफ एों ड इफेक्करव मेडीसीनस फॉर ऑल
5)फामेसी स्ट्र ें ग्हेसनोंग हेल्थ ससस्ट्म्स
उत्तर-5)फामेसी स्ट्रें ग्हेनिंग हेल्थ नसस्ट्म्स
स्पष्टीिरण:
13वाों सवश्व फामाधससस्ट् सदवस (WPD) 2023 25 नसतंबर 2023 को दु सनया भर में "फामेसी स्ट्रें ग्हेनिंग हेल्थ नसस्ट्म्स"
की निषय के साि मनाया गया।
• यह सदन फामेसी, फामाध स्युसटकल सवज्ञान और फामाधस्युसटकल सशक्षा के सलए वैसश्वक सनकाय, इों टरनेशनल
फामाधस्युसटकल फेडरे शन (FIP- फेडरे शन इों टरनेशनेल फामाधस्युसटक) के थिापना सदवस का भी प्रतीक है।
• WPD 2023 का सवषय "फामेसी स्ट्र ें िेसनोंग हेल्थ ससस्ट्म्स" है।
• पहला सवश्व फामाधससस्ट् सदवस 25 ससतोंबर 2010 को मनाया गया िा।

37. पंनित दीिदयाि उपाध्याय िी 107िी ं ियंती िे उपिक्ष्य में पूरे भारत में अंत्योदय नदिस 2023 िब मिाया
गया?
1)21 ससतोंबर 2023
2)23 ससतोंबर 2023
3)25 ससतोंबर 2023
4)22 ससतोंबर 2023
5)24 ससतोंबर 2023
उत्तर-3)25 नसतंबर 2023
स्पष्टीिरण:
भारतीय जनता पाटी (BJP) के पूवधवती राजनीसतक दल भारतीय जनसोंघ (BJS) के पूवध नेता पंनित दीिदयाि उपाध्याय
की जयोंती मनाने के सलए हर साल 25 नसतंबर 2023 को पूरे भारत में अंत्योदय नदिस िे 2023 मनाया जाता है।

Report Errors in the PDF - ebooks@affairscloud.com Copyright 2014-2023 @ AffairsCloud.com 201


i.25 ससतोंबर 2023 को पोंसडत दीनदयाल उपाध्याय की 107िी ं ियंती है।
ii.'अोंत्योदय' शब्द का अिध "समाज के अोंसतम व्यक्कक्त का उत्थान और सवकास" है , जो सक दीनदयाल उपाध्याय िारा
प्रचाररत अवर्ारिाओों में से एक है । अोंत्योदय का उद्दे श्य समाज में गरीबोों का उत्थान करना है।
iii.पहला अोंत्योदय सदवस 25 ससतोंबर 2014 को मनाया गया िा।

38. दु निया भर में परमाणु हनथयारों िे पूणा उन्मूिि िे निए संयुक्त राष्टर (UN) अंतरााष्टरीय नदिस 2023 िब
मिाया गया?
1)26 ससतोंबर 2023
2)25 ससतोंबर 2023
3)24 ससतोंबर 2023
4)23 ससतोंबर 2023
5)22 ससतोंबर 2023
उत्तर- 1)26 नसतंबर 2023
स्पष्टीिरण:
परमािु हसियारोों से मानवता के सलए उत्पन्न खतरे और उनके पूिध उन्मूलन की आवश्यकता के बारे में सावधजसनक
जागरूकता बढाने के सलए 26 नसतंबर 2023 को दु सनया भर में सोंयुक्त राष्ट्र (UN) परमाणु हनथयारों िे पूणा उन्मूिि
िे निए अंतरााष्टरीय नदिस 2023 मनाया गया।
i.परमािु हसियारोों के पूिध उन्मूलन के सलए पहला अोंतराध ष्ट्रीय सदवस 26 ससतोंबर 2014 को मनाया गया।
ii.परमािु-हसियार-मुक्त क्षेत्र (NWFZ) क्षेत्रीय समझौते हैं सजनका उद्दे श्य शाोंसत और सुरक्षा को बढावा दे ते हुए वैसश्वक
परमािु अप्रसार और सनरस्त्रीकरि प्रयासोों को मजबूत करना है।

39. निश्व गभानिरोधि नदिस 2023 िा निषय क्ा था िो 26 नसतंबर 2023 िो दु निया भर में मिाया गया?
1)एर रे स्पोोंससबल , सक्रएटे ड बाइ चॉइस
2)इट् स योर लाइफ, इट् स योर ररस्पाोंसससबसलटी
3)हेअल्थी फैसमलीज़ आर सक्रएटे ड बाई चॉइस, नॉट बाई चाोंस
4)नो योर ऑप्शोंस
5)द पावर ऑफ ऑप्शोंस
उत्तर- 5)द पािर ऑफ ऑप्शंस
स्पष्टीिरण:
सवश्व गभधसनरोर्क सदवस 2023 26 ससतोंबर 2023 को "द पावर ऑफ ऑप्शोंस" सवषय के साि दु सनया भर में मनाया गया।
i.26 नसतंबर 2023 को 17िां सवश्व गभधसनरोर्क सदवस मनाया जाता है।
ii.पहला सवश्व गभधसनरोर्क सदवस 26 ससतोंबर 2007 को मनाया गया िा।
iii.पररवार सनयोजन में एक सचेत सवकल्प के रूप में गभधसनरोर्क के महत्व को उजागर करने के सलए 10 अोंतराध ष्ट्रीय
पररवार सनयोजन सोंगठनोों िारा सवश्व गभधसनरोर्क सदवस की थिापना की गई िी।

40. निश्व पयाािरण स्वास्थ्य नदिस (WEHD) 2023 िो दु निया भर में "ग्लोबि एििायरिमेंट पखब्लि हेल्थ:
स्ट्ैं निं ग अप टू प्रोटे र एिरीिि हेल्थ ईच एं ि एव्री िे " निषय िे साथ िब मिाया गया था?
1)22 ससतोंबर 2023
2)24 ससतोंबर 2023

Report Errors in the PDF - ebooks@affairscloud.com Copyright 2014-2023 @ AffairsCloud.com 202


3)26 ससतोंबर 2023
4)23 ससतोंबर 2023
5)25 ससतोंबर 2023
उत्तर-3)26 नसतंबर 2023
स्पष्टीिरण:
सवश्व पयाधवरि स्वास्थ्य सदवस (WEHD) 2023 26 नसतंबर 2023 को दु सनया भर में "ग्लोबल एनवायरनमेंट पक्किक
हेल्थ: स्ट्ैं सडों ग अप टू प्रोटे र एवरीवर्न् हेल्थ ईच एों ड एव्री डे " निषय के साि मनाया गया।
i.26 ससतोंबर 2011 को, इों टरनेशनल फेडरे शन ऑफ एनवायनधमेंटल हेल्थ (IFEH) की पररषद ने हर साल 26 ससतोंबर
को सवश्व पयाधवरि स्वास्थ्य सदवस (WEHD) के रूप में घोसषत सकया।
ii.भारत में 2023 घटिाएाँ :
• WEHD को मनाने के सलए, MS रामैया इों स्ट्ीट्यूट ऑफ टे क्नोलॉजी, बेंगलुरु (कनाधटक) के केसमकल इों जीसनयररों ग
सवभाग ने 26 ससतोंबर 2023 से 3 अरू बर 2023 तक कायधक्रमोों की एक श्रृोंखला आयोसजत की, जैसे:
• SAFI इों स्ट्ीट्यूट ऑफ एडवाोंथड स्ट्डी (SIAS), मलप्पुरम (केरल) ने सवशेषज्ञ वाताध , सामुदासयक आउटरीच
कायधक्रम और कॉलेज के छात्रोों के बीच एक बहस जैसे कायधक्रमोों के साि WEHD का अवलोकन सकया।

41. संयुक्त राष्टर (UN) निश्व पयाटि नदिस (WTD ) 2023 िा निषय क्ा था िो 27 नसतंबर 2023 िो दु निया भर
में मिाया गया?
1)टू ररज्म एों ड जॉब्स : ए बेटर फ्यूचर फॉर ऑल
2)टू ररज्म फॉर इों क्लूससव ग्रोि
3)रसिोंसकोंग टू ररज्म
4)टू ररज्म एों ड रूरल डे वलपमेंट
5)टू ररज्म एों ड ग्रीन इन्वेस्ट्मेंट
उत्तर-5)टू ररज्म एं ि ग्रीि इन्वेस्ट्मेंट
स्पष्टीिरण:
सोंयुक्त राष्ट्र (UN) निश्व पयाटि नदिस (WTD ) 2023 27 नसतंबर 2023 को "टू ररज्म एों ड ग्रीन इन्वेस्ट्मेंट" की सवषय
के साि दु सनया भर में मनाया गया।
• WTD के वासषधक पालन का नेतृत्व सोंयुक्त राष्ट्र सवश्व पयध टन सोंगठन (UNWTO ) िारा सकया जाता है , जो UN की
एक सवशेष एजेंसी है जो सजम्मेदार, सटकाऊ और सावधभौसमक रूप से सुलभ पयधटन को बढावा दे ने के सलए
सजम्मेदार है।
i.27 ससतोंबर 2023 को 43िां निश्व पयाटि नदिस मनाया गया। पहली बार WTD 27 ससतोंबर 1980 को मनाया गया िा।
ii.वासषधक रूप से, UNWTO WTD समारोहोों के सलए एक मेजबान दे श को नासमत करता है । WTD 2023 का मेजबान
दे श सऊदी अरब है।
• WTD 2023 के सलए कायधक्रमोों की एक श्रृोंखला आयोसजत की गई और 27 से 28 ससतोंबर 2023 तक फोर सीजर्न्
होटल, ररयाद, सऊदी अरब में आयोसजत की गई।
iii.पयधटन मोंत्रालय (MoT) ने पयाधवरि, वन और जलवायु पररवतधन मोंत्रालय (MoEFCC), सोंयुक्त राष्ट्र सवश्व पयधटन सोंगठन
(UNWTO) और सोंयुक्त राष्ट्र पयाधवरि कायधक्रम (UNEP) के साि साझेदारी में पयधटन क्षेत्र पर ध्यान केंसद्रत करते हुए
समशन LiFE(लाइफस्ट्ाइल फॉर एनवायरनमेंट) के तहत एक क्षेत्रीय कायधक्रम 'टर ै वल फॉर लाइफ' LiFE’(TFL) के वैसश्वक
लॉन्च का आयोजन सकया है ।

Report Errors in the PDF - ebooks@affairscloud.com Copyright 2014-2023 @ AffairsCloud.com 203


42. निश्व फेफडे नदिस 2023 िब था निसे दु निया भर में 'एक्सेस टू नप्रिेंशि एं ि टर ीटमेंट फॉर ऑि, िीि िो िि
नबहाइं ि” निषय िे साथ मिाया गया।
1)26 ससतोंबर 2023
2)23 ससतोंबर 2023
3)25 ससतोंबर 2023
4)27 ससतोंबर 2023
5)24 ससतोंबर 2023
उत्तर-3)25 नसतंबर 2023
स्पष्टीिरण:
निश्व फेफडे नदिस 2023 को दु सनया भर में 25 नसतंबर 2023 को “एक्सेस टू सप्रवेंशन एों ड टर ीटमेंट फॉर ऑल, लीव नो
वन सबहाइों ड” सवषय के साि मनाया गया।
• 25 ससतोंबर 2023 को छठा सवश्व फेफडा सदवस है।
i.सवश्व फेफडे सदवस की शुरुआत फोरम ऑफ इों टरनेशनल रे क्कस्परे टरी सोसाइटीज (FIRS) िारा की गई िी, जो एक
सहयोगी सों गठन है सजसमें दु सनया की अग्रिी अोंतरराष्ट्रीय श्वसन सोसायटी शासमल हैं ।
ii.पहिा निश्व फेफडे नदिस 2017 में मनाया गया िा।
iii.FIRS की थिापना 2001 में हुई िी; FIRS वैसश्वक स्तर पर फेफडोों के स्वास्थ्य को आगे बढाने के सलए समसपधत है।
iv.डे सवड CL लैम FIRS और एसशयन पैसससफक सोसाइटी ऑफ रे क्कस्परोलॉजी (APSR) के अध्यक्ष हैं।

43. निश्व रे बीि नदिस (WRD) 2023 िा निषय क्ा था िो 28 नसतंबर 2023 िो दु निया भर में मिाया गया?
1)रे बीज: शेयर द मैसेज, सेव ए लाइफ
2)वन हेल्थ, जीरो डे ि
3)एन्ड रे बीज: कॉलेबोरे ट एों ड वेक्सीनेट
4)रे बीज: ऑल फॉर 1, वन हेल्थ फॉर ऑल
5)रे बीज: फैर् स, नॉट सफयर
उत्तर- 4)रे बीि: ऑि फॉर 1, िि हेल्थ फॉर ऑि
स्पष्टीिरण:
रे बीज की रोकिाम के बारे में जागरूकता बढाने और इस भयावह बीमारी को हराने में प्रगसत को उजागर करने के सलए
28 नसतंबर को दु सनया भर में निश्व रे बीि नदिस (WRD) मनाया जाता है। इस सदन का उद्दे श्य सवश्व स्तर पर रे बीज
उन्मूलन की वकालत करना भी है।
i.28 नसतंबर 2023 को WRD का 17िां उत्सव है।
ii.WRD 2023 का निषय "रे बीि: ऑि फॉर 1, िि हेल्थ फॉर ऑि" है।

44. संयुक्त राष्टर (UN) द्वारा 28 नसतंबर 2023 िो दु निया भर में __________ निश्व समुद्री नदिस (WMD) 2023
मिाया गया।
1)46वाों
2)42वाों
3)44वाों
4)43वाों
5)45वाों

Report Errors in the PDF - ebooks@affairscloud.com Copyright 2014-2023 @ AffairsCloud.com 204


उत्तर- 1)46िां
स्पष्टीिरण:
समुद्री उद्योग में काम करने वाले लोगोों के योगदान को मान्यता दे ने के सलए सों युक्त राष्ट्र (UN) का सवश्व समुद्री सदवस
(WMD) हर साल ससतोंबर के आक्कखरी गुरुवार को मनाया जाता है।
• WMD 2023, 46िां निश्व समुद्री नदिस, 28 ससतोंबर 2023 को मनाया गया।
• WMD 2022 29 ससतोंबर 2022 को मनाया गया और WMD 2024 26 ससतोंबर 2024 को मनाया जाएगा।
• सवश्व समुद्री सदवस 2023 का सवषय “MARPOL एट 50 – आवर कसमटमेंट गोज ऑन” है
• WMD 2023 जहाजोों से प्रदू षि की रोकिाम के सलए अोंतराध ष्ट्रीय सम्मेलन (MARPOL) को अपनाने की 50वीों
वषधगाोंठ का प्रतीक है।

45. संयुक्त राष्टर (UN) सूचिा ति सािाभौनमि पहुं च िे निए अंतरााष्टरीय नदिस (IDUAI) 2023 िो दु निया भर
में 'द इम्पोटें स ऑफ द ऑििाइि स्पेस फॉर एक्सेस टू इिफामेशि' निषय िे साथ िब मिाया गया?
1)27 ससतोंबर 2023
2)25 ससतोंबर 2023
3)28 ससतोंबर 2023
4)26 ससतोंबर 2023
5)24 ससतोंबर 2023
उत्तर-3)28 नसतंबर 2023
स्पष्टीिरण:
सूचना से सोंबोंसर्त कानूनोों के सवस्तार और उनके वास्तसवक कायाधन्वयन की आवश्यकता के बारे में जागरूकता फैलाने
के सलए सोंयुक्त राष्ट्र (UN) सूचना तक सावधभौसमक पहुोंच के सलए अोंतराध ष्ट्रीय सदवस (IDUAI) हर साल 28 नसतंबर को
दु सनया भर में मनाया जाता है।
• इन कानूनोों के सवस्तार से दु सनया तक पहुोंच के सलए समावेशी सोंथिानोों के सनमाधि में मदद समलती है।
• 28 नसतंबर 2023 को UN का चौिा IDUAI मनाया गया।
• IDUAI 2023 का सवषय "द इम्पोटें स ऑफ द ऑनलाइन स्पेस फॉर एक्सेस टू इनफामेशन" है।

46. संयुक्त राष्टर (UN) अंतरााष्टरीय िाद्य हानि और अपनशष्ट िागरूिता नदिस (IDAFLW) 2023 दु निया भर में
िब मिाया गया?
1)28 ससतोंबर 2023
2)26 ससतोंबर 2023
3)29 ससतोंबर 2023
4)27 ससतोंबर 2023
5)25 ससतोंबर 2023
उत्तर-3)29 नसतंबर 2023
स्पष्टीिरण:
सोंयुक्त राष्ट्र (UN) खाद्य हासन और बबाधदी के बारे में जागरूकता का अोंतराध ष्ट्रीय सदवस (IDAFLW)2023 को दु सनया भर
में 29 नसतंबर 2023 को "ररड्यूससोंग फूड लॉस एों ड वेस्ट्: टे सकोंग एक्शन टू टर ाोंसफॉमध फूड ससस्ट्म्स" निषय के साि
मनाया गया।

Report Errors in the PDF - ebooks@affairscloud.com Copyright 2014-2023 @ AffairsCloud.com 205


• यह सदन UN के खाद्य और कृसष सोंगठन (FAO) और सों युक्त राष्ट्र पयाध वरि कायधक्रम (UNEP) िारा सह-आयोसजत
सकया जाता है।
• 29 ससतोंबर 2023 को चौिा IDAFLW मनाया गया।
• 2023 सवषय 2030 के एजेंडे की उपलक्कब्ध में योगदान दे ने के सलए कृसष खाद्य प्रिासलयोों को बदलने की सदशा में
खाद्य हासन और बबाधदी (FLW) को कम करने की कारध वाई का आह्वान करती है ।
• पहला IDAFLW 29 ससतोंबर 2020 को मनाया गया।

47. निश्व हृदय नदिस (WHD) 2023 िा निषय क्ा था िो 29 नसतंबर 2023 िो दु निया भर में मिाया गया?
1)बी ए हाटध हीरो
2)यूज़ हाटध , नो हाटध
3)यूज़ हाटध टू बीट CVD
4)यूज़ हाटध टू कनेर
5)मेक योर हाटध प्रॉसमस
उत्तर- 2)यूज़ हाटा , िो हाटा
स्पष्टीिरण:
निश्व हृदय नदिस (WHD) 2023 दु सनया भर में 29 नसतंबर 2023 को "यूज़ हाटा , िो हाटा " सवषय के साि मनाया
गया।
i,सवश्व हृदय सदवस हृदय स्वास्थ्य के बारे में जागरूकता बढाने और हृदय रोगोों (CVD) की रोकिाम, पता लगाने और
प्रबोंर्न के सलए कायों में तेजी लाने के सलए मनाया जाता है।
ii.सवषय पहले अपने हृदय को जानने की महत्वपूिध प्रसक्रया पर जोर दे ता है ; इसका इरादा CVD में कमी लाने की सदशा
में सोचने और कायध करने का है।
iii.WHD की थिापना 2000 में वडध हाटध फेडरे शन (WHF) िारा सवश्व स्वास्थ्य सोंगठन (WHO) के साि साझेदारी में की
गई िी।
iv. WHD का सवचार सवश्व स्वास्थ्य महासोंघ (1997-99) के पूवध अध्यक्ष एों टोनी बाई डी लूना िारा पेश सकया गया िा और
यह सदन पहली बार 24 ससतोंबर, 2000 को मनाया गया िा।

ENVIRONMENT
1. नसतंबर 2023 में पहचािे गए स्वगीय िॉ. A.P.J अब्दु ि ििाम िे िाम पर समुद्री टानिा ग्रेि िी एि िई प्रिानत
"बैनटनिप्स ििामी" निस राज्य में पहचािी गई?
1)आों ध्र प्रदे श
2)केरल
3)ओसडशा
4)तसमलनाडु
5)कनाधटक
उत्तर-4)तनमििािु

Report Errors in the PDF - ebooks@affairscloud.com Copyright 2014-2023 @ AffairsCloud.com 206


स्पष्टीिरण:
केरल के कोचीन में कोचीन सवज्ञान और प्रौद्योसगकी सवश्वसवद्यालय (कुसैट) के समुद्री जीवसवज्ञान सवभाग के शोर्कताधओों
ने रामनािपुरम (दसक्षिपूवध) तनमििािु (TN) में मोंडपम तट के अोंतर्ज्ाधरीय समुद्र तट तलछट से समुद्री टासडध ग्रेड की
एक नई प्रजासत, "बैनटनिप्स ििामी" की पहचान की है। ।
i.नई प्रजासत का नाम स्वगीय डॉ. A.P.J. अब्दु ल कलाम, भारत के 11 वें राष्ट्रपसत (2002-2007) और एक एयरोस्पेस
वैज्ञासनक के नाम पर रखा गया है , क्ोोंसक खोज उनके मूल थिान रामेश्वरम (TN) के करीब की गई िी।
ii.शोर् के सनष्कषध वैज्ञासनक पसत्रका, ज़ूटाक्सा के 18 ससतोंबर 2023 अोंक में प्रकासशत सकए गए हैं ।
iii.समुद्री टासडध ग्रेड की नई प्रजासत 'बैसटसलप्स कलामी' जीनस 'बैसटसलप्स' से सोंबोंसर्त है। यह जीनस बैसटसलप्स की 37वीों
प्रजासत है।

2. निस राज्य में, िूिॉनििि सिे ऑफ इं निया (ZSI) िे िैज्ञानििों िे हाि ही में (नसतंबर'23 में) िीिंत िारं गी
गहरे पािी िी समुद्री मछिी िी एि िई प्रिानत िी पहचाि िी है , निसिा िाम 'पटरीगोनटर ग्ला इं टरमेनििा'
है?
1)ओसडशा
2)आों ध्र प्रदे श
3)पसिम बोंगाल
4)तसमलनाडु
5)महाराष्ट्र
उत्तर-3)पनिम बंगाि
स्पष्टीिरण:
जूलॉसजकल सवे ऑफ इों सडया(ZSI) के वैज्ञासनकोों ने पनिम बंगाि के दीघा मोहना से जीवोंत नारों गी गहरे पानी की समुद्री
मछली की एक नई प्रजासत 'पटरीगोनटर ग्ला इं टरमेनििा' की पहचान की है।
• नई प्रजासत का सववरि 20 ससतोंबर, 2023 को अोंतराध ष्ट्रीय समु द्री सवज्ञान जनध ल िैलासस में प्रकासशत सकया गया
िा।
• नई प्रजासत, सजसे आमतौर पर गनाधड्धस या सी-रॉसबर्न् के नाम से जाना जाता है , पररवार टर ाइक्कग्लडे और जीनस
पसटध गोसटर ग्ला के इों डो-पैसससफक सबजेनस ओटोसहम से सोंबोंसर्त है।
• दु सनया भर में कुल 178 टर ाइक्कग्लडे प्रजासतयोों में से यह भारत में पाई जाने वाली चौिी टे रीगोसटर ग्ला प्रजासत है।
• वैज्ञासनकोों की टीम का नेतृत्व ZSI में एस्ट्ु रीन बायोलॉजी रीजनल सेंटर (EBRC) के असनल महापात्रा, सनसमत्र रॉय
और सुभ्रेंदु शेखर समश्रा ने सकया।

3. नसतंबर 2023 में, िैज्ञानििों िे 2012 और 2013 में हांगिांग में िब्त निए गए 27 स्केिों िे निश्लेषण िे
माध्यम से एि िई पैंगोनिि प्रिानत िी पहचाि िी, निसे अस्थायी रूप से "मैनिस नमस्ट्ीररया" िाम नदया गया।
मैनिस नमस्ट्ीररया _________ पैंगोनिि प्रिानत और 5िी ं एनशयाई पैंगोनिि बि गई।
1)6वीों
2)8वीों
3)4वीों
4)7वीों
5)9वीों
उत्तर- 5)9िी ं

Report Errors in the PDF - ebooks@affairscloud.com Copyright 2014-2023 @ AffairsCloud.com 207


स्पष्टीिरण:
वैज्ञासनकोों ने 2012 और 2013 में हाोंगकाोंग में जब्त सकए गए 27 स्केलोों के सवश्लेषि के माध्यम से एक नई पैंगोनिि
प्रिानत की पहचान की है , सजसे अथिायी रूप से "मैनिस नमस्ट्ीररया" नाम सदया गया है । यह नई प्रजासत पैंगोसलन की
9िी ं प्रिानत और 5िी ं एनशयाई पैंगोनिि बन गई है।
i.पैंगोसलन की 8 मौजूदा प्रजासतयोों को तीन प्रजासतयोों: मैसनस (एसशया), फाटासगनस (अफ्रीका), और स्मुट्ससया (अफ्रीका)
में वगीकृत सकया गया है।
ii.यह सनष्कषध जनधल प्रोसीसडों ग्स ऑफ द नेशनल एकेडमी ऑफ साइों सेज (PNAS) में प्रकासशत हुआ िा।

APP and WEB PORTAL


1. नसतंबर 2023 में, भारत सरिार िे एि मोबाइि ऐप "G20 इं निया" िा अिािरण निया, िो घटिा-संबंधी
िाििारी िे निए एि व्यापि, इं टरै खरि गाइि पेश िरे गा।
G20 इं निया ऐप निस मंत्रािय िे बिाया है ?
1)सवज्ञान एवों प्रौद्योसगकी मोंत्रालय
2)सूचना एवों प्रसारि मोंत्रालय
3)सवदे श मोंत्रालय
4)गृह मोंत्रालय
5)इलेररॉसनक्स और सूचना प्रौद्योसगकी मोंत्रालय
उत्तर-3)निदे श मंत्रािय
स्पष्टीिरण:
भारत सरकार ने एक मोबाइि ऐप "G20 इं निया" का अनावरि सकया है जो घटना-सोंबोंर्ी जानकारी के सलए एक
व्यापक, इों टरै क्करव गाइड पेश करे गा। पहली बार G20 के सलए एक मोबाइल ऐप बनाया गया है ।
• ऐप को 2023 G20 नई सदल्ली सशखर सम्मेलन से पहले लॉन्च सकया गया िा, जो G20 की 18वीों बैठक है , जो
9-10 ससतोंबर, 2023 को नई सदल्ली, सदल्ली में भारत मों डपम अोंतराध ष्ट्रीय प्रदशधनी-कन्वेंशन सेंटर, प्रगसत मैदान
में होने वाली है।
• निदे श मंत्रािय (MEA) िारा बनाया गया G20 इों सडया ऐप प्रसतभासगयोों और वैसश्वक दशधकोों के सलए एक
उपयोगकताध -अनुकूल सडसजटल प्लेटफॉमध है।
i.30 अगस्त, 2023 को, प्रर्ान मोंत्री (PM) के प्रर्ान ससचव, डॉ प्रमोद कुमार (PK) समश्रा ने G20 (बीस का समूह) नई
सदल्ली नेताओों के नशिर सम्मेिि 2023 की तैयाररयोों की दे खरे ख के सलए 9िी ं G20 समन्वय सनमनत िी बैठि की
अध्यक्षता की, सजसमें रसद, प्रोटोकॉल, सुरक्षा और मीसडया व्यवथिा शासमल िी।

2. निस राज्य िे राज्यपाि िे हाि ही में (नसतंबर'23 में) िानिटी िाउं नसि ऑफ इं निया (QCI) द्वारा नििनसत
एि मोबाइि ऐप 'सरपंच संिाद' िॉन्च निया है ?
1)C.V. आनोंद बोस (पसिम बोंगाल)
2)S अब्दु ल नज़ीर (आों ध्र प्रदे श)
3)गुलाब चोंद कटाररया (असम)
4)आचायध दे व व्रत (गुजरात)
5)िावरचोंद गेहलोत (कनाधटक)
उत्तर-3)गुिाब चंद िटाररया (असम)

Report Errors in the PDF - ebooks@affairscloud.com Copyright 2014-2023 @ AffairsCloud.com 208


स्पष्टीिरण:
असम के राज्यपाल गुलाब चोंद कटाररया ने राजभवन, गुवाहाटी, असम में क्वासलटी काउों ससल ऑफ इों सडया (QCI) िारा
सवकससत एक मोबाइल ऐप 'सरपंच संिाद' लॉन्च सकया।
• इस पहल का उद्दे श्य पूरे भारत में सरपोंचोों को व्यापक समिधन प्रदान करना और नेताओों के रूप में उनके सवकास
को प्रोत्सासहत करना है ।
• यह ऐप पूरे भारत में लगभग 2.5 लाख सरपोंचोों को जोडने के सलए तैयार है।
• यह जमीनी स्तर के नेताओों के सलए नेटवसकिंग, ज्ञान प्रसार और सहयोग के सलए एक मोंच के रूप में भी कायध
करता है।
• सरपोंच इस ऐप का उपयोग थिानीय गाोंवोों में सवकास गसतसवसर्योों को उजागर करने , सवोत्तम प्रिाओों के बारे में
जानने और पूरे भारत में कनेक्शन थिासपत करने के सलए कर सकते हैं।

3. निस राज्य सरिार िे हाि ही में (नसतंबर'23 में) सरिारी पररयोििाओं िे नििास िे निए भूनम िरीदिे
िी प्रनक्रया िो और आसाि बिािे िे निए एि िया ई-भूनम पोटा ि िॉन्च निया है ?
1)मध्य प्रदे श
2)ओसडशा
3)हररयािा
4)झारखण्ड
5)पसिम बोंगाल
उत्तर-3)हररयाणा
स्पष्टीिरण:
हररयाणा के मुख्यमोंत्री (CM) मनोहर लाल खट्टर ने सरकारी पररयोजनाओों के सवकास के सलए भूसम खरीदने की प्रसक्रया
को और आसान बनाने के सलए एक नया ई-भूनम पोटा ि लॉन्च सकया।
• उन्ोोंने हररयािा में अवैर् खनन को रोकने के सलए पहले से लॉन्च सकए गए ई-रावि पोटध ल को बदलने के सलए
HMJIS पोटध ल भी लॉन्च सकया।
• मुख्यमोंत्री ने "मुख्यमंत्री शहरी आिास योििा पोटा ि" का भी अनावरि सकया, सजसे 1.80 लाख रुपये से कम
की वासषधक आय वाले आसिधक रूप से कमजोर वगों के सलए आवास प्रदान करने के सलए सडज़ाइन सकया गया है ,
सजन्ें आवास की आवश्यकता है।
• इों डक्कस्ट्रयल मॉडल टाउनसशप (IMT) मानेसर (हररयािा) के सवस्तार के सलए वषध 2011 के भूसम असर्ग्रहि मुद्दोों
को हल करने की सदशा में एक कदम में, CM ने 'िो-निनटगेशि पॉनिसी -2023 पोटा ि' का उद् घाटन सकया।
• हररयािा सरकार ने PPP पररवार ID प्रिाली के माध्यम से अन्य सपछडा वगध (OBC) प्रमाि पत्र जारी करने की
भी शुरुआत की।

4. उस मंत्रािय िा िाम बताइए निसिे हाि ही में (नसतंबर'23 में) स्वच्छता अनभयाि 3.0 और िंनबत मामिों
िे निपटाि िे निए निशेष अनभयाि (SCDPM) 2023 िी निगरािी िे निए एि समनपात िेबसाइट िॉन्च िी
है।
1)ग्रामीि सवकास मोंत्रालय
2)आवास और शहरी मामलोों का मोंत्रालय
3)गृह मोंत्रालय
4)सवज्ञान एवों प्रौद्योसगकी मोंत्रालय

Report Errors in the PDF - ebooks@affairscloud.com Copyright 2014-2023 @ AffairsCloud.com 209


5)पोंचायती राज मोंत्रालय
उत्तर-4)निज्ञाि एिं प्रौद्योनगिी मंत्रािय
स्पष्टीिरण:
14 ससतोंबर 2023 को, सवज्ञान और प्रौद्योसगकी राज्य मोंत्री (स्वतोंत्र प्रभार) िॉ. नितेंद्र नसंह ने राष्ट्रीय मीसडया सें टर, नई
सदल्ली, सदल्ली में स्वच्छता अनभयाि 3.0 और लोंसबत मामलोों के सनपटान के सलए सवशेष असभयान (SCDPM) 2023
की सनगरानी के सलए एक समनपात िेबसाइट लॉन्च की।
i.कासमधक, लोक सशकायत और पेंशन मोंत्रालय के तहत प्रशाससनक सुर्ार और लोक सशकायत सवभाग (DARPG) इस
असभयान के कायाधन्वयन के सलए नोडल सवभाग है ।
ii.पोटध ल को राष्ट्रीय सूचना सवज्ञान केंद्र (NIC), इलेररॉसनक्स और सूचना प्रौद्योसगकी मोंत्रालय (MeitY) िारा सडजाइन,
सवकससत और होस्ट् सकया गया है।
िोट: स्वच्छता असभयान, एक जन जागरूकता असभयान, पहली बार 15 अगस्त 2014 को प्रर्ान मोंत्री (PM) नरें द्र मोदी
िारा शुरू सकया गया िा।

5. नसतंबर 2023 में, भारत िे मुख्य न्यायाधीश(CJI) D Y चंद्रचूड िे घोषणा िी नि भारत िे सिोच्च न्यायािय
(SCI) िे ई-िोटा पहि िी एि प्रमुि पररयोििा, राष्टरीय न्यानयि िे टा नग्रि (NJDG) पोटा ि िो शानमि निया
है।
NJDG पोटा ि _______________ (सरिारी िायाािय) द्वारा नििनसत निया गया है।
1)राष्ट्रीय सूचना सवज्ञान केंद्र
2)उन्नत कोंप्यूसटों ग के सवकास केंद्र
3)इमेसजोंग प्रौद्योसगकी सवकास केंद्र
4)सडसजटल इों सडया कॉपोरे शन
5)राष्ट्रीय ई-गवनेंस प्रभाग
उत्तर- 1)राष्टरीय सूचिा निज्ञाि िेंद्र
स्पष्टीिरण:
भारत के मुख्य न्यायार्ीश(CJI) D Y चोंद्रचूड ने घोषिा की सक भारत के सवोच्च न्यायालय (SCI) ने ई-कोटध पहल की
एक प्रमुख पररयोजना, राष्ट्रीय न्यासयक डे टा सग्रड (NJDG) पोटा ि को शासमल सकया है।
• NJDG पोटध ल भारत भर की सवसभन्न अदालतोों िारा शुरू सकए गए, लोंसबत और सनपटाए गए मामलोों से सोंबोंसर्त
डे टा का एक राष्ट्रीय भोंडार है ।
• NJDG पोटध ल में SCI का एकीकरि ई-कोटध पररयोजना के सफल समापन का प्रतीक है , जो भारतीय
न्यायपासलका के सभी तीन स्तरोों को NJDG प्लेटफॉमध पर लाता है।
• NJDG 'सुई जेनररस' (लैसटन में इसका अिध 'अपनी तरह का') है , इससलए NJDG पर डे टा अपलोड करने से
न्यासयक क्षेत्र में पारदसशधता और जवाबदे ही सुसनसित होगी।
• NJDG पोटध ल को राष्ट्रीय सू चना सवज्ञान केंद्र (NIC) िारा एक इों टरै क्करव इों टरफेस और एनासलसटक्स डै शबोडध के
साि SC रसजस्ट्र ी के कोंप्यूटर सेल की इन-हाउस सॉफ्टवेयर डे वलपमेंट टीम के साि सनकट समन्वय में
सवकससत सकया गया है।
• सोंपूिध डे टाबेस को समय-समय पर वास्तसवक समय के आर्ार पर NJDG पोटध ल पर अपडे ट सकया जाएगा।
NJDG-SCI पोटध ल को SC वेबसाइट के माध्यम से एक्सेस सकया जा सकता है।

Report Errors in the PDF - ebooks@affairscloud.com Copyright 2014-2023 @ AffairsCloud.com 210


6. हाि ही में (लसतंबर'23 में) लकस राज्य ने पूरे संगठन में कागि रलहत काया संिृलत स्थालपत करने के लिए
एकीकृत लबििी उपभोक्ता पोटा ि िॉन्च लकया है?
1)असम
2)श्चसस्िम
3)पश्चिम बंगाल
4)श्चहमाचल प्रदे श
5)ओश्चिशा
उत्तर-4)लहमाचि प्रदे श
स्पष्टीकरण:
लहमाचि प्रदे श(HP) के मुख्यमंत्री (CM) सुखश्चवंदर श्चसंह सुक्खू ने 15 श्चसतंबर 2023 को इं िीश्चनयर श्चदवस के अवसर पर
श्चहमाचल प्रदे श राज्य श्चबिली बोिग श्चलश्चमटे ि (HPSEBL) का एकीकृत लबििी उपभोक्ता पोटा ि लॉन्च श्चकया।
i.यह प्रयास प्रशासश्चनक प्रश्चक्रयाओं को सुव्यवस्थित करने और सेवाओं में तेिी लाने के लक्ष्य के साि पूरे संगठन में एक
कागि रश्चहत कायग संस्कृश्चत थिाश्चपत करना चाहता है।
ii.यह 31 माचग, 2026 तक हररत ऊिाग राज्य प्राप्त करने के श्चहमाचल प्रदे श के प्रयासों का एक श्चहस्सा है।

CURRENT STATIC BANKING


1. इं िसइं ि बैंि िी टै गिाइि क्ा है ?
1)वी मेक यू फील ररचर
2)योर परफेर बैंसकोंग पाटध नर
3)ऑलवेज यू फस्ट्ध
4)वी अोंडरस्ट्ैं ड योर वडध
5)गुड पीपल टू ग्रो सवद
उत्तर- 1)िी मेि यू फीि ररचर
स्पष्टीिरण:
इं िसइं ि बैंि िे बारे में :
इों डसइों ड बैंक का कर पिात लाभ सदसोंबर 2022 को समाप्त सतमाही में 68.71% बढकर 1,959.20 करोड रुपये हो
गया, जो इससे सपछली सतमाही में 1,161.27 करोड रुपये िा।
MD & CEO – सुमोंत कठपासलया
स्थापिा - 1994
मुख्यािय – मुोंबई, महाराष्ट्र
टै गिाइि - िी मेि यू फीि ररचर

2. NEFT एि राष्टरव्यापी भुगताि प्रणािी है िो एि-से-एि धि हस्तांतरण िी सुनिधा प्रदाि िरती है।
NEFT में, 'E' िा क्ा मतिब है ?
1)एक्सचेंज
2)इलेररॉसनक
3)ई-कॉमसध
4)अनेड

Report Errors in the PDF - ebooks@affairscloud.com Copyright 2014-2023 @ AffairsCloud.com 211


5)इक्कक्वटी
उत्तर- 2)इिेररॉनिि
स्पष्टीिरण:
नेशनल इिेररॉनिि फोंड टर ाोंसफर (NEFT) एक राष्ट्रव्यापी भुगतान प्रिाली है जो एक-से-एक फोंड टर ाोंसफर की
सुसवर्ा प्रदान करती है।
i.इस योजना के तहत, व्यक्कक्त, फमध और कॉपोरे ट सकसी भी बैंक शाखा से योजना में भाग लेने वाले दे श के सकसी भी
अन्य बैंक शाखा में खाता रखने वाले सकसी भी व्यक्कक्त, फमध या कॉपोरे ट को इलेररॉसनक रूप से र्न हस्ताोंतररत कर
सकते हैं।

3. हाि ही में (लसतंबर’23 में) AU स्मॉि फाइनेंस बैंक लिलमटे ि के प्रबंि लनदे शक (MD) और मुख्य कायाकारी
अलिकारी (CEO) के रूप में लकसे लनयुक्त लकया गया है?
1) इश्चत्तरा िे श्चवस
2) सश्चमत घोष
3) कदम्बेश्चलल पॉल िॉमस
4)अिय कंवल
5)संिय अग्रवाल
उत्तर-5)संिय अग्रवाि
स्पष्टीकरण:
AU स्मॉि फाइनेंस बैंक लिलमटे ि (लिलमटे ि) के बारे में:
प्रबंि लनदे शक (MD) और मुख्य कायाकारी अलिकारी (CEO) - संिय अग्रवाि
मुख्यािय - ियपुर, रािथिान
स्थालपत - 1996

4. उस भारतीय लक्रकेटर का नाम बताइए लिसे हाि ही में (लसतंबर ‘23 में) पेस्किको के स्पोट्ा स लिर ं क गेटोरे ि
का िांि एं बेसिर लनयुक्त लकया गया है।
1) KL राहुल
2)श्चवराट कोहली
3) हाश्चदगक पंड्या
4)रवींद्र िड़े िा
5)शुभमन श्चगल
उत्तर-3)हालदा क पंड्या
स्पष्टीकरण:
पेस्किको ने भारतीय श्चक्रकेटर हालदा क पंड्या को अपने स्पोट्ग स श्चिरंक गेटोरे ि के श्चलए िांि एं बेसिर के रूप में साइन
श्चकया है, यह पहली बार है श्चक श्चकसी श्चक्रकेटर द्वारा स्पोट्ग स िांि का समिगन श्चकया िाएगा।
i.इस कदम का उद्दे श्य खेल समुदाय के साि गेटोरे ि के संबंर् को मिबूत करना है। खेल और िलयोिन श्रेिी में
उल्लेखनीय वृस्ि हो रही है, खेल और ऊिाग पेय का घरे लू बािार 2028 तक 5.7 श्चबश्चलयन अमेररकी िॉलर तक पहुंचने
का अनुमान है।

Report Errors in the PDF - ebooks@affairscloud.com Copyright 2014-2023 @ AffairsCloud.com 212


5. िीलबया की राििानी क्या है?
1) मापुटो
2) श्चत्रपोली
3) मसेरू
4) बमाको
5) अबुिा
उत्तर- 2) लत्रपोिी
स्पष्टीकरण:
लीश्चबया की रािर्ानी लत्रपोिी है और लीश्चबया की मुद्रा लीश्चबयाई दीनार है।
i.लीश्चबया में भूमध्यसागरीय तूफान िै श्चनयल के कारि आई श्चवनाशकारी बा़ि के कारि लगभग 2,000 लोगों के मारे
िाने की आशंका है, श्चिससे पूरे पड़ोस बह गए और उत्तरी अफ्रीकी राष्ट्र के कई तटीय शहरों में घर बबाग द हो गए।
ii.स्वास्थ्य अश्चर्काररयों के अनुसार, सप्ताहांत में आई बा़ि से मरने वालों की पुश्चष्ट् की गई संख्या 38 है। लेश्चकन सूची में
सबसे अश्चर्क प्रभाश्चवत शहर िनाग शाश्चमल नहीं है, िो दु गगम हो गया है।

6. उस दे श का नाम बताइए लिसने हाि ही में (लसतंबर’23 में) बायो-लफलिकि लसमुिेशन के माध्यम से
कालिा योवािुिर साइं स में लिलिटि पररवतान के लिए िसॉल्ट् लसस्ट्म्स के साथ सहयोग लकया है ।
1) इं फोश्चसस श्चलश्चमटे ि
2) HCL टे क्नोलॉिीि श्चलश्चमटे ि
3)श्चवप्रो श्चलश्चमटे ि
4)टे क मश्चहंद्रा
5) टाटा कंसल्ट्ें सी सश्चवगसेि
उत्तर-5)टाटा कंसल्ट्ें सी सलवासेि
स्पष्टीकरण:
IT-प्रमुख टाटा कंसल्ट्ें सी सश्चवगसेि (TCS) ने घोषिा की श्चक वह बायो-श्चफश्चिकल श्चसमुलेशन के माध्यम से
काश्चिगयोवास्कुलर साइं स में श्चिश्चिटल पररवतगन के श्चलए िसॉल्ट् श्चसस्ट्म्स के साि सहयोग कर रही है ।
i.TCS वचुगअल ह्यूमन श्चिन के लक्ष्य के श्चलए िसॉल्ट् श्चसस्ट्म्स और अन्य के साि भी सहयोग करे गा, श्चिसमें क्षेत्र की
उन्नश्चत के श्चलए महत्वपूिग पहचाने गए चुश्चनंदा अंगों पर ध्यान केंश्चद्रत श्चकया िाएगा।

7. लसतंबर 2023 में, सू बिा को ___________ में FIBA मलहिा बािेटबॉि लवश्व कप 2026 के वैलश्वक रािदू त के
रूप में नालमत लकया गया था।
1)न्यूयॉकग, संयुक्त राज्य अमेररका
2) लंदन, यूनाइटे ि श्चकंगिम
3) मनीला, श्चफलीपींस
4)बश्चलगन, िमगनी
5) िकाताग, इं िोनेश्चशया
उत्तर-4)बलिान, िमानी
स्पष्टीकरण:
अमेररकी पूवग बास्केटबॉल स्खलाड़ी सुज़ैन लिलगट बिा (सू बिा ) को बश्चलगन, िमगनी में FIBA मलहिा बािेटबॉि लवश्व
कप 2026 के श्चलए वैश्चश्वक रािदू त के रूप में नाश्चमत श्चकया गया है।

Report Errors in the PDF - ebooks@affairscloud.com Copyright 2014-2023 @ AffairsCloud.com 213


i.टीम USA की चार बार की श्चवश्व कप चैंश्चपयन को FIBA द्वारा 4 से 13 श्चसतंबर 2026 तक मश्चहला बास्केटबॉल वैश्चश्वक
तमाशा के चेहरे के रूप में पेश श्चकया गया िा।

8. स्थायी सेवालनवृलत्त िाता संख्या (PRAN) राष्टरीय पेंशन प्रणािी के तहत राज्य और केंद्रीय कमाचाररयों को
िीवन भर पेंशन प्राप्त करने के लिए िारी की गई एक अलद्वतीय ____ अंक संख्या है।
1) 7
2) 12
3) 10
4) 9
5) 11
उत्तर- 2) 12
स्पष्टीकरण:
थिायी सेवाश्चनवृश्चत्त खाता संख्या (PRAN) राज्य और केंद्रीय कमगचाररयों को राष्ट्रीय पेंशन प्रिाली (NPS) के तहत िीवन
भर पेंशन प्राप्त करने के श्चलए िारी श्चकया गया एक अश्चद्वतीय 12 अंकों का नंबर है। (यह NPS योिना 2004 में शुरू
की गई िी)।
i.यह भारत की संसद के एक अश्चर्श्चनयम द्वारा श्चनश्चमगत पेंशन फंि रे ग्युलेटरी ऐंि श्चिवेलपमेंट अिॉररटी (PFRDA) द्वारा
श्चवश्चनयश्चमत है।

9. भारतीय नित्तीय प्रणािी िोि (IFSC) िोि 11 अल्फ़ान्यूमेररि िोि है।


IFSC िोि में 5िााँ अक्षर क्ा है ?
1)चार
2)एक
3)शून्य
4)दो
5)पााँच
उत्तर-3)शून्य
स्पष्टीिरण:
भारतीय सवत्तीय प्रिाली कोड (IFSC) के 11 अल्फ़ान्यूमेररक कोड को एक पैटनध में सोंरसचत सकया गया है जहाों पहले
चार अक्षर बैंक के नाम का प्रसतसनसर्त्व करते हैं , जबसक अोंसतम छह अक्षर बैंक की शाखा का प्रसतसनसर्त्व करते हैं।
पााँचवााँ अक्षर आम तौर पर भसवष्य में उपयोग के सलए 0 (शून्य) आरसक्षत होता है ।
i.IFSC एक 11 अोंकोों का अल्फ़ान्यूमेररक कोड है , सजसमें पहले चार अोंक बैंक की पहचान करते हैं , पााँचवााँ सोंख्यात्मक
है (0 रखा गया है ) और अोंसतम छह अोंक बैंक शाखा का प्रसतसनसर्त्व करते हैं। उदाहरि: सदल्ली में पोंजाब नेशनल बैंक
की एक शाखा का IFS कोड PUNB0614800 है।
ii.यह एक अल्फ़ान्यूमेररक कोड है जो भारत में दो मुख्य इलेररॉसनक फोंड सेटलमेंट ससस्ट्म: ररयल टाइम ग्रॉस
सेटलमेंट (RTGS) और नेशनल इलेररॉसनक फोंड टर ाोंसफर (NEFT) ससस्ट्म में भाग लेने वाले बैंक-शाखा की सवसशष्ट्
पहचान करता है।

Report Errors in the PDF - ebooks@affairscloud.com Copyright 2014-2023 @ AffairsCloud.com 214


10. हाि ही में (नसतंबर'23 में) निस िंपिी िे DP नसंह िो उप प्रबंध निदे शि और संयुक्त मुख्य िायािारी
अनधिारी (CEO) नियुक्त निया है ?
1)श्रीराम एसेट मैनेजमेंट
2)UTI म्यूचुअल फोंड
3)ICICI प्रूडेंसशयल म्यूचुअल फोंड
4)SBI फोंड् स मैनेजमेंट
5)HDFC एसेट मैनेजमेंट
उत्तर-4)SBI फंि् स मैिेिमेंट
स्पष्टीिरण:
सबसे बडी पररसोंपसत्त प्रबोंर्न कोंपनी SBI फंि् स मैिेिमेंट ने DP नसंह को उप प्रबंध निदे शि और सोंयुक्त मुख्य
िायािारी अनधिारी (CEO) के पद पर पदोन्नत सकया है।
i.अपनी नई भूसमका में, वह सोंगठन के सभी व्यावसासयक कायधक्षेत्रोों और क्षेत्रोों की दे खरे ख करना जारी रखेंगे। म्यूचुअल
फोंड उद्योग के सदग्गज, उन्ोोंने SBI म्यूचुअल फोंड के साि 25 साल सबताए हैं और कई वररष्ठ नेतृत्व भूसमकाएाँ सनभाई
हैं।
ii.उन्ोोंने शहरी और ग्रामीि दोनोों क्षेत्रोों में फोंड हाउस की पहुोंच बढाने में महत्वपूिध भूसमका सनभाई है। वह पहले AMC
में उप प्रबोंर् सनदे शक और मुख्य व्यवसाय असर्कारी िे।

11. ICICI िोम्बािा ििरि इं श्योरें स निनमटे ि िी स्थापिा निस िषा हुई थी?
1)2004
2)2001
3)2010
4)2000
5)2005
उत्तर- 2)2001
स्पष्टीिरण:
ICICI िोम्बािा ििरि इं श्योरें स निनमटे ि िे बारे में :
प्रबंध निदे शि एिं मुख्य िायािारी अनधिारी (CEO)-भागधव दासगुप्ता
मुख्यािय – मुोंबई, महाराष्ट्र
स्थानपत – 2001

12. इं नियास इं टरिेशिि बैंि __________ (बैंि) िी टै गिाइि है।


1)भारतीय स्ट्े ट बैंक
2)बैंक ऑफ इों सडया
3)पोंजाब राष्ट्रीय उद्यान
4)इों सडयन बैंक
5)बैंक ऑफ बडौदा
उत्तर-5)बैंि ऑफ बडौदा

Report Errors in the PDF - ebooks@affairscloud.com Copyright 2014-2023 @ AffairsCloud.com 215


स्पष्टीिरण:
बैंि ऑफ बडौदा (BoB) िे बारे में:
प्रबंध निदे शि (MD) और मुख्य िायािारी अनधिारी (CEO) - सोंजीव चड्ढा
मुख्यािय – वडोदरा, गुजरात (मुख्य कायाध लय), मुोंबई, महाराष्ट्र (कॉपोरे ट केंद्र)
स्थापिा - 20 जुलाई 1908
टै ग िाइि - इं नियास इं टरिेशिि बैंि

13. इं िसइं ि बैंि िी टै गिाइि क्ा है ?


1)वी अोंडरस्ट्ैं ड योर वडध
2)सपोटध ऑल द वे
3)वी मेक यू फील ररचर
4)इों सडयाज इों टरनेशनल बैंक
5)योर परफेर बैंसकोंग पाटध नर
उत्तर- 3)िी मेि यू फीि ररचर
स्पष्टीिरण:
इं िसइं ि बैंि निनमटे ि िे बारे में :
प्रबंध निदे शि (MD) और मुख्य िायािारी अनधिारी (CEO) - सुमोंत कठपासलया
मुख्यािय – मुोंबई, महाराष्ट्र
स्थापिा – 1994
टै गिाइि - िी मेि यू फीि ररचर

14. पंिाब िेशिि बैंि (PNB) िी स्थापिा निस िषा हुई थी?
1)1934
2)1956
3)1880
4)1894
5)1945
उत्तर- 4)1894
स्पष्टीिरण:
पंिाब िेशिि बैंि (PNB) िे बारे में:
MD & CEO– अतुल कुमार गोयल
मुख्यािय– नई सदल्ली, सदल्ली
टै गिाइि- द नेम यू कैन BANK अपॉन!
स्थापिा- 1894

15. ििााटि बैंि िी टै गिाइि क्ा है ?


1)वी अोंडरस्ट्ैं ड योर वडध
2)सपोटध ऑल द वे
3)योर फैसमली बैंक अक्रॉस इों सडया

Report Errors in the PDF - ebooks@affairscloud.com Copyright 2014-2023 @ AffairsCloud.com 216


4)इों सडयाज इों टरनेशनल बैंक
5)योर परफेर बैंसकोंग पाटध नर
उत्तर-3)योर फैनमिी बैंि अक्रॉस इं निया
स्पष्टीिरण:
ििााटि बैंि निनमटे ि (KBL) िे बारे में:
प्रबंध निदे शि और मुख्य िायािारी अनधिारी– श्रीकृष्णन हरर हारा सरमा
मुख्यािय– मोंगलुरु, कनाधटक
टै गिाइि-योर फैनमिी बैंि अक्रॉस इं निया
स्थापिा- 1924

16. _________ िह रे ट है निस पर भारतीय ररििा बैंि (RBI) िानणखज्यि बैंिों से पैसा उधार िेता है।
1)रे पो रे ट
2)ररवसध रे पो रे ट
3)वैर्ासनक तरलता अनुपात
4)बैंक रे ट
5)नकद आरसक्षत अनुपात
उत्तर- 2)ररिसा रे पो रे ट
स्पष्टीिरण:
रे पो रे ट - रे पो रे ट वह रे ट है सजस पर RBI वासिक्कज्यक बैंकोों को सरकारी प्रसतभूसतयोों के बदले ऋि दे ता है ।
ररिसा रे पो दर - ररवसध रे पो रे ट वह रे ट है सजस पर RBI वासिक्कज्यक बैंकोों से पैसा उर्ार लेता है।

CA STATIC GK
1. पंिाब िेशिि बैंि (PNB) िे ितामाि (अगस्त’23 ति) प्रबंध निदे शि (MD) और मुख्य िायािारी
अनधिारी (CEO) िौि हैं?
1)अतनु कुमार दास
2)शसशर्र जगदीशन
3)चोंद्र शेखर घोष
4)अतुल कुमार गोयल
5)A S राजीव
उत्तर-4)अतुि िुमार गोयि
स्पष्टीिरण:
पंिाब िेशिि बैंि (PNB) िे बारे में:
भारत के पहले स्वदे शी बैंक PNB ने 12 अप्रैल, 1895 को अपना पररचालन शुरू सकया।
प्रबंध निदे शि और मुख्य िायािारी अनधिारी– अतुि िुमार गोयि
मुख्यािय– नई सदल्ली, सदल्ली
स्थानपत- 19 मई, 1894
टै गिाइि- द नेम यू कैन बैंक अपॉन

Report Errors in the PDF - ebooks@affairscloud.com Copyright 2014-2023 @ AffairsCloud.com 217


2. महाराणा प्रताप हिाई अड्डा निस राज्य में खस्थत है ?
1)राजथिान
2)झारखण्ड
3)पसिम बोंगाल
4)ओसडशा
5)मध्य प्रदे श
उत्तर- 1)रािस्थाि
स्पष्टीिरण:
रािस्थाि िे बारे में:
मुख्यमंत्री– अशोक गहलोत
राज्यपाि– कलराज समश्र
हिाई अड्डे – जयपुर अोंतराधष्ट्रीय हवाई अड्डा और महाराणा प्रताप हिाई अड्डा

3. िेशिि पेमेंट्स िॉरपोरे शि ऑफ इं निया (NPCI) िे ितामाि (अगस्त’23 ति) प्रबंध निदे शि और मुख्य
िायािारी अनधिारी (CEO) िौि हैं ?
1)ऋसषकेश कृष्णन
2)सदलीप अस्बे
3)सवश्वमोहन महापात्रा
4)शैलेन्द्र सत्रवेदी
5)असमता सहगल
उत्तर- 2)नदिीप अस्बे
स्पष्टीिरण:
िेशिि पेमेंट्स िॉरपोरे शि ऑफ इं निया (NPCI)िे बारे में:
NPCI भारत में एक मजबूत भुगतान और सनपटान बुसनयादी ढाोंचा बनाने के सलए भुगतान और सनपटान प्रिाली
असर्सनयम, 2007 के प्रावर्ानोों के तहत भारतीय ररजवध बैंक (RBI) और भारतीय बैंक सोंघ (IBA) की एक पहल है।
प्रबंध निदे शि और मुख्य िायािारी अनधिारी - नदिीप अस्बे
मुख्यािय– मुोंबई, महाराष्ट्र
शानमि - 2008

4. निस राज्य सरिार िे हाि ही में (अगस्त'23 में) 0-5 िषा िी आयु िे बच्चों िे निए िन्म पंिीिरण िी
सुनिधा िे निए आधार निंक्ि िन्म पंिीिरण (ALBR) पहि शुरू िी है ?
1)अरुिाचल प्रदे श
2)नागालैंड
3)समजोरम
4)असम
5)सत्रपुरा
उत्तर- 2)िागािैंि

Report Errors in the PDF - ebooks@affairscloud.com Copyright 2014-2023 @ AffairsCloud.com 218


स्पष्टीिरण:
िागािैंि सरकार ने 0-5 िषा की आयु के बच्चोों के सलए जन्म पोंजीकरि की सुसवर्ा के सलए आर्ार सलोंक्ड जन्म
पोंजीकरि (ALBR) पहल शुरू की है।
i.सेवा का उद्दे श्य यह सुसनसित करना है सक कोई भी बच्चा सामासजक कल्ाि योजनाओों से वोंसचत न रहे। नागालैंड में
अिधशास्त्र और साोंक्कख्यकी सनदे शालय को UIDAI िाराALBR प्रसक्रया का उपयोग करके बच्चोों का नामाोंकन करने के
सलए रसजस्ट्र ार सह नामाोंकन एजेंसी के रूप में शासमल सकया गया है।

5. उस बैंि िा िाम बताइए िो हाि ही में (अगस्त'23 में) भारतीय ररििा बैंि (RBI) सेंटरि बैंि निनिटि
िरें सी (CBDC) ऐप पर यूनिफाइि पेमेंट्स इं टरफेस (UPI) इं टरऑपरे नबनिटी िे साथ िाइि हुआ है।
1)RBL बैंक
2)बोंर्न बैंक
3)YES बैंक
4)IDBI बैंक
5)इों डसइों ड बैंक
उत्तर-3)YES बैंि
स्पष्टीिरण:
YES बैंि ने घोषिा की सक वह भारतीय ररजवध बैंक (RBI) सेंटरल बैंक सडसजटल करें सी (CBDC) ऐप पर यूसनफाइड
पेमेंट इों टरफेस (UPI) इों टरऑपरे सबसलटी के साि लाइव हो गया है।
i.इस कदम के साि, उपयोगकताध अब YES बैंक सडसजटल रुपया ऐप के माध्यम से सकसी भी UPI QR िोि को स्कैन
कर सकेंगे और सनबाधर् तरीके से लेनदे न कर सकेंगे।
ii.यह कदम सडसजटल रुपए (e₹) की पहुोंच को महत्वपूिध रूप से सवस्ताररत करने के सलए तैयार है और सडसजटल
नवाचारोों को बढावा दे ने और दे श भर में उपयोगकताध ओों के सलए लेनदे न सवकल्पोों की एक सवस्तृत श्रृोंखला प्रदान करने
के प्रसत बैंक की प्रसतबद्धता को रे खाोंसकत करता है।
iii.सडसजटल रुपया-िोक खोंड (e रुपया-W) में पहला पायलट 1 नवोंबर, 2022 को शुरू हुआ। इसके बाद खुदरा
सडसजटल रुपया (ई रुपया-R) के सलए पहला पायलट 1 सदसोंबर, 2022 को घोसषत सकया गया।

6. अगस्त 2023 में, भारतीय अंतररक्ष अिुसंधाि संगठि (ISRO) िे पूिा अध्यक्ष िॉ K नसिि िो तीि साि िे
निए भारतीय प्रौद्योनगिी संस्थाि (IIT) िे बोिा ऑफ गििासा िे अध्यक्ष िे रूप में नियुक्त निया गया था।
1)IIT मद्रास
2)IIT बॉम्बे
3)IIT गुवाहाटी
4)IIT खडगपुर
5)IIT इों दौर
उत्तर-5)IIT इं दौर
स्पष्टीिरण:
प्रख्यात अोंतररक्ष वैज्ञासनक और भारतीय अोंतररक्ष अनुसोंर्ान सोंगठन (ISRO) के पूवध अध्यक्ष डॉ. K ससवन को तीन साल
के सलए भारतीय प्रौद्योसगकी सोंथिान (IIT) इं दौर के बोडध ऑफ गवनधसध का अध्यक्ष सनयुक्त सकया गया है ।
i.डॉ. ससवन ने प्रोफेसर दीपक B फाटक की जगह ले ली है सजनका कायधकाल 21 अगस्त, 2023 को समाप्त हो गया।

Report Errors in the PDF - ebooks@affairscloud.com Copyright 2014-2023 @ AffairsCloud.com 219


ii.बेंगलुरु मुख्यालय वाले भारतीय अोंतररक्ष अनुसोंर्ान सोंगठन (ISRO) ने अपने चोंद्र अन्वेषि समशन चोंद्रयान -3 की
सफलता के बाद भारत को दे शोों की एक सवसशष्ट् लीग में पहुोंचा सदया। भारत चोंद्रमा के दसक्षिी ध्रुव के पास
सफलतापूवधक अोंतररक्ष यान उतारने वाला पहला दे श भी बन गया।
iii.डॉ ससवन ने 2018 से 2022 तक ISRO का नेतृत्व सकया और 22 जुलाई, 2019 को लॉन्च सकए गए चोंद्रयान -2
समशन के प्रभारी िे।

7. अगस्त 2023 में, निटे ि िी रािी िैनमिा िे रॉयि एयर फोसा (RAF) िब में उििे नचत्र िा अिािरण
िरिे भारतीय मूि िे िासूस और टीपू सुल्ताि िे िंशि _______________ िो श्रिांिनि अनपात िी।
1)अमीना बेगम
2)नूर इनायत खान
3)सारा मेगन
4)सजया इनायत खान
5)एों डर ी बोरे ल
उत्तर-2)िूर इिायत िाि
स्पष्टीिरण:
निटे ि िी रािी िैनमिा ने रॉयल एयर फोसध (RAF) िब में भारतीय मूल के जासूस और टीपू सुल्तान के वोंशज िूर
इिायत िाि के सचत्र का अनावरि करके उन्ें श्रद्धाोंजसल दी।
• 76 वषीय वररष्ठ शाही ने आसर्काररक तौर पर उनके सम्मान में RAF क्लब में एक कमरे का नाम भी रखा।
RAF की मसहला सहायक वायु सेना (WAAF) की सदस्य नूर इनायत खान सितीय सवश्व युद्ध के दौरान सब्रटे न के
सवशेष सोंचालन कायधकारी(SOE) में शासमल हुईों, अपनी असार्ारि वीरता और चरम पररक्कथिसतयााँ में साहस के
सलए जॉजध क्रॉस प्राप्त करने वाले दो WAAF सदस्योों में से एक बन गईों।
• सचत्र अनावरि समारोह में सब्रसटश भारतीय लेक्कखका श्रबानी बसु ने भाग सलया, सजन्ोोंने रानी कैसमला को नूर पर
उनकी जीवनी की एक प्रसत भेंट की।

8. 'िौरादे ही िन्यिीि अभयारण्य' निस राज्य में खस्थत है ?


1)सहमाचल प्रदे श
2)उत्तराखोंड
3)महाराष्ट्र
4)अरुिाचल प्रदे श
5)मध्य प्रदे श
उत्तर-5)मध्य प्रदे श
स्पष्टीिरण:
मध्य प्रदे श िे बारे में:
मुख्यमंत्री–सशवराज ससोंह चौहान
राज्यपाि– मोंगुभाई छगनभाई पटे ल
राष्टरीय उद्याि– कुनो राष्ट्रीय उद्यान; घुघवा राष्ट्रीय जीवाश्म पाकध
िन्यिीि अभयारण्य- िौरादे ही िन्यिीि अभयारण्य; ओरछा वन्यजीव अभयारण्य

Report Errors in the PDF - ebooks@affairscloud.com Copyright 2014-2023 @ AffairsCloud.com 220


9. 'मेिघाट टाइगर ररििा' निस राज्य में खस्थत है ?
1)गुजरात
2)राजथिान
3)महाराष्ट्र
4)झारखण्ड
5)मध्य प्रदे श
उत्तर-3)महाराष्टर
स्पष्टीिरण:
महाराष्टर िे बारे में:
मुख्यमंत्री - एकनाि सशोंदे
राज्यपाि – रमेश बैस
िन्यिीि अभयारण्य (WLS) - नागजीरा WLS; सचखलदरा WLS
टाइगर ररििा - मेिघाट टाइगर ररििा; सह्याद्री टाइगर ररजवध

10. भारतीय िघु उद्योग नििास बैंि (SIDBI) िे ितामाि (नसतंबर’23 ति) अध्यक्ष और प्रबंध निदे शि िौि
हैं?
1)आशीष गुप्ता
2)दे वेन्द्र कुमार ससोंह
3)सशवसुब्रमण्यन रमन
4)पोंकज जैन
5)सुदत्त मोंडल
उत्तर-3)नशिसुिमण्यि रमि
स्पष्टीिरण:
भारतीय िघु उद्योग नििास बैंि (SIDBI) िे बारे में:
अध्यक्ष & प्रबंध निदे शि– नशिसुिमण्यि रमि
मुख्यािय– लखनऊ, उत्तर प्रदे श
स्थापिा- 1990

11. 'महुआदािर िन्यिीि अभयारण्य' निस राज्य में खस्थत है ?


1)उत्तर प्रदे श
2)पसिम बोंगाल
3)ओसडशा
4)झारखण्ड
5)राजथिान
उत्तर-4)झारिंि
स्पष्टीिरण:
झारिंि िे बारे में:
मुख्यमंत्री (CM)-हेमोंत सोरे न
राज्यपाि – C.P. रार्ाकृष्णन

Report Errors in the PDF - ebooks@affairscloud.com Copyright 2014-2023 @ AffairsCloud.com 221


िन्यिीि अभयारण्य - पालकोट वन्यजीव अभयारण्य; महुआदािर िन्यिीि अभयारण्य
टाइगर ररििा- पलामू टाइगर ररजवध

12. 'िल्लािािु ब्लैि बि अभयारण्य' निस राज्य में खस्थत है ?


1)आों ध्र प्रदे श
2)तसमलनाडु
3)केरल
4)कनाधटक
5)तेलोंगाना
उत्तर- 2)तनमििािु
स्पष्टीिरण:
तनमििािु (TN) िे बारे में:
मुख्यमंत्री– मुिुवेल करुिासनसर् स्ट्ासलन
राज्यपाि– रवीोंद्र नारायि रसव
िन्यिीि अभयारण्य- िल्लािािु ब्लैि बि अभयारण्य, सग्रजड जाइों ट क्किरल वन्यजीव अभयारण्य, कन्याकुमारी
वन्यजीव अभयारण्य, सत्यमोंगलम वन्यजीव अभयारण्य
टाइगर ररििा- मुदुमलाई टाइगर ररजवध; अनामलाई टाइगर ररजवध

13. 'पािुई टाइगर ररििा' निस राज्य में खस्थत है ?


1)ससक्किम
2)समजोरम
3)अरुिाचल प्रदे श
4)मसिपुर
5)असम
उत्तर-3)अरुणाचि प्रदे श
स्पष्टीिरण:
अरुणाचि प्रदे श िे बारे में :
मुख्यमंत्री- पेमा खाोंडू
राज्यपाि- लेक्कफ्टनेंट जनरल (सेवासनवृत्त) कैवल् सत्रसवक्रम परनायक
िन्य िीिि अभयारण्य- सदबाोंग वन्यजीव अभयारण्य; पिे वन्यजीव अभयारण्य
टाइगर ररििा- नामदाफा टाइगर ररजवध; पािुई टाइगर ररििा

14. निम्ननिखित में से िौि सी झीि िद्दाि िा पहिा िैि निनिधता निरासत स्थि है ?
1)मानसबल झील
2)नाको झील
3)हाफलोोंग झील
4)सतसलयार झील
5)यायाल त्सो झी
उत्तर-5)याया त्सो झीि

Report Errors in the PDF - ebooks@affairscloud.com Copyright 2014-2023 @ AffairsCloud.com 222


स्पष्टीिरण:
याया त्सो झीि जैसवक सवसवर्ता असर्सनयम, 2002 के तहत घोसषत िद्दाि िी पहिी िैि निनिधता निरासत स्थि
है।
i.4,820 मीटर की ऊोंचाई पर क्कथित अपनी खूबसूरत झील के कारि इसे पसक्षयोों का स्वगध कहा जाता है।

15. न्यूखियर पािर िॉरपोरे शि ऑफ इं निया निनमटे ि (NPCIL) िे ितामाि (23’नसतंबर ति) अध्यक्ष और
प्रबंध निदे शि (CMD) िौि हैं ?
1)सोंजय कुमार
2)सुरेश बाबू
3)सवश्वदीप डे
4)राजनाि राम
5)भुवन चोंद्र पाठक
उत्तर-5)भुिि चंद्र पाठि
स्पष्टीिरण:
न्यूखियर पािर िॉपोरे शि ऑफ इं निया निनमटे ि (NPCIL) िे बारे में:
अध्यक्ष एिं प्रबंध निदे शि– भुिि चंद्र पाठि
मुख्यािय– मुोंबई, महाराष्ट्र
स्थापिा- 1987

16. नमस्र _________ नमस्र िी मुद्रा है।


1)पेसो
2)यूरो
3)डॉलर
4)ररयाल
5)पाउों ड
उत्तर- 5)पाउं ि
स्पष्टीिरण:
नमस्र िे बारे में:
प्रधाि मंत्री– मुस्तफा मैडबौली
रािधािी– कासहरा
मुद्रा– नमस्र पाउं ि (EGP)

17. इं नियि पोस्ट् िे ितामाि (23’नसतंबर ति) महानिदे शि िाि सेिाएाँ िौि हैं ?
1)गौरव कुमार सैनी
2)आलोक शमाध
3)नीरज मेहरोत्रा
4)असभषेक कुमार ससोंह
5)अजय ससोंह
उत्तर- 2)आिोि शमाा

Report Errors in the PDF - ebooks@affairscloud.com Copyright 2014-2023 @ AffairsCloud.com 223


स्पष्टीिरण:
इं निया पोस्ट् िे बारे में :
इों सडया पोस्ट् भारत में सरकार िारा सोंचासलत डाक प्रिाली है और सोंचार मोंत्रालय के तहत डाक सवभाग का व्यापार
नाम है।
महानिदे शि िाि सेिाएाँ – आिोि शमाा
मुख्यािय– नई सदल्ली, सदल्ली

18. गैबॉि िी रािधािी क्ा है ?


1)मोरोनी
2)बोंजुल
3)सबसाऊ
4)श्चलिेश्चवल
5)मलाबो
उत्तर-4)लििेलवि
स्पष्टीिरण:
गैबॉि िे बारे में:
रािधािी– नििेनिि
मुद्रा– मध्य अफ्रीकी CFA फ्रैंक

19. ICICI प्रूिेंनशयि िाइफ इं श्योरें स िंपिी निनमटे ि िे ितामाि (नसतंबर’23 ति) प्रबंध निदे शि (MD)
और मुख्य िायािारी अनधिारी (CEO) िौि हैं ?
1)सौसवक जश
2)जुर्ाजीत दास
3)अनूप बागची
4)मनीष कुमार
5)असमत पलटा
उत्तर-3)अिूप बागची
स्पष्टीिरण:
ICICI प्रूिेंनशयि िाइफ इं श्योरें स िंपिी निनमटे ि (ICICI प्रूिेंनशयि िाइफ) िे बारे में:
ICICI प्रूडेंसशयल लाइफ इों श्योरें स कोंपनी सलसमटे ड (ICICI प्रूडेंसशयल लाइफ) को ICICI बैंक सलसमटे ड और प्रूडेंसशयल
कॉपोरे शन होक्कडोंग्स सलसमटे ड िारा प्रवसतधत सकया जाता है।
प्रबंध निदे शि (MD) और मुख्य िायािारी अनधिारी (CEO) - अिूप बागची
मुख्यािय – मुोंबई, महाराष्ट्र
स्थानपत – 2001

20. निम्ननिखित में से िौि सा शहर FIFA निश्व िप 2026 और AFC एनशयाई िप 2027 प्रारं नभि संयुक्त
योग्यता दौर 2 में भारत िे शुरुआती दो घरे िू मैचों िी मेिबािी िरे गा?
1)नई सदल्ली, सदल्ली
2)भुवनेश्वर, ओसडशा

Report Errors in the PDF - ebooks@affairscloud.com Copyright 2014-2023 @ AffairsCloud.com 224


3)गुवाहाटी, असम
4)1 और 2 दोनोों
5)2 और 3 दोनोों
उत्तर- 5)2 और 3 दोिों
स्पष्टीिरण:
अक्कखल भारतीय फुटबॉल महासोंघ ने घोषिा की, भुििेश्वर (ओनिशा) और गुवाहाटी (असम) FIFA निश्व िप 2026
और AFC एसशयाई कप 2027 प्रारों सभक सोंयुक्त योग्यता दौर 2 में भारत के शुरुआती दो घरे लू मैचोों की मेजबानी करें गे।
i.भारत को एसशयाई क्वालीफायर के ग्रुप A में कतर, कुवैत और अफगासनस्तान और मोंगोसलया के बीच प्रारों सभक सोंयुक्त
क्वालीसफकेशन राउों ड 1 के सवजेताओों के साि रखा गया है।
ii.िू टाइगसध 21 नवोंबर 2023 को भुवनेश्वर के कसलोंगा स्ट्े सडयम में मौजूदा AFC एसशयाई कप चैंसपयन कतर की
मेजबानी करने से पहले 16 नवोंबर को कुवैत के क्कखलाफ अपने असभयान की शुरुआत करें गे।
iii.2024 में, भारत अफगासनस्तान या मोंगोसलया के क्कखलाफ बैक-टू -बैक मैच खेलेगा, सजसकी शुरुआत 21 माचध को
होगी और सफर गुवाहाटी के इों सदरा गाोंर्ी स्ट्े सडयम में घर लौट आएगा, जहाों वे 26 माचध को मुकाबले का दू सरा चरि
खेलेंगे।

21. हाि ही में (नसतंबर’23 में) नफि एं ि टे िीनििि इं स्ट्ीट्यूट ऑफ इं निया में अध्यक्ष पद िे निए निसे
िामांनित निया गया है ?
1)R मार्वन
2)सववेक रों जन असग्नहोत्री
3)सनक्कखल महाजन
4)पोंकज सत्रपाठी
5)अरुि अशोक
उत्तर- 1)R माधिि
स्पष्टीिरण:
अनुभवी असभनेता R माधिि, सजनकी सफल् 'रॉिेटर ी' को राष्ट्रीय पुरस्कार समला िा, को अब सफल् एों ड टे लीसवजन
इों स्ट्ीट्यूट ऑफ इों सडया में अध्यक्ष पद के सलए नामाोंसकत सकया गया है। वह 'समस्ट्र इों सडया' के सनदे शक शेखर कपूर
की जगह लेंगे।

22. हाि ही में (नसतंबर’23 में) निस शहर में िेंद्रीय िपडा मंत्री पीयूष गोयि िे स्वतंत्र भारत िे पहिे नित्त
मंत्री RK शिमुगम चेट्टी िी प्रनतमा िा अिािरण निया है ?
1)हैदराबाद, तेलोंगाना
2)बेंगलुरु, कनाधटक
3)कोयोंबटू र, तसमलनाडु
4)कोक्कच्च, केरल
5)मुोंबई, महाराष्ट्र
उत्तर-3)िोयंबटू र, तनमििािु
स्पष्टीिरण:
केंद्रीय कपडा मोंत्री पीयूष गोयल ने आज तनमििािु िे िोयंबटू र में साउि इों सडयन समल्स एसोससएशन कॉम्प्लेक्स
(SIMA) में स्वतोंत्र भारत के पहले सवत्त मों त्री R K शनमुगम चेट्टी की प्रसतमा का अनावरि सकया।

Report Errors in the PDF - ebooks@affairscloud.com Copyright 2014-2023 @ AffairsCloud.com 225


• इस अवसर पर केंद्रीय रे ल और कपडा राज्य मोंत्री श्रीमती दशधना V. झारदोश, कोयोंबटू र दसक्षि MLA श्रीमती
वनिी श्रीसनवासन और साउि इों सडयन समल्स एसोससएशन के वररष्ठ असर्कारी भी उपक्कथित िे।
• R K शनमुगम चेट्टी साउि इों सडयन समल्स एसोससएशन की थिापना के पीछे एक प्रेरक शक्कक्त िे , एक ऐसा
सोंगठन सजसने दसक्षि भारत में कपडा उद्योग को आकार दे ने में महत्वपूिध भूसमका सनभाई है।
• यह दे श के दसक्षिी क्षेत्र में कपडा उद्योग के सहतोों को बढावा दे ने के सलए समसपधत एक प्रसतसष्ठत सोंगठन है। R K
शनमुगम चेट्टी िारा थिासपत, यह क्षेत्र के भीतर सवकास और नवाचार को बढावा दे ने में एक महत्वपूिध क्कखलाडी
बना हुआ है।

23. नसतंबर 2023 में, भारतीय ग्रैंिमास्ट्र ______________ िे निश्विाथि आिंद िो भारत िे शीषा शतरं ि
खििाडी िे रूप में प्रनतस्थानपत निया।
1)रमेशबाबू प्रग्गनानोंद
2)D गुकेश
3)अजुधन एररगैसी
4)सवसदत गुजराती
5)अनीश सगरी
उत्तर- 2)D गुिेश
स्पष्टीिरण:
सकशोर भारतीय ग्रैंडमास्ट्र D गुिेश तीन दशक से असर्क समय के बाद भारत के शीषध शतरों ज क्कखलाडी के रूप में
महान सवश्वनािन आनोंद की जगह ले लेंगे।
• आनोंद जुलाई 1986 से भारत के नोंबर 1 रहे हैं।
i.चेन्नई के 17 वषीय GM, जो हाल ही में बाकू में सफडे सवश्व कप के क्वाटध र फाइनल में मैग्नस कालधसन से हार गए िे ,
आनोंद से आगे सनकलकर सवश्व में 8वें नोंबर पर पहुोंच गए। गुकेश पहली बार FIDE रे सटों ग सूची के शीषध 10 में शासमल
हुए।
ii.पाोंच बार के सवश्व चैंसपयन आनोंद वतधमान में 9वें थिान पर हैं। 1 ससतोंबर से प्रभावी FIDE रे सटों ग के अनुसार, गुकेश की
रे सटों ग 2758 है जबसक आनोंद की 2754 है।

24. अगस्त 2023 में, __________ (िािेनियि फुटबॉिर) िे सिाश्रेष्ठ पुरुष खििाडी िे निए UEFA पुरस्कार
िीता और स्पेि िी ऐटािा बोिमती िे सिाश्रेष्ठ मनहिा खििाडी िा पुरस्कार िीता।
1)हेसनोंग क्कस्ट्ल बगध
2)मासटध न ओडे गाडध
3)एसलिंग हालैंड
4)अलेक्जेंडर सोरलोि
5)स्ट्ीफन इवसधन
उत्तर-3)एनििंग हािैंि
स्पष्टीिरण:
नॉवे के मैनचेस्ट्र ससटी के स्ट्र ाइकर एनििंग हािैंि और मैनेजर पेप गासडध योला ने सवधश्रेष्ठ पुरुष क्कखलाडी का UEFA
पुरस्कार जीता और वषध के कोच स्पेन की ऐटाना बोनमती ने सवधश्रेष्ठ मसहला क्कखलाडी का पुरस्कार जीता।
i.नॉवेसजयन हालैंड ने ससटी को तीन प्रमुख टर ॉसफयाों - चैंसपयोंस लीग, इों क्कग्लश प्रीसमयर लीग और FA कप जीतने में मदद
करने के बाद अपनी टीम के सािी केसवन डी ब्रुने और 2022 सवश्व कप चैंसपयन सलयोनेल मेसी को हराया।

Report Errors in the PDF - ebooks@affairscloud.com Copyright 2014-2023 @ AffairsCloud.com 226


25. 'सत्यमंगिम िन्यिीि अभ्यारण्य' निस राज्य में खस्थत है ?
1)केरल
2)तसमलनाडु
3)आों ध्र प्रदे श
4)तेलोंगाना
5)कनाधटक
उत्तर- 2)तनमििािु
स्पष्टीिरण:
तनमििािु िे बारे में:
मुख्यमंत्री- M.K. स्ट्ासलन
राज्यपाि- R.N.रसव
िन्यिीि अभ्यारण्य- कन्याकुमारी वन्यजीव अभ्यारण्य; सत्यमंगिम िन्यिीि अभ्यारण्य
टाइगर ररििा- श्रीसवक्कल्लपुिुर - मेघमलाई टाइगर ररजवध

26. अंतरााष्टरीय स्मारि एिं स्थि पररषद (ICOMOS) िा मुख्यािय िहााँ खस्थत है ?
1)सजनेवा, क्कस्वट् जरलैंड
2)पेररस, फ़्ाोंस
3)लोंदन, यूनाइटे ड सकोंगडम
4)रोम, इटली
5)सवयना, ऑक्कस्ट्रया
उत्तर-2)पेररस, फ्रांस
स्पष्टीिरण:
अंतरााष्टरीय स्मारक एवं स्थि पररषद (ICOMOS) के बारे में:
अध्यक्ष- टे रेसा पेटरीससयो
महासनचि- माररयो सैन्टाना
मुख्यािय- पेररस, फ्रांस
स्थापिा- 1965

27. 'मंगिििम पक्षी अभ्यारण्य' निस राज्य में खस्थत है ?


1)केरल
2)तसमलनाडु
3)कनाधटक
4)तेलोंगाना
5)आों ध्र प्रदे श
उत्तर- 1)िेरि
स्पष्टीिरण:
िेरि िे बारे में:
मुख्यमंत्री– सपनाराई सवजयन
राज्यपाि– आररफ मोहम्मद खान

Report Errors in the PDF - ebooks@affairscloud.com Copyright 2014-2023 @ AffairsCloud.com 227


िन्यिीि अभ्यारण्य- कोसट्टयूर वन्यजीव अभ्यारण्य; मंगिििम पक्षी अभ्यारण्य
हिाई अड्डा– कालीकट अोंतराधष्ट्रीय हवाई अड्डा; कन्नूर अोंतराध ष्ट्रीय हवाई अड्डा

28. 'िोरपोिरी िन्यिीि अभ्यारण्य' निस राज्य में खस्थत है ?


1)मसिपुर
2)पसिम बोंगाल
3)मध्य प्रदे श
4)केरल
5)झारखण्ड
उत्तर- 2)पनिम बंगाि
स्पष्टीिरण:
पनिम बंगाि िे बारे में :
मुख्यमंत्री–ममता बनजी
गििार– CV आनोंद बोस
राष्टरीय उद्याि– बक्सा राष्ट्रीय उद्यान; गोरुमारा राष्ट्रीय उद्यान
िन्यिीि अभ्यारण्य-िोरपोिरी िन्यिीि अभ्यारण्य; सेंचल वन्यजीव अभ्यारण्य

29. 'अिामुिी शोिा राष्टरीय उद्याि' निस राज्य में खस्थत है ?


1)तसमलनाडु
2)कनाधटक
3)केरल
4)आों ध्र प्रदे श
5)तेलोंगाना
उत्तर-3)िेरि
स्पष्टीिरण:
िेरि िे बारे में:
मुख्यमंत्री- सपनाराई सवजयन
राज्यपाि- आररफ मोहम्मद खान
राष्टरीय उद्याि- अिामुिी शोिा राष्टरीय उद्याि; पेररयार राष्ट्रीय उद्यान
िन्यिीि अभ्यारण्य- सचन्नार वन्यजीव अभ्यारण्य; सचम्मोनी वन्यजीव अभ्यारण्य

30. राष्टरीय शैनक्षि अिुसंधाि और प्रनशक्षण पररषद (NCERT) िे ितामाि (नसतंबर’23 ति) निदे शि िौि हैं ?
1)सुभाष चोंदर
2)सदनेश प्रसाद सकलानी
3)दीपक पालीवाल
4)श्रीर्र श्रीवास्तव
5)जयदीप मोंडल
उत्तर-2)नदिेश प्रसाद सििािी

Report Errors in the PDF - ebooks@affairscloud.com Copyright 2014-2023 @ AffairsCloud.com 228


स्पष्टीिरण:
राष्टरीय शैनक्षि अिुसंधाि और प्रनशक्षण पररषद (NCERT) िे बारे में:
NCERT की थिापना स्कूली सशक्षा से सोंबोंसर्त मामलोों पर सरकार को सहायता और सलाह दे ने के उद्दे श्य से की गई
िी।
निदे शि-नदिेश प्रसाद सििािी
मुख्यािय – नई सदल्ली, सदल्ली
स्थापिा – 1961

31. पेटीएम िा ितामाि (नसतंबर’23 ति) मुख्य िायािारी अनधिारी (CEO) िौि है ?
1)सवजय शेखर शमाध
2)सुररों दर चावला
3)अनुब्रत सबस्वास
4)ऋसष गुप्ता
5)सवनोद ईश्वरन
उत्तर- 1)नििय शेिर शमाा
स्पष्टीिरण:
वन97 कम्युसनकेशोंस सलसमटे ड (OCL) में एों टसफन की सहस्सेदारी घटकर 9.90 प्रसतशत हो गई है , जो पेटीएम का
मासलक है , इसके सोंथिापक और मुख्य कायधकारी असर्कारी (CEO) नििय शेिर शमाा एकमात्र महत्वपूिध लाभकारी
मासलक (SBO) बन गए हैं।
i.एों टसफन, सजसने हाल ही में OCL में अपनी सहस्सेदारी 23.79 प्रसतशत से घटाकर 9.90 प्रसतशत कर दी है , अब कोंपनी
का SBO नहीों है , पेटीएम ने स्ट्ॉक एक्सचेंजोों को एक फाइसलोंग में कहा।
ii.एों टसफन की पेटीएम में कुल शेयरर्ाररता सपछले कुछ हफ्तोों में 23.79 प्रसतशत से काफी कम हो गई है , जब उसने
अपनी 10.3 प्रसतशत सहस्सेदारी सवजय शेखर शमाध की 100 प्रसतशत स्वासमत्व वाली सवदे शी इकाई रे सजसलएों ट एसेट
मैनेजमेंट B.V को बेच दी िी। इसके बाद िॉक डील में अन्य 3.6 प्रसतशत की सबक्री हुई।

32. भारतीय अंतररक्ष और अिुसंधाि संगठि (ISRO) िे िैज्ञानिि __________, राष्टरीय अंतररक्ष एिेंसी िे
रॉिेट िाउं टिाउि िॉन्च िे पीछे िी आिाि, िा नसतंबर 2023 में निधि हो गया।
1)मीनल रोसहत
2)ररतु कररर्ल
3)N वलारमिी
4)मौसमता दत्ता
5)अनुरार्ा T.K
उत्तर-3)N ििारमथी
स्पष्टीिरण:
भारतीय अोंतररक्ष और अनुसोंर्ान सोंगठन (ISRO) के वैज्ञासनक N ििारमथी, जो राष्ट्रीय अोंतररक्ष एजेंसी के रॉकेट
काउों टडाउन लॉन्च के पीछे की आवाज िे , का सनर्न हो गया है।
• 14 जुलाई को लॉन्च सकया गया बेहद सफल चोंद्रयान-3 उसके सलए अोंसतम उलटी सगनती सासबत हुआ।
• तसमलनाडु के अररयालुर की मूल सनवासी वलारमिी का जन्म 31 जुलाई 1959 को हुआ िा, वलारमिी 1984 में
ISRO में शासमल हुईों और कई असभयानोों में भाग सलया। वह भारत के पहले स्वदे शी रूप से सवकससत रडार

Report Errors in the PDF - ebooks@affairscloud.com Copyright 2014-2023 @ AffairsCloud.com 229


इमेसजोंग सैटेलाइट (RIS) और दे श के दू सरे ऐसे उपग्रह RISAT-1 की पररयोजना सनदे शक िीों। RISAT-1 को
अप्रैल 2012 में सफलतापूवधक लॉन्च सकया गया िा।
• इसके अलावा, 2015 में स्वतोंत्रता सदवस पर, वलारमिी पूवध राष्ट्रपसत डॉ. APJ अब्दु ल कलाम, सजन्ें 'भारत के
समसाइल मैन' के रूप में भी जाना जाता है , की स्मृसत में तसमलनाडु सरकार िारा थिासपत प्रसतसष्ठत अब्दु ल
कलाम पुरस्कार के पहले प्राप्तकताध बने।

33. इटािा सफारी पािा निस राज्य में खस्थत है ?


1)कनाधटक
2)महाराष्ट्र
3)मध्य प्रदे श
4)उत्तर प्रदे श
5)झारखण्ड
उत्तर-4)उत्तर प्रदे श
स्पष्टीिरण:
उत्तर प्रदे श के इटावा सफारी पाकध में शेरनी रूपा के पैदा हुए दो शावकोों में से एक को कुछ घोंटोों बाद नरभक्षि के
एक दु लधभ मामले में बडी सबल्ली ने खा सलया। सफारी असर्काररयोों के मुतासबक, शावक मृत पैदा हुआ िा। दू सरा जो
जीसवत पैदा हुआ िा उसे शेरनी िारा ध्यान न दे ने के कारि तुरोंत अलग कर सदया गया और पाकध की नवजात दे खभाल
इकाई में रखा गया।
• इटावा सफारी ने 2023 में छह शावकोों को खो सदया, सजनमें शेरनी सोना के पाों च शावक भी शासमल िे। सोना ने
6 जुलाई से 10 जुलाई के बीच सफारी पाकध में पाोंच शावकोों को जन्म सदया िा। इनमें से चार शावकोों की 13
जुलाई तक मौत हो चुकी िी।

34. उस बैंि िा िाम बताइए निसिे हाि ही में (नसतंबर’23 में) निनिटि रुपए िे साथ UPI
इं टरऑपरे नबनिटी िे िायाान्वयि िी घोषणा िी है।
1)RBL बैंक
2)ICICI बैंक
3)भारतीय स्ट्े ट बैंक
4)IDBI बैंक
5)HDFC बैंक
उत्तर-3)भारतीय स्ट्े ट बैंि
स्पष्टीिरण:
भारतीय स्ट्े ट बैंि ने सडसजटल रुपए के साि UPI इं टरऑपरे नबनिटी लागू करने की घोषिा की है। इस कदम के
साि, SBI का लक्ष् अपने ग्राहकोों को सुसवर्ा और पहुोंच प्रदान करना है। 'eRupee by SBI' एक्कप्लकेशन के माध्यम से
सुलभ यह सुसवर्ा उपयोगकताधओों को लेनदे न के सलए सकसी भी व्यापारी के UPI QR कोड को आसानी से स्कैन करने
में सक्षम बनाएगी।
• SBI RBI की खुदरा सडसजटल eRupee पररयोजना में भाग लेने वाले पहले कुछ बैंकोों में से एक िा।

Report Errors in the PDF - ebooks@affairscloud.com Copyright 2014-2023 @ AffairsCloud.com 230


35. 'अचाििमार िन्यिीि अभ्यारण्य' निस राज्य में खस्थत है ?
1)मध्य प्रदे श
2)पसिम बोंगाल
3)झारखण्ड
4)महाराष्ट्र
5)छत्तीसगढ
उत्तर-5)छत्तीसगढ़
स्पष्टीिरण:
छत्तीसगढ़ िे बारे में:
राज्यपाि – सबस्वा भूषि हररचोंदन
मुख्यमंत्री (CM)-भूपेश बघेल
राष्टरीय उद्याि - इों द्रावती (कुटरू) राष्ट्रीय उद्यान, काों गेर घाटी राष्ट्रीय उद्यान
िन्यिीि अभ्यारण्य - अचानकमार वन्यजीव अभ्यारण्य, बादलखोल वन्यजीव अभ्यारण्य

36. िाद्य एिं िृनष संगठि (FAO) िी स्थापिा िब हुई थी?


1)1981
2)1950
3)1978
4)1945
5)1962
उत्तर- 4)1945
स्पष्टीिरण:
िाद्य एिं िृनष संगठि (FAO) िे बारे में:
महानिदे शि– QU डोोंगयु
मुख्यािय– रोम, इटली
स्थापिा- 1945

37. 'रामगढ़ निषधारी टाइगर ररििा' निस राज्य में खस्थत है ?


1)उत्तर प्रदे श
2)महाराष्ट्र
3)मध्य प्रदे श
4)गुजरात
5)राजथिान
उत्तर-5)रािस्थाि
स्पष्टीिरण:
रािस्थाि िे बारे में:
मुख्यमंत्री (CM) – अशोक गहलोत
राज्यपाि – कलराज समश्र
िन्यिीि अभ्यारण्य - गजनेर वन्यजीव अभ्यारण्य; कैला दे वी अभ्यारण्य

Report Errors in the PDF - ebooks@affairscloud.com Copyright 2014-2023 @ AffairsCloud.com 231


टाइगर ररििा - रििोंभौर टाइगर ररजवध; रामगढ़ निषधारी टाइगर ररििा

38. अंतरााष्टरीय ऊिाा एिेंसी (IEA) िा मुख्यािय िहााँ है ?


1)सजनेवा, क्कस्वट् जरलैंड
2)रोम, इटली
3)पेररस, फ़्ाोंस
4)लोंदन, यूनाइटे ड सकोंगडम
5)सवयना, ऑक्कस्ट्रया
उत्तर-3)पेररस, फ्रांस
स्पष्टीिरण:
अंतरााष्टरीय ऊिाा एिेंसी (IEA) िे बारे में:
िायािारी निदे शि - डॉ. फ़सतह सबरोल
मुख्यािय – पेररस, फ़्ांस

39. 'िेििादे ि राष्टरीय उद्याि' निस राज्य में खस्थत है ?


1)कनाधटक
2)पसिम बोंगाल
3)राजथिान
4)महाराष्ट्र
5)उत्तर प्रदे श
उत्तर-3)रािस्थाि
स्पष्टीिरण:
रािस्थाि राज्य वन सवभाग ने िेििादे ि राष्टरीय उद्याि के अोंदर एक सचसडयाघर बनाने का प्रस्ताव सदया है , जो एक
सवश्व र्रोहर थिल है , सजसे लोकसप्रय रूप से भरतपुर पक्षी अभ्यारण्य के रूप में जाना जाता है , सजसमें गैंडे, जल भैंस,
मगरमच्छ, डॉक्कल्फ़न और सवदे शी प्रजासतयोों ससहत आद्रध भूसम प्रजासतयोों की एक श्रृोंखला प्रदसशधत की जाएगी।
• 1982 में, केवलादे व को एक राष्ट्रीय उद्यान घोसषत सकया गया िा और सफर बाद में 1985 में UNESCO िारा
सवश्व र्रोहर थिल के रूप में सूचीबद्ध सकया गया िा।

40. 'चंद्रताि िन्यिीि अभ्यारण्य' निस राज्य में खस्थत है ?


1)सहमाचल प्रदे श
2)केरल
3)महाराष्ट्र
4)तसमलनाडु
5)अरुिाचल प्रदे श
उत्तर- 1)नहमाचि प्रदे श
स्पष्टीिरण:
नहमाचि प्रदे श िे बारे में:
मुख्यमंत्री– सुखसवोंदर ससोंह सुक्खू
राज्यपाि– सशव प्रताप शुक्ला

Report Errors in the PDF - ebooks@affairscloud.com Copyright 2014-2023 @ AffairsCloud.com 232


िन्यिीि अभ्यारण्य- चंद्रताि िन्यिीि अभ्यारण्य; चूडर्ार वन्यजीव अभ्यारण्य

41. 'परखम्बिुिम टाइगर ररििा' निस राज्य में खस्थत है ?


1)तेलोंगाना
2)कनाधटक
3)तसमलनाडु
4)आों ध्र प्रदे श
5)केरल
उत्तर-5)िेरि
स्पष्टीिरण:
िेरि िे बारे में:
मुख्यमंत्री- सपनाराई सवजयन
राज्यपाि- आररफ मोहम्मद खान
िन्य िीिि अभ्यारण्य- अरलम वन्यजीव अभ्यारण्य; मालाबार वन्यजीव अभ्यारण्य
टाइगर ररििा- पेररयार टाइगर ररजवध; परखम्बिुिम टाइगर ररििा

42. HDFC बैंि िे ितामाि (लसतंबर’23 ति) प्रबंध निदे शि (MD) और मुख्य िायािारी अनधिारी (CEO)
िौि हैं ?
1)सगरीश चोंद्र चतुवेदी
2)सोंदीप बख्शी
3)अतनु चक्रवती
4)शसशर्र जगदीशन
5)सुमोंत कठपासलया
उत्तर-4)शनशधर िगदीशि
स्पष्टीिरण:
HDFC बैंि निनमटे ि िे बारे में :
MD & CEO - शनशधर िगदीशि
स्थापिा- 1994
मुख्यािय – मुोंबई, महाराष्ट्र
टै गिाइि - वी अोंडरस्ट्ैं ड योर वडध

43. 'दे िमािी हाथी ररििा' निस राज्य में खस्थत है ?


1)समजोरम
2)ओसडशा
3)पसिम बोंगाल
4)असम
5)अरुिाचल प्रदे श
उत्तर-5)अरुणाचि प्रदे श

Report Errors in the PDF - ebooks@affairscloud.com Copyright 2014-2023 @ AffairsCloud.com 233


स्पष्टीिरण:
दे वमाली हािी ररजवध अरुणाचि प्रदे श में क्कथित है। इसकी थिापना 2002 में हुई िी। यह पहासडयोों, चाय बागानोों,
जोंगलोों और बुरही-सदसहोंग नदी से सघरा हुआ है ।

44. निश्व स्वास्थ्य संगठि (WHO) िे महानिदे शि िा मुख्यािय िहााँ खस्थत है ?


1)सजनेवा, क्कस्वट् जरलैंड
2)रोम, इटली
3)पेररस, फ़्ाोंस
4)लोंदन, यूनाइटे ड सकोंगडम
5)सवयना, ऑक्कस्ट्रया
उत्तर-1)नििेिा, खस्वट् िरिैंि
स्पष्टीिरण:
निश्व स्वास्थ्य संगठि (WHO) िे बारे में:
महानिदे शि– डॉ. टे डरोस एडनोम घेब्रेयेसस
मुख्यािय– सजनेवा, क्कस्वट् जरलैंड
स्थानपत– 7 अप्रैल 1948

45. िमाचारी भनिष्य निनध संगठि (EPFO) िा ितामाि (नसतंबर’23 ति) मुख्य िायािारी अनधिारी (CEO)
िौि है ?
1)चन्रामौली चक्रवती
2)शोसभत श्रीवास्तव
3)नीलम शमी राव
4)राकेश डबास
5)मीनू मोंडोला
उत्तर-3)िीिम शमी राि
स्पष्टीिरण:
िमाचारी भनिष्य निनध संगठि (EPFO ) िे बारे में:
िेंद्रीय भनिष्य निनध आयुक्त (CPFO)/मुख्य िायािारी अनधिारी (CEO)– सुश्री नीलम शमी राव
मुख्यािय – नई सदल्ली
स्थापिा– 1952

46. 'िंनदिी िन्यिीि अभ्यारण्य' __________ में खस्थत है।


1)लद्दाख
2)जम्मू और कश्मीर
3)उत्तर प्रदे श
4)सहमाचल प्रदे श
5)उत्तराखोंड
उत्तर- 2)िम्मू-िश्मीर

Report Errors in the PDF - ebooks@affairscloud.com Copyright 2014-2023 @ AffairsCloud.com 234


स्पष्टीिरण:
िम्मू और िश्मीर (J&K) िे बारे में:
उपराज्यपाि-मनोज ससन्ा
िन्यिीि अभ्यारण्य - ओवरा-अरु वन्यजीव अभ्यारण्य, िंनदिी िन्यिीि अभ्यारण्य
हिाई अड्डा - श्रीनगर अोंतराध ष्ट्रीय हवाई अड्डा (शेख उल-आलम अोंतराध ष्ट्रीय हवाई अड्डा), जम्मू हवाई अड्डा (सतवारी
हवाई अड्डा)

47. निश्व आनथाि मंच (WEF) िे ितामाि (नसतंबर’23 ति) िायािारी अध्यक्ष िौि हैं ?
1) क्लॉस श्वाब
2)तुलसी गबाडध
3)बोगे ब्रेंडे
4)इक्कजज़क याहव
5)हेनरी सकससोंजर
उत्तर- 1) िॉस श्वाब
स्पष्टीिरण:
निश्व आनथाि मंच (WEF) िे बारे में:
संस्थापि और िायािारी अध्यक्ष - प्रोफेसर िॉस श्वाब
प्रबंध निदे शि (MD)- सासदया जासहदी
मुख्यािय - कोलोनी, सजनेवा कैंटन, क्कस्वट् जरलैंड

48. 'बेतिा राष्टरीय उद्याि' निस राज्य में खस्थत है ?


1)ओसडशा
2)मध्य प्रदे श
3)पसिम बोंगाल
4)झारखण्ड
5)उत्तर प्रदे श
उत्तर-4)झारिंि
स्पष्टीिरण:
झारिंि िे बारे में:
मुख्यमंत्री (CM)-हेमोंत सोरे न
राज्यपाि- रमेश बैस
राष्टरीय उद्याि- बेतिा राष्टरीय उद्याि
िन्य िीिि अभ्यारण्य- दलमा वन्यजीव अभ्यारण्य, लावालोोंग वन्यजीव अभ्यारण्य

49. िन्य िीिों और ििस्पनतयों िी िुप्तप्राय प्रिानतयों में अंतरााष्टरीय व्यापार पर िन्वेंशि (CITES) िा
मुख्यािय िहााँ है ?
1)रोम, इटली
2)सवयना, ऑक्कस्ट्रया
3)पेररस, फ़्ाोंस

Report Errors in the PDF - ebooks@affairscloud.com Copyright 2014-2023 @ AffairsCloud.com 235


4)सजनेवा, क्कस्वट् जरलैंड
5)जकाताध , इों डोनेसशया
उत्तर-4)नििेिा, खस्वट् िरिैंि
स्पष्टीिरण:
CITES वन्य जीवोों और वनस्पसतयोों की लुप्तप्राय प्रजासतयोों में अोंतराध ष्ट्रीय व्यापार पर एक सम्मे लन है और इसका
मुख्यालय नििेिा, खस्वट् िरिैंि में है।

50. 'अंसुपा झीि' निस राज्य/UT में खस्थत है ?


1)ओसडशा
2)तसमलनाडु
3)जम्मू और कश्मीर
4)केरल
5)आों ध्र प्रदे श
उत्तर- 1)ओनिशा
स्पष्टीिरण:
अंसुपा झीि ओसडशा के कटक सजले के बाोंकी उपखोंड में क्कथित है , यह महानदी नदी िारा सनसमधत एक ऑक्सबो झील
है और 231 हेरेयर क्षेत्र में फैली हुई है। यह मैक्रोफाइट् स की 244 प्रजासतयोों के अलावा पसक्षयोों की कम से कम 194
प्रजासतयोों, मछसलयोों की 61 प्रजासतयोों और स्तनर्ाररयोों की 26 प्रजासतयोों का घर है।

51. चौधरी चरण नसंह अंतरााष्टरीय हिाई अड्डा निस राज्य में खस्थत है ?
1)महाराष्ट्र
2)उत्तर प्रदे श
3)मध्य प्रदे श
4)गुजरात
5)झारखण्ड
उत्तर- 2)उत्तर प्रदे श
स्पष्टीिरण:
उत्तर प्रदे श (UP) िे बारे में:
मुख्यमंत्री (CM)- योगी आसदत्यनाि
राज्यपाि – आनोंदीबेन पटे ल
प्राणी उद्याि - शहीद अशफाक उल्ला खान प्रासि उद्यान; सारनाि सडयर पाकध
हिाई अड्डे - चौधरी चरण नसंह अंतरााष्टरीय हिाई अड्डा; लाल बहादु र शास्त्री हवाई अड्डा

52. िेंद्रीय बंदरगाह, िहािरािी और ििमागा मंत्री सबाािंद सोिोिाि िे नसतंबर 2023 में निस बंदरगाह में
333 िरोड रुपये िी पररयोििाओं िा उद् घाटि निया?
1)जवाहरलाल नेहरू बोंदरगाह
2)पारादीप बोंदरगाह
3)दीनदयाल बोंदरगाह
4)सवशाखापत्तनम बोंदरगाह

Report Errors in the PDF - ebooks@affairscloud.com Copyright 2014-2023 @ AffairsCloud.com 236


5)V.O. सचदों बरनार बोंदरगाह
उत्तर-4)निशािापत्तिम बंदरगाह
स्पष्टीिरण:
केंद्रीय बोंदरगाह, जहाजरानी और जलमागध मोंत्री सबाािंद सोिोिाि ने सवशाखापत्तनम बोंदरगाह पर नििाग
इं टरिेशिि क्रूज़ टनमािि (VICT) और 333 करोड रुपये की कुछ और पररयोजनाओों का उद् घाटन सकया।
i.क्रूज़ टसमधनल के अलावा, उन्ोोंने एक कवडध स्ट्ोरे ज शेड, सवश्व स्तरीय टर क पासकिंग टसमधनल और एक सोंशोसर्त बिध
(OR-1 बिध) का उद् घाटन सकया।
ii.ढका हुआ भोंडारि शेड र्ूल को दबाने के सलए र्ुोंर् की व्यवथिा के साि 84,000 टन िोक और बैग में रखे गए कागो
को सोंभालने के सलए सुसक्कित है।

53. उस बैंि िा िाम बताइए निसिे हाि ही में (नसतंबर’23 में) अपिे सेंटरि बैंि निनिटि िरें सी (CBDC)
ऐप िे साथ यूनिफाइि पेमेंट्स इं टरफेस (UPI) QR िोि िे एिीिरण िी घोषणा िी है।
1)RBL बैंक
2)कनाधटक बैंक
3)IDFC फस्ट्ध बैंक
4)IDBI बैंक
5)इों डसइों ड बैंक
उत्तर-3)IDFC फस्ट्ा बैंि
स्पष्टीिरण:
IDFC फस्ट्ा बैंि ने सडसजटल रुपये को अपनाने और इसके उपयोग को बढावा दे ने के उद्दे श्य से अपने सेंटरल बैंक
सडसजटल करें सी (CBDC) ऐप के साि यूसनफाइड पेमेंट इों टरफेस (UPI) QR िोि के एकीकरि की घोषिा की।
i.उन बैंकोों में से एक होने के नाते जो खुदरा उपयोगकताधओों के सलए RBI की CBDC पहल का सहस्सा हैं , यह नई सुसवर्ा
व्यापाररयोों के सलए भुगतान स्वीकृसत को सरल बनाएगी, सजससे वे सडसजटल रुपये से सकए गए भुगतान को सहजता से
स्वीकार करने में सक्षम होोंगे।
ii. UPI इों टरऑपरे सबसलटी सुसवर्ा दे श भर में CBDC को अपनाने में नाटकीय रूप से वृक्कद्ध करे गी।

54. निस बैंि िे हाि ही में (नसतंबर’23 में) व्हाट् सएप िेंनिं ग प्लेटफॉमा िी शुरुआत िी घोषणा िी है निसिे
माध्यम से ग्राहि पूिा-अिुमोनदत व्यखक्तगत ऋण िे निए आिेदि िर सिते हैं ?
1)RBL बैंक
2)IDBI बैंक
3)इों डसइों ड बैंक
4)फेडरल बैंक
5)कनाधटक बैंक
उत्तर-4)फेिरि बैंि
स्पष्टीिरण:
फेिरि बैंि सलसमटे ड ने व्हाट् सएप िेंनिं ग प्लेटफॉमा की शुरुआत की घोषिा की है सजसके माध्यम से ग्राहक पूवध-
अनुमोसदत व्यक्कक्तगत ऋि के सलए आवेदन कर सकते हैं।
i.मोंच का उद् घाटन फेडरल बैंक के अध्यक्ष A.P. होता ने MD और CEO श्याम श्रीसनवासन, कायधकारी सनदे शक
शासलनी वाररयर, उपाध्यक्ष और प्रमुख - सडसजटल सुमोि C और बैंक स्ट्ाफ की उपक्कथिसत में सकया।

Report Errors in the PDF - ebooks@affairscloud.com Copyright 2014-2023 @ AffairsCloud.com 237


55. भारतीय िायु सेिा (IAF) िी पनिमी िायु िमाि िे िानषाि प्रनशक्षण अभ्यास िा िाम बताइए िो हाि ही
में (नसतंबर’23 में) चीि और पानिस्ताि िे साथ सीमाओं पर शुरू हुआ है।
1)खोंजर
2)सोंप्रीसत
3)मैत्री
4)ससन्धुराज
5)सत्रशूल
उत्तर-5)नत्रशूि
स्पष्टीिरण:
भारतीय वायु सेना (IAF) की पसिमी वायु कमान (WAC) का वासषधक प्रसशक्षि अभ्यास, नत्रशूि, 4 ससतोंबर 2023 को
शुरू हुआ। इस अभ्यास में पासकस्तान के साि सनयोंत्रि रे खा से लेकर चीन के साि वास्तसवक सनयोंत्रि रे खा (LAC) तक
फैले सभी हवाई सोंसार्नोों और बल गुिकोों को ससक्रय सकया जाएगा।
• यह अभ्यास 4-14 ससतोंबर तक सनर्ाधररत है और यह कमाोंड की पररचालन तैयाररयोों को मान्य करे गा और
लडाकू जेट, पररवहन सवमान, हेलीकॉप्टर और अन्य बल मल्टीप्लायरोों से सभी फ्रोंटलाइन सोंपसत्तयोों को उच्च गसत
में सनयोसजत करे गा।
• G-20 सशखर सम्मेलन के साि कुछ सदनोों के सलए अभ्यास को रोक सदया जाएगा, जब हाई-प्रोफाइल घटना
और खतरे की आशोंका के साि सशस्त्र बल हाई अलटध पर होोंगे।

56. नसतंबर 2023 में, संयुक्त राज्य अमेररिा में ____________ राज्य िे औपचाररि रूप से अरू बर िो "नहं दू
निरासत माह" िे रूप में िानमत निया।
1)टे क्सास
2)िोररडा
3)अलास्का
4)जॉसजधया
5)वजीसनया
उत्तर-4)िॉनिाया
स्पष्टीिरण:
सोंयुक्त राज्य अमेररका में िॉनिाया राज्य ने राज्य में रहने वाले जीवोंत सहों दू-अमेररकी समुदाय िारा सकए गए मूल्वान
योगदान को पहचानने और सराहना करने के सलए अरू बर को औपचाररक रूप से "नहं दू निरासत माह" के रूप में
नासमत सकया है।
i.यह घोषिा अपनी समृद्ध सोंस्कृसत और भारत से उत्पन्न सवसवर् आध्याक्कत्मक परों पराओों पर जोर दे कर सहोंदू सवरासत का
जश्न मनाने के इरादे पर प्रकाश डालती है।
ii.23 अगस्त को जॉसजधया के गवनधर की उद् घोषिा में कहा गया है , जीवोंत सहोंदू अमेररकी समुदाय ने अपने नागररकोों के
जीवन को समृद्ध करके जॉसजधया राज्य की जीवन शक्कक्त में जबरदस्त योगदान सदया है।

57. अंतरााष्टरीय आपरानधि पुनिस संगठि (INTERPOL) िा मुख्यािय िहााँ खस्थत है ?


1)सवयना, ऑक्कस्ट्रया
2)ब्रुसेल्स, बेक्कियम
3)ल्ोन, फ्राोंस

Report Errors in the PDF - ebooks@affairscloud.com Copyright 2014-2023 @ AffairsCloud.com 238


4)सजनेवा, क्कस्वट् जरलैंड
5)लोंदन, यूनाइटे ड सकोंगडम
उत्तर-3)ल्योि, फ्रांस
स्पष्टीिरण:
अंतरााष्टरीय आपरानधि पुनिस संगठि (INTERPOL) िे बारे में:
महासनचि– जुगधन स्ट्ॉक
अध्यक्ष– सोंयुक्त अरब अमीरात (UAE) के अहमद नासेर अल-रायसी
मुख्यािय– ल्योि, फ्रांस
स्थापिा – 1923

58. "रायमोिा राष्टरीय उद्याि" निस राज्य में खस्थत है ?


1)ओसडशा
2)महाराष्ट्र
3)असम
4)गुजरात
5)मध्य प्रदे श
उत्तर-3)असम
स्पष्टीिरण:
असम िे बारे में:
राज्यपाि – गुलाब चोंद कटाररया
मुख्यमंत्री – सहमोंत सबस्वा सरमा
राष्टरीय उद्याि - ओरों ग राष्ट्रीय उद्यान, सदसहोंग पटकाई राष्ट्रीय उद्यान, रायमोिा राष्टरीय उद्याि
िन्य िीिि अभ्यारण्य - बुराचापोरी वन्यजीव अभ्यारण्य, पसिडीसहोंग वन्यजीव अभ्यारण्य

59. इं नियि ररन्यूएबि एििी िे ििपमेंट एिेंसी निनमटे ि (IREDA) िे ितामाि (नसतंबर'23 ति) अध्यक्ष और
प्रबंध निदे शि (CMD) िौि हैं ?
1)रमेश चोंद्र शमाध
2)रोसहिी रावत
3)पदम लाल नेगी
4)प्रदीप कुमार दास
5)अजय यादव
उत्तर-4)प्रदीप िुमार दास
स्पष्टीिरण:
इं नियि ररन्यूएबि एििी िे ििपमेंट एिेंसी निनमटे ि (IREDA) िे बारे में:
IREDA समसनस्ट्र ी ऑफ़ न्यू एों ड ररन्यूएबल एनजी (MNRE) के तहत एक समनी रत्न (श्रेिी- I) केंद्रीय सावधजसनक क्षेत्र
उद्यम (CPSE) है।
अध्यक्ष और प्रबंध निदे शि– प्रदीप िुमार दास
स्थापिा- 1987
मुख्यािय– नई सदल्ली, सदल्ली

Report Errors in the PDF - ebooks@affairscloud.com Copyright 2014-2023 @ AffairsCloud.com 239


60. इं टरगििामेंटि साइं स-पॉनिसी प्लेटफामा ऑि बायोिायिनसाटी एं ि इिोनसस्ट्म सनिासेि (IPBES) िा
मुख्यािय िहााँ खस्थत है ?
1)मॉक्कन्टरयल, कनाडा
2)बॉन, जमधनी
3)लोंदन, यूनाइटे ड सकोंगडम
4)ब्रुसेल्स, बेक्कियम
5)जकाताध , इों डोनेसशया
उत्तर-2)बॉि, िमािी
स्पष्टीिरण:
इं टरगििामेंटि साइं स-पॉनिसी प्लेटफामा ऑि बायोिायिनसाटी एं ि इिोनसस्ट्म सनिासेि (IPBES) िे बारे में:
िायािारी सनचि– डॉ. ऐनी लैरीगौडे री
मुख्यािय– बॉि, िमािी
स्थानपत – 21 अप्रैल 2012

61. निब्बर िन्यिीि अभ्यारण्य निस राज्य में खस्थत है ?


1)सहमाचल प्रदे श
2)महाराष्ट्र
3)मध्य प्रदे श
4)पसिम बोंगाल
5)ओसडशा
उत्तर- 1)सहमाचल प्रदे श
स्पष्ट्ीकरि:
नहमाचि प्रदे श (HP) िे बारे में:
मुख्यमंत्री– सुखसवोंदर ससोंह सुक्खू
राज्यपाि– सशव प्रताप शुक्ला
िन्यिीि अभ्यारण्य- खोखन वन्यजीव अभ्यारण्य, निब्बर िन्यिीि अभ्यारण्य
त्यौहार- नलवाडी मेला (माचध), सदयोटससद्ध मेला (माचध और अप्रैल)

62. हाि ही में (नसतंबर'23 में) बैटिग्राउं ि मोबाइि इं निया (BGMI) िे िांि एं बेसिर िे रूप में निसे नियुक्त
निया गया है ?
1)सवराट कोहली
2)दीसपका पादु कोन
3)रिवीर ससोंह
4)ससचन तेंदुलकर
5)ररसतक रोशन
उत्तर-3)रणिीर नसंह
स्पष्टीिरण:
BGMI के पीछे की कोंपनी क्राफ्टि इं निया ने भारत में सहयोग की घोषिा की है। बॉलीवुड असभनेता रिवीर ससोंह को
बैटलग्राउों ड मोबाइल इों सडया (BGMI) का आसर्काररक िांि एं बेसिर नासमत सकया गया है।

Report Errors in the PDF - ebooks@affairscloud.com Copyright 2014-2023 @ AffairsCloud.com 240


i.इस सहयोग के साि, क्राफ्टन प्ले प्योर असभयान शुरू करे गा सजसका उद्दे श्य सुपरस्ट्ार रिवीर ससोंह के लेंस के माध्यम
से क्कखलासडयोों में शुद्धता का जश्न मनाना है।

63. उस िंपिी िा िाम बताइए निसे हाि ही में (नसतंबर'23 में) 19िें एनशयाई िेि 2022 िे निए भारतीय दि
िे आनधिाररि प्रायोिि िे रूप में िानमत निया गया है।
1)पेटीएम
2)OPPO
3)MRF
4)अमूल
5)PUMA
उत्तर-4)अमूि
स्पष्टीिरण:
अमूि को 23 ससतोंबर से 8 अरू बर, 2023 तक हाों गझू, चीन में आयोसजत होने वाले 19वें एसशयाई खेल 2022 के सलए
भारतीय दि िे आनधिाररि प्रायोिि के रूप में नासमत सकया गया है।
i.अमूल ने लोंदन 2012 ओलोंसपक के बाद से ओलोंसपक, राष्ट्रमोंडल खेलोों और एसशयाई खेलोों के सलए सभी भारतीय दल के
सलए भारतीय ओलोंसपक सोंघ के माध्यम से भारतीय क्कखलासडयोों के साि साझेदारी की है।
ii.इस सहयोग के सहस्से के रूप में , अमूल क्कखलाडी के प्रयासोों का जश्न मनाने के सलए अपने सोंचार में एकीकृत लोगो का
उपयोग करे गा।

64. उस नक्रिेटर िा िाम बताइए िो हाि ही में (नसतंबर'23 में) T20 इं टरिेशिि (T20I) नक्रिेट में 100 नििेट
िेिे िािा एसोनसएट िेशि िा पहिा गेंदबाि बि गया है।
1)ऐसलस कैप्सी (ऑस्ट्र े सलया)
2)मासहका गौर (इों ग्लैंड)
3)नट्टाया बूचािम (िाईलैंड)
4)तारा नॉररस (आयरलैंड)
5)सोफी एक्लेस्ट्ोन (नीदरलैंड)
उत्तर-3)िट्टाया बूचाथम (थाईिैंि)
स्पष्टीिरण:
िाईलैंड के क्कस्पनर िट्टाया बूचाथम ने T20 इों टरनेशनल (T20I) नक्रिेट में 100 नििेट लेने वाले एसोससएट नेशन के
पहले गेंदबाज बनकर एक ऐसतहाससक उपलक्कब्ध हाससल की।
i.बूचािम ने मलेसशया के बाोंगी में कुवैत के क्कखलाफ ICC मसहला T20 सवश्व कप एसशया क्षेत्र क्वालीफायर मैच के दौरान
यह उपलक्कब्ध हाससल की। उन्ोोंने 3/3 के प्रभावशाली आों कडे के साि खेल में तीन सवकेट हाससल सकए, सजसने उन्ें T20I
में 100 सवकेट के ऐसतहाससक लक्ष् तक पहुोंचा सदया।
ii.यह उल्लेखनीय उपलक्कब्ध सलोंग की परवाह सकए सबना सकसी एसोससएट दे श से T201 सक्रकेट में इस महत्वपूिध उपलक्कब्ध
तक पहुोंचने वाली पहली क्कखलाडी के रूप में उनकी क्कथिसत को मजबूत करती है ।

65. भारतीय नक्रिेट िंटर ोि बोिा (BCCI) िे ितामाि (नसतंबर'23 ति) सनचि िौि हैं ?
1)सौरव गाोंगुली
2)शशाोंक मनोहर

Report Errors in the PDF - ebooks@affairscloud.com Copyright 2014-2023 @ AffairsCloud.com 241


3)C.K खन्ना
4)रोजर सबन्नी
5)जय शाह
उत्तर-5)िय शाह
स्पष्टीिरण:
भारतीय नक्रिेट िंटर ोि बोिा (BCCI) के ससचव िय शाह ने महान असभनेता असमताभ बच्चन को ICC पुरुष सवश्व कप
का गोडन सटकट प्रदान सकया।
i.भारत ने आज सवश्व कप टीम की घोषिा की और सवकेटकीपर बल्लेबाज KL राहुल, सजनकी सफटनेस सवालोों के घेरे में
है, को भारत की 15 सदस्यीय सवश्व कप टीम में नासमत सकया गया है क्ोोंसक चयनकताध ओों ने टू नाधमेंट के सलए सात
बल्लेबाजोों और चार ऑलराउों डरोों का चयन सकया है।

66. इन्वेस्ट्र एिुिेशि एं ि प्रोटे क्शि फंि अथॉररटी (IEPFA) िे 5 नसतंबर 2023 िो अपिा __________ स्थापिा
नदिस मिाया।
1)3वाों
2)7वाों
3)5वाों
4)10वाों
5)8वाों
उत्तर- 2)7िां
स्पष्टीिरण:
अपने 7िें थिापना सदवस (5 ससतोंबर 2023)पर, इन्वेस्ट्र एजुकेशन एों ड प्रोटे क्शन फोंड अिॉररटी (IEPFA) ने इों स्ट्ीट्यूट
ऑफ कोंपनी सेक्रेटरीज इन इों सडया (ICSI) और नेशनल काउों ससल ऑफ एप्लाइड इकोनॉसमक ररसचध (NCAER) के
सहयोग से िई नदल्ली, नदल्ली में “अोंडरस्ट्ैं सडों ग द साइकोलॉजी ऑफ़ स्कैम्स: हाउ टू अवॉइड फ़्ॉडु लेंट स्कीम्स” पर एक
सम्मेलन आयोसजत सकया।
i.सम्मेलन में सवसभन्न वगों से ससक्रय भागीदारी प्राप्त हुई, सजसमें र्ोखार्डी वाली योजनाओों का मुकाबला करने के महत्व
पर जोर सदया गया। सवशेषज्ञोों और उपक्कथित लोगोों के बीच आकषधक चचाधओों ने महत्वपूिध प्रभाव डाला।

67. इं नियि बैंि िी टै गिाइि क्ा है ?


1)वी अोंडरस्ट्ैं ड योर वडध
2)सपोटध ऑल द वे
3)योर ओन बैंक
4)टु गेदर वी कैन
5)योर परफेर बैंसकोंग पाटध नर
उत्तर-3)योर ओि बैंि
स्पष्टीिरण:
ि बैंि िे बारे में:
प्रबंध निदे शि (MD) & मुख्य िायािारी अनधिारी (CEO) - शाोंसत लाल जैन
मुख्यािय – चेन्नई, तसमलनाडु

Report Errors in the PDF - ebooks@affairscloud.com Copyright 2014-2023 @ AffairsCloud.com 242


स्थापिा – 1907
टै गिाइि - योर ओि बैंि

68. अगस्त 2023 ति, निम्ननिखित में से निस अंतररक्ष एिेंसी िे अंतररक्ष में सौर नमशि िॉन्च िही ं निया है ?
1)राष्ट्रीय वैमासनकी एवों अोंतररक्ष प्रशासन
2)भारतीय अोंतररक्ष अनुसोंर्ान सोंगठन
3)जापान एयरोस्पेस एक्सप्लोरे शन एजेंसी
4)चीन राष्ट्रीय अोंतररक्ष प्रशासन
5)रोस्कोस्मोस
उत्तर-5)रोस्कोस्मोस
स्पष्टीिरण:
ISRO का आसदत्य-L1 सौर समशन सूयध का अध्ययन करने का भारत का पहला प्रयास है। इससे पहले , यूरोप, सोंयुक्त
राज्य अमेररका, जापान और चीन की अोंतररक्ष एजेंससयाों ऐसे समशन लॉन्च कर चुकी हैं।
i.भारतीय अोंतररक्ष अनुसोंर्ान सोंगठन (ISRO) 2 ससतोंबर 2023 को भारत का पहला सौर समशन, आसदत्य-L1 लॉन्च करने
के सलए तैयार है। आसदत्य-L1 अोंतररक्ष यान पृथ्वी और सूयध के बीच लैग्रेंज सबों दु L-1 तक 125 सदन की यात्रा करे गा। जो
ग्रह से 1.5 समसलयन सकलोमीटर दू र है। आसदत्य-L1 समशन का प्रक्षेपि श्रीहररकोटा के सतीश र्वन अोंतररक्ष केंद्र में
होगा।
• जापान: जापान एयरोस्पेस एक्सप्लोरे शन एजेंसी (JAXA)
• सोंयुक्त राज्य अमेररका: US अोंतररक्ष एजेंसी राष्ट्रीय वैमासनकी एवों अोंतररक्ष प्रशासन
• यूरोप: यूरोपीय अोंतररक्ष एजेंसी ने NASA और JAXA के सहयोग से कई सौर समशन लॉन्च सकए हैं
• चीन: चीन राष्ट्रीय अोंतररक्ष सवज्ञान केंद्र, चीनी सवज्ञान अकादमी (CAS)

69. िम्बििोंिा िन्यिीि अभ्यारण्य निस राज्य में खस्थत है ?


1)केरल
2)कनाधटक
3)तसमलनाडु
4)आों ध्र प्रदे श
5)ओसडशा
उत्तर-4)आं ध्र प्रदे श
स्पष्टीिरण:
िम्बििोंिा िन्यिीि अभ्यारण्य आं ध्र प्रदे श के पास सवशाखापत्तनम के पास क्कथित एक जोंगल है।
i.अभ्यारण्य एक उष्णकसटबोंर्ीय पिधपाती जोंगल है सजसमें झासडयोों और घास के मैदानोों के साि-साि समसश्रत सोंरचना के
वृक्षोों की छतरी है । यह पसिम में ससम्हाचलम पहाडी श्रृोंखला और उत्तर-पूवध में गोंभीरम जलाशय से सघरा है।

70. 'तािोबा-अंधारी टाइगर ररििा' निस राज्य में खस्थत है ?


1)महाराष्ट्र
2)उत्तर प्रदे श
3)गुजरात
4)मध्य प्रदे श

Report Errors in the PDF - ebooks@affairscloud.com Copyright 2014-2023 @ AffairsCloud.com 243


5)झारखण्ड
उत्तर- 1)महाराष्टर
स्पष्टीिरण:
महाराष्टर िे बारे में:
राज्यपाि – रमेश बैस
मुख्यमंत्री - एकनाि सशोंदे
िन्यिीि अभ्यारण्य - रे हेकुरी िैकबक अभ्यारण्य, सपोंगोंगा वन्यजीव अभ्यारण्य
टाइगर ररििा - बोर टाइगर ररजवध, तािोबा-अंधारी टाइगर ररििा, नवेगाोंव - नागजीरा टाइगर ररजवध (NNTR)

71. भीमगढ़ िन्यिीि अभ्यारण्य निस राज्य में खस्थत है ?


1)आों ध्र प्रदे श
2)तसमलनाडु
3)केरल
4)तेलोंगाना
5)कनाधटक
उत्तर-5)ििााटि
स्पष्टीिरण:
भीमगढ़ िन्यिीि अभ्यारण्य ििााटि के बेलगाम सजले में पसिमी घाट में एक सोंरसक्षत क्षेत्र है।
i.भीमगढ के जोंगल बारापेड गुफाओों के सलए उल्लेखनीय हैं , जो रॉटन के मुक्त पूोंछ वाले चमगादड का एकमात्र ज्ञात
प्रजनन क्षेत्र है , जो सवलुप्त होने के कगार पर एक खतरनाक प्रजासत है।

72. 'िरिािा पक्षी अभ्यारण्य' निस राज्य में खस्थत है ?


1)केरल
2)कनाधटक
3)मध्य प्रदे श
4)महाराष्ट्र
5)तेलोंगाना
उत्तर-4)महाराष्टर
स्पष्टीिरण:
िरिािा पक्षी अभ्यारण्य महाराष्टर के रायगढ सजले में क्कथित है।
i.यह पसिमी घाट में वुडलैंड पसक्षयोों का सनवास थिान है और प्रससद्ध पक्षी सवज्ञानी डॉ सलीम अली के पसोंदीदा क्षेत्रोों में से
एक िा।
ii.यह पातालगोंगा नदी का जलग्रहि क्षेत्र भी है।

73. इं टरिेशिि शूनटं ग स्पोटा फेिरे शि (ISSF) िी स्थापिा निस िषा िी गई थी?
1)1964
2)1907
3)1936
4)1911

Report Errors in the PDF - ebooks@affairscloud.com Copyright 2014-2023 @ AffairsCloud.com 244


5)1940
उत्तर- 2)1907
स्पष्टीिरण:
इं टरिेशिि शूनटं ग स्पोटा फेिरे शि (ISSF) िे बारे में:
अध्यक्ष - लुससयानो रॉसी
मुख्यािय - म्यूसनख, जमधनी
स्थापिा - 1907

74. इं टरिेशिि नबििेस मशीि (IBM) िॉपोरे शि िा ितामाि (नसतंबर'23 ति) मुख्य िायािारी अनधिारी
(CEO) िौि है ?
1)सनमधल शमाध
2)सदनेश कुमार
3)अरसवोंद कृष्ण
4)राजेश
5)जयोंत मािुर
उत्तर-3)अरनिंद िृष्ण
स्पष्टीिरण:
इं टरिेशिि नबििेस मशीि(IBM) िॉपोरे शि िे बारे में:
IBM की थिापना 1911 में कोंप्यूसटों ग-टे बुलेसटों ग-ररकॉसडिं ग कोंपनी (CTR) के रूप में की गई िी और बाद में 1924 में
इसका नाम बदलकर इों टरनेशनल सबजनेस मशीन कर सदया गया।
मुख्य िायािारी अनधिारी (CEO)- अरनिंद िृष्णा
मुख्यािय- न्यूयॉकध, सोंयुक्त राज्य अमेररका (USA)

75. बैंि ऑफ बडौदा िी टै गिाइि क्ा है ?


1)वी अोंडरस्ट्ैं ड योर वडध
2)सपोटध ऑल द वे
3)योर ओन बैंक
4)इों सडयाज इों टरनेशनल बैंक
5)योर परफेर बैंसकोंग पाटध नर
उत्तर-4)इं नियाि इं टरिेशिि बैंि
स्पष्टीिरण:
बैंि ऑफ बडौदा (BoB) िे बारे में:
MD & CEO– दे बदत्त चोंद
मुख्यािय– वडोदरा, गुजरात
टै गिाइि- इं नियाि इं टरिेशिि बैंि
स्थापिा - 1908

Report Errors in the PDF - ebooks@affairscloud.com Copyright 2014-2023 @ AffairsCloud.com 245


76. 'अष्टधातु' से बिी ___________ फीट ऊंची 'िटराि' िी मूनता िई नदल्ली, नदल्ली िे भारत मंिपम में स्थानपत
है िहां नसतंबर 2023 में G20 नशिर सम्मेिि आयोनित निया िाएगा।
1)45
2)32
3)27
4)56
5)42
उत्तर- 3)27
स्पष्टीिरण:
अष्टधातु से बनी िटराि िी मूनता नई सदल्ली, सदल्ली के भारत मंिपम में थिासपत है , जहााँ 9 और 10 ससतोंबर, 2023
को G20 नशिर सम्मेिि आयोसजत सकया जाएगा।
i.27 फीट ऊंची, 18 टन वजनी यह मूसतध अष्ट्र्ातु से बनी सबसे ऊोंची मूसतध है और इसे तसमलनाडु के स्वामी मलाई के
प्रससद्ध मूसतधकार रार्ाकृष्णन थिापसत और उनकी टीम ने ररकॉडध 7 महीने में तैयार सकया है। चोल साम्राज्य काल से ही
रार्ाकृष्णन की 34 पीसढयााँ मूसतधयााँ बना रही हैं।
ii.ब्रह्ाोंडीय ऊजाध , रचनात्मकता और शक्कक्त का महत्वपूिध प्रतीक नटराज की यह प्रसतमा G20 सशखर सम्मेलन में
आकषधि बनने जा रही है। यह प्रसतसष्ठत पररयोजना सोंस्कृसत मोंत्रालय की टीम IGNCA िारा सोंचासलत की जाती है।

77. भारत इिेररॉनिक्स निनमटे ि (BEL) िे ितामाि (नसतंबर’23 ति) अध्यक्ष और प्रबंध निदे शि (CMD)
िौि हैं ?
1)मनोज जैन
2)दामोदर भट्टड
3)K V सुरेश कुमार
4)भानु प्रकाश श्रीवास्तव
5)सवक्रमि N
उत्तर-4)भािु प्रिाश श्रीिास्ति
स्पष्टीिरण:
भारत इिेररॉनिक्स निनमटे ि (BEL) िे बारे में:
BEL भारत सरकार के रक्षा मोंत्रालय के अर्ीन एक नवरत्न सावधजसनक क्षेत्र उपक्रम (PSU) है।
अध्यक्ष और प्रबंध निदे शि (CMD)- भािु प्रिाश श्रीिास्ति
मुख्यािय– बेंगलुरु, कनाधटक
स्थापिा- 1954

78. उस िंपिी िा िाम बताइए िो हाि ही में (नसतंबर’23 में) ररस्पॉखिबि ज्वेिरी िाउं नसि िा सदस्य बिी
है।
1)सेनको गोड सलसमटे ड
2)PC र्ज्ैलर सलसमटे ड
3)जोस अलुिास समूह
4)तसनष्क
5)मालाबार सोना और हीरे

Report Errors in the PDF - ebooks@affairscloud.com Copyright 2014-2023 @ AffairsCloud.com 246


उत्तर-1)सेििो गोड निनमटे ि
स्पष्टीिरण:
दलाल स्ट्र ीट का नया बच्चा सेििो गोड निनमटे ि आभूषि और घडी उद्योग के सलए दु सनया के अग्रिी मानक-
सनर्ाधरि सोंगठन, ररस्पॉक्कर्न्बल र्ज्ेलरी काउों ससल का सदस्य बन गया है।
• RJC की आचार सोंसहता सजम्मेदार आभूषि और घडी उद्योग के सलए वैसश्वक मानक है , जो व्यावसासयक
नैसतकता और सजम्मेदार आपूसतध श्रृोंखलाओों पर ध्यान केंसद्रत करती है , जो आभूषिोों:सोना, चाोंदी, प्लैसटनम समूह
र्ातुओ,ों हीरे और रों गीन रत्नोों के सनमाधि में उपयोग सकए जाने वाले सभी प्रािसमक खसनजोों और र्ातुओों को
कवर करती है ।
• यह कदम कोंपनी की क्कथिरता यात्रा की सदशा में एक कदम है और सेनको ने कहा सक वे ररस्पॉक्कर्न्बल र्ज्ेलरी
काउों ससल का सदस्य बनने वाले भारत के पहले बडे आभूषि खुदरा सवक्रेता हैं।

79. बिाि आनियांि िाइफ इं श्योरें स िंपिी िे ितामाि (नसतंबर’23 ति) प्रबंध निदे शि (MD) और मुख्य
िायािारी अनधिारी (CEO) िौि हैं ?
1)शसश झा
2)तरूि चुघ
3)रिर्ीर कुमार
4)चोंद्रकाोंत ससोंह
5)राकेश पाोंडे
उत्तर- 2)तरूण चुघ
स्पष्टीिरण:
बिाि आनियांि िाइफ इं श्योरें स िंपिी निनमटे ि:
बजाज आसलयाोंज लाइफ इों श्योरें स कोंपनी सलसमटे ड बजाज सफनसवध सलसमटे ड और आसलयाोंज SE के बीच एक सोंयुक्त
उद्यम है।
प्रबंध निदे शि (MD) और मुख्य िायािारी अनधिारी (CEO) - तरूण चुघ
मुख्यािय – पुिे, महाराष्ट्र
स्थानपत – 2001

80. ितनिायाघाट िन्यिीि अभयारण्य निस राज्य में खस्थत है ?


1)मध्य प्रदे श
2)महाराष्ट्र
3)सबहार
4)ओसडशा
5)उत्तर प्रदे श
उत्तर-5)उत्तर प्रदे श
स्पष्टीिरण:
लखनऊ से 200 km दू र ितनिायाघाट िन्यिीि अभयारण्य, दु र्वा बाघ अभयारण्य का एक सहस्सा है। यह उत्तर
प्रदे श के बहराईच सजले के तराई बेल्ट में 400.6 वगध km के क्षेत्र को कवर करता है।
i.1987 में इसे 'प्रोजेर टाइगर' के दायरे में लाया गया।

Report Errors in the PDF - ebooks@affairscloud.com Copyright 2014-2023 @ AffairsCloud.com 247


ii.यह भारत में दु र्वा और सकशनपुर के बाघ आवासोों और नेपाल में बसदध या राष्ट्रीय उद्यान के बीच रिनीसतक
कनेक्करसवटी प्रदान करता है।

81. िािी टाइगर ररििा _____________ (राज्य) में खस्थत है।


1)कनाधटक
2)पसिम बोंगाल
3)आों ध्र प्रदे श
4)ओसडशा
5)तसमलनाडु
उत्तर- 1)ििााटि
स्पष्टीिरण:
िािी टाइगर ररििा, सजसे पहले दाोंदेली-अोंशी टाइगर ररजवध के नाम से जाना जाता िा, ििााटि राज्य के उत्तर
कन्नड सजले में क्कथित है। उत्तर कन्नड सजले की यह बेल्ट पसिमी घाट का एक सहस्सा है।
i.टाइगर ररजवध में इस क्षेत्र के दो महत्वपूिध सोंरसक्षत क्षे त्र शासमल हैं जो दाों देली वन्यजीव अभयारण्य और अोंशी राष्ट्रीय
उद्यान हैं।

82. दनक्षण पूिा एनशयाई दे शों िे संगठि (ASEAN) िा मुख्यािय िहााँ खस्थत है ?
1)जकाताध , इों डोनेसशया
2)मनीला, सफलीपीोंस
3)ढाका, बाोंग्लादे श
4)काठमाोंडू, नेपाल
5)कोलोंबो, श्रीलोंका
उत्तर-1)ििाताा, इं िोिेनशया
स्पष्टीिरण:
दनक्षण पूिा एनशयाई दे शों िे संगठि (ASEAN) िे बारे में:
स्थानपत - 8 अगस्त 1967
अध्यक्ष (2023)- इों डोनेसशया
2023 िी अध्यक्षता िा निषय- ASEAN मैटर: एसपसेंटरम ऑफ़ ग्रोि
महासनचि– डॉ. काओ सकम होनध (कोंबोसडया)
मुख्यािय– ििाताा, इं िोिेनशया

83. िोटागढ़ िन्यिीि अभयारण्य _______________ (राज्य) में खस्थत है।


1)कनाधटक
2)झारखण्ड
3)पसिम बोंगाल
4)ओसडशा
5)आों ध्र प्रदे श
उत्तर-4)ओनिशा

Report Errors in the PDF - ebooks@affairscloud.com Copyright 2014-2023 @ AffairsCloud.com 248


स्पष्टीिरण:
ओनिशा िे बारे में:
मुख्यमंत्री - नवीन पटनायक
राज्यपाि- गिेशी लाल
िन्यिीि अभयारण्य - डे ब्रीगढ वन्यजीव अभयारण्य, िोटागढ़ िन्यिीि अभयारण्य, सचल्का वन्यजीव अभयारण्य

84. संयुक्त राष्टर िी स्थापिा निस िषा हुई?


1)1971
2)1956
3)1988
4)1960
5)1945
उत्तर- 5)1945
स्पष्टीिरण:
संयुक्त राष्टर (UN) िे बारे में:
स्थानपत- 1945
महासनचि– एों टोसनयो गुटेरेस
मुख्यािय– न्यूयॉकध, सोंयुक्त राज्य अमेररका (USA)

85. सुल्तािपुर राष्टरीय उद्याि निस राज्य में खस्थत है ?


1)उत्तर प्रदे श
2)सबहार
3)हररयािा
4)मध्य प्रदे श
5)महाराष्ट्र
उत्तर-3)हररयाणा
स्पष्टीिरण:
सुल्तािपुर राष्टरीय उद्याि, सजसे पहले सुल्तानपुर पक्षी अभयारण्य के नाम से जाना जाता िा, हररयाणा के गुडगाोंव
सजले में क्कथित है । यह 1.42 वगध km में फैला हुआ है , सजसमें मुख्य रूप से आद्रध भूसमयााँ शासमल हैं ।
i.यह राष्ट्रीय उद्यान प्रवासी और सनवासी पसक्षयोों ससहत जलीय पसक्षयोों का एक प्रमु ख सनवास थिान है । इसे 2021 में
रामसर साइट, अोंतराधष्ट्रीय महत्व की आद्रध भूसम के रूप में असर्सूसचत सकया गया िा।

86. िेट्टिगुिी पक्षी अभयारण्य निस राज्य में खस्थत है ?


1)केरल
2)तसमलनाडु
3)आों ध्र प्रदे श
4)कनाधटक
5)तेलोंगाना
उत्तर- 2)तनमििािु

Report Errors in the PDF - ebooks@affairscloud.com Copyright 2014-2023 @ AffairsCloud.com 249


स्पष्टीिरण:
तनमििािु िे बारे में:
मुख्यमंत्री- M.K स्ट्ासलन
राज्यपाि- R.N रसव
पक्षी अभयारण्य- िेट्टिगुिी पक्षी अभयारण्य (सशवगोंगई), पुसलकट झील पक्षी अभयारण्य (सतरुवल्लूर)

87. नचल्का झीि निस राज्य में खस्थत है ?


1)महाराष्ट्र
2)पसिम बोंगाल
3)मध्य प्रदे श
4)ओसडशा
5)कनाधटक
उत्तर-4)ओनिशा
स्पष्टीिरण:
नचल्का झीि भारतीय उपमहािीप में ओनिशा की सबसे बडी खारे पानी की झील और पसक्षयोों की शीतकालीन आश्रय
थिल है।
i.सचल्का एसशया का सबसे बडा और सवश्व का दू सरा सबसे बडा लैगून है। 1981 में, सचल्का झील को रामसर कन्वेंशन
के तहत अोंतराधष्ट्रीय महत्व की पहली भारतीय आद्रध भूसम नासमत सकया गया िा। सचल्का का प्रमुख आकषधि इरावदी
डॉक्कल्फ़न हैं सजन्ें अक्सर सातपाडा िीप पर दे खा जाता है।

88. बैराबी-सैरांग रे ििे पररयोििा ____________ (राज्य) में खस्थत है।


1)असम
2)समजोरम
3)पसिम बोंगाल
4)ओसडशा
5)महाराष्ट्र
उत्तर- 2)नमिोरम
स्पष्टीिरण:
समजोरम में एक सनमाधिार्ीन रे लवे पुल का स्ट्ील गाडध र सगरने से उस पर काम कर रहे 23 लोगोों की मौत हो गई.
• यह पुल नमिोरम में 51.38 km लोंबी बैराबी-सैराोंग रे लवे पररयोजना के 185 पुलोों में से एक है।
• केंद्रीय रे ल मोंत्री असश्वनी वैष्णव ने मृतकोों के पररजनोों को 10-10 लाख रुपये और गोंभीर रूप से घायलोों को दो-
दो लाख रुपये का मुआवजा दे ने की घोषिा की है।
• यह पुल लहरदार पहाडी जोंगली इलाके और एक नदी पर बनाया जा रहा है और यह राज्य की राजर्ानी
आइजोल तक रे लवे कनेक्करसवटी ले जाने के सलए लोंबे समय से चल रही पररयोजना का सहस्सा है।
• ढहने का थिान आइजोल से लगभग 19 km दू र सायरों ग में िा, और पुल इस ब्रॉड गेज रे लवे पररयोजना के
अोंसतम सबोंदु, सायरों ग स्ट्े शन के रास्ते पर है ।

Report Errors in the PDF - ebooks@affairscloud.com Copyright 2014-2023 @ AffairsCloud.com 250


89. नचंचोिी िन्यिीि अभयारण्य निस राज्य में खस्थत है ?
1)ओसडशा
2)तेलोंगाना
3)पसिम बोंगाल
4)कनाधटक
5)तसमलनाडु
उत्तर-4)ििााटि
स्पष्टीिरण:
ििााटि िे बारे में:
मुख्यमंत्री– ससद्धारमैया
राज्यपाि– िावर चोंद गेहलोत
िन्यिीि अभयारण्य- कावेरी वन्यजीव अभयारण्य और नचंचोिी िन्यिीि अभयारण्य

90. REC निनमटे ि िे ितामाि (नसतंबर'23 ति) अध्यक्ष और प्रबंध निदे शि (CMD) िौि हैं ?
1)दु गेश नोंसदनी
2)सववेक कुमार दे वाोंगन
3)अजोय चौर्री
4)शशाोंक समश्रा
5)मनोज शमाध
उत्तर-2)नििेि िुमार दे िांगि
स्पष्टीिरण:
REC निनमटे ि (पूिा में ग्रामीण निद् युतीिरण निगम निनमटे ि) िे बारे में:
यह सवद् युत मोंत्रालय के अर्ीन एक महारत्न केंद्रीय सावधजसनक क्षेत्र उपक्रम है।
अध्यक्ष और प्रबंध निदे शि (CMD)– नििेि िुमार दे िांगि
मुख्यािय–गुरुग्राम, हररयािा
स्थापिा- 1969

91. िेहरू िूिॉनििि पािा निस राज्य/UT में खस्थत है ?


1)तेलोंगाना
2)सदल्ली
3)जम्मू और कश्मीर
4)गुजरात
5)महाराष्ट्र
उत्तर- 1)तेिंगािा
स्पष्टीिरण:
तेिंगािा िे बारे में:
मुख्यमंत्री– K.चोंद्रशेखर राव
राज्यपाि– तसमसलसाई सौोंदयधराजन
िूिॉनििि पािा- िेहरू िूिॉनििि पािा और वाना सवज्ञान केंद्र समनी जू(काकतीय जूलॉसजकल पाकध)

Report Errors in the PDF - ebooks@affairscloud.com Copyright 2014-2023 @ AffairsCloud.com 251


92. निस गैर-सरिारी संगठि (NGO) िे हाि ही में (नसतंबर’23 में) एि तििीिी समाधाि- िोिेट बुि िा
अिािरण निया है ?
1)स्माइल फाउों डेशन
2)उडान इों सडया फाउों डेशन
3)ईसवद्यालोक
4)Vi फाउों डेशन
5)वानासवल टर स्ट्
उत्तर- 4)Vi फाउं िेशि
स्पष्टीिरण:
दू रसोंचार ऑपरे टर Vi की CSR शाखा, Vi फाउं िेशि ने एक तकनीकी समार्ान- डोनेट बुक का अनावरि सकया है ,
सजसका उद्दे श्य पुस्तक दाताओों, सकताबोों की जरूरत वाले सोंगठनोों/स्कूलोों और इस उद्दे श्य के सलए काम करने वाले गैर
सरकारी सोंगठनोों को एक साि लाना है।
i.यह पहल Vi फाउों डेशन के कनेक्करोंग फॉर गुड कायधक्रम के तहत शुरू की जा रही है , सजसका उद्दे श्य सकताबोों तक
पहुोंच रखने वाले लोगोों और सकताबोों की जरूरत वाले लोगोों के बीच एक पुल बनाना है।

93. भारतीय िौसेिा िे िहाि (INS) िा िाम बताएं िो नसतंबर 2023 में 'एक्सरसाइि िाइट स्ट्ार- 23' में
भाग िेिे िे निए नमस्र िे पोटा अिेक्जेंनिर या पहुंचे।
1)INS सरयू
2)INS शारदा
3)INS सुमेर्ा
4)INS सासवत्री
5)INS सयुराला
उत्तर-3)INS सुमेधा
स्पष्टीिरण:
भारतीय नौसेना के जहाज (INS) सुमेधा 'एक्सरसाइि िाइट स्ट्ार- 23' में भाग लेने के सलए 06 ससतोंबर 2023 को
पोटध अलेक्जेंसडर या, समस्र पहुों चे। बहुराष्ट्रीय सत्र-सेवा सैन्य एक्सरसाइज के इस सोंस्करि में 34 दे शोों की भागीदारी होगी।
• यह मध्य पूवध और उत्तरी अफ़्ीका क्षेत्र में अब तक का सबसे बडा सोंयुक्त सैन्य एक्सरसाइजहै।
• एक्स ब्राइट स्ट्ार 23 को दो चरिोों में आयोसजत सकया जाना है। हाबधर चरि में समुद्री चरि की योजना और
सोंचालन के सलए क्रॉस-डे क दौरे , पेशेवर आदान-प्रदान, खेल सफक्स्चर और बातचीत जैसी व्यापक गसतसवसर्याों
शासमल हैं।

94. निस राज्य में पुथुर िूिॉनििि पािा 2024 में िुिेगा?
1)तसमलनाडु
2)केरल
3)कनाधटक
4)तेलोंगाना
5)आों ध्र प्रदे श
उत्तर- 2)िेरि

Report Errors in the PDF - ebooks@affairscloud.com Copyright 2014-2023 @ AffairsCloud.com 252


स्पष्टीिरण:
एसशया के सबसे बडे जूलॉसजकल पाकों में से एक पुथुर िूिॉनििि पािा को 2024 की शुरुआत तक जनता के सलए
खोल सदया जाएगा।
• केंद्रीय सचसडयाघर प्रासर्करि ने जानवरोों को सत्रशूर सचसडयाघर से िेरि के पुिुर प्रािी उद्यान में थिानाोंतररत
करने की अनुमसत दे दी है।
• सचसडयाघर में वतधमान में 117 पक्षी, 279 स्तनर्ारी और 48 सवसवर् प्रजासतयोों के 43 सरीसृप हैं। इन्ें छह माह
के भीतर नई सुसवर्ा में थिानाोंतररत करने की अनुमसत समल गई है।
• थिानाोंतरि चरिबद्ध तरीके से सकया जाएगा। पुिुर जूलॉसजकल पाकध को 360 करोड रुपये की लागत से
सवकससत सकया गया है।

95. 'ििि िन्यिीि अभयारण्य' निस राज्य में खस्थत है ?


1)झारखोंड
2)तेलोंगाना
3)आों ध्र प्रदे श
4)केरल
5)कनाधटक
उत्तर- 2)तेिंगािा
स्पष्टीिरण:
तेिंगािा िे बारे में:
मुख्यमंत्री– K.चोंद्रशेखर राव
राज्यपाि– तसमसलसाई सौोंदयधराजन
िन्यिीि अभयारण्य- जन्नाराम वन्यजीव अभयारण्य; ििि िन्यिीि अभयारण्य
टाइगर ररििा- नागाजुधनसागर श्रीशैलम टाइगर ररजवध (आों ध्र प्रदे श के साि साझा); कवल टाइगर ररजवध

96. एनशयाई नििास बैंि (ADB) िे ितामाि (नसतंबर'23 ति) अध्यक्ष िौि हैं ?
1)मासात्सुगु असकावा
2)सक्रस्ट्ासलना जॉजीवा
3)असचम स्ट्ीनर
4)डे सवड रॉबटध माल्पास
5)रीता गुियाधसत लुसबस
उत्तर- 1)मासत्सुगु असिािा
स्पष्टीिरण:
एनशयाई नििास बैंि (ADB) िे बारे में:
राष्टरपनत– मासात्सुगु असिािा
मुख्यािय– माोंडलुयॉन्क्ग ससटी, मेटरो मनीला, सफलीपीोंस
स्थापिा- 1966
सदस्य- 68 सदस्य (एसशयाई और प्रशाोंत क्षेत्र से 49 और बाहर से 19)

Report Errors in the PDF - ebooks@affairscloud.com Copyright 2014-2023 @ AffairsCloud.com 253


97. 'संिय गांधी राष्टरीय उद्याि' निस राज्य में खस्थत है ?
1)उत्तर प्रदे श
2)सबहार
3)महाराष्ट्र
4)आों ध्र प्रदे श
5)कनाधटक
उत्तर-3)महाराष्टर
स्पष्टीिरण:
संिय गांधी राष्टरीय उद्याि महाराष्टर राज्य में क्कथित है सजसे पहले बोरीवली राष्ट्रीय उद्यान के नाम से जाना जाता िा।
2400 साल पुरानी कन्ेरी गुफाएाँ पाकध के भीतर क्कथित चट्टानी चट्टान से बनाई गई हैं।

98. नसतंबर 2023 में िारी िेशिि पेमेंट्स िॉरपोरे शि ऑफ इं निया (NPCI) िे आं िडों िे अिुसार, भारत में
___________ यूनिफाइि पेमेंट्स इं टरफेस (UPI) िेिदे ि पहिी बार अगस्त 2023 में निए गए थे।
1)6 सबसलयन
2)12 सबसलयन
3)9 सबसलयन
4)8 सबसलयन
5)10सबसलयन
उत्तर- 5)10सबसलयन
स्पष्टीिरण:
नेशनल पेमेंट्स कॉरपोरे शन ऑफ इों सडया (NPCI) के आों कडोों के मुतासबक, भारत में अगस्त में पहली बार 10 नियि
यूसनफाइड पेमेंट्स इों टरफेस (UPI) लेनदे न सकए गए।
• वास्तसवक समय के मोबाइल भुगतान प्लेटफॉमध के अगस्त के अोंत में लगभग 10.5 सबसलयन लेनदे न के साि
होने की सोंभावना है , सजसका सोंचयी मूल् लगातार दू सरे महीने 15 लाख करोड रुपये को पार कर जाएगा।

99. अंतरााष्टरीय मुद्रा िोष (IMF) िी स्थापिा िब हुई थी?


1)1960
2)1948
3)1944
4)1974
5)1971
उत्तर- 3)1944
स्पष्टीिरण:
अंतरााष्टरीय मुद्रा िोष (IMF) िे बारे में:
प्रबंध निदे शि - सक्रस्ट्ासलना जॉजीवा
मुख्यािय– वासशोंगटन, D.C, USA
स्थापिा- 1944

Report Errors in the PDF - ebooks@affairscloud.com Copyright 2014-2023 @ AffairsCloud.com 254


100. 'ओििा पक्षी अभयारण्य' निस राज्य में खस्थत है ?
1)सबहार
2)उत्तर प्रदे श
3)मध्य प्रदे श
4)कनाधटक
5)महाराष्ट्र
उत्तर- 2)उत्तर प्रदे श
स्पष्टीिरण:
उत्तर प्रदे श िे बारे में:
मुख्यमंत्री (CM)- योगी आसदत्यनाि
राज्यपाि- आनोंदीबेन मफतभाई पटे ल
पक्षी अभयारण्य- समसपुर पक्षी अभयारण्य; ओििा पक्षी अभयारण्य
उत्सि- आयुर् पूजा; लट्ठ मार होली

101. आनथाि सहयोग एिं नििास संगठि (OECD) िा मुख्यािय िहााँ खस्थत है ?
1)सजनेवा, क्कस्वट् जरलैंड
2)पेररस, फ़्ाोंस
3)लोंदन, यूनाइटे ड सकोंगडम
4)रोम, इटली
5)न्यूयॉकध शहर, सोंयुक्त राज्य अमेररका
उत्तर-2)पेररस, फ्रांस
स्पष्टीिरण:
आनथाि सहयोग और नििास संगठि (OECD) िे बारे में:
महासनचि– मासियास कॉमधन
मुख्यािय– पेररस, फ्रांस

102. मुंबई पुनिस िे हाि ही में (नसतंबर'23 में) _________ (ऑपरे शि) िे तहत 5,000 से अनधि िाबानिगों
िो बचाया है।
1)ऑपरे शन कावेरी
2)ऑपरे शन गोंगा
3)ऑपरे शन सवसजल
4)ऑपरे शन यमुना
5)ऑपरे शन मुस्कान
उत्तर-5)ऑपरे शि मुस्काि
स्पष्टीिरण:
गृह मोंत्रालय की पररयोजना 'ऑपरे शि मुस्काि' के तहत मुोंबई पुसलस ने वषध (2023)तक 5,000 से असर्क नाबासलगोों
को बचाया।
i.इसका उद्दे श्य लापता बच्चोों को ढू ों ढना, उन्ें बचाना और पुनवाधस करना है। अगस्त 2023 तक मुोंबई पुसलस िारा
सपछले पाोंच वषों में बचाए गए बच्चोों की तुलना में असर्क बच्चोों को बचाया गया।

Report Errors in the PDF - ebooks@affairscloud.com Copyright 2014-2023 @ AffairsCloud.com 255


103. पिामू टाइगर ररिवा लकस राज्य में स्कस्थत है?
1)पश्चिम बंगाल
2)ओश्चिशा
3)झारखण्ड
4)मध्य प्रदे श
5)महाराष्ट्र
उत्तर-3)झारिंि
स्पष्टीकरण:
पिामू टाइगर ररिवा छोटानागपुर पठार के पश्चिमी भाग में स्थित है और झारिंि में दो श्चिलों, लातेहार और ग़िवा में
फैला हुआ है।
i.यह भूभाग घाश्चटयों, पहाश्चड़यों और मैदानों से युक्त है। तीन नश्चदयााँ अिागत् उत्तरी कोयल, औरं गा और बुरहा घाश्चटयों से
होकर बहती हैं। बुरहा एकमात्र बारहमासी नदी होने के कारि यह क्षेत्र सूखाग्रस्त है।

104. UN मनहिा िा मुख्यािय िहााँ खस्थत है ?


1)लोंदन, यूनाइटे ड सकोंगडम
2)न्यूयॉकध, सोंयुक्त राज्य अमेररका
3)सजनेवा, क्कस्वट् जरलैंड
4)रोम, इटली
5)पेररस, फ़्ाोंस
उत्तर-2)न्यूयॉिा, संयुक्त राज्य अमेररिा
स्पष्टीिरण:
UN मनहिा (िैंनगि समािता और मनहिाओं िे सशखक्तिरण िे निए UN इिाई) िे बारे में:
िायािारी निदे शि - ससमा बाहौस
मुख्यािय – न्यूयॉिा, संयुक्त राज्य अमेररिा (USA)
स्थानपत – 2010

105. यूनियि बैंि ऑफ इं निया (UBI) िी टै गिाइि क्ा है ?


1)वी अोंडरस्ट्ैं ड योर वडध
2)सपोटध ऑल द वे
3)टु गेदर वी कैन
4)गुड पीपल टू बैंक सवि
5)ऑलवेज यू फस्ट्ध
उत्तर- 4)गुि पीपि टू बैंि निथ
स्पष्टीिरण:
यूनियि बैंि ऑफ इं निया (UBI) िे बारे में:
प्रबंध निदे शि & CEO– A मसिमेखलाई
मुख्यािय– मुोंबई, महाराष्ट्र

Report Errors in the PDF - ebooks@affairscloud.com Copyright 2014-2023 @ AffairsCloud.com 256


टै गिाइि- गुड पीपल टू बैंक सवि
स्थापिा – 1919
राष्टरीयिरण – 1969 में हुआ

106. उस दु िाभ हरे धूमिेतु िा िाम बताइए निसिे हाि ही में (नसतंबर'23 में) चार शताखब्दयों से अनधि
समय िे बाद आिाशीय क्षेत्र में अपिी शुरुआत िी।
1)सनएों डरिल
2)सनसशमुरा
3)बल्लाल सेना
4)आल्टोस्ट्र े टस
5)ससरस
उत्तर- 2)निनशमुरा
स्पष्टीिरण:
निनशमुरा नाम का एक दु िाभ हरा धूमिेतु, चार शताक्कब्दयोों से असर्क समय के बाद हमारे आकाशीय क्षेत्र में अपनी
शुरुआत कर रहा है। यह दे खते हुए सक यह 400 वषों तक हमारे क्षेत्र में दोबारा नहीों आएगा, उत्तरी गोलार्ध के खगोल
सवज्ञान प्रशोंसकोों के पास इस खगोलीय आगोंतुक को दे खने के सलए केवल अगले सप्ताह की शुरुआत तक का समय है।
• 12 ससतोंबर को, यह र्ूमकेतु , सजसका आकार लगभग एक सकलोमीटर (लगभग आर्ा मील) है , 78 समसलयन
मील (125 समसलयन सकलोमीटर) की दू री पर पृथ्वी से शानदार ढों ग से गुजरे गा।

107. नसतंबर 2023 में, िेमार िूनियर बोिीनिया िे खििाफ 2 गोि िरिे __________ (दे श) िे शीषा गोि
स्कोरर बि गए।
1)अजेंटीना
2)पुतधगाल
3)फ्राोंस
4)बेक्कियम
5)ब्राज़ील
उत्तर-5)िािीि
स्पष्टीिरण:
08 ससतोंबर, 2023 को बेलेम के जोनधसलस्ट्ा एडगर ऑगस्ट्ो प्रोएनका स्ट्े ट स्ट्े सडयम में बोसलसवया के क्कखलाफ अपने दो
गोलोों में से पहला गोल करने के बाद िेमार िूनियर िािीि के सलए अग्रिी गोल स्कोरर के रूप में पेले से आगे
सनकल गए।
• नेमार को पेले से आगे करने वाला गोल 61वें समनट में आया जब ब्राजील पहले से ही 3-0 से आगे िा। नेमार को
पेनल्टी बॉक्स में एक सनचला क्रॉस समला, सजसे स्ट्र ाइकर ने करीब से सटीकता के साि समाप्त सकया।
• ऐसा तब हुआ जब खेल के 17वें समनट में अल-सहलाल का क्कखलाडी पेनल्टी चूक गया िा। उन्ोोंने इों जुरी टाइम
में सफर से गोल करके ब्राजील को 5-1 से जीत सदला दी.

Report Errors in the PDF - ebooks@affairscloud.com Copyright 2014-2023 @ AffairsCloud.com 257


108. हाि ही में (नसतंबर'23 में) निस राज्य में आनथाि रूप से िमिोर और िमिोर िगों िे निए 8 रुपये
प्रनत भोिि पर पौनष्टि भोिि उपिब्ध िरािे िे निए इं नदरा रसोई योििा िा निस्तार निया गया है ?
1)पसिम बोंगाल
2)राजथिान
3)हररयािा
4)ओसडशा
5)तेलोंगाना
उत्तर- 2)रािस्थाि
स्पष्टीिरण:
मसहलाओों को सशक्त बनाने के साि-साि भूख से सनपटने की सदशा में एक कदम उठाते हुए, काोंग्रेस महाससचव
सप्रयोंका गाों र्ी वाडर ा ने आसर्काररक तौर पर इं नदरा रसोई योििा को रािस्थाि के ग्रामीि सहस्सोों तक बढा सदया है।
i.राज्य के आगामी सवर्ानसभा चुनाव से पहले शसनवार को टोोंक सजले के सझलाई गाोंव में कायधक्रम का उद् घाटन सकया
गया। इस योजना का लक्ष् आसिधक रूप से कमजोर और कमजोर वगों के सलए 8 रुपये प्रसत भोजन पर पौसष्ट्क भोजन
उपलब्ध कराना है।
ii. प्रारों भ में, कायधक्रम राज्य के शहरी सहस्सोों में शुरू सकया गया िा, जहाों राजथिान के मुख्यमोंत्री अशोक गहलोत ने
इसका समिधन सकया िा। अब इसे पूरे राजथिान में 1,000 रसोई खोलने के साि शहरी क्षेत्रोों से परे राज्य के ग्रामीि
सहस्सोों तक सवस्ताररत सकया गया है।
iii.योजना के तहत पूरे राजथिान में 1,000 रसोई खोली जाएों गी, सजससे मसहलाओों के सलए 10,000 से असर्क रोजगार
के अवसर उपलब्ध होोंगे।

109. उस व्यखक्तत्व िा िाम बताइए निसे हाि ही में (नसतंबर'23 में) मनहिाओं िे फैशि िांि W िा िांि
एं बेसिर नियुक्त निया गया है।
1)आसलया भट्ट
2)सप्रयोंका चोपडा
3)अनुष्का शमाध
4)कैटरीना कैफ
5)श्रद्धा कपूर
उत्तर-3)अिुष्का शमाा
स्पष्टीिरण:
मनहिाओं िे फैशि िांि W ने अिुष्का शमाा को अपना ब्राोंड एों बेसडर बनाया है। इस साझेदारी के साि, ब्राोंड अपने
उच्च-डे सीबल उत्सव असभयान को लॉन्च करने की तैयारी कर रहा है जो उत्सव की गमी को आर्ुसनकता की भावना के
साि खूबसूरती से जोडता है ।
i.यह असभयान हर त्यौहार की एक कहानी के बारे में बात करता है जो मसहलाओों की कहानी का पयाधय है।

110. दु निया भर में निश्व EV नदिस 2023 िब मिाया गया?


1)8 ससतोंबर 2023
2)6 ससतोंबर 2023
3)10 ससतोंबर 2023
4)7 ससतोंबर 2023

Report Errors in the PDF - ebooks@affairscloud.com Copyright 2014-2023 @ AffairsCloud.com 258


5)9 ससतोंबर 2023
उत्तर- 5)9 नसतंबर 2023
स्पष्टीिरण:
निश्व EV नदिस 2023 9 नसतंबर 2023 को मनाया गया। यह सदन ई-मोसबसलटी के उत्सव का प्रतीक है।
• पहला सवश्व EV सदवस 2020 में मनाया गया िा।

111. नसतंबर 2023 में, अिीत नििाि, प्रनसि ____________, िो 'टाइम्स ऑफ इं निया' में अपिी श्रृंििा
'नििि िडा ' िे निए सबसे ज्यादा िािे िाते थे , िा निधि हो गया।
1)लेखक
2)पत्रकार
3)सोंपादक
4)काटू ध सनस्ट्
5)फोटोग्राफर
उत्तर- 4)िाटू ा निस्ट्
स्पष्टीिरण:
प्रससद्ध िाटू ा निस्ट् अिीत नििाि, जो 'टाइम्स ऑफ इों सडया' में अपनी श्रृोंखला 'सननर्न् वडध ' के सलए सबसे ज्यादा जाने
जाते िे, का कनाधटक के मैसूर में सनर्न हो गया। वह 68 वषध के िे.
i.सननान की दु सनया टीओआई पाठकोों के बीच एक बहुत लोकसप्रय जगह िी जहाों वह सदन-ब-सदन अपनी प्रसतभा का
प्रदशधन करते िे।

112. िाहर िन्यिीि अभ्यारण्यनिस राज्य में खस्थत है ?


1)पसिम बोंगाल
2)महाराष्ट्र
3)मध्य प्रदे श
4)उत्तर प्रदे श
5)हररयािा
उत्तर-5)हररयाणा
स्पष्टीिरण:
हररयाणा िे बारे में:
राज्यपाि – बोंडारू दत्तात्रेय
मुख्यमंत्री – मनोहर लाल खटटर
िन्यिीि अभ्यारण्य- सचलसछला वन्यजीव अभयारण्य, िाहर िन्यिीि अभयारण्य

113. "पािीिीनहंग िन्यिीि अभयारण्य" निस राज्य में खस्थत है ?


1)ओसडशा
2)महाराष्ट्र
3)असम
4)गुजरात
5)मध्य प्रदे श

Report Errors in the PDF - ebooks@affairscloud.com Copyright 2014-2023 @ AffairsCloud.com 259


उत्तर-3)असम
स्पष्टीिरण:
असम िे बारे में:
राज्यपाि-जगदीश मुखी
मुख्यमंत्री – सहमोंत सबस्वा सरमा
राष्टरीय उद्याि - ओरों ग राष्ट्रीय उद्यान, सदसहोंग पटकाई राष्ट्रीय उद्यान, रायमोना राष्ट्रीय उद्यान
िन्य िीिि अभ्यारण्य- बुराचापोरी वन्यजीव अभयारण्य, पािीिीनहंग िन्यिीि अभयारण्य

114. निम्ननिखित में से िौि सा संगठि निश्व बैंि िा हनथयार िही ं है ?


1)पुनसनधमाधि और सवकास के सलए अोंतराध ष्ट्रीय बैंक
2)अोंतराधष्ट्रीय सवकास सोंघ
3)अोंतराधष्ट्रीय सवत्त सनगम
4)बहुपक्षीय सनवेश गारों टी एजेंसी
5)जलवायु पररवतधन पर अोंतर सरकारी पैनल
उत्तर-5)िििायु पररिताि पर अंतर सरिारी पैिि
स्पष्टीिरण:
निश्व बैंि िे अधीि पााँच संगठि हैं :
• पुनसनधमाध ि और सवकास के सलए अोंतराध ष्ट्रीय बैं क (IBRD)
• अोंतराधष्ट्रीय सवकास सोंघ (IDA)
• अोंतराधष्ट्रीय सवत्त सनगम (IFC)
• बहुपक्षीय सनवेश गारों टी एजेंसी (MIGA)
• सनवेश सववादोों के सनपटान के सलए अोंतराध ष्ट्रीय केंद्र (ICSID)।

115. संयुक्त राष्टर शैनक्षि, िैज्ञानिि और सांस्कृनति संगठि (UNESCO) िे ितामाि (नसतंबर'23 ति)
महानिदे शि (DG) िौि हैं ?
1)माकोस प्राडो टर ॉयजो
2)ऑडर े अज़ोले
3)डे सवड माल्पास
4)न्क्गोजी ओकोन्क्जो-इवेला
5)सक्रस्ट्ासलना जॉजीवा
उत्तर-2)ऑिर े अज़ोिे
स्पष्टीिरण:
संयुक्त राष्टर शैनक्षि, िैज्ञानिि और सांस्कृनति संगठि (UNESCO) िे बारे में:
महानिदे शि- ऑिर े अज़ोिे
मुख्यािय- पेररस, फ्राोंस
स्थापिा- 1945 (1946 में लागू हुआ)
सदस्य- 193 सदस्य और 11 सहयोगी सदस्य।

Report Errors in the PDF - ebooks@affairscloud.com Copyright 2014-2023 @ AffairsCloud.com 260


116. 'िुमारिोम पक्षी अभयारण्य' निस राज्य में खस्थत है ?
1)केरल
2)तसमलनाडु
3)कनाधटक
4)तेलोंगाना
5)आों ध्र प्रदे श
उत्तर- 1)िेरि
स्पष्टीिरण:
वेम्बनाड झील िेरि की सबसे बडी झील है। झील समुद्र तल पर क्कथित है और एक सोंकीिध बार्ा िीप िारा लक्षिीप
सागर से अलग की गई है
i.भारी असतक्रमि और पाररक्कथिसतकी के सवनाश के कारि सपछले 120 वषों में वेम्बनाड झील की जल र्ारि क्षमता
85% कम हो गई िी।
ii.िुमारिोम पक्षी अभ्यारण्यभारत के केरल राज्य में कोट्टायम सजले के कोट्टायम तालुक में वेम्बनाड झील के तट पर
कुमारकोम में क्कथित है।

117. िं पा टाइगर ररििा निस राज्य में खस्थत है ?


1)ससक्किम
2)प्रदे श
3)समजोरम
4)मेघालय
5)असम
उत्तर-3)नमिोरम
स्पष्टीिरण:
िं पा टाइगर ररििा भारत के पूवोत्तर राज्य नमिोरम में क्कथित है। यह दे श के प्रमुख बाघ अभ्यारण्योों में से एक है और
अपनी समृद्ध जैव सवसवर्ता और प्राचीन प्राकृसतक सुोंदरता के सलए जाना जाता है।
i.ररज़वध लगभग 500 वगध सकलोमीटर के क्षेत्र को कवर करता है । डम्पा टाइगर ररज़वध सवसभन्न प्रकार की वनस्पसतयोों और
जीवोों का घर है , सजनमें कई लुप्तप्राय और थिासनक प्रजासतयााँ शासमल हैं ।
ii.बाघोों के अलावा, ररज़वध एसशयाई हासियोों, क्लाउडे ड तेंदुए, तेंदुए सबक्कल्लयोों, जोंगली सूअर, भौोंकने वाले सहरि, साोंभर
सहरि और प्राइमेट्स की सवसभन्न प्रजासतयोों जैसे जानवरोों की आबादी का समिधन करता है।

118. 'संयुक्त राष्टर बाि कोष (UNICEF)' की स्थापना लकस वषा की गई थी?
1) 1936
2) 1956
3) 1946
4) 1949
5) 1959
उत्तर- 3) 1946

Report Errors in the PDF - ebooks@affairscloud.com Copyright 2014-2023 @ AffairsCloud.com 261


स्पष्टीकरण:
संयुक्त राष्टर बाि कोष (UNICEF) के बारे में:
कायाकारी लनदे शक- कैिरीन M. रसेल
मुख्यािय- न्यूयॉकग, USA
स्थापना- 1946

119. 'गंगऊ वन्यिीव अभयारण्य' लकस राज्य में स्कस्थत है?


1)मध्य प्रदे श
2)महाराष्ट्र
3)गुिरात
4)उत्तर प्रदे श
5)श्चबहार
उत्तर- 1)मध्य प्रदे श
स्पष्टीकरण:
मध्य प्रदे श (MP) के बारे में:
मुख्यमंत्री (CM)-श्चशवराि श्चसंह चौहान
राज्यपाि – मंगुभाई पटे ल
राष्टरीय उद्यान - पन्ना राष्ट्रीय उद्यान; पेंच राष्ट्रीय उद्यान
वन्यिीव अभयारण्य (WLS) - गंगऊ वन्यिीव अभयारण्य; घाटीगांव वन्यिीव अभयारण्य

120. लििू-सैिोवा राष्टरीय उद्यान लकस राज्य में स्कस्थत है ?


1)अरुिाचल प्रदे श
2)असम
3)पश्चिम बंगाल
4)श्चत्रपुरा
5) श्चमिोरम
उत्तर- 2)असम
स्पष्टीकरण:
असम के बारे में:
राज्यपाि-िगदीश मुखी
राष्टरीय उद्यान - कािीरं गा राष्ट्रीय उद्यान, लििू - सैिोवा राष्टरीय उद्यान
वन्यिीव अभयारण्य - गरमपानी वन्यिीव अभयारण्य, लाओखोवा वन्यिीव अभयारण्य

121. दनक्षण-दनक्षण सहयोग िे निए संयुक्त राष्टर िायाािय (UNOSSC) िा मुख्यािय िहााँ खस्थत है ?
1)न्यूयॉकध, सोंयुक्त राज्य अमेररका
2)सजनेवा, क्कस्वट् जरलैंड
3)रोम, इटली
4)सवयना, ऑक्कस्ट्रया
5)पेररस, फ़्ाोंस

Report Errors in the PDF - ebooks@affairscloud.com Copyright 2014-2023 @ AffairsCloud.com 262


उत्तर-1)न्यूयॉिा, संयुक्त राज्य अमेररिा
स्पष्टीिरण:
दनक्षण-दनक्षण सहयोग िे निए संयुक्त राष्टर िायाािय (UNOSSC) िे बारे में:
निदे शि– दीमा अल-खतीब
मुख्यािय– न्यूयॉिा, संयुक्त राज्य अमेररिा (USA)
स्थानपत- 1974

122. ओपि िेटििा फॉर निनिटि िॉमसा (ONDC) िे ितामाि (नसतंबर’23 ति) प्रबंध निदे शि (MD) और
मुख्य िायािारी अनधिारी (CEO) िौि हैं ?
1)श्रीकाोंत श्रीसनवासन
2)T कोशी
3)सोंदीप बहल
4)गोपाल शमाध
5)आशीष अग्रवाल
उत्तर- 2)T िोशी
स्पष्टीिरण:
ओपि िेटििा फॉर निनिटि िॉमसा (ONDC) िे बारे में:
ONDC, दु सनया की पहली समावेशी बडे पैमाने की ई-कॉमसध प्रिाली, को सदसोंबर 2021 में र्ारा 8 (कोंपनी असर्सनयम
2013 के तहत गैर-लाभकारी सोंगठन) कोंपनी के रूप में शासमल सकया गया िा।
प्रबंध निदे शि एिं मुख्य िायािारी अनधिारी– T िोशी
मुख्यािय– नई सदल्ली, सदल्ली

123. उस व्यखक्तत्व िा िाम बताइए निसे हाि ही में (नसतंबर’23 में) िेशिि एसोनसएशि ऑफ सॉफ्टिेयर
एं ि सनिास िंपिीि (NASSCOM) िा उपाध्यक्ष नियुक्त निया गया है।
1)सोंगीता गुप्ता
2)दे बजानी घोष
3)कृसतका मुरुगेसन
4)ससन्धु गोंगार्रन
5)कीसतध सेठ
उत्तर-4)नसंधु गंगाधरि
स्पष्टीिरण:
नेशनल एसोससएशन ऑफ सॉफ्टवेयर एों ड ससवधस कोंपनीज (NASSCOM) ने SAP लैब्स इों सडया के वररष्ठ उपाध्यक्ष
(SVP) और प्रबोंर् सनदे शक (MD) और SAP उपयोगकताध सक्षमता के प्रमुख ससोंर्ु गोंगार्रन को अपना नया उपाध्यक्ष
सनयुक्त सकया है । इस नई भूसमका में , गोंगार्रन भारत के टे कएड को आकार दे ने में मदद करें गे।
i.ससोंर्ु गोंगार्रन को 2023 की शुरुआत में NASSCOM GCC काउों ससल 2023-25 के अध्यक्ष के रूप में सनयुक्त सकया
गया िा।

Report Errors in the PDF - ebooks@affairscloud.com Copyright 2014-2023 @ AffairsCloud.com 263


124. 'मराट िोंगरी िन्यिीि अभ्यारण्य' निस राज्य में खस्थत है ?
1)असम
2)ससक्किम
3)अरुिाचल प्रदे श
4)उत्तर प्रदे श
5)उत्तराखोंड
उत्तर- 1)असम
स्पष्टीिरण:
असम िे बारे में:
मुख्यमंत्री- सहमोंत सबस्वा सरमा
राज्यपाि- गुलाब चोंद कटाररया
हिाई अड्डे - जोरहाट हवाई अड्डा (रोवररया हवाई अड्डा); सलोनीबारी हवाई अड्डा
िन्यिीि अभ्यारण्य- दीपोर बील वन्यजीव अभ्यारण्य; मराट िोंगरी िन्यिीि अभ्यारण्य

125. नसतंबर 2023 में, िेशिि ग्रीि नटर ब्यूिि, पूिी क्षेत्र िे ______________ (राज्य) सरिार िो तम्पारा झीि
और उसिे आसपास 'अिैध' निमााण िो रोििे िा निदे श नदया।
1)महाराष्ट्र
2)ओसडशा
3)पसिम बोंगाल
4)मध्य प्रदे श
5)गुजरात
उत्तर- 2)ओनिशा
स्पष्टीिरण:
नेशनल ग्रीन सटर ब्यूनल, पूवी क्षेत्र ने ओनिशा सरकार को तम्पारा झील और उसके आसपास 'अवैर्' सनमाधि को आगे
नहीों बढाने का सनदे श सदया है।
i.यह एक मीठे पानी की झील है जो रुसशकुल्ा नदी के दासहने सकनारे पर क्कथित है । इसे 2021 से सोंरसक्षत रामसर
साइट के रूप में नासमत सकया गया है।

126. ििेगांि-िागिीरा टाइगर ररििा निस राज्य में खस्थत है ?


1)महाराष्ट्र
2)उत्तर प्रदे श
3)गुजरात
4)मध्य प्रदे श
5)झारखण्ड
उत्तर- 1)महाराष्टर
स्पष्टीिरण:
अगस्त 2023 में, नवेगाोंव-नागजीरा टाइगर ररजवध के बफर जोन में मुडोली गाोंव के पास सडक की चपेट में आने से एक
युवा नर बाघ की मौत हो गई। यह गोोंसदया, महाराष्टर में क्कथित है।

Report Errors in the PDF - ebooks@affairscloud.com Copyright 2014-2023 @ AffairsCloud.com 264


i.नवेगाोंव पूरे महाराष्ट्र में पाई जाने वाली लगभग 60% पक्षी प्रजासतयोों का घर भी है । इसे सदसोंबर 2013 में भारत के
46वें टाइगर ररजवध के रूप में नासमत सकया गया िा।

127. अगस्त 2023 में, िेंद्र िे िुिसेिरपनट्टिम, ___________ (राज्य) में एि िए स्पेसपोटा िी स्थापिा िो
मंिूरी दी।
1)केरल
2)आों ध्र प्रदे श
3)तेलोंगाना
4)कनाधटक
5)तसमलनाडु
उत्तर-5)तनमििािु
स्पष्टीिरण:
अगस्त 2023 में, केंद्र ने कुलसेकरपसट्टनम में एक नए स्पेसपोटध की थिापना को मोंजूरी दी। यह तनमििािु के िूिुकुडी
(पूवध में तूतीकोररन सजला) में एक शहर है। इसका नाम पाोंसडयन शासक मारवमधन कुलशेखर पाोंसडयन प्रिम के नाम पर
रखा गया है।
• एक स्पेसपोटध , सजसे कॉस्मोडर ोम या लॉन्च सुसवर्ा के रूप में भी जाना जाता है , एक सवशेष साइट है सजसे
अोंतररक्ष यान, रॉकेट और उपग्रहोों को अोंतररक्ष में लॉन्च करने के सलए सडज़ाइन सकया गया है।
• यह प्रक्षेपि, लैंसडों ग और रखरखाव ससहत अोंतररक्ष यान सोंचालन के सलए आर्ार के रूप में कायध करता है।
• ISRO छोटे उपग्रह प्रक्षेपि यान (SSLV), सजसका उद्दे श्य छोटे उपग्रहोों को कक्षा में लचीली और सकफायती
पहुोंच प्रदान करना है , को स्पेसपोटध से लॉन्च सकया जाएगा।
• भारतीय अोंतररक्ष नीसत 2023 के अनुसार, गैर-सरकारी सोंथिाएों (NGE) भी स्पेसपोटध का उपयोग तब तक कर
सकती हैं जब तक वे कुछ तकनीकी और सुरक्षा शतों को पूरा करते हैं।

128. UNESCO निश्व धरोहर स्थि एिोरा गुफाएाँ निस राज्य में खस्थत है ?
1)उत्तर प्रदे श
2)सबहार
3)महाराष्ट्र
4)गुजरात
5)मध्य प्रदे श
उत्तर-3)महाराष्टर
स्पष्टीिरण:
ऐसतहाससक एिोरा गुफाएाँ , यह महाराष्टर के औरों गाबाद में चरिोंद्री पहासडयोों में क्कथित है।
i.इसे 1983 में सों युक्त राष्ट्र शैसक्षक, वैज्ञासनक और साों स्कृसतक सोंगठन (UNESCO ) िारा सवश्व र्रोहर थिल के रूप में
सूचीबद्ध सकया गया िा और यह अपने सहोंदू, बौद्ध और जैन मोंसदरोों और स्मारकोों के सलए जाना जाता है।
ii.ये गुफाएाँ सोंख्या में 34 (17 ब्राह्िवादी, 12 बौद्ध और 5 जैन) हैं और 6ठी और 11वीों शताब्दी AD के बीच की हैं। वे
सुोंदर मूसतधयोों, सचत्रोों और सभसत्तसचत्रोों से सुशोसभत हैं।

Report Errors in the PDF - ebooks@affairscloud.com Copyright 2014-2023 @ AffairsCloud.com 265


129. मुदुमिाई राष्टरीय उद्याि ___________ (राज्य) में खस्थत है।
1)तसमलनाडु
2)ओसडशा
3)आों ध्र प्रदे श
4)तेलोंगाना
5)कनाधटक
उत्तर- 1)तनमििािु
स्पष्टीिरण:
मुदुमलाई वन्यजीव अभ्यारण्य की थिापना 1942 में तसमलनाडु में क्कथित मुदुमलाई वन क्षेत्र में की गई िी।
i.मुदुमलाई वन्यजीव अभ्यारण्य में मुदुमिाई राष्टरीय उद्याि (तनमििािु ) शासमल है। मुदुमलाई वन्यजीव अभ्यारण्य
को 1990 में मुदुमलाई राष्ट्रीय उद्यान के रूप में नासमत सकया गया िा, जो 103.24 km 2 के क्षेत्र को कवर करता है।
ii.मुदुमलाई वन्यजीव अभ्यारण्य कुल 321 km 2 क्षेत्र में फैला है। मुदुमलाई को अप्रैल 2007 में तसमलनाडु राज्य
सरकार िारा बाघ अभ्यारण्य के रूप में नासमत सकया गया िा।

130. 'ितनिायाघाट िन्यिीि अभ्यारण्य' निस राज्य में खस्थत है ?


1)उत्तर प्रदे श
2)सबहार
3)पसिम बोंगाल
4)ओसडशा
5)झारखण्ड
उत्तर- 1)उत्तर प्रदे श
स्पष्टीिरण:
कतसनधयाघाट वन्यजीव अभ्यारण्य (KWS) उत्तर प्रदे श में ऊपरी गोंगा के मैदान में एक सोंरसक्षत क्षेत्र है।
• इसे 1987 में 'प्रोजेर टाइगर' के दायरे में लाया गया िा।

131. राष्टरीय िृनष और ग्रामीण नििास बैंि (NABARD) िे ितामाि (नसतंबर'23 ति) अध्यक्ष िौि हैं ?
1)रामा श्रीसनवासन
2)उसवधश शाह
3)रे वती अय्यर
4)शाजी K V
5)रवीोंद्र H ढोलसकया
उत्तर-4)शािी K V
स्पष्टीिरण:
राष्टरीय िृनष और ग्रामीण नििास बैंि (NABARD) िे बारे में:
अध्यक्ष– शािी K V
मुख्यािय– मुोंबई, महाराष्ट्र
12 जुलाई 1982 को स्थानपत

Report Errors in the PDF - ebooks@affairscloud.com Copyright 2014-2023 @ AffairsCloud.com 266


132. ____________ PM-KUSUM (प्रधािमंत्री निसाि ऊिाा सुरक्षा एिं उत्थाि महानभयाि योििा) योििा िा
िोिि मंत्रािय है।
1)नवीन एवों नवीकरिीय ऊजाध मोंत्रालय
2)जल शक्कक्त मोंत्रालय
3)सवद् युत मोंत्रालय
4)आवास और शहरी मामलोों का मोंत्रालय
5)पेटरोसलयम और प्राकृसतक गैस मोंत्रालय
उत्तर- 1)ििीि एिं ििीिरणीय ऊिाा मंत्रािय
स्पष्टीिरण:
सकसानोों को खेती के सलए सौर ससोंचाई पोंप थिासपत करने के सलए सक्कब्सडी दे ने के सलए नवीन और नवीकरिीय ऊजाध
मोंत्रालय (MNRE) िारा माचध 2019 में प्रर्ान मोंत्री सकसान ऊजाध सुरक्षा उत्थान महासभयान योजना (PM-KUSUM
योििा) शुरू की गई िी।
i.इसके 3 घटक हैं :
• घटक A: 2 MW तक की क्षमता के व्यक्कक्तगत सोंयोंत्रोों के छोटे सौर ऊजाध सोंयोंत्रोों की थिापना के माध्यम से
10,000 (मेगा वाट) MW की सौर क्षमता।
• घटक B: 20 लाख स्ट्ैं डअलोन सौर ऊजाध सोंचासलत कृसष पोंपोों की थिापना।
• घटक C: 15 लाख सग्रड से जुडे कृसष पोंपोों का सौयीकरि।

133. निस िंपिी िे हाि ही में (नसतंबर'23 में) अपिी सेिा िो भारत िाटा िे रूप में पुिः िांि निया है ?
1)एकाटध
2)िू डाटध एक्सप्रेस सलसमटे ड
3)फेडे क्स
4)ईकॉम एक्सप्रेस
5)डे ल्हीवेरी कूररयर सेवा
उत्तर-2)ब्लू िाटा एक्सप्रेस निनमटे ि
स्पष्टीिरण:
ब्लू िाटा एक्सप्रेस निनमटे ि ने अपनी सेवा, सजसे पहले डाटध प्लस के नाम से जाना जाता िा, को भारत िाटा के रूप में
पुनः ब्राोंड सकया है। कोंपनी ने कहा सक नाम पररवतधन भारत, भारत के मध्यम वगध और उभरते बाजारोों की सवसवर्
आवश्यकताओों को पूरा करने के प्रसत उसकी प्रसतबद्धता को दशाधता है। िू डाटध ने सटयर II और सटयर III शहरोों पर
ध्यान केंसद्रत करते हुए अपने सेवा योग्य थिानोों का सवस्तार सकया है , सजनकी सवत्त वषध-23 में ई-कॉमसध बाजार में क्रमशः
18.6% और 37.1% सहस्सेदारी िी।

134. नसतंबर 2023 में, िडा रे सनिंग एं टरटे िमेंट (WWE) और अल्टीमेट फाइनटं ग चैंनपयिनशप (UFC) िे
TKO ग्रुप होखडं ग्स िामि एि िई िंपिी बिािे िे निए अपिा नििय पूरा निया।
UFC िे मानिि एं िे िर ग्रुप होखडं ग्स िे पास TKO में _____________ नियंत्रि नहत होगा।
1)60%
2)49%
3)34%
4)51%

Report Errors in the PDF - ebooks@affairscloud.com Copyright 2014-2023 @ AffairsCloud.com 267


5)40%
उत्तर- 4)51%
स्पष्टीिरण:
वडध रे ससलोंग एों टरटे नमेंट (WWE) और अल्टीमेट फाइसटों ग चैंसपयनसशप (UFC) ने TKO ग्रुप होक्कडोंग्स नामक एक नई
कोंपनी बनाने के सलए अपना सवलय पूरा कर सलया है।
i.TKO एक प्रीसमयम खेल और मनोरों जन कोंपनी होगी, जो 180 दे शोों में एक अरब से असर्क प्रशोंसकोों को सेवा प्रदान
करे गी।
ii.UFC के मासलक एों डे वर ग्रुप होक्कडोंग्स के पास TKO में 51% सनयोंत्रि सहस्सेदारी होगी, जबसक मौजूदा WWE
शेयरर्ारकोों के पास शेष 49% सहस्सेदारी होगी।

135. नसतंबर 2023 में, यूरोपीय संघ िे नहमाचि प्रदे श और उत्तरािंि राज्यों में हाि ही में आई बाढ़ से
प्रभानित पररिारों िो आपातिािीि माििीय सहायता िे रूप में _____________ यूरो िी घोषणा िी।
1)2 लाख
2)5 लाख
3)1 लाख
4)3 लाख
5)8 लाख
उत्तर- 3)1 िाि
स्पष्टीिरण:
यूरोपीय संघ ने उत्तर भारत के कुछ सहस्सोों में हाल ही में आई बाढ से प्रभासवत पररवारोों को आपातकालीन मानवीय
सहायता के रूप में 1 िाि यूरो की घोषिा की। इस सहायता से सहमाचल प्रदे श और उत्तराखोंड राज्योों में सबसे
असर्क प्रभासवत इलाकोों में 40,000 लोगोों को लाभ होने की उम्मीद है।
i.EU फोंसडों ग भारतीय रे ड क्रॉस सोसाइटी (IRCS) को सतरपाल, रसोई सेट, ऊनी कोंबल और स्वच्छता सकट ससहत
आपातकालीन आश्रय और घरे लू सामान पहुोंचाने में सहायता करती है।
ii.सवत्त पोषि इों टरनेशनल फेडरे शन ऑफ रे ड क्रॉस एों ड रे ड सक्रसेंट सोसाइटीज(IFRC) के आपदा प्रसतसक्रया
आपातकालीन सनसर्(DREF) में यूरोपीय सोंघ के समग्र योगदान का सहस्सा है।

136. हाि ही में (नसतंबर'23 में) निस राज्य सरिार िे अदाितों में संपनत्त संबंधी पाररिाररि नििादों िो िम
िरिे िे निए नगफ्ट िीि योििा शुरू िी है ?
1)झारखोंड
2)सबहार
3)महाराष्ट्र
4)ओसडशा
5)उत्तर प्रदे श
उत्तर-5)उत्तर प्रदे श
स्पष्टीिरण:
सद्भाव को बढावा दे ने और सोंपसत्त से सोंबोंसर्त पाररवाररक सववादोों को कम करने के सलए, उत्तर प्रदे श सरकार ने एक
निशेष नगफ्ट िीि योजना शुरू की, सजसने 5 अगस्त, 2023 से 12 ससतोंबर, 2023 तक 43,574 से असर्क लोगोों को
लाभाक्कन्वत सकया है । इस योजना के तहत जनता को 1807.31 करोड रुपये का लाभ समला।

Report Errors in the PDF - ebooks@affairscloud.com Copyright 2014-2023 @ AffairsCloud.com 268


i.स्ट्ाम्प एवों न्यायालय शुल्क एवों पोंजीयन राज्य मोंत्री (स्वतोंत्र प्रभार) रवीन्द्र जयसवाल ने कहा सक उत्तर प्रदे श में रक्त
सोंबोंसर्योों को सोंपसत्त दान से जुडी स्ट्ाम्प ड्यूटी में महत्वपूिध बदलाव सकया गया है।
ii.नई योजना के तहत, ऐसे लेनदे न के सलए स्ट्ाोंप शुल्क घटाकर 5,000 रुपये कर सदया गया है।

137. हाि ही में (नसतंबर'23 में) निस िंपिी िे 21,935 िरोड रुपये िी पररयोििा िे तहत निमािों िी
नििीिरी शुरू िरते हुए 56 C295 पररिहि निमािों में से पहिा भारतीय िायु सेिा िो सौप
ं नदया?
1)लॉकहीड मासटध न कॉपोरे शन
2)नॉथ्रॉधप ग्रुम्मन कॉपोरे शन
3)रे सियॉन कोंपनी
4)बोइों ग कोंपनी
5)एयरबस सडफेंस एों ड स्पेस
उत्तर-5)एयरबस निफेंस एं ि स्पेस
स्पष्टीिरण:
एयरबस निफेंस एं ि स्पेस ने 21,935 करोड रुपये की पररयोजना के तहत 56 C295 पररवहन सवमानोों में से पहला
सवमान भारतीय िायु सेिा को सौोंप सदया, सजसे दो साल पहले भारत की हवाई सोंपसत्तयोों के आर्ुसनकीकरि के समग्र
उद्दे श्य के तहत सील कर सदया गया िा।
i.वायु सेना प्रमुख एयर चीफ माशधल VR चौर्री ने दसक्षिी स्पेसनश शहर सेसवले में एयरोस्पेस प्रमुख की उत्पादन सुसवर्ा
में सवमान प्राप्त सकया।
ii.सौदे के तहत, एयरबस 2025 तक सेसवले में अपनी अोंसतम असेंबली लाइन से 'िाई-अवे' क्कथिसत में पहले 16 सवमानोों
की सडलीवरी करे गा और बाद के 40 सवमानोों को दोनोों कोंपसनयोों के बीच एक औद्योसगक साझेदारी के सहस्से के रूप में
भारत में टाटा एडवाोंथड ससस्ट्म्स (TASL) िारा सनसमधत और इकट्ठा सकया जाएगा।

138. उस व्यखक्तत्व िा िाम बताइए निसे हाि ही में (नसतंबर'23 में) एनतहाद एयरिेि िा िांि एं बेसिर
नियुक्त निया गया है।
1)आसलया भट्ट
2)कैटरीना कैफ
3)दीसपका पादु कोन
4)सप्रयोंका चोपडा
5)अनुष्का शमाध
उत्तर- 2)िैटरीिा िैफ
स्पष्टीिरण:
सोंयुक्त अरब अमीरात की राष्ट्रीय एयरलाइन एनतहाद एयरिेि ने बॉलीवुड सदवा िैटरीिा िैफ को अपना िांि
एं बेसिर सनयुक्त सकया है । एक दशक से असर्क समय में यह दू सरी बार है जब खाडी क्कथित एयरलाइन ने बॉलीवुड
स्ट्ार के साि सहयोग सकया है।
i.एसतहाद के ब्राोंड एों बेसडर के रूप में , कैटरीना को रचनात्मक और आकषधक असभयान वीसडयो की एक श्रृोंखला में
सदखाया जाएगा।

Report Errors in the PDF - ebooks@affairscloud.com Copyright 2014-2023 @ AffairsCloud.com 269


139. निम्ननिखित में से िौि सा/से भारतीय राष्टरीय भुगताि निगम (NPCI) िा उत्पाद है /हैं ?
1)राष्ट्रीय स्वचासलत समाशोर्न गृह
2)तत्काल भुगतान सेवा
3)भारत सबल भुगतान प्रिाली
4)1 और 2 दोनोों
5)सभी 1, 2 और 3
उत्तर- 5)सभी 1, 2 और 3
स्पष्टीिरण:
भारतीय राष्टरीय भुगताि निगम (NPCI) िे बारे में:
CEO & MD– सदलीप अस्बे
मुख्यािय– मुोंबई, महाराष्ट्र
उत्पाद- रुपे, तत्काल भुगतान सेवा (IMPS), राष्ट्रीय स्वचासलत समाशोर्न गृह (NACH), आर्ार भुगतान सब्रज (APB)
प्रिाली, आर्ार सक्षम भुगतान प्रिाली (AePS), राष्ट्रीय सवत्तीय क्कस्वच (NFS), UPI, भारत सबल भुगतान प्रिाली (BBPS),
और राष्ट्रीय इलेररॉसनक टोल सोंग्रह (NETC)

140. संयुक्त राष्टर पयाािरण िायाक्रम (UNEP) िा मुख्यािय िहााँ खस्थत है ?


1)सजनेवा, क्कस्वट् जरलैंड
2)मैसडर ड, स्पेन
3)सवयना, ऑक्कस्ट्रया
4)लोंदन, यूनाइटे ड सकोंगडम
5)नैरोबी, केन्या
उत्तर-5)िैरोबी, िेन्या
स्पष्टीिरण:
संयुक्त राष्टर पयाािरण िायाक्रम (UNEP) िे बारे में:
िायािारी निदे शि– इों गर एों डरसन
मुख्यािय– िैरोबी, िेन्या

141. ियििाडी पक्षी अभ्यारण्य निस राज्य में खस्थत है ?


1)महाराष्ट्र
2)मध्य प्रदे श
3)राजथिान
4)उत्तर प्रदे श
5)झारखण्ड
उत्तर- 1)महाराष्टर
स्पष्टीिरण:
ियििाडी पक्षी अभ्यारण्य महाराष्टर में क्कथित है । यह सनवासी और प्रवासी पसक्षयोों की लगभग 234 प्रजासतयोों का घर
है, सजन्ें अन्य प्रासियोों के साि बाोंर् के खुले पानी के सहस्सोों से दजध सकया गया है ।

Report Errors in the PDF - ebooks@affairscloud.com Copyright 2014-2023 @ AffairsCloud.com 270


142. िम्बििोंिा िन्यिीि अभ्यारण्य निस राज्य में खस्थत है ?
1)केरल
2)तसमलनाडु
3)कनाधटक
4)तेलोंगाना
5)आों ध्र प्रदे श
उत्तर-5)आं ध्र प्रदे श
स्पष्टीिरण:
िम्बििोंिा िन्यिीि अभ्यारण्य आं ध्र प्रदे श के पास सवशाखापत्तनम के पास क्कथित एक जोंगल है।
i.अभ्यारण्य एक उष्णकसटबोंर्ीय पिधपाती जोंगल है सजसमें झासडयोों और घास के मैदानोों के साि-साि समसश्रत सोंरचना
के वृक्षोों की छतरी है।
ii.यह पसिम में ससम्हाचलम पहाडी श्रृोंखला और उत्तर-पूवध में गोंभीरम जलाशय से सघरा है।

143. रामगढ़ निषधारी टाइगर ररििा निस राज्य में खस्थत है ?


1)मध्य प्रदे श
2)उत्तर प्रदे श
3)राजथिान
4)सबहार
5)कनाधटक
उत्तर-3)रािस्थाि
स्पष्टीिरण:
रामगढ़ निशधारी टाइगर ररििा रािस्थाि के दसक्षिपूवी सहस्से में सबोंदी सजले में क्कथित है , सजसमें सवोंध्य और अरावली
दोनोों तत्वोों का प्रसतसनसर्त्व है।
i.यह उत्तर-पूवी सहस्से में रििोंभौर बाघ अभ्यारण्य और दसक्षिी सहस्से में मुकुोंदरा सहल्स बाघ अभ्यारण्य के बफर क्षेत्र
की सनरों तरता में है। मेज़, चोंबल नदी की एक सहायक नदी, बाघ अभ्यारण्य से होकर गुजरती है ।

144. िेओरा िैिी राष्टरीय उद्याि निस राज्य में खस्थत है ?


1)सबहार
2)ओसडशा
3)पसिम बोंगाल
4)उत्तराखोंड
5)गुजरात
उत्तर-3)पनिम बंगाि
स्पष्टीिरण:
नेओरा वैली नेशनल पाकध (NVNP) पनिम बंगाि के कसलम्पोोंग सजले में क्कथित है ।
i.इसकी सवशेषता उष्णकसटबोंर्ीय उपोष्णकसटबोंर्ीय, शीतोष्ण और अल्पाइन वनस्पसत है। जोंगलोों में रोडोडें डरोन, बाोंस,
ओक, फ़नध, साल आसद शासमल हैं ।

Report Errors in the PDF - ebooks@affairscloud.com Copyright 2014-2023 @ AffairsCloud.com 271


145. अमेनियाई_________ आमेनिया िी मुद्रा है।
1)दीनार
2)पाउों ड
3)रुसपया
4)यूरो
5)डर ाम
उत्तर- 5)िर ाम
स्पष्टीिरण:
आमेनिया िे बारे में:
राष्टरपनत-वाहगन खाचटु ररयन
रािधािी – येरेवान
मुद्रा - अमेनियाई िर ाम

146. भारतीय राष्टरीय अंतररक्ष संिधाि और प्रानधिरण िेंद्र (IN-SPACe) िे ितामाि (नसतंबर'23 ति) अध्यक्ष
िौि हैं ?
1)सोंध्या वेिुगोपाला शमाध
2)पवन कुमार गोयनका
3)सुर्ीर कुमार एन
4)गौतमी सुदशधन
5)सुरेश शमाध
उत्तर-2)पिि िुमार गोयििा
स्पष्टीिरण:
भारतीय राष्टरीय अंतररक्ष संिधाि और प्रानधिरण िेंद्र (IN-SPACe) िे बारे में:
IN-SPACe की थिापना अोंतररक्ष सवभाग (DOS) के तहत सनजी सोंथिाओों की सभी अोंतररक्ष क्षेत्र की गसतसवसर्योों के सलए
एकल क्कखडकी एजेंसी के रूप में की गई िी।
अध्यक्ष – िॉ. पिि िुमार गोयििा
मुख्यािय - अहमदाबाद, गुजरात
स्थानपत - 2020

147. निश्व स्वास्थ्य संगठि (WHO) िा मुख्यािय िहााँ खस्थत है ?


1)सजनेवा, क्कस्वट् जरलैंड
2)मैसडर ड, स्पेन
3)सवयना, ऑक्कस्ट्रया
4)लोंदन, यूनाइटे ड सकोंगडम
5)नैरोबी, केन्या
उत्तर-1)नििेिा, खस्वट् िरिैंि
स्पष्टीिरण:
निश्व स्वास्थ्य संगठि (WHO) िे बारे में
महानिदे शि– डॉ. टे डरोस एडनोम घेब्रेयेसस

Report Errors in the PDF - ebooks@affairscloud.com Copyright 2014-2023 @ AffairsCloud.com 272


मुख्यािय– नििेिा, खस्वट् िरिैंि
7 अप्रैल 1948 को स्थानपत

148. नसतंबर 2023 में, ___________ िास्तनिि समय में एि पररक्रमा उपग्रह िी मदद से हाई-ररज़ॉल्यूशि
छनियां िेिे िे निए एि िंप्यूनटं ग िा उपयोग िरिे िािी पहिी भारतीय फमा बि गई।
1)न्यूस्पेस इों सडया सलसमटे ड
2)गोदरे ज एयरोस्पेस
3)KaleidEO स्पेस ससस्ट्म
4)ध्रुव अोंतररक्ष
5)स्काईरूट एयरोस्पेस
उत्तर-3)KaleidEO स्पेस नसस्ट्म
स्पष्टीिरण:
KaleidEO स्पेस नसस्ट्म्स, वास्तसवक समय में पररक्रमा करने वाले उपग्रह की मदद से हाई-ररज़ॉल्ूशन छसवयाों लेने
के सलए एज कोंप्यूसटों ग का उपयोग करने वाली पहली भारतीय फमध बन गई।
• इसके साि, बेंगलुरु क्कथित स्ट्ाटध अप ने अोंतररक्ष में एज कोंप्यूसटों ग का प्रदशध न करने वाली पहली भारतीय फमध
बनकर एक नया मील का पत्थर हाससल सकया।
• कोंपनी ने कक्षा में इमेजरी का सवश्लेषि करने के सलए गहन सशक्षि आर्ाररत एल्गोररदम का उपयोग सकया,
जैसा सक मोोंटेवीसडयो, उरुग्वे में क्कथित एक उपग्रह समूह और डे टा प्रदाता कोंपनी - सैटेलॉसजक िारा कैप्चर
सकया गया िा।
• इस प्रसक्रया में, एल्गोररदम और कायाधन्वयन समिधन को चलाने के सलए हाडध वेयर ससडनी, ऑस्ट्र े सलया क्कथित एक
स्ट्ाटध अप स्पाइरल िू िारा प्रदान सकया गया िा।

149. निम्ननिखित में से निस भारतीय व्यखक्तत्व िो हाि ही में (नसतंबर'23 में) टाइम पनत्रिा िी दु निया िो
आिार दे िे िािे शीषा 100 उभरते िेताओं िी सूची में िानमत निया गया है ?
1)हरमनप्रीत कौर
2)नोंसदता वेंकटे शन
3)वीनू डे सनयल
4)1 और 2 दोनोों
5)सभी 1, 2 और 3
उत्तर- 5)सभी 1, 2 और 3
स्पष्टीिरण:
भारतीय सक्रकेटर हरमिप्रीत िौर, दो अन्य भारतीयोों, िंनदता िेंिटे शि और िीिू िै नियि के साि, टाइम पसत्रका
की दु सनया को आकार दे ने वाले शीषध 100 उभरते नेताओों की सूची में नासमत सकया गया है। मसहला सक्रकेट में कौर के
योगदान और उनकी हासलया उपलक्कब्धयोों पर प्रकाश डाला गया। तपेसदक से बचे वेंकटे शन को टीबी के इलाज को और
असर्क सकफायती बनाने के सलए उनके वकालत कायध के सलए पहचाना गया िा। इसमें भारतीय मूल के नबारुि
दासगुप्ता भी िे।
• डे सनयल, जो एक स्ट्ूसडयो, वॉलमेकसध के मासलक हैं , को यह कहते हुए उद् र्ृत सकया गया िा सक "उनके सबसे
अच्छे सशक्षक भारत के केरल में राजसमस्त्री, श्रसमक और थिानीय लोग िे।"

Report Errors in the PDF - ebooks@affairscloud.com Copyright 2014-2023 @ AffairsCloud.com 273


• भारतीय मूल के वैज्ञासनक दासगुप्ता ने एक गैर-लाभकारी सोंथिा के माध्यम से एक कायधक्रम शुरू करने में
मदद की, सजसने ओसपयोइड-ओवरडोज़-ररवससिंग दवा नालोक्सोन को असग्रम पोंक्कक्त में आने से रोकने वाली
बार्ाओों को दू र सकया।

150. निस भारतीय IT िंपिी िे हाि ही में (नसतंबर'23 में) िमािी िे िसेििोफा में साइबर निफेंस सेंटर
(CDC) िॉन्च िरिे िी योििा िी घोषणा िी है ?
1)इों फोससस सलसमटे ड
2)सवप्रो सलसमटे ड
3)TATA कोंसल्टें सी ससवधसेज सलसमटे ड
4)HCL टे क्नोलॉजीज सलसमटे ड
5)LTIमाइों डटर ी
उत्तर- 2)निप्रो
स्पष्टीिरण:
IT प्रमुख निप्रो ने जमधनी के डसेलडोफध में अपने साइबर सडफेंस सेंटर (CDC) के शुभारों भ की घोषिा की। केंद्र ग्राहकोों
को चौबीसोों घोंटे साइबर सुरक्षा सनगरानी, घटना प्रसतसक्रया, साि ही उपचारात्मक सहायता प्रदान करे गा।
i.सवप्रो और माइक्रोसॉफ्ट की वैसश्वक साझेदारी के सहस्से के रूप में , यह केंद्र माइक्रोसॉफ्ट के साइबर सुरक्षा उत्पादोों के
पोटध फोसलयो का लाभ उठाएगा, सवशेष रूप से माइक्रोसॉफ्ट सेंसटनल, एों डपॉइों ट के सलए माइक्रोसॉफ्ट सडफेंडर और IoT
समार्ानोों के सलए सडफेंडर है।

151. नसतंबर 2023 में भारत िी पहिी निरासत चंबि ररिर फ्रंट िा उद् घाटि निस राज्य में निया गया?
1)राजथिान
2)उत्तर प्रदे श
3)गुजरात
4)महाराष्ट्र
5)मध्य प्रदे श
उत्तर- 1)रािस्थाि
स्पष्टीिरण:
भारत िी पहिी धरोहर चंबि ररिर फ्रंट का रािस्थाि सवर्ानसभा अध्यक्ष डॉ. CP जोशी ने समारोह पूवधक
उद् घाटन सकया. उन्ोोंने वैसदक मोंत्रोच्चार, राजथिानी सोंगीत की र्ुनोों और लोक कलाकारोों की प्रस्तुसत के बीच नयापुरा
बावडी घाट पर पसट्टका का अनावरि कर कोटावाससयोों को सौगात दी.
• मोंगलवार शाम को कोटा सजले में दु सनया के पहले ररवरफ्रोंट के उद् घाटन समारोह में असतसियोों ने 12 घाटोों का
भ्रमि सकया और चोंबल नदी में नौकायन सकया.
• इसे चोंबल नदी के सकनारे करीब 1400 करोड रुपये की लागत से सवश्वस्तरीय चोंबल ररवर फ्रोंट और करीब 125
करोड रुपये की लागत से बने ऑक्सीजन पाकध की सौगात समली है . UDH मोंत्री शाोंसत र्ारीवाल ने कहा सक
कोटा में साकार हुआ यह डर ीम प्रोजेर शहर की दशा और सदशा बदल दे गा.
• 17 जून, 2020 को शुरू हुई यह पररयोजना कोटा बैराज बाोंर् के ठीक नीचे नदी पर बनाई गई है। यह कई
मायनोों में साबरमती ररवरफ्रोंट से अलग और बेहतर है ।

Report Errors in the PDF - ebooks@affairscloud.com Copyright 2014-2023 @ AffairsCloud.com 274


152. नसतंबर 2023 में, भारत िी पारं पररि समुद्री निरासत िो पुििीनित िरिे और सम्माि दे िे िी पहि िे
नहस्से िे रूप में, 21 मीटर िा एि "नसिा हुआ िहाि" ििंबर 2025 में ओनिशा से इं िोिेनशया िे बािी ति
13 सदस्यीय भारतीय िौसेिा दि िे साथ यात्रा शुरू िरिे िे निए तैयार है।
यह पररयोििा, भारतीय िौसेिा, संस्कृनत मंत्रािय और _____________ (िहाि निमााण िंपिी) िे साथ िेंद्र
सरिार िी एि पहि है।
1)कोचीन सशपयाडध सलसमटे ड
2)होदी इनोवेशन प्राइवेट सलसमटे ड
3)गाडध न रीच सशपसबडसध एों ड इों जीसनयसध सलसमटे ड
4)मझगाोंव डॉक सशपसबडसध सलसमटे ड
5)सहोंदुस्तान सशपयाडध सलसमटे ड
उत्तर-2)होदी इिोिेशि प्राइिेट निनमटे ि
स्पष्टीिरण:
रस्सी, तार, नाररयल फाइबर, प्राकृसतक रे सजन और तेलोों का उपयोग करके लकडी के तख्त की ससलाई करके एक
जहाज का सनमाधि करने की एक प्राचीन तकनीक, 21 मीटर का "ससले जहाज", भारत की पारों पररक समुद्री सवरासत
को पुनजीसवत करने और सम्मासनत करने की पहल के सहस्से के रूप में नवोंबर 2025 में ओसडशा से इों डोनेसशया के
बाली तक 13 सदस्यीय भारतीय नौसेना के चालक दल के साि यात्रा शुरू करने के सलए तैयार है।
i.यह पररयोजना, नौसेना, सोंस्कृसत मोंत्रालय और गोवा क्कथित जहाज सनमाधि कोंपनी होदी इिोिेशि प्राइिेट निनमटे ि के
साि केंद्र की एक पहल है , सजसकी लागत 9 करोड रुपये है और इसे पूरा होने में 22 महीने लगने की उम्मीद है।

153. 'िेल्लई िन्यिीि अभ्यारण्य' निस राज्य में खस्थत है ?


1)पसिम बोंगाल
2)तेलोंगाना
3)तसमलनाडु
4)केरल
5)कनाधटक
उत्तर-3)तनमििािु
स्पष्टीिरण:
तनमििािु (TN) िे बारे में:
िन्यिीि अभ्यारण्य- गोंगईकोोंडन सचत्तीदार सहरि अभ्यारण्य, कावेरी उत्तरी वन्यजीव अभ्यारण्य, िेल्लई िन्यिीि
अभ्यारण्य
िूिॉनििि पािा- सचडरेन पाकध, कुरुोंबपट्टी जूलॉसजकल पाकध

154. रायमोिा राष्टरीय उद्याि निस राज्य में खस्थत है ?


1)पसिम बोंगाल
2)असम
3)ओसडशा
4)उत्तर प्रदे श
5)महाराष्ट्र
उत्तर- 2)असम

Report Errors in the PDF - ebooks@affairscloud.com Copyright 2014-2023 @ AffairsCloud.com 275


स्पष्टीिरण:
रायमोिा राष्टरीय उद्याि (असम) बोडोलैंड प्रादे सशक क्षेत्र के अोंतगधत है। पाकध के क्षेत्र में असर्सूसचत ररपु ररजवध वन का
उत्तरी भाग शासमल है , जो भारत-भूटान सीमा पर फैले मानस राष्ट्रीय उद्यान के सबसे पसिमी बफर का सनमाध ि करता
है।
i.यह पसिम में सोनकोश नदी और पूवध में सरलभोंगा नदी से सघरा है। दोनोों नसदयााँ ब्रह्पुत्र की सहायक नसदयााँ हैं।
पेकुआ नदी रायमोना की दसक्षिी सीमा को पररभासषत करती है।

155. प्रत्येि िषा िौि सा संगठि निश्व आनथाि आउटिुि प्रिानशत िरे गा?
1)सवश्व बैंक
2)अोंतराधष्ट्रीय मु द्रा कोष
3)सोंयुक्त राष्ट्र सवकास कायध क्रम
4)सोंयुक्त राष्ट्र मानव सनपटान कायधक्रम
5)सवश्व व्यापार सोंगठन
उत्तर-2)अंतरााष्टरीय मुद्रा िोष
स्पष्टीिरण:
सवश्व आसिधक आउटलुक अोंतराध ष्ट्रीय मु द्रा कोष (IMF) िारा वषध में दो बार प्रकासशत सकया जाता है। यह सनकट और
मध्यम अवसर् के दौरान वैसश्वक आसिधक सवकास का सवश्लेषि प्रस्तुत करता है। यह सवश्व अिधव्यवथिा का एक
ससोंहावलोकन के साि-साि असर्क सवस्तृत सवश्लेषि भी दे ता है ।

156. अचाििमार टाइगर ररििा निस राज्य में खस्थत है ?


1)महाराष्ट्र
2)ओसडशा
3)झारखण्ड
4)पसिम बोंगाल
5)छत्तीसगढ
उत्तर-5)छत्तीसगढ़
स्पष्टीिरण:
अचानकमार टाइगर ररजवध के बारे में :
• यह छत्तीसगढ़ के सबलासपुर सजले में क्कथित है।
• इसे 2009 में बाघ अभ्यारण्य घोसषत सकया गया िा।
• यह सवशाल अचानकमार-अमरकोंटक बायोस्फीयर ररजवध का एक सहस्सा है ।
• मसनयारी नदी इस अभ्यारण्य के ठीक मध्य से बहती है , जो जोंगल की जीवन रे खा है।

157. निस राज्य सरिार िे हाि ही में (नसतंबर'23 में) निपाह िायरस (NiV) िे प्रसार िे िारण स्वास्थ्य
चेताििी िारी िी है ?
1)कनाधटक
2)तसमलनाडु
3)तेलोंगाना
4)महाराष्ट्र

Report Errors in the PDF - ebooks@affairscloud.com Copyright 2014-2023 @ AffairsCloud.com 276


5)केरल
उत्तर-5)िेरि
स्पष्टीिरण:
केरल स्वास्थ्य सवभाग ने सजले में "अप्राकृसतक" मौत के कारि मरने वाले दो लोगोों के निपाह िायरस (NiV) से
सोंक्रसमत होने का सोंदेह होने के बाद कोसझकोड में स्वास्थ्य अलटध जारी सकया।
• सनपाह वायरस सोंक्रमि एक ज़ूनोसटक बीमारी है जो जानवरोों से मनुष्योों में फैलती है।
• सनपाह वायरस एर्न्ेफलाइसटस का कारि बनने वाला जीव पैरामाइक्सोसवररडे , जीनस हेसनपावायरस पररवार
का एक RNA या राइबोन्यूक्कक्लक एससड वायरस है , और हेंडरा वायरस से सनकटता से सोंबोंसर्त है।
• यह पहली बार 1998 और 1999 में मलेसशया और ससोंगापुर में फैला िा।
• यह बीमारी टे रोपस जीनस के फल चमगादड या 'िाइों ग फॉक्स' के माध्यम से फैलती है , जो सनपाह और हेंडरा
वायरस के प्राकृसतक भोंडार मेजबान हैं।
• यह वायरस चमगादड के मूत्र और सोंभासवत रूप से चमगादड के मल, लार और जन्म दे ने वाले तरल पदािध में
मौजूद होता है।
• लक्षि: आम तौर पर, मानव सोंक्रमि बुखार, ससरददध , उनीोंदापन, भटकाव, मानससक भ्रम, कोमा और सोंभासवत
मृत्यु िारा सचसह्नत एक एर्न्ेफेसलसटक ससोंडरोम के रूप में प्रस्तुत होता है।

158. मोरिो िी रािधािी क्ा है ?


1)सलब्रेसवले
2)रबात
3)बोंजुल
4)मापुटो
5)सबसाऊ
उत्तर- 2)रबात
स्पष्टीिरण:
हाल ही में, मराकेश से 72 सकलोमीटर दसक्षि पसिम में मध्य मोरिो में 6.8 तीव्रता का एक शक्कक्तशाली भूकोंप आया।
i.मोरिो िी रािधािी रबात है और यह पसिमी उत्तरी अफ्रीका में एक पहाडी दे श है जो सीर्े सजब्राल्टर
जलडमरूमध्य के पार क्कथित है।
ii.यह अपनी असर्काों श प्राचीन वास्तुकला और यहाों तक सक अपने पारों पररक रीसत-ररवाजोों को भी बरकरार रखता है।
iii. मोरिो का सबसे बडा शहर और प्रमुख अटलाोंसटक महासागर बोंदरगाह कैसािाोंका है , जो एक औद्योसगक और
वासिक्कज्यक केंद्र है ।
iv. इसकी सीमा पूवध और दसक्षि-पूवध में अिीररया, दसक्षि में पसिमी सहारा, पसिम में अटलाोंसटक महासागर और उत्तर
में भूमध्य सागर से लगती है ।

159. अंतर-संसदीय संघ (IPU) िा मुख्यािय िहााँ खस्थत है ?


1)सजनेवा, क्कस्वट् जरलैंड
2)न्यूयॉकध, सोंयुक्त राज्य अमेररका
3)लोंदन, यूनाइटे ड सकोंगडम
4)रोम, इटली
5)पेररस, फ़्ाोंस

Report Errors in the PDF - ebooks@affairscloud.com Copyright 2014-2023 @ AffairsCloud.com 277


उत्तर-1)नििेिा, खस्वट् िरिैंि
स्पष्टीिरण:
अंतर-संसदीय संघ (IPU) िे बारे में:
िायािारी सनमनत िे अध्यक्ष और प्रमुि– डु आटे पाचेको
मुख्यािय– नििेिा, खस्वट् िरिैंि
स्थानपत- 1889

160. YES बैंि िे ितामाि (नसतंबर’23 ति) प्रबंध निदे शि (MD) और मुख्य िायािारी अनधिारी (CEO)
िौि हैं ?
1)श्याम श्रीसनवासन
2)प्रशाोंत कुमार
3)सोंदीप बख्शी
4)सुमोंत कठपासलया
5)अजय कुमार श्रीवास्तव
उत्तर- 2)प्रशांत िुमार
स्पष्टीिरण:
YES बैंि निनमटे ि िे बारे में :
प्रबंध निदे शि (MD) और मुख्य िायािारी अनधिारी (CEO)- प्रशांत िुमार
मुख्यािय– मुोंबई, महाराष्ट्र
स्थानपत- 2004

161. नसतंबर 2023 में, ऑयि इं निया निनमटे ि िे ________ (िषा) ति शुि-शून्य खस्थनत प्राप्त िरिे िे उद्दे श्य
से स्वच्छ ऊिाा पररयोििाओं में 25,000 िरोड रुपये नििेश िरिे िी योििा िी घोषणा िी।
1)2030
2)2050
3)2027
4)2040
5)2035
उत्तर- 4)2040
स्पष्टीिरण:
दे श की दू सरी सबसे बडी सरकारी खोज कोंपनी ऑयि इं निया निनमटे ि ने 2040 तक नेट-शून्य क्कथिसत हाससल करने
के लक्ष् के साि स्वच्छ ऊजाध पररयोजनाओों में 25,000 िरोड रुपये का सनवेश करने की योजना बनाई है।
i.नेट ज़ीरो रिनीसत गैस के र्ज्लन को कम करने , फोंसे हुए गैस का व्यावसासयक रूप से दोहन करने के साि-साि
हररत सबजली उत्पादन और हररत हाइडर ोजन क्षमताओों के सनमाधि के साि-साि बायोगैस और इिेनॉल सोंयोंत्र थिासपत
करने के सोंयोजन पर आर्ाररत है।
ii.कोंपनी ने प्रदू षि फैलाने वाले तरल ईोंर्न को बदलने में मदद के सलए अरुिाचल प्रदे श के खेतोों से असम तक
प्राकृसतक गैस लाने के सलए 80 -km लोंबी पाइपलाइन सबछाने की योजना बनाई है। OIL कुछ कच्ची पाइपलाइनोों को
गैस सलोंक में बदलने पर भी सवचार कर रही है।

Report Errors in the PDF - ebooks@affairscloud.com Copyright 2014-2023 @ AffairsCloud.com 278


162. िाहर िन्यिीि अभयारण्य निस राज्य में खस्थत है ?
1)ओसडशा
2)पसिम बोंगाल
3)महाराष्ट्र
4)मध्य प्रदे श
5)हररयािा
उत्तर-5)हररयाणा
स्पष्टीिरण:
हररयाणा िे बारे में:
मुख्यमंत्री– मनोहर लाल खटटर
राज्यपाि– बोंडारू दत्तात्रेय
िन्यिीि अभयारण्य- सचलसछला वन्यजीव अभयारण्य और िाहर िन्यिीि अभयारण्य

163. िौि सी िंपिी हाि ही में (नसतंबर’23 में) नसतंबर 2023 में िोएिा, उत्तर प्रदे श में आयोनित होिे िािे
भारत िे पहिे ग्रैंि नप्रक्स मोटोGP भारत िी शीषाि प्रायोिि बि गई है ?
1)भारत पेटरोसलयम कॉपोरे शन सलसमटे ड
2)सहोंदुस्तान पेटरोसलयम कॉपोरे शन सलसमटे ड
3)इों सडयन ऑयल कॉपोरे शन सलसमटे ड
4)ऑयल एों ड नेचुरल गैस कॉपोरे शन सलसमटे ड
5)ऑयल इों सडया सलसमटे ड
उत्तर-3)इं नियि ऑयि िॉपोरे शि निनमटे ि
स्पष्टीिरण:
भारतीय ऊजाध सदग्गज इं नियि ऑयि िॉपोरे शि निनमटे ि ने मोटोGP भारत के सोंस्करि के शीषधक प्रायोजन पर
कब्जा कर सलया है , जो भारत में पहला ग्रैंड सप्रक्स है , जो 22 से 24 ससतोंबर 2023 तक उत्तर प्रदे श के ग्रेटर नोएडा में
बुद्ध इों टरनेशनल ससकधट में आयोसजत सकया जाएगा। .
i.मोटोGP भारत, सजसे डोनाध स्पोट्ध स के सहयोग से फेयर स्ट्र ीट स्पोट्ध स िारा आयोसजत सकया जा रहा है , मोटोGP,
मोटो2 और मोटो3 श्रेसियोों में प्रसतस्पर्ाध करने वाली 42 टीमोों और 84 सवारोों के साि एक रोमाोंचक तमाशा होने वाला
है।

164. नसतंबर 2023 में, िागररि उड्डयि महानिदे शि (DGCA) िे 15 नसतंबर, 2028 ति _____________
अंतरााष्टरीय हिाई अड्डे (MIA) िे एयरोिर म िाइसेंस िा ििीिीिरण निया।
1)केम्पेगौडा अोंतराधष्ट्रीय हवाई अड्डा
2)मोंगलुरु अोंतराधष्ट्रीय हवाई अड्डा
3)सरदार वल्लभभाई पटे ल अोंतराध ष्ट्रीय हवाई अड्डा
4)नेताजी सुभाष चोंद्र बोस अोंतराध ष्ट्रीय हवाई अड्डा
5)छत्रपसत सशवाजी महाराज अोंतराध ष्ट्रीय हवाई अड्डा
उत्तर-2)मंगिुरु अंतरााष्टरीय हिाई अड्डा

Report Errors in the PDF - ebooks@affairscloud.com Copyright 2014-2023 @ AffairsCloud.com 279


स्पष्टीिरण:
नागररक उड्डयन महासनदे शक (DGCA) ने मंगिुरु अंतरााष्टरीय हिाई अड्डे (MIA) के एयरोडर म लाइसेंस को 15
नसतंबर, 2028 तक नवीनीकृत कर सदया है। 16 ससतोंबर, 2023 से प्रभावी नवीनीकृत लाइसेंस पाोंच साल की अवसर् के
सलए वैर् होगा।
• 16 माचध, 2022 को जारी वतधमान हवाई अड्डा लाइसेंस की वैर्ता 15 ससतोंबर, 2023 को समाप्त हो गई। हवाई
अड्डे को 12 ससतोंबर, 2023 को नवीनीकृत लाइसेंस प्राप्त हुआ।
• DGCA ने शुरू में 16 ससतोंबर, 2021 को 15 माचध, 2022 तक छह महीने की अवसर् के सलए MIA का सोंचालन
करने वाली मोंगलुरु इों टरनेशनल एयरपोटध सलसमटे ड को हवाई अड्डा लाइसेंस प्रदान सकया िा।
• नागररक उड्डयन सनयामक ने बाद में इसे 16 माचध, 2022 से 15 ससतोंबर, 2023 तक और 18 महीने के सलए
बढा सदया।
• सवमान सनयम, 1937 के सनयम 78 के तहत जारी सकया गया, हवाई अड्डा लाइसेंस हवाई अड्डा सोंचालक के सलए
एक महत्वपूिध दस्तावेज है ।

165. नसतंबर 2023 में, __________ (िंपिी) और यूिाइटे ि निंगिम सरिार पोटा टै िबोट साइट पर इिेखरर ि
आिा फिेस स्ट्ीिमेनिंग में £1.25 नबनियि िा नििेश िरिे पर सहमत हुए।
1)JSW स्ट्ील सलसमटे ड
2)आसेलरसमत्तल सनप्पॉन स्ट्ील इों सडया सलसमटे ड
3)सहोंडाल्को इों डस्ट्र ीज सलसमटे ड
4)टाटा स्ट्ील सलसमटे ड
5)एस्सार स्ट्ील इों सडया सलसमटे ड
उत्तर-4)टाटा स्ट्ीि निनमटे ि
स्पष्टीिरण:
टाटा स्ट्ीि निनमटे ि और यूिाइटे ि निंगिम सरिार पोटध टै लबोट साइट पर इलेक्कररक आकध फनेस स्ट्ीलमेसकोंग में
£1.25 सबसलयन का सनवेश करने पर सहमत हुए हैं। सरकार (यूरो) £500 समसलयन तक प्रदान करे गी।
i.इस पररयोजना का लक्ष् दस वषों में प्रत्यक्ष उत्सजधन में 50 समसलयन टन की कमी लाना और टाटा स्ट्ील की बैलेंस
शीट का पुनगधठन करना है ।

166. हाि ही में (नसतंबर'23 में) बेस मेटल्स िे मुख्य िायािारी अनधिारी और िेदांता ररसोसेि िे अंतरााष्टरीय
व्यिसायों िे अध्यक्ष िे रूप में निसे नियुक्त निया गया है ?
1)मर्ु श्रीवास्तव
2)सप्रया अग्रवाल
3)असनल अग्रवाल
4)सोनल श्रीवास्तव
5)सक्रस सग्रसफ़ि
उत्तर-5)नक्रस नग्रनफ़थ
स्पष्टीिरण:
िेदांता ररसोसेि ने बेस मे टल्स के मुख्य कायधकारी असर्कारी और अोंतराध ष्ट्रीय व्यवसायोों के अध्यक्ष के रूप में नक्रस
नग्रनफ़थ की सनयुक्कक्त की घोषिा की। उनकी सनयुक्कक्त 2 अरू बर 2023 से प्रभावी होगी.

Report Errors in the PDF - ebooks@affairscloud.com Copyright 2014-2023 @ AffairsCloud.com 280


i.वेदाोंता ररसोसेज के पास अपनी भारतीय सहायक कोंपनी वेदाोंता सलसमटे ड का 68.11 प्रसतशत सहस्सा है , सजसका तेल
और गैस, जस्ता, लौह अयस्क, एल्ूमीसनयम, सबजली और ताोंबे में महत्वपूिध सोंचालन है।

167. इं स्ट्ीट्यूशि ऑफ इं िीनियसा इं निया (IEI) िे ितामाि (नसतंबर'23 ति) अध्यक्ष िौि हैं ?
1)नीरज A. शाह
2)सशवानोंद रॉय
3)अशोक परमार
4)M चौर्री
5)सासहल भरवाड
उत्तर- 2)नशिािंद राय
स्पष्टीिरण:
इं स्ट्ीट्यूशि ऑफ इं िीनियसा इं निया (IEI) िे बारे में:
IEI को 9 ससतोंबर 1935 को तत्कालीन महामसहम सकोंग जॉजध V िारा रॉयल चाटध र के तहत शासमल सकया गया िा। स्वतोंत्रता
(1947) के बाद, IEI भारत के सोंसवर्ान के अनुच्छेद 372 के तहत सोंरसक्षत एक "बॉडी कॉपोरे ट" बन गया और इसे एक
राष्ट्रीय पररषद िारा प्रशाससत सकया जाता है ।
अध्यक्ष– Er. सशवानोंद राय
मुख्यािय– कोलकाता, पसिम बोंगाल
स्थानपत– 1920

168. निम्ननिखित में से िौि सा भारत िे 8 प्रमुि उद्योगों में से एि 'िही ं' है?
1)कोयला
2)कच्चा तेल
3)ररफाइनरी उत्पाद
4)सीमेंट
5)कपास
उत्तर-5)िपास
स्पष्टीिरण:
भारत के 8 मुख्य उद्योग कोयला, कच्चा तेल, प्राकृसतक गैस, ररफाइनरी उत्पाद, उवधरक, इस्पात, सीमेंट और सबजली
हैं।

169. DICGC िा पूणा रूप क्ा है ?


1)जमा बीमा और नकद गारों टी सनगम
2)प्रत्यक्ष बीमा और नकद गारों टी कोंपनी
3)जमा बीमा और ऋि गारों टी सनगम
4)प्रत्यक्ष बीमा और क्रेसडट गारों टी कोंपनी
5)जमा बीमा और क्रेसडट गारों टी कोंपनी
उत्तर-3)जमा बीमा और ऋि गारों टी सनगम

Report Errors in the PDF - ebooks@affairscloud.com Copyright 2014-2023 @ AffairsCloud.com 281


स्पष्टीिरण:
जमा बीमा और ऋि गारों टी सनगम (DICGC) भारतीय ररजवध बैंक (RBI) की पूिध स्वासमत्व वाली सहायक कोंपसनयोों में से
एक है।
i.इसकी थिापना 15 जुलाई 1978 को जमा बीमा और क्रेसडट गारों टी सनगम असर्सनयम, 1961 के तहत बैंकोों के ग्राहकोों
को जमा का बीमा प्रदान करने और क्रेसडट सुसवर्ाओों की गारों टी दे ने के उद्दे श्य से की गई िी।

170. 'िुिर झीि' िहााँ खस्थत है ?


1)राजथिान
2)समजोरम
3)पसिम बोंगाल
4)जम्मू और कश्मीर
5)असम
उत्तर-4)िम्मू और िश्मीर
स्पष्टीिरण:
िम्मू और िश्मीर (J&K) िे बारे में:
उपराज्यपाि -मनोज ससन्ा
रामसर स्थि- िुिर झीि, सुररों सर-मानसर झीलें, होकरा वेटलैंड

171. 'ििदापारा राष्टरीय उद्याि' निस राज्य में खस्थत है ?


1)उत्तर प्रदे श
2)असम
3)मध्य प्रदे श
4)पसिम बोंगाल
5)सबहार
उत्तर-4)पनिम बंगाि
स्पष्टीिरण:
पनिम बंगाि िे बारे में:
मुख्यमंत्री–ममता बनजी
राज्यपाि– डॉ. C.V. आनोंद बोस
राष्टरीय उद्याि- ससोंगालीला राष्ट्रीय उद्यान; नेओरा वैली राष्ट्रीय उद्यान, ििदापारा राष्टरीय उद्याि
टाइगर ररििा- बक्सा टाइगर ररजवध; सुोंदरबन टाइगर ररजवध

172. 'परखम्बिुिम टाइगर ररििा' निस राज्य में खस्थत है ?


1)तेलोंगाना
2)कनाधटक
3)तसमलनाडु
4)आों ध्र प्रदे श
5)केरल
उत्तर-5)िेरि

Report Errors in the PDF - ebooks@affairscloud.com Copyright 2014-2023 @ AffairsCloud.com 282


स्पष्टीिरण:
िेरि िे बारे में:
मुख्यमंत्री- सपनाराई सवजयन
राज्यपाि- आररफ मोहम्मद खान
िन्य िीिि अभयारण्य- अरलम वन्यजीव अभयारण्य; मालाबार वन्यजीव अभयारण्य
टाइगर ररििा- पेररयार टाइगर ररजवध; परखम्बिुिम टाइगर ररििा

173. िान्ा टाइगर ररििा निस राज्य में खस्थत है ?


1)उत्तर प्रदे श
2)महाराष्ट्र
3)सबहार
4)मध्य प्रदे श
5)झारखण्ड
उत्तर-4)मध्य प्रदे श
स्पष्टीिरण:
मध्य प्रदे श िे बारे में:
मुख्यमंत्री–सशवराज ससोंह चौहान
राज्यपाि– मोंगुभाई C. पटे ल
टाइगर ररििा- बाोंर्वगढ टाइगर ररजवध, िान्ा टाइगर ररििा

174. निश्व पशु स्वास्थ्य संगठि (WOAH) िा मुख्यािय िहााँ खस्थत है ?


1)न्यूयॉकध, सोंयुक्त राज्य अमेररका
2)पेररस, फ़्ाोंस
3)रोम, इटली
4)सवयना, ऑक्कस्ट्रया
5)सजनेवा, क्कस्वट् जरलैंड
उत्तर-2)पेररस, फ्रांस
स्पष्टीिरण:
निश्व पशु स्वास्थ्य संगठि (WOAH) िे बारे में:
महानिदे शि – मोसनक एलोइट
मुख्यािय – पेररस, फ़्ांस
स्थापिा – 1924

175. एग्रीिल्चर इं श्योरें स िंपिी ऑफ इं निया निनमटे ि (AIC) िे ितामाि (नसतंबर’23 ति) अध्यक्ष-सह-
प्रबंध निदे शि (CMD) िौि हैं ?
1)ररतेश चौहान
2)नीरजा कपूर
3)दे वेश श्रीवास्तव
4)सुसचता गुप्ता

Report Errors in the PDF - ebooks@affairscloud.com Copyright 2014-2023 @ AffairsCloud.com 283


5)सगररजा सुब्रमण्यन
उत्तर-5)नगररिा सुिमण्यि
स्पष्टीिरण:
एग्रीिल्चर इं श्योरें स िंपिी ऑफ इं निया निनमटे ि (AIC) िे बारे में:
अध्यक्ष-सह-प्रबंध निदे शि (CMD)– नगररिा सुिमण्यि
मुख्यािय – नई सदल्ली, सदल्ली
शानमि - 2002

176. नसतंबर 2023 में, सुनििा चैिि िे िे न्यूब िदी िो ________ (समुद्र) से िोडा।
1)भूमध्य सागर
2)लाल सागर
3)काला सागर
4)बाक्कल्टक सागर
5)उत्तरी सागर
उत्तर-3)िािा सागर
स्पष्टीिरण:
रूस के डर ोन हमलोों का उद्दे श्य डे न्यूब नदी के माध्यम से यूक्रेन के अनाज व्यापार मागध को बासर्त करना िा। काला
सागर अनाज सौदे से रूस के हटने के बाद इस मागध ने यूक्रेनी अनाज सनयाधत के सलए एक वैकक्कल्पक मागध प्रदान सकया।
i.सुनििा चैिि, िे न्यूब िदी का एक सवतरक, प्रमुख यूक्रेनी बोंदरगाहोों से िािा सागर तक माल पररवहन के सलए
"नदी एक्सप्रेसवे" के रूप में कायध करता है । यह अोंतदे शीय थिानोों को वैसश्वक बाजार से जोडने में महत्वपूिध भूसमका
सनभाता है।
ii.इस मागध पर यूक्रेनी बोंदरगाहोों और अनाज सुसवर्ाओों पर सोंभासवत रूसी हमलोों का ख़तरा है। इसके असतररक्त,
सुसलना चैनल के मुहाने पर भीडभाड के कारि सशसपोंग में दे री होती है। अनाज सनयाधत में अचानक वृक्कद्ध के सलए यूक्रेन
की सीसमत बोंदरगाह क्षमता भी एक सचोंता का सवषय है।

177. अगस्त 2023 में निस राज्य िी पहाडी झीिों में , भारतीय और रूसी िैज्ञानििों िे नटिोटोनटर टोि ज़ैमेंग
िामि सैिामैंिर िी एि िई प्रिानत िी पहचाि िी है ?
1)समजोरम
2)असम
3)अरुिाचल प्रदे श
4)मसिपुर
5)ससक्किम
उत्तर-4)मनणपुर
स्पष्टीिरण:
भारतीय और रूसी वैज्ञासनकोों ने मनणपुर की पहाडी झीलोों में टायिोटोनटर टोि ज़ैमेंग नामक सैलामैंडर की एक नई
प्रजासत की पहचान की है ।
• पहले इसे समान प्रजासत समझ सलया गया िा, लेसकन नई खोज आिसवक और रूपात्मक साक्ष्ोों को समलाकर
की गई है ।

Report Errors in the PDF - ebooks@affairscloud.com Copyright 2014-2023 @ AffairsCloud.com 284


i.इस सैलामैंडर की सवशेषताएों इसके करीबी ररश्तेदारोों से अलग पाई गईों, सजसमें एक सवस्तृत ससर, अच्छी तरह से
सवकससत र्नु पवधतमाला और असितीय रों ग शासमल िे।
ii.यह प्रजासत मसिपुर की खोोंगिेंग पवधत श्रृोंखला तक ही सीसमत है और इसके सीसमत सवतरि के कारि, वैज्ञासनक
सोंरक्षि प्रयासोों की आवश्यकता पर प्रकाश डालते हुए इसे IUCN रे ड सलस्ट् में एक कमजोर प्रजासत के रूप में शासमल
करने की सलाह दे ते हैं।
iii.सैलामैंडर छोटे उभयचर हैं जो कॉडाटा क्रम से सोंबोंसर्त हैं। उनके लोंबे शरीर, छोटे पैर और सचकनी, नम त्वचा होती
है।

178. नसतंबर 2023 में, संरक्षण िाया िे निए "इं नियािा िोि" अनभिेता िे समथाि िे सम्माि में
____________ िी एि िई प्रिानत, टै चीमेिोइि् स हैररसिफोिी िा िाम अनभिेता हैररसि फोिा िे िाम पर
रिा गया था।
1)सााँप
2)कछु ए
3)मेढक
4)सछपकली
5)मछली
उत्तर- 1)सााँप
स्पष्टीिरण:
सााँप की एक नई प्रजासत, टै चीमेिोइि् स हैररसिफोिी का नाम असभनेता हैररसन फोडध के नाम पर रखा गया है ,
क्ोोंसक उन्ोोंने पयाधवरिीय कारिोों का समिधन सकया है । 40 cm सरीसृप की खोज मई 2022 में ओसतशी नेशनल पाकध
में की गई िी, लेसकन शोर्कताधओों ने हाल ही में सनर्ाधररत सकया सक यह एक नई प्रजासत िी।
i.पेरू में काम करने वाले वैज्ञासनकोों ने सोंरक्षि कायध के सलए "इों सडयाना जोर्न्" असभनेता के समिधन के सम्मान में
हैररसन फोडध के नाम पर सााँप की एक नई प्रजासत का नाम रखा है।
ii.40 cm(16 इों च) सरीसृप को पहली बार मई 2022 में ओसतशी नेशनल पाकध, सैन माकोस नेशनल यूसनवससधटी के
जोंगल पहाडोों में खोजा गया िा, लेसकन यह अब तक नहीों िा सक शोर्कताधओों ने सनष्कषध सनकाला सक यह वास्तव में
पहले से अज्ञात प्रजासत िी।

179. अगस्त 2023 में नितिे भारतीय युिाओं िो इं टरिेशिि यंग इिो-हीरो अिािा 2023 िे निए िामांनित
निया गया है ?
1)3
2)8
3)5
4)10
5)4
उत्तर- 3)5
स्पष्टीिरण:
भारत के पांच युवाओों को 2023 इं टरिेशिि यंग इिो-हीरो पुरस्कार प्राप्त करने के सलए दु सनया भर के 17
निशोर पयाािरण िायाितााओ ं में नासमत सकया गया है , सजन्ोोंने दु सनया की सबसे गोंभीर पयाधवरिीय चुनौसतयोों से
सनपटने के सलए पहल की है ।

Report Errors in the PDF - ebooks@affairscloud.com Copyright 2014-2023 @ AffairsCloud.com 285


i.सजन योंग इको-योद्धाओों को उनके प्रयासोों के सलए US क्कथित गैर-लाभकारी सोंगठन, "एक्शन फॉर नेचर" िारा मान्यता
दी गई, वे मेरठ से ईहा दीसक्षत, बेंगलुरु से मान्या हषध, नई सदल्ली से सनवाधि सोमानी और मन्नत कौर और मुोंबई के
करनव रस्तोगी हैं।

180. भारतीय एथिेनटक्स महासंघ (AFI) िे ितामाि (नसतंबर’23 ति) अध्यक्ष िौि हैं ?
1)आसदल सुमाररवलम
2)लेसमन डायक
3)सेबक्कस्ट्यन कोए
4)एसडर यान पॉलेन
5)प्राइमो नेसबयोलो
उत्तर- 1)आनदि सुमाररििम
स्पष्टीिरण:
आनदि सुमाररििम को सवश्व एिलेसटक्स के चार उपाध्यक्षोों में से एक के रूप में चुिा गया है , जो वैसश्वक टर ै क और
फीड गवसनिंग बॉडी में सकसी भारतीय िारा अब तक का सवोच्च पद है।
• 65 वषीय सुमाररवलम, जो भारतीय एिलेसटक्स महासोंघ (AFI) के अध्यक्ष हैं , को होंगरी के बुडापेस्ट् में हुए WA
चुनावोों के दौरान तीसरे सबसे असर्क वोट समले।
• वह चार साल का कायधकाल पूरा करें गे। उपाध्यक्ष के रूप में अपनी नई भूसमका में , सुमाररवलम वैसश्वक स्तर पर
एिलेसटक्स के भसवष्य को आकार दे ने में मदद करने के सलए सजम्मेदार होोंगे।
• चुने गए अन्य तीन उपराष्ट्रपसत सज़मेना रे स्ट्र े पो (कोलोंसबया), राउल चैपाडो (स्पेन), और जैक्सन तुवेई (केन्या) िे।

181. ‘मराट िोंगरी िन्यिीि अभ्यारण्य' निस राज्य में खस्थत है ?


1)असम
2)ससक्किम
3)अरुिाचल प्रदे श
4)उत्तर प्रदे श
5)उत्तराखोंड
उत्तर- 1)असम
स्पष्टीिरण:
असम िे बारे में:
मुख्यमंत्री- सहमोंत सबस्वा सरमा
राज्यपाि- गुलाब चोंद कटाररया
हिाई अड्डे - जोरहाट हवाई अड्डा (रोवररया हवाई अड्डा); सलोनीबारी हवाई अड्डा
िन्यिीि अभ्यारण्य- दीपोर बील वन्यजीव अभ्यारण्य; मराट िोंगरी िन्यिीि अभ्यारण्य

182. 'िृष्णा िन्यिीि अभ्यारण्य' निस राज्य में खस्थत है ?


1)ओसडशा
2)उत्तर प्रदे श
3)केरल
4)तेलोंगाना

Report Errors in the PDF - ebooks@affairscloud.com Copyright 2014-2023 @ AffairsCloud.com 286


5)आों ध्र प्रदे श
उत्तर-5)आं ध्र प्रदे श
स्पष्टीिरण:
आं ध्र प्रदे श िे बारे में:
मुख्यमंत्री– YS जगन मोहन रे ड्डी
राज्यपाि– सवश्वभूषि हररचोंदन
राष्टरीय उद्याि– श्री वेंकटे श्वर राष्ट्रीय उद्यान
िन्यिीि अभ्यारण्य- श्रीलोंकामल्लेश्वर वन्यजीव अभ्यारण्य; िृष्णा िन्यिीि अभ्यारण्य

183. ‘माल्यािी पक्षी अभ्यारण्य' निस राज्य में खस्थत है ?


1)केरल
2)तसमलनाडु
3)कनाधटक
4)आों ध्र प्रदे श
5)तेलोंगाना
उत्तर-3)ििााटि
स्पष्टीिरण:
माल्यािी पक्षी अभ्यारण्य ििााटि के उडु पी सजले के कुोंडापुरा तालुक में क्कथित है और हाल ही में , वेटलैंड में
भारतीय ऊदसबलाव पक्षी अभ्यारण्य में दे खे गए हैं।

184. िि निहार राष्टरीय उद्याि नचनडयाघर निस राज्य में खस्थत है ?


1)पसिम बोंगाल
2)मध्य प्रदे श
3)ओसडशा
4)महाराष्ट्र
5)झारखण्ड
उत्तर- 2)मध्य प्रदे श
स्पष्टीिरण:
मध्य प्रदे श िे बारे में:
मुख्यमंत्री–सशवराज ससोंह चौहान
राज्यपाि– मोंगुभाई C. पटे ल
प्राणी उद्याि- िि निहार राष्टरीय उद्याि सचसडयाघर, कमला नेहरू प्रािी सोंग्रहालय सचसडयाघर (इों दौर सचसडयाघर)

185. पािर फाइिेंस िॉपोरे शि निनमटे ि (PFC) िे ितामाि (नसतंबर’23 ति) अध्यक्ष और प्रबंध निदे शि
(MD) िौि हैं ?
1)उषा सजीव नायर
2)अजय सतवारी
3)राजीव रों जन झा
4)परसमोंदर चोपडा

Report Errors in the PDF - ebooks@affairscloud.com Copyright 2014-2023 @ AffairsCloud.com 287


5)मनोज शमाध
उत्तर-4)परनमंदर चोपडा
स्पष्टीिरण:
पािर फाइिेंस िॉपोरे शि निनमटे ि (PFC) िे बारे में:
अध्यक्ष एिं प्रबंध निदे शि (MD)- परनमंदर चोपडा
मुख्यािय- नई सदल्ली, सदल्ली
स्थापिा- 1986

186. सांभर साल्ट िेि निस राज्य में खस्थत है ?


1)मध्य प्रदे श
2)महाराष्ट्र
3)पसिम बोंगाल
4)ओसडशा
5)राजथिान
उत्तर-5)रािस्थाि
स्पष्टीिरण:
सांभर साल्ट लेक पूवध-मध्य रािस्थाि में जयपुर से लगभग 80 km दसक्षि पसिम में क्कथित है ।
i.साल्ट लेक भारत की सबसे बडी अोंतदे शीय साल्ट लेक है। यह अरावली पवधतमाला के अवसाद का प्रसतसनसर्त्व करता
है।

187. संयुक्त राष्टर शैनक्षि, िैज्ञानिि और सांस्कृनति संगठि (UNESCO) िा मुख्यािय िहााँ खस्थत है ?
1)न्यूयॉकध, सोंयुक्त राज्य अमेररका
2)सजनेवा, क्कस्वट् जरलैंड
3)रोम, इटली
4)सवयना, ऑक्कस्ट्रया
5)पेररस, फ़्ाोंस
उत्तर-5)पेररस, फ्रांस
स्पष्टीिरण:
संयुक्त राष्टर शैनक्षि, िैज्ञानिि और सांस्कृनति संगठि (UNESCO) िे बारे में:
महानिदे शि – ऑडर े अज़ोले
मुख्यािय – पेररस, फ़्ांस
स्थानपत – 1945 (1946 में लागू हुआ)

188. निस राज्य/UT में िॉ. RML अस्पताि िे नसतंबर 2023 में टर ांसिेंिरों िे निए भारत िी पहिी समनपात
OPD िा उद् घाटि निया?
1)आों ध्र प्रदे श
2)सदल्ली
3)तसमलनाडु
4)जम्मू और कश्मीर

Report Errors in the PDF - ebooks@affairscloud.com Copyright 2014-2023 @ AffairsCloud.com 288


5)कनाधटक
उत्तर- 2)नदल्ली
स्पष्टीिरण:
सेवा पखवाडा की शुरुआत पर, डॉ. RML अस्पताल ने नई सदल्ली, नदल्ली में टर ाोंसजेंडरोों के सलए भारत की पहली
समसपधत OPD का उद् घाटन सकया। उद् घाटन समारोह का नेतृत्व RML अस्पताल के सनदे शक डॉ. (प्रो.) अजय शुक्ला ने
सकया।
i.यह पहल टर ाोंसजेंडरोों के सलए भारत की पहली समसपधत OPD में बडे पैमाने पर असुसवर्ा और भेदभाव और सामासजक
उदासीनता के डर के कारि स्वास्थ्य सेवाओों तक पहुों चने के सलए टर ाोंसजेंडर समुदाय िारा सामना की जाने वाली
कसठनाइयोों को समझने के साि शुरू की गई है।

189. नसतंबर 2023 में, ______________ (ऑपरे शि), तटीय सुरक्षा निमााण िे सभी नहतधारिों िो शानमि
िरिे िािा एि अभ्यास, पनिमी तट पर भारतीय तट रक्षि द्वारा आयोनित निया गया था।
1)ऑपरे शन सजग
2)ऑपरे शन गोंगा
3)ऑपरे शन सवसजल
4)ऑपरे शन यमुना
5)ऑपरे शन मुस्कान
उत्तर- 1)ऑपरे शि सिग
स्पष्टीिरण:
'ऑपरे शि सिग', तटीय सुरक्षा सनमाधि के सभी सहतर्ारकोों को शासमल करने वाला एक अभ्यास, 18 ससतोंबर, 2023
को पसिमी तट पर भारतीय तट रक्षि िारा आयोसजत सकया गया िा।
i.यह सडर ल तटीय सुरक्षा तोंत्र के पुनवैर्ीकरि की सुसवर्ा प्रदान करती है और समुद्र में मछु आरोों के बीच जागरूकता
लाती है।
ii.सडर ल के दौरान, समुद्र में मछली पकडने वाली सभी नौकाओों, नौकाओों और सशल्पोों के दस्तावेजोों और चालक दल के
पासोों की व्यापक जाोंच और सत्यापन सकया गया। इस अभ्यास में सीमा शुल्क, समुद्री पुसलस, बोंदरगाहोों और भारतीय
नौसेना ससहत कुल 118 जहाजोों ने भाग सलया।
iii.तटीय सुरक्षा सोंरचना को मजबूत करने के सलए, मछु आरोों के सलए बायोमेसटर क काडध जारी करने, प्रत्येक राज्य के
अनुसार मछली पकडने वाली नौकाओों की कलर कोसडों ग, मछली लैंसडों ग केंद्रोों की मैसनोंग और प्रवेश/सनकास चेक
पॉइों ट्स पर पहुोंच सनयोंत्रि, तटीय मानसचत्रि, सुरक्षा एजेंससयोों के सलए सवसशष्ट् समुद्री बैंड आवृसत्त नासमत करने से लेकर
कई उपायोों को शासमल सकया गया है। अन्य बातोों के साि-साि भारतीय तटरक्षक बल िारा समुद्री पुसलस कसमधयोों का
प्रसशक्षि।

190. 'िुिो राष्टरीय उद्याि' निस राज्य में खस्थत है ?


1)सहमाचल प्रदे श
2)उत्तराखोंड
3)महाराष्ट्र
4)अरुिाचल प्रदे श
5)मध्य प्रदे श
उत्तर-5)मध्य प्रदे श

Report Errors in the PDF - ebooks@affairscloud.com Copyright 2014-2023 @ AffairsCloud.com 289


स्पष्टीिरण:
17 ससतोंबर, 2022 को भारत में वन्यजीव सोंरक्षि के क्षेत्र में एक इसतहास दजध सकया गया, जब दु सनया का सबसे तेज़
ज़मीनी जानवर दे श से लगभग 75 वषों के थिानीय सवलुप्त होने के बाद अोंततः भारत वापस आ गया। 17 नसतंबर
2023 को भारत में प्रोिेर चीता के सफल कायाध न्वयन के एक वषध का जश्न मनाया जा रहा है।
i.पहले अोंतरमहािीपीय वन्यजीव थिानाोंतरि में और भारत में अपने एसशयाई समकक्षोों के सवलुप्त होने के दशकोों बाद,
नामीसबया से आठ अफ्रीकी चीतोों (एससनोसनक्स जुबेटस जुबेटस) को प्रर्ान मोंत्री श्री नरें द्र मोदी िारा मध्य प्रदे श के
िुिो राष्टरीय उद्याि में थिानाोंतररत सकया गया। इसके बाद, फरवरी, 2023 में दसक्षि अफ्रीका से बारह चीतोों को भी
थिानाोंतररत सकया गया और कुनो राष्ट्रीय उद्यान में छोड सदया गया।
ii.ये चीते प्राकृसतक खजाने को बहाल करने के सलए भारत की टोपी में एक बडे पोंख का प्रसतसनसर्त्व करते हैं। पूरी
पररयोजना नामीसबया, दसक्षि अफ्रीका और भारत से सोंबोंसर्त सरकारी असर्काररयोों, वैज्ञासनकोों, वन्यजीव जीवसवज्ञानी
और पशुसचसकत्सकोों की सवशेषज्ञ टीम की सावर्ानीपूवधक सनगरानी में कायाधक्कन्वत की गई िी।

191. मराठिाडा मुखक्त नदिस िब मिाया गया?


1)17 ससतोंबर 2023
2)15 ससतोंबर 2023
3)18 ससतोंबर 2023
4)16 ससतोंबर 2023
5)19 ससतोंबर 2023
उत्तर- 1)17 नसतंबर 2023
स्पष्टीिरण:
प्रर्ानमोंत्री श्री नरें द्र मोदी ने 17 नसतंबर 2023 को मराठिाडा मुखक्त नदिस पर शुभकामनाएों दी हैं।
• वतधमान महाराष्ट्र का मराठवाडा क्षेत्र तत्कालीन है दराबाद ररयासत का सहस्सा िा, जो सनज़ाम के शासन के
अर्ीन िा।
• मराठवाडा मुक्कक्त सदवस 17 ससतोंबर को पूरे महाराष्ट्र में मनाया जाता है । यह वह सदन है सजस सदन मराठवाडा
पर सनज़ाम का शासन समाप्त हुआ िा। और मराठवाडा हमारे भारत का सहस्सा बन गया। इस लेख में हम
मराठवाडा क्षेत्र के स्वतोंत्रता सोंग्राम के बारे में जानेंगे।

192. ितामाि (नसतंबर’23 ति) िेंद्रीय युिा मामिे और िेि मंत्री िौि हैं ?
1)सकरे न ररसजजू
2)परषोत्तम रूपाला
3)G.सकशन रे ड्डी
4)अनुराग ससोंह ठाकुर
5)हरदीप ससोंह पुरी
उत्तर-4)अिुराग नसंह ठािुर
स्पष्टीिरण:
केंद्रीय युवा मामले और खेल मोंत्री श्री अिुराग नसंह ठािुर ने 19 ससतोंबर 2023 को नई सदल्ली में पोंसडत दीनदयाल
उपाध्याय राष्ट्रीय क्कखलाडी कल्ाि कोष (PDUNWFS) के तहत क्कखलासडयोों को सम्मासनत सकया।

Report Errors in the PDF - ebooks@affairscloud.com Copyright 2014-2023 @ AffairsCloud.com 290


i.इस अवसर पर श्री अनुराग ससोंह ठाकुर ने घोषिा की सक पोंसडत दीनदयाल उपाध्याय नेशनल वेलफेयर फोंड फॉर
स्पोट्ध स पसधन (PDUNWFS) उन क्कखलासडयोों की मदद के सलए बनाया गया है जो अच्छा खेलते हैं लेसकन गरीब और
जरूरतमोंद पररवारोों से हैं ।
ii.यह योजना खेल उपकरि और प्रसशक्षि की खरीद में मदद करती है और अब तक 270 क्कखलासडयोों को लगभग 8
करोड 15 लाख रुपये की सहायता प्रदान की गई है।

193. अंतरााष्टरीय समुद्री संगठि (IMO) िा मुख्यािय िहााँ खस्थत है ?


1)रोम, इटली
2)पेररस, फ़्ाोंस
3)सजनेवा, क्कस्वट् जरलैंड
4)लोंदन, यूनाइटे ड सकोंगडम
5)न्यूयॉकध शहर, सोंयुक्त राज्य अमेररका
उत्तर-4)िंदि, यूिाइटे ि निंगिम
स्पष्टीिरण:
अंतरााष्टरीय समुद्री संगठि (IMO) िे बारे में:
महासनचि- सकटै क सलम
मुख्यािय- िंदि, यूिाइटे ि निंगिम
स्थापिा- 1958

194. 'मंगिििम पक्षी अभ्यारण्य' निस राज्य में खस्थत है ?


1)केरल
2)ओसडशा
3)कनाधटक
4)तेलोंगाना
5)आों ध्र प्रदे श
उत्तर- 1)िेरि
स्पष्टीिरण:
िेरि िे बारे में:
मुख्यमंत्री– सपनाराई सवजयन
राज्यपाि– आररफ मोहम्मद खान
िन्यिीि अभ्यारण्य- कोसट्टयूर वन्यजीव अभ्यारण्य; मंगिििम पक्षी अभ्यारण्य
हिाई अड्डा– कालीकट अोंतराधष्ट्रीय हवाई अड्डा; कन्नूर अोंतराधष्ट्रीय हवाई अड्डा

195. बरदा िन्यिीि अभ्यारण्य निस राज्य में खस्थत है ?


1)महाराष्ट्र
2)झारखण्ड
3)मध्य प्रदे श
4)पसिम बोंगाल
5)गुजरात

Report Errors in the PDF - ebooks@affairscloud.com Copyright 2014-2023 @ AffairsCloud.com 291


उत्तर-5)गुिरात
स्पष्टीिरण:
गुिरात िे बारे में:
मुख्यमंत्री– भूपेन्द्र रजनीकाोंत पटे ल
राज्यपाि – आचायध दे वव्रत
िन्यिीि अभ्यारण्य - शूलपनेश्वर वन्यजीव अभ्यारण्य, बरदा िन्यिीि अभ्यारण्य

196. निश्व बैंि िी स्थापिा निस िषा हुई थी?


1)1919
2)1969
3)1961
4)1944
5)1948
उत्तर- 4)1944
स्पष्टीिरण:
निश्व बैंि िे बारे में :
स्थापिा - िुिाई, 1944
मुख्यािय – वासशोंगटन DC, सोंयुक्त राज्य अमेररका (USA)
अध्यक्ष – अजय बोंगा

197. गुंििा िह्मेश्वरम िन्यिीि अभ्यारण्य निस राज्य में खस्थत है ?


1)ओसडशा
2)आों ध्र प्रदे श
3)तेलोंगाना
4)कनाधटक
5)केरल
उत्तर- 2)आं ध्र प्रदे श
स्पष्टीिरण:
आं ध्र प्रदे श िे बारे में:
मुख्यमंत्री- Y.S. जगन मोहन रे ड्डी
राज्यपाि- S. अब्दु ल नज़ीर
िन्यिीि अभ्यारण्य- नागाजुधन सागर-श्रीशैलम वन्यजीव अभ्यारण्य, गुंििा िह्मेश्वरम िन्यिीि अभ्यारण्य

198. एनशयाई नक्रिेट पररषद िे ितामाि (नसतंबर'23 ति) अध्यक्ष िौि हैं ?
1)सौरव गाोंगुली
2)शशाोंक मनोहर
3)C.K. खन्ना
4)रोजर सबन्नी
5)जय शाह

Report Errors in the PDF - ebooks@affairscloud.com Copyright 2014-2023 @ AffairsCloud.com 292


उत्तर-5)िय शाह
स्पष्टीिरण:
एनशया िप िे बारे में :
स्थापिा िषा- एसशया कप सक्रकेट टू नाधमेंट 1984 में सोंयुक्त अरब अमीरात (सवजेता - भारत) में शुरू हुआ।
प्रारूप- परों परागत रूप से यह एक ODI टू नाधमेंट रहा है , इसमें कभी-कभी T20I मैच , जैसे सक 2016 और 2022 भी
शासमल होते हैं ।
प्रारूप हर सोंस्करि में सभन्न हो सकता है।
आयोिि संस्था- एसशयाई सक्रकेट पररषद (ACC)
एनशयाई नक्रिेट पररषद िे अध्यक्ष - िय शाह

199. इनत्तरा िे निस ___________ स्मॉि फाइिेंस बैंि (SFB) िे ितामाि (नसतंबर'23 ति) प्रबंध निदे शि (MD)
हैं।
1)कैसपटल SFB
2)उिीवन SFB
3)इक्कक्वटास SFB
4)A U SFB
5)ESAF SFB
उत्तर-2)उज्ज्िीिि SFB
स्पष्टीिरण:
उज्जीिि स्मॉि फाइिेंस बैंि (SFB) निनमटे ि िे बारे में:
प्रबंध निदे शि - इसत्तरा डे सवस
मुख्यािय – बेंगलुरु, कनाधटक
स्थानपत – 2017

200. िंसदा राष्टरीय उद्याि निस राज्य में खस्थत है ?


1)सबहार
2)उत्तर प्रदे श
3)महाराष्ट्र
4)गुजरात
5)मध्य प्रदे श
उत्तर-4)गुिरात
स्पष्टीिरण:
िंसदा राष्टरीय उद्याि भारत के गुिरात के नवसारी सजले में क्कथित एक लुभावनी सुों दर पाकध है । यह राष्ट्रीय उद्यान 24
वगध सकलोमीटर क्षेत्र में फैला है और प्रकृसत प्रेसमयोों और वन्यजीव उत्साही लोगोों के सलए एक लोकसप्रय गोंतव्य है ।

201. संयुक्त राष्टर बाि िोष (UNICEF) िा मुख्यािय िहााँ खस्थत है ?


1)रोम, इटली
2)सजनेवा, क्कस्वट् जरलैंड
3)न्यूयॉकध, सोंयुक्त राज्य अमेररका

Report Errors in the PDF - ebooks@affairscloud.com Copyright 2014-2023 @ AffairsCloud.com 293


4)सवयना, ऑक्कस्ट्रया
5)पेररस, फ़्ाोंस
उत्तर-3)न्यूयॉिा, संयुक्त राज्य अमेररिा
स्पष्टीिरण:
संयुक्त राष्टर बाि िोष (UNICEF) िे बारे में:
िायािारी निदे शि - कैिरीन M. रसेल
मुख्यािय – न्यूयॉकध, सोंयुक्त राज्य अमेररका (USA)
स्थापिा - 1943

202. बैंि ऑफ महाराष्टर िे ितामाि (नसतंबर’23 ति) प्रबंध निदे शि (MD) और मुख्य िायािारी अनधिारी
(CEO) िौि हैं ?
1)A S राजीव
2)अतनु कुमार दास
3)चोंद्र शेखर घोष
4)अतुल कुमार गोयल
5)शसशर्र जगदीशन
उत्तर- 1)A S रािीि
स्पष्टीिरण:
बैंि ऑफ महाराष्टर (BoM) िे बारे में:
प्रबंध निदे शि (MD) और मुख्य िायािारी अनधिारी (CEO) - A S रािीि
मुख्यािय- पुिे, महाराष्ट्र
स्थापिा- 1935
टै गिाइि - एक पररवार, एक बैंक (वन फैसमली वन बैंक)

203. िेि िन्यिीि अभ्यारण्य निस राज्य में खस्थत है ?


1)सत्रपुरा
2)ससक्किम
3)समजोरम
4)पसिम बोंगाल
5)अरुिाचल प्रदे श
उत्तर-5)अरुणाचि प्रदे श
स्पष्टीिरण:
अरुणाचि प्रदे श (AR) िे बारे में:
मुख्यमंत्री– पेमा खाोंडू
राज्यपाि– लेक्कफ्टनेंट जनरल कैवल् सत्रसवक्रम परनायक
िन्यिीि अभ्यारण्य- डी'एररों ग मेमोररयल (लाली) वन्यजीव अभ्यारण्य, िेि िन्यिीि अभ्यारण्य
टाइगर ररजवध-कमलाोंग टाइगर ररजवध

Report Errors in the PDF - ebooks@affairscloud.com Copyright 2014-2023 @ AffairsCloud.com 294


204. निस एयरोस्पेस िंपिी िे हाि ही में (नसतंबर’23 में) अपिी पहिी अत्याधुनिि नगयर और नगयर बॉक्स
निनिमााण सुनिधा िा उद् घाटि निया है ?
1)गरुड एयरोस्पेस प्राइवेट सलसमटे ड
2)अरून एसवएशन ससवधसेज प्राइवेट सलसमटे ड
3)स्कोंद एयरोस्पेस टे क्नोलॉजी प्राइवेट सलसमटे ड
4)BAE ससस्ट्म्स इों सडया ससवधसेज प्राइवेट सलसमटे ड
5)स्काईरूट एयरोस्पेस प्राइवेट सलसमटे ड
उत्तर-3)स्कंद एयरोस्पेस टे क्नोिॉिी प्राइिेट निनमटे ि
स्पष्टीिरण:
हैदराबाद क्कथित स्कंद एयरोस्पेस टे क्नोिॉिी प्राइिेट निनमटे ि (SATPL), जो भारत के है दराबाद में रघु वामसी
मशीन टू ल्स और टे क्सास, USA में रे व सगयसध LLC के बीच एक सहयोगी उद्यम है , ने अपनी पहली अत्यार्ुसनक सगयर
और सगयर बॉक्स सवसनमाधि सुसवर्ा का उद् घाटन सकया।
i.यह अग्रिी प्रयास, भारत में पहला, एयरोस्पेस-ग्रेड सगयर के उत्पादन के सलए समसपधत है , जो दे श के भीतर एयरोस्पेस
सवसनमाधि को आगे बढाने में एक मामूली लेसकन आशाजनक प्रगसत का प्रतीक है ।
ii.इस अत्यार्ुसनक सुसवर्ा के उद् घाटन में जयेश रों जन, IAS, I&C और IT के प्रमुख ससचव और तेलोंगाना सरकार में
एयरोस्पेस और रक्षा सनदे शक प्रवीि P A ससहत गिमान्य लोग उपक्कथित िे।

205. संयुक्त राज्य अमेररिा (USA) में निम्ननिखित में से िौि सा शहर 2024 में अंतरााष्टरीय नक्रिेट पररषद
(ICC) पुरुष ट्वें टी 20 (T20) निश्व िप िी मेिबािी िरे गा?
1)न्यूयॉकध
2)िोररडा
3)डलास
4)1 और 2 दोनोों
5)सभी 1, 2 और 3
उत्तर- 5)सभी 1, 2 और 3
स्पष्टीिरण:
अोंतराधष्ट्रीय सक्रकेट पररषद (ICC) ने घोषिा की है सक सोंयुक्त राज्य अमेररका (USA) के तीन शहर - ििास, फ्लोररिा
और न्यूयॉिा - 2024 में पुरुष T20 निश्व िप की मेजबानी करें गे। यह आयोसजत होने वाला पहला ICC टू नाधमेंट होगा।
USA, और चुने गए थिान डलास में ग्रैंड प्रेयरी, िोररडा में ब्रोवाडध काउों टी और न्यूयॉकध में नासाउ काउों टी हैं।
i.अपने पहले ICC टू नाधमेंट की मेजबानी कर रहे अमेररका के डलास में ग्रैंड प्रेयरी, िोररडा में ब्रोवाडध काउों टी और
न्यूयॉकध में नासाउ काउों टी को सबसे छोटे प्रारूप के सवश्व कप के आयोजन थिल के रूप में चुना गया है।

206. निस सोशि मीनिया प्लेटफॉमा िे हाि ही में (नसतंबर 23 में) चैिि िामि एि िई सुनिधा िॉन्च िी है ,
िो िोगों िे निए उििे निए महत्वपूणा अपिे ट प्राप्त िरिे िा एि नििी तरीिा है ?
1)टे लीग्राम
2)यूट्यूब
3)सलोंक्डइन
4)व्हाट् सएप
5)सट्वटर

Report Errors in the PDF - ebooks@affairscloud.com Copyright 2014-2023 @ AffairsCloud.com 295


उत्तर- 4)व्हाट् सएप
स्पष्टीिरण:
भारत में लॉन्च होने के एक हफ्ते बाद, मेटा का नया फीचर व्हाट् सएप चैिि लोकसप्रयता हाससल कर रहा है। प्रर्ान
मोंत्री नरें द्र मोदी चैनलोों से जुडे, और उनके दस लाख से असर्क अनुयायी िे।
• मेटा के अनुसार, कैटरीना कैफ, अक्षय कुमार, भारतीय सक्रकेट टीम, सदलजीत दोसाोंझ और सवजय दे वरकोोंडा
अपने व्हाट् सएप चैनल लॉन्च करने वाले कुछ भारतीय हक्कस्तयोों में से हैं।
• मेटा का कहना है सक व्हाट् सएप चैनल "लोगोों के सलए व्हाट् सएप के भीतर ही उनके सलए महत्वपूिध अपडे ट
प्राप्त करने का एक सनजी तरीका है।"
• चैनल फ़ॉलोअसध का फ़ोन नोंबर और प्रोफ़ाइल छसव व्यवथिापक या अन्य फ़ॉलोअसध को नहीों सदखाई जाएगी।
सकसी चैनल पर साझा सकए गए अपडे ट केवल 30 सदनोों के सलए सहेजे जाएों गे।

207. 42िें संिैधानिि संशोधि, 1976 द्वारा भारत िे संनिधाि िी प्रस्ताििा में निम्ननिखित में से िौि सा
शब्द िोडा गया?
1)र्मधसनरपेक्ष
2)समाजवादी
3)लोकताोंसत्रक
4)1 और 2 दोनोों
5)1 और 3 दोनोों
उत्तर- 4)1 और 2 दोिों
स्पष्टीिरण:
प्रस्तावना सकसी दस्तावेज़ में एक पररचयात्मक किन है जो दस्तावेज़ के दशधन और उद्दे श्योों को समझाता है।
• 1949 में सोंसवर्ान सभा िारा अपनाई गई मूल प्रस्तावना में भारत को एक "सोंप्रभु लोकताोंसत्रक गिराज्य" घोसषत
सकया गया िा।
• 1976 के 42वें सोंवैर्ासनक सोंशोर्न िारा, "समाििादी" और "धमानिरपेक्ष" शब्द जोडे गए; प्रस्तावना में अब
"सोंप्रभु समाजवादी र्मधसनरपेक्ष लोकताोंसत्रक गिराज्य" सलखा है। यह अदालत में लागू करने योग्य नहीों है
• प्रस्तावना 13 सदसोंबर, 1946 को सोंसवर्ान सभा में जवाहरलाल नेहरू िारा पेश सकए गए उद्दे श्य सोंकल्प पर
आर्ाररत है। यह प्रस्ताव 22 जनवरी, 1947 को अपनाया गया िा।

208. भारत िे संनिधाि में संशोधि दे श िे मौनिि िािूि या सिोच्च िािूि में बदिाि िरिे िी प्रनक्रया है ,
िो भारत िे संनिधाि िे _________ (अिुच्छेद) में निधााररत है।
1)352 अनुच्छेद
2)362 अनुच्छेद
3)350 अनुच्छेद
4)360 अनुच्छेद
5)368 अनुच्छेद
उत्तर- 5)368 अिुच्छेद

Report Errors in the PDF - ebooks@affairscloud.com Copyright 2014-2023 @ AffairsCloud.com 296


स्पष्टीिरण:
भारत के सोंसवर्ान में सोंशोर्न करना दे श के मौसलक कानून या सवोच्च कानून में बदलाव करने की प्रसक्रया है। सोंसवर्ान
में सोंशोर्न की प्रसक्रया भारत के सोंसवर्ान के भाग XX (अिुच्छेद 368) में सनर्ाधररत है। यह प्रसक्रया भारत के सोंसवर्ान
की पसवत्रता सुसनसित करती है और भारत की सोंसद की मनमानी शक्कक्त पर सनयोंत्रि रखती है।
• इसे दनक्षण अफ़्ीिी संनिधाि से उर्ार सलया गया िा
• "अनुच्छेद 368 के तहत सोंसद मौसलक असर्कारोों ससहत सोंसवर्ान के सकसी भी सहस्से में सोंशोर्न कर सकती है
लेसकन सोंसवर्ान की 'बुसनयादी सोंरचना' को प्रभासवत सकए सबना।" भारत के सोंसवर्ान के भाग XX का अनुच्छेद
368 दो प्रकार के सोंशोर्नोों का प्रावर्ान करता है।
❖ सोंसद के सवशेष बहुमत से
❖ कुल राज्योों में से आर्े के अनुसमिधन के साि सोंसद के सवशेष बहुमत िारा।

209. 'माधि राष्टरीय उद्याि' निस राज्य में खस्थत है?


1)मध्य प्रदे श
2)महाराष्ट्र
3)गुजरात
4)उत्तर प्रदे श
5)सबहार
उत्तर- 1)मध्य प्रदे श
स्पष्टीिरण:
मध्य प्रदे श (MP) िे बारे में:
मुख्यमंत्री (CM)-सशवराज ससोंह चौहान
राज्यपाि – मोंगुभाई छगनभाई पटे ल
राष्टरीय उद्याि - पन्ना राष्ट्रीय उद्यान; पेंच राष्ट्रीय उद्यान, माधि राष्टरीय उद्याि
िन्यिीि अभ्यारण्य (WLS) - गोंगऊ वन्यजीव अभ्यारण्य; घाटीगाोंव वन्यजीव अभ्यारण्य

210. नामदाफा टाइगर ररिवा ____________ (राज्य) में स्कस्थत है।


1)पसिम बोंगाल
2)अरुिाचल प्रदे श
3)ओसडशा
4)समजोरम
5)उत्तर प्रदे श
उत्तर- 2)अरुणाचि प्रदे श
स्पष्टीिरण:
िामदाफा टाइगर ररििा अरुणाचि प्रदे श के चाोंगलाोंग सजले में क्कथित है ।
i.नामदाफा एक नदी का नाम है जो दाफा बम से सनकलती है और नोआ-सदसहोंग नदी में समलती है।
ii.यह नदी राष्ट्रीय उद्यान के ठीक उत्तर-दसक्षि सदशा में बहती है और इससलए इसे नामदाफा नाम सदया गया है ।

Report Errors in the PDF - ebooks@affairscloud.com Copyright 2014-2023 @ AffairsCloud.com 297


211. निस राज्य सरिार िो हाि ही में (नसतंबर’23 में) िेशिि ग्रीि नटर ब्यूिि (NGT) से भोि िेटिैंि में क्रूि
िहािों िे साथ-साथ अन्य मोटर-चानित िौिाओं िे संचािि िो रोििे िा आदे श नमिा है ?
1)असम
2)अरुिाचल प्रदे श
3)मध्य प्रदे श
4)समजोरम
5)नागालैंड
उत्तर-3)मध्य प्रदे श
स्पष्टीिरण:
हाल ही में, नेशनल ग्रीन सटर ब्यूनल (NGT) ने मध्य प्रदे श सरकार को भोज वेटलैंड में क्रूज जहाजोों के साि-साि अन्य
मोटर-चासलत नौकाओों के सोंचालन को रोकने का आदे श सदया।
• इसमें मध्य प्रदे श के भोपाल शहर में क्कथित दो ससन्नसहत मानव सनसमधत जलाशय, ऊपरी और सनचली झीलें
शासमल हैं।
• ऊपरी झील को भोजताल (बडा तालाब) और सनचली झील को छोटा तालाब कहा जाता है।
i.िेशिि ग्रीि नटर ब्यूिि की थिापना नेशनल ग्रीन सटर ब्यूनल असर्सनयम 2010 के तहत की गई है। इसमें अध्यक्ष,
न्यासयक सदस्य और सवशेषज्ञ सदस्य शासमल हैं।
• वे 5 वषध की अवसर् के सलए पद पर रहें गे और पुनसनधयुक्कक्त के सलए पात्र नहीों होोंगे।
• अध्यक्ष की सनयुक्कक्त भारत के मुख्य न्यायार्ीश (CJI) के परामशध से केंद्र सरकार िारा की जाती है।
• न्यासयक सदस्योों और सवशेषज्ञ सदस्योों की सनयुक्कक्त के सलए केंद्र सरकार िारा एक चयन ससमसत का गठन सकया
जाएगा।
• सटर ब्यूनल में कम से कम 10 और असर्कतम 20 पूिधकासलक न्यासयक सदस्य और सवशेषज्ञ सदस्य होने चासहए।

212. तुंगारे श्वर िन्यिीि अभ्यारण्य निस राज्य में खस्थत है ?


1)महाराष्ट्र
2)पसिम बोंगाल
3)झारखण्ड
4)उत्तर प्रदे श
5)सबहार
उत्तर- 1)महाराष्टर
स्पष्टीिरण:
सूयाध क्षेत्रीय जल आपूसतध पररयोजना के सहस्से के रूप में , तुंगारे श्वर िन्यिीि अभ्यारण्य को पार करने के सलए 4.6 km
लोंबी भूसमगत सुरोंग पर काम हाल ही में पूरा हुआ।
i.तुोंगारे श्वर वन्यजीव अभ्यारण्य मुोंबई से 75 km दू र महाराष्टर के पालघर सजले में क्कथित है । यह सोंजय गाों र्ी राष्ट्रीय उद्यान
और तानसा वन्यजीव अभ्यारण्य के बीच एक गसलयारा बनाता है।

213. सेंटर फॉर िे विपमेंट ऑफ टे िीमैलटक्स (C-DOT) के वतामान (लसतंबर'23 तक) मुख्य कायाकारी
अलिकारी (CEO) कौन हैं?
1)ियंत कुमार
2)रािकुमार उपाध्याय

Report Errors in the PDF - ebooks@affairscloud.com Copyright 2014-2023 @ AffairsCloud.com 298


3)आश्चतश मािुर
4)अिय कुमार
5)प्रवीि शमाग
उत्तर- 2)रािकुमार उपाध्याय
स्पष्टीकरण:
सेंटर फॉर िे विपमेंट ऑफ टे िीमैलटक्स (C-DOT) के बारे में:
मुख्य कायाकारी अलिकारी (CEO) – रािकुमार उपाध्याय
मुख्यािय – नई श्चदल्ली, श्चदल्ली
स्थापना – 1984

214. अंतरााष्टरीय मानकीकरण संगठन (ISO) का मुख्यािय कहाँ स्कस्थत है?


1)रोम, इटली
2)श्चिनेवा, स्स्वट् िरलैंि
3)न्यूयॉकग, संयुक्त राज्य अमेररका
4)श्चवयना, ऑस्स्ट्र या
5)पेररस, फ़्ांस
उत्तर-2)लिनेवा, स्कस्वट् िरिैंि
स्पष्टीकरण:
अंतरााष्टरीय मानकीकरण संगठन (ISO) के बारे में:
राष्टरपलत – उलररका फ़्ैंके
मुख्यािय - लिनेवा, स्कस्वट् िरिैंि
सदस्य - 169 राष्ट्रीय मानक श्चनकाय।
स्थापना - 1947

215. नॉवेलियन _________ नॉवे की मुद्रा है।


1)कोरुना
2)स्ट्श्चलिंग
3)क्रोन
4)श्चशश्चलंग
5)लैट्स
उत्तर-3)क्रोन
स्पष्टीकरण:
नॉवे के बारे में:
प्रिान मंत्री (PM) - िोनास गहर स्ट्ोरे
राििानी – ओस्लो
मुद्रा - नॉवेश्चियन क्रोन

Report Errors in the PDF - ebooks@affairscloud.com Copyright 2014-2023 @ AffairsCloud.com 299


216. हाि ही में (लसतंबर'23 में) लकस राज्य सरकार ने 29 िून को व्यापारी कल्याण लदवस घोलषत लकया है?
1)महाराष्ट्र
2)मध्य प्रदे श
3)आं ध्र प्रदे श
4)उत्तर प्रदे श
5)कनागटक
उत्तर-4)उत्तर प्रदे श
स्पष्टीकरण:
उत्तर प्रदे श की योगी आश्चदत्यनाि सरकार ने हर 29 िून को व्यापारी कल्याण लदवस मनाने का फैसला श्चकया है।
i.सरकार उस श्चदन प्रमुख थिानीय व्यापाररक नेताओं और उद्योगपश्चतयों को पहचानने और सम्माश्चनत करने के श्चलए हर
श्चिले में एक श्चनवेशक श्चशखर सम्मेलन भी आयोश्चित करे गी, िो श्चक महारािा प्रताप के सहयोगी और श्चिनके नैश्चतक
और श्चवत्तीय समिगन के श्चलए माना िाता है, भामाशाह की ियंती के साि मेल खाता है। सरकार द्वारा िारी एक प्रेस
श्चवज्ञस्प्त के अनुसार, रािपूत योिा रािा को अपने खोए हुए अश्चर्कां श क्षेत्र को पुनः प्राप्त करने में मदद श्चमली।

217. उस राज्य सरकार का नाम बताइए लिसने हाि ही में (लसतंबर'23 में) ओंकारे श्वर में आलद शंकराचाया की
'एकात्मता की प्रलतमा' या 'स्ट्ै च्यू ऑफ वननेस' का अनावरण लकया है।
1)मध्य प्रदे श
2)तश्चमलनािु
3)आं ध्र प्रदे श
4)केरल
5)कनागटक
उत्तर- 1)मध्य प्रदे श
स्पष्टीकरण:
मध्य प्रदे श के मुख्यमंत्री श्चशवराि श्चसंह चौहान ने ओंकारे श्वर में आलद शंकराचाया की 'एकात्मता की प्रलतमा' या
'स्ट्ै च्यू ऑफ वननेस' का अनावरि श्चकया। राज्य कैश्चबनेट ने 8वीं सदी के श्चहंदू दाशगश्चनक और संत के िीवन और दशगन
का िश्न मनाने के श्चलए श्चपछले साल इस पररयोिना को मंिूरी दी िी। 108 फीट ऊंची यह प्रश्चतमा ओंकारे श्वर में मांर्ाता
पवगत पर स्थित है।
i.नमगदा नदी के सुरम्य तट पर स्थित ओंकारे श्वर, इं दौर शहर से लगभग 80 km दू र है।

218. लसतंबर 2023 में केंद्रीय रे ि मंत्रािय द्वारा िारी सकाु िर के अनुसार, टर े न के साथ-साथ मानवयुक्त
समपार दु घाटनाओं में मृत यालत्रयों के ररश्तेदारों को ___________ रुपये लमिेंगे।
1)1 लाख
2)3 लाख
3)6 लाख
4)5 लाख
5)2 लाख
उत्तर- 4)5 िाि
स्पष्टीकरण:
केंद्रीय रे ि मंत्रािय ने मृत्यु के मामले में आश्चश्रतों को दे य अनुग्रह राहत की राश्चश को दस गुना तक संशोश्चर्त श्चकया है।

Report Errors in the PDF - ebooks@affairscloud.com Copyright 2014-2023 @ AffairsCloud.com 300


• सकुगलर के मुताश्चबक, टर े न और मानवयुक्त समपार दु घगटनाओं में मृत याश्चत्रयों के ररश्तेदारों को अब 5 िाि
रुपये श्चमलेंगे, िबश्चक गंभीर रूप से घायल लोगों को 2.5 लाख रुपये श्चदए िाएं गे। सार्ारि चोट वाले याश्चत्रयों को
50,000 रुपये श्चमलेंगे। पहले ये रकम क्रमश: 50,000 रुपये, 25,000 रुपये और 5,000 रुपये िी
• सकुगलर में आगे कहा गया है श्चक मृत, गंभीर रूप से घायल और श्चकसी अश्चप्रय घटना में सार्ारि रूप से घायल
याश्चत्रयों के आश्चश्रतों को क्रमशः 1.5 लाख रुपये, 50,000 रुपये और 5,000 रुपये श्चमलेंगे। श्चपछली अनुग्रह
योिना में यह राश्चश क्रमशः 50,000 रुपये, 25,000 रुपये और 5,000 रुपये िी।
• बोिग ने स्पष्ट् श्चकया है श्चक मानव रश्चहत क्रॉश्चसंग पर दु घगटना, अश्चतक्रश्चमयों, OHE(ओवरहेि इस्िपमेंट) द्वारा करं ट
से मारे गए व्यस्क्तयों के मामले में सड़क उपयोगकताग ओं को कोई अनुग्रह राहत स्वीकायग नहीं होगी। रे लवे
अश्चर्श्चनयम 1989 में टर े न दु घगटनाओं और अश्चप्रय घटनाओं में याश्चत्रयों की मृत्यु या चोट के श्चलए मुआविा दाश्चयत्व
श्चनर्ागररत श्चकया गया है।

219. 'महानंदा वन्यिीव अभ्यारण्य' लकस राज्य में स्कस्थत है?


1)उत्तर प्रदे श
2)पश्चिम बंगाल
3)श्चबहार
4)मध्य प्रदे श
5)महाराष्ट्र
उत्तर- 2)पलश्चम बंगाि
स्पष्टीकरण:
महानंदा वन्यिीव अभ्यारण्य (क्षेत्रफल 158 वगग श्चकमी) पलश्चम बंगाि के दाश्चिगश्चलंग श्चिले के दश्चक्षिी भाग में, श्चतस्ता
नदी के पश्चिमी तट पर स्थित है। ऊाँचाई और वनस्पश्चत की श्चवश्चवर्ता के कारि, अभ्यारण्य में पाई िाने वाली िीव-िंतु
प्रिाश्चतयााँ भी श्चवश्चवर् हैं ।

220. 'पोरबंदर पक्षी अभ्यारण्य' लकस राज्य में स्कस्थत है?


1)गुिरात
2)मध्य प्रदे श
3)श्चबहार
4)पश्चिम बंगाल
5)असम
उत्तर- 1)गुिरात
स्पष्टीकरण:
पोरबंदर पक्षी अभ्यारण्य, गुिरात के पोरबंदर शहर में एक सुंदर प्राकृश्चतक क्षेत्र है िहां लोग और प्रकृश्चत
सौहादग पूवगक रहते हैं।
i.अभ्यारण्य 1 वगग श्चकलोमीटर के क्षेत्र में फैला है और पेड़ों और पौर्ों से श्चघरा हुआ है। इसकी थिापना 1988 में वहां
रहने वाली पश्चक्षयों की कई प्रिाश्चतयों की रक्षा के श्चलए की गई िी।

221. 'लसंगािीिा राष्टरीय उद्यान' लकस राज्य में स्कस्थत है ?


1)उत्तर प्रदे श
2)असम

Report Errors in the PDF - ebooks@affairscloud.com Copyright 2014-2023 @ AffairsCloud.com 301


3)मध्य प्रदे श
4)पश्चिम बंगाल
5)श्चबहार
उत्तर-4)पलश्चम बंगाि
स्पष्टीकरण:
पलश्चम बंगाि के बारे में:
मुख्यमंत्री–ममता बनिी
राज्यपाि– िॉ. C.V. आनंद बोस
राष्टरीय उद्यान- लसंगािीिा राष्टरीय उद्यान; नेओरा वैली राष्ट्रीय उद्यान
टाइगर ररिवा- बक्सा टाइगर ररिवग; सुंदरबन टाइगर ररिवग

222. 'गांिी सागर वन्यिीव अभ्यारण्य' लकस राज्य में स्कस्थत है?
1)ओश्चिशा
2)झारखण्ड
3)पश्चिम बंगाल
4)मध्य प्रदे श
5)कनागटक
उत्तर-4)मध्य प्रदे श
स्पष्टीकरण:
गांिी सागर वन्यिीव अभ्यारण्य मध्य प्रदे श में मंदसौर और नीमच श्चिलों की उत्तरी सीमा पर स्थित है।
i.यह रािथिान राज्य से सटे 368.62 km2 (142.32 वगग मील) क्षेत्र में फैला हुआ है।
ii.चीता पररयोिना को लागू करने वाली पयाग वरि मं त्रालय की एिेंसी, राष्ट्रीय बाघ संरक्षि प्राश्चर्करि (NTCA) के प्रमुख
S.P. यादव के अनुसार, चीतों का अगला बैच दश्चक्षि अफ्रीका से आयात श्चकया िाएगा और मध्य प्रदे श के गांर्ी सागर
वन्यिीव अभ्यारण्य में लाया िाएगा।

223. किाकड मुंिनथुराई टाइगर ररिवा' __________ (राज्य) में स्कस्थत है।
1)तेलंगाना
2)केरल
3)तश्चमलनािु
4)आं ध्र प्रदे श
5)कनागटक
उत्तर-3)तलमिनािु
स्पष्टीकरण:
तलमिनािु के बारे में:
मुख्यमंत्री (CM)- MK स्ट्ाश्चलन
राज्यपाि- R.N.रश्चव
वन्यिीव अभ्यारण्य- इं श्चदरा गांर्ी वन्यिीव अभ्यारण्य; कलाकड़ वन्यिीव अभ्यारण्य
टाइगर ररिवा- किाकड मुंिनथुराई टाइगर ररिवा ; अनामलाई टाइगर ररिवग

Report Errors in the PDF - ebooks@affairscloud.com Copyright 2014-2023 @ AffairsCloud.com 302


224. लसतंबर 2023 में इं स्ट्ीट्यूट ऑफ इं टरनेशनि फाइनेंस (IIF) के आं कडों के अनुसार, वैलश्वक ऋण 2023
की दू सरी लतमाही (Q2)में __________ USD के एक नए ररकॉिा पर पहुंच गया।
1)307 श्चटरश्चलयन
2)134 श्चटरश्चलयन
3)235 श्चटरश्चलयन
4)406 श्चटरश्चलयन
5)189 श्चटरश्चलयन
उत्तर- 1)307 लटर लियन
स्पष्टीकरण:
इं स्ट्ीट्यूट ऑफ इं टरनेशनल फाइनेंस (IIF) के अनुसार, US और िापान िैसे दे शों द्वारा संचाश्चलत, वैश्चश्वक ऋि Q2
2023 में 307 लटर लियन अमेररकी िॉिर के नए ररकॉिग पर पहुंच गया।
i.IIF ररपोटग में कहा गया है श्चक िॉलर के संदभग में वैश्चश्वक ऋि H1 2023 में 10 श्चटरश्चलयन अमेररकी िॉलर और श्चपछले
दशक में 100 श्चटरश्चलयन अमेररकी िॉलर ब़ि गया है।
ii.इस वृस्ि ने वैश्चश्वक ऋि-से-GDP अनुपात को 336% तक ब़िा श्चदया है, िो लगातार दू सरी श्चतमाही वृस्ि है।

225. मोंिो िु िांलटस ने लसतंबर 2023 में यूिीन में 2023 िायमंि िीग फाइनि में ____________ बार पोि
वॉल्ट् वल्डा ररकॉिा दिा लकया।
1)चौिा
2)सातवां
3)पांचवां
4)छठा
5)आठवां
उत्तर- 2)सातवां
स्पष्टीकरण:
मोंिो िु िांलटस ने 17 श्चसतंबर 2023 को यूिीन में 2023 िायमंि लीग फाइनल में एिलेश्चटक्स सीज़न का शानदार
अंत श्चकया।
• स्वीिन के श्चवश्व और ओलंश्चपक पोल वॉल्ट् चैंश्चपयन ने पहले प्रयास में 6.23 m की दू री तय की, श्चिससे
फरवरी, 2023 में क्लेरमोंट-फेरािग में बनाए गए अपने वर्ल्ग ररकॉिग में एक सेंटीमीटर िुड़ गया।
• यह सातवी ं बार है िब उन्होंने श्चवश्व ररकॉिग तोड़ा है और दू सरी बार उन्होंने आउटिोर में ऐसा श्चकया
है, दोनों ने श्चपछले साल यूिीन में श्चवश्व चैंश्चपयनश्चशप में 6.21 m की दू री तय की िी।
• िु िांश्चटस को प्रश्चतयोश्चगता िीतने के श्चलए केवल तीन प्रयासों की आवश्यकता िी, श्चिसमें उन्होंने िोड़ी
सी परे शानी के साि 5.62 m, 5.82 m और 6.02 m की दू री तय की।

226. लसतंबर 2023 में, अकासा एयर ने अंतरराष्टरीय मागों को संचालित करने के लिए प्रालिकरण प्राप्त लकया।
अकासा एयर के वतामान (लसतंबर’तक) मुख्य कायाकारी अलिकारी (CEO) कौन हैं?
1)राहुल भाश्चटया
2)गौरव M. नेगी
3)श्चवनय दु बे
4)अिय श्चसंह

Report Errors in the PDF - ebooks@affairscloud.com Copyright 2014-2023 @ AffairsCloud.com 303


5)राकेश गंगवाल
उत्तर-3)लवनय दु बे
स्पष्टीकरण:
श्चवनय दु बे अकासा एयर के वतगमान मुख्य कायगकारी अश्चर्कारी (CEO) हैं।
i.कम लागत वाली भारतीय एयरलाइन अकासा एयर ने अंतरराष्टरीय मागों पर पररचालन के श्चलए प्राश्चर्करि हाश्चसल
कर श्चलया है।
ii.एयरलाइन की योिना अं तरराष्ट्रीय उड़ानें शुरू करने की है , िो मुख्य रूप से मध्य पूवी गंतव्यों को लश्चक्षत करे गी,
संभवतः श्चदसंबर में शुरू होगी।
iii.अगले चरि में सरकार द्वारा यातायात अश्चर्कारों का आवंटन शाश्चमल है, श्चिसके बाद संबंश्चर्त दे शों से अनुमोदन
श्चलया िाता है। ये यातायात अश्चर्कार सरकारों और उनकी नाश्चमत एयरलाइनों के बीच पारस्पररक आर्ार पर श्चदए िाते
हैं।
iv. श्चवशेष रूप से, भारत और मध्य पूवग के केंद्रों िैसे दु बई और दोहा के बीच थिाश्चपत मागों का पहले से ही पूरी तरह से
उपयोग श्चकया िा रहा है और एयरलाइं स आवंश्चटत मागों से अश्चर्क उड़ानें संचाश्चलत नहीं कर सकती हैं।

227. लसतंबर 2023 तक, भारत ने नेशनि इिेस्कक्टर क बस प्रोग्राम (NEBP) शुरू लकया और चरणबि तरीके से
________ (वषा) तक दे श भर में 50,000 नई ई-बसें तैनात की ं।
1)2030
2)2026
3)2028
4)2029
5)2027
उत्तर- 5)2027
स्पष्टीकरण:
भारत और संयुक्त राज्य ने भारतीय सड़कों पर 10000 इलेस्रर क बसें लॉन्च करने के श्चलए हाि श्चमलाया है ।
i.यह साझेदारी पूरे भारत में 10,000 इलेस्रर क बसों के बेड़े के श्चलए श्चवत्तपोषि िुटाएगी, भारत में इलेस्रर क
सावगिश्चनक पररवहन के श्चवकल्पों का श्चवस्तार करे गी, स्वि शहरों और स्वथि समुदायों का श्चनमागि करे गी।
ii.भारत ने नेशनल इलेस्रर क बस प्रोग्राम (NEBP) शु रू श्चकया और 2027 तक चरिबि तरीके से दे श भर में
50,000 नई ई-बसें तैनात करने का लक्ष्य रखा है।

228. लसतंबर 2023 में, प्रिान मंत्री नरें द्र मोदी ने प्रस्ताव लदया लक पुराने संसद भवन को ‘संलविान सदन’के
रूप में िाना िाएगा।
पुराना संसद भवन लकसके द्वारा लिज़ाइन लकया गया था?
1)हबगटग बेकर
2)एिश्चवन लुश्चटयंस
3)लॉिग इरश्चवन
4)1 & 2 दोनों
5)2 & 3 दोनों
उत्तर- 4)1 & 2 दोनों

Report Errors in the PDF - ebooks@affairscloud.com Copyright 2014-2023 @ AffairsCloud.com 304


स्पष्टीकरण:
पुराने संसद भवन को अब "संलविान सदन" के नाम से िाना िाएगा। लोकसभा अध्यक्ष और राज्यसभा सभापश्चत
दोनों ने नए संसद भवन, श्चिसे "भारत का संसद भवन" का नाम भी श्चदया गया है, में सदन की कायगवाही शुरू होने से
ठीक पहले प्रर्ान मंत्री नरें द्र मोदी द्वारा श्चदए गए प्रस्ताव को स्वीकार कर श्चलया।
i.पुराने संसद भवन (नया संश्चवर्ान सदन) को श्चिश्चटश आश्चकगटे र एिलवन िुलटयंस और हबाटा बेकर द्वारा श्चििाइन
श्चकया गया िा और इसका श्चनमागि 1921 और 1927 के बीच श्चकया गया िा।

229. लवश्व स्वास्थ्य संगठन (WHO) की स्थापना कब हुई थी?


1)1977
2)1948
3)1966
4)1978
5)1945
उत्तर- 2)1948
स्पष्टीकरण:
लवश्व स्वास्थ्य संगठन (WHO) के बारे में:
महालनदे शक– िॉ. टे िरोस एिनोम घेिेयेसस
मुख्यािय– श्चिनेवा, स्स्वट् िरलैंि
स्थापना- 1948

230. राष्टरीय लचलकत्सा आयोग (NMC) के वतामान (लसतंबर’23 तक) अध्यक्ष कौन हैं?
1)रिदीप गुलेररया
2)िगत राम
3)रािीव गगग
4)श्चनस्खल टं िन
5)सुरेश चंद्र शमाग
उत्तर-5)सुरेश चंद्र शमाा
स्पष्टीकरण:
राष्टरीय लचलकत्सा आयोग (NMC) के बारे में:
अध्यक्ष- िॉ. सुरेश चंद्र शमाा
मुख्यािय- नई श्चदल्ली, श्चदल्ली
NMC का गठन संसद के एक अश्चर्श्चनयम द्वारा श्चकया गया है श्चिसे राष्ट्रीय श्चचश्चकत्सा आयोग अश्चर्श्चनयम, 2019 के रूप में
िाना िाता है िो 25 श्चसतंबर, 2020 को लागू हुआ।

231. लवश्व सरकार लशिर सम्मेिन (WGS) संगठन का मुख्यािय कहाँ स्कस्थत है?
1)दु बई, संयुक्त अरब अमीरात
2)श्चिनेवा, स्स्वट् िरलैंि
3)श्चवयना, ऑस्स्ट्र या
4)हेग, नीदरलैंि

Report Errors in the PDF - ebooks@affairscloud.com Copyright 2014-2023 @ AffairsCloud.com 305


5)लंदन, यूनाइटे ि श्चकंगिम
उत्तर-1)दु बई, संयुक्त अरब अमीरात
स्पष्टीकरण:
लवश्व सरकार लशिर सम्मेिन (WGS) संगठन के बारे में:
यह एक वैश्चश्वक, तटथि, गैर-लाभकारी संगठन है िो सरकारों के भश्चवष्य को आकार दे ने के श्चलए समश्चपगत है।
अध्यक्ष- मोहम्मद अब्दु ल्ला अल गेरगावी
मुख्यािय- दु बई, संयुक्त अरब अमीरात (UAE)
स्थापना- 2013

232. बेथुदाहारी वन्यिीव अभ्यारण्य लकस राज्य में स्कस्थत है?


1)महाराष्ट्र
2)कनागटक
3)पश्चिम बंगाल
4)तेलंगाना
5)ओश्चिशा
उत्तर-3)पलश्चम बंगाि
स्पष्टीकरण:
पलश्चम बंगाि के बारे में:
मुख्यमंत्री - ममता बनिी
राज्यपाि - C. V. आनंद बोस
वन्यिीव अभ्यारण्य - बेथुदाहारी वन्यिीव अभ्यारण्य, बल्लावपुर वन्यिीव अभ्यारण्य
टाइगर ररिवा - बक्सा टाइगर ररिवग, सुंदरबन टाइगर ररिवग

233. बांिवगढ टाइगर ररिवा _____ (राज्य) में स्कस्थत है|


1)मध्य प्रदे श
2)उत्तर प्रदे श
3)श्चबहार
4)झारखण्ड
5)पश्चिम बंगाल
उत्तर- 1)मध्य प्रदे श
स्पष्टीकरण:
मध्य प्रदे श के बारे में:
मुख्यमंत्री-श्चशवराि श्चसंह चौहान
राज्यपाि- मंगूभाई C. पटे ल
टाइगर ररिवा- पन्ना टाइगर ररिवग, बांिवगढ टाइगर ररिवा

234. पविगढ कंिवेशन ररिवा लकस राज्य में स्कस्थत है?


1)पश्चिम बंगाल
2)उत्तराखंि

Report Errors in the PDF - ebooks@affairscloud.com Copyright 2014-2023 @ AffairsCloud.com 306


3)झारखण्ड
4)उत्तर प्रदे श
5)ओश्चिशा
उत्तर- 2)उत्तरािंि
स्पष्टीकरण:
भारत के उत्तराखंि में श्चहमालय की तलहटी में बसा पविगढ कंिवेशन ररिवा, क्षेत्र की श्चवश्चवर् िैव श्चवश्चवर्ता की
सुरक्षा के श्चलए समश्चपगत एक संरश्चक्षत क्षेत्र के रूप में कायग करता है।
i.यह ररज़वग बाघ, तेंदुए, हािी और गैंिे सश्चहत श्चवश्चभन्न प्रकार की लुप्तप्राय प्रिाश्चतयों का घर है। इन शानदार िानवरों को
ररिवग के भीतर सांत्वना श्चमलती है, श्चिससे उन्हें पनपने और क्षेत्र के नािुक पाररस्थिश्चतक संतुलन में योगदान करने की
अनुमश्चत श्चमलती है।

235. ‘रं गनालथटु पक्षी अभ्यारण्य’ लकस राज्य में स्कस्थत है?
1)कनागटक
2)तश्चमलनािु
3)तेलंगाना
4)केरल
5)आं ध्र प्रदे श
उत्तर- 1)कनााटक
स्पष्टीकरण:
रं गनालथटु पक्षी अभ्यारण्य कनााटक में स्थित है। यह एक छोटा लेश्चकन महत्वपूिग पक्षी अभ्यारण्य है िो कई थिाश्चनक
प्रिाश्चतयों सश्चहत श्चवश्चभन्न प्रकार की पक्षी प्रिाश्चतयों का समिगन करता है।
i.अगस्त 2023 में, कनागटक में कृष्णा रािा सागर बांर् (KRS) से भारी पानी छोड़े िाने के बाद बा़ि आने के बाद
अभ्यारण्य को आगंतुकों के श्चलए श्चफर से खोल श्चदया गया।

236. उस व्यस्कक्तत्व का नाम बताइए लिसने हाि ही में (लसतंबर’23 में) MP के मुख्यमंत्री लशवराि लसंह चौहान
से मध्य प्रदे श (MP) एकिव्य पुरिार िीता है।
1)माया श्चपल्लई
2)रामानन्द शमाग
3)पलक शमाग
4)श्चसिािग परदे शी
5)श्चलनेश ियन्त
उत्तर-3)पिक शमाा
स्पष्टीकरण:
मध्य प्रदे श के इं दौर की रहने वाली एक उल्लेिनीय युवा गोतािोर पिक शमाा ने महि 16 साल की उम्र में मध्य
प्रदे श एकलव्य पुरस्कार िीतकर खेल की दु श्चनया में तहलका मचा श्चदया है। यह पुरस्कार 16 श्चसतंबर, 2023 को भोपाल
के तात्या टोपे स्ट्े श्चियम में आयोश्चित एक समारोह में मध्य प्रदे श के मुख्यमंत्री श्चशवराि श्चसंह चौहान द्वारा प्रदान श्चकया
गया।
• मध्य प्रदे श एकलव्य पुरस्कार पलक शमाग की गोताखोरी में असार्ारि उपलस्ियों का प्रमाि है, यह
खेल उन्होंने 8 साल की उम्र में शुरू श्चकया िा।

Report Errors in the PDF - ebooks@affairscloud.com Copyright 2014-2023 @ AffairsCloud.com 307


• अपनी कम उम्र के बाविूद, पलक शमाग ने एश्चशयन एि ग्रुप चैंश्चपयनश्चशप 2019 में भारतीय
गोताखोरी टीम का प्रश्चतश्चनश्चर्त्व श्चकया, िहां उन्होंने फाइनल में एक स्विग और दो रित पदक िीते।
• पलक की उल्लेखनीय उपलस्ियााँ राष्ट्रीय स्तर तक भी फैली हुई हैं , श्चिनमें: सब-िूश्चनयर नेशनल में 2
स्विग और 1 रित, िूश्चनयर नेशनल में 9 स्विग शाश्चमल हैं।

237. लसतंबर 2023 तक उत्तर अटिांलटक संलि संगठन (NATO) में लकतने सदस्य दे श हैं?
1)20
2)15
3)28
4)31
5)25
उत्तर- 4)31
स्पष्टीकरण:
उत्तर अटिांलटक संलि संगठन (NATO) के बारे में:
गठन- 1949
महासलचव- िेन्स स्ट्ोलटे नबगग
मुख्यािय- िुसेल्स, बेस्ल्जयम
सदस्य राज्य- 31 (वतगमान में)

238. भारतीय िघु उद्योग नििास बैंि (SIDBI) िे ितामाि (नसतंबर'23 ति) अध्यक्ष और प्रबंध निदे शि
(CMD) िौि हैं ?
1)सशवसुब्रमण्यन रमन
2)दे वेन्द्र कुमार ससोंह
3)आशीष गुप्ता
4)पोंकज जैन
5)सुदत्त मोंडल
उत्तर- 1)सशवसुब्रमण्यन रमन
स्पष्ट्ीकरि:
भारतीय िघु उद्योग नििास बैंि (SIDBI) िे बारे में:
इसकी थिापना 1990 में सोंसद के एक असर्सनयम के तहत की गई िी।
अध्यक्ष & प्रबंध निदे शि– नशिसुिमण्यि रमि
मुख्यािय– लखनऊ, उत्तर प्रदे श

239. निश्व बैंि िी स्थापिा निस िषा हुई थी?


1)1919
2)1969
3)1961
4)1944
5)1948

Report Errors in the PDF - ebooks@affairscloud.com Copyright 2014-2023 @ AffairsCloud.com 308


उत्तर- 4)1944
स्पष्टीिरण:
निश्व बैंि िे बारे में :
स्थापिा - जुलाई, 1944
मुख्यािय – वासशोंगटन DC, सोंयुक्त राज्य अमेररका (USA)
अध्यक्ष – अजय बोंगा

240. उत्कषा िघु नित्त बैंि िे ितामाि (नसतंबर'23 ति) प्रबंध निदे शि और मुख्य िायािारी अनधिारी
(CEO) िौि हैं ?
1)राजीव यादव
2)सोंजय अग्रवाल
3)गोसवोंद ससोंह
4)पॉल िॉमस
5)P N वासुदेवन
उत्तर-3)गोनिंद नसंह
स्पष्टीिरण:
उत्कषा स्मॉि फाइिेंस बैंि निनमटे ि (USFBL) िे बारे में:
प्रबंध निदे शि और मुख्य िायािारी अनधिारी (CEO) – गोसवोंद ससोंह
मुख्यािय – वारािसी, उत्तर प्रदे श
में शानमि -2016

241. उस अंतरााष्टरीय सूचिा प्रौद्योनगिी संस्थाि (IIIT) िा िाम बताइए निसे हाि ही में (नसतंबर'23 में) ग्रीि
यूनििनसाटी अिािा 2023 से सम्मानित निया गया है।
1)IIIT सदल्ली
2)IIIT बैंगलोर
3)IIIT हैदराबाद
4)IIIT पुिे
5)IIIT भुवनेश्वर
उत्तर -2)IIIT बैंगिोर
स्पष्टीिरण:
अोंतराधष्ट्रीय सूचना प्रौद्योसगकी सोंथिान बैंगलोर (IIIT-बैंगिोर) को न्यूयॉकध में आयोसजत सातवें NYC ग्रीन स्कूल सम्मेलन
के दौरान 'ग्रीि यूनििनसाटी अिािा 2023' से सम्मासनत सकया गया है।
i.यह पुरस्कार पूरे IIIT -बैंगलोर पररसर समुदाय की ओर से गवसनिंग बॉडी के सदस्य और IIIT -बैंगलोर के पूवध छात्र श्री
उदय हेगडे ने प्राप्त सकया।

242. सुिाबेडा िन्यिीि अभ्यारण्य निस राज्य में खस्थत है ?


1)ओसडशा
2)पसिम बोंगाल
3)झारखण्ड

Report Errors in the PDF - ebooks@affairscloud.com Copyright 2014-2023 @ AffairsCloud.com 309


4)मध्य प्रदे श
5)महाराष्ट्र
उत्तर- 1)ओनिशा
स्पष्टीिरण:
ओनिशा िे बारे में:
मुख्यमंत्री- नवीन पटनायक
राज्यपाि – गिेशी लाल
िन्यिीि अभ्यारण्य - डे ब्रीगढ वन्यजीव अभ्यारण्य, हदागढ वन्यजीव अभ्यारण्य, सुिाबेडा िन्यिीि अभ्यारण्य

243. उदं ती सीतािदी बाघ अभ्यारण्य निस राज्य में खस्थत है ?


1)ओसडशा
2)महाराष्ट्र
3)मध्य प्रदे श
4)पसिम बोंगाल
5)छत्तीसगढ
उत्तर-5)छत्तीसगढ़
स्पष्टीिरण:
उदं ती सीतािदी बाघ अभ्यारण्य छत्तीसगढ़ में क्कथित है।
i.वषध 2008-09 में इसे बाघ अभ्यारण्य घोसषत सकया गया िा। उदों ती और सीतानदी नसदयााँ ररजवध से होकर बहती हैं।
यहाों कुछ बारहमासी झरने हैं सजनमें प्रससद्ध दे वर्रा और गोडे ने झरने भी शासमल हैं । एसशयाई जोंगली भैंस कोर क्षेत्र में
पाई जाने वाली प्रमुख लुप्तप्राय प्रजासत है।

244. अंतरााष्टरीय समुद्री संगठि (IMO) िा मुख्यािय िहााँ खस्थत है ?


1)रोम, इटली
2)पेररस, फ़्ाोंस
3)सजनेवा, क्कस्वट् जरलैंड
4)लोंदन, यूनाइटे ड सकोंगडम
5)न्यूयॉकध शहर, सोंयुक्त राज्य अमेररका
उत्तर-4)िंदि, यूिाइटे ि निंगिम
स्पष्टीिरण:
अंतरााष्टरीय समुद्री संगठि (IMO) िे बारे में:
महासनचि- सकटै क सलम
मुख्यािय- िंदि, यूिाइटे ि निंगिम
स्थापिा- 1958

245. 'थट्टे िड पक्षी अभ्यारण्य' निस राज्य में खस्थत है ?


1)केरल
2)ओसडशा
3)कनाधटक

Report Errors in the PDF - ebooks@affairscloud.com Copyright 2014-2023 @ AffairsCloud.com 310


4)तेलोंगाना
5)आों ध्र प्रदे श
उत्तर- 1)िेरि
स्पष्टीिरण:
िेरि िे बारे में:
मुख्यमंत्री– सपनाराई सवजयन
राज्यपाि– आररफ मोहम्मद खान
पक्षी अभ्यारण्य- मोंगलवनम पक्षी अभ्यारण्य; थट्टे िड पक्षी अभ्यारण्य
हिाई अड्डा– कालीकट अोंतराधष्ट्रीय हवाई अड्डा; कन्नूर अोंतराधष्ट्रीय हवाई अड्डा

246. 'िुद्रे मुि राष्टरीय उद्याि' निस राज्य में खस्थत है ?


1)आों ध्र प्रदे श
2)कनाधटक
3)झारखण्ड
4)पसिम बोंगाल
5)ओसडशा
उत्तर- 2)ििााटि
स्पष्टीिरण:
िुद्रे मुि राष्टरीय उद्याि ििााटि के सचिमगलुरु और दसक्षि कन्नड सजलोों में क्कथित है।
i.इसमें तेंदुआ, मालाबार सवशाल सगलहरी, स्लॉि भालू, गौर, साोंभर, ससयार, नेवला, बाघ, जोंगली कुत्ता, आम लोंगूर, साही,
सचत्तीदार सहरि, भौोंकने वाले सहरि और सवशाल उडने वाली सगलहरी जैसे सवसभन्न प्रकार के वन्यजीव हैं।

247. निस अंतररक्ष एिेंसी िा ओनसररस-रे क्स अंतररक्ष याि िा िमूिा िैप्सूि नसतंबर 2023 में यूटा
रे नगस्ताि पर उतरा?
1)चीन राष्ट्रीय अोंतररक्ष प्रशासन
2)यूरोपीय अोंतररक्ष एजेंसी
3)स्पेसएक्स
4)राष्ट्रीय वैमासनकी एवों अोंतररक्ष प्रशासन
5)कनाडाई अोंतररक्ष एजेंसी
उत्तर-4)राष्टरीय िैमानििी एिं अंतररक्ष प्रशासि
स्पष्टीिरण:
राष्ट्रीय वैमासनकी एवों अोंतररक्ष प्रशासन (NASA) के पहले क्षुद्रग्रह नमूने सात साल की यात्रा के सलए 24 ससतोंबर को गहरे
अोंतररक्ष से पैराशूट के जररए यूटा रे सगस्तान में लाए गए िे।
i.पृथ्वी के एक चिर में, ओससररस-रे क्स अोंतररक्ष यान ने 100,000 km(63,000 मील) दू र से नमूना कैप्सूल छोडा।
छोटा कैप्सूल चार घोंटे बाद सैन्य भूसम के सुदूर सवस्तार पर उतरा, क्ोोंसक मदरसशप एक अन्य क्षुद्रग्रह के बाद रवाना हुई
िी।
ii.वैज्ञासनकोों का अनुमान है सक कैप्सूल में बेन्नू नामक काबधन-समृद्ध क्षुद्रग्रह से कम से कम एक कप मलबा रखा हुआ
है, लेसकन कोंटे नर खोले जाने तक सनसित रूप से पता नहीों चलेगा। तीन साल पहले सोंग्रहि के दौरान जब अोंतररक्ष यान
बहुत असर्क ऊपर चला गया और चट्टानोों ने कोंटे नर के ढिन को जाम कर सदया तो कुछ छलक गए और तैरने लगे।

Report Errors in the PDF - ebooks@affairscloud.com Copyright 2014-2023 @ AffairsCloud.com 311


iii. ओससररस-रे क्स, मदरसशप, 2016 में 1 सबसलयन डॉलर के समशन पर रवाना हुई। यह दो साल बाद बेन्नू तक पहुोंची
और एक लोंबी छडी वैक्ूम का उपयोग करके, 2020 में छोटे गोलाकार अोंतररक्ष चट्टान से मलबे को पकड सलया। जब
तक यह वापस लौटा, अोंतररक्ष यान 6.2 सबसलयन km (4 सबसलयन मील) लॉग इन सकया िा।

248. अगस्त 2023 में, िॉबेट टाइगर ररििा में पहिा आनिाि निनिधता सिेक्षण __________ (राज्य) में
आयोनित निया गया था।
1)ओसडशा
2)असम
3)ससक्किम
4)उत्तराखोंड
5)उत्तर प्रदे श
उत्तर-4)उत्तरािंि
स्पष्टीिरण:
िॉबेट टाइगर ररििा में पहली बार आसकधड सवसवर्ता सवेक्षि में रों गीन ऑसकधड के एक समृद्ध खजाने का पता चला।
• यह सवेक्षि 2020 से अगस्त 2023 तक कॉबेट टाइगर ररजवध में उत्तरािंि वन के अनुसोंर्ान सवोंग िारा सकया
गया िा।
• इस पररयोजना को 2020 में HoFF, उत्तराखोंड की अध्यक्षता में अनुसोंर्ान सलाहकार ससमसत (RAC) िारा
अनुमोसदत सकया गया िा।
• सवेक्षि से पता चला है सक कॉबेट टाइगर ररजवध में समृद्ध आसकधड सवसवर्ता है। इसने 31 आसकधड प्रजासतयोों के
अक्कस्तत्व को दशाध या है, जो ऑसकधडे सी पररवार के भीतर 23 प्रजासतयोों का प्रसतसनसर्त्व करते हैं । इन प्रजासतयोों में
से, 11 थिलीय हैं , दो मृतोपजीवी हैं , और 18 एसपफाइसटक हैं।

249. िेंद्रीय िागररि उड्डयि मंत्री, ज्योनतरानदत्य नसंनधया िे हाि ही में (नसतंबर'23 में) निस राज्य में तेिू
हिाई अड्डे पर िए टनमािि भिि िा उद् घाटि निया है ?
1)पसिम बोंगाल
2)ससक्किम
3)मेघालय
4)समजोरम
5)अरुिाचल प्रदे श
उत्तर-5)अरुणाचि प्रदे श
स्पष्टीिरण:
केंद्रीय नागररक उड्डयन मोंत्री, ज्योसतरासदत्य ससोंसर्या ने अरुिाचल प्रदे श के तेिू हिाई अड्डे पर नए टसमधनल भवन का
उद् घाटन सकया।
i.इस क्षेत्र के फल जमधनी, लोंदन और ससोंगापुर तक पहुोंच रहे हैं। उन्ोोंने बताया सक तेजू टसमधनल भवन का सनमाध ि 212
एकड भूसम पर 170 करोड रुपये की लागत से सकया गया है .
ii.इसका रनवे 1,500 मीटर है और यह ATR-72 प्रकार के सवमानोों को सोंभालने के सलए पूरी तरह सुसक्कित है । तेजू के
अब हवाई मानसचत्र पर आने के साि, राज्य में कुल चार हवाई अड्डे हैं , सजनके नाम तेजू, जीरो, होलोोंगी और पासीघाट
हैं। तेजू वतधमान में इों फाल, सडब्रूगढ और गुवाहाटी से जुडा हुआ है।

Report Errors in the PDF - ebooks@affairscloud.com Copyright 2014-2023 @ AffairsCloud.com 312


250. नसतंबर 2023 में, मनहंद्रा फाइिेंस िे मनहंद्रा इं श्योरें स िोिसा निनमटे ि (MIBL) में 206.39 िरोड रुपये
में ___________ प्रनतशत नहस्सेदारी हानसि िर इसे िंपिी िी पूणा स्वानमत्व िािी सहायि िंपिी बिा नदया।
1)45
2)30
3)50
4)20
5)40
उत्तर- 4)20
स्पष्टीिरण:
मनहंद्रा फाइिेंस ने 206.39 िरोड रुपये में मसहोंद्रा इों श्योरें स ब्रोकसध सलसमटे ड (MIBL) में 20 प्रनतशत नहस्सेदारी
का असर्ग्रहि कर इसे कोंपनी की पूिध स्वासमत्व वाली सहायक कोंपनी बना सदया।
i.मसहोंद्रा फाइनेंस ने शेयर बाजारोों को भेजी सूचना में कहा सक भारतीय बीमा सनयामक एवों सवकास प्रासर्करि (IRDAI)
से मोंजूरी समलने के बाद कोंपनी ने 1,001 रुपये प्रसत शेयर की दर से MIBL के 10 रुपये मूल् के 20,61,856 इक्कक्वटी
शेयरोों का असर्ग्रहि पूरा कर सलया है।
ii.नतीजतन, 22 ससतोंबर, 2023 से MIBL कोंपनी की पूिध स्वासमत्व वाली सहायक कोंपनी बन गई है।

251. इं नियि ओिरसीि बैंि िे ितामाि (नसतंबर'23 ति) प्रबंध निदे शि (MD) और मुख्य िायािारी
अनधिारी (CEO) िौि हैं?
1)श्याम श्रीसनवासन
2)प्रशाोंत कुमार
3)सोंदीप बख्शी
4)सुमोंत कठपासलया
5)अजय कुमार श्रीवास्तव
उत्तर-5)अिय िुमार श्रीिास्ति
स्पष्टीिरण:
इं नियि ओिरसीि बैंि िे बारे में:
स्थापिा- 10 फरवरी 1937
प्रबंध निदे शि (MD) और मुख्य िायािारी अनधिारी (CEO)- अजय कुमार श्रीवास्तव
मुख्यािय- चेन्नई, तसमलनाडु

252. िाखम्बया िी रािधािी क्ा है ?


1)माजुरो
2)होसनयारा
3)लुसाका
4)कोंपाला
5)पासलसकर
उत्तर-3)िुसािा

Report Errors in the PDF - ebooks@affairscloud.com Copyright 2014-2023 @ AffairsCloud.com 313


स्पष्टीिरण:
िाखम्बया िे बारे में:
रािधािी– िुसािा
राष्टरपनत– हाकैंडे सहसचलेमा
मुद्रा– जाक्कम्बयन क्वाचा/न्क्गवी

253. िूिॉनििि सिे ऑफ इं निया (ZSI) िे ितामाि (नसतंबर'23 ति) निदे शि िौि हैं ?
1)A सुब्रमण्यम
2)रघुनािन
3)गोपीनािन महेश्वरन
4)कोसससगन ससोंह
5)र्ृसत बनजी
उत्तर-5)धृनत बििी
स्पष्टीिरण:
भारतीय प्राणी सिेक्षण (ZSI) िे बारे में:
ZSI पयाधवरि, वन और जलवायु पररवतधन मोंत्रालय (MoEFCC) के तहत एक वैर्ासनक सलाहकार सनकाय है।
निदे शि– डॉ. धृनत बििी
मुख्यािय– कोलकाता, पसिम बोंगाल
स्थापिा- 1916

254. इं टरिेशिि फामाास्युनटिि फेिरे शि (FIP) िा मुख्यािय िहााँ खस्थत है ?


1)दु बई, सोंयुक्त अरब अमीरात
2)सजनेवा, क्कस्वट् जरलैंड
3)सवयना, ऑक्कस्ट्रया
4)हेग, नीदरलैंड
5)लोंदन, यूनाइटे ड सकोंगडम
उत्तर-4)हेग, िीदरिैंि
स्पष्टीिरण:
इं टरिेशिि फामाास्युनटिि फेिरे शि (FIP) िे बारे में:
अध्यक्ष (िायािाहि)- पॉल ससोंक्लेयर (ऑस्ट्र े सलया)
मुख्यािय- द हेग, िीदरिैंि्स

255. नसतंबर 2023 में, पहिा C-295 पररिहि निमाि ____________ (राज्य/UT) िे नहंिि िायु सेिा स्ट्े शि पर
भारतीय िायु सेिा (IAF) में शानमि निया गया था।
1)लद्दाख
2)पोंजाब
3)असम
4)उत्तर प्रदे श
5)जम्मू और कश्मीर

Report Errors in the PDF - ebooks@affairscloud.com Copyright 2014-2023 @ AffairsCloud.com 314


उत्तर-4)उत्तर प्रदे श
स्पष्टीिरण:
भारतीय वायु सेना (IAF) ने स्पेन के सेसवले में एयरबस सुसवर्ा में सवमान की सडलीवरी प्राप्त करने के कुछ सदनोों बाद
सोमवार को उत्तर प्रदे श के सहोंडन वायु सेना स्ट्े शन पर एक औपचाररक समारोह में पहिा C-295 पररिहि निमाि
शासमल सकया।
• सवमान से सुसक्कित पहला IAF िाडर न, नोंबर 11 िाडर न, सजसे द राइनोस भी कहा जाता है , वडोदरा में क्कथित
है।
• यह HS-748 एवरो सवमान की जगह लेगा। C-295 के शासमल होने से भारतीय वायुसेना की मध्यम सलफ्ट
सामररक क्षमता में वृक्कद्ध होगी
• ऑडध र पर 56 सवमानोों में से पहले 16 C295 को सेसवले में सैन पािो सुर साइट पर असेंबल सकया जाएगा,
दू सरे सवमान की सडलीवरी मई 2024 में की जाएगी और अगले 14 को अगस्त 2025 तक प्रसत माह एक की दर
से तैयार सकया जाएगा।
• IAF ऑडध र के शेष 40 C295 का सनमाधि और सोंयोजन - टाटा एडवाोंथड ससस्ट्म्स सलसमटे ड (TASL) के साि
साझेदारी में - गुजरात के वडोदरा में एक फाइनल असेंबली लाइन (FAL) में सकया जाएगा।

256. निस एयरिाइि िंपिी िे हाि ही में (नसतंबर'23 में) 21 मागों पर प्रनतनदि 100 से अनधि उडािें
संचानित िरिे िे निए AIX ििेर िे साथ एि िोिशेयर समझौता निया है ?
1)इों सडगो
2)सवस्तारा
3)स्पाइसजेट
4)एयर इों सडया
5)एलायोंस एयर
उत्तर-4)एयर इं निया
स्पष्टीिरण:
एयर इं निया ने AIX कनेर (सजसे पहले एयर एसशया इों सडया के नाम से जाना जाता िा) के साि एक कोडशेयर
समझौता सकया है। एयर इों सडया 21 मागों पर AIX कनेर िारा सोंचासलत प्रसतसदन 100 से असर्क उडानोों में अपना 'AI'
सडज़ाइनर कोड जोडे गी।
i.कोडशेयर समझौते के तहत और असर्क मागध उत्तरोत्तर जोडे जाएों गे। 27 ससतोंबर, 2023 से शुरू होने वाली यात्रा के
सलए सबक्री केंद्रोों पर कोडशेयर उडानोों की बुसकोंग खोली जा रही है।

257. 8 अरू बर, 2023 िो निस दे श िो अपिा सबसे बडा नहं दू मंनदर, BAPS स्वामीिारायण अक्षरधाम
नमििे िािा है ?
1)रूस
2)कनाडा
3)सोंयुक्त राज्य अमेररका
4)फ्राोंस
5)जमधनी
उत्तर-3)संयुक्त राज्य अमेररिा

Report Errors in the PDF - ebooks@affairscloud.com Copyright 2014-2023 @ AffairsCloud.com 315


स्पष्टीिरण:
संयुक्त राज्य अगले महीने 8 अरू बर, 2023 को अपना सबसे बडा सहों दू मोंसदर बनाने के सलए पूरी तरह तैयार है।
BAPS स्वामीिारायण अक्षरधाम न्यू जसी में टाइम्स िायर से 90 km दसक्षि में क्कथित है। 183 एकड के मोंसदर को
बनने में लगभग 12 साल लगे और इसके सनमाधि में पूरे अमेररका से 12,500 से असर्क स्वयोंसेवक शासमल हुए हैं।
i.ररपोटों के अनुसार, मोंसदर में एक मुख्य मोंसदर, 12 उप-मोंसदर, नौ सशखर (सशखर जैसी सोंरचनाएों ) और नौ सपरासमडनुमा
सशखर हैं। इसमें पारों पररक पत्थर वास्तुकला का सबसे बडा अण्डाकार गुोंबद भी है ।
ii. यह राजसी भव्य मोंसदर 19वीों सदी के सहोंदू आध्याक्कत्मक नेता भगवान स्वामीनारायि को समसपधत है , और यह उनके
5वें आध्याक्कत्मक उत्तरासर्कारी और प्रससद्ध सोंत प्रमुख स्वामी महाराज से प्रेररत िा।
iii. अक्षरर्ाम का सवसर्वत उद् घाटन महोंत स्वामी महाराज ने सकया. यह 18 अरू बर से आगोंतुकोों के सलए खुला रहेगा।
यह मोंसदर कोंबोसडया में अोंगकोर वाट के बाद सोंभवतः दू सरा सबसे बडा मोंसदर है , जो 500 एकड में फैला हुआ
UNESCO सवश्व र्रोहर थिल है। नई सदल्ली का अक्षरर्ाम मोंसदर 100 एकड में फैला हुआ है।

258. िैरे नबयि प्रीनमयर िीग (CPL) 2023 िे 2023 संस्करण िी चैंनपयिनशप निस टीम िे िीती?
1)सेंट सकट् स और नेसवस पैसटर यट् स
2)सेंट लूससया सकोंग्स
3)जमैका तल्लावाह
4)सटर नबागो नाइट राइडसध
5)गुयाना अमेज़न वॉररयसध
उत्तर-5)गुयािा अमेज़ि िॉररयसा
स्पष्टीिरण:
गुयाना के प्रोसवडें स में नटर िबागो िाइट राइिसा पर नौ सवकेट से जीत के साि गुयािा अमेज़़ॅि िॉररयसा कैरे सबयन
प्रीसमयर लीग (CPL) के 2023 सोंस्करि के चैंसपयन बने।
i.ड् वेन प्रीटोररयस ने 26 रन दे कर चार सवकेट सलए, जबसक नाइट राइडसध 94 रन पर आउट हो गए। इमरान तासहर (4-
0-8-2)और गुडाकेश मोती (4-1-7-2)सकफायती स्पैल लेकर आए।

259. उस राज्य सरिार िा िाम बताइए निसिे हाि ही में (नसतंबर'23 में) सरिारी स्कूिों में िीिि िौशि
नशक्षा िायाक्रम 'सक्षम' पररयोििा शुरू िरिे िे निए मै निि बस इं निया फाउं िेशि िे साथ साझेदारी िी
है।
1)गुजरात
2)मध्य प्रदे श
3)ओसडशा
4)महाराष्ट्र
5)पसिम बोंगाल
उत्तर- 2)मध्य प्रदे श
स्पष्टीिरण:
मैनिि बस इं निया फाउं िेशि, एक गैर-लाभकारी सोंगठन जो सशक्षा और कौशल प्रदान करता है , ने सरकारी स्कूलोों
में एक जीवन कौशल सशक्षा कायधक्रम - 'सक्षम' पररयोजना शुरू करने के सलए मध्य प्रदे श सरकार के साि साझेदारी
की है।

Report Errors in the PDF - ebooks@affairscloud.com Copyright 2014-2023 @ AffairsCloud.com 316


i.मध्य प्रदे श के जनजातीय मामलोों के सवभाग ने मास्ट्र प्रसशक्षकोों और सरकारी स्कूल के सशक्षकोों को सशक्त बनाने के
सलए मैसजक बस इों सडया फाउों डेशन के साि साझेदारी की है , जो उन्ें राज्य के 20 सजलोों और 89 िॉकोों में सकशोरोों के
सवािंगीि सवकास का पोषि करते हुए जीवन कौशल सशक्षा प्रदान करने में सक्षम बनाता है।

260. निम्ननिखित में से िौि सा राष्टरीय उद्याि िेरि में खस्थत िही ं है ?
1)मसिकेत्तन शोला राष्ट्रीय उद्यान
2)पम्पादु म शोला राष्ट्रीय उद्यान
3)अनामुडी शोला राष्ट्रीय उद्यान
4)दे सहोंग पटकाई राष्ट्रीय उद्यान
5)एरासवकुलम राष्ट्रीय उद्यान
उत्तर-4)दे नहंग पटिाई राष्टरीय उद्याि
स्पष्टीिरण:
दे नहंग पटिाई राष्टरीय उद्याि असम में क्कथित है। इसे 2020 में वन्यजीव अभ्यारण्य से राष्ट्रीय उद्यान में उन्नत सकया
गया िा, यह बडे दे सहों ग पटकाई हािी ररजवध के भीतर क्कथित है , जो ऊपरी असम (सडब्रूगढ और सतनसुसकया सजले) के
कोयला और तेल समृद्ध सजलोों में फैला हुआ है।
i.सडगबोई में एसशया की सबसे पुरानी ररफाइनरी और सलडो में 'ओपन कास्ट्' कोयला खनन अभ्यारण्य के पास क्कथित
हैं।
ii. दे सहों ग पटकाई वन्यजीव अभ्यारण्य को जेपोर वषाधवन के नाम से भी जाना जाता है।

261. 'सतिोनसया टाइगर ररििा' निस राज्य में खस्थत है ?


1)पसिम बोंगाल
2)कनाधटक
3)आों ध्र प्रदे श
4)केरल
5)ओसडशा
उत्तर-5)ओनिशा
स्पष्टीिरण:
ओनिशा िे बारे में:
रािधािी-भुवनेश्वर
मुख्यमंत्री- नवीन पटनायक
राज्यपाि- गिेशी लाल
टाइगर ररििा- सससमसलपाल टाइगर ररजवध, सतिोनसया टाइगर ररििा

262. छारी ढांि पक्षी अभ्यारण्य निस राज्य में खस्थत है ?


1)तेलोंगाना
2)पसिम बोंगाल
3)गुजरात
4)तसमलनाडु
5)आों ध्र प्रदे श

Report Errors in the PDF - ebooks@affairscloud.com Copyright 2014-2023 @ AffairsCloud.com 317


उत्तर-3)गुिरात
स्पष्टीिरण:
छारी ढांि पक्षी अभ्यारण्य गुिरात में क्कथित है। यह एक आद्रध भूसम सोंरक्षि अभ्यारण्य है।
i.'छारी' का अिध है नमक प्रभासवत और 'ढाोंड' का अिध उिली आद्रध भूसम है । छारी-ढाोंड एक कानूनी रूप से सोंरसक्षत
आद्रध भूसम सोंरक्षि ररजवध है।
ii.बन्नी घास के मैदान और छडी-ढाोंड भारत के रे सगस्तानी पाररक्कथिसतकी तोंत्र में सबसे महत्वपूिध पक्षी क्षेत्रोों में से एक हैं ।

263. नदहांग नदबांग बायोस्फीयर ररििा ____________ (राज्य) में खस्थत है ।


1)असम
2)सत्रपुरा
3)ससक्किम
4)सहमाचल प्रदे श
5)अरुिाचल प्रदे श
उत्तर-5)अरुणाचि प्रदे श
स्पष्टीिरण:
नदहांग-नदबांग बायोस्फीयर ररििा भारत के उत्तरपूवी भाग, सवशेष रूप से अरुणाचि प्रदे श राज्य में क्कथित एक
महत्वपूिध पाररक्कथिसतक क्षेत्र है।
i.बायोस्फीयर ररज़वध का नाम क्षेत्र की दो प्रमुख नसदयोों, सदहाोंग (सजसे ससयाों ग या ब्रह्पुत्र के नाम से भी जाना जाता है )
और सदबाोंग के नाम पर रखा गया है।
ii.ये नसदयााँ पररदृश्य को आकार दे ने और ररजवध के भीतर सवसवर् पाररक्कथिसतक तोंत्र का समिधन करने में महत्वपूिध
भूसमका सनभाती हैं।

264. फेि िन्यिीि अभ्यारण्य निस राज्य में खस्थत है ?


1)उत्तर प्रदे श
2)पसिम बोंगाल
3)महाराष्ट्र
4)ओसडशा
5)मध्य प्रदे श
उत्तर-5)मध्य प्रदे श
स्पष्टीिरण:
मध्य प्रदे श (MP) िे बारे में:
राज्यपाि– मोंगुभाई छगनभाई पटे ल
मुख्यमंत्री–सशवराज ससोंह चौहान
िन्यिीि अभ्यारण्य- पनपिा वन्यजीव अभ्यारण्य, फेि िन्यिीि अभ्यारण्य

265. तेि और प्रािृनति गैस निगम (ONGC) िे ितामाि (नसतंबर'23 ति) अध्यक्ष और मुख्य िायािारी
अनधिारी (CEO) िौि हैं?
1)ऐश्वयाध सबस्वाल
2)C राजीव

Report Errors in the PDF - ebooks@affairscloud.com Copyright 2014-2023 @ AffairsCloud.com 318


3)अरुि कुमार ससोंह
4)G कृष्णकुमार
5)वेत्सा रामकृष्ण गुप्ता
उत्तर-3)अरुण िुमार नसंह
स्पष्टीिरण:
तेि और प्रािृनति गैस निगम (ONGC) िे बारे में:
अध्यक्ष & CEO– अरुण िुमार नसंह
मुख्यािय– नई सदल्ली, सदल्ली
स्थानपत- 14 अगस्त 1956

266. िेंद्र सरिार िे हाि ही में (नसतंबर'23 में) प्रधाि मंत्री िरें द्र मोदी िे सिाहिार िे रूप में ___________
िा िायािाि बढ़ा नदया है।
1)राजीव महसषध
2)नृपेंद्र समश्रा
3)प्रदीप कुमार ससन्ा
4)असमत खरे
5)राजीव गौबा
उत्तर-4)अनमत िरे
स्पष्टीिरण:
प्रर्ान मोंत्री कायाध लय में एक शीषध प्रशासक के सलए सवस्तार को मोंजूरी दे ते हुए, केंद्र सरकार ने 1985-बैच के IAS
असर्कारी अनमत िरे के सलए सेवा सवस्तार की घोषिा की, जो वतधमान में प्रर्ान मोंत्री नरें द्र मोदी के सिाहिार के
रूप में तैनात हैं।
i.कहा जाता है सक झारखोंड कैडर के सेवासनवृत्त असर्कारी खरे को प्रर्ान मोंत्री के कायधकाल के बराबर अवसर् के सलए
या 'अगले आदे श तक' सदल्ली में ससचव स्तर पर सफर से सनयुक्त सकया गया है।
ii.खरे ने उच्च सशक्षा ससचव के रूप में कायध सकया है और 30 ससतोंबर, 2021 को सेवासनवृत्त हुए। उन्ें शुरू में अरू बर
2021 में दो साल के कायधकाल के सलए सलाहकार पद पर सनयुक्त सकया गया िा।

267. िोिरे ि ररि राष्टरीय उद्याि निस राज्य/UT में खस्थत है ?


1)ससक्किम
2)जम्मू और कश्मीर
3)मेघालय
4)उत्तराखोंड
5)मध्य प्रदे श
उत्तर-3)मेघािय
स्पष्टीिरण:
िोिरे ि राष्टरीय उद्याि पूवोत्तर भारत के मेघािय राज्य में क्कथित एक प्रमुख राष्ट्रीय उद्यान है । यह क्षेत्र में सबसे
महत्वपूिध जैव सवसवर्ता हॉटस्पॉट में से एक है और इसे UNESCO बायोस्फीयर ररजवध के रूप में नासमत सकया गया है।
i.नोकरे क नेशनल पाकध सवसवर् पाररक्कथिसतक तोंत्रोों का घर है , सजसमें उष्णकसटबोंर्ीय और उपोष्णकसटबोंर्ीय वन, घास के
मैदान और उच्च ऊोंचाई वाले पठार शासमल हैं।

Report Errors in the PDF - ebooks@affairscloud.com Copyright 2014-2023 @ AffairsCloud.com 319


ii.इसमें नोकरे क बायोस्फीयर ररज़वध शासमल है , सजसमें नोकरे क पीक शासमल है , जो गारो सहल्स क्षेत्र की सबसे ऊोंची
चोटी है।

268. धौिपुर-िरौिी टाइगर ररििा निस राज्य में खस्थत है ?


1)पसिम बोंगाल
2)कनाधटक
3)राजथिान
4)महाराष्ट्र
5)उत्तर प्रदे श
उत्तर-3)रािस्थाि
स्पष्टीिरण:
धौिपुर-िरौिी टाइगर ररििा रािस्थाि में क्कथित है। यह रािस्थाि का 5वाों टाइगर ररजवध बन गया है।
i.यह जोंगल 1,058 वगध सकलोमीटर (कोर क्षेत्र का 368 sq km और बफर क्षेत्र का 690 वगध सकलोमीटर) क्षेत्र में फैला है
और वतधमान में इसमें 9 बाघ हैं।

269. पचमढ़ी बायोस्फीयर ररििा निस राज्य में खस्थत है ?


1)राजथिान
2)पोंजाब
3)कनाधटक
4)गुजरात
5)मध्य प्रदे श
उत्तर-5)मध्य प्रदे श
स्पष्टीिरण:
पचमढ़ी बायोस्फीयर ररििा भारत के मध्य प्रदे श के सतपुडा रें ज में क्कथित एक सोंरसक्षत क्षेत्र है। इसकी थिापना 1999
में हुई िी और यह 4,926 वगध सकलोमीटर क्षेत्र में फैला है , जो इसे मध्य प्रदे श का सबसे बडा बायोस्फीयर ररजवध बनाता
है।
i.यह ररज़वध सवसभन्न प्रकार की वनस्पसतयोों और जीवोों का घर है , सजसमें 1,000 से असर्क पौर्ोों की प्रजासतयााँ , 50
स्तनपायी प्रजासतयााँ और 250 पक्षी प्रजासतयााँ शासमल हैं ।
ii.पचमढी बायोस्फीयर ररजवध अपनी समृद्ध साोंस्कृसतक सवरासत के सलए भी जाना जाता है और बैगा, गोोंड और कोरकू
जनजासतयोों ससहत कई आसदवासी समुदायोों का घर है।

270. िॉ. सिीम अिी पक्षी अभ्यारण्य निस राज्य/UT में खस्थत है ?
1)तसमलनाडु
2)गोवा
3)केरल
4)सदल्ली
5)उत्तराखोंड
उत्तर- 2)गोिा

Report Errors in the PDF - ebooks@affairscloud.com Copyright 2014-2023 @ AffairsCloud.com 320


स्पष्टीिरण:
िॉ. सिीम अिी पक्षी अभ्यारण्य गोिा में माोंडोवी नदी के सकनारे चोराओ िीप के पसिमी ससरे पर क्कथित है ।
अभ्यारण्य एक मुहाना मैंग्रोव सनवास थिान है और इसका नाम प्रससद्ध भारतीय पक्षी सवज्ञानी सलीम अली के नाम पर
रखा गया है।
i.अभ्यारण्य सलसटल सबटनध, िैक सबटनध, रे ड नॉट, जैक सस्नप, पाइड एवोसेट, र्ारीदार बगुला, वेस्ट्नध रीफ हे रॉन,
मडक्कस्कपसध, सफडलर केकडे आसद का मेजबान है।

271. िेम्बिाि झीि निस राज्य में खस्थत है ?


1)कनाधटक
2)गुजरात
3)ओसडशा
4)केरल
5)पसिम बोंगाल
उत्तर-4)िेरि
स्पष्टीिरण:
िेम्बिाि झीि, पसिम बोंगाल में सुोंदरबन के बाद भारत (िेरि) में दू सरी सबसे बडी रामसर साइट है , जो ससकुड रही
है और इसकी असितीय जैव सवसवर्ता पाररक्कथिसतक क्षय के खतरे में है , बावजूद इसके सक इसे 20 साल पहले रामसर
साइट घोसषत सकया गया िा।
i.रामसर साइट: 2002 में, झील को रामसर कन्वेंशन िारा पररभासषत अोंतराध ष्ट्रीय महत्व की आद्रध भूसम की सूची में
शासमल सकया गया िा।

272. अंगोिा __________ अंगोिा िी मुद्रा है।


1)क्वान्क्ज़ा
2)फ़्ैंक
3)दीनार
4)पेसो
5)डॉलर
उत्तर- 1)िान्ज़ा
स्पष्टीिरण:
अंगोिा िे बारे में:
रािधािी-लुआोंडा
मुद्रा- अंगोिि िान्ज़ा

273. दनक्षण पूिा एनशयाई दे शों िे संगठि (ASEAN) िा मुख्यािय िहााँ खस्थत है ?
1)मॉक्कन्टरयल, कनाडा
2)जकाताध , इों डोनेसशया
3)ढाका, बाोंग्लादे श
4)शोंघाई, चीन
5)लोंदन, यूनाइटे ड सकोंगडम

Report Errors in the PDF - ebooks@affairscloud.com Copyright 2014-2023 @ AffairsCloud.com 321


उत्तर-2)ििाताा, इं िोिेनशया
स्पष्टीिरण:
दनक्षण पूिा एनशयाई दे शों िे संगठि (ASEAN) िे बारे में:
यह एक क्षेत्रीय अोंतरसरकारी सोंगठन है सजसमें दस दसक्षि पूवध एसशयाई दे श शासमल हैं।
थिापना - इसकी थिापना 8 अगस्त 1967 को बैंकॉक, िाईलैंड में सोंथिापक सदस्योों इों डोनेसशया, मलेसशया, सफलीपीोंस,
ससोंगापुर और िाईलैंड िारा ASEAN घोषिा (बैंकॉक घोषिा) पर हस्ताक्षर के साि की गई िी।
महासनचि– H.E.DR.काओ सकम होनध
मुख्यािय– ििाताा, इं िोिेनशया

274. संयुक्त राष्टर नििास िायाक्रम (UNDP) िी स्थापिा निस िषा हुई थी?
1)1947
2)1978
3)1965
4)1972
5)1961
उत्तर- 3)1965
स्पष्टीिरण:
संयुक्त राष्टर नििास िायाक्रम (UNDP) िे बारे में:
स्थापिा- 1965
प्रशासि– असचम स्ट्ीनर
मुख्यािय– न्यूयॉकध, सोंयुक्त राज्य अमेररका

275. िैज्ञानिि और औद्योनगि अिुसंधाि पररषद (CSIR) िे ितामाि (नसतंबर’23 ति) महानिदे शि िौि हैं ?
1)नल्लािम्बी कलैसेल्वी
2)ररतु कररर्ल
3)असदसत पोंत
4)नोंसदनी हररनाि
5)टे स्सी िॉमस
उत्तर- 1)िल्लाथम्बी ििैसेल्वी
स्पष्टीिरण:
िैज्ञानिि एिं औद्योनगि अिुसंधाि पररषद (CSIR) िे बारे में:
अध्यक्ष– नरें द्र मोदी
उपाध्यक्ष– डॉ. जीतेन्द्र ससोंह
महानिदे शि – िॉ. िल्लाथम्बी ििैसेल्वी
मुख्यािय– नई सदल्ली, सदल्ली
स्थानपत-1942

Report Errors in the PDF - ebooks@affairscloud.com Copyright 2014-2023 @ AffairsCloud.com 322


276. संयुक्त राष्टर निश्व पयाटि संगठि (UNWTO) िी स्थापिा निस िषा हुई थी?
1)1947
2)1978
3)1975
4)1972
5)1961
उत्तर- 3)1975
स्पष्टीिरण:
संयुक्त राष्टर निश्व पयाटि संगठि(UNWTO) िे बारे में:
यह सजम्मेदार, सटकाऊ और सावधभौसमक रूप से सुलभ पयधटन को बढावा दे ने के सलए सजम्मेदार UN एजेंसी है ।
महासनचि - ज़ुराब पोलोसलकासश्वली
मुख्यािय – मैसडर ड, स्पेन
स्थानपत – 1975

277. अखिि भारतीय तििीिी नशक्षा पररषद (AICTE) िे ितामाि (नसतंबर’23 ति) अध्यक्ष िौि हैं ?
1)प्रवीि शमाध
2)अजय कुमार
3)आसतश मािुर
4)अभय जेरे
5)T.G. सीिाराम
उत्तर-5)T.G. सीथाराम
स्पष्टीिरण:
अखिि भारतीय तििीिी नशक्षा पररषद (AICTE) िे बारे में:
अक्कखल भारतीय तकनीकी सशक्षा पररषद (AICTE) सशक्षा मोंत्रालय (MoE) के तहत तकनीकी सशक्षा में गुिवत्ता को
बढावा दे ने के सलए राष्ट्रीय स्तर की सवोच्च सलाहकार सोंथिा है ।
अध्यक्ष – T.G. सीथाराम
मुख्यािय – नई सदल्ली, सदल्ली
स्थापिा - 1945

278. नसतंबर 2023 में, नमस्र िे िे नमएटा बंदरगाह से 37.4 टि हरे अमोनिया िािे हरे अमोनिया िंटे िरों िो
______________ (पोटा ) प्रानधिरण द्वारा सफितापूिाि संभािा गया है।
1)पारादीप बोंदरगाह
2)जवाहरलाल नेहरू बोंदरगाह
3)दीनदयाल बोंदरगाह
4)VO सचदों बरनार बोंदरगाह
5)चेन्नई बोंदरगाह
उत्तर-4)VO नचदं बरिार बंदरगाह

Report Errors in the PDF - ebooks@affairscloud.com Copyright 2014-2023 @ AffairsCloud.com 323


स्पष्टीिरण:
समस्र के डे समएटा बोंदरगाह से 37.4 टन हरे अमोसनया वाले हरे अमोसनया कोंटे नरोों को 23 ससतोंबर 2023 को VO
नचदं बरिार बंदरगाह प्रासर्करि िारा सफलतापूवधक सोंभाला गया िा। हरा अमोसनया तूतीकोररन अल्कली केसमकल
एों ड फसटध लाइजसध सलसमटे ड (TFL) के सलए है।
i.परों परागत रूप से, ग्रे अमोसनया का उपयोग सोडा ऐश उत्पादन के सलए सकया जाता है। हररत पहल के रूप में , TFL ने
परीक्षि के आर्ार पर हररत सोडा ऐश का उत्पादन करने के सलए हररत अमोसनया का आयात सकया है।
ii.TFL हररत अमोसनया की उपलब्धता के अर्ीन इस वषध 2000 मीसटर क टन आयात करने की योजना बना रहा है। VO
सचदों बरनार बोंदरगाह ने 24 ससतोंबर को एक ही सदन में 2,01,204 मीसटर क टन का प्रबोंर्न करके एक नया ररकॉडध
बनाया, जो 26 अगस्त को 2,00,642 मीसटर क टन के पहले एक सदन के ररकॉडध को पार कर गया।

279. गूगि िूिि 27 नसतंबर 2023 िो अपिा _________ िन्मनदि मिाता है।
1)50वाों
2)35वाों
3)15वाों
4)20वाों
5)25वाों
उत्तर- 5)25िां
स्पष्टीिरण:
गूगल डूडल ने 27 ससतोंबर 2023 को एक सवशेष Doodle के साि अपना 25िां जन्मसदन मनाया। गूगल हमेशा भसवष्य
पर ध्यान केंसद्रत करता है लेसकन जन्मसदन सचोंतन का अवसर प्रदान करता है।
i.गूगल इों क की आसर्काररक थिापना 27 ससतोंबर 1998 को अमेररकी कोंप्यूटर वैज्ञासनक लैरी पेज और सगेई सब्रन िारा
की गई िी।

280. नसतंबर 2023 में, सरिार िे नियाानतत उत्पादों पर शुल्क और िरों िी छूट (RoDTEP) योििा िे तहत
30 नसतंबर, 2023 से __________ ति समथाि बढ़ाया।
1)30 जून 2024
2)31 जनवरी 2024
3)30 अप्रैल 2024
4)31 माचध 2024
5)31 जुलाई 2024
उत्तर- 1)30 िूि 2024
स्पष्टीिरण:
सरकार ने सनयाधसतत उत्पादोों पर शुल्क और करोों में छूट (RoDTEP) की योजना के तहत 30 ससतोंबर, 2023 से 30 िूि,
2024 तक समिधन बढाया। वासिज्य और उद्योग मों त्रालय ने कहा सक इससे सनयाध तकोों को वतध मान अोंतरराष्ट्रीय माहौल
में बेहतर शतों पर अनुबोंर् पर बातचीत करने में मदद समलेगी।
i.यह योजना करोों, शुल्कोों और शुल्कोों की प्रसतपूसतध के सलए एक तोंत्र प्रदान करती है , जो वतधमान में केंद्र, राज्य और
थिानीय स्तरोों पर सकसी अन्य तोंत्र के तहत वापस नहीों सकए जा रहे हैं , लेसकन जो सनयाधत सत उत्पादोों के सनमाधि और
सवतरि की प्रसक्रया में सनयाध त सोंथिाओों िारा सकए जाते हैं।

Report Errors in the PDF - ebooks@affairscloud.com Copyright 2014-2023 @ AffairsCloud.com 324


281. मोिेम राष्टरीय उद्याि निस राज्य में खस्थत है ?
1)झारखोंड
2)गुजरात
3)ओसडशा
4)गोवा
5)उत्तर प्रदे श
उत्तर-4)गोिा
स्पष्टीिरण:
मोिेम राष्टरीय उद्याि गोिा में क्कथित है । भगवान महावीर वन्यजीव अभ्यारण्य के मु ख्य क्षेत्र को राष्ट्रीय उद्यान घोसषत
सकया गया िा।
i.भगवान महावीर वन्यजीव अभ्यारण्य और मोल्लेम राष्ट्रीय उद्यान के रूप में लोकसप्रय है । इसमें 12वीों सदी का ताोंबडी
सुरला मोंसदर, डे सवल्स कैन्यन, सनसेट पॉइों ट जैसे आकषधि हैं

282. पीिीभीत टाइगर ररििा ___________ में खस्थत है।


1)उत्तराखोंड
2)महाराष्ट्र
3)असम
4)उत्तर प्रदे श
5)पसिम बोंगाल
उत्तर-4)उत्तर प्रदे श
स्पष्टीिरण:
पीिीभीत टाइगर ररििा उत्तर प्रदे श के पीलीभीत और शाहजहााँपुर सजले में क्कथित है
• इसे 2014 में टाइगर ररजवध के रूप में नासमत सकया गया िा।
• 2020 में, सपछले चार वषों में बाघोों की सोंख्या दोगुनी करने के सलए इसे अोंतराध ष्ट्रीय पुरस्कार TX2 प्राप्त हुआ।
• यह ऊपरी गोंगा के मैदान में तराई आकध लैंडस्केप का सहस्सा है।
• ररज़वध का उत्तरी सकनारा भारत-नेपाल सीमा पर क्कथित है जबसक दसक्षिी सीमा शारदा और खकरा नदी िारा
सचसह्नत है।

283. शेषचिम बायोस्फीयर ररििा निस राज्य में खस्थत है ?


1)तसमलनाडु
2)आों ध्र प्रदे श
3)गुजरात
4)पसिम बोंगाल
5)पोंजाब
उत्तर- 2)आं ध्र प्रदे श
स्पष्टीिरण:
शेषचलम पहासडयााँ आं ध्र प्रदे श में पूवी घाट का सहस्सा हैं। 2010 में इसे शेषचिम बायोस्फीयर ररििा के रूप में
नासमत सकया गया िा।

Report Errors in the PDF - ebooks@affairscloud.com Copyright 2014-2023 @ AffairsCloud.com 325


i.शेषचलम जीवमोंडल सचत्तूर और कडप्पा सजलोों में फैला हुआ है। प्रमुख सहोंदू तीिधथिल सतरूपसत और श्रीवेंकटे श्वर
राष्ट्रीय उद्यान इन श्रेसियोों में क्कथित हैं ।
ii.यह प्रससद्ध रे ड सैंडसध और स्लेंडर लोररस ससहत कई थिासनक प्रजासतयोों का घर है।

284. 'होिरसर पक्षी अभ्यारण्य' निस राज्य/UT में खस्थत है ?


1)नई सदल्ली
2)चोंडीगढ
3)असम
4)राजथिान
5)जम्मू & कश्मीर
उत्तर-5)िम्मू & िश्मीर
स्पष्टीिरण:
िम्मू और िश्मीर (J&K) िे बारे में:
उपराज्यपाि–मनोज ससन्ा
पक्षी अभ्यारण्य– होिरसर पक्षी अभ्यारण्य
त्यौहार– लोसर; मािो नागराोंग

285. अंतरााष्टरीय रं गमंच संस्थाि (ITI) िा मुख्यािय िहााँ खस्थत है ?


1)सजनेवा, क्कस्वट् जरलैंड
2)लोंदन, यूनाइटे ड सकोंगडम
3)शोंघाई, चीन
4)पेररस, फ़्ाोंस
5)न्यूयॉकध शहर, सोंयुक्त राज्य अमेररका
उत्तर-3)शंघाई, चीि
स्पष्टीिरण:
अंतरााष्टरीय रं गमंच संस्थाि (ITI) िे बारे में:
ITI की थिापना 1948 में पहले सोंयुक्त राष्ट्र शैसक्षक, वैज्ञासनक और साोंस्कृसतक सोंगठन (UNESCO) के महासनदे शक सर
जूसलयन हक्सले और नाटककार और उपन्यासकार JB प्रीस्ट्ली की पहल पर की गई िी।
अध्यक्ष– मोहम्मद सैफ अल-अफखाम
महानिदे शि– टोसबयास सबयानकोन
मुख्यािय– शंघाई, चीि

286. िािाटोप-िनियार िन्यिीि अभ्यारण्य निस राज्य में खस्थत है ?


1)ससक्किम
2)पसिम बोंगाल
3)मसिपुर
4)अरुिाचल प्रदे श
5)सहमाचल प्रदे श
उत्तर-5)नहमाचि प्रदे श

Report Errors in the PDF - ebooks@affairscloud.com Copyright 2014-2023 @ AffairsCloud.com 326


स्पष्टीिरण:
नहमाचि प्रदे श िे बारे में:
मुख्यमंत्री– सुखसवोंदर ससोंह सुक्खू
राज्यपाि– सशव प्रताप शुक्ला
िन्यिीि अभ्यारण्य- िािाटोप-िनियार िन्यिीि अभ्यारण्य; कनावर वन्यजीव अभ्यारण्य
त्यौहार- बसोंत पोंचमी; लोसर

287. एसोनसएटे ि चैंबसा ऑफ िॉमसा एं ि इं िस्ट्र ी ऑफ इं निया (ASSOCHAM) िी स्थापिा निस िषा हुई थी?
1)1920
2)1942
3)1930
4)1946
5)1952
उत्तर- 1)1920
स्पष्टीिरण:
एसोनसएटे ि चैंबसा ऑफ िॉमसा एं ि इं िस्ट्र ी ऑफ इं निया (ASSOCHAM) िे बारे में:
अध्यक्ष- अजय ससोंह
मुख्यािय- नई सदल्ली, सदल्ली
स्थापिा- 1920

288. दनक्षण पूिा एनशयाई राष्टर संघ (ASEAN) िी स्थापिा िब हुई थी?
1)1957
2)1962
3)1967
4)1958
5)1952
उत्तर- 3)1967
स्पष्टीिरण:
दनक्षण पूिा एनशयाई दे शों िे संगठि (ASEAN) िे बारे में:
महासनचि – डॉ. काओ सकम होनध
मुख्यािय - जकाताध, इों डोनेसशया
स्थापिा - 1967

289. संयुक्त राष्टर ििसंख्या िोष (UNFPA) िा मुख्यािय िहााँ खस्थत है ?


1)सवयना, ऑक्कस्ट्रया
2)सजनेवा, क्कस्वट् जरलैंड
3)न्यूयॉकध, सोंयुक्त राज्य अमेररका (USA)
4)पेररस, फ़्ाोंस
5)वासशोंगटन DC, सोंयुक्त राज्य अमेररका (USA)

Report Errors in the PDF - ebooks@affairscloud.com Copyright 2014-2023 @ AffairsCloud.com 327


उत्तर-3)न्यूयॉिा, संयुक्त राज्य अमेररिा (USA)
स्पष्टीिरण:
संयुक्त राष्टर ििसंख्या िोष (UNFPA) िे बारे में:
िायािारी निदे शि - नतासलया कनेम
मुख्यािय – न्यूयॉिा, संयुक्त राज्य अमेररिा (USA)
स्थापिा - 1969 (जनसोंख्या गसतसवसर्योों के सलए सोंयुक्त राष्ट्र कोष के रूप में, बाद में 1987 में इसका नाम बदलकर
सोंयुक्त राष्ट्र जनसोंख्या कोष कर सदया गया)

290. नसतंबर 2023 में, __________, िो हैरी पॉटर नफिों में "एल्बस िं बििोर" िे रूप में अपिी भूनमिा िे
निए प्रनसि थे, िा निधि हो गया।
1)जॉन हटध
2)ररचडध सग्रसफथ्स
3)माइकल गैंबोन
4)एलन ररकमैन
5)रोबी कोलटर ै न
उत्तर-3)माइिि गैंबोि
स्पष्टीिरण:
अनुभवी असभनेता माइिि गैंबोि, जो आठ हैरी पॉटर नफिों में से छह में हॉगवट्ध स के हेडमास्ट्र 'एल्बस
िं बििोर' के सकरदार के सलए जाने जाते िे, का 28 ससतोंबर 2023 को 82 वषध की आयु में सनर्न हो गया।
• उन्ोोंने 2001 के "गोस्फोडध पाकध" और "द सकोंग्स स्पीच" के सलए तीन ओसलसवयर पुरस्कार और दो कलाकारोों
की टु कडी के स्क्रीन एरसध सगड पुरस्कार भी जीते।
• गैंबोन को 1992 में सब्रसटश साम्राज्य का कमाोंडर बनाया गया िा और 1998 में नाटक की सेवाओों के सलए
नाइट की उपासर् दी गई िी।

291. िेंद्रीय मत्स्यपािि, पशुपािि और िे यरी मंत्री परषोत्तम रूपािा िे हाि ही में (नसतंबर'23 में) निस
शहर में भारतीय मछिी सिेक्षण (FSI) में आधुनिि समुद्री संग्रहािय िा उद् घाटि निया है ?
1)चेन्नई
2)सवशाखापत्तनम
3)मुोंबई
4)कोचीन
5)मैंगलोर
उत्तर- 2)निशािापत्तिम
स्पष्टीिरण:
25 ससतोंबर 2023 को, केंद्रीय मत्स्य पालन, पशुपालन और डे यरी मोंत्री, परषोत्तम रूपािा ने भारतीय मछली सवेक्षि
(FSI), निशािापत्तिम, आों ध्र प्रदे श में आर्ुसनक समुद्री संग्रहािय का उद् घाटन सकया।
• सोंग्रहालय 200 से असर्क सकस्मोों की मछली और अन्य समुद्री उत्पादोों को सोंरसक्षत कर रहा है।
• सोंग्रहालय में सभी नमूनोों को आर्ुसनक तकनीक (जैसे QR कोसडों ग) के साि प्रदसशधत सकया गया िा।

Report Errors in the PDF - ebooks@affairscloud.com Copyright 2014-2023 @ AffairsCloud.com 328


292. सेशेल्स िी रािधािी िौि सी है ?
1)नगेरुलमुद
2)वेसलोंगटन
3)सवरोररया
4)माजुरो
5)पोटध -सवला
उत्तर-3)निरोररया
स्पष्टीिरण:
सेशेल्स िे बारे में:
सेशेल्स पूवी अफ्रीका से दू र सहोंद महासागर में 115 िीपोों का एक िीपसमूह है।
राष्टरपनत– वेवल रामकलावन
रािधािी– निरोररया
मुद्रा– सेशेलोइस रुपया

293. राष्टरीय आपदा प्रबंधि प्रानधिरण (NDMA) िे ितामाि (नसतंबर'23 ति) अध्यक्ष िौि हैं ?
1)सनमधला सीतारमि, सवत्त मोंत्रालय (MoF)
2)नरें द्र मोदी, प्रर्ान मोंत्री (PM)
3)असमत शाह, गृह मोंत्रालय (MHA)
4)राजनाि ससोंह, रक्षा मोंत्रालय (MoD)
5)द्रौपदी मुमूध, राष्ट्रपसत
उत्तर -2)िरें द्र मोदी, प्रधाि मंत्री (PM)
स्पष्टीिरण:
राष्टरीय आपदा प्रबंधि प्रानधिरण (NDMA) िे बारे में:
भारत के प्रर्ान मोंत्री (PM) के नेतृत्व में, यह भारत में आपदा प्रबोंर्न के सलए शीषध सनकाय है।
अध्यक्ष– िरें द्र मोदी
मुख्यािय– नई सदल्ली, सदल्ली

294. िोड िे िटा बायोस्फीयर ररििा निस राज्य में खस्थत है ?


1)सहमाचल प्रदे श
2)मेघालय
3)गुजरात
4)ससक्किम
5)मध्य प्रदे श
उत्तर- 1)नहमाचि प्रदे श
स्पष्टीिरण:
िोड िे िटा बायोस्फीयर ररििा की थिापना 2009 में नहमाचि प्रदे श राज्य में की गई िी।
• यह सहमाचल प्रदे श में पसिमी सहमालय क्षेत्र में क्कथित है और स्पीसत घाटी ररजवध का एक सहस्सा है।
• सहम तेंदुए जीवमोंडल में पाए जा सकते हैं।

Report Errors in the PDF - ebooks@affairscloud.com Copyright 2014-2023 @ AffairsCloud.com 329


• बायोस्फीयर ररजवध सोंयुक्त राष्ट्र शैसक्षक, वैज्ञासनक और साोंस्कृसतक सोंगठन (UNESCO) िारा प्रदान की गई
अोंतराधष्ट्रीय मान्यता है और अोंतर सरकारी मानव और बायोस्फीयर (MAB) कायधक्रम के तहत नासमत है।

295. दु निया िे 8िें महाद्वीप िा िाम बताइए िो प्रशांत महासागर िे िीचे है , निसे हाि ही में (नसतंबर'23 में)
भूिंपनिज्ञानियों और भूिैज्ञानििों िी एि टीम िे समुद्र ति से बरामद चट्टाि िे िमूिों से प्राप्त आं िडों िा
उपयोग िरिे िोिा था।
1)जॉडध सैंड
2)लेमुररया
3)जीलैंसडया
4)डोगरलैंड
5)गोोंडवाना
उत्तर-3)िीिैंनिया
स्पष्टीिरण:
भूकोंपसवज्ञासनयोों और भूवैज्ञासनकोों के एक समूह ने एक ऐसे महािीप की खोज की जो लगभग 375 वषों से जलमग्न िा।
टीम ने िीिैंनिया का एक अत्यसर्क सोंशोसर्त मानसचत्र तैयार सकया है , सजसे ते ररउ-ए-माउई के नाम से भी जाना
जाता है , जो 'पृथ्वी का आठिां महाद्वीप' है जो प्रशाों त महासागर के नीचे क्कथित है।
• जीलैंसडया दसक्षि प्रशाोंत महासागर में एक लोंबा, सोंकीिध भूभाग है जो ज्यादातर पानी के नीचे है। यह
मेडागास्कर के आकार से छह गुना असर्क और ऑस्ट्र े सलया के लगभग आर्े आकार का है , सजसका क्षेत्रफल
4.9 समसलयन वगध सकलोमीटर है।
• यह न्यूजीलैंड के दसक्षि से उत्तर की ओर न्यू कैलेडोसनया तक और पसिम में ऑस्ट्र े सलया के पूवध में केन पठार
तक फैला हुआ है।
• इसके अक्कस्तत्व का पहला प्रमाि 1642 में समला, जब एक डच नासवक एबेल तस्मान दसक्षिी गोलार्ध में एक
सवशाल महािीप की खोज के सलए एक समशन पर सनकले।
• बाद में, इसे 2017 में भूवैज्ञासनकोों के एक समूह िारा खोजा गया िा और यह एक समय गोोंडवाना के प्राचीन
महािीप का सहस्सा िा, सजसकी उत्पसत्त लगभग 550 समसलयन वषध पहले हुई िी।

296. िुनिि पक्षी अभ्यारण्य निस राज्य में खस्थत है ?


1)झारखोंड
2)ओसडशा
3)मध्य प्रदे श
4)पसिम बोंगाल
5)उत्तर प्रदे श
उत्तर-4)पनिम बंगाि
स्पष्टीिरण:
रायगोंज वन्यजीव अभ्यारण्य को िुनिि पक्षी अभ्यारण्य के नाम से भी जाना जाता है। यह पनिम बंगाि राज्य में
उत्तर सदनाजपुर सजले के रायगोंज के पास क्कथित है। पक्षी अभ्यारण्य पसक्षयोों की 164 प्रजासतयोों का घर है और हर साल
लगभग 90,000 से 1,00,000 प्रवासी पक्षी अभ्यारण्य में आते हैं।

Report Errors in the PDF - ebooks@affairscloud.com Copyright 2014-2023 @ AffairsCloud.com 330


297. अंतरााष्टरीय समुद्री संगठि (IMO) िा मुख्यािय िहााँ खस्थत है ?
1)कोपेनहेगन, डे नमाकध
2)एम्स्ट्डध म, नीदरलैंड
3)लोंदन, यूनाइटे ड सकोंगडम
4)सलस्बन, पुतधगाल
5)पेररस, फ़्ाोंस
उत्तर-3)िंदि, यूिाइटे ि निंगिम
स्पष्टीिरण:
अंतरााष्टरीय समुद्री संगठि (IMO) िे बारे में:
महासनचि – सकटै क सलम
मुख्यािय- िंदि, यूिाइटे ि निंगिम
स्थापिा – 1958

298. िम्बििोंिा िन्यिीि अभ्यारण्य निस राज्य में खस्थत है ?


1)केरल
2)कनाधटक
3)तसमलनाडु
4)आों ध्र प्रदे श
5)ओसडशा
उत्तर-4)आं ध्र प्रदे श
स्पष्टीिरण:
िम्बििोंिा िन्यिीि अभ्यारण्य आं ध्र प्रदे श के पास सवशाखापत्तनम के पास क्कथित एक जोंगल है।
i.अभ्यारण्य एक उष्णकसटबोंर्ीय पिधपाती जोंगल है सजसमें झासडयोों और घास के मैदानोों के साि-साि समसश्रत सोंरचना
के वृक्षोों की छतरी है। यह पसिम में ससम्हाचलम पहाडी श्रृोंखला और उत्तर-पूवध में गोंभीरम जलाशय से सघरा है ।

299. गैल्विाइनिंग ऑगेनिि बायो-एग्रो ररसोसेि धि (GOBARdhan) योििा निस िषा शुरू िी गई थी?
1)2017
2)2015
3)2018
4)2016
5)2019
उत्तर- 3)2018
स्पष्टीिरण:
स्वच्छ भारत समशन ग्रामीि SBM(G)-चरि II कायधक्रम के तहत 2018 में शुरू की गई गैल्वनाइसजोंग ऑगेसनक बायो-
एग्रो ररसोसेज र्न (GOBARdhan) योििा, भारत सरकार की एक व्यापक पहल है सजसका उद्दे श्य चक्रीय
अिधव्यवथिा को बढावा दे ने के सलए कचरे को र्न में बदलना है।
• िि शखक्त मोंत्रालय के तहत पेयजल और स्वच्छता सवभाग (DDWS) GOBARdhan के सलए नोडल सवभाग
है।

Report Errors in the PDF - ebooks@affairscloud.com Copyright 2014-2023 @ AffairsCloud.com 331


• उद्दे श्य: मवेसशयोों के गोबर, कृसष अवशेषोों और अन्य जैसवक कचरे को बायोगैस, CBG और जैव उवधरक में
पररवसतधत करके र्न और ऊजाध उत्पन्न करना है।
• िक्ष्य:इस योजना का लक्ष् भारत के जलवायु कारध वाई लक्ष्ोों, सवशेष रूप से 2070 तक शुद्ध शून्य उत्सजधन
प्राप्त करने की सदशा में योगदान करना है ।

300. 'अमराबाद टाइगर ररििा' निस राज्य में खस्थत है ?


1)कनाधटक
2)आों ध्र प्रदे श
3)तेलोंगाना
4)केरल
5)मध्य प्रदे श
उत्तर-3)तेिंगािा
स्पष्टीिरण:
तेिंगािा िे बारे में:
मुख्यमंत्री - कल्वाकुोंतला चन्द्रशेखर राव
राज्यपाि – डॉ. तसमसलसाई सौोंदयधराजन
टाइगर ररििा - अमराबाद टाइगर ररििा

301. िागोिो-िाराबाि क्षेत्र िा स्व-घोनषत गणराज्य 1 िििरी, 2024 से समाप्त हो िाएगा।


िागोिो-िाराबाि निस दे श िी सीमा में खस्थत है ?
1)रूस
2)ईरान
3)जॉसजधया
4)अज़रबैजान
5)आमेसनया
उत्तर-4)अज़रबैिाि
स्पष्टीिरण:
िागोिो-िाराबाि का स्व-घोसषत गिराज्य 1 जनवरी, 2024 से अक्कस्तत्व में नहीों रहेगा।
• यह तब हुआ जब नागोनो-काराबाख के राष्ट्रपसत सै मवेल शाहरामनयन ने अजरबै जान से अपनी हार के बाद
राज्य सोंथिानोों को भोंग करने वाले एक सडक्री पर हस्ताक्षर सकए।
• नागोनो-काराबाख अज़रबैिाि की सीमाओों के भीतर क्कथित है , लेसकन दशकोों से यह अपनी वास्तसवक सरकार
के साि स्वायत्त रूप से सोंचासलत है।
• इस समझौते का पररिाम यह हुआ सक 120,000 जातीय अमेसनयाई लोगोों में से आर्े से असर्क जो नागोनो-
काराबाख में रहते िे, अमेसनया में भाग गए

302. हाि ही में (नसतंबर 23 में) दू रसंचार निभाग में सनचि िे रूप में निसे नियुक्त निया गया है ?
1)उमोंग नरूला
2)नीरज समत्तल
3)VL काोंिा राव

Report Errors in the PDF - ebooks@affairscloud.com Copyright 2014-2023 @ AffairsCloud.com 332


4)S कृष्णन
5)सववेक भारिाज
उत्तर-2)िीरि नमत्ति
स्पष्टीिरण:
कैसबनेट की सनयुक्कक्त ससमसत (ACC) ने 1992 बैच के अनुभवी भारतीय प्रशाससनक सेवा (IAS) असर्कारी िीरि नमत्ति
को दू रसंचार निभाग में सनचि सनयुक्त सकया है।
• एक और महत्वपूिध सनयुक्कक्त तसमलनाडु कैडर के 1989 बैच के प्रसतसष्ठत IAS असर्कारी S कृष्णन की है ।
वतधमान में, कृष्णन तसमलनाडु राज्य सरकार में उद्योग ससचव के पद पर हैं। प्रशाससनक भूसमकाओों में उनके
व्यापक अनुभव के कारि उन्ें इलेररॉसनक्स और सूचना प्रौद्योसगकी मोंत्रालय के नए ससचव के रूप में चुना
गया है।
• कृष्णन की सनयुक्कक्त सपछले ससचव अलकेश कुमार शमाध के 31 अगस्त को सेवासनवृत्त होने के बाद हुई है।
i.उमोंग नरूला को सोंसदीय कायध मोंत्रालय में ससचव के रूप में नासमत सकया गया है , सजससे उनकी व्यापक प्रशाससनक
सवशेषज्ञता इस महत्वपूिध पोटध फोसलयो में आ गई है।'
ii.1992 के मसिपुर कैडर के IAS असर्कारी वुमलुनमोंग वुअलनाम को नागररक उड्डयन मोंत्रालय में ससचव की भूसमका
सौोंपी गई है। उन्ोोंने पहले सवत्त मोंत्रालय के आसिधक मामलोों के सवभाग में असतररक्त ससचव के रूप में कायध सकया िा।

303. संयुक्त राष्टर बाि िोष (UNICEF) िी स्थापिा निस िषा िी गई थी?
1)1959
2)1946
3)1964
4)1952
5)1972
उत्तर- 2)1946
स्पष्टीिरण:
संयुक्त राष्टर बाि िोष (UNICEF) िे बारे में:
िायािारी निदे शि– कैिरीन रसेल
मुख्यािय– न्यूयॉकध, सोंयुक्त राज्य अमेररका (USA)
स्थापिा- 11 सदसोंबर 1946

304. व्यापार और नििास पर संयुक्त राष्टर सम्मेिि (UNCTAD) िा मुख्यािय िहााँ खस्थत है ?
1)सजनेवा, क्कस्वट् जरलैंड
2)लोंदन, यूनाइटे ड सकोंगडम
3)इटली, रोम
4)पेररस, फ़्ाोंस
5)न्यूयॉकध शहर, सोंयुक्त राज्य अमेररका
उत्तर-1)नििेिा, खस्वट् िरिैंि
स्पष्टीिरण:
व्यापार और नििास पर संयुक्त राष्टर सम्मेिि (UNCTAD) िे बारे में:
यह एक अोंतरसरकारी सोंगठन है जो सवश्व व्यापार में सवकासशील दे शोों के सहतोों को बढावा दे ता है।

Report Errors in the PDF - ebooks@affairscloud.com Copyright 2014-2023 @ AffairsCloud.com 333


महासनचि– रे बेका सग्रनस्पैन
मुख्यािय– नििेिा, खस्वट् िरिैंि

305. मेहाओ िन्यिीि अभ्यारण्य निस राज्य में खस्थत है ?


1)समजोरम
2)ससक्किम
3)ओसडशा
4)पसिम बोंगाल
5)अरुिाचल प्रदे श
उत्तर-5)अरुणाचि प्रदे श
स्पष्टीिरण:
अरुणाचि प्रदे श िे बारे में :
मुख्यमंत्री - पेमा खाोंडू
राज्यपाि - लेक्कफ्टनेंट जनरल केटी पारनाइक
िन्यिीि अभ्यारण्य - मेहाओ िन्यिीि अभ्यारण्य, योडी -राबे सुपसे
िूिॉनििि पािा - ईटानगर जूलॉसजकल पाकध, रोइों ग सचसडयाघर

306. ितांगिी राष्टरीय उद्याि _____________ (राज्य) में खस्थत है।


1)झारखोंड
2)पसिम बोंगाल
3)असम
4)नागालैंड
5)ससक्किम
उत्तर-4)िागािैंि
स्पष्टीिरण:
ितांगिी राष्टरीय उद्याि िागािैंि में क्कथित है।
i.पेरेन शहर से 40 km और दीमापुर से लगभग 37 km दू र क्कथित नताोंगकी राष्ट्रीय उद्यान को कई लोग इों ताों की के नाम
से भी जानते हैं। 200 वगध km की दू री में फैले इस पाकध की थिापना सब्रसटश प्रशासकोों ने वषध 1923 में की िी।

307. अचाििमार टाइगर ररििा निस राज्य में खस्थत है ?


1)छत्तीसगढ
2)अरुिाचल प्रदे श
3)ससक्किम
4)पसिम बोंगाल
5)सत्रपुरा
उत्तर- 1)छत्तीसगढ़

Report Errors in the PDF - ebooks@affairscloud.com Copyright 2014-2023 @ AffairsCloud.com 334


स्पष्टीिरण:
अचाििमार िन्यिीि अभ्यारण्य की थिापना 1975 में हुई िी और 2009 में इसे छत्तीसगढ़ में बाघ अभ्यारण्य
घोसषत सकया गया िा। सकोंवदों ती है सक इस जोंगल में एक बाघ के आकक्कस्मक('अचानक') हमले ('मार') से एक सब्रसटश
व्यक्कक्त की मौत हो गई िी।
i.अभ्यारण्य बोंगाल टाइगर, भारतीय तेंदुआ, र्ारीदार लकडबग्घा, भारतीय ससयार, ढोले, स्लॉि भालू, गौर, साोंभर,
सचत्तीदार सहरि, नीलगाय, चार सीोंग वाले मृग, बासकिंग सहरि, जोंगली सूअर और कई अन्य प्रजासतयोों का घर है । इसके
पररदृश्य में 150 से असर्क प्रजासतयोों के पक्षी उडते हैं।

308. अगखस्तयार मिाई बायोस्फीयर ररििा _________________ (राज्य) में खस्थत है।
1)तेलोंगाना
2)केरल
3)कनाधटक
4)आों ध्र प्रदे श
5)तसमलनाडु
उत्तर-5)तनमििािु
स्पष्टीिरण:
तनमििािु िे बारे में:
मुख्यमंत्री– मुिुवेल करुिासनसर् स्ट्ासलन
िन्यिीि अभ्यारण्य-प्वाइों ट कैसलमेरे वन्यजीव अभ्यारण्य
बायोस्फीयर ररििा- नीलसगरी बायोस्फीयर ररजवध, मन्नार की खाडी बायोस्फीयर ररजवध, और अगखस्तयार मिाई
बायोस्फीयर ररििा

309. रं गिानथटु पक्षी अभ्यारण्य निस राज्य में खस्थत है ?


1)तेलोंगाना
2)मध्य प्रदे श
3)कनाधटक
4)तसमलनाडु
5)आों ध्र प्रदे श
उत्तर-3)ििााटि
स्पष्टीिरण:
रं गिानथटु पक्षी अभ्यारण्य ििााटि में क्कथित है। सवसभन्न प्रकार के थिानीय और प्रवासी पसक्षयोों के सलए मुख्य प्रजनन
थिल हैं।
i.इसमें कावेरी नदी में 6 (छह) िीप और 6 (छह) िीप शासमल हैं।

310. िेशिि थमाि पािर िॉरपोरे शि (NTPC ) निनमटे ि िी स्थापिा निस िषा हुई थी?
1)1992
2)1958
3)1982
4)1965

Report Errors in the PDF - ebooks@affairscloud.com Copyright 2014-2023 @ AffairsCloud.com 335


5)1975
उत्तर- 5)1975
स्पष्टीिरण:
NTPC निनमटे ि (पूिा में िेशिि थमाि पािर िॉपोरे शि निनमटे ि) िे बारे में:
अध्यक्ष एिं प्रबंध निदे शि (CMD) – गुरदीप ससोंह
मुख्यािय – नई सदल्ली, सदल्ली
स्थापिा- 1975

311. 'रािपेररयि िन्यिीि अभ्यारण्य' निस राज्य/UT में खस्थत है ?


1)राजथिान
2)जम्मू और कश्मीर
3)पोंजाब
4)लद्दाख
5)आों ध्र प्रदे श
उत्तर- 2)िम्मू और िश्मीर
स्पष्टीिरण:
िम्मू & िश्मीर (J&K) िे बारे में:
उपराज्यपाि (LG)-मनोज ससन्ा
िन्यिीि अभ्यारण्य - रामनगर वन्यजीव अभ्यारण्य, रािपेररयि िन्यिीि अभ्यारण्य
हिाई अड्डा - श्रीनगर अोंतराध ष्ट्रीय हवाई अड्डा (शेख उल-आलम अोंतराध ष्ट्रीय हवाई अड्डा), जम्मू हवाई अड्डा सतवारी हवाई
अड्डा

312. दनक्षण पूिा एनशयाई राष्टर संघ (ASEAN) िी स्थापिा 1967 में ________ घोषणा पर हस्ताक्षर िे साथ हुई
थी।
1)सफलीपीोंस घोषिा
2)ब्रुनेई घोषिा
3)म्याोंमार घोषिा
4)बैंकॉक घोषिा
5)सवयतनाम घोषिा
उत्तर-4)बैंिॉि घोषणा
स्पष्टीिरण:
दसक्षि पूवध एसशयाई राष्ट्र सों घ (ASEAN) की थिापना 1967 में बैंकॉक, िाईलैंड में ASEAN घोषिा (बैंिॉि घोषणा) पर
हस्ताक्षर के साि हुई िी।
i.उद्दे श्य: दसक्षि एसशयाई क्षेत्र में आसिधक सवकास, सामासजक प्रगसत और साोंस्कृसतक सवकास में तेजी लाना और क्षेत्रीय
शाोंसत और क्कथिरता को बढावा दे ना है।
ii.सदस्य - ब्रुनेई, कोंबोसडया, इों डोनेसशया, लाओस, मलेसशया, म्याोंमार, सफलीपीोंस, ससोंगापुर, िाईलैंड और सवयतनाम।

Report Errors in the PDF - ebooks@affairscloud.com Copyright 2014-2023 @ AffairsCloud.com 336


Aspirant Queries
Aspirant: Does Affairscloud covers all the Current affairs topics
related to examinations?
Affairscloud: We Guaranteed All the Important topics related to
examination are covered in Our Daily CA content and Daily CA
Quizzes.
Aspirant: Why is there a delayal in news?
Affairscloud: As some of the major news sites doesn't provide the
required data on the exact day, we take extra time for important
data to be presented to the aspirants on the examination basis to
ensure nothing is missed.
Example: In 'Important Days' topics the International
Organisations publish their reports and Rankings in the evenings, to
make sure every data is covered, we delay the topics to the next day

Candidates appearing for Competitive Exams. Kindly Share the


General Awareness questions, which asked in their respective
exams to “gaanalysis.ac@gmail.com”

GA Questions Asked in Exams

• Affairscloud’s Self Analysis for General Awareness Section

Report Errors in the PDF - ebooks@affairscloud.com Copyright 2014-2023 @ AffairsCloud.com 337


Report Errors in the PDF - ebooks@affairscloud.com Copyright 2014-2023 @ AffairsCloud.com 338
Report Errors in the PDF - ebooks@affairscloud.com Copyright 2014-2023 @ AffairsCloud.com 339

You might also like